You are on page 1of 760

/a/

EXAMPLES OF QUESTIONS

Enzymes. Coenzymes. Bioenergetics. Citric Acid Cycle of Krebs. Tissue respiration.

1. Pain along large nervous stems and increased amount of pyruvate in the blood were revealed
in the patient. Insufficiency of what vitamin can cause such change?
A В1*
B В2
C РР
D Pantothenic acid
E Biotin

2. In case of enterobiasis acrihine - the structural analogue of vitamin B2 - is administered. The


synthesis disorder of which enzymes does this medicine cause in microorganisms?
A FAD-dependent dehydrogenases *
B Cytochromeoxidases
C Peptidases
D NAD-dependet dehydrogenases
E Aminotransferases

3. A 10-year-old girl often experiences acute respiratory infections with multiple spotty
haemorrages in the places of clothes friction. Hypovitaminosis of what vitamin is present at the
girl?
A С*
B В6
C В1
D А
E В2

4. There is observed inhibited fibrillation in the patients with bile ducts obstruction, bleeding due
to low level of absorbtion of some vitamin. What vitamin is in deficit?
A К*
B А
C D
D Е
E Carotene

5. During endotoxemia active forms of the oxygen including superoxide anion radical are formed
in the human body. With help of what enzyme is this anion activated?
A Superoxide dismutase*
B Catalase
C Peroxidase
D Glutathioneperoxidase
E Glutathionereductase

6. A patient presents high activity of LDH1,2, aspartate aminotransferase, creatine


phosphokinase. In what organ (organs) is the development of a pathological process the most
probable?
A In the heart muscle (initial stage of myocardium infarction) *
B In skeletal muscles (dystrophy, atrophy)
C In kidneys and adrenals
D In connective tissue
E In liver and kidneys
7. While examining the child the doctor revealed symmetric cheeks roughness, diarrhea,
disfunction of the nervous system. Lack of what food components caused it?
A Nicotinic acid, tryptophane*
B Lysine, ascorbic acid
C Threonine, pantothenic acid
D Methionine, lipoic acid
E Phenylalanine, pangamic acid

8. A13-year-old boy complains of general weakness, dizziness, tiredness. He is mentally


retarded. Increased level of valine, isoleucine, leucine is in the blood and urine. Urine has
specific smell. What is the diagnosis?
A Maple syrup urine disease*
B Addison's disease
C Tyrosinosis
D Histidinemia
E Graves' disease

9. Increased breaking of vessels, enamel and dentine destruction in scurvy patients are caused by
disorder of collagen maturing. What stage of modification of procollagen is disordered in this
avitaminosis?
A Hydroxylation of proline*
B Formation of polypeptide chains
C Glycosylation of hydroxylysine residues
D Removal of C-ended peptide from procollagen
E Detaching of N-ended peptide

10. Aspirin has antiinflammatory effect due to inhibition of the cyclooxygenase activity. Level of
what biological active acids will decrease?
A Prostaglandins*
B Leucotriens
C Catecholamines
D Biogenic amines
E Iodinethyronyns

11. Examination of a patient revealed typical presentations of collagenosis. This pathology is


characterized by increase of the following urine index:
A Hydroxyproline*
B Arginine
C Glucose
D Mineral salts
E Ammonium salts

12. Marked increase of activity of МВ-forms of CPK (creatinephosphokinase) and LDH1 were
revealed on the examination of the patient's blood. What is the most likely pathology?
A Miocardial infarction*
B Hepatitis
C Rheumatism
D Pancreatitis
E Cholecystitis
13. Examination of a patient suffering from frequent haemorrhages in the inner organs and
mucous membranes revealed proline and lysine being included in collagen fibers. Impairment of
their hydroxylation is caused by lack of the following vitamin:
A C*
B E
C K
D A
E D

14. A patient has an increased pyruvate concentration in blood. A large amount of it is excreted
with the urine. What vitamin is lacking in this patient?
A B1*
B E
C B3
D B6
E B2

15. Increased production of thyroidal hormones T_3 and T_4, weight loss, tachycardia, psychic
excitement and so on present on thyrotoxicosis. How do thyroidal hormones effect energy
metabolism in the mitochondrion of cells?
A Disconnect oxidation and oxidated phosphorylation *
B Activates phosphorylation of substance
C Stops phosphorylation of substance
D Stops respiratory chain
E Activates oxidated phosphorylation

16. A patient consulted a doctor about symmetric dermatitis of open skin areas. It was found out
that the patient lived mostly on cereals and ate too little meat, milk and eggs. What vitamin
deficiency is the most evident?
A Nicotinamide *
B Calciferol
C Folic acid
D Biotin
E Tocopherol

17. 12 hours after an accute attack of retrosternal pain a patient presented a jump of aspartate
minotransferase activity in blood serum. What pathology is this deviation typical for?
A Myocardium infarction *
B Viral hepatitis
C Collagenosis
D Diabetes mellitus
E Diabetes insipidus

18. Profuse foam appeared when dentist put hydrogen peroxide on the mucous of the oral cavity.
What enzyme caused such activity?
A Catalase *
B Cholinesterase
C Acetyltransferase
D Glucose-6-phosphatdehydrogenase
E Methemoglobin reductase
19. A newborn child has convulsions that have been observed after prescription of vitamin B6.
This most probable cause of this effect is that vitamin B6 is a cofactor of the following enzyme:
A Glutamate decarboxylase *
B Pyruvate dehydrostase
C Netoglubarate dehydromine
D Aminolevulinate synthase
E Glycogen phosphorylase

20. A woman who has been keeping to a clean-rice diet for a long time was diagnosed with
polyneuritis (beriberi). What vitamin deficit results in development of this disease?
A Thiamine *
B Ascorbic acid
C Pyridoxine
D Folic acid
E Riboflavin

21. Removal of gall bladder of a patient has disturbed processes of Ca absorption through the
intestinal wall. What vitamin will stimulate this process?
A D3 *
B PP
C C
D B12
E K

22. Examination of a patient with frequent hemorrhages from internals and mucous membranes
revealed proline and lysine being a part of collagene fibers. What vitamin absence caused
disturbance of their hydroxylation?
A Vitamin C *
B Vitamin K
C Vitamin A
D Thiamine
E Vitamin E

23. Vitamin A together with specific cytoreceptors penetrates through the nuclear membranes,
induces transcription processes that stimulate growth and differentiation of cells. This biological
function is realized by the following form of vitamin A:
A Trans-retinoic acid *
B Trans-retinal
C Cis-retinal
D Retinol
E Carotin

24. A newborn child suffers from milk curdling in stomach, this means that soluble milk proteins
(caseins) transform to insoluble proteins (paracaseins) by means of calcium ions and a certain
enzyme. What enzyme takes part in this process?
A Renin *
B Pepsin
C Gastrin
D Secretin
E Lipase
25. To prevent postoperative bleeding a 6 y.o. child was administered vicasol that is a synthetic
analogue of vitamin K. Name post-translational changes of blood coagulation factors that will be
activated by vicasol:
A Carboxylation of glutamin acid *
B Phosphorylation of serine radicals
C Partial proteolysis
D Polymerization
E Glycosylation

26. According to clinical indications a patient was administered pyridoxal phosphate. What
processes is this medication intended to correct?
A Transamination and decarboxylation of aminoacids *
B Oxidative decarboxylation of ketoacids
C Desamination of purine nucleotide
D Synthesis of purine and pyrimidine bases
E Protein synthesis

27. As a result of posttranslative modifications some proteins taking part in blood coagulation,
particularly prothrombin, become capable of calcium binding. The following vitamin takes part
in this process:
A K*
B C
C A
D B1
E B2

28. A 3 year old child with symptoms of stomatitis, gingivitis and dermatitis of open skin areas
was delivered to a hospital. Examination revealed inherited disturbance of neutral amino acid
transporting in the bowels. These symptoms were caused by the deficiency of thefollowing
vitamin:
A Niacin *
B Pantothenic acid
C Vitamin A
D Cobalamin
E Biotin

29. Surgical removal of a part of stomach resulted in disturbed absorption of vitamin B12, it is
excreted with feces. The patient was diagnosed with anemia. What factor is necessary for
absorption of this vitamin?
A Gastromucoprotein *
B Gastrin
C Hydrochloric acid
D Pepsin
E Folic acid

30. In clinical practice tuberculosis is treated with izoniazid preparation - that is an antivitamin
able to penetrate into the tuberculosis bacillus. Tuberculostatic effect is induced by the
interference with replication processes and oxidation-reduction reactions due to the buildup of
pseudo-coenzyme:
A NAD *
B FAD
C FMN
D TDP
E CoQ

31. In a patient's blood the activities of lactate dehydrogenase (LDH4,LDH5), alanine


aminotransferase, carbamoyl ornithine transferase are increased. What organ is the pathological
process developing in?
A In skeletal muscles
B In the myocardium (myocardial infarction is possible).
C. In the liver (hepatitis is possible)*.
D In kidneys
E In connective tissue CoQ
32. An ophthalmologist found that an outpatient had the increase of the time of sight adaptation
for darkness. What kind of vitamin deficiency can be the cause of the symptom?
A. E.
B. A*
C. C.
D. K
E. D.

33. A patient was diagnosed with megaloblastic anemia. The lack of which substance in the human
organism can cause this disease''
A. Copper.
B. Glycine.
C. Cobalamine*
D Cholecalciferol.
E. Magnesium.

34. Pathological processes associated with the development of hypoxia can be caused by incomplete
reduction of an oxygen molecule in the electron transport chain and accumulation of hydrogen
peroxide. Choose the enzyme which breaks the hydrogen peroxide.
A. Catalase.*
B. Cytochrome oxidase.
C. Succinate dehydrogenase.
D. a-Ketoglutarate dehydrogenase.
E. Aconitase.

35. There is an increase of the pyruvate level in the patient's blood and urine. What kind of
avitaminosis developed in this case?
A. B2 avitaminosis.
B. E avitaminosis.
C. B3 avitaminosis.
D. B6 avitaminosis.
E. B1 avitaminosis.*

36. Potassium cyanide is a very dangerous poison that causes instantaneous death of a human
organism. What mitochondrial enzyme is affected by potassium cyanide?
A. Cytochrome P450.
B. Flavine enzymes.
C. Cytochrome b.
D. NAD+-dependent dehydrogenases.
E. Cytochrome oxidase (cytochrome aa3)*.
37. A 47-year-old patient was brought to an emergency department with the diagnosis of
myocardial infarction. What lactate dehydrogenase (LDH) fractions activity would prevail in the
patient's blood serum during the first two days after hospitalization?
A. LDH4.
B. LDH2
c. LDH3
D. LDH1*
E. LDH5

38. A 50-year-old woman was brought to an emergency clinic with the diagnosis of myocardial
infarction. The activity of what enzyme will prevail in her blood plasma during the first two days
after hospitalization?
A. Alkaline phosphatase.
B. g-Glutamyl transpeptidase.
C. Aspartate aminotransferase.*
D. Acidic phosphatase.
E. Hexokinase.

39. A 2-year-old child developed intestinal dysbacteriosis with a hemorrage syndrome. The most
probable cause of this syndrome is:
A. Vitamin K deficiency.*
B. Activation of tissue thromboplastin.
C. Vitamin PP deficiency.
D. Fibrinogen deficiency.
E. Hypocalcemia.

40. The living organisms that did develop the system of defence against the unfavorable action
of H2O2 during the evolution can exist only in anaerobic conditions. Which of the enzymes can
destroy hydrogen peroxide?
A. Oxygenases and hydroxylases.
B. Peroxidase and catalase.*
C. Cytochrome oxidase, cytochrome b.
D. Oxygenase and catalase.
E. Flavin-linked oxidases.

41. The central intermediate which is common for the catabolic pathways of proteins,
carbohydrates and lipids is:
A. Succinyl-CoA.
B. Acetyl-CoA.*
C. Oxaloacetate.
D. Lactate.
E. Citrate.

42. During the necropsy of a 20-year old girl a pathologist concluded that the death of the patient
had resulted from poisoning by cyanides. The activity of what enzyme is mostly inhibited by
cyanides?
A. Malate dehydrogenase.
B. Cytochrome oxidase.*
C. Heme synthase.
D. Aspartate aminotransferase.
E. Carbamoyl phosphate synthetase.
Metabolism of carbohydrates
1. Galactosemia is revealed in the child. Concentration of glucose in the blood is not
considerably changed. Deficiency of what enzyme caused this illness?
A Galactose-1-phosphate uridyltransferase*
B Amylo-1,6-glucosidase
C Phosphoglucomutase
D Galactokinase
E Hexokinase

2. Characteristic sign of glycogenosis is muscle pain during physical work. Blood examination
reveals usually hypoglycemia. This pathology is caused by congenital deficiency of the
following enzyme:
A. Glycogen phosphorylase*
B. Glucose 6-phosphate dehydrogenase
C. Alpha amylase
D. Gamma amylase
E. Lysosomal glycosidase

3. A 34-year-old patient's resistance to heavy physical load is reduced while the skeletal muscles
glycogen level is increased. By decreasing of the activity of what enzyme can this phenomenon
be explained?
A. Phosphofructokinase.
B. Glucose-6-phosphate dehydrogenase.
C. Glycogen phosphorylase. *
D. Glycogen synthetase.
E. Glucose-6-phosphatase.

4. A patient is ill with diabetes mellitus that is accompanied by hyperglycemia of over 7,2
millimole/l on an empty stomach. The level of what blood plasma protein allows to estimate the
glycemia rate retrospectively (4-8 weeks before examination)?
A. Glycated hemoglobin *
B. Albumin
C. Fibrinogen
D. C-reactive protein
E. Ceruloplasmin

5. A 62-year-old female patient has developed a cataract (lenticular opacity) secondary to the
diabetes mellitus. What type of protein modification is observed in case of diabetic cataract?
A. Glycosylation *
B. Phosphorylation
C. ADP-ribosylation
D. Methylation
E. Limited proteolysis

6. The B cells of endocrine portion of pancreas are selectively damaged by alloxan poisoning.
How will it be reflected in blood plasma?
A. The content of sugar increases *
B. The content of fibrinogen decrease
C. The level of sugar decreases
D. The content of globulins decreases
E. The content of albumins decreases
7. Untrained people often have muscle pain after sprints as a result of lactate accumulation. This
might be caused by intensification of the following biochemical process:
A. Glycolysis *
B. Gluconeogenesis
C. Pentose phosphate pathway
D. Lipogenesis
E. Glycogenesis

8. A patient was delivered to the hospital by an emergency team. Objectively: grave condition,
unconscious, adynamy. Cutaneous surfaces are dry, eyes are sunken, face is cyanotic. There is
tachycardia and smell of acetone from the mouth. Analysis results: blood glucose - 20,1
micromole/l, urine glucose - 3,5% . What is the most probable diagnosis?
A. Hyperglycemic coma *
B. Hypoglycemic coma
C. Acute heart failure
D. Acute alcoholic intoxication
E. Anaphylactic shock

9. Patient with diabetes mellitus experienced loss of consciousness and convulsions after
injection of insulin. What is the result of biochemical blood analysis for concentration of the
sugar?
A. 1,5 mmol/L *
B. 8,0 mmol/L
C. 10,0 mmol/L
D. 3,3 mmol/L
E. 5,5 mmol/L

10. On the empty stomach in the patients blood glucose level was 5,65 mmol/L, in an hour after
usage of sugar it was 8,55 mmol/L, in a 2 hours - 4,95 mmol/L. Such indicators are typical for:
A. Healthy person*
B. Patient with hidden diabetes mellitus
C. Patient with insulin-dependent diabetes mellitus
D. Patient with non-insulin dependent diabetes mellitus
E. Patient with thyrotoxicosis

11. A child is languid, apathetic. Liver is enlarged and liver biopsy revealed a significant excess
of glycogene. Glucose concentration in the blood stream is below normal. What is the cause of
low glucose concentration?
A. Low (absent) activity of glycogene phosphorylase in liver *
B. Low (absent) activity of hexokinase
C. High activity of glycogen synthetase
D. Low (absent) activity of alfa-1,4- glucosidase
E. Deficit of glucose 1-phosphaturidine transferase

12. After a sprint an untrained person develops muscle hypoxia. This leads to the accumulation
of the following metabolite in muscles:
A. Lactate *
B. Ketone bodies
C. Acetyl CoA
D. Glucose 6-phosphate
E. Oxaloacetate
13. Myocyte cytoplasm contains a big number of dissolved metabolites of glucose oxidation.
Name one of them that turns directly into a lactate:
A. Pyruvate *
B. Oxaloacetate
C. Glycerophosphate
D. Glucose 6-phosphate
E. Fructose 6-phosphate

14. A child's blood presents high content of galactose, glucose concentration is low. There are
such presentations as cataract, mental deficiency, adipose degeneration of liver. What disease
is it?
A. Galactosemia *
B. Diabetes mellitus
C. Lactosemia
D. Steroid diabetes
E. Fructosemia

15. A 45 y.o. woman suffers from Cushing's syndrome - steroid diabetes. Biochemical
examination revealed: hyperglycemia, hypochloremia. Which of the under-mentioned processes
is the first to be activated?
A. Gluconeogenesis *
B. Glycogenolysis
C. Glucose reabsorption
D. Glucose transport to the cell
E. Glycolysis

16. The patient with complaints of permanent thirst applied to the doctor.
Hyperglycemia,polyuria and increased concentration of 17-ketosteroids in the urine were
revealed. What disease is the most likely?
A. Steroid diabetes *
B. Insulin-dependent diabetes mellitus
C. Myxoedema
D. Type I glycogenosis
E. Addison's disease

17. When blood circulation in the damaged tissue is restored, then lactate accumulation comes to
a stop and glucose consumption decelerates. These metabolic changes are caused by activation
of the following process:
A. Aerobic glycolysis *
B. Anaerobic glycolysis
C. Lipolysis
D. Gluconeogenesis
E. Glycogen biosynthesis

18. During starvation muscle proteins break up into free amino acids. These compounds will be
the most probably involved into the following process:
A. Cori cycle *
B. Gluconeogenesis in muscles
C. Synthesis of higher fatty acids
D. Glycogenolysis
E. Decarboxylation
19. A newborn develops dyspepsia after the milk feeding. When the milk is substituted by the
glucose solution the dyspepsia symptoms disappear. The newborn has the subnormal activity of
the following enzyme:
A. Lactase *
B. Invertase
C. Maltase
D. Amylase
E. Isomaltase

20. After taking sulfonamides and aspirin by a 38-year-old patient, hemolysis of erythrocytes
caused by the insufficiency of glucose-6-phosphate dehydrogenase developed. The disturbance
of what coenzyme formation does this pathology result from?
A. Ubiquinone.
B. FADH2
C. Pyridoxal phosphate
D. FMNH2.
E. NADPH*

21. A child with point mutation has the absence of glucose- 6- phosphate body tissues,
hypoglycemia and hepatomegaly detected. Define the type of pathology which these symptoms
are characteristic of:
A. Girke's disease. *
B. Measles.
C. Addison's disease.
D. Parkinson's disease.
E. McArdle's disease.

22. The concentration of glucose in the blood plasma of a healthy man varies within the
following limits:
A. 2.0-4.0 mM/L.
B. 3.3-5.5 mM/L. *
C. 10.0-25.0 mM/L.
D. 6.0-9.5 mM/L.
E. 1.0-2.0 mM/L.

23. Some hours after an intensive physical training a sportsman showed activated
gluconeogenesis. Which of the following is the basic substrate of gluconeogenesis?
A. Serine.
B. Aspartate.
C. Glutamate.
D. a-Ketoglutarate.
E. Lactate. *

24. A newborn child had dyspepsia phenomena (diarrhea, vomiting) detected after feeding with
milk. After additional feeding with glucose the morbid symptoms disappeared. The insufficient
activity of what enzyme that takes part in the carbohydrates breakdown causes the indicated
disorders?
A. Saccharase.
B. Amylase.
C. Lactase. *
D.lsomaltase.
E. Maltase.
25. A 2-year-old boy has the increase of liver and spleen sizes detected and eye cataract present.
The total sugar level in blood is increased, but glucose tolerance is within the normal range. The
inherited disturbance of the metabolism of what substance is the cause of the indicated state?
A. Glucose.
B. Fructose.
C. Galactose. *
D. Maltose.
E. Saccharose.

26. A 57-year-old patient, suffering from insulin dependent diabetes mellitus, showed the
development of ketoacidosis. The biochemical mechanism of the development of this pathology
is decreasing of acetyl-CoA utilization due to the deficiency of:
A. 2-0xoglutarate.
B. Oxaloacetate.*
C. Glutamate.
D. Aspartate.
E. Succinate.

27. A 38-year-old man is receiving treatment for schizophrenia in hospital. The initial levels of
glucose, ketone bodies and urea in the blood are within the normal range. Shock therapy put into
practice by regular insulin injections resulted in the development of the comatose state which
improved the clinical status of the patient. What is the most probable cause of insulin coma?
A. Hyperglycemia.
B. Dehydratation of tissues.
C. Metabolic acidosis.
D. Ketonemia.
E. Hypoglycemia. *

28. A 7-year-old girl manifests obvious signs of anemia. Laboratory tests showed the deficiency
of pyruvate kinase activity in erythrocytes. The disorder of what biochemical process is a major
factor in the development of anemia?
A. Deamination of amino acid.
B. Oxidative phosphorylation.
C. Tissue respiration.
D. Breaking up of peroxides.
E. Anaerobic glycolysis. *

29. A 45-year-old woman does not have any symptoms of insulin dependent diabetes mell itus
but testing on an empty stomach showed the increase of the blood glucose level (7.5 mM/l).
What additional laboratory test needs to be done to substantiate the diagnosis?
A. Determination of tolerance to glucose.
B. Determination of ketone bodies concentration in the urine.
C. Determination of rest nitrogen level in the blood.
D. Determination of tolerance to glucose on an empty stomach. *
E. Determination of glycosylated hemoglobin level.

30. What biochemical process is stimulated in the liver and kidneys of a patient exhausted by
starvation?
A. Synthesis of bilirubin.
B. Synthesis of urea.
C. Gluconeogenesis. *
D.Formation of hippuric acid.
E. Synthesis of uric acid.
Metabolism of lipids
1. After consumption of rich food a patient has nausea and heartburn, steatorrhea. Thiscondition
might be caused by:
A Bile acid deficiency*
B Increased lipase secretion
C Disturbed tripsin synthesis
D Amylase deficiency
E Disturbed phospholipase synthesis

2. Fatty of phospholipids is disordered due to fat infiltration of the liver. Indicate which of the
presented substances can enhance the process of methylation during phospholipids synthesis?
A Methionine *
B Ascorbic acid
C Glucose
D Glycerin
E Citrate

3. Increased amount of free fatty acids is observed in the blood of the patients with diabetes
mellitus. It can be caused by:
A Increased activity of triglyceridelipase adipocytes*
B Storage of palmitatoil-CoA
C Activation of the ketone bodies utilization
D Activation of the synthesis of the apolipoproteins
E Decreased activity of phosphatidylcholine-cholesterol-acyltransferase blood plasma

4. A patient with high rate of obesity was advised to use carnitine as a food additive in order to
enhance "fat burning". What is the role of carnitine in the process of fat oxidation?
A Transport of FFA (free fatty acids) from cytosol to the mitochondria *
B Transport of FFA from fat depots to the tissues
C It takes part in one of reactions of FFA beta-oxidation
D FFA activation
E Activation of intracellular lipolysis

5. An experimantal animal that was kept on protein-free diet developed fatty liver infiltration, in
particular as a result of deficiency of methylating agents. This is caused by disturbed generation
of the following metabolite:
A Choline *
B DOPA
C Cholesterol
D Acetoacetate
E Linoleic acid

6. Carnitine including drug was recomended to the sportsman for improving results. What
process is activated most of all with help of carnitine?
A Transport of fatty acids to the mitochondria*
B Synthesis of steroid hormones
C Synthesis of ketone bodies
D Synthesis of lipids
E Tissue respiratio
7. After intake of rich food a patient feels nausea and sluggishness; with time there appeared
signs of steatorrhea. Blood cholesterine concentration is 9,2 micromole/l. This condition was
caused by lack of:
A Bile acids *
B Triglycerides
C Fatty acids
D Phospholipids
E Chylomicrons

8. Examination of a man who hadn't been consuming fats but had been getting enough
carbohydrates and proteins for a long time revealed dermatitis, poor wound healing, vision
impairment. What is the probable cause of metabolic disorder?
A Lack of linoleic acid, vitamins A, D, E, K *
B Lack of palmitic acid
C Lack of vitamins PP, H
D Low caloric value of diet
E Lack of butiric acid

9. An experimental animal has been given excessive amount of carbon-labeled glucose for a
week. What compound can the label be found in?
A Palmitic acid *
B Methionine
C Vitamin A
D Choline
E Arachidonic acid

10. A sportsman was recommended to take a medication that contains carnitine in order to
improve his results. What process is activated by carnitine the most?
A Fatty acids transport to mitochondrions *
B Synthesis of steroid hormones
C Synthesis of ketone bodies
D Synyhesis of lipids
E Tissue respiration

11. Examination of a patient suffering from chronic hepatitis revealed a significant decrease in
the synthesis and secretion of bile acids. What process will be mainly disturbed in the patient’s
bowels?
A Fats emulsification *
B Protein digestion
C Carbohydrate digestion
D Glycerin absorption
E Amino acid absorption

12. A 6 year old child was delivered to a hospital. Examination revealed that the child couldn't
fix his eyes, didn't keep his eyes on toys, eye ground had the cherry-red spot sign. Laboratory
analyses showed that brain, liver and spleen had high rate of ganglioside glycometide. What
congenital disease is the child ill with?
A Tay-Sachs disease *
B Wilson's syndrome
C Turner's syndrome
D Niemann-Pick disease
E MacArdle disease
13. NSAID blockade the utilization of arachidonic acid via cyclooxigenase pathway, which
results in formation of some bioactive substances. Name them:
A Prostaglandins *
B Thyroxine
C Biogenic amins
D Somatomedins
E Insulin-like growth factors

14. Arachidonic acid, an essential component of a human diet, acts as a precursor of the vitally
important physiologically active biomolecules. Which substances are synthesized via
cyclooxigenase pathway from arachidonic acid?
A. Ethanolamine.
B. Choline.
C. Noradrenaline.
D. Prostaglandins *
E. Triiodothyronine.

15. A 1-year-old child with symptoms of muscle involvement was admitted to the hospital.
Examination revealed carnitine deficiency in his muscles. What process disturbance is the
biochemical basis of this pathology?
A Transporting of fatty acids to mitochodrions *
B Regulation of Са2+ level in mitochondrions
C Substrate phosphorylation
D Lactic acid utilization
E Actin and myosin synthesis

16. Laboratory investigation of the patient's blood plasma, which was performed 4 hours after a
consumption of a fat diet, displayed a marked increase of plasma turbidity. The most credible
cause of this phenomenon is the increase of …….... in the plasma.
A. HDL.
B. Chylomicrons.*
C. LDL.
D. Cholesterol.
E. Phospholipids.

17. Patients who suffer from severe diabetes and don't receive insulin have metabolic acidosis.
This is caused by increased concentration of the following metabolites:
A Ketone bodies *
B Fatty acids
C Unsaturated fatty acids
D Triacylglycerols
E Cholesterol

18. In a human body the adipose tissue is the basic location of triacylglycerols (TAG) deposit. At
the same time their synthesis takes place in hepatocytes. In the form of what molecular complex
are TAG transported from the liver into the adipose tissue?
A. Chylomicrons.
B. VLDL.*
C. LDL.
D. HDL.
E. Complexes with albumin.
19. Laboratory investigation of a patient revealed a high level of plasma LDL. What disease can
be diagnosed?
A. Gastritis.
B. Nephropathy.
C. Acute pancreatitis.
D.Atherosclerosis*.
E. Pneumonia.

20. Aerobic oxidation of substrates is typical for cardiac myocytes. Which of the following is the
major oxidation substrate of cardiac muscles?
A. Fatty acids.*
B. Triacylglycerols.
C. Glycerol.
D. Glucose.
E. Amino acids.

21. Which of the following enzymes accelerates the lipolysis under the action of epinephrine in
stress situations?
A. Triacylglycerol lipase. *
B. Lypoprotein lipase.
C. Phospholipase A2
D. Phospholi pase C.
E. Cholesterol esterase.

22. Clinical signs and laboratory testing of a patient allow make the assumption of gall-bladder
inflammation, colloid properties of bile disorder and occurrence of gall-stones. Which substances
can underlie the formation of gall-stones?
A. Oxalates.
B. Urates.
C. Cholesterol. *
D. Chlorides.
E. Phosphates.

23. Emotional stress causes activation of hormon-sensitive triglyceride lipase in the adipocytes.
What secondary mediator takes part in this process?
A Cyclic adenosine monophosphate *
B Cyclic guanosine monophosphate
C Adenosine monophosphate
D Diacylglycerol
E Ions of Са2+

24. The insufficient secretion of what enzyme is the cause of incomplete fats degradation in the
digestive tract and appearance of great quantity of neutral fats in feces?
A. Pepsin.
B. Phospholipase.
C. Enterokinase.
D.Amylase.
E. Pancreatic lipase *

Metabolism of simple proteins


1. Patient with encephalopathy was admitted to the neurological in-patient department.
Correlation of increasing of encephalopathy and substances absorbed by the bloodstream from
the intestines was revealed. What substances that are created in the intestines can cause
endotoxemia?
A Indole*
B Butyrate
C Acetacetate
D Biotin
E Ornithine

2. Examination of a patient suffering from cancer of urinary bladder revealed high rate of
serotonin and hydroxyanthranilic acid. It is caused by excess of the following amino acid in
the organism:
A Tryptophan*
B Alanine
C Histidine
D Methionine
E Tyrosine

3. A 4 y.o. child with signs of durative protein starvation was admitted to the hospital. The signs
were as follows: growth inhibition, anemia, edema, mental deficiency. Choose a cause of edema
development:
A Reduced synthesis of albumins *
B Reduced synthesis of globulins
C Reduced synthesis of hemoglobin
D Reduced synthesis of lipoproteins
E Reduced synthesis of glycoproteins

4. The concentration of albumins in human blood sample is lower than normal. This leads to
edema of tissues. What blood function is damaged?
A Maintaining the oncotic blood pressure *
B Maintaining the Ph level
C Maintaining the body temperature
D Maintaining the blood sedimentation system
E All answers are correct

5. Ammonia is a very toxic substance, especially for nervous system. What substance takesthe
most active part in ammonia detoxication in brain tissues?
A Glutamic acid *
B Lysine
C Proline
D Histidine
E Alanine

6. A patient has pellagra. Interrogation revealed that he had lived mostly on maize for a long
time and eaten little meat. This disease had been caused by the deficit of the following substance
in the maize:
A Tryptophan *
B Tyrosine
C Proline
D Alanine
E Histidine
7. A patient with serious damage of muscular tissue was admitted to the traumatological
department. What biochemical urine index will be increased in this case?
A Creatinine *
B Common lipids
C Glucose
D Mineral salts
E Uric acid

8. Nappies of a newborn have dark spots that witness of formation of homogentisic acid.
Metabolic imbalance of which substance is it connected with?
A Thyrosine *
B Galactose
C Methionine
D Cholesterine
E Tryptophane

9. A 1,5-year-old child presents with both mental and physical lag, decolorizing of skin and hair,
decrease in catecholamine concentration in blood. When a few drops of 5% solution of
trichloroacetic iron had been added to the child’s urine it turned olive green. Such alteration are
typical for the following pathology of the amino acid metabolism:
A Phenylketonuria *
B Alkaptonuria
C Tyrosinosis
D Albinism
E Xanthinuria

10. The greater amount of nitrogen is excreted from the organism in form of urea. Inhibition of
urea synthesis and accumulation of ammonia in blood and tissues are induced by the
decreased activity of the following liver enzyme:
A Carbamoyl phosphate synthetase *
B Aspartate aminotransferase
C Urease
D Amylase
E Pepsin

11. After a serious viral infection a 3-year-old child has repeated vomiting, loss of
consciousness, convulsions. Examination revealed hyperammoniemia. What may have caused
changes of biochemical blood indices of this child?
A Disorder of ammonia neutralization in ornithinic cycle *
B Activated processes of aminoacids decarboxylation
C Disorder of biogenic amines neutralization
D Increased purtefaction of proteins in intestines
E Inhibited activity of transamination enzymes

12. Albinos can't stand sun impact - they don't aquire sun-tan but get sunburns. Disturbed
metabolism of what aminoacid underlies this phenomenon?
A Phenilalanine *
B Methionine
C Tryptophan
D Glutamic acid
E Histidine

13. Glutamate decarboxylation results in formation of inhibitory transmitter in CNS. Name it:
A GABA *
B Glutathione
C Histamine
D Serotonin
E Asparagine

14. In course of histidine catabolism a biogenic amin is formed that has powerful vasodilatating
effect. Name it:
A Histamine *
B Serotonin
C Dioxyphenylalanine
D Noradrenalin
E Dopamine

15. A patient diagnosed with carcinoid of bowels was admitted to the hospital. Analysis revealed
high production of serotonin. It is known that this substance is formed of tryptophane
aminooacid. What biochemical mechanism underlies this process?
A Decarboxylation *
B Desamination
C Microsomal oxydation
D Transamination
E Formation of paired compounds

16. During hypersensitivity test a patient got subcutaneous injection of an antigen which caused
reddening of skin, edema, pain as a result of histamine action. This biogenic amine is generated
as a result of transformation of the following histidine amino acid:
A Decarboxylation *
B Methylation
C Phosphorylation
D Isomerization
E Deaminization

17. A patient complained about dizziness, memory impairment, periodical convulsions. It was
revealed that these changes were caused by a product of decarboxylation of glutamic acid. Name
this product:
A GABA *
B Pyridoxal phosphate
C TDP
D ATP
E THFA

18. Laboratory examination of a child revealed increased concentration of leucine, valine,


isoleucine and their ketoderivatives in blood and urine. Urine smelt of maple syrup. This disease
is characterized by the deficit of the following enzyme:
A Dehydrogenase of branched amino acids *
B Aminotransferase
C Glucose-6-phosphatase
D Phosphofructokinase
E Phosphofructomutase
19. A newborn child was found to have reduced intensity of sucking, frequent vomiting,
hypotonia. In urine and blood exhibit increased concentration of citrulline. What metabolic
process is disturbed?
A Ornithinic cycle *
B Tricarboxylic acid cycle
C Glycolysis
D Glyconeogenesis
E Cori cycle

20. Plasmic factors of blood coagulation are exposed to post-translational modification with the
participation of vitamin K. It is necessary as a cofactor in the enzyme system of gamma-
carboxylation of protein factors of blood coagulation due to the increased affinity of their
molecules with calcium ions. What amino acid is carboxylated in these proteins?
A Glutamate *
B Valine
C Serine
D Phenylalanine
E Arginine

21. Pharmacological effects of antidepressants are connected with inhibition of an enzyme


catalyzing biogenic amines noradrenaline and serotonine in the mitochondrions of cerebral
neurons. What enzyme participates in this process?
A Monoamine oxidase *
B Transaminase
C Decarboxylase
D Peptidase
E Lyase

22. A child manifests epileptic seizures caused by vitamin B6 deficiency. This is conditioned by
the decrease of the gamma-aminobutyrate level in the nervous tissue which acts as an inhibiting
neurotransmitter. The activity of which enzyme is decreased in this case?
A. Pyridoxal kinase.
B. Alanine aminotransferase.
C. Glutamate dehydrogenase.
D. Glutamate decarboxylase.*
E. Glutamate synthetase.

Metabolism of complex proteins


1. Methotrexate (competitive inhibitor of the dihydrofolatreductase) is prescribed for treatment
of the tumour.On which level does methotrexate inhibit synthesis of the nucleic acids?
A Mononucleotide synthesis *
B Replication
C Transcription
D Reparation
E Processing

2. Blood of a 12 year old boy presents low concentration of uric acid and accumulation of
xanthine and hypoxanthine. This child has genetic defect of the following enzyme:
A Xanthine oxidase*
B Arginase
C Urease
D Ornithine carbamoyltransferase
E Glycerylkinase
3.An experiment proved that UV-radiated cells of patients with xeroderma pigmentosum restore
the native DNA structure slower than cells of healthy individuals as a result of reparation
enzyme defect. What enzyme helps this process?
A Endonuclease *
B RNA ligase
C Primase
D DNA polymerase III
E DNA gyirase

4. A 20 year old patient complains of general weakness, dizziness, quick fatigability. Blood
analysis results: Hb- 80 g/l. Microscopical examination results: erythrocytes are of modified
form. This condition might be caused by:
A Sickle-cell anemia *
B Hepatocellular jaundice
C Acute intermittent porphyria
D Obturative jaundice
E Addison's disease

5. A 48 year old patient complained about intense pain, slight swelling and reddening of skin
over the joints, temperature rise up to 38oC. Blood analysis revealed high concentration of urates.
This condition might be caused by disturbed metabolism of:
A Purines *
B Collagen
C Cholesterol
D Pyrimidines
E Carbohydrates

6. A patient has yellow skin colour, dark urine, achromatic feces. What substance will have
strengthened concentration in the blood serum?
A Unconjugated bilirubin
B Conjugated bilirubin *
C Mesobilirubin
D Verdoglobin
E Biliverdin

7. A 46 year old woman suffering from chololithiasis developed jaundice. Her urine became
dark-yellow and feces became colourless. Blood serum will have the highest concentration of the
following substance:
A Conjugated bilirubin *
B Unconjugated bilirubin
C Biliverdin
D Mesobilirubin
E Urobilinogen

8. A 46 year old patient applied to a doctor complaining about joint pain that becomes stronger
the day before weather changes. Blood examination revealed strengthened concentration of uric
acid. The most probable cause of the disease is the intensified disintegration of the following
substance:
A Adenosine monophosphate *
B Cytidine monophosphate
C Uridine triphosphate
D Uridine monophosphate
E Thymidine monophosphate

9. A 42-year man suffering from gout has increased level of urinary acid in the blood.
Allopurinol was prescribed to decrease the level of urinary acid. Competitive inhibitor of what
enzyme is allopurinol?
A Xanthinoxidase *
B Adenosinedeaminase
C Adeninephosphoribosiltransferase
D Hypoxantinphosphoribosiltransferase
E Guaninedeaminase

10. Patient experienced increased susceptibility of the skin to the sunlight. His urine after some
time became dark-red. What is the most likely cause of this?
A Porphyria *
B Hemolytic jaundice
C Albinism
D Pellagra
E Alkaptonuria

11. A 65 year old man suffering from gout complains of kidney pain. Ultrasound examination
revealed renal calculi. The most probable cause of calculi formation is the strengthened
concentration of the following substance:
A Uric acid *
B Cholesterol
C Bilirubin
D Urea
E Cystine

12. It was found out that some compounds, for instance fungi toxins and some antibiotics can
inhibit activity of RNA-polymerase. What process will be disturbed in a cell in case of inhibition
of this enzyme?
A Transcription *
B Processing
C Replication
D Translation
E Reparation

13. An oncological patient was prescribed methotrexate. With the lapse of time target cells of the
tumour lost susceptibility to this drug. There is change of gene expression of the folowing
enzyme:
A Dehydrofolate reductase *
B Thiaminase
C Deaminase
D Folate oxidase
E Folate decarboxylase

Functional biochemistry
1. An infant has apparent diarrhea resulting from improper feeding. One of the main diarrhea
effects is plentiful excretion of sodium bicarbonate. What form of acid-base balance disorder is
the case?
A Metabolic acidosis*
B Metabolic alkalosis
C Respiratory acidosis
D Respiratory alkalosis
E No disorders of acid-base balance will be observed

2. Buffer capacity of blood was decreased in the worker due to exhausting muscular work. Entry
of what acid substance to the blood can this state be explained?
A Lactate*
B Pyruvate
C 1,3-bisphosphoglycerate
D alpha-ketoglutarate
E 3-phosphoglycerate

3. A 38 year old patient suffers from rheumatism in its active phase. What laboratory
characteristic of blood serum is of diagnostic importance in case of this pathology?
A C-reactive protein *
B Uric acid
C Urea
D Creatinine
E Transferrin

4. A 30 y.o. woman had been ill for a year when she felt pain in the area of joints for the first
time, they got swollen and skin above them became reddened. Provisional diagnosis is
rheumatoid arthritis. One of the most probable causes of this disease is a structure alteration of a
connective tissue protein:
A Collagen *
B Mucin
C Myosin
D Ovoalbumin
E Troponin

5. Examination of a 27-year-old patient revealed pathological changes in liver and brain. Blood
plasma analysis revealed an abrupt decrease in the copper concentration, urine analysis revealed
an increased copper concentration. The patient was diagnosed with Wilson’s degeneration. To
confirm the diagnosis it is necessary to study the activity of the following enzyme in blood
serum:
A Ceruloplasmin *
B Carbonic anhydrase
C Xanthine oxidase
D Leucine aminopeptidase
E Alcohol dehydrogenase

6. A patient complains about dyspnea provoked by the physical activity. Clinical examination
revealed anaemia and presence of the paraprotein in the zone of gamma-globulins. To confirm
the myeloma diagnosis it is necessary to determine the following index in the patient’s urine:
A Bence Jones protein *
B Bilirubin
C Haemoglobin
D Ceruloplasmin
E Antitrypsin
7. A 62 y.o. woman complains of frequent pains in the area of her chest and backbone, rib
fractures. A doctor assumed myelomatosis (plasmocytoma). What of the following laboratory
characteristics will be of the greatest diagnostical importance?
A Paraproteinemia *
B Hyperalbuminemia
C Proteinuria
D Hypoglobulinemia
E Hypoproteinemia

8. Pathological changes of the liver and brain were revealed in a 27-year-old patient.The copper
concentration is abruptly decreased in blood plasma and increased in the urine. Wilson's disease
was diagnosed. Activity of what enzyme in the blood serum should be examined to prove
diagnisis?
A Ceruloplasmin*
B Carboanhydraze
C Xanthioxidase
D Leucinamineopeptidaze
E Alcoholdehydrogenaze

9. A 50-year-old patient complains about general weakness, appetite loss and cardiac arrhythmia.
The patient presents with muscle hypotonia, flaccid paralyses, weakened peristaltic activity of
the bowels. Such condition might be caused by:
A Hypopotassemia *
B Hypoproteinemia
C Hyperkaliemia
D Hypophosphatemia
E Hyponatremia

10. A 63-year-old woman developed signs of rheumatoid arthritis. Increase of which indicated
blood values level could be helpful in proving diagnosis?
A Additive glycosaminoglycans*
B Lipoproteids
C Acid phosphatase
D General cholesterol
E R-glycosidase

11. A 35 y.o. patient who often consumes alcohol was treated with diuretics. There appeared
serious muscle and heart weakness, vomiting, diarrhea, AP- 100/60 mm Hg, depression. This
condition is caused by intensified excretion with urine of:
A Potassium *
B Sodium
C Chlorine
D Calcium
E Phosphates

12. A patient suffers from hepatic cirrhosis. Examination of which of the following substances
excreted by urine can characterize the state of antitoxic function of liver?
A Hippuric acid *
B Ammonium salts
C Kreatinine
D Uric acid
E Aminoacids
13. Products of some proteins hydrolysis and modification are the biologically active substances
called hormones. Lipotropin, corticotropin, melanotropin and endorphins are synthesized in the
hypophysis of the following protein:
A Proopiomelanocortin (POMC) *
B Neuroalbumin
C Neurostromin
D Neuroglobulin
E Thyreoglobulin

14. During examination of an 11-month-old infant a pediatrician revealed osteoectasia of the


lower extremities and delayed mineralization of cranial bones. Such pathology is usually
provoked by the deficit of the following vitamin:
A Cholecalciferol *
B Thiamin
C Pantothenic acid
D Bioflavonoids
E Riboflavin

15. Desulfiram is widely used in medical practice to prevent alcocholism. It inhibits aldehyde
dehydrogenase. Increased level of what metabolite causes aversion to alcochol?
A Acetaldehyde *
B Ethanol
C Malonyl aldehyde
D Propionic aldehyde
E Methanol

16. Index of pH of the blood changed and became 7,3 in the patient with diabetus mellitus.
Detecting of the components of what buffer system is used while diagnosing disorder of the acid-
base equilibrium?
A Bicarbonate*
B Phosphate
C Hemoglobin
D Oxyhemoglobin
E Protein

17. Diabetes mellitus causes ketosis as a result of activated oxidation of fatty acids. What
disorders of acid-base equilibrium may be caused by excessive accumulation of ketone bodies in
blood?
A Metabolic acidosis *
B Metabolic alcalosis
C Any changes woun't happen
D Respiratory acidosis
E Respiratory alcalosis

18. Depressions and emotional insanities result from the deficit of noradrenalin, serotonin and
other biogenic amines in the brain. Their concentration in the synapses can be increased by
means of the antidepressants that inhibit the following enzyme:
A Monoamine oxidase *
B Diamine oxidase
C L-amino-acid oxidase
D D-amino-acid oxidase
E Phenylalanine-4-monooxygenase
19. A patient with suspected diagnosis "progressing muscular dystrophy" got his urine tested.
What compound will confirm this diagnosis if found in urine?
A Kreatine *
B Collagen
C Porphyrin
D Myoglobin
E Calmodulin

20. Dietary intake of a 30 year old nursing woman contains 1000 mg of calcium, 1300 mg of
phosphorus and 20 mg of iron per day. It is necessary to change content of these mineral
substances in the following way:
A To increase phosphorus content *
B To increase calcium content
C To reduce fluorine content
D To increase iron content
E To reduce iron content

21. Cardinal symptoms of primary hyperparathyroidism are osteoporosis and renal lesion along
with development of urolithiasis. What substance makes up the basis of these calculi in this
disease?
A Calcium phosphate *
B Uric acid
C Cystine
D Bilirubin
E Cholesterol

22. Study of conversion of a food colouring agent revealed that neutralization of this xenobiotic
takes place only in one phase - microsomal oxydation. Name a component of this phase:
A Cytochrome Р-450 *
B Cytochrome B
C Cytochrome C
D Cytochrome A
E Cytochrome oxidase

23. A patient had hemorrhagic stroke. Blood examination revealed strengthened kinin
concentration.The patient was prescribed contrical. It was administered in order to inhibit the
following proteinase:
A Kallikrein *
B Pepsin
C Trypsin
D Chemotrypsin
E Collagenase

24. A 42-year-old man was hospitalized to a cardiologic department with the diagnosis of
stenocardia. The inhibitor of phosphodiesterase was included in the medicinal treatment of the
patient The concentration of what substance will be increased in the cardiac muscle?
A.ADP.
B.GMP.
C.AMP
D.cAMP.*
E. ATP.
25. Under different pathological states the level of active forms of oxygen rises, which results in
the destruction of cellular membranes. In order to prevent the damage of membranes,
antioxidants are used. The most powerful natural antioxidant is:
A.a-Tocoferol.*
B. Glucose.
C. Vitamin A.
D.Fatty acids.
E. Glycerol.

26. A 4 year old child with hereditary renal lesion has signs of rickets, vitamin D concentration
in blood is normal. What is the most probable cause of rickets development?
A Impaired synthesis of calcitriol *
B Increased excretion of calcium
C Hyperfunction of parathyroid glands
D Hypofunction of parathyroid glands
E Lack of calcium in food

27. A doctor examined a child and revealed symptoms of rachitis. Development of this desease
was caused by deficiency of the following compound:
A 1,25 [ОН]-dichydroxycholecalciferol *
B Biotin
C Tocopherol
D Naphtaquinone
E Retinol
V ebf68bc9

Krok \ Dentistry \ Krok 1 \ Bases \ Biochemistry \ 2010


1 Roentgenological examination of a patient revealed a cyst enclosing a tooth in its cavity in
the area of the premolar. Microscopical examination revealed that the cyst wall consisted of
connective tissue and was lined with multilayer squamous epithelium. What is the most
probable diagnosis?

A Follicular cyst
B Radicular cyst
C Primordial cyst
D Eosinophilic granuloma
E Epulis

2 A dentist was examining oral cavity of a 9 year old child in the buccal surface of gingiva in
the area of the lower canine he revealed a red, soft, node-like formation 1 cm in diameter
that started immediately bleeding when touched. Microscopical examination revealed that
this formation consisted of many small vessels like venules and capillaries separated by thin
layers of connective tissue, with focal infiltration by lymphoid and plasmatic cells. Such
changes are typical for:

A Angiomatous epulis
B Capillary hemangioma
C Radicular granuloma
D Fibrous epulis
E Papilloma

3 Analysis of urine from a 24-year-old man revealed the following changes: daily diuresis -
10 l, relative density - 1,001, qualitative alterations are absent. A patient complains of
excessive thirst, frequent urination. What is the most likely cause of this disease?

A Vasopressin hyposecretion
B Glucocorticoid hypersecretion
C Vasopressin hypersecretion
D Relative insulin insufficiency
E Aldosteron hypersecretion

4 A patient suffering from syphilis was prescribed a drug the action of which based upon
disturbed generation of murein leading to death of the causative agent. What drug is it?

A Benzylpenicillin sodium salt


B Bijochinol
C Ciprofloxacin
D Azithromycin
E Doxycycline hydrochloride
V ebf68bc9

5 Osteolathyrism is characterized by a loss of tensile strength of collagen, which is induced


by a significant decrease in the formation of cross-links in collagen fibrils. The cause for it is
the reduced activity of:

A Lysyl oxidase
B Monoamine oxidase
C Prolyl hydroxylase
D Lysyl hydroxylase
E Collagenase

6 Periodontitis is accompanied by activation of proteolysis in the periodontium tissues. The


evidence of proteolysis activation is increase of the following component of oral liquid:

A Amino acids
B Organic acids
C Glucose
D Biogenic amines
E Cholesterol

7 Two weeks after hemotransfusion a patient developed fever. What protozoal disease can
be suspected?

A Malaria
B Toxoplasmosis
C Leishmaniasis
D Amebiasis
E Trypanosomiasis

8 A patient applied to a doctor complaining about dizziness, memory impairment,


periodical convulsions. It was found out that such changes were caused by a product of
glutamic acid decarboxylation. What product is meant?

A GABA
B Pyridoxalphosphate
C Thymidine diphosphate
D ATP
E Tetrahydrofolate

9 Laboratory examination of a child revealed high content of leucine, valine, isoleucine and
their ketoderivates in blood and urine. Urine had the typical smell of maple syrup. This
disease was caused by deficiency of the following enzyme:

A Dehydrogenase of branched amino acids


B Aminotransferase
C Glucose-6-phosphatase
D Phosphofructokinase
V ebf68bc9

E phosphofructomutase

10 Hepatic disfunctions accompanied by insufficient inflow of bile to the bowels result in


coagulation failure. This phenomenon can be explained by:

A Vitamin K deficiency
B Iron deficiency
C Thrombocytopenia
D Erythropenia
E Leukopenia

11 A child has disturbed processes of ossification and punctate enamel. What microelement
metabolism is disturbed?

A Fluorine
B Chromium
C Copper
D Iron
E Zinc

12 During examination of a 36-year-old woman a dentist revealed a formation in form of a


nodule up to 0,8 cm in diameter, of dark brown-red colour, soft, on a wide base. The
formation was found on the buccal surface of gum in the region of the 2nd molar.
Histological examination revealed that the formation had plenty of sinusoid vessels and a
lot of roundish mononuclear and big multinuclear cells; in some parts accumulations of
hemosiderin granules could be found. What is the most likely diagnosis?

A Giant-cell epulis
B Root granuloma
C Angiomatous epulis
D Ameloblastoma
E Mandibular osteoclastoma

13 While on holiday in the countryside a boy found a spider with the following
morphological peculiarities: body length at the rate of 2 cm, round black abdomen with two
rows of red dots on its dorsal surface, four pairs of segmented extremities covered with tiny
black hairs. Identify this arthropod:

A Steppe spider (Latrodectus tredeci-mguttatus)


B Scorpion
C Solifugae
D Mite
E Tarantula

14 A girl who was provisionally diagnosed with Turner’s syndrome came to a genetic
consultation. The diagnosis can be specified by means of the following genetic method:
V ebf68bc9

A Sex chromatin test


B Genealogical
C Hybridological
D Biochemical
E Dermatoglyphics

15 Cyanide poisoning causes immediate death. What is the mechanism of cyanide effect at
the molecular level?

A They inhibit cytochromoxidase


B They bind substrates of tricarboxylic acid cycle
C They block succinate dehydrogenase
D They inactivate oxygene
E They inhibit cytochrome B

16 A blood smear of a patient who has recently recovered from flu contains 10% of
roundish cells 4,5-7 micrometer large with a big round nucleus and basophi-lically stained
cytoplasm in form of a narrow border around the nucleus. What blood status are they
typical for?

A Lymphocytopenia
B Thrombopenia
C Leukopenia
D Lymphocytosis
E Monocytopenia

17 A patient was diagnosed with seborrheic dermatitis associated with vitamin H (biotin)
deficiency. The patient has disturbed activity of the following enzyme:

A Acetyl-CoA-carboxylase
B Pyruvate decarboxylase
C Alcohol dehydrogenase
D Amino transferase
E Carbomoyl phosphate synthetase

18 Active physical work induces rise of concentration of carbonic acid in blood. This causes
deepening and acceleration of respiration thus reducing concentration of carbonic acid and
hydrogen ions in blood. This maintains the following process:

A Homeostasis
B Immunity
C Ontogenesis
D Orthobiosis
E Anabiosis
V ebf68bc9

19 The activity of parotides reduces with age. Activity of what enzyme in saliva will be
reducing?

A Amylase
B Lysozime
C Phosphatase
D Hexokinase
E Maltase

20 Chronic overdosage of glucocorticoids leads to the development of hyperglycemia. What


process of carbohydrate metabolism is responsible for this effect?

A Gluconeogenesis
B Glycogenolysis
C Aerobic glycolisis
D Pentose-phosphate cycle
E Glycogenesis

21 A patient has mental retardation, small height, brachydactyly, mongoloid slant. Analysys
of his karyotype revealed trisomy 21. What chromosomal anomaly is it?

A Down’s disease
B Klinefelter’s syndrome
C Turner’s syndrome
D Trisomy X
E Specific fetopathy

22 Examination of urine in a newborn revealed presence of citrulline and high ammonia


concentration. This baby is most likely to have the disorder of the following substance
production:

A Urea
B Uric acid
C Ammonia
D Creatinine
E Creatine

23 In the mountains some clinically healthy people present with anaemia symptoms. Blood
test can reveal sickle cells. What is the genotype of such people?

A Aa
B aa
C AA
D XcXc
E XCXc
V ebf68bc9

24 To what total ATP quantity is the full glucose oxidation and its linking with
phosphorylation equivalent?

A 38
B8
C 12
D 52
E 58

25 While examining a blood smear taken form a patient and stained by Romanovsky’s
method a doctor revealed some protozoa and diagnozed the patient with Chagas disease.
What protozoan is the causative agent of this disease?

A Trypanosoma cruzi
B Toxoplasma gondii
C Leishmania donovani
D Leishmania tropica
E Trypanosoma brucei

26 A 10 year old child underwent Mantoux test (with tuberculin). 48 hours later there
apperaed a papule up to 8 mm in diameter on the site of tuberculin injection. Tuberculin
injection caused the following hypersensitivity reaction:

A IV type hypersensitivity reaction


B Arthus reaction
C Seroreaction
D Atopic reaction
E II type hypersensitivity reaction

27 A chiled was diagnosed with acute renal failure. What biochemic saliva indices can
confirm this diagnosis?

A Increased level of rest nitrogen


B Increase of immunoglobuline A
C Reduction of alkaline phosphatase
D Increase of alpha amylase
E Decreased level of phosphate

28 A 28 year old pregnant woman had the enzymes in the cells of amniotic fluid analyzed.
The analysis revealed insufficient activity of beta-glucuronidase. What pathological process
is it?

A Mucopolysaccharidosis
B Glycogenosis
C Collagenosis
D Lipidosis
V ebf68bc9

E Aglycogenosis

29 A patient with chronic hypoglycemia had adrenaline introduction. After introduction


blood test has not changed essentially. Doctor assumed liver pathology. What liver function
may have been changed?

A Function of glycogen depositing


B Ketogenic function
C Function of cholesterin production
D Glycolytic function
E Excretory function

30 In order to speed up healing of the thermal injury it is required to prescribe a drug that
facilitates epithelization of skin and mucous membranes. What drug is it?

A Retinol acetate
B Tocopherol acetate
C Nicotinic acid
D Ergocalciferol
E Ascorbic acid

31 A child has abnormal formation of tooth enamel and dentin as a result of low
concentration of calcium ions in blood. Such abnormalities might be caused by deficiency of
the following hormone:

A Parathormone
B Thyrocalcitonin
C Thyroxin
D Somatotropic hormone
E Triiodothyronine

32 Microscopical examination of discharges from the gums of a patient ill with paradontosis
revealed some protozoan pear-shaped organisms 6-13 micrometer long. The parasite has
one nucleus and undulating membrane, there are four flagella at the front of its body. What
protozoan were found?

A Trichomonads
B Leishmania
C Amoebae
D Balantidia
E Lamblia

33 A patient has painfulness along big nerve trunks and excessive content of pyruvate in
blood. What vitamin deficit may cause such changes?

ABi
B B2
V ebf68bc9

C PP
D Pantothenic acid
E Biotin

34 A patient with diabetes mellitus had an insuline injection. It caused loss of consciousness
and convulsions. What was the result of biochemic blood analysis on glucose content?

A 2,5 mmole/l
B 3,3 mmole/l
C 8,0 mmole/l
D 10 mmole/l
E 5,5 mmole/l

35 Examination of a 6 days old infant revealed phenyl pyruvate and phenyl acetate excess
in his urine. What ami-noacid metabolism is disturbed in the child’s organism?

A Phenylalanine
B Tryptophan
C Methionine
D Histidine
E Arginine

36 For assessment of the neutralizing function of liver a patient with chronic hepatitis went
through a test with natrium benzoate load. The excretion of what acid with urine will
characterize the neutralizing function of liver?

A Hippuric acid
B Phenylacetic acid
C Citric acid
D Valeric acid
E Oxalic acid

37 A 42 year old woman diagnosed with diabetes mellitus was admitted the
endocrinological department with complaints of thirst, excessive appetite. What
pathological components are revealed in course of laboratory examination of the patients
urine?

A Glucose, ketone bodies


B Protein, aminoacids
C Bilirubin, urobilin
D Blood
E Protein, creatine

38 A 2 year old child suffers from intestinal dysbacteriosis that lead to the development
hemorrhagic syndrome. The most probable cause of hemorrhage is:

A Vitamin K deficiency
V ebf68bc9

B Activation of tissue thromboplastin


C Hypovitaminosis PP
D Fibrinogen deficiency
E Hypocalcemia

39 Decreased ratio of adenylic nucleotides ATP/ADP results in intensified glycolysis in


parodentium tissues in hypoxic conditions. What reaction is activated in this case?

A Phosphofructokinase
B Lactate dehydrogenase
C Triosophosphate isomerase
D Aldolase
E Enolase

40 Helminthological examination of patient’s feces revealed oval brown eggs with tuberous
external membrane. Name the type of helminth:

A Ascarid
B Pinworm
C Whipworm
D Dwarf tapeworm
E Broad tapeworm

41 A patient has the folowing changes: disorder of twilight vision, drying out of conjunctiva
and cornea. Such disorders may be caused by deficiency of vitamin:

A Vitamin A
B Vitamin B
C Vitamin C
D Vitamin D
E Vitamin B12

42 Examination of a 10 y.o. child revealed on the alveolar submandibular process a fixed


tumourous mass 1,5 cm in diameter closing premolar crown on the vestibular side. Mucous
membrane of its surface is reddish-brown, it bleeds as a reaction to a slight mechanical
intervention. Biopsy results: the mass consists of small size vessels separated by thin layers
of connective tissue and infiltrated by plasmocytes, mucous membrane is here and there
ulcered. What is the most probable diagnosis?

A Angiomatous form of epulis


B Gingival fibromatosis
C Giant cell form of epulis
D Hypertrophic gingivitis
E Fibrous form of epulis
V ebf68bc9

43 A non trained man has usually muscular hypoxy after a sprint. What metabolite
accumulates in the muscles as a result of it?

A Lactate
B Ketone bodies
C Glucose 6-phosphate
D Oxaloacetate
E-

44 A man got poisoned with mushrooms. They contain muscarine that stimulates
muscarinic cholinoreceptors. What symptom is typical for poisoning with inedible
mushrooms?

A Miosis
B Mydriasis
C Bronchi dilation
D Heart rate rise
E Arterial pressure rise

45 Myocyte cytoplasm contains a big number of dissolved metabolites of glucose oxidation.


Name one of them that turns directly into lactate:

A Pyruvate
B Oxaloacetate
C Glycerophosphate
D Glucose 6-phosphate
E Fructose 6-phosphate

46 Roentgenological examination of mandible of a 27 year old man revealed a focus of


osseous tissue destruction. Histological examination revealed a tumour consisting of
odontogenous epithelium cords, immature connective tissue and dysplastic dentin rests.
What tumour is it?

A Dentinoma
B Ameloblastic fibro-odontoma
C Odontoameloblastoma
D Odontogenous fibroma
E Complex odontoma

47 Patient with pigmentary xeroderma are characterized by anamalously high sensivity to


ultraviolet rays that causes skin cancer as a result of enzyme systems incapability to restore
damages of hereditary apparatus of cells. What process abnormality is this pathology
connected with?

A DNA reparation
B Genetic conversion
V ebf68bc9

C DNA recombination
D Genetic complementation
E DNA reduplication

48 A patient has the sudden decrease of Са2+ content in blood. What hormone secretion
will increase?

A Parathormone
B Somatotropin
C Aldosterone
D Thyrocalcitonin
E Vasopressin

49 A female patient has symptoms of inflammation of urogenital tracts. A smear from the
vaginal mucous membrane contained big unicellular pyriform organisms with a sharp spike
on the back end of their bodies; big nucleus and undulating membrane. What protozoa were
revealed in the smear?

A Trichomonas vaginalis
B Trichomonas hominis
C Trichomonas buccalis
D Trypanosoma gambiense
E Lamblia intestinalis

50 A patient has increased content of uric acid in his blood that is clinically presented by
pain syndrome as a result of urate deposition in the joints. What process does this acid
result from?

A Lysis of purine nucleotides


B Lysis of pyrimidine nucleotides
C Heme catabolism
D Proteolysis
E Reutilization of purine bases

51 Among public catering workers examined by doctors of sanitary-and-epidemiologic


station often occur asymptomatic parasite carriers. This means that a healthy person
carries cysts that infect other people. Such parasitizing is impossible for the following
causative agent:

A Dysenteric amoeba
B Malarial plasmodium
C Intestinal trichomonad
D Dermatotropic leishmania
E Viscerotropic leishmania
V ebf68bc9

52 The 16th tooth of a patient is missing. X-ray picture shows in the depth of alveolar
process rarefaction of bone and a well-defined cavity that contained the underdeveloped
tooth crown. Microscopical examination revealed that the cavity wall was lined with
stratified squamous epithelium and enclosed within a fibrous capsule. Make a diagnosis:

A Follicular gnathic cyst


B Radicular gnathic cyst
C Cyst of the incisive canal
D Cystic ameloblastoma of jaw
E Primordial gnathic cyst

53 A patient has increased permeability of blood-vessel walls, increased gingival


hemorrhage, small punctate hematomas on his skin, falling of teeth. What disturbance of
vitamun metabolism can account for these symptoms?

A Hypovitaminosis C
B Hypervitaminosis D
C Hypervitaminosis C
D Hypovitaminosis D
E Hypovitaminosis A

54 A patient has liver cirrhosis. Analysis of what substance excreted with urine may
characterize the state of antitoxic liver function?

A Hippuric acid
B Ammonian salts
C Creatinine
D Uric acid
E Amino acids

55 A sportsman needs to improve his sporting results. He was recommended a drug


containing carnitine. What process is activated by this compound in the first place?

A Transport of fatty acids


B Transport of amino acids
C Transport of calcium ions
D Transport of glucose
E Transport of vitamin K

56 A child has disturbed enamel and dentine formation as a result of decreased content of
calcium ions in his blood. What hormone deficiency may cause such changes?

A Thyreocalcitonin
B Somatotropin
C Thyroxin
D Parathormone
V ebf68bc9

E Triiodothyronine

57 A hospital admitted a patient with complaints about abdominal swelling, diarrhea,


meteorism after consumption of food rich in proteins. It is indicative of disturbed protein
digestion and their intensified decaying. What substance is the product of this process in the
bowels?

A Indole
B Bilirubin
C Cadaverine
D Agmatine
E Putrescine

58 A 38 year old patient takes aspirin and sulfanilamides. After their intake intensified
erythrocyte haemolysis is observed which is caused by deficiency of glucose 6-phosphate
dehydrogenase. This pathology is caused by failure of the following coenzyme:

A NADP - H
B FAD - H2
C Pyridoxal phosphate
D FMN - H2
E Ubiquinone

59 Up to 50% of world population aged above thirty is affected by paradontosis. The


leading part in pathogenesis of this disease is played by:

A Neurodystrophic factor
B Parodontium tissues damaged by kalli-krein
C Parodontium damaged by active cells
D Dental calculus caused by microflora
E Immune damage of tissues

60 A 57 year old patient with diabetes mellitus was developed ketoacedosis. Biochemical
base of this condition is decrease of acetyl-CoA utilization. What cell compound deficit
causes this effect?

A Oxaloacetate
B Glutamate
C 2-oxoglutarate
D Aspartate
E Succinate

61 A patient was taken to the hospital with preliminary diagnosis progressive muscle
distrophy. What substance will be excessively contained in urine and confirm this
diagnosis?

A Creatine
V ebf68bc9

B Pyruvate
C Carnosine
D Troponine
E Hydroxiproline

62 Microscopical examination of a surgical specimen (an ulcered part of a lip) revealed in


the connective tissue of mucous membrane near the borders and under the floor of the
ulcer some epithelial complexes consisting of atypic multi-stratal epithelium with
accumulations of bright pink concentric formations. What pathology is it?

A Squamous cell keratinous carcinoma


B Squamous cell nonkeratinous carcinoma
C Transitional cell carcinoma
D Basal cell carcinoma
E Papilloma

63 Examination of a child who hasn’t got fresh fruit and vegetables during winter revealed
numerous subcutaneous hemorrhages, gingivitis, carious cavities in teeth. What vitamin
combination should be prescribed in this case?

A Ascorbic acid and rutin


B Thiamine and pyridoxine
C Folic acid and cobalamin
D Riboflavin and nicotinamide
E Calciferol and ascorbic acid

64 Coprological examination of a patient’s feces revealed small operculate eggs. It is known


from the anamnesis that the patient often consumes fish. What fluke parasitizes in the
patient’s organism?

A Cat liver fluke


B Blood fluke
C Lung fluke
D Liver fluke
E Lancet fluke

65 While the examination of patient’s oral cavity the dentist found xerostomia, numerous
erosions. What vitamin deficit caused this effect?

A Vitamin A
B Vitamin K
C Vitamin P
D Vitamin H
E Vitamin PP
V ebf68bc9

66 A sportsman was recommended to take a preparation with carnitine in order to improve


his achievements. What process is activated by carnitine to the most extent?

A Transporting of fatty acids to the mi-tochondrions


B Synthesis of steroid hormones
C Synthesis of ketone bodies
D Lypide synthesis
E Tissue respiration

67 After implantation of a cardiac valve a young man constantly takes indirect


anticoagulants. His state was complicated by hemorrhage. What substance content has
decreased in blood?

A Prothrombin
B Haptoglobin
C Heparin
D Creatin
E Ceruloplasmin

68 A 28 year old woman consulted a doctor about sterility. Examination revealed


underdeveloped ovaries and uterus, irregular menstrual cycle. Study of sex chromatin
revealed 2 Barr’s bodies in most somatic cells. What chromosome disease is the most
probable in this case?

A Triplo-X syndrome
B Edwards’ syndrome
C Patau’s syndrome
D Klinefelter’s syndrome
E Turner’s syndrome

69 A patient who is ill with scurvy displays disturbed processes of connective tissue
formation that leads to loosening and falling of teeth. Disturbed activity of what enzyme
causes these symptomps?

A Lisilhydroxylase
B Glycosiltransferase
C Elastase
D Procollagenpeptidase of N-terminal peptide
E Procollagenpeptidase of C-terminal peptide

70 During physical exercise people are less sensitive to pain. The reason for it is the
activation of:

A Antinociceptive system
B Nociceptive system
C Thyroid gland functions
V ebf68bc9

D Sympathoadrenal system
E Adrenal gland functions

71 A one year old child has enlarged head and belly, retarded cutting of teeth, destruction of
enamel structure. What hypovitaminosis causes these changes?

A Hypovitaminosis D
B Hypovitaminosis C
C Hypovitaminosis A
D Hypovitaminosis B 1
E Hypovitaminosis B2

72 A sample taken from the pharynx of a patient with angina was inoculated on the blood-
tellurite agar. This resulted in growth of grey, radially striated (in form of rosettes) colonies
up to 4-5 mm in diameter. Microscopically there can be seen gram-positive rods with club-
shaped ends arranged in form of spread fingers. What microorganisms are these?

A Corynebacteria diphtheriae
B Clostridium botulinum
C Diphtheroids
D Streptococci
E Streptobacilli

73 A patient with high obesity was recommended to take carnitine as a food additive for
better fat burning. What function is fulfilled by carnitine in the process of fat oxidation?

A Transport of fatty acids from the cytosol to the mitochondria


B Transport of fatty acids from the fat depots to the tissues
C Participation in one of the reactions of beta-oxidation of fatty acids
D Fatty acid activation
E Intracellular lipolysis activation

74 Laboratory of extremely dangerous infections received a sample taken from a patient


with assumed cholera. What express-diagnostics method can confirm this diagnosis?

A Immunofluorescence test
B Complement binding reaction
C Agglutination test
D Precipitation reaction
E Hemagglutination reaction

75 A patient with focal tuberculosis of superior lobe of his right lung takes isoni-azid as a
part of combined therapy. After a time he started complaining of muscular weakness,
decrease of skin sensitivity, sight and movement coordination disorder. What vitamin
preparation will be right for elimination of these occurances?

A Vitamin B6
V ebf68bc9

B Vitamin A
C Vitamin D
D Vitamin B12
E Vitamin C

76 The first grade pupils went through a medical examination aimed at selection of children
needing tuberculosis revaccination. What test was applied?

A Mantoux test
B Schick test
C Supracutaneous tularin test
D Burne test
E Anthracene test

77 A 15 year old girl was delivered to the hospital with inflammation of vermiform
appendix. Blood analysis revealed signs of anaemia. Her feces contained lemon-shaped
helminthic eggs (50x30 micrometer) with "plugs" on the poles. What type of helminth is it?

A Trichuris
B Hookworm
C Hymenolepis nana
D Echinococcus
E Pinworm

78 A patient has roundish ulcers on his face, inflammation and enlargement of lymph nodes.
These symptoms turned up as a result of mosquito bites. Laboratory examination of
discharge from the ulcers revealed unicellular aflagellar organisms. What is the most
probable diagnosis?

A Dermatotropic leishmaniasis
B Toxoplasmosis
C Scabies
D Trypanosomiasis
E Myasis

79 A patient has deformation of jaw bones. Histological examination revealed there growth
of fibrocellular tumourlike ill-defined tissue with primitive osteogenesis. What disease are
these presentations typical for?

A Fibrous dysplasia
B Ameloblastoma
C Osteosarcoma
D Eosinophilic granuloma
E Parathyroid osteodystrophy
V ebf68bc9

80 Cytogenetic examination of a patient with dysfunction of the reproductive system


revealed normal karyotype 46,XY in some cells, but most cells have Klinefelter’s syndrome
karyotype - 47,XXY Such phenomenon of cell inhomogeneity is called:

A Mosaicism
B Inversion
C Transposition
D Duplication
E Heterogeneity

81 Examination of an ill child’s blood revealed inherited hyperlipoproteinemia. Genetic


defect of what enzyme synthesis causes this phenomenon?

A Lipoprotein lipase
B Glycosidase
C Proteinase
D Hemsynthetase
E Phenylalanine hydroxylase

82 Dehelmintization of a patient revealed some long fragments of a helminth with


segmented structure. Mature segments were rectangular, 30x12 mm large, closed-type
matrix was in form of a stem with 17-35 lateral branches. Specify this helminth:

A Hookless tapeworm
B Alveococcus
C Echinococcus
D Dwarf tapeworm
E Armed tapeworm

83 Examination of a 30-year-old man mandible revealed in the region of his molar a dense
tumour-like formation that significantly deformed the mandible. The formation wasnt fully
detached from the bone tissue. Microscopical examination of a tissue sampling revealed that
stroma had some cords and follicles with odontogenous cylindric epithelial cells in
peripheria and stellate cells resembling of the enamel organ pulp in the centre. What is the
most likely diagnosis?

A Ameloblastoma
B Primary intraosteal cancer
C Adenomatoid tumour
D Adenocarcinoma
E Osteoclastoma

84 Histological examination of a micro specimen presenting a malignant lung tumour


revealed that the tumor consisted of lymphocyte-like cells forming any structures. Stroma is
mildely marked, there are a lot of mitoses and necroses. What tumour is it?
V ebf68bc9

A Small cell carcinoma


B Squamous cell nonkeratinous carcinoma
C Fibroma
D Squamous cell keratinous carcinoma
E Adenocarcinoma

85 A 40-year-old male patient had a tumour-like formation 8x7 cm large on his neck. A
surgeon removed it only partially because of close connection with large vessels.
Microscopical examination revealed marked cellular and tissue atypism, lipoblast-type cells
in different stages of maturity, with polymorphism and nuclear hyperchromia, pathological
mitoses, necrosis foci. Specify the histological form of the tumour:

A Liposarcoma
B Lipoma
C Fibroma
D Fibrosarcoma
E Hibernoma

86 In compliance with the clinical presentations a man was prescribed pyri-


doxalphosphate. What processes are corrected by this preparation?

A Transamination and decarboxylation of amino acids


B Oxidative decarboxilation of keto acids
C Desamination of purine nucleotides
D Synthesis of purine and pyrimidine bases
E Protein synthesis
V 02b91030

Krok \ Dentistry \ Krok 1 \ Bases \ Biochemistry \ 2010


1 Active physical work induces rise of concentration of carbonic acid in blood. This causes
deepening and acceleration of respiration thus reducing concentration of carbonic acid and
hydrogen ions in blood. This maintains the following process:

A Orthobiosis
B Homeostasis
C Ontogenesis
D Anabiosis
E Immunity

2 The first grade pupils went through a medical examination aimed at selection of children
needing tuberculosis revaccination. What test was applied?

A Anthracene test
B Supracutaneous tularin test
C Mantoux test
D Schick test
E Burne test

3 Cyanide poisoning causes immediate death. What is the mechanism of cyanide effect at
the molecular level?

A They inhibit cytochrome B


B They bind substrates of tricarboxylic acid cycle
C They block succinate dehydrogenase
D They inhibit cytochromoxidase
E They inactivate oxygene

4 Periodontitis is accompanied by activation of proteolysis in the periodontium tissues. The


evidence of proteolysis activation is increase of the following component of oral liquid:

A Glucose
B Organic acids
C Amino acids
D Biogenic amines
E Cholesterol

5 A 40-year-old male patient had a tumour-like formation 8x7 cm large on his neck. A
surgeon removed it only partially because of close connection with large vessels.
Microscopical examination revealed marked cellular and tissue atypism, lipoblast-type cells
in different stages of maturity, with polymorphism and nuclear hyperchromia, pathological
mitoses, necrosis foci. Specify the histological form of the tumour:

A Fibroma
V 02b91030

B Liposarcoma
C Fibrosarcoma
D Hibernoma
E Lipoma

6 A child has disturbed processes of ossification and punctate enamel. What microelement
metabolism is disturbed?

A Chromium
B Zinc
C Fluorine
D Iron
E Copper

7 Two weeks after hemotransfusion a patient developed fever. What protozoal disease can
be suspected?

A Leishmaniasis
B Trypanosomiasis
C Amebiasis
D Malaria
E Toxoplasmosis

8 In compliance with the clinical presentations a man was prescribed pyri-doxalphosphate.


What processes are corrected by this preparation?

A Transamination and decarboxylation of amino acids


B Desamination of purine nucleotides
C Oxidative decarboxilation of keto acids
D Protein synthesis
E Synthesis of purine and pyrimidine bases

9 The activity of parotides reduces with age. Activity of what enzyme in saliva will be
reducing?

A Lysozime
B Maltase
C Phosphatase
D Amylase
E Hexokinase

10 Laboratory examination of a child revealed high content of leucine, valine, isoleucine and
their ketoderivates in blood and urine. Urine had the typical smell of maple syrup. This
disease was caused by deficiency of the following enzyme:

A Glucose-6-phosphatase
B Phosphofructokinase
V 02b91030

C Dehydrogenase of branched amino acids


D phosphofructomutase
E Aminotransferase

11 In the mountains some clinically healthy people present with anaemia symptoms. Blood
test can reveal sickle cells. What is the genotype of such people?

A AA
B aa
C XcXc
D XCXc
E Aa

12 Examination of a 10 y.o. child revealed on the alveolar submandibular process a fixed


tumourous mass 1,5 cm in diameter closing premolar crown on the vestibular side. Mucous
membrane of its surface is reddish-brown, it bleeds as a reaction to a slight mechanical
intervention. Biopsy results: the mass consists of small size vessels separated by thin layers
of connective tissue and infiltrated by plasmocytes, mucous membrane is here and there
ulcered. What is the most probable diagnosis?

A Gingival fibromatosis
B Angiomatous form of epulis
C Giant cell form of epulis
D Fibrous form of epulis
E Hypertrophic gingivitis

13 Examination of a 6 days old infant revealed phenyl pyruvate and phenyl acetate excess
in his urine. What ami-noacid metabolism is disturbed in the child’s organism?

A Phenylalanine
B Histidine
C Methionine
D Tryptophan
E Arginine

14 Laboratory of extremely dangerous infections received a sample taken from a patient


with assumed cholera. What express-diagnostics method can confirm this diagnosis?

A Hemagglutination reaction
B Agglutination test
C Precipitation reaction
D Complement binding reaction
E Immunofluorescence test
V 02b91030

15 A man got poisoned with mushrooms. They contain muscarine that stimulates
muscarinic cholinoreceptors. What symptom is typical for poisoning with inedible
mushrooms?

A Heart rate rise


B Arterial pressure rise
C Bronchi dilation
D Mydriasis
E Miosis

16 Helminthological examination of patient’s feces revealed oval brown eggs with tuberous
external membrane. Name the type of helminth:

A Whipworm
B Broad tapeworm
C Dwarf tapeworm
D Ascarid
E Pinworm

17 A girl who was provisionally diagnosed with Turner’s syndrome came to a genetic
consultation. The diagnosis can be specified by means of the following genetic method:

A Genealogical
B Sex chromatin test
C Dermatoglyphics
D Hybridological
E Biochemical

18 While examining a blood smear taken form a patient and stained by Romanovsky’s
method a doctor revealed some protozoa and diagnozed the patient with Chagas disease.
What protozoan is the causative agent of this disease?

A Trypanosoma brucei
B Toxoplasma gondii
C Leishmania tropica
D Trypanosoma cruzi
E Leishmania donovani

19 A 28 year old pregnant woman had the enzymes in the cells of amniotic fluid analyzed.
The analysis revealed insufficient activity of beta-glucuronidase. What pathological process
is it?

A Lipidosis
B Collagenosis
C Mucopolysaccharidosis
D Aglycogenosis
V 02b91030

E Glycogenosis

20 A patient has painfulness along big nerve trunks and excessive content of pyruvate in
blood. What vitamin deficit may cause such changes?

ABi
B Pantothenic acid
C PP
D Biotin
E B2

21 A 15 year old girl was delivered to the hospital with inflammation of vermiform
appendix. Blood analysis revealed signs of anaemia. Her feces contained lemon-shaped
helminthic eggs (50x30 micrometer) with "plugs" on the poles. What type of helminth is it?

A Hymenolepis nana
B Trichuris
C Echinococcus
D Hookworm
E Pinworm

22 A sample taken from the pharynx of a patient with angina was inoculated on the blood-
tellurite agar. This resulted in growth of grey, radially striated (in form of rosettes) colonies
up to 4-5 mm in diameter. Microscopically there can be seen gram-positive rods with club-
shaped ends arranged in form of spread fingers. What microorganisms are these?

A Diphtheroids
B Corynebacteria diphtheriae
C Streptococci
D Streptobacilli
E Clostridium botulinum

23 A patient was diagnosed with seborrheic dermatitis associated with vitamin H (biotin)
deficiency. The patient has disturbed activity of the following enzyme:

A Alcohol dehydrogenase
B Carbomoyl phosphate synthetase
C Pyruvate decarboxylase
D Amino transferase
E Acetyl-CoA-carboxylase

24 Roentgenological examination of mandible of a 27 year old man revealed a focus of


osseous tissue destruction. Histological examination revealed a tumour consisting of
odontogenous epithelium cords, immature connective tissue and dysplastic dentin rests.
What tumour is it?

A Odontogenous fibroma
V 02b91030

B Ameloblastic fibro-odontoma
C Complex odontoma
D Dentinoma
E Odontoameloblastoma

25 A patient suffering from syphilis was prescribed a drug the action of which based upon
disturbed generation of murein leading to death of the causative agent. What drug is it?

A Benzylpenicillin sodium salt


B Doxycycline hydrochloride
C Azithromycin
D Ciprofloxacin
E Bijochinol

26 A hospital admitted a patient with complaints about abdominal swelling, diarrhea,


meteorism after consumption of food rich in proteins. It is indicative of disturbed protein
digestion and their intensified decaying. What substance is the product of this process in the
bowels?

A Bilirubin
B Putrescine
C Indole
D Agmatine
E Cadaverine

27 A patient has roundish ulcers on his face, inflammation and enlargement of lymph nodes.
These symptoms turned up as a result of mosquito bites. Laboratory examination of
discharge from the ulcers revealed unicellular aflagellar organisms. What is the most
probable diagnosis?

A Toxoplasmosis
B Trypanosomiasis
C Scabies
D Myasis
E Dermatotropic leishmaniasis

28 A patient has increased permeability of blood-vessel walls, increased gingival


hemorrhage, small punctate hematomas on his skin, falling of teeth. What disturbance of
vitamun metabolism can account for these symptoms?

A Hypovitaminosis A
B Hypovitaminosis D
C Hypervitaminosis C
D Hypovitaminosis C
E Hypervitaminosis D
V 02b91030

29 Hepatic disfunctions accompanied by insufficient inflow of bile to the bowels result in


coagulation failure. This phenomenon can be explained by:

A Vitamin K deficiency
B Thrombocytopenia
C Leukopenia
D Iron deficiency
E Erythropenia

30 Analysis of urine from a 24-year-old man revealed the following changes: daily diuresis -
10 l, relative density - 1,001, qualitative alterations are absent. A patient complains of
excessive thirst, frequent urination. What is the most likely cause of this disease?

A Vasopressin hyposecretion
B Vasopressin hypersecretion
C Relative insulin insufficiency
D Aldosteron hypersecretion
E Glucocorticoid hypersecretion

31 Dehelmintization of a patient revealed some long fragments of a helminth with


segmented structure. Mature segments were rectangular, 30x12 mm large, closed-type
matrix was in form of a stem with 17-35 lateral branches. Specify this helminth:

A Dwarf tapeworm
B Hookless tapeworm
C Armed tapeworm
D Alveococcus
E Echinococcus

32 Coprological examination of a patient’s feces revealed small operculate eggs. It is known


from the anamnesis that the patient often consumes fish. What fluke parasitizes in the
patient’s organism?

A Liver fluke
B Lancet fluke
C Blood fluke
D Cat liver fluke
E Lung fluke

33 Examination of a child who hasn’t got fresh fruit and vegetables during winter revealed
numerous subcutaneous hemorrhages, gingivitis, carious cavities in teeth. What vitamin
combination should be prescribed in this case?

A Thiamine and pyridoxine


B Calciferol and ascorbic acid
C Folic acid and cobalamin
V 02b91030

D Ascorbic acid and rutin


E Riboflavin and nicotinamide

34 A patient with high obesity was recommended to take carnitine as a food additive for
better fat burning. What function is fulfilled by carnitine in the process of fat oxidation?

A Transport of fatty acids from the fat depots to the tissues


B Intracellular lipolysis activation
C Transport of fatty acids from the cytosol to the mitochondria
D Participation in one of the reactions of beta-oxidation of fatty acids
E Fatty acid activation

35 A one year old child has enlarged head and belly, retarded cutting of teeth, destruction of
enamel structure. What hypovitaminosis causes these changes?

A Hypovitaminosis C
B Hypovitaminosis A
C Hypovitaminosis B 1
D Hypovitaminosis D
E Hypovitaminosis B2

36 A non trained man has usually muscular hypoxy after a sprint. What metabolite
accumulates in the muscles as a result of it?

A Glucose 6-phosphate
B Ketone bodies
C Lactate
D Oxaloacetate
E-

37 During physical exercise people are less sensitive to pain. The reason for it is the
activation of:

A Nociceptive system
B Antinociceptive system
C Adrenal gland functions
D Sympathoadrenal system
E Thyroid gland functions

38 A patient applied to a doctor complaining about dizziness, memory impairment,


periodical convulsions. It was found out that such changes were caused by a product of
glutamic acid decarboxylation. What product is meant?

A ATP
B GABA
C Thymidine diphosphate
D Tetrahydrofolate
V 02b91030

E Pyridoxalphosphate

39 A patient has liver cirrhosis. Analysis of what substance excreted with urine may
characterize the state of antitoxic liver function?

A Uric acid
B Creatinine
C Hippuric acid
D Amino acids
E Ammonian salts

40 A child has abnormal formation of tooth enamel and dentin as a result of low
concentration of calcium ions in blood. Such abnormalities might be caused by deficiency of
the following hormone:

A Thyroxin
B Triiodothyronine
C Somatotropic hormone
D Thyrocalcitonin
E Parathormone

41 While the examination of patient’s oral cavity the dentist found xerostomia, numerous
erosions. What vitamin deficit caused this effect?

A Vitamin H
B Vitamin P
C Vitamin PP
D Vitamin K
E Vitamin A

42 A 42 year old woman diagnosed with diabetes mellitus was admitted the
endocrinological department with complaints of thirst, excessive appetite. What
pathological components are revealed in course of laboratory examination of the patients
urine?

A Blood
B Glucose, ketone bodies
C Protein, aminoacids
D Bilirubin, urobilin
E Protein, creatine

43 Microscopical examination of a surgical specimen (an ulcered part of a lip) revealed in


the connective tissue of mucous membrane near the borders and under the floor of the
ulcer some epithelial complexes consisting of atypic multi-stratal epithelium with
accumulations of bright pink concentric formations. What pathology is it?

A Squamous cell keratinous carcinoma


V 02b91030

B Basal cell carcinoma


C Papilloma
D Transitional cell carcinoma
E Squamous cell nonkeratinous carcinoma

44 A patient has the folowing changes: disorder of twilight vision, drying out of conjunctiva
and cornea. Such disorders may be caused by deficiency of vitamin:

A Vitamin C
B Vitamin D
C Vitamin B12
D Vitamin B
E Vitamin A

45 A chiled was diagnosed with acute renal failure. What biochemic saliva indices can
confirm this diagnosis?

A Decreased level of phosphate


B Increased level of rest nitrogen
C Reduction of alkaline phosphatase
D Increase of alpha amylase
E Increase of immunoglobuline A

46 For assessment of the neutralizing function of liver a patient with chronic hepatitis went
through a test with natrium benzoate load. The excretion of what acid with urine will
characterize the neutralizing function of liver?

A Valeric acid
B Hippuric acid
C Oxalic acid
D Citric acid
E Phenylacetic acid

47 Microscopical examination of discharges from the gums of a patient ill with paradontosis
revealed some protozoan pear-shaped organisms 6-13 micrometer long. The parasite has
one nucleus and undulating membrane, there are four flagella at the front of its body. What
protozoan were found?

A Amoebae
B Trichomonads
C Balantidia
D Leishmania
E Lamblia

48 A female patient has symptoms of inflammation of urogenital tracts. A smear from the
vaginal mucous membrane contained big unicellular pyriform organisms with a sharp spike
V 02b91030

on the back end of their bodies; big nucleus and undulating membrane. What protozoa were
revealed in the smear?

A Trichomonas buccalis
B Trypanosoma gambiense
C Lamblia intestinalis
D Trichomonas vaginalis
E Trichomonas hominis

49 A 57 year old patient with diabetes mellitus was developed ketoacedosis. Biochemical
base of this condition is decrease of acetyl-CoA utilization. What cell compound deficit
causes this effect?

A 2-oxoglutarate
B Succinate
C Aspartate
D Glutamate
E Oxaloacetate

50 A sportsman needs to improve his sporting results. He was recommended a drug


containing carnitine. What process is activated by this compound in the first place?

A Transport of vitamin K
B Transport of calcium ions
C Transport of amino acids
D Transport of glucose
E Transport of fatty acids

51 Patient with pigmentary xeroderma are characterized by anamalously high sensivity to


ultraviolet rays that causes skin cancer as a result of enzyme systems incapability to restore
damages of hereditary apparatus of cells. What process abnormality is this pathology
connected with?

A DNA reparation
B DNA reduplication
C Genetic complementation
D DNA recombination
E Genetic conversion

52 Among public catering workers examined by doctors of sanitary-and-epidemiologic


station often occur asymptomatic parasite carriers. This means that a healthy person
carries cysts that infect other people. Such parasitizing is impossible for the following
causative agent:

A Malarial plasmodium
B Viscerotropic leishmania
V 02b91030

C Dermatotropic leishmania
D Intestinal trichomonad
E Dysenteric amoeba

53 A sportsman was recommended to take a preparation with carnitine in order to improve


his achievements. What process is activated by carnitine to the most extent?

A Transporting of fatty acids to the mi-tochondrions


B Synthesis of steroid hormones
C Tissue respiration
D Lypide synthesis
E Synthesis of ketone bodies

54 The 16th tooth of a patient is missing. X-ray picture shows in the depth of alveolar
process rarefaction of bone and a well-defined cavity that contained the underdeveloped
tooth crown. Microscopical examination revealed that the cavity wall was lined with
stratified squamous epithelium and enclosed within a fibrous capsule. Make a diagnosis:

A Cystic ameloblastoma of jaw


B Follicular gnathic cyst
C Primordial gnathic cyst
D Radicular gnathic cyst
E Cyst of the incisive canal

55 A 38 year old patient takes aspirin and sulfanilamides. After their intake intensified
erythrocyte haemolysis is observed which is caused by deficiency of glucose 6-phosphate
dehydrogenase. This pathology is caused by failure of the following coenzyme:

A NADP - H
B Ubiquinone
C FAD - H2
D Pyridoxal phosphate
E FMN - H2

56 A patient has the sudden decrease of Са2+ content in blood. What hormone secretion
will increase?

A Somatotropin
B Vasopressin
C Parathormone
D Aldosterone
E Thyrocalcitonin

57 Examination of urine in a newborn revealed presence of citrulline and high ammonia


concentration. This baby is most likely to have the disorder of the following substance
production:
V 02b91030

A Creatinine
B Uric acid
C Creatine
D Ammonia
E Urea

58 Histological examination of a micro specimen presenting a malignant lung tumour


revealed that the tumor consisted of lymphocyte-like cells forming any structures. Stroma is
mildely marked, there are a lot of mitoses and necroses. What tumour is it?

A Squamous cell keratinous carcinoma


B Adenocarcinoma
C Small cell carcinoma
D Squamous cell nonkeratinous carcinoma
E Fibroma

59 A patient has mental retardation, small height, brachydactyly, mongoloid slant. Analysys
of his karyotype revealed trisomy 21. What chromosomal anomaly is it?

A Trisomy X
B Down’s disease
C Turner’s syndrome
D Klinefelter’s syndrome
E Specific fetopathy

60 To what total ATP quantity is the full glucose oxidation and its linking with
phosphorylation equivalent?

A 38
B8
C 58
D 52
E 12

61 A 2 year old child suffers from intestinal dysbacteriosis that lead to the development
hemorrhagic syndrome. The most probable cause of hemorrhage is:

A Vitamin K deficiency
B Hypovitaminosis PP
C Fibrinogen deficiency
D Hypocalcemia
E Activation of tissue thromboplastin

62 A patient who is ill with scurvy displays disturbed processes of connective tissue
formation that leads to loosening and falling of teeth. Disturbed activity of what enzyme
causes these symptomps?
V 02b91030

A Procollagenpeptidase of N-terminal peptide


B Elastase
C Glycosiltransferase
D Procollagenpeptidase of C-terminal peptide
E Lisilhydroxylase

63 A patient has increased content of uric acid in his blood that is clinically presented by
pain syndrome as a result of urate deposition in the joints. What process does this acid
result from?

A Heme catabolism
B Lysis of purine nucleotides
C Lysis of pyrimidine nucleotides
D Proteolysis
E Reutilization of purine bases

64 A child has disturbed enamel and dentine formation as a result of decreased content of
calcium ions in his blood. What hormone deficiency may cause such changes?

A Thyroxin
B Parathormone
C Triiodothyronine
D Thyreocalcitonin
E Somatotropin

65 While on holiday in the countryside a boy found a spider with the following
morphological peculiarities: body length at the rate of 2 cm, round black abdomen with two
rows of red dots on its dorsal surface, four pairs of segmented extremities covered with tiny
black hairs. Identify this arthropod:

A Mite
B Steppe spider (Latrodectus tredeci-mguttatus)
C Tarantula
D Solifugae
E Scorpion

66 Examination of a 30-year-old man mandible revealed in the region of his molar a dense
tumour-like formation that significantly deformed the mandible. The formation wasnt fully
detached from the bone tissue. Microscopical examination of a tissue sampling revealed that
stroma had some cords and follicles with odontogenous cylindric epithelial cells in
peripheria and stellate cells resembling of the enamel organ pulp in the centre. What is the
most likely diagnosis?

A Osteoclastoma
B Adenomatoid tumour
C Ameloblastoma
V 02b91030

D Primary intraosteal cancer


E Adenocarcinoma

67 Roentgenological examination of a patient revealed a cyst enclosing a tooth in its cavity


in the area of the premolar. Microscopical examination revealed that the cyst wall consisted
of connective tissue and was lined with multilayer squamous epithelium. What is the most
probable diagnosis?

A Follicular cyst
B Primordial cyst
C Radicular cyst
D Eosinophilic granuloma
E Epulis

68 Examination of an ill child’s blood revealed inherited hyperlipoproteinemia. Genetic


defect of what enzyme synthesis causes this phenomenon?

A Lipoprotein lipase
B Phenylalanine hydroxylase
C Proteinase
D Glycosidase
E Hemsynthetase

69 Osteolathyrism is characterized by a loss of tensile strength of collagen, which is induced


by a significant decrease in the formation of cross-links in collagen fibrils. The cause for it is
the reduced activity of:

A Prolyl hydroxylase
B Lysyl hydroxylase
C Collagenase
D Lysyl oxidase
E Monoamine oxidase

70 Up to 50% of world population aged above thirty is affected by paradontosis. The


leading part in pathogenesis of this disease is played by:

A Immune damage of tissues


B Parodontium tissues damaged by kalli-krein
C Parodontium damaged by active cells
D Dental calculus caused by microflora
E Neurodystrophic factor

71 A 10 year old child underwent Mantoux test (with tuberculin). 48 hours later there
apperaed a papule up to 8 mm in diameter on the site of tuberculin injection. Tuberculin
injection caused the following hypersensitivity reaction:

A II type hypersensitivity reaction


V 02b91030

B Arthus reaction
C Seroreaction
D Atopic reaction
E IV type hypersensitivity reaction

72 During examination of a 36-year-old woman a dentist revealed a formation in form of a


nodule up to 0,8 cm in diameter, of dark brown-red colour, soft, on a wide base. The
formation was found on the buccal surface of gum in the region of the 2nd molar.
Histological examination revealed that the formation had plenty of sinusoid vessels and a
lot of roundish mononuclear and big multinuclear cells; in some parts accumulations of
hemosiderin granules could be found. What is the most likely diagnosis?

A Giant-cell epulis
B Angiomatous epulis
C Mandibular osteoclastoma
D Ameloblastoma
E Root granuloma

73 A patient with chronic hypoglycemia had adrenaline introduction. After introduction


blood test has not changed essentially. Doctor assumed liver pathology. What liver function
may have been changed?

A Excretory function
B Function of cholesterin production
C Glycolytic function
D Ketogenic function
E Function of glycogen depositing

74 A 28 year old woman consulted a doctor about sterility. Examination revealed


underdeveloped ovaries and uterus, irregular menstrual cycle. Study of sex chromatin
revealed 2 Barr’s bodies in most somatic cells. What chromosome disease is the most
probable in this case?

A Turner’s syndrome
B Patau’s syndrome
C Edwards’ syndrome
D Klinefelter’s syndrome
E Triplo-X syndrome

75 A patient with focal tuberculosis of superior lobe of his right lung takes isoni-azid as a
part of combined therapy. After a time he started complaining of muscular weakness,
decrease of skin sensitivity, sight and movement coordination disorder. What vitamin
preparation will be right for elimination of these occurances?

A Vitamin A
B Vitamin D
V 02b91030

C Vitamin C
D Vitamin B6
E Vitamin B12

76 A dentist was examining oral cavity of a 9 year old child in the buccal surface of gingiva
in the area of the lower canine he revealed a red, soft, node-like formation 1 cm in diameter
that started immediately bleeding when touched. Microscopical examination revealed that
this formation consisted of many small vessels like venules and capillaries separated by thin
layers of connective tissue, with focal infiltration by lymphoid and plasmatic cells. Such
changes are typical for:

A Fibrous epulis
B Papilloma
C Capillary hemangioma
D Radicular granuloma
E Angiomatous epulis

77 After implantation of a cardiac valve a young man constantly takes indirect


anticoagulants. His state was complicated by hemorrhage. What substance content has
decreased in blood?

A Ceruloplasmin
B Heparin
C Prothrombin
D Creatin
E Haptoglobin

78 Decreased ratio of adenylic nucleotides ATP/ADP results in intensified glycolysis in


parodentium tissues in hypoxic conditions. What reaction is activated in this case?

A Lactate dehydrogenase
B Phosphofructokinase
C Enolase
D Triosophosphate isomerase
E Aldolase

79 A patient has deformation of jaw bones. Histological examination revealed there growth
of fibrocellular tumourlike ill-defined tissue with primitive osteogenesis. What disease are
these presentations typical for?

A Ameloblastoma
B Fibrous dysplasia
C Osteosarcoma
D Parathyroid osteodystrophy
E Eosinophilic granuloma
V 02b91030

80 In order to speed up healing of the thermal injury it is required to prescribe a drug that
facilitates epithelization of skin and mucous membranes. What drug is it?

A Ascorbic acid
B Retinol acetate
C Tocopherol acetate
D Nicotinic acid
E Ergocalciferol

81 A patient with diabetes mellitus had an insuline injection. It caused loss of consciousness
and convulsions. What was the result of biochemic blood analysis on glucose content?

A 3,3 mmole/l
B 10 mmole/l
C 2,5 mmole/l
D 5,5 mmole/l
E 8,0 mmole/l

82 A blood smear of a patient who has recently recovered from flu contains 10% of
roundish cells 4,5-7 micrometer large with a big round nucleus and basophi-lically stained
cytoplasm in form of a narrow border around the nucleus. What blood status are they
typical for?

A Lymphocytopenia
B Leukopenia
C Monocytopenia
D Thrombopenia
E Lymphocytosis

83 A patient was taken to the hospital with preliminary diagnosis progressive muscle
distrophy. What substance will be excessively contained in urine and confirm this
diagnosis?

A Troponine
B Hydroxiproline
C Carnosine
D Creatine
E Pyruvate

84 Chronic overdosage of glucocorticoids leads to the development of hyperglycemia. What


process of carbohydrate metabolism is responsible for this effect?

A Aerobic glycolisis
B Glycogenolysis
C Gluconeogenesis
D Pentose-phosphate cycle
V 02b91030

E Glycogenesis

85 Cytogenetic examination of a patient with dysfunction of the reproductive system


revealed normal karyotype 46,XY in some cells, but most cells have Klinefelter’s syndrome
karyotype - 47,XXY Such phenomenon of cell inhomogeneity is called:

A Duplication
B Heterogeneity
C Transposition
D Mosaicism
E Inversion

86 Myocyte cytoplasm contains a big number of dissolved metabolites of glucose oxidation.


Name one of them that turns directly into lactate:

A Fructose 6-phosphate
B Oxaloacetate
C Glucose 6-phosphate
D Pyruvate
E Glycerophosphate
V 02b91030

Keys: 1 B, 2 C, 3 D, 4 C, 5 B, 6 C, 7 D, 8 A, 9 D, 10 C, 11 E, 12 B, 13 A, 14 E, 15 E, 16 D, 17 B,
18 D, 19 C, 20 A, 21 B, 22 B, 23 E, 24 D, 25 A, 26 C, 27 E, 28 D, 29 A, 30 A, 31 B, 32 D, 33 D,
34 C, 35 D, 36 C, 37 B, 38 B, 39 C, 40 E, 41 E, 42 B, 43 A, 44 E, 45 B, 46 B, 47 B, 48 D, 49 E,
50 E, 51 A, 52 E, 53 A, 54 B, 55 A, 56 C, 57 E, 58 C, 59 B, 60 A, 61 A, 62 E, 63 B, 64 D, 65 B,
66 C, 67 A, 68 A, 69 D, 70 E, 71 E, 72 A, 73 E, 74 E, 75 D, 76 E, 77 C, 78 B, 79 B, 80 B, 81 C,
82 A, 83 D, 84 C, 85 D, 86 D,
Ministry of Public Health of Ukraine

O. O. BOGOMOLETS NATIONAL MEDICAL UNIVERSITY

Department of Bioorganic and Biological Chemistry

LIST OF TEST QUESTIONS

for preparation of Content module № 2

"General regularities of metabolism of


carbohydrates, lipids, amino acids"
FOR STUDENTS OF THE 2ST YEAR OF STUDY

OF MEDICAL and STOMATOLOGICAL FACULTIES

Kyiv-2018
Enzymes and coenzymes. Regulation of metabolism
1. A denaturation of proteins can befound in B. Aspartate, glutamate, glycine
some substances. Specify thesubstance that is C. Aspartate, arginine, glutamate
used for the incompletedenaturation of D. Glutamate, valine, leucine
hemoglobin: E. Cysteine, glycine, proline
A. Urea
B. Toluene 7. Parodontitis is accompanied byactivation of
C. Sulfuric acid proteolysis in parodentiumtissues. Increase of
D. Nitric acid what oral fluid’scomponent is the evidence of
E. Sodium hydroxide proteolysisactivation?
A. Aminoacids
2. Some proteins of saliva have aprotective B. Organic acids
function. Which of themprotects the oral C. Glucose
mucosa from themechanical damage? D. Biogenic amines
A. Mucin E. Cholesterol
B. Lysozyme
C. Catalase 8. The student used conserved donor blood to
D. Peroxidase determine the time of its clotting. However, any
E. Renin positive result had not been obtained. The
reason for this is the lack in this sample of
3. For the study of serum proteinsvarious blood:
physical and physicochemicalmethods can be A. Ionized calcium
used. In particular, serumalbumins and B. Factor Hageman
globulins can be separatedby this method: C. Thromboplastin
A. Electrophoresis D. Fibrinogen
B. Polarography E. Vitamin K
C. Dialysis
D. Spectrography 9. Long-term effects on the body of toxic
E. Refractometry substances led to a significant reduction in
protein synthesis in hepatocytes. What
4. Electrophoretic study of blood serumof a organelles suffered from intoxication most?
patient with pneumonia revealed anincrease in A. Granular endoplasmic reticulum
one of the protein fractions.What fraction is it? B. Mitochondria
A. γ-globulins C. Microtubules
B. Albumins D. Lysosomes
C. α1-globulins E. The Golgi’s complex
D. α2-globulins
E. β-globulins 10. Children with kwashiorkor, along with other
signs, violations of the process of dentition were
5. A woman, 36 years after surgery, received revealed. At the heart of this phenomenon is the
intravenous injection of a concentrated albumin insufficient receipt into the body:
solution. This resulted in the increased A. Lipids
movement of water in the following direction: B. Proteins
A. From intercellular fluid to capillaries C. Carbohydrates
B. From the intercellular fluid to the cells D. Vitamin C
C. From cells to the intercellular fluid E. Vitamin B1
D. From capillaries to intercellular fluid
E. There will be no change in the flow of water 11. In the daily diet of an adult healthy person,
there must be fats, proteins, carbohydrates,
6. Cationic glycoproteins are the vitamins, mineral salts and water. Specify the
majorcomponents of parotid saliva. What daily amount of protein (g) that provides normal
aminoacids are responsible for their life activity of the body:
positivecharge? A. 100-120
A. Lysine, arginine, histidine B. 50-60
C. 10-20 E. Glucose
D. 70-80
E. 40-50 17. 60-year-old man went to the doctor after the
appearance of pain in the chest. In blood serum
12. A patient with a hepatic failure has been showed a significant increase in the activity of
examined for the electrophoretic spectrum of enzymes: CPK and its MB isoform, aspartate
blood serum proteins. Which of the physico- aminotransferase. On the development of the
chemical properties of the protein molecules lie pathological process in which tissue do these
in the basis of this method? changes indicate?
A. Presence of a charge A. Heart muscle
B. Hydrophilicity B. Lungs
C. Ability to swell C. Skeletal muscle
D. Optical activity D. Liver
E. Non-dialysable E. Smooth muscles

13. The surgeon used a 70% solution of ethyl 18. There are several groups of molecular
alcohol to disinfect the hands before surgery. mechanisms that are important in the
What is the main mechanism of antiseptic action pathogenesis of cell damage, which contributes
of the drug on microorganisms? to the development of pathology. What
A. Denaturation of protoplasmic proteins. processes provide protein damage mechanisms?
B. Interaction with amino groups of A. Inhibition of enzymes
protoplasmic proteins. B. Peroxide oxidation of lipids
C. Interaction with hydroxyl groups of enzymes. C. Acidosis
D. Blockade of sulfhydryl groups of enzyme D. Osmotic stretching of membranes
systems. E. Activation of phospholipases
E. Oxidation of organic components of
protoplasm. 19. It is known that the activity of parotid
salivary glands susceptible to age-related
14. What substance makes saliva viscousand changes. The activity of which enzyme will
mucous, has protective function, protects decrease sharply in saliva?
mucous membrane of oralcavity from A. Amylase.
mechanical damage? B. Hexokinase.
A. Mucin C. Renin.
B. Glucose D. Maltase.
C. Kallikrein E. Phosphatase.
D. Amylase
E. Lysozyme 20. For biochemical diagnostics of cardiac
infarction it is necessary to determine activity of
15. Only one factor can affect the charge of a number of enzymes and their izoenzymes in
amino acid radicals in the active center of the the blood. What enzyme assay is considered to
enzyme. Name of this factor is: be optimal for confirming or ruling out cardiac
A. pH infarction at the early stage, after the patient
B. Pressure develops thoracic pain?
C. Temperature A. Creatine kinase MB isoenzyme
D. The presence of a competitive inhibitor B. Creatine kinase MM isoenzyme
E. Excessproduct C. LDH1 isoenzyme
D. LDH5 isoenzyme
16. The conjugated protein necessarily contains E. Cytoplasmic isoenzyme of
special component as a nonprotein part. Choose aspartateaminotransferase
the substance that can’t carry out this function:
A. HNO3 21. For the treatment of some infectious
B. АТP diseases caused by bacteria, sulfonamides drugs
C. Thiamine pyrophosphate are used. What is the mechanism of action of
D. AMP these drugs?
A. They are antivitamins of p-aminobenzoic the most likely development of the pathological
acid. process?
B. They are alosteric enzyme inhibitors. A. Heart
C. They are alosteric enzyme activators. B. Pancreas
D. They are participate in redox processes. C. Liver
E. They are inhibit folic acid absorption. D. Kidneys
E. Skeletal muscle
22. In the intensive care unit entered a man 47-
years old with a diagnosis of myocardial 27. An increase in the activity of LDH 4,5,
infarction. Which of the lactate dehydrogenase AlAT, carbamoyl ornithine transferase was
(LDH) fractions will prevail in the blood serum detected in the patient's blood. In what organ
during the first two days? can assume development of the pathological
A. LDH 1 process?
B. LDH 2 A. Cardiac muscle (possible myocardial
C. LDH 3 infarction)
D. LDH 4 B. Liver (possible hepatitis)
E. LDH 5 C. Skeletal muscle
D. Kidneys
23. Researchers isolated 5 isoenzymicforms of E. Connective tissue
lactate dehydrogenase from thehuman blood
serum and studied theirproperties. What 28. Albumin concentration in the patient’s blood
property indicates thattheisoenzymic forms is 2.8 g/l, increased concentration of lactate
were isolated fromthe same enzyme? dehydrogenase 5 (LDH5). On the disease of
A. Catalyzation of the same reaction what organ does that indicate?
B. The same molecular weight A. Liver
C. The same physicochemical properties B. Kidney
D. Tissue localization C. Heart
E. The same electrophoretic mobility D. Lung
E. Spleen
24. One of the means of regulating enzyme
activity in a human body is the covalent 29. There is increased activity of AST, LDH1,
modification. Glycogen phosphorylase and LDH2, and CPK in the patient’s blood.
glycogen synthetase activity is regulated by the Pathological process most likely occurs in the:
following type of covalent modification: A. Heart
A. Phosphorylation-dephosphorylation B. Skeletal muscles
B. ADP-ribosylation C. Kidneys
C. Methylation D. Liver
D. Hydrolysis E. Adrenal glands
E. Sulfonation
30. In the patient's blood plasma the activity of
25. When the patient's blood analysis revealed a LDH1 and LDH2 isoenzymes increased. About
significant increase in the activity of MB-forms which pathology of the body does this mean?
CPK (CPK) and lactate dehydrogenase-1. What A. Myocardium
pathology can be assumed? B. Liver
A. Myocardial infarction C. Kidney
B. Hepatitis D. Brain
C. Rheumatism E. Skeletal muscle
D. Pancreatitis
E. Cholecystitis 31. In a patient revealed increased the activity of
AsAT, LDH1,2, and creatine phosphokinase. In
26. An increase in the activity of LDH1, LDH2, which organ (s) is the most likely development
AsAT, and creatine kinase was detected in the of the pathological process?
patient's blood. In which organ of the patient is A. Heart muscle
B. Skeletal muscle
C. Kidney and adrenal gland D. LDH 4
D. Connective tissue E. LDH 5
E. Liver and kidneys
34. The high level of lactate dehydrogenase
32. 49-year-old patient, a driver by profession, (LDH) isozymes concentration showed the
complains of unbearable compressive pain increase of LDH-1 and LDH-2 in a patient’s
behind the sternum, "giving away" to the neck. blood plasma. Point out the most probable
Pain occurred 2 hours ago. Objectively: the diagnosis:
condition is severe, pallor, heart sounds are A. Myocardial infarction
weakened. Laboratory studies have shown high B. Skeletal muscle dystrophy
activity of creatine kinase and LDH1. What C. Diabetes mellitus
diseases are characterized by such symptoms? D. Viral hepatitis
A. Acute myocardial infarction E. Acute pancreatitis
B. Acute pancreatitis
C. Stenocardia 35. Succinate dehydrogenase catalyses the
D. Cholelithiasis dehydrogenation of succinate. Malonic acid
E. Diabetes HOOC − CH2− COOH is used to interrupt the
action of this enzyme. Choose the inhibition
33. Six hours after myocardial infarction, the type:
lactate dehydrogenase activity in the patient A. Competitive
increased in the blood. What isoenzymes should B. Allosteric
be expected in this case? C. Non-competitive
A. LDH 1 D. Limited proteolysis
B. LDH 2 E. Dephosphorylation
C. LDH 3
Water-soluble vitamins
1. Examination of a patient revealed dermatitis, C. Retinol
diarrhea, dementia. What vitamin deficiency is D. Calciferol
the cause of this condition? E. Riboflavin
A. Nicotinamide
B. Ascorbic acid 6. A patient has painfulness along bignerve
C. Folic acid trunks and excessive content of pyruvate in
D. Biotin blood. What vitamin deficit may cause such
E. Rutin changes?
A. В1
2. A patient consulted a doctor about symmetric B. Biotin
dermatitis of open skin areas. It was found out C. РР
that the patient lived mostly on cereals and ate D. Pantothenic acid
too little meat, milk and eggs. What vitamin E. В2
deficiency is the most evident?
A. Nicotinamide 7. A woman who has been keeping to a clean-
B. Calciferol rice diet for a long time was diagnosed with
C. Folic acid polyneuritis (beri-beri). What vitamin deficit
D. Biotin results in development of this disease?
E. Tocopherol A. Thiamine
B. Ascorbic acid
3. A 3 year old child with symptoms of C. Pyridoxine
stomatitis, gingivitis and dermatitis of open skin D. Folic acid
areas was delivered to a hospital. Examination E. Riboflavin
revealed inherited disturbance of neutral amino
acid transporting in the bowels. These 8. A patient has an increased pyruvate
symptoms were caused by the deficiency of the concentration in blood. A large amount of it is
following vitamin: excreted with the urine. What vitamin is lacking
A. Niacin in this patient?
B. Pantothenic acid A. B1
C. Vitamin A B. E
D. Cobalamin C. B3
E. Biotin D. B6
E. B2
4. A patient is ill with dermatitis, diarrhea,
dementia. During history taking it was revealed 9. A patient, who has been subsisting
that the main food stuff of the patient was exclusively on polished rice, has developed
maize. These disturbances are caused by polyneuritis due to thiamine deficiency. What
deficiency of the following vitamin: substance is an indicator of such avitaminosis,
A. PP when it is excreted with urine?
B. B1 A. Pyruvic acid
C. B2 B. Malate
D. B9 C. Methylmalonic acid
E. B8 D. Uric acid
E. Phenyl pyruvate
5. A 50-year-old man addressed a hospital
withcomplaints of memory disorders, painful 10. Vitamin B1 deficiency causes disturbance of
sensations along the nerve trunks, decreased oxidative decarboxylation of α-ketoglutaric
mental ability, circulatory disorders and acid. This leads to the impaired synthesis of the
dyspepsia. Anamnesis states excessive alcohol following coenzyme:
consumption. What vitamin deficiency can A. Thiamine pyrophosphate
result in such signs? B. Nicotinamide adenine dinucleotide
A. Thiamine C. Flavine adenine dinucleotide
B. Niacin D. Lipoic acid
E. Coenzyme A 16. In case of enterobiasis acrihine – the
structural analogue of vitamin B2 – is
11. To improve the trophism of the heart administered. The synthesis disorder of which
muscle, the patient is prescribed a drug that enzymes does this medicine cause in
includes cocarboxylase (thiamine diphosphate) - microorganisms?
a coenzyme form of the vitamin: A. FAD-dependent dehydrogenases
A. B1 B. Cytochromeoxidases
B. B2 C. Peptidases
C. B5 D. NAD-depended dehydrogenases
D. B6 E. Aminotransferases
E. B12
17. The preparation comlex for periodontitis
12. Pyruvic acid as an intermediate metabolite treatment includes the medicine from the group
of carbohydrate, lipid and amino acid of water soluble vitamins, bioflavonide
metabolism can undergo oxidative derivative, which is prescribed together with
decarboxylation. The cause of this process is the ascorbic acid. This preparation has anti-
lack of the following nutrient in the diet: oxidative properties, decreases gingival
A. Thiamin hemorrhage. What preparation is meant?
B. Pyridoxine A. Rutin
C. Ascorbic acid B. Calcium pantothenate
D. Citrine C. Calcium panganate
E. Pangamic acid D. Cyanocobalamin
E. Folic acid
13. A patient with chronic alcoholism has
symptoms of polyneuritis and cardiac 18. Examination of a child who hasn’t got fresh
insufficiensy. What vitamin preparation should fruit and vegetables during winter revealed
be prescribed to this patient? numerous subcutaneous hemorrhages,
A. Thiamine gingivitis, carious cavities in teeth. What
B. Ergocalciferol vitamin combination should be prescribed in
C. Retinol this case?
D. Rutin A. Ascorbic acid and rutin
E. Phylloquinone B. Thiamine and pyridoxine
C. Folic acid and cobalamin
14. A 36-year-old female patient has a history of D. Riboflavin and nicotinamide
B2-hypovitaminosis. The most likely cause of E. Calciferol and ascorbic acid
specific symptoms (epithelial, mucosal,
cutaneous, corneal lesions) is the deficiency of: 19. Examination of a patient with frequent
A. Flavin coenzymes hemorrhages from internals and mucous
B. Cytochrome A1 membranes revealed proline and lysine being a
C. Cytochrome oxidase part of collagene fibers. What vitamin absence
D. Cytochrome B caused disturbance of their hydroxylation?
E. Cytochrome C A. Vitamin С
B. Vitamin K
15. Malaria is treated with structural analogs of C. Vitamin A
vitamin B2 (riboflavin). These drugs disrupt the D. Thiamine
synthesis of the following enzymes in E. Vitamin Е
plasmodium:
A. FAD-dependent dehydrogenase 20. In spring a patient experiences petechial
B. NAD-dependent dehydrogenase haemorrhages, loosening of teeth, high liability
C. Peptidase to colds. A doctor suspects hypovitaminosis C.
D. Cytochrome oxidase In this respect loosening of teeth can be
E. Aminotransferase explained by:
A. Structural failure of collagen in the
periodontal ligaments
B. Structural change of glycosaminoglycans 25. A patient diagnosed with focal tuberculosis
C. Increased permeability of periodont of the upper lobe of the right lung had been
membranes taking isoniazid as a part of combination
D. Mechanical damage of teeth therapy. After some time, the patient reported of
E. Disturbed oxidation-reduction process in the muscle weakness, decreased skin sensitivity,
periodont blurred vision, impaired motor coordination.
Which vitamin preparation should be used to
21. Most of the members of the expedition of address these phenomena?
Magellan to America died from vitamin A. Vitamin B6
deficiency. This disease was manifested by B. Vitamin A
general weakness, subcutaneous hemorrhage, C. Vitamin D
tooth loss, bleeding from the gums. As the name D. Vitamin B12
of this vitamin deficiency? E. Vitamin C
A. scurvy (scurvy)
B. Pellagra 26. A 40-year-old male patient with pulmonary
C. Rickets tuberculosis was administered isoniazid. What
D. Polyneuritis (beriberi) vitamin deficiency can develop as a result of
E. Biermer’s anemia taking this drug for a long time?
A. Pyridoxine.
22. A 10-year-old girl has a history of repeated B. Cobalamin.
acute respiratory viral infection. After C. Biotin.
recovering she presents with multiple petechial D. Thiamine.
hemorrhages on the sites of friction from E. Folic acid.
clothing rubbing the skin. What kind of
hypovitaminosis has this girl? 27. What vitamin is a component of glutamic
A. C acid decarboxylase, participates in the
B. B6 production of GABA, and its deficiency is
C. B1 manifested by seizures?
D. A A. Pyridoxine
E. B2 B. Cobalamin
C. Tocopherol
23. The patient has an increase in the D. Folic acid
permeability of the walls of blood vessels with E. Ascorbic acid
the development of increased bleeding and the
appearance of small point hemorrhages on the 28. A 9-month-old child feeds on artificial
skin, tooth loss. How does the vitamin exchange mixtures that are not balanced in terms of
disorder explain these symptoms? vitamin B6 content. A child has pellagra similar
A. Hypovitaminosis C dermatitis, convulsions, anemia. The
B. Hypervitaminosis D development of seizures may be associated with
C. Hypervitaminosis C impaired formation:
D. Hypovitaminosis D A. GABA
E. Hypovitaminosis A B. Histamine
C. Serotonin
24. A 20-year-old male patient complains of D. DOPA
general weakness, rapid fatigability, irritability, E. Dopamine
decreased performance, bleeding gums,
petechiae on the skin. What vitamin deficiency 29. The infant has epileptiform convulsions
may be a cause of these changes? caused by vitamin B6 deficiency. This is due to
A. Ascorbic acid a decrease in the nerve tissue of the inhibitory
B. Riboflavin mediator - γ-aminobutyric acid. What enzyme
C. Thiamine activity is reduced due to this?
D. Retinol A. Glutamate decarboxylase
E. Folic acid B. Alanine aminotransferase
C. Glutamate dehydrogenase
D. Pyridoxal kinase B. Carboxybiotin
E. Glutamate synthetase C. Thiamine diphosphate
D. Pyridoxal phosphate
30. In compliance with the clinical presentations E. Nicotinamide adenine dinucleotide
a man was prescribed pyridoxalphosphate. What
processes are corrected by this preparation? 35. Pterin derivatives (aminopterin and
A. Transamination and decarboxylation of methotrexate) are the inhibitors of dihydrofolate
amino acids reductase, so that they inhibit the regeneration
B. Oxidative decarboxilation of keto acids of tetrahydrofolic acid from dihydrofolate.
C. Desamination of purine nucleotides These drugs inhibit the intermolecular tranfer of
D. Synthesis of purine and pyrimidine bases monocarbon groups, thus suppressing the
E. Protein synthesis synthesis of the following polymer:
A. DNA
31. In the clinical practice for the treatment of B. Protein
tuberculosis drug isoniazid used - antivitamin C. Homopolysaccharides
which is able to penetrate into the tubercle D. Gangliosides
bacillus. The tuberculostatic effect is due to the E. Glycosaminoglycans
violation of replication processes, redox
reactions, due to the formation of not real 36. After an extended treatment with
coenzyme from: sulfanamides a patient has developed
A. NAD+ macrocyticanemia. Production of active forms
B. FAD of the following vitamin is disrupted in such a
C. FMN condition:
D. TDP A. Folic acid
E. KoQ B. Thiamine
C. Riboflavin
32. Reactions of intermolecular transport of D. Pyridoxine
one-carbon radicals are necessary for the E. Cyanocobalamin
synthesis of proteins and nucleic acids. From
which of the following vitamins does the 37. It is known that part of carbon dioxide is
coenzyme form necessary for the above used in the body in the biosynthesis of fatty
reactions? acids, urea, gluconeogenesis, etc. Which
A. Folic acid vitamin forms the CO2-transporting form for
B. Thiamine these reactions?
C. Pantothenic acid A. Biotin
D. Ascorbic acid B. thymine
E. Riboflavin C. Riboflavin
D. Nicotinamide
33. In the synthesis of purine nucleotides E. Retinol
involved some amino acids, derivatives of
vitamins, phosphorus esters of ribose. What 38. A patient was diagnosed with seborrheic
coenzyme form of vitamin provides one-carbon dermatitis associated with vitamin H (biotin)
fragments for this synthesis? deficiency. The patient has disturbed activity of
A. Folic acid the following enzyme:
B. Pantothenic acid A. Acetyl-CoA-carboxylase
C. Nicotinic acid B. Pyruvate decarboxylase
D. Riboflavin C. Alcohol dehydrogenase
E. Pyridoxine D. Amino transferase
E. Carbomoyl phosphate synthetase
34. In the normal course of a replication process
need thymidylic nucleotides, the synthesis of 39. The patient was diagnosed megaloblastic
which occurs with the participation of anemia. Specify a vitamin deficiency which can
thymidylate synthetase, are used as a coenzyme: lead to the development of this disease.
A. Methylenetetrahydrofolate A. Cyanocobalamin.
B. Rutin. 44. In the patient with complaints about pain in
C. Nicotinamide. the stomach found a decrease in its secretory
D. Thiamine. function, which is accompanied by anemia.
E. Cholecalciferol. What substance deficiency causes the
development of B12 hypovitaminosis in a patient
40. 47. A patient 43 years old with chronic and the appearance of anemia?
atrophic gastritis and hyperchromic A. Castle factor
megaloblastic anemia increased methylmalonic B. Thiamine
acid excretion in the urine. The lack of which C. Biotin
vitamin caused the occurrence of this symptom D. Pyridoxine
complex? E. Calciferol
A. В12
B. В2 45. After the surgical removal of part of the
C. В3 stomach of the patient disrupted the absorption
D. В5 of vitamin B12, it is excreted in the feces.
E. В6 Anemia has developed. What factor is necessary
for the absorption of this vitamin?
41. After removing 2/3 of the stomach in the A. Gastromukoprotein
patient’s blood, the hemoglobin content B. Gastrin
decreased, the number of red blood cells C. Hydrochloric acid
increased, the size of these blood cells D. Pepsin
increased. What vitamin deficiency leads to E. Folic acid
such changes in the blood?
A. B12 46. In a 65-year-old patient with long-lasting
B. C complaints characteristic of chronic gastritis,
C. P megalocytes were found in peripheral blood and
D. B6 megaloblastic erythropoiesis in the bone
E. PP marrow. What is the most likely diagnosis?
A. B12-folic deficiency anemia
42. In examining the oral cavity of the patient, B. Aplastic anemia
the dentist paid attention to the presence of an C. Hypoplastic anemia
inflammatory and dystrophic process in the D. Hemolytic anemia
mucous membrane (Hunter’s glossitis, atrophic E. Iron deficiency anemia
stomatitis). A blood test revealed hyperchromic
anemia. What factor is the cause of this disease? 47. A 13-year-old girl has been prescribed a
A. Hypovitaminosis B12 certain drug for treatment of megaloblastic
B. Hypovitaminosis B1 anemia. This drugstimulates a transfer from
C. Hypovitaminosis B6 megaloblastic haemopoiesis to normoblastic,
D. Increasing pH of the gastric juice participates in synthesis of purine and
E. Hypovitaminosis A pyrimidine bases, activates proteine and
methionine synthesis.What drug does the patient
43. A year after subtotal stomach resection on take?
account of ulcer of lesser curvature the A. Cyanocobalamin
following blood changes were revealed: anemia, B. Erythropoietin
leukocytopenia and thrombocytopenia, color C. Haemostimulinum
index - 1,3, megaloblasts and megalocytes. D. Ferricsulfate
What factor deficiency caused the development E. Rosehiptea
of thos pathology?
A. Castle’s factor 48. A 50-year-old patient has been examined by
B. Hydrochloride acid a dentist and found to have crimson smooth
C. Mucin tongue. Blood analysis revealed a decrease in
D. Pepsin RBC level and hemoglobin concentration,
E. Gastrin colour index of 1,3, symptoms of megaloblastic
hematopoiesis, degenerative changes in
WBCs.What blood disorder was found in this E. Ubiquinone
patient?
A. B12-folic-acid-deficiency anemia 50. A doctor recommends a patient with
B. Iron deficiency anemia duodenal ulcer to drink cabbage and potato juice
C. Myeloid leukemia after the therapy course. Which substances
D. Aplastic anemia contained in these vegetables help to heal and
E. Hemolytic anemia prevent the ulcers?
A. Vitamin U
49. Coenzym A participates in numerous B. Pantothenic acid
important metabolic reactions. It is aderivative C. Vitamin C
of the following vitamin: D. Vitamin B1
A. Pantothenic acid E. Vitamin K
B. Thiamine
C. Niacin
D. Calciferol
Fat -soluble vitamins
1. A 2 year old child suffers fromintestinal blood - decrease in activity II, VII, X blood
dysbacteriosis that lead to thedevelopment clotting factors; lengthening of blood clotting
hemorrhagic syndrome. Themost probable cause time. What vitamin deficiency are caused by
of hemorrhage is: these changes?
A. Vitamin K deficiency A. Vitamin K
B. Activation of tissue thromboplastin B. Vitamin A
C. Hypovitaminosis PP C. Vitamin C
D. Fibrinogen deficiency D. Vitamin D
E. Hypocalcemia E. Vitamin E

2. Surgery in patients with obstructive jaundice 7. Plasma coagulation factors are subject to
and malabsorption in the intestine complicated posttranslational modification with the
by bleeding. What vitamin insufficiency has led participation of vitamin K. As a cofactor, it is
to this? needed in the γ-carboxylation enzyme system of
A. Vitamin K protein coagulation factors due to an increase in
B. Vitamin B12 the affinity of their molecules by calcium ions.
C. Vitamin C What amino acid is carboxylated in these
D. Vitamin B6 proteins?
E. Folic acid A. Glutamine
B. Valin
3. Hepatic disfunctions accompaniedby C. Serine
insufficient in flow of bile to the bowels result D. Phenylalanine
in coagulation failure. This phenomenon can be E. Arginine
explained by:
A. Vitamin K deficiency 8. A 49-year-old man is observed in the clinic
B. Leucopenia with a significant increase in blood clotting
C. Thrombocytopenia time, gastrointestinal bleeding, subcutaneous
D. Erythropenia hemorrhage. What vitamin deficiency can
E. Iron deficiency explain these symptoms?
A. K
4. The function of protein synthesis in the liver B. B1
due to a lack of vitamin K is reduced in a young C. PP
man of 16 years after suffering the disease. This D. H
can lead to a violation of: E. E
A. Blood coagulation
B. Erythrocyte sedimentation rate 9. As a result of post-translational modifications
C. Education anticoagulants of certain proteins involved in blood
D. Formation of erythropoietin coagulation, in particular prothrombin, they
E. Blood pH acquire the ability to bind calcium. In this
process participate vitamin
5. In the patient observed hemorrhage, reduced A. K
the concentration of prothrombin in the blood. B. C
What vitamin deficiency led to a violation of the C. A
synthesis of this blood clotting factor? D. B1
A. K E. B2
B. A
C. D 10. To prevent postoperative bleeding, a 6-year-
D. C old child is recommended to take vikasol, which
E. E is a synthetic analogue of vitamin K. Specify
which post-translational changes of blood
6. In the patient 37 years against the background clotting factors are activated under the influence
of long-term use of antibiotics observed of vikasol?
increasing bleeding after small injuries. In the A. Carboxylation of glutamic acid
B. Phosphorylation of serine radicals 15. In the patient with renal insufficiency has
C. Partial proteolysis developed osteodystrophy, accompanied by
D. Polymerization intense bone demineralization. Violation of the
E. Glycosylation formation of which vitamin active form was the
cause of this complication?
A. Calciferol
11. In patients with the biliary tract obstruction B. Retinol
the blood coagulation isinhibited; the patients C. Thiamine
have frequent haemorrhages caused by the D. Naphthoquinone
subnormal assimilation of the following E. Riboflavin
vitamin:
A. К 16. The child has a delayed teething, their
B. А wrong arrangement, upon examination
C. D noticeable dryness of the oral cavity, in the
D. E corners of the mouth there are cracks with
E. C suppuration. With the lack of the vitamin this
condition may be related?
12. A few days before an operation apatient A. Vitamin D.
should be administered vitamin K or its B. Vitamin C.
synthetic analogue Vicasol. Vitamin K takes C. Vitamin E.
part in the following posttranslational D. Vitamin K.
modification of the II, VII, IX, X blood clotting E. Vitamin A.
factors:
A. Carboxylation 17. In a child of the first year of life is observed
B. Decarboxylation an increase in the size of the head and abdomen,
C. Deamination late teething, violation of the enamel structure.
D. Transamination The consequence of vitamin deficiencies are
E. Glycosylation these changes?
A. Hypovitaminosis D
13. A patient, who has been suffering for a long B. Hypovitaminosis C
time from intestine disbacteriosis, has increased C. Hypovitaminosis A
hemorrhaging caused by disruptionof D. Hypovitaminosis B1
posttranslational modification of blood- E. Hypovitaminosis B2
coagulation factors II, VII, IХ, and Х in the
liver. What vitamin deficiency is thecause of 18. A 6-year-old child suffers from delayed
this condition? growth, disrupted ossification processes,
A. К decalcification of the teeth.What can be the
B. B12 cause?
C. B9 A. Vitamin D deficiency
D. С B. Decreased glucagon production
E. Р C. Insulin deficiency
D. Hyperthyroidism
14. 10 month old child has high excitability, E. Vitamin C deficiency
sleep disturbance, amyotonia, retarded dentition,
teeth erupt withinadequate enamel calcification. 19. A 35-year-old female patient with a chronic
Thesechanges are caused by deficiency of renal disease has developed osteoporosis. The
thefollowing vitamin: cause of this complication is the deficiency of
A. Cholecalciferol the following substance:
B. Riboflavin A. 1,25-dihydroxy-D3
C. Thiamine B. 25-hydroxy-D3
D. Retinol C. D3
E. Nicotinamide D. D2
E. Cholesterol
20. A child with renal insufficiency exhibits C. K
delayed teeth eruption. This is mostlikely D. B1
caused by the abnormal formationof the E. PP
following substance:
A. 1,25 (OH)2D3 26. A 5 years old child has insufficient
B. Glycocyamine calcification of enamel, tooth decay. Which
C. Glutamate vitamin hypovitaminosis leads to the
D. α-ketoglutarate development of this process?
E. Hydroxylysine A. Calciferol
B. Tocopherol
21. When examining a child, the doctor revealed C. Biotin
signs of rickets. Which compounds D. Nicotinic acid
insufficiency in the child’s body facilitates the E. Folic acid
development of this disease?
A. 1,25 [OH] -dihydroxycholecalciferol 27. On examination of 11 months’ child, the
B. Biotin pediatrician found a curvature of the bones of
C. Tocopherol the lower extremities and a delay in the
D. Naphthoquinone mineralization of the bones of the skull. What
E. Retinol vitamin deficiency leads to this pathology?
A. Cholecalciferol
22. During regular check-up a child is detected B. Thiamine
with interrupted mineralization of the bones. C. Pantothenic acid
What vitamin deficiency can bethe cause? D. Bioflavonoids
A. Calciferol E. Riboflavin
B. Riboflavin
C. Tocopherol 28. A child with signs of rickets has been
D. Folic acid prescribed a certain liposoluble vitamin drug by
E. Cobalamin pediatrician and dentist. This drug affects the
metabolism of phosphorus and calcium in the
23. In patients after gallbladder removal body and facilitates calcium accumulation in
processes are hampered Ca2+ absorption through bone tissue and dentine. If its content in the
the intestinal wall. What vitamin prescription body is insufficient, there developdisruptions of
will stimulate this process? ossification process, dental
A. D3 structureandocclusion. Name this drug:
B. RR A. Ergocalciferol
C. C B. Retinolacetate
D. B12 C. Tocopherolacetate
E. K D. Menadione (Vicasolum)
E. Thyroidin
24. A patient who suffers from chronic renal
insufficiency fell ill with osteoporosis. 29. Hormonal form of a certain vitamin induces
Disturbed synthesis of what mineral genome level synthesis of Ca binding proteins
metabolism’s regulator is the cause of and enterocytes thus regulating the intestinal
osteoporosis? absorption of Ca2+ ions required for dental
A. Formation of 1, 25(OH)2D3 tissuedevelopment. What vitamin is it?
B. Proline hydroxylation A. D3
C. Lysine hydroxylation B. A
D. Glutamate carboxylation C. B1
E. Cortisol hydroxylation D. E
E. K
25. A patient has enamel erosion. What vitamin
should be administered for itstreatment?
A. D3
B. C
30. A 4-year-old child with hereditary renal 35. In order to accelerate healing of a radiation
lesion has signs of rickets; vitamin D ulcer a vitamin drug was administered. What
concentration in blood is normal. What is the drug is it?
most probable cause of rickets development? A. Retinol acetate
A. Impaired synthesis of calcitriol B. Retabolil
B. Increased excretion of calcium C. Prednisolone
C. Hyperfunction of parathyroid glands D. Levamisole
D. Hypofunction of parathyroid glands E. Methyluracil
E. Lack of calcium in food
36. While the examination of patient’s oral
31. Vitamin A together with specific cavity the dentist found xerostomia, numerous
cytoreceptors penetrates through thenuclear erosions. What vitamin deficit caused this
membranes, induces transcriptionprocesses that effect?
stimulate growth anddifferentiation of cells. A. Vitamin A
This biologicalfunction is realized by the B. Vitamin K
following formof vitamin A: C. Vitamin P
A. Trans-retinoic acid D. Vitamin H
B. Carotin E. Vitamin PP
C. Cis-retinal
D. Retinol 37. In order to prevent gum inflammation and to
E. Trans-retinal improve regeneration of epithelial periodontium
cells manufacturers add to the tooth pastes one
32. A patient suffers from vision impairment- of the following vitamins:
hemeralopy (night blindness). What vitamin A. Retinol
preparation should beadministered the patient in B. Calciferol
order torestore his vision? C. Thiamine
A. Retinol acetate D. Biotin
B. Vicasol E. Phyloquinone
C. Pyridoxine
D. Thiamine chloride 38. A 64 year old woman has impairment of
E. Tocopherol acetate twilight vision (hemeralopy). What vitamin
should be recommended in the first place?
33. An oculist detected increased time of A. A
darkness adaptation of a patient’s eye. What B. B2
vitamin deficiency can cause such symptom? C. E
A. А D. C
B. Е E. B6
C. С
D. К 39. The patient has worsened twilight vision.
E. D Which of the vitamin preparations should be
prescribed to the patient?
34. A patient complains of photoreception A. Retinol acetate
disorder and frequent acute viral diseases. He B. Cyanocobalamin
has been prescribed a vitamin that affects C. Pyridoxine hydrochloride
photoreception processes by producing D. Ascorbic acid
rhodopsin, the photosensitive pigment. What E. Nicotinic acid
vitamin is it?
A. Retinol acetate 40. The patient with periodontal disease was
B. Tocopherol acetate prescribed a fat-soluble vitamin preparation,
C. Pyridoxine hydrochloride which is actively involved in the redox
D. Cyanocobalamin processes in the body. Antioxidant is a growth
E. Thiamine factor, antixerphthalmic, provides normal
vision. In dental practice it is used to accelerate
epithelization in case of mucosal diseases with A. Tocopherol
periodontitis. Identify this drug: B. Thiamine
A. Retinol acetate C. Gluconate
B. Ergocalciferol D. Pyridoxine
C. Tocopherol acetate E. Choline
D. Vikasol
E. Cyanocobalamin 46. When treating sialadenitis (inflammation of
the salivary glands), preparations of vitamins
41. A patient has the folowing changes: disorder are used. Which of the following vitamins plays
of twilight vision, drying out of conjunctiva and an important role in antioxidant defense?
cornea. Such disordersmay be caused by A. Tocopherol
deficiency of vitamin: B. Pantothenic acid
A. Vitamin A C. Riboflavin
B. Vitamin B D. Thiamine
C. Vitamin C E. Pyridoxine
D. Vitamin D
E. Vitamin B12 47. A woman, who had undergone mastectomy
due to breast cancer, wasprescribed a course of
42. A patient complains of photoreception radiation therapy.What vitamin preparation has
disorder and frequent acute viral diseases. He markedantiradiation effect due to its
has been prescribed a vitamin that affects antioxidantactivity?
photoreception processes by producing A. Tocopherol acetate
rhodopsin, the photosensitive pigment. B. Ergocalciferol
What vitamin is it? C. Riboflavin
A. Retinol acetate D. Cyanocobalamin
B. Tocopherol acetate E. Folic acid
C. Pyridoxine hydrochloride
D. Cyanocobalamin 48. What vitamin deficiency leads to both
E. Thiamine disorder of reproductive function and distrophy
of skeletal muscles?
43. There are various diseases that cause sharp A. Vitamin E
increase of active oxygen, leading tocell B. Vitamin A
membranes destruction. Antioxidantsare used to C. Vitamin K
prevent it from happening. Themost potent D. Vitamin D
natural antioxidant is: E. Vitamin B1
A. Alpha-tocopherol
B. Glycerol 49. Ionizing radiation or vitamin E
C. Vitamin D deficiencyaffect the cell by increasing
D. Fatty acids lysosomemembrane permeability.What are the
E. Glucose possibleconsequences of this pathology?
A. Partial or complete cell destruction
44. A pregnant woman with several B. Intensive protein synthesis
miscarriagesin anamnesis is prescribed atherapy C. Intensive energy production
that includes vitamin preparations.What vitamin D. Restoration of cytoplasmic membrane
facilitates carrying of apregnancy? E. Formation of maturation spindle
A. Alpha-tocopherol
B. Folic acid 50. Examination of a man who hadn’t been
C. Cyanocobalamin consuming fats but had been gettingenough
D. Pyridoxal phosphate carbohydrates and proteinsfor a long time
E. Rutin revealed dermatitis, poor wound healing, vision
impairment. Whatis the probable cause of
45. Parodontosis is treated by meansof metabolic disorder?
antioxidants. Which of the followingnatural A. Lack of linoleic acid, vitamins A, D, E, K
compounds is used as an antioxidant: B. Lack of palmitic acid
C. Lack of vitamins PP, H
D. Low caloric value of diet
E. Lack of oleic acid

51. For the prevention of atherosclerosis,


coronary heart disease, cerebrovascular accident,
the consumption of high-fat polyunsaturated fatty
acids is recommended. One of the following fatty
acids is:
A. Linolic
B. Oleic
C. Lauric
D. Palmitooleic
E. Stearic
Basic laws of metabolism and energy
1. The Krebs cycle plays an important role in E. They inhibit cytochrome B
the realization of the glucoplastic effect of
amino acids. This is due to the mandatory 6. A patient with poisoning with an insecticide-
transformation of a nitrogen-free residue into: rotenone was taken to the hospital. Which part
A. Oxaloacetate. of the mitochondrial electron transfer chain is
B. Malate. blocked by this substance?
C. Succinate. A. NADH coenzyme Q-reductase.
D. Fumarat. B. ATP synthetases.
E. Citrate. C. Coenzyme Q-cytochrome C-reductase.
D. Succinate-coenzyme Q-reductase.
2. At the patient of 57 years, suffering from a E. Cytochrome C-oxidase.
diabetes, the keto-acidosis has developed. The
biochemical basis of this condition is a decrease 7. Rotenone is known to inhibit respiratory
in the degree of utilization of acetyl-CoA. The chain. What complex of mitochondrial
disadvantage of which is the connection in cells respiratory chain is inhibited by this substance?
that is due? A. NADH-coenzymeQreductase
A. Oxaloacetate B. Cytochromeoxidase
B. 2-oxoglutarate C. CoenzymeQ-cytochromecreductase
C. Glutamate D. Succinate-coenzyme Qreductase
D. Aspartate E. Adenosine triphosphate synthetase
E. Succinate
8. In the intensive care unit in a serious
3. During the oxidation of carbohydrates, lipids, condition, unconscious, the patient was
a large amount of energy is formed, the bulk of admitted. Diagnosed overdose of barbiturates,
which is formed due to the oxidation of acetyl- which caused the phenomenon of tissue
CoA. How many ATP molecules are formed hypoxia. At what level was blocking the
when a single molecule of acetyl-CoA is electron transport?
completely oxidized? A. NADH-Coenzyme-Q-reductase
A. 12. B. Cytochrome oxidase
B. 8. C. Cytochrome b - cytochrome c1
C. 38. D. Ubiquinone
D. 24. E. ATP synthase
E. 36
9. Hyperthermia, bulimia, weight loss, which is
4. Prussic acid and cyanide are among the observed in patients with thyrotoxicosis,
strongest poisons. Depending on the dose, death associated with a violation:
occurs in a few seconds or minutes. The A. Conjugation of oxidation and
inhibition of the activity of which enzyme is the phosphorylation.
cause of death? B. Reactions of fat synthesis.
A. Cytochrome oxidase. C. Decay of ATP.
B. Acetyl holinesesterase. D. Reactions of the citric acid cycle
C. Catalase. E. Reactions of beta-oxidation of fatty acids.
D. Methemoglobin reductase.
E. ATP synthetases. 10. With thyrotoxicosis, the production of
thyroid hormones T3 and T4 increases, weight
5. Cyanide poisoning causes immediate death. loss develops, tachycardia, mental excitability
What is the mechanism of cyanide effect at the and so on. The mechanism of their action?
molecular level? A. separates oxidation and oxidative
A. They inhibit cytochromoxidase phosphorylation.
B. They bind substrates of tricarboxylic acid B. Activate substrate phosphorylation.
cycle C. Block the substrate by phosphorylation.
C. They block succinate dehydrogenase D. Block the respiratory chain.
D. They inactivate oxygene E. Oxidative phosphorylation is activated.
11. A woman who is receiving treatment for cytochrome C. What is its function in a normal
hyperthyroidism, there is an increase in body cell?
temperature. What is the basis of this A. Enzyme of respiratory chain of
phenomenon? electrontransport
A. Separation of oxidative phosphorylation B. Enzyme of tricarboxylic acid cycle
B. Decreased glucose utilization by tissues C. Enzyme of beta-oxidation of fatty acids
C. Reduced fat oxidation in the liver D. Component of H+ATP system
D. Violation of amino acid deamination E. Component of pyruvate-
E. Violation of glycogen synthesis dehydrogenasesystem

12. A 38-year-old woman complains of 16. Cyanide is a poison that causes instant death
increased sweating, heartbeat, and an increase in of the organism. What enzymes found in
temperature in the evening. The main exchange mitochondria are affected by cyanide?
is increased by 60%. The doctor diagnosed A. Cytochrome oxidase (aa3)
thyrotoxicosis. What properties of thyroxin lead B. Flavin enzymes
to increased heat production? C. Cytochrome 5
A. Separation of oxidative phosphorylation D. NAD+-dependent dehydrogenase
B. Increases conjugation of oxidation and E. Cytochrome P-450
phosphorylation
C. Reduces β-oxidation of fatty acids 17. Hydrocyanic acid and cyanides are the most
D. Reduces the deamination of amino acids violent poisons. According to the dose the death
E. Contributes to the accumulation of acetyl follows after a few seconds or minutes. The
CoA death is caused by the inhibited activity of the
following enzyme:
13. Cells were treated with a substance that A. Cytochrome oxidase
blocks the phosphorylation of nucleotides in the B. Acetylcholinesterase
mitochondria. The process of cell activity will C. ATP-synthetase
be violated in the first place? D. Catalase
A. Oxidative phosphorylation. E. Methemoglobin reductase
B. Glycolysis.
C. Integration of functional protein molecules. 18. Potassium cyanide that is a poison came into
D. Aerobic oxidation of glucose a patient’s organism and caused death a few
E. Synthesis of mitochondria proteins. minutes after it. The most probable cause of its
toxic effect was abnormal activity of:
14. In the presence of 2,4-dinitrophenol, the A. Cytochrome oxidase
oxidation of substrates can continue, but the B. Catalase
synthesis of ATP molecules is impossible. What C. ATP-synthetase
is the mechanism of its action? D. NADP − H-dehydrogenase
A. Separation of oxidation and phosphorylation E. Haemoglobin synthesis
in mitochondria
B. Activation of the enzyme ATPase 19. The resuscitation unit has admitted a patient
C. Transfer of substrates beyond mitochondria in grave condition. It is known that he had
D. Stimulation of hydrolysis of formed ATP mistakenly taken sodium fluoride which blocks
E. Inhibition of the enzyme cytochrome oxidase cytochrome oxidase. What type of hypoxia
developed in the patient?
15. Researches of the latest decades established A. Tissue
that immediate "executors"ofcell apoptosis are B. Hemic
special enzymes calledcaspases. Generation of C. Cardiovascular
one of themproceeds with participation of D. Hypoxic
E. Respiratory
Carbohydrate metabolism and its regulation
1. In a patient of 60 years the activity of the 6. Analysis of a patient’s saliva revealedhigh
main digestive enzyme of saliva is reduced. In concentration of lactate. This is mostprobably
this case, the primary hydrolysis of which caused by activation of thefollowing process:
organic compounds is disturbed? A. Anaerobic glucose breakdown
A. Carbohydrates. B. Aerobic glucose breakdown
B. Fats. C. Glycogen breakdown
C. Proteins. D. Carbohydrate hydrolysis
D. Cellulose. E. Glucose-lactate cycle
E. Lactose.
7. Clinical examination enabled to make a
2. With age, the activity of the parotid glands provisional diagnosis: stomach cancer. Gastric
decreases.The activity of what enzyme of juice contained lactic acid.What type of glucose
carbohydrate metabolism will decrease? catabolism turns upin the cancerous cells?
A. Amilase. A. Anaerobic glycolysis
B. Lysozyme. B. Pentose-phosphate cycle
C. Phosphatase. C. Gluconeogenesis
D. Hexokinase. D. Aerobic glycolysis
E. Maltase. E. Glucose-alanine cycle

3. A newborn develops dyspepsia after the milk 8. What process provides erythrocytes with the
feeding. When the milk is substituted by the required amount of energy in the form of ATP
glucose solution the dyspepsia symptoms for their vital activity?
disappear. The newborn has the subnormal A. Glycolysis.
activity of the following enzyme: B. Aerobic oxidation of glucose.
A. Lactase C. β-Oxidation of fatty acids.
B. Invertase D. Pentose phosphate cycle.
C. Maltase E. Tricarbonic acid cycle.
D. Amylase
E. Isomaltase 9. Human red blood cells do notcontain
mitochondria. What is themain pathway for
4. While determining power inputs of apatient’s ATP production inthese cells?
organism it was established thatthe respiratory A. Anaerobic glycolysis
coefficient equaled 1,0. This means that in the B. Aerobic glycolysis
cells of the patientthe following substances are C. Oxidative phosphorylation
mainlyoxidized: D. Creatine kinase reaction
A. Carbohydrates E. Cyclase reaction
B. Proteins
C. Fats 10. During consumption of biscuits, sweets in
D. Proteins and carbohydrates the mixed saliva temporarily increases the level
E. Carbohydrates and fats of lactate. The activation of which biochemical
process leads to this?
5. Аfter the transition to a mixed diet in a A. Anaerobic glycolysis
newborn child has arisen dyspepsia with B. Tissue breathing
diarrhea, meteorism developmental delay.The C. Aerobic glycolysis
biochemical basis of this pathology is D. Gluconeogenesis
insufficiency of: E. Microsomal oxidation
A. Saccharaseandisomaltase.
B. Lactaseandcellobiase. 11. During long-distance running, skeletal
C. Trypsinandchymotrypsin. muscle of a trained person uses glucose to
D. Lipaseandcreatinekinase. obtain the energy of ATP for muscle
E. Cellulase. contraction. Indicate the main process of
utilization of glucose in these conditions:
A. Aerobic glycolysis
B. Anaerobic glycolysis D. Enolase
C. Glycogenolysis E. Lactate dehydrogenase
D. Gluconeogenesis
E. Glycogenesis 17. Cytoplasm of the myocytes contains a lot of
dissolved metabolites resulting fromglucose
12. After restoration of blood circulationin oxidation. Name the metabolite that turns
damaged tissue accumulation oflactate comes to directly into lactate:
a stop and speed ofglucose consumption slows A. Pyruvate
down. Thesemetabolic changes are caused by B. Oxaloacetate
activationof the following process: C. Glycerophosphate
A. Aerobic glycolysis D. Glucose-6-phosphate
B. Anaerobic glycolysis E. Fructose-6-phosphate
C. Lipolysis
D. Gluconeogenesis 18. Treatment of many diseasesinvolves use of
E. Glycogen biosynthesis cocarboxylase (thiaminepyrophosphate) for
supplying cells withenergy. What metabolic
13. When blood circulation in the damaged process is activatedin this case?
tissue is restored, then lactateaccumulation A. Oxidizing decarboxylation of pyruvate
comes to a stop andglucose consumption B. Glutamate deamination
decelerates. These metabolic changes are caused C. Amino acids decarboxylation
by activation of the following process: D. Decarboxylation of biogenic amines
A. Aerobic glycolysis E. Detoxication of harmful substances inliver
B. Anaerobic glycolysis
C. Lipolysis 19. A worker of a chemical enterprise was taken
D. Gluconeogenesis to the hospital with signs of poisoning. In the
E. Glycogen biosynthesis hair of this woman found increased
concentration of arsenate, which blocks lipoic
14. A 32-year-old female patientsuffers from acid. Violation of which process is the most
gingivitis accompaniedby gum hypoxia. What likely cause of poisoning?
metabolite ofcarbohydrate metabolism is A. Oxidation decarboxylation of pyruvate.
produced inthe periodontium tissues more B. Microsomal oxidation.
actively inthis case? C. Restoration of methemoglobin.
A. Lactate D. Restoration of organic peroxides.
B. Ribose 5-phosphate E. Removal of superoxide ions.
C. Glycogen
D. Glucose 6-phosphate 20. Oxidative decarboxylation of pyruvic acid is
E. NADPH-H catalyzed by a multienzyme complex with
several functionally linked coenzymes. Name
15. Untrained people often have musclepain this complex:
after sprints as a result of lactateaccumulation. A. Thymidine diphosphate (TDP), flavin
This might be caused byintensification of the adenine dinucleotide (FAD), coenzyme A
following biochemicalprocess: (CoASH), nicotine amide adenine dinucleotide
A. Glycolysis (NAD), lipoic acid
B. Gluconeogenesis B. Flavin adenine dinucleotide (FAD),
C. Pentose phosphate pathway tetrahydrofolic acid, pyridoxal-5-phosphate,
D. Lipogenesis thymidine diphosphate (TDP), choline
E. Glycogenesis C. Nicotine amide adenine dinucleotide (NAD),
pyridoxal-5-phosphate, thymidine diphosphate
16. Anaerobic splitting of glucose to lactic acid (TDP), methylcobalamin, biotin
is regulated by the relevant enzymes. What D. Coenzyme A (CoASH), flavin adenine
enzyme is the main regulator of this process? dinucleotide (FAD), pyridoxal-5- phosphate,
A. Phosphofructokinase tetrahydrofolic acid,carnitine
B. Glucose-6-phosphate isomerase E. Lipoic acid, tetrahydrofolic acid, pyridoxal-
C. Aldolase 5-phosphate, methylcobalamin
21. It has been found out that one of pesticide consumed a small amount of alcohol, resulting
components is sodium arsenate that blocks in severe poisoning. Explain the cause of
lipoic acid. Which enzyme activityis impaired poisoning:
by this pesticide? A. Accumulation of acetaldehyde
A. Pyruvate dehydrogenase complex B. Allergic reaction
B. Microsomal oxidation C. Neuralgia disorders
C. Methemoglobin reductase D. Cardiovascular insufficiency
D. Glutathione peroxidase E. Violation of the function of the kidneys
E. Glutathione reductase
27. The biosynthesis of the purine ring occurs
22. In erythrocytes of the patient with hemolytic on ribose-5-phosphate by gradual build-up of
anemia, the activity of pyruvate kinase was nitrogen and carbon atoms and the closure of
significantly reduced. What metabolic process is the rings. The source of ribose-5-phosphate is
violated under these conditions? the following process:
A. Glycolysis A. Pentose phosphate cycle
B. Glycogenolysis B. Glycolysis
C. Gluconeogenesis C. Glyconeogenesis
D. Pentosophosphate pathway of glucose D. Gluconeogenesis
oxidation E. Glycogenolysis
E. Synthesis of glycogen
28. Due to the long-term use of sulfanilamide
23. Decreased ratio of adenylic nucleotides drugs in a young woman appeared signs of
ATP/ADP results in intensifiedglycolysis in hemolytic anemia due to hereditary violation of
parodentium tissues underhypoxia conditions. the synthesis of the enzyme pentosophosphate
What reaction is activatedin this case? pathway glucose-6-phosphate dehydrogenase,
A. Phosphofructokinase which provides formation in the body:
B. Aldolase A. NADРН.
C. Triosophosphate isomerase B. ATP.
D. Enolase C. NAD
E. Lactate dehydrogenase D. FAD
E. FMN.
24. In some anaerobic bacteria the pyruvate
produced by glycolysis is converted to the ethyl 29. 22 year old woman has been taking
alcohol (alcoholic fermentation). What is the sulfanilamides for a long time that led to
biological significance of this process? symptoms of hemolytic anaemia caused by
A. NAD+ replenishment hereditary disturbance of synthesis of glucose 6-
B. Lactate production phosphate dehydrogenase. This enzyme of
C. ADP production pentose-phosphate cycle is responsible for
D. Providing the cells with NADPH generation of:
E. ATP production A. NADP − H2
B. NAD
25. In medical practice for preventionof C. FAD
alcoholism, a drug that is an aldehyde D. FMN
dehydrogenase inhibitor is widely used. The E. ATP
increase of which metabolite in the blood causes
disgust to alcohol? 30. Sulfanilamides are applied as antimicrobal
A. Acetaldehyde agents in clinical practice. Sulfanilamide
B. Ethanol treatment, however, can result in hemolytic
C. Malone aldehyde anemia development in patients that suffer from
D. Propionic aldehyde genetic defect of the following enzyme of
E. Methanol pentose phosphatemetabolismin erythrocytes:
A. Glucose-6-phosphate dehydrogenase
26. The patient during the course of treatment B. Hexokinase
with a drug that blocks alcohol dehydrogenase, C. Transketolase
D. Transaldolase deficiency in the galactose-1-
E. Pyruvatekinase phosphate uridylyltransferase. What is the
pathological process in the child?
31. 38 year old patient takes aspirin and A. Galactosaemia
sulfanilamides. After their intake intensified B. Fructosemia
erythrocyte haemolysis is observed which is C. Hyperglycemia
caused by deficiency of glucose 6-phosphate D. Hypoglycemia
dehydrogenase. This pathology is caused by E. Hyperlactate acidemia
failure of the following coenzyme:
A. NADP − H 36. In the 2-year-old boy, an increase in the size
B. FAD − H2 of the liver and spleen, cataract is observed. The
C. Pyridoxal phosphate concentration of sugar is elevated in the blood,
D. FMN − H2 but the test of glucose tolerance is normal. An
E. Ubiquinone hereditary violation of the metabolism of which
substance is the cause of this condition?
32. The patient has a chronic inflammatory A. Galactose
process of the tonsils. Due to the what B. Fructose
biochemical process in the inflammation C. Glucose
centers, the concentration of NADPH, which is D. Maltose
necessary for the implementation of the E. Saccharose
mechanism of phagocytosis, is maintained?
A. Pentose phosphate pathway 37. Indicate, hereditary insufficiency of which
B. Cori Cycle enzyme is the cause of vomiting and diarrhea
C. Krebs cycle after taking the fruit juices in a 9-month-old
D. Ornithine cycle child whose fructose intake has led to
E. Glycolysis hypoglycemia?
A. Fructose-1-phosphataldolase.
33. A 7-year-old child has symptoms of B. Phosphofructokinase
hemolytic anemia. In the biochemical analysis C. Hexokinase.
of erythrocytes, a reduced concentration of D. Fructose-1,6-diphosphatase.
NADPH and reduced glutathione has been E. Fructokinase.
established. The deficiency of which enzyme in
this case causes biochemical changes and 38. In the 8-month-old child, vomiting and
clinical manifestations? diarrhea are observed after taking fruit juices.
A. Glucose-6-phosphate dehydrogenase Fructose intake led to hypoglycemia. Hereditary
B. Hexokinase insufficiency of which enzyme is the cause of
C. Fructokinase these disorders?
D. Pyruvate kinase A. Fructose-1-phosphataldolase
E. Lactate dehydrogenase B. Fructokinase
C. Hexokinase
34. A child’s blood presents high content of D. Phosphofructokinase
galactose, glucose concentration is low. There E. Fructose-1,6-diphosphatase
are such presentations as cataract, mental
deficiency, adipose degeneration of liver.What 39. The excess concentration of glucose in the
disease is it? oral solution in diabetes leads to development:
A. Galactosemia A. Multiple caries
B. Diabetes mellitus B. Hyperplasia of enamel
C. Lactosemia C. Hypoplasia of enamel
D. Steroid diabetes D. Fluorosis
E. Fructosemia E. Intensifiedcalcification of enamel

35. A sick child has a delayed mental 40. In the biochemical study of the patient's
development, enlarged liver, and decreased blood, hyperglycemia, hyperketonemia, glucose
vision. The physician binds these symptoms to and ketone bodies in urine were detected. On
the electrocardiogram diffuse changes in the A. Decrease of proteosynthesis
myocardium was detected. She complains of dry B. Increase of lipolysis
mouth, thirst, frequent urination, general C. Acceleration of gluconeogenesis
weakness. The patient suffers from: D. Reduction of lipolysis
A. Diabetes mellitus. E. Increase of catabolism
B. Alimentary hyperglycemia.
C. Acute pancreatitis. 45. Diabetes mellitus causes ketosis as a result
D. Ischemic heart disease. of activated oxidation of fatty acids. What
E. Non-diabetes mellitus. disorders of acid-base equilibrium may be
caused by excessive accumulation of ketone
41. A 42 year old woman diagnosed with bodies in blood?
diabetes mellitus was admitted the A. Metabolic acidosis
endocrinological department with complaints of B. Metabolic alcalosis
thirst, excessive appetite. What pathological C. Any changes woun’t happen
components are revealed in course of laboratory D. Respiratory acidosis
examination of the patient’s urine? E. Respiratory alcalosis
A. Glucose, ketone bodies
B. Protein, aminoacids 46. A patient is ill with diabetes mellitus that is
C. Protein, creatine accompanied by hyperglycemia of over 7,2
D. Bilirubin, urobilin millimole/l on an empty stomach. The level of
E. Blood what blood plasma protein allows to estimate
the glycemia rate retrospectively (4-8 weeks
42. A patient with insulin-dependent diabetes before examination)?
mellitus has been injected with insulin. After A. Glycated hemoglobin
some time at the patient has developed B. Albumin
weakness, irritability, and increased sweating. C. Fibrinogen
What is the main mechanism of development of D. C-reactive protein
hypoglycemic coma? E. Ceruloplasmin
A. Carbohydrate starvation of the brain.
B. Decrease of gluconeogenesis. 47. A 62-year-old female patient has developed
C. Increaseof glycogenolysis. a cataract (lenticular opacity) secondary to the
D. Increase of ketogenesis. diabetes mellitus. What type of protein
E. Increase of lipogenesis. modification is observed in case of diabetic
cataract?
43. Examination of a 56-year-oldfemale patient A. Glycosylation
with a history of type 1 diabetes revealed a B. Phosphorylation
disorder ofprotein metabolism that is C. ADP-ribosylation
manifestedby aminoacidemia in the laboratory D. Methylation
blood test values, and clinically by the delayed E. Limited proteolysis
wound healing and decreased synthesis of
antibodies. Which ofthe following mechanisms 48. After recovering from epidemic parotiditis a
causes the development of aminoacidemia? patient began to put off weight, he was
A. Increased proteolysis permanently thirsty, drank a lot of water, had
B. Albuminosis frequent urination, voracious appetite. Now he
C. Decrease in the concentration ofamino acids has complaints of skin itch, weakness,
in blood furunculosis. His blood contains: glucose - 16
D. Increase in the oncotic pressure inthe blood mmole/L, ketone bodies - 100 mcmole/L;
plasma glucosuria. What disease has developed?
E. Increase in low-density lipoproteinlevel A. Insulin-dependent diabetes
B. Insulin-independent diabetes
44. It is known that in patients with diabetes C. Steroid diabetes
mellitus often found inflammatory processes, D. Diabetes insipidus
reduced regeneration, and healing of wounds E. Malnutrition diabetes
decreases. The reason for this is:
49. Patient with diabetes mellitus experienced D. Itsenko-Kushing's disease
loss of consciousness and convulsions after an E. Akromegalia
injection of insulin. What might be the result of
biochemical blood analysis for concentration of 54. In a patient 15 years of age, glucose
sugar? concentration is 4,8 mmol/l, an hour after the
A. 1,5 mmol/L sugar intake - 9.0 mmol/l, after 2 hours - 7.0
B. 8,0 mmol/L mmol/l, after 3 hours - 4.8 mmol/l . These
C. 10,0 mmol/L indices are characteristic of such a disease:
D. 3,3 mmol/L A. Hidden diabetes mellitus
E. 5,5 mmol/L B. Type I diabetes
C. Type II diabetes
50. A 46-year-old patient complains of dry D. Itsenko-Kushing's disease
mouth, thirst, urination, general weakness. In E. –
the blood: hyperglycemia, hyperketonemia. In
the urine: glucose, ketone body. On the ECG: 55. A nurse accidentally injected a nearly
diffuse changes in myocardium. What is the double dose of insulin to a patient with diabetes
most probable diagnosis? mellitus. The patient lapsed into a hypoglycemic
A. Diabetes mellitus coma. What drug should be injected in order to
B. Alimentary hyperglycemia help him out of coma?
C. Acute pancreatitis A. Glucose
D. Non-diabetes mellitus B. Lidase
E. Ischaemic heart disease C. Insulin
D. Somatotropin
51. After the epidemic parotitis, the patient loses E. Noradrenaline
weight, constantly feels thirst, drinks a lot of
water, indicates frequent urination, increased 56. For type II diabetes, the characteristic
appetite, itching, weakness, furunculosis. In the features are hyperglycemia, hypochloremia.
blood: glucose - 16 mmol/l, ketone bodies - 100 Which of the following processes is activated in
μmol/l; glucosuria What disease has developed the first place?
in the patient? A. Gluconeogenesis.
A. Insulin-dependent diabetes mellitus B. Glycolysis.
B. Non-dependent diabetes mellitus C. Glycogenolysis.
C. Steroidal diabetes D. Glucose reabsorption.
D. Non-diabetes mellitus E. Transport of glucose to the cell.
E. Malnutrition-related diabetes mellitus
57. A 57 year old patient with diabetes mellitus
52. A 38-year-old patient was delivered to the was developed ketoacedosis. Biochemical base
reanimation unit with unconscious. Reflexes are of this condition is smaller extent of acetyl-
absent. Blood sugar is 2.1 mmol/L. In CoAutilization.What cell compound deficit
anamnesis - diabetes since 18 years of age. causes this effect?
What coma does a patient have? A. Oxaloacetate
A. Hypoglycemic B. 2-oxoglutarate
B. Ketoacidotic C. Glutamate
C. Lactacidotic D. Aspartate
D. Hyperosmolar E. Succinate
E. Hyperglycemic
58. A patient with diagnose of diabetes mellitus,
53. In the patient's blood, glucose is 5.6 in the morning received an assigned dose of
mmol/L, 1 hour after the sugar intake - 13.8 insulin of prolonged action. He missed the next
mmol/l, and after 3 hours - 9.2 mmol/l. What meal, and soon felt weakness, headache,
pathology is characterized by such indicators? dizziness, greediness, trembling of the body,
A. Hidden form of diabetes mellitus convulsions, hunger, and the phenomenon of
B. Healthy person hypoglycemia. The use of glucose does not
C. Thyrotoxicosis
improve the condition. What drug should be D. Low (absent) activity of glucose 6-
entered to alleviate this condition? phosphatase
A. Adrenaline E. Deficit of a gene that is responsible for
B. Triamcinolone synthesis of glucose 1-phosphaturidine
C. Noradrenaline transferase
D. Prednisolone
E. Hydrocortisone 63. After the introduction of adrenaline in a
patient with persistent hypoglycemia, blood test
59. A patient for 5 years suffering from has not changed significantly. In such
diabetes. As a result of a violation of a diet, he circumstances, there is a possibility of
developed a coma. An ambulance doctor gave disturbances in the liver. What function of the
him glucose. The condition of the patient has liver is changed?
improved. What kind of coma was in the A. Glycogendeposing.
patient? B. Glycolytic.
A. Hypoglycemic C. Excretory.
B. Acidotic D. Ketogenic.
C. Hyperglycemic E. Cholesterol-forming.
D. Hepatic
E. Hypothyroid 64. A 2-year-old child has been diagnosed with
Girke's disease, which is manifested by severe
60. Patients with Itsenko-Cushing's syndrome hypoglycemia. The reason for this condition is
are observed with a hyperglycemia and the absence of the enzyme glucose-6-
glucosuria. Synthesis and secretion of which phosphatase. With the violation of which
hormone is increased in this patient? process is this pathology involved?
A. Cortisol A. Mobilization of glycogen.
B. Adrenaline B. Gluconeogenesis.
C. Glucagone C. Glycolysis.
D. Thyroxine D. Ketogenesis.
E. Aldosterone E. Synthesis of glycogen.

61. Pancreas is known as a mixed gland. 65. The glycogen that came from the food was
Endocrine functions include production of digested in the gastrointestinal tract. Which final
insulin by beta cells. This hormone affects the product was formed as a result of this process?
metabolism of carbohydrates. What is its effect A. Glucose
upon the activity of glycogen phosphorylase B. Lactate
(GP) and glycogen synthase (GS)? C. Lactose
A. It inhibits GP and activates GS D. Galactose
B. It activates both GP and GS E. Fructose
C. It inhibits both GP and GS
D. It activates GP and inhibits GS 66. In the human diet there are a large number
E. It does not affect the activity of GP and GS of carbohydrates. The number of which
structures will increase in cytoplasm of
62. Medical ambulance delivered a 2 year old hepatocytes?
girl to the children’s department.Objectively: A. Glycogen granules
the child is languid, apathetic. Liver is enlarged B. Drops of fat
and liver biopsy revealed a significant excess of C. The lysosomes
glycogene. Glucose concentration in the blood D. Free ribosomes
streamis below normal. What is the cause of low E. Inclusion of lipofuscine
glucose concentration?
A. Low (absent) activity of glycogene 67. Andersen's disease belongs to a group of
phosphorylase in liver hereditary diseases that develop due to the
B. Low (absent) activity of hexokinase congenital malformation of the synthesis of
C. High activity of glycogen synthetase certain enzymes of glycogenolysis. The
inadequacy of which enzyme is the molecular B. Gluconeogenesis in the muscles.
basis of this glycogenose? C. Glycogenolysis.
A. Amilo (1-4 → 1-6) transglycosidases. D. Decarboxylation.
B. Glycogen synthase. E. Synthesis of higher fatty acids.
C. Glucose-6-phosphatase.
D. Lysosomal glycosidase. 73. In a patient undergoing a course of medical
E. Phosphofructokinase. starvation, the normal level of glucose in the
blood is maintained mainly due to
68. Characteristic sign of glycogenosis is gluconeogenesis. From what amino acids at the
muscle pain during physical work. Blood same time in the human liver most actively
examination reveals usually hypoglycemia. This synthesized glucose?
pathology is caused by congenital deficiency of A. Alanine
the following enzyme: B. Lysine
A. Glycogen phosphorylase C. Valine
B. Glucose 6-phosphate dehydrogenase D. Glutamic acid
C. Alpha amylase E. Leucine
D. Gamma amylase
E. Lysosomal glycosidase 74. The gluconeogenesis is activated in the liver
after intensive physical trainings. What
69. During starvation normal rate ofglucose is substance is utilized in gluconeogenesis first of
maintained by means ofactivation of all in this case:
gluconeogenesis. Whatsubstance can be used as A. Lactate
a substrate forthis process? B. Pyruvate
A. Alanine C. Glucose
B. Ammonia D. Glutamate
C. Adenine E. Alanine
D. Urea
E. Guanine 75. During intensive physical work, muscle
tissue accumulates lactic acid, which diffuses
70. It is known that human carbohydrate into the blood and is absorbed by the liver and
reserves rapidly disappear as a result of heart. What process ensures the recovery of
prolonged fasting. Which of the metabolic glycogen stores in the muscles?
processes restores glucose in the blood? A. Cori Cycle
A. Gluconeogenesis. B. Citric acid cycle
B. Aerobic oxidation of glucose. C. Ornithine cycle
C. Glycolysis. D. Cycle of tricarboxylic acids
D. Glycogenolysis. E. Pentose phosphate pathway
E. Pentose phosphate pathway.
76. The patient was diagnosed with beri-beri.
71. Hyperglycemia develops due to chronic What is the enzyme's activity affected by the
overdose of glucocorticoids in a patient. Specify patient?
the process of carbohydrate metabolism, due to A. Pyruvate dehydrogenase.
which the concentration of glucose increases: B. Citrate synthase.
A. Gluconeogenesis. C. Malate dehydrogenase.
B. Aerobic oxidation of glucose. D. Succinate dehydrogenase.
C. Glycogenolysis. E. Fumarase.
D. Glycogenenesis.
E. Pentose phosphate cycle. 77. If insufficient thiamine - vitamin B1 occurs
disease beriberi (polyneuritis) and disturbed
72. During the fasting the muscle proteins break carbohydrate metabolism. What metabolite thus
down into free amino acids. In which process accumulates in the blood?
will most likely be used amino acids under such A. Pyruvate.
conditions? B. Lactate.
A. Gluconeogenesis in the liver. C. Succinate.
D. Citrate. 82. Glycogen polysaccharide is synthesizedfrom
E. Malat. the active form of glucose. Theimmediate donor
of glucose residues duringthe glycogenesis is:
78. Some students developed myodynia after A. UDP-glucose
continuous physical activity duringphysical B. Glucose-1-phosphate
education. The reason for suchcondition was C. ADP-glucose
accumulation of lactic acidin the skeletal D. Glucose-6-phosphate
muscles. It was generatedin the students’ bodies E. Glucose-3-phosphate
after activation ofthe following process:
A. Glycolysis 83. A child with point mutation presents with
B. Gluconeogenesis absence of glucose-6-phosphatase,
C. Lipolysis hypoglycemia, and hepatomegaly. What
D. Pentose-phosphate cycle pathology are these signs characteristic of?
E. Glyconeogenesis A. Von Gierke’s disease (Glycogen storage
disease type I)
79. It is known that the pentosephosphate B. Cori’s disease (Glycogen storage disease
pathway occuring in the adipocytesof adipose type III)
tissue acts as a cycle.What is the main function C. Addison’s disease (Primary adrenal
of this cycle inthe adipose tissue? insufficiency)
A. NADPH2 generation D. Parkinson’s disease
B. Ribose-phosphate production E. McArdle’s disease (Glycogen storage disease
C. Xenobiotic detoxification type V)
D. Energy generation
E. Glucose oxidation to end products 84. A child has a history ofhepatomegaly,
hypoglycemia, seizures,especially on an empty
80. Fructosuria is known to be connected with stomach andin stressful situations. The child is
inherited deficiency of fructose-1-phosphate diagnosedwith Gierke disease. This diseaseis
aldolase.What product of fructose metabolism caused by the genetic defect ofthe following
will accumulate in the organism resulting in enzyme:
toxicaction? A. Glucose-6-phosphatase
A. Fructose-1-phosphate B. Amyloid-1,6-glycosidase
B. Glucose-1-phosphate C. Phosphoglucomutase
C. Glucose-6-phosphate D. Glycogen phosphorylase
D. Fructose-1,6-biphosphate E. Glucokinase
E. Fructose-6-phosphate
85. In patients with glycogenosis, that is von
81. The genetic defect of pyruvatecarboxylase Gierke’s disease, the conversion of glucose-6-
deficiency is the cause of delayed physical and phosphate in to glucose is inhibited, which is
mental development and early death in children. accompanied by the improper breakdown of
This defectis characterized by lacticemia, glycogen in the liver. The cause of this
lactaciduria, disorder of a number of metabolic condition is the following enzyme deficiency:
path ways. In particular, the following process is A. Glucose-6-phosphatase
inhibited: B. Glycogen phosphorylase
A. Citric acid cycle and gluconeogenesis C. Glucose-6-phosphate dehydrogenase
B. Glycolysis and glycogenolysis D. Phosphofructokinase
C. Glycogenesis and glycogenolysis E. Phosphoglucomutase
D. Lipolysis and lipogenesis
E. Pentose phosphate pathway andglycolysis
Metabolism of lipids and its regulation
1. Examination of a patient revealed that dental E. Electrolytes
hypoplasia was caused by hypovitaminosis of
vitamins A and D. These vitamins were 6. A patient was prescribed a drug with apparent
administered perorally but they didn’t have any lipophilic properties. What is the main
medicinal effect. What is the probable cause of mechanism of its absorption?
disturbed vitamin assimilation? A. Passive diffusion
A. Bile acid deficiency B. Active transporting
B. Phospholipase A2 deficiency C. Binding with transport proteins
C. Cholesterolesterase deficiency D. Pinocytosis
D. Colipase deficiency E. Filtration
E. Pancreatic lipase deficiency
7. Due to the blockage of the common bile duct
2. Examination of a man who hadn’t been (which was radiographically confirmed), the
consuming fats but had been getting enough biliary flow to the duodenum was stopped. We
carbohydrates and proteins for long time should expect the impairment of:
revealed dermatitis, poor wound healing, vision A. Fat emulsification
impairment. Whatis the probable cause of B. Protein absorption
metabolic disorder? C. Carbohydrate hydrolysis
A. Lack of linoleic acid, vitamins A, D, E, K D. Secretion of hydrochloric acid
B. Lack of palmitic acid E. Salivation inhibition
C. Lack of vitamins PP, H
D. Low caloric value of diet 8. After eating fatty foods, the patient has
E. Lack of oleic acid nausea and heartburn, steatorrhea occurs. The
reason for such a state can be:
3. In humans, the absorption of products of the A. Lack of bile
hydrolysis of fats is impaired. The reason for B. Increased lipase secretion
this may be a deficiency in the cavity of the C. Violation of trypsin synthesis
small intestine: D. Lack of amylase
A. Bile acids E. Impaired phospholipase synthesis
B. Lipolytic enzymes
C. Bile pigments 9. A coprological study found that the feces are
D. Sodium ions discolored, there are found drops of neutral fat.
E. Fat-soluble vitamins The most likely cause of this is a violation:
A. Flow of bile into the intestine
4. A patient has normally colored stool B. pH of gastric juice
including a large amount of free fatty acids. The C. Secretions of pancreatic juice
reason for this is a disturbance of the following D. Secretion of intestinal juice
process: E. Absorption processes in the intestine
A. Fat absorption
B. Fat hydrolysis 10. The respiratory ratio of the patient is 0.7.
C. Biliary excretion This indicates that in human cells prevails:
D. Bilification A. Oxidation of fats
E. Lipase secretion B. Oxidation of carbohydrates
C. Oxidation of proteins
5. A 65-year-old patient suffers from D. Mixed oxidation of fats and carbohydrates
cholelithiasis. Recently, there were signs of E. Mixed oxidation of fats and proteins
achilic syndrome due to obturation of the biliary
tract. Which components of food will be 11. A sportsman was recommended to take a
mastered the most? preparation with carnitine in order to improve
A. Fats his achievements. What process is activated by
B.Carbohydrates carnitine to the most extent?
C. Proteins A. Transporting of fatty acids to the
D. Nucleic acids mitochondria
B. Lipid synthesis B. Pancreatic amylase
C. Synthesis of ketone bodies C. Pancreatic proteases
D. Synthesis of steroid hormones D. Bile lipase
E. Tissue respiration E. Gastric proteases

12. One of the factors that cause obesity is the 17. Obesity is a common disease. The aim of its
inhibition of fatty acids oxidation due to: treatment is to lower content of neutral fats in
A. Low level of carnitine the body. What hormone-sensitive enzyme is
B. Impaired phospholipid synthesis the most important for intracellular lipolysis?
C. Excessive consumption of fatty foods A. Triacylglycerol lipase
D. Choline deficiency B. Protein kinase
E. Lack of carbohydrates in the diet C. Adenylate cyclase
D. Diacylglycerol lipase
13. A patient with a high degree of obesity in E. Monoacylglycerol lipase
the quality of a dietary supplement is
recommended carnitine to improve the 18. Disorderthe splitting of the lipids in the
"burning" of fat. What is the direct involvement small intestine is due to a disorder of lipase
of carnitine in the process of oxidation of fats? activity. Which of the following factors
A. Transport of fatty acids from cytosol to activates the lipase?
mitochondria A. Bile acids
B. Activation of fatty acids B. Hydrochloric acid
C. Involved in one of the reactions of beta- C. Enterokinase
oxidation of fatty acids D. Pepsin
D. Transport of fatty acids from fat depots to E. Solts of Na+
tissues
E. Activation of intracellular lipolysis 19. A man has 35 years of pheochromocytoma.
In the blood there is an elevated level of
14. A 1-year-old child entered the clinic with adrenaline and norepinephrine, the
signs of damage to the muscles of the concentration of free fatty acids increased in 11-
extremities and trunk. After the examination fold. Indicate which activation of any enzyme
revealed a deficit of carnitine in the muscles. under the influence of adrenaline increases
The biochemical basis of this pathology is the lipolysis?
disruption of the process: A. TAG-lipase.
A. Transport of fatty acids in mitochondria B. Lipoprotein lipase.
B. Regulation of Ca2+level in mitochondria C. Phospholipases A2.
C. Substrate phosphorylation D. Phospholipase C.
D. Utilization of lactic acid E. Cholesterol esterase
E. Oxidizing phosphorylation
20. The patient, who was in the clinic about
15. Emotional stress causes activation of pneumonia complicated by pleurisy, was
hormone-sensitive triglyceride lipase in the included in the complex therapy of
adipocytes. What secondary mediator takes part prednisolone. The anti-inflammatory action of
in this process? this synthetic glucocorticoid is associated with
A. Cyclic adenosine monophosphate blocking the release of arachidonic acid by
B. Cyclic guanosine monophosphate inhibition:
C. Adenosine monophosphate A. A2 phospholipases
D. Diacylglycerol B. Lipoxygenase
E. Ions of Ca2+ C. Phospholipases C.
D. Peroxidase
16. A significant amount of undigested fat was E. Cyclooxygenase
found in the patient's coprogram.The violation
of which enzymes secretion most likely occurs 21. In the laboratory examination of blood of a
in this person? person who was bitten by a change, hemolysis
A. Pancreatic lipase of erythrocytes, hemoglobinuria was revealed.
The action of a mixed poison is due to the C. Interferons
presence of an enzyme in it: D. Catecholamins
A. Phospholipase A2 E. Corticosteroids
B. Phospholipase D
C. Phospholipase S. 27. The drug "Geptral", which is used in liver
D. Phospholipase A1 diseases contains S-adenosylmethionine. This
E. Sphingomyelinase active amino acid is involved in the synthesis
of:
22. Patients with pain syndrome in the joints A. Phospholipids
permanently appoint aspirin. Which of the B. Fatty acids
enzymes that he enfolds? C. Triacylglycerols
A. Phospholipase A2 D. Cholesterol
B. Cyclooxygenase E. Heme
C. Phospholipase D
D. Lipoxygenase 28. It is known that a part of carbon dioxide is
E.PhospholipaseC used in the body in the biosynthesis of fatty
acids, urea, gluconeogenesis, and the like. What
23. Thromboxanes belong to the bioregulators kind of vitamin forms a CO2-transporting form
of cellular functions of lipid nature. The source for these reactions?
for the synthesis of these compounds is: A. Biotine
A. Arachidonic acid B. Timine
B. Stearic acid C. Riboflavin
C. Palmitic acid D. Nicotinamide
D. Phosphatidic acid E. Retinol
E. Palmiotoleic acid
29. A young man of 25 years consumes an
24. Methyl groups (-CH3) are used in the body excessive amount of carbohydrates (600 g per
for synthesis such important compounds as day) that exceeds its energy needs. What
creatine, choline, adrenaline, and others. The process will be activated in the human organism
source of these groups is one of the essential in this case?
amino acids, namely: A. Lypogenesis
A. Methionine B. Glycolysis
B. Valine C. Lypolis
C. Leicine D. Gluconeogenesis
D. Isoleucine E. Oxidation of fatty acids
E. Tryptophan
30. Disorders of the processes of mielinisation
25. At deficiency of biotin, there is a disorder of neuronal fibers' leads to neurological disorders
higher fatty acids synthesis. Which of these and mental retardation. Such symptoms are
metabolites can be affected by this? characteric for hereditary and acquired
A. Malonyl-CoA metabolic disturbances:
B. Succinyl-CoA A. Sphingolipids
C. Pyruvate B. Neutral fat
D. Alanine C. Higher fatty acids
E. Serotonin D. Cholesterol
E. Phosphatidic acid
26. Deficiency in the body of linoleic and
linoleic acids leads to skin damage, hair loss, 31. The mother asked the doctor about the bad
delayed healing of wounds, thrombocytopenia, health of the child - the lack of appetite, poor
and reduced probability of infection. Disorders sleep, irritability. At the biochemical
the synthesis of which substances most likely examination in the blood, the absence of the
causes these symptoms? enzyme glucocerebrosidase was detected. For
A. Eicosanoids which pathology is this characteristic?
B. Interleukins A. Gaucher’s disease
B. Tay-Sachs disease A. Choline
C. Niemann-Pika's disease B. DOPA
D. Girke's disease C. Cholesterol
E. Pompe’s disease D. Acetoacetate
E. Linolic acid
32. A patient is diagnosed with
glucocerebrosidelipidosis (Gaucher’s disease) 37. Synthesis of phospholipids is
that manifests as splenomegaly, liver disorderedunder the liver fat infiltration.
enlargement, affected bone tissue, and Indicate which of the following substances can
neuropathies. What enzyme of complex lipid enhance the process of methylation during
catabolism is deficient, thus causing this phospholipids synthesis?
disease? A. Methionine
A. Sphingomyelinase B. Ascorbic acid
B. β-galactosidase C. Glucose
C. Glucocerebrosidase D. Glycerin
D. Hexosaminidase E. Citrate
E. Hyaluronidase
38. Examination of cell culture gotfrom a
33. At utilization of arachidonic acid by patient with lysosomal pathology revealed
cyclooxygenase pathways, biologically active accumulation of great quantity of lipids in the
substances are formed. Indicate them: lysosomes. What of the following diseases is
A. Prostaglandins this disturbance typical for?
B. Thyroxine A. Tay-Sachs disease
C. Biogenic amines B. Gout
D. Somatomedin C. Phenylketonuria
E. Insulin-like growth factors D.Wilson disease
E. Galactosemia
34. The patient appointed a lipotropic drug a
donor of methyl groups, to prevent a fatty liver 39. An experimental animal has been
distrophia. This is sensible: givenexcessive amount of carbon-
A. S-Adenosylmethionine labeledglucose for a week. What compound
B. Cholesterol canthe label be found in?
C. Bilirubin A. Palmitic acid
D. Valine B. Methionine
E. Glucose C. Vitamin A
D. Choline
35. Steatosis occurs as a result of the E. Arachidonic acid
accumulation of triacylglycerols in hepatocytes.
One of the mechanisms of development of this 40. A 6 years old child was delivered to
disease is reduction of utilization neutral fat ahospital. Examination revealed that thechild
LDL. What lipotropic substances prevent the couldn’t fix his eyes, didn’t keephis eyes on
development of steatosis? toys, eye ground had thecherry-red spot sign.
A. Methionine, BC, B12 Laboratory analyses showed that brain, liver and
B. Arginine, B2, B3 spleen had highrate of ganglioside glycometide.
C. Alanine, B1, PP What congenital disease is the child ill with?
D. Valine, B3, B2 A. Tay-Sachs disease
E. Isoleucine, B1, B2 B.Wilson’s syndrome
C. Turner’s syndrome
36. In an experimental animal, receiving non- D. Niemann-Pick disease
protein diet, fatty infiltration of the liver has E. MacArdle disease
developed due to the deficiency of the
methylating agents. The formation of which
metabolite is disturbed in the experimental
animal?
41. A dry cleaner’s worker has been found to 46. The doctor gave the woman a
have hepatic steatosis. This pathology can be recommendation to continuethe low-calorie diet.
caused by the disruption of synthesis of the She decided to get the same amount of calories,
following substance: but to replace carbohydrates on fats. Which of
A. Phosphatidylcholine the following lipoprotein fractions will bee
B. Tristearin levated as a result of this diet?
C. Urea A. Chylomicrons
D. Phosphatidic acid B. VLDL
E. Cholic acid C. LDL
D. LPID
42. The key reaction of fatty acid synthesis is E. HDL
production of malonyl-CoA. What metabolite is
the source of malonyl-CoA synthesis? 47. For the prevention of atherosclerosis,
A. Acetyl-CoA coronary heart disease, cerebrovascular
B. Succinyl-CoA accident, the consumption of high-fat
C. Acyl-CoA polyunsaturated fatty acids is recommended.
D. Malonate One of the following fatty acids is:
E. Citrate A. Linolic
B. Oleic
43. A 2-year-old child presents with acute C. Lauric
psychomotor retardation, vision and hearing D. Palmitooleic
impairment, sharp enlargement of the liver and E. Stearic
spleen. The child is diagnosed with hereditary
Niemann-Pick disease. What genetic defect is 48. Hereditary hyperlipoproteinemia of type I is
the cause of this disease? duetoin sufficiency of the lipoproteinlipase.
A. Sphingomyelinase deficiency Increasing of which transport forms of lipid in
B. Glucose 6-phosphatase deficiency plasma is characteric even on an empty
C. Amylo-1,6-glucosidase deficiency stomach?
D. Acid lipase deficiency A. Lipoproteinsoflowdensity
E. Xanthine oxidase deficiency B. Chylomicrons
C. Lipoproteinsofverylowdensity
44. A 3-year-oldgirlwithmentalretardation D. High-densitylipoproteins
hasbeendiagnosedwithsphingomyelin lipidosis E. Modifiedlipoproteins
(Niemann-Pickdisease). Inthis
conditionsynthesisofthefollowing substance 49. After 4 hours taking of fatty food is found
isdisrupted: that patient blood plasma is muddy. The most
A. Sphingomyelinase probable reason for this state is increasing the
B. Glycosyltransferase concentration in plasma:
C. Sphingosine A. Chilomicrons
D. Ceramides B. VLDL
E. Gangliosides C. LDL
D. Cholesterol
45. Modern antiatherosclerotic drugs are used E. Phospholipids
for the preventions and treatment of
atherosclerosis. Such drugs as gemfibrozil and 50. In the diseased child is established
fenfibrate is inhibiting cholesterol biosynthesis hyperlipoproteinemia, inherited. Genetic defect
by enzyme digestion: of what synthesis enzyme causes this
A. β-HMG-reductase phenomenon?
B. Hexokinase A. Lipoproteinlipase
C. Glucose-6-phosphatase B. Glycosidase
D. Acyltransferase C. Proteinase
E. Acyl-CoA-cholesterolacyltransferase D. Hemesynthetase
E. Fenylanininehydroxylase
51. Rabbits were fed with cholesterol
supplementation. After 5 months, 56. During examination of a teenagerwith
atherosclerotic changes were detected in the xanthomatosis the family history
aorta. Name the main cause of atherogenesis in ofhypercholesterolemia is revealed.
this case: Whattransportable lipids are increased
A. Exogenous hypercholesterinemia inconcentration in case of such a disease?
B. Overeating A. Low-density lipoproteins
C. Hypodynamia B. Chylomicrons
D. Endogenous hypercholesterinemia C. Very low-density lipoproteins
E. Stress D. High-density lipoproteins
E. Intermediate-density lipoproteins
52. In the examination, was established the
female has insufficient activity of 57. Inthebloodofpatientswithdiabetesmellitus,
lipoproteinlipase which hydrolizes the increasesthecontentoffreefattyacids (HFAs)
cholymocrons triglycerides on the surface of the areobserved. Thereasonforthismaybe:
endothelium adipose tissue. What biochemical A.
disorders should wait? Increasingtheactivityofadipocytestriglyceridelip
A. Hyperlipoproteinemia type I ase
B. Hyperlipoproteinemia II A type B. Accumulationinthecytosolof palmitic-CoA
C. Hyperlipoproteinemia of type III C. Activationofketone bodies utilization
D. Hyperlipoproteinemia of type IV D. Activationofthesynthesisofapolipoproteins
E. Hyperlipoproteinemia II B type A-1, А-2, А-4
E. Reducedplasma phosphatidylcholine-
53. A 58-year-old patient suffers from the cholesteine acyltransferaseactivity
cerebral atherosclerosis. Examination revealed
hyperlipoidemia. What class of lipoproteins will 58. In the case of diabetes mellitus and
most probably show increase in concentration in starvation in the blood, the amount of acetone
this patient’s blood serum? bodies used in the quality of energy material
A. Low-density lipoproteins increases. Name the substance from which they
B. High-density lipoproteins are synthesized:
C. Fatty acid complexes with albumins A. Acetyl-CoA
D. Chylomicrons B. Succinyl-CoA
E. Cholesterol C. Citrate
D. Malate
54. A patient underwent a course oftreatment E. Ketoglutarate
for atherosclerosis. Laboratorytests revealed an
increase in the antiatherogeniclipoprotein 59. A 67-year-old man consumes eggs, pork fat,
fraction in theblood plasma. The treatment butter, milk and meat. Blood test results:
efficacy isconfirmed by the increase in: cholesterol - 12,3 mmol/l, total lipids - 8,2 g/l,
A. HDL increased low-density lipoprotein fraction
B. VLDL (LDL). What type of hyperlipoproteinemia is
C. IDL observed in the patient?
D. LDL A. Hyperlipoproteinemia type IIa
E. Chylomicrons B. Hyperlipoproteinemia type I
C. Hyperlipoproteinemia type IIb
55. During the examination of the patient, the D. Hyperlipoproteinemia type IV
increase in blood serum in low density E. Cholesterol, hyperlipoproteinemia
lipoproteins was detected. What disease can be
foreseen in this patient? 60. Cholesterol content in blood serum of a 12-
A. Atherosclerosis year-old boy is 25 mmol/l. Anamnesis states
B. Kidney damage hereditary familial hypercholesterolemia caused
C. Acute pancreatitis by synthesis disruption of receptor-related
D. Gastritis proteins for:
E. Lung inflammation A. Low-density lipoproteins
B. High-density lipoproteins 61. Increased HDL levels decrease the risk of
C. Chylomicrons atherosclerosis. What is the mechanism of HDL
D. Very low-density lipoproteins antiatherogenic action?
E. Middle-density lipoproteins A. They remove cholesterol from tissues
B. They supply tissues with cholesterol
C. They are involved in the breakdown of
cholesterol
D. They activate the conversion of cholesterol
to bile acids
E. They promote absorption of cholesterol in the
intestine
Amino- acid metabolism and its regulation
1. A 36-year-old female patientwho has been A. Gastrin
limiting the number offoodstuffs in her diet for B. Secretin
3 monthspresents with a decrease in body C. Cholecystokinin
weight, deterioration of physical andmental D. Somatostatin
health, face edemata. Thesechanges may be E. Neurotensin
caused by the deficiencyof the following
nutrients: 6. Young man, due to the irritation of the carotic
A. Proteins plexus by the inflammatory process (solarium),
B. Vitamins the functional activity of the glands of the
C. Fats stomach is increased, which is expressed, in
D. Carbohydrates particular, in the increase of the product of
E. Micronutrients chloride acid. Which of the following
substances causes hypochlorhydria in this case?
2. In the daily diet of an adult healthy person A. Gastrin
should be fats, proteins, carbohydrates, B. Gastrointing peptide
vitamins, mineral salts and water. Specify the C. Urologist
amount of protein per day, which provides D. Glukagon
normal functioning of the body. E. Kalikrein
A. 100-120 g
B. 50-60 g 7. The pyloric part of the stomach has been
C.10-20 g removed. Reducing the secretion of which
D. 70-80 g hormone should be seen first of all?
E. 40-50 g A. Gastrin
B. Gistamin
3. A 30 years old patient with acute C. Secretin
inflammation of the liver (pancreatitis), D. Kholetsistokinin
violations of cavity digestion of proteins were E. Stinging digestive peptide
detected. This may be due to insufficient
synthesis and secretion of the gland of such an 8. In order to determine the maximum secretion
enzyme: of hydrochloric acid in gastric juice, a solution
A. Trypsin of histaminewas given to 42-year-old patient.
B. Pepsin This led to increasing of the secretion of the
C. Lipase parietal gland of such component of juice as:
D. Dipeptidase A. Bicarbonates
E. Amilaza B. Trypsinogen
C. Lipase
4. The patient was brought to the hospital with D. Amilaza
burns of skin. To clean the wounds from dead E. Slime
tissues and mucus, the doctor prescribed an
enzyme preparation for local treatment. Name 9. In the acute experiment, a weak solution of
it: hydrochloric acid was introduced into the cavity
A. Tripsin of the duodenum of animal. Will it increase the
B. Pansinorm secrecy of the gastrointestinal hormone?
C. Asparaginase A. Sekretin
D. Pepsin B. Gastrin
E. Streptokinase C. Motilin
D. Neurotensin
5. A patient with gastric juice hypersecretion E. Gistamin
has been recommended to exclude from the
dietric broths and vegetable in fused water. A 10. Transformation of proteins in the stomach is
doctor recommended it, because these food the initial stage of protein digestion in the
products stimulate production of the following human digestive tract. What are the enzymes
hormone: involved in digestive proteins in the stomach:
A. Pepsin and gastriksin noradrenaline and serotonine in the
B. Trypsin and Cathepsin mitochondrions of cerebral neurons. What
C. Chymotropins and lysozyme enzyme participates in this process?
D. Enteropeptidase and elastase A. Monoamine oxidase
E. Carboxypeptidase and aminopeptidase B. Transaminase
C. Decarboxylase
11. A newborn child suffers from milkcurdling D. Peptidase
in stomach, this means thatsoluble milk proteins E. Lyase
(caseins) transformto insoluble proteins
(paracaseins) bymeans of calcium ions and a 16. It is known that in the metabolism of
certainenzyme. What enzyme takes part in catecholamine mediators a special role belongs
thisprocess? to the enzyme monoamine oxidase (MAO).
A. Renin How does this enzyme inactivate mediators
B. Pepsin (noradrenaline, adrenaline, dopamine)?
C. Gastrin A. The oxidative deamination
D. Secretin B. Addition of an amino group
E. Lipase C. Removal of the metal group
D. Carboxylation
12. A patient has been prescribed E. Hydrolisis
pyridoxalphosphate. What processes are
correctedwith this drug? 17. During the catabolism of histidine, a
A. Transamination and decarboxylation biogenic amine is formed, which has a powerful
ofaminoacids vasodilating effect. Name it.
B. Oxidative decarboxylation of keto acids A. Histamine.
C. Deaminization of amino acids B. DOPA.
D. Synthesis of purine and pyrimidine bases C. Dopamine.
E. Protein synthesis D. Serotonin.
E. Noradrenalin.
13. To the hospital delivered 7 years-old
childwith allergic shock, which developed after 18. When decarboxylating glutamate in the
she was stomped by bee. In the blood, the central nervous system, a mediator of inhibition
concentration of histamine is increased.The is formed. Name it.
result of which reaction formed this amine? A. GAMK.
A. Decarboxylation B. Glutathione.
B. Hydroxylation C. Histamine.
C. Degradation D. Serotonin.
D. Deamination E. Asparagine.
E. Restoration
19. Biogenic amines are used in psychiatry for
14. The allergen was administered to the patient, the treatment of a number of diseases of the
tested for hypersensitivity, under the skin,and central nervous system. Specify the drug of this
than saw reddening, edema, and even after the group, which is a mediator of inhibition:
action of histamine. As a result of which the A. γ-aminobutyric acid.
amino acid conversion of histidine forms this B. Dopamine.
biogenic amine? C. Histamine.
A. Decarboxylation D. Serotonin.
B. Methylation E. Taurin.
C. Phosphorylation
D. Isolation 20. A child 9 months old eats artificial sweeties
E. Depositing that are not balanced by the vitamin B6. A child
have a pelagic dermatitis, convulsions, anemia.
15. Pharmacological effects of The development of a cramps may be related
antidepressantsare connected with inhibition of with a violation of forming of:
an enzyme catalyzing biogenic amines A. GAMK
B. Gistamin E. Cysteine, glucine, proline
C. Serotonin
D. DOPA 26. There are several ways to dispose of
E. Dopamin ammonia in the body, but for specific organs are
specific. What is the way to neutralize this toxic
21. What neurotransmitter in the brain tissue substance is typical for brain cells?
can be synthesized from the product of A. Formation of glutamine.
reamination of alpha-ketoglutaric acid? B. Formation NH 4 +.
A. GAMK C. Formation of asparagine.
B. Tryptamine D. Formation of Creatine.
C. Dopamine E. Formation of urea.
D. Serotonin
E. Noradrenalin 27. Ammonia is a very poisonous substance,
especially for the nervous system. What
22. In a patient with a diagnosis of malignant substance is particularly active in ammonia
carcinoma, the severely increased amount of disinfection in brain tissues?
serotonin in the blood.Choose the amino acid A. Glutamic acid.
from which this compound is formed: B. Lizin.
A. Tryptophan C. Proline.
B. Allanin D. Hystidine.
C. Leicin E. Alanin.
D. Treonin
E. Methionin 28. It is known that accumulation of ammonia is
the main cause of cerebral coma in hepatic
23. Hospitalized patient with diagnosis of insufficiency. What is a free amino acid plays a
intestinal carcinoids. Analizes showed an primary role in the use of this toxic substance in
increasing secretion of serotonin. It is known the brain?
that this substance is formed from an amino acid A. Glutamic acid.
tryptophan. What biomechanical mechanism is B. Alanin.
the basis of this process? C. Histidine.
A. Decarboxylation D. Tryptophan.
B. Deamination E. Cysteine.
C. Microsomal oxidation
D. Transamination 29. After an injury to the brain, the patient has
E. Formation of paired compounds an increased ammonia formation. What amino
acid is involved in removing ammonia from this
24. Methyl groups (-CH3) are used in the body tissue?
to synthesize such important compounds as A. Glutaminov.
creatine, choline, adrenaline, and others. The B. Valin.
source of these groups is one of the essential C. Lizin.
amino acids, namely: D. Tyrosin
A. Methionin E. Tryptophan
B. Valin
C. Leicin 30. The patient entered the clinic with a
D. Isoleucine concussion of the brain. On the background of
E. Tryptophan neurological symptoms, the concentration of
ammonia in the blood increases. What substance
25. Cationic glycoproteins are the main should be used to neutralize this substance in
components of salivation of parotid glands. the brain tissue?
What amino acids cause their positive charge? A. Glutamic acid
A. Lizin, arginine, histystine B. Gistamin
B. Aspartate, glutamate, glycine C. Ascorbic acid
C. Aspartate, arginine, glutamate D. Serotonin
D. Glutamate, valine, leucine E. Nicotinic acid
31. After the operation on the intestines, the 35. In a 2 year old child, there is a Intellectual
patient appeared symptoms of poisoning with disability, intolerance of protein foods, severe
ammonia according to the type of stomach hyperammonia as the result of reduced level of
coma. What mechanism of action of ammonia urea in the plasma of blood, which is connected
on the energy supply of the central nervous with birth deficiency of such mythohondrial
system? enzyme as:
A. Braking (slowdown) CTK due to the binding A. Carbamoyl phosphate synthase
of alpha ketoglutarate B. Citrate synthase
B. Braking of glycolysis C. Succinate dehydrogenase
C. Braking of beta-oxidation of fatty acids D. Malatedehydrogenase
D. Inactivation of the enzymes of respiratory E. Monoamine oxidase
chain
E. Dissociation of oxidative phosphorylation 36. In the biosynthesis of urea in the liver, the
formation of ornithine and urea is stimulated.
32. In a newborn child there is a decrease in What amino acid is the intermediate product of
intensity of sucking, frequent vomiting, this synthesis?
hypotension. In urine and blood, the A. Аrginine
concentration of citrulline is significantly B. Leicinc.
increased. What metabolic process is affected? C. Сitrate.
A. Ornithine cycle D. Valin.
В. Glycolysis E. Tryptophan.
C. Core's cycle
D. СТК 37. In the urine of the newborn, determined by
E. Gluconeogenesis citrulline and high levels of ammonia. Indicate
which substance is most likely to be inflicted on
33. In a 3 year old child, which outlived the a baby?
severe viral infection, the doctor has indicated A. Urea.
re-vomiting, unconsciousness, convulsions. The B. Bilirubin.
hyperammonemia was detected during the C. Creatine.
analysis. What can be connected with changing D. Creatinine.
of the biochemical indicators of blood in this E. Uric acid.
child?
A. Violation of ammonia neutralization in the 38. In a patient with reduced singular function
ornithine cycle of the kidneys, an unpleasant odor is noted from
B. Activation of the processes of the mouth. Increased excretion of the salivary
decarboxylation of amino acids glands of what substance is the cause of this?
C. Violation of neutralizing of biogenic amines A. Urea
D. Strengthening of decay of proteins in the B. α-amylase.
intestines C. Mutsina.
E. Depressing the activity of transamination D. Lysozyme.
enzymes E. Fosfatasi.

34. The main part of nitrogen is taking out of 39. A patient in an unconscious state is
organism as element of urea. Reducing the delivered by an ambulance brigade to the
activity of what liver enzyme leads to inhibition hospital. Objectively: reflexes are absent,
of the synthesis of urea and increasing the seizures occur periodically, breathing is
accumulation of ammonia in blood and tissues? unequal. After a laboratory examination, the
A. Carbamoyl phosphate synthase liver was diagnosed with a coma. The
B. Aspartate aminotransferase accumulation of which metabolites in the blood
C. Urease is essential for the appearance of central nervous
D. Amilaza system disorders?
E. Pepsin A. Ammonia.
B. Bilirubin.
C. Gistamin.
D. Glutamin. D. Methionine
E. Urea. E. Glutamine

40. In a boy of 4 years after suffering from 45. A 46-year-old patient suffers from
severe viral hepatitis, vomiting, episodes of progressing Duchenne muscular dystrophy.
nephropathy, seizures are observed. In the blood Changes of the following blood enzyme level
there is hyperammonia. Violation of which of may diagnose this condition:
the biochemical process in the liver caused such A. Creatine phosphokinase
a condition of the patient? B. Lactate dehydrogenase
A. Disposal of ammonia. C. Pyruvate dehydrogenase
B. Decarboxylation of amino acids. D. Glutamate dehydrogenase
C. Disposal of Biogenic Amines. E. Adenylate kinase
D. Synthesis of alphabets.
E. Gluconeogenesis. 46. It was found that the cause of dizziness,
memory impairment, and periodic judgment in
41. Examination of a 2 year-old child with renal the patient is a violation of the decarboxylation
insufficiency revealed hyperoxaluria that caused of glutamic acid. What is the product of this
depositing of calcium oxalate stones in the reaction:
kidneys. The reason for this condition is a A. GAMK.
disturbance of metabolism of the following B. PAUL
aminoacid: C. TDP
A. Glycine D. ATP.
B. Lysine E. TGFK.
C. Methionine
D. Arginine 47. A 24 year-old patient that suffers from
E. Histidine epilepsy was administered glutamic acid. What
product of its decarboxylation caused the
42. Antioxidant enzymes inhibit lipid therapeutic effect?
peroxidation processes. Decrease of glutathione A. GABA
peroxidase activity is caused by deficiency of B. Histamine monooxygenase
the following microelement: C. Serotonin
A. Selenium D. Dopamine
B. Molybdenum E. Taurine
C. Cobalt
D. Manganese 48. A baby has epileptical convulsions that are
E. Copper caused by vitamin B6 defficiency. It is due to
the decrease of GABA in the neural tissue.
43. Blood vessels endothelium has a What enzyme activity is decrased in this case?
significantly high metabolic activity as it A. Glutamate decarboxylase
synthesizes a variety of vasoactive substances. B. Alanine aminotransferase
What powerful vasodilatator is synthesized from C. Glutamate degydrogenase
L-arginine? D. Pyridoxal kinase
A. Nitric oxide E. Glutamate synthetase
B. Histamine
C. Bradykinin 49. In a patient with a sharp increase in
D. Acetylcholine serotonin level, the diagnosis of "malignant
E. Epinephrine carcinoid" is observed in the blood. Choose the
amino acid from which the given biogenic
44. Nitric oxide is a powerful vasodilatator and amine can be formed.
blood pressure regulator. In human organism it A. Tryptophan.
is synthesized from: B. Alanine.
A. Arginine C. Leicin.
B. Proline D. Methionine
C. Lysine E. Threonine.
50. А patient suffering from pellagra was 55. A patient has the initial stage of gingivitis.
examined. It was found that his diet included Dilatation of the microcirculatory vessels
almost no meat and consisted mostly of corn. caused gum hyperaemia. What substance
His condition was caused by the deficiency of produced by mast cells caused the changes
the following substance in corn: A. Tryptophan mentioned above?
B. Tyrosine A. Histamine
C. Proline B. Epinephrine
D. Alanine C. Substance P
E. Histidine D. Endorphins
E. Actylcholine
51. A patient that is suffering from congenital
Hartnup disease has pellagra-like dermatosis 56. A cook burned his hand incautiously. What
and mental retardation due to the deficiency of substance has caused the reddening, swelling
nicotinic acid. The reason for this condition is and pain of the damaged area?
the violation of the following process: A. Histamine
A. Tryptophan absorption and reabsorption in B. Thiamine
the kidneys C. Glutamine
B. Phenylalanine transamination D. Lysine
C. Tryptophan decarboxylation E. Galactosamine
D. Methionine absorption and reabsorption in
the kidneys 57. L-DOPA is the precursor chemical of
E. Cysteine absorption and reabsorption dopamine. It is prescribed for Parkinson’s
disease treatment. What aminoacid is used for
52. A woman was bitten by a wasp. Shortly its biosynthesis?
after the bite a painful and itching blister was A. Tyrosine
formed. After some more time hives and B. Alanine
dyspnea developed. What factor was the reason C. Cysteine
for it? D. Histidine
A. Histamine E. Tryptophan
B. Hagemann factor
C. Lysosomal enzymes 58. A patient suffering from Parkinson’s disease
D. Norepinephrine has tremor of both hands. This symptom is
E. Epinephrine caused by the deficiency of the following
neurotransmitter in the striatopallidal structures:
53. Shortly after the dental treatment the patient A. Dopamine
has developed red itching blemishes on the B. GABA
facial skin and oral mucosa. Urticaria was C. Substance P
diagnosed. Vasodilatation and itching were D. Epinephrine
caused by the following bioactive substance: E. Serotonin
A. Histamine
B. Prostaglandin Е2 59. An 84 year-old patient suffers from the
C. Leucotriene В4 Parkinson’s disease. One of its pathogenetic
D. Interleukin-1 causes is the defficiency of a neurotransmitter in
E. Bradykinin certain neural structures. What neurotransmitter
is it?
54. The patient has an allergic reaction, which is A. Dopamine
accompanied by itching, swelling and reddening B. Epinephrine
of the skin. Concentration of which biogenic C. Norepinephrine
amine has increased in tissues? D. Histamine
A. Histamine. E. Acetylcholine
B. Gamma-aminobutyric acid.
C. Dopamine. 60. At the examination in the clinic, the man
D. Serotonin. was diagnosed with acute radiation sickness.
E. Triptamin. Laboratory has established a sharp decrease in
serotonin in platelets. The metabolism of a D. Gout.
substance is a possible cause of this condition? E. Fenilketonuria.
A. 5-oxytriptophane.
B. Tyrosine. 65. What is the most probable diagnosis in a
C. Histidine. child of infancy, in which there is darkening of
D. Phenylalanine. sclera, mucous membranes, auricles, separated
E. Serine. urine darkens in the air, in the blood and urine
found homogensic acid?
61. A 20-year-old woman came to the doctor A. Alkaptonuria.
with complaints of general weight loss, loss of B. Albinism.
appetite, weakness, skin discoloration C. Hemolytic anemia.
resembling bronze tan. In addtition to D. Porphyria.
hyperpigmentation, examination in the hospital E. Cystinuria.
revealed bilateral adrenal tuberculosis. What
substance leads to skin hyperpigmentation, 66. A baby has coloured sclera and mucouses
when accumulated excessively? and her urine is darkening when exposed to air.
A. Melanin Homogentisic acid was found in urine and blood
B. Bilirubin samples. What is the reason for this condition?
C. Hemozoin A Alcaptonuria
D. Lipofuscin B. Albinism
E. Adrenochrome C. Galactosemia
D. Cystinuria
62. A person consumed methionine-poor E. Histidinemia
products for a long period of time. As a result
neural and endocrine disorders have developed. 67. Dark stains on baby’s diapers indicate
They may be caused by the disturbance of the homogentisic acid synthesis. What substance’s
following substance synthesis: metabolism is disturbed?
A. Adrenaline A. Tyrosine
B. Pyruvate B. Galactose
C. Thyronine C. Methionine
D. Fatty acids D. Cholesterol
E. Glucagon E. Tryptophan

63. A 12-year-old boy in the urine revealed high 68. Excretion of homogentisic acid with the
levels of all amino acids in the aliphatic row. In urine is the reason for the condition called
this case, the highest excretion of cysteine and alcaptonuria. Disturbance of which amino acid
cysteine was noted. In addition, ultrasound of metabolism causes this condition?
the kidneys showed the presence of stones in A. Tyrosine
them. Choose a possible pathology. B. Phenylalanine
A. Cystinuria. C. Alanine
B. Alpathururia. D. Methionine
C. Cystitis E. Asparagine
D. Phenylketonuria.
E. Hartnup's disease. 69. A 5-year old child has darkened urine but no
bile enzymes were found in it. The child is
64. Parents of the 3-year-old child drew diagnosed with alcaptonuria. Which enzyme
attention to the fact that the child's urine deficiency is the reason for it?
becomes dark when restrained. Objectively: the A. Homogentisic acid oxydase
body temperature is normal, the skin is pink, B. Phenylalanine hydroxylase
clean, the stove is not elevated. What disease is C. Tyrosinase
accompanied by such manifestations. D. Oxyphenylpyruvate oxydase
A. Alkaptonuria. E. Phenylpyruvate decarboxylase
B. Hemolytic anemia.
C. Itsenko-Cushing syndrome.
70. What kind of treatment is needed in a child 75. Excess of phenylpyruvate and phenylacetate
with phenylketonuria, in whose blood there is was detected in the urine sample of a 6 day-old
an increased amount of phenylpyruvic acid? baby. Which amino acid metabolism is
A. Diet therapy. disturbed?
B. Antibacterial therapy. A. Phenylalanine
C. Vitamin therapy. B. Tryptophan
D. Hormonotherapy. C. Methionine
E. Fermentotherapy. D. Histidine
E. Arginine
71. A 9-year-old child has a mental and physical
lag. In the biochemical analysis of blood, an 76. A 1.5 year-old child is mentally and
increased amount of phenylalanine has been physically retarded, hair and skin are whitened,
detected. Blocking of an enzyme can lead to this the level of catecholamines in the blood is
condition? decreased. After adding few drops of 5%
A. Phenylalanine-4-monooxygenase. solution of ferrum trichloracetate to the urine it
B. Aspartate aminotransferases. changes its colour to olive green. This
C. Glutamintransaminase. symptoms are typical for the following
D. Glutamate-carboxylase. condition:
E. Oxidazes of homogenetic acetic acid. A. Phenylketonuria
B. Alcaptonuria
72. In a sick child, a raised level of C. Tyrosinosis
phenylpyruvate was detected in the urine (in the D. Albinism
norm practically absent). The amount of E. Xanthinuria
phenylalanine in the blood is 350 mg/l (about 15
mg/l). Which disease is characterized by the 77. A child with mental and physical
above symptoms? development retardation was delivered to the
A. Fenilketonuria. hospital. Phenylpyruvate was found in the urine.
B. Albin. Disturbance of which process is the reason for
C. Alpathonuria. this pathology?
D. Gout A. Amino acid metabolism
E. Tyrosinosis. B. Lipid metabolism
C. Carbohydrate metabolism
73. A certain type of congenital pathology is D. Water-salt balance regulation
accompanied with the inhibition of tyrosine E. Calcium and phosphate homeostasis
formation from phenylalanine. Biochemical
indicator of this condition is depositing of the 78. A patient with complaints about intolerance
following acid in the organism: to solar radiation was contacted by a doctor.
A. Phenylpyruvate There is burn skin and visual impairment.
B. Citric acid Preliminary diagnosis of albinism. Disturbance
C. Pyruvate of the exchange of amino acids observed in this
D. Lactic acid patient?
E. Glutamine A. Tyrosine.
B. Proline
74. A 6 month-old baby has mental and physical C. Lizin.
development retardation, seizures, pale D. Alanin.
eczematous skin, light hair, blue eyes. What E. Tryptophan
substance presence in blood and urine can help
to make a diagnosis in this case? 79. Albinoses has increased sensitivity to
A. Phenylpyruvate sunlight therefore they get burns instead of tan.
B. Tryptophan Which amino acid metabolism disturbance is
C. Histidine the reason for the condition?
D. Leucine A. Phenylalanine
E. Valine B. Methionine
C. Tryptophan
D. Glutamine a specific smell of maple syrup. Which enzyme
E. Histidine deficiency is typical for this condition?
A. Amino acid dehydrogenase
80. The patient is 12 years old, he has a general B. Aminotranspherase
weakness, dizziness, fatigability, lag in mental C. Glucose-6-phosphatase
development. In the laboratory examination, D. Phosphofructokinase
high concentrations of valine, isoleucine, E. Phosphofructomutase
leucine in blood and urine were detected. Urine
specific odor. What disease does this happen? 85. A hospital admitted a patient with
A. Disease of maple syrup. complaints about abdominal swelling, diarrhea,
B. Basetic disease. meteorism after consumption of food rich in
C. Hystidominemia. proteins. It is indicative of disturbed protein
D. Tyrosinosis. digestion and their intensified decaying. What
E. Addison's Illness. substance is the product of this process in the
bowels?
81. A 13 year-old boy is complaining about A. Bilirubin
general weakness, dizziness, fatigue. Mental B. Indole
retardation is detected. The examination C. Cadaverine
revealed high concentrations of valine, D. Agmatine
isoleucine, and leucine in blood and urine. The E. Putrescine
urine has a specific smell. What is the most
possible diagnosis? 86. A 60 year-old man suffers from chronic
A. Maple syrup urine disease intestinal obstruction. The processes of protein
B. Addison’s disease decomposition in the large intestine are
C. Tyrosinosis intensified. What is the evidence of these
D. Histidinemia processes?
E. Grave’s disease A. Indicanuria
B. Bilirubinuria
82. The child's urine had a characteristic smell C. Hyperuricosuria
of maple syrup. In the laboratory study, high D. Creatinuria
levels of leucine, valine, isoleucine and their E. Glucosuria
keto-derivatives were found in blood and urine.
What kind of enzyme is inadequate for this 87. A patient has been operated due to acute
disease? abdomen. His urine is brown, concentration of
A. Dehydrogenases of branched-chain amino indican is higher than 93 mmol/day. What does
acids. it indicate?
B. Aminotransferases. A. High intensity of protein decomposition
C. Glucose-6-phosphatase. processes in the intestine
D. Phosphorutokinase. B. Decreased activity of the urea cycle enzymes
E. Phosphofructomatase. C. Acceleration of aromatic amino acids
desamination
83. A baby refuses breastfeeding, it is nervous, D. Disturbance of the kidney absorption
its respiration is arrhythmic and urine has a E. Decrease of ammonia detoxification
specific smell of maple syrup. What congenital
enzymopathy is the reason for this condition? 88. A 43 year-old woman has been operated due
A. Keto acid dehydrogenase to acute abdomen. Her urine is brown and the
B. Glucose-6-phospate dehydrogenase concentration of indican in the blood has
C. Glycerol kinase increased rapidly. What does it indicate?
D. Aspartate aminotranspherase A.High intensity of protein decomposition
E. UDP-glucoronosyltranspherase B. Acceleration of amino acids desamination
C. Supression of glomerular filtration
84. A lab test revealed increased concentration D. Decreased intensity of the urea cycle
of leucine, valine, isoleucine and ketone E. Inhibition of gluconeogenesis
derivatives in the blood and urine. The urine has
89. Indican is a byproduct of certain amino acid A. Tryptophan
decomposition in the large intestine. Its B. Valine
excretion with the urine is used to indicate the C. Glycine
functional condition of the liver. What is this D. Serine
amino acid? E. Cysteine
Ministry of Public Health of Ukraine

O. O. BOGOMOLETS NATIONAL MEDICAL UNIVERSITY

Department of Bioorganic and Biological Chemistry

LIST OF TEST QUESTIONS

for preparation of Content module № 3

" Biochemistry of tissues and physiological


functions "
FOR STUDENTS OF THE 2ST YEAR OF STUDY
OF MEDICAL and STOMATOLOGICAL FACULTIES

Kyiv-2019
1
Content
Fundamentals of Molecular Biology and Genetics ................................... 3

Biochemistry of intercellular communications:

hormones of protein-peptide nature .................................... 14

hormones of steroid and thyroid nature .............................. 22

Functional and clinical biochemistry of organs and tissues:

Biochemistry and pathobiochemistry of blood ......................................... 29

Biochemistry of immune system................................................................ 46

Biochemistry of liver .................................................................................. 51

Pathologic urine components .................................................................... 59

Biochemistry of nervous tissue.................................................................. 65

Biochemistry of muscle tissue ................................................................... 68

Biochemistry of connective tissue ............................................................. 71

Biochemistry of tooth and saliva (for stomat. faculty) ............................. 74

2
Fundamentals of Molecular Biology and Genetics
1. Gastroenterologists were assigned allopurinol, C. Adenine phosphoribosiltransferase
which inhibits the synthesis of uric acid by D. Hypoxanthine phosphoribosiltransferase
inactivating such an enzyme: E. Guanine deaminase
A. Xanthine oxidase
B. Deaminase 6. Allopurinol is a competitor of xanthine
C. Hyaluronidase
oxidase, who was prescribed for urolithiasis
D. Lactate dehydrogenase
E. Transaminase after the examination. The basis for this was the
chemical analysis of kidney stones, which
2. On the basis of laboratory analysis, the consisted mainly of:
patient was diagnosed with gout. Based on A. Urat sodium
laboratory analysis, the patient confirmed the B. Dihydrate calcium oxalate
diagnosis of gout. What was the analysis for the C. Monohydrate calcium oxalate
diagnosis? D. Phosphate calcium
A. Determination of uric acid in the blood and E. Calcium Sulphate
urine
B. Determination of urinary creatinine 7. A 48 year old patient complained about
C. Determination of residual nitrogen in the intense pain, slight swelling and reddening of
blood skin over the joints, temperature rise up to
D. The determination of urea in the blood and 38oC. Blood analysis revealed high
urine concentration of urates. This condition might be
E. Determination of urine ammonia caused by disturbed metabolism of:
A. Purines
3. A patient suffering from gout was prescribed B. Collagen
allopurinol.What pharmacological property of C. Cholesterol
allopurinol provides the rapeutic effect in this D. Pyrimidines
case? E. Carbohydrates
A. Competitive inhibition of xanthine oxidase
B. Acceleration of nitrogen-containing 8. A 46 year old patient applied to a doctor
substances excretion complaining about joint pain that becomes
C. Acceleration of pyrimidine nucleotides stronger the day before weather changes. Blood
catabolism examination revealed strengthened
D. Deceleration of pyrimidine nucleotides concentration of uric acid. The most probable
salvage cause of the disease is the intensified
E. Acceleration of nucleic acids synthesis disintegration of the following substance:
A. Adenosine monophosphate
4. Blood of a 12 year old boy presents low B. Cytidine monophosphate
concentration of uric acid and accumulation of C. Uridine triphosphate
xanthine and hypoxanthine. This child has D. Uridine monophosphate
genetic defect of the following enzyme: E. Thymidine monophosphate
A. Xanthine oxidase
B. Arginase 9. A patient has increased content of uric acid in
C. Urease his blood that is clinically presented by pain
D. Ornithine carbamoyltransferase syndrome as a result of urate deposition in the
E. Glycerylkinase joints. What process does this acid result from?
A. Lysis of purine nucleotides
5. A 42-year-old man suffering from gout has B. Lysis of pyrimidine nucleotides
increased level of urinary acid in blood. C. Heme catabolism
Allopurinol was prescribed to decrease the level D. Proteolysis
of urinary acid. Competitive inhibitor of what E. Reutilization of purine bases
enzyme is allopurinol?
A. Xanthine oxidase 10. A 65 year old man suffering from gout
B. Adenosine deaminase complains of kidney pain. Ultrasound
3
examination revealed renal calculi. The most the formation of tophi, urate calculi in the
probable cause of calculi formation urinary tracts, as well as serious neuro-
is the strengthened concentration of the psychiatric disorders. The cause of this disease
following substance: is the reduced activity of the following enzyme:
A. Uric acid A.Hypoxanthine-guanine
B. Cholesterol phosphoribosyltransferase
C. Bilirubin B. Xanthine oxidase
D. Urea C. Dihydrofolate reductase
E. Cystine D. Thymidylate synthase
E. Karbamoyl phosphate synthetase
11. In the synthesis of purine nucleotides, some
amino acids, indigenous vitamins, phosphoric 16. A 1,7-year-old child with a developmental
ribozymes are involved. Name the most delay and manifestations of self-agression has
important vitamin needed for purine nucleotide the concentration of uric acid in blood at the rate
synthesis de novo. of 1,96 millimole/l. What metabolic disoder is
A. Folic acid this typical for?
B. Pantothenic acid A. Lesch-Nyhan syndrome
C. Nicotinic acid B. Podagra
D. Ryboflavin C. Acquired immunodeficiency syndrome
E. Pyridoxine D. Gierke’s disease
E. Cushing’s basophilism
12. The patient has a hypovitaminosis of folate,
which may lead to a violation of the synthesis: 17. A newborn child gains weight very slowly,
A. Purin and Tymidyl nucleotides his urine contains too much orotic acid that is
B. Purine nucleotides and cholesterol indicative of disturbed synthesis of pyrimidine
C. Tymidyl nucleotides and fatty acids nucleotides. What metabolite should be used in
D. Heme and creatine order to normalize metabolism?
E. Citrate and ketone bodies A. Uridine
B. Adenosine
13. Purine ring biosynthesis occurs in ribose-5- C. Guanosine
phosphate through gradual accumulation of D. Thymidine
nitrogen and carbon atoms and closing of the E. Histidine
rings. The source of ribose phosphate is the
process of: 18. In orоtaciduria the release of Orotic acid is
A. Pentose phosphate cycle many times higher than normal. Synthesis of
B. Glycolysis what substances will be disturbed in this
C. Glyconeogenesis pathology?
D. Gluconeogenesis A. Pyrimidine nucleotides.
E. Glycogenolysis B. Biogenic Amines.
C. Purine nucleotides.
14. A 8 year old boy suffering from the Lesh- D. Urea
Nichan disease.He has increased content of uric E. Uric acid
acid in his blood. Indicate which violation of
the process is the cause of this hereditary 19. Pterin derivatives (aminopterin and
disease? methotrexate) are the inhibitors of dihydrofolate
A. Lysis of purine nucleotides reductase, so that they inhibit the regeneration
B. Synthesis of purine nucleotides of tetrahydrofolic acid from dihydrofolate.
C. Synthesis of pyrimidine nucleotides These drugs inhibit the intermolecular tranfer of
D. Lysis of pyrimidine nucleotides monocarbon groups, thus suppressing the
E. Formation of deoxyribonucleotides synthesis of the following polymer:
A. DNA
15. Children with Lesch-Nyhan syndrome have B. Protein
a severe form of hyperuricemia accompanied by C. Homopolysaccharides

4
D. Gangliosides D. Pyridoxalphosphate
E. Glycosaminoglycans E. Nicotinamidadenindynucleotide

20. Methotrexate (structural analogue of the 25. radiation and chemotherapy. The complex
folic acid which is competitive inhibitor of the of medicinal products included 5-fluoro-
dihydrofolatreductase) is prescribed for deoxyuridine - thymidylate synthetase inhibitor.
treatment of the malignant tumour. On which Synthesis of what substance is blocked by this
level does methotrexate hinder synthesis of the drug?
nucleic acids? A. DNA
A. Mononucleotide synthesis B. i-RNA
B. Replication C. p-RNA
C. Transcription D. t-RNA
D. Reparation E. -
E. Processing
26. According to the model of double DNA
21. Methotrexate (structural analogue of the helix that was suggested by Watson and Creek,
folic acid which is competitive inhibitor of the it was established that one of chains would not
dihydrofolatreductase) is prescribed for be lost during replication and the second chain
treatment of the malignant tumour. On which would be synthesized complementary to the first
level does methotrexate hinder synthesis of the one. What way of replication is it?
nucleic acids? A. Semiconservative
A. Mononucleotide synthesis B. Analogous
B. Replication C. Identical
C. Transcription D. Dispersed
D. Reparation E. Conservative
E. Processing
27. Among organic substances of a cell there is
22. An oncological patient was administered a polymer composed of dozens, hundreds, and
methotrexate. With the lapse of time the target thousands of monomers. This molecule is
cells of the tumour lost sensitivity to this capable of self-reproduction and can be an
preparation. We can observe changes in the information carrier. X-ray structure analysis
gene expression of the following enzyme: shows this molecule to consist of two
A. Dihydrofolate reductase complementary spiral threads. Name this
B. Thiminase compound:
C. Desaminase A. DNA
D. Folate oxidase B. RNA
E. Folate decarboxylase C. Cellulose
D. Carbohydrate
23. Leukoses are treated with antimetabolite E. Hormone
methotrexate. What vitamin is its antagonist?
A. Folic acid 28. Ability to divide is characteristic of
B. Cyanocobalamin procariotic and eukaryotic cells. Procariotic cell
C. Phyllochinone division is different from that of eukaryotic, but
D. Piridoxine there is one molecular process that is the basis
E. Rutin of both types of division. Name this process.
A. DNA replication
24. For the normal of the replication process B. Transcription
necessary Thimidyl nucleotides that are C. Reparation
synthesized by the enzyme Thymidylate D. Translation
synthase are , as coenzyme is used: E. Gene amplification
A. Methylenetetrahydrofolate
B. Carboxybiotin 29. Epithelium regeneration of mucous
C. Thiamindyfosphate membrane of oral cavity (cell reproduction) was

5
accompanied by semiconservative DNA 34. You are studying functioning of a bacteria
replication (selfreproduction). Nucleotides of a operon. The operator gene has been released
new DNA chain are complementary to: from the repressor gene. Immediately after this
A. Maternal chain the following process will start in the cell:
B. Sense codons A. Transcription
C. DNA-polymerase enzyme B. Translation
D. Introns C. Replication
E. RNA-polymerase enzyme D. Processing
E. Repression
30. During cell division DNA replication occurs
after a signal is received from the cytoplasm, 35. It was proved that a molecule of immature
then a certain portion of the DNA helix unwinds mRNA (precursor mRNA) contained more
and splits into two individual strains. What triplets than amino acids found in the
enzyme facilitates this process? synthesized protein. The reason for that is that
A. Helicase translation is normally preceded by:
B. RNA polymerase A. Processing
C. Ligase B. Initiation
D. Restrictase C. Reparation
E. DNA polymerase D. Mutation
E. Replication
31. A group of researchers set an experiment
and obtained anucleatemutant cells. In the first 36. Nowadays about 50 minor bases have been
place they will have disturbed synthesis of the found in the t-RNA structure besides the main
following compounds: four nitrogenous bases. Choose the minor
A. Ribosomal RNA nitrogenous base:
B. Transfer RNA A. Dihydrouracil
C. Lipids B. Uracil
D. Monosaccharides C. Cysteine
E. Polysaccharides D. Adenine
E. Cytosine
32. Nucleolus organizers of human
chromosomes 13-15, 21, 22 include about 200 37. Inside a human cell the informational RNA
gene clusters that synthesize RNA. These containing both exons and introns was delivered
chromosomal regions contain the information to the granular endoplasmic reticulum to the
on the following type of RNA: ribosomes. What process does NOT take place?
A. rRNA A. Processing
B. tRNA B. Replication
C. mRNA C. Transcription
D. snRNA D. Translation
E. tRNA + rRNA E. Prolongation

33. Amino acids join to each other in ribosomes 38. Synthesis of i-RNA passes on the DNA matrix,
of granular endoplasmic reticulum. Knowing taking into account the principle of
the sequence of amino acids and applying complementarity. If the triplets in the DNA are the
genetic code, it is possible to determine the following - ATG-CGT, then the corresponding i-
RNA codons will be:
sequence of nucleoids in:
A. UAC-GCA
A. mRNA B. AUG-CGU
B. Introns C. ATG-CGT
C. Proteins D. UAG-CGU
D. Carbohydrates E. TAG-UGU
E. rRNA
39. General structure of eukaryotic genes is as
follows: exon-intron-exon. Such functional

6
structure of a gene leads to certain specifics of E. Regulator
the transcription process. What sequence will
correspond with precursor mRNA (immature)? 44. A patient with pulmonary tuberculosis is
A. Exon-intron-exon prescribed the most effective antituberculous
B. Exon-exon-intron antibiotic. Name this drug:
C. Exon-exon A. Rifampicin
D. Intron-exon B. Tetracycline
E. Exon-intron C. Streptocide
D. Furasolidone
40. In the nucleus of eukaryote’s cells the first E. Bactrim (Co-trimoxazole)
pro-RNA molecule is synthesized that is
complementary to the exons and introns of the 45. Tuberculosis can be treated by means of
structural gene. But the ribosome receives such combined chemotherapy that includes
i-RNA, which is complementary to exons only. substances with different mechanisms of
This indicates that the nucleus takes place: action.What antituberculous medication inhibits
A. Processing transcription of RNA into DNA in
B. Transcription mycobacteria?
C. Reparation A. Rifampicin
D. Replication B. Isoniazid
E. Reverse transcription C. Streptomycin
D. Ethionamide
41. As a result of intoxication in the epithelial E. Para-aminosalicylic acid
cell of the mucous membrane of the oral cavity,
enzymes that provide splicing are not 46. RNA that contains AIDS virus penetrated
synthesized. What is the reason for the into a leukocyte and by means of reverse
termination of protein biosynthesis in this case? transcriptase forced a cell to synthetize a viral
A. The i-RNA is not synthesised DNA. This process is based upon:
B. ATP is not synthesised A. Reverse transcription
C. The p-RNA is not synthesised B. Operon repression
D. Amino acids are not activated C. Reverse translation
E. Transport of amino acids has been disturbed D. Operon depression
E. Convariant replication
42. During reproduction of some RNA-
containing viruses that cause tumors in animals, 47. T-lymphocytes are determined to be affected
genetic information can be transmitted in the with HIV. In this case viral enzyme reverse
opposite direction from the RNA to the DNA transcriptase (RNA-dependent DNA-
via a specific enzyme. The enzyme of reverse polymerase) catalyzes the synthesis of:
transcription is called: A. DNA based on the viral RNA matrix
A. Reverse transcriptase B. Viral RNA based on the DNA matrix
B. DNA polymerase C. Viral protein based on the viral RNA matrix
C. Ligase D. Viral DNA based on the DNA matrix
D. Primase E. Informational RNA based on the viral protein
E. Topoisomerase matrix

43. In the body of a person transcription occurs. 48. For the treatment of urogenital infections
The RNA polymerase, moving along the DNA use hinolones - inhibitors of the enzyme DNA
molecule, has reached a certain sequence of gyrase. What process is disrupted by hinolones
nucleotides. After this transcription stopped. in the first place?
This section of the DNA is called: A. Replication of DNA
A. Terminator B. Reparation of DNA
B. Promoter C. Amplification of the genes
C. Repressor D. Recombination of the genes
D. Operator E. Reverse transcription

7
by different quantity of triplets ranging from
49. It was found out that some compounds, for one to six. Name this property of genetic code:
instance fungi toxins and some antibiotics can A. Degeneracy
inhibit activity of RNA-polymerase. What B. Universality
process will be disturbed in a cell in case of C. Disjointness
inhibition of this enzyme? D. Triplety
A. Transcription E. Specificity
B. Processing
C. Replication 54. Formation of ribosome subunits in a cell
D. Translation was disturbed in course of an experiment
E. Reparation (bymeans of . activated mutagenic factors). This
will have an effect on the following metabolic
50. At the stage of translation in the rough process:
endoplasmic reticulum, the ribosome moves A. Protein biosynthesis
along the mRNA. Amino acids are joined B. Carbohydrate biosynthesis
together by peptide bonds in a specific C. ATP synthesis
sequence, and thus polypeptide synthesis takes D. Photosynthesis
place. The sequence of amino acids in a E. Biological oxidation
polypeptide corresponds to the sequence of:
A. mRNA codons 55. Amino acids join to each other in ribosomes
B. tRNA nucleotides of granular endoplasmic reticulum. Knowing
C. tRNA anticodons the sequence of amino acids and applying
D. rRNA nucleotides genetic code, it is possible to determine the
E. rRNA anticodons sequence of nucleoids in:
A. mRNA
51. Students studying the features of the genetic B. Introns
code found that there are amino acids that are C. Proteins
answered by 6 codons, 5 amino acids - 4 D. Carbohydrates
different codons. Other amino acids are encoded E. rRNA
by three or two codons and only two amino
acids by one codon. What feature of the genetic 56. A tissue sample of benign tumor was studied
code did students discover? under the electron microscope. A lot of small
A. Degeneracy (Redundancy) (15-20 nm) spherical bodies, consisting of 2
B. Universality unequal subunits were
C. Collinearity detected. These are:
D. Unidirectional A. Ribosomes
E. Compose of three nucleotide B. Golgi complex
C. Smooth endoplasmic reticulum
52. The mutation of the structural gene did not D. Microtubules
lead to the substitution of amino acids in protein E. Mitochondria
molecules. This revealed the following property
of the genetic code: 57. A 36-year-old patient under went tooth
A. Degeneracy (Redundancy) extraction at a dental clinic. After two weeks the
B. Mutable stratified squamous epithelium regenerate dat
C. Collinearity the site of extraction.What organelles were
D. Insufficiency involved in there storation of the mucous
E. Universality membrane?
A. Ribosomes
53. It is known that information about amino B. Centrosomes
acid sequence in a protein molecule is stored as C. Postlysosomes
a sequence of four nucleotide types in a DNA D. Smooth EPR
molecule, and different amino acids are encoded E. Mitochondria

8
58. Genetic information is stored in DNA but
does not participate directly in protein synthesis 63. One of the protein synthesis stages is
with in DNA cells. What process ensures recognition. The first iRNA triplet starts with
transfer of genetic information into polypeptide UAU triplet. What complementary triplet is
chain? found in tRNA?
A. Translation A. AUA
B. FormationofrRNA B. AAA
C. FormationoftRNA C. GUG
D. FormationofiRNA D. UGU
E. Replication E. CUC

59. A patient has decreased concentration of 64. Cytological studies revealed a large number
magnesium ions that are required for ribosomes of different t-RNA molecules that deliver amino
connection to granular endoplasmic reticulum. acids to the ribosome. The number of different
This condition is known to disrupt the process types of t-RNA in a cell will be equal to the
of protein biosynthesis. Disruption occurs at the number of:
following stage: A. Triplets encoding amino acids
A. Translation B. Nucleotides
B. Transcription C. Amino acid
C. Replication D. Proteins synthesised in the cell
D. Amino acids activation E. Different types of RNA
E. Processing
65. Labelled amino acids alanine and
60. For the formation of a transport form of tryptophane were injected to a mouse in order to
amino acids for the synthesis of a protein, it is study localization of protein synthesis in its
necessary: cells. The labelled amino acids will be
A. Aminoacyl-tRNA synthetase accumulated near the following organellas:
B. GTP A. Ribosomes
C. m-RNA B. Smooth endoplasmic reticulum
D. Ribosome C. Cell centre
E. Revertase D. Lysosomes
E. Golgi apparatus
61. During the study of cells, a high content of
the aminoacyl-tRNA synthetase enzyme was 66. The patient was prescribed antibiotic
established in their cytoplasm. This enzyme chloramphenicol (levometsitin), which disrupts
provides the following process in a cell: protein synthesis in the body by inhibiting the
A. Activation of amino acids process:
B. Reparation A. Translation elongation
C. Elongation B. Formation of polyribosomes
D. Transcription C. Transcription
E. Replication D. Processing
E. Gene amplification
62. In the cell there is a process of translation.
When the ribosome reaches the codons of UAA, 67. Infectious diseases are treated with
UAG, or UGA, the polypeptide chain synthesis antibiotics (streptomycin, erythromycin,
is terminated. These codons are not recognized chloramphenicol). They inhibit the following
by any t-RNA in the biosynthesis of the stage of protein synthesis:
polypeptide and this is a signal: A. Translation
A. Termination B. Transcription
B. Post-translational modification C. Replication
C. Begin transcription D. Processing
D. Elongation E. Splicing
E. Initiation

9
68. A 28-year-old patient with bacterial E.coli. What portion of the lactose operon will
pneumonia was prescribed a course of treatment be unlocked from the repressor in these
with erythromycin. Its antibacterial properties conditions?
are known to be due to the ability of this A. Operator
substance to combine with the free 50S-subunit B. Promoter
of the ribosome. What substances synthesis does C. Structural gene
this antibiotic block in bacterial cells? D. Regulatory gene
A. Proteins E. Primer
B. RNA
C. DNA 73. It is known that the gene responsible for the
D. Fat development of the MN blood groups has two
E. Polysaccharides allelic states. If the gene M is considered as the
initial gene, the allelic gene N appeared due to:
69. Streptomycin and other aminoglycosides A. Mutations
prevent the joining of formylmethionyl tRNA B. Gene combinations
by bonding with the 30S ribosomal subunit. C. DNA repair
This effect leads to disruption of the following D. DNA replication
process: E. Crossing over
A. Translation initiation in procaryotes
B. Translation initiation in eucaryotes 74. As a result of treatment of viral RNA with
C. Transcription initiation in procaryotes nitrous acid, UCA triplet mutated to UGA
D. Transcription initiation in eucaryotes triplet. What kind of mutation occurred?
E. Replication initiation in procaryotes A. Transition
B. Nucleotide deletion
70. In a genetical laboratory in course of work C. Missense
with DNA molecules of white rats of Wistar’s D. Nucleotide insertion
line a nucleotide was substituted for another E. Inversion
one. At that only one amino acid was
substituded in the peptide. This 75. A mutation has occurred in a cell in the first
result is caused by the following mutation: exon of the structural gene. The number of
A. Transversion nucleotide pairs changed from 290 to 250.
B. Deletion Name this type of mutation:
C. Duplication A. Deletion
D. Displacement of reading frame B. Inversion
E. Translocation C. Duplication
D. Translocation
71. Hurtnup’s disease is caused by point E. Nullisomy
mutation of only one gene. This results in
abnormal absorption of tryptophane in the 76. When examining a 2-month-old child, the
intestine as well as its abnormal pediatrician noticed that the crying of the child
reabsorption in renal tubules. This causes resembles a cat's cry. Diagnosed microcephaly
synchronous disorders in digestive and urinary and heart disease. Using the cytogenetic
excretion systems. What genetic phenomenon is method, the child's cartiotype: 46, XX, 5p. This
observed in this case? disease is a consequence of this process:
A. Pleiotropy A. Deletion
B. Complementary interaction B. Duplication
C. Polymery C. Inversion
D. Codominance D. Translocation
E. Semidominance E. Pleiotropy

72. During the experiment, an increase in β- 77. Parents of a sick 5-year-old girl visited a
galactosidase activity was demonstrated after genetic consultation. Karyotype investigation
lactose was added to the culture medium with revealed 46 chromosomes. One chromosome of

10
the 15th pair was abnormally long, having a part D. Translocation
of the chromosome belonging to the21st pair E. Replication
attached to it. What mutation occurred in this
girl? 82. An experiment proved that UV irradiated
A. Translocation skin cells of patients with xeroderma
B. Deletion pigmentosum restore the native structure of
C. Inversion DNA slower than the cells of healthy people
D. Deficiency due to the defect in repair enzyme. What
E. Duplication enzyme takes part in this process?
A. Endonuclease
78. In some regions of South Africa there is a B. RNA ligase
spread sickle-shaped cell anemia, in which C. Primase
erythrocytes have shape of a sickle as a result of D. DNA polymerase
substitution of glutamin by valine in the E. DNA gyrase
hemoglobin molecule. What is the cause of this
disease? 83. In the course of evolution there developed
A. Gene mutation molecular mechanisms for correction of
B. Disturbance of mechanisms of genetic damaged DNA molecules. This process is
information realization called:
C. Crossingover A. Reparation
D. Genomic mutations B. Transcription
E. Transduction C. Translation
D. Replication
79. Sickle-shaped e cell anemia in humans is E. Processing
accompanied by the appearance of abnormal
hemoglobin in the blood, a change in the form 84. In the human cells under the action of
of red blood cells and the development of ultraviolet radiation the DNA molecule has been
anemia. This disease is the result of: damaged. The system of repairing the damaged
A. Gene mutations area of the DNA molecule to the intact chain
B. Polythene through a specific enzyme was revised. What is
C. Chromosomal aberration the name of this phenomenon?
D. Polyplody A. Reparation
E. Mitochondrial mutation B. Duplication
C. Replication
80. A 15-year-old man is complaining of D. Initiation
general weakness, dizziness, rapid fatigability. E. Termination
During the examination, erythrocytes of a
modified form were detected, their number was 85. Cells of a person working in the Chornobyl
reduced. Preliminary diagnosis: sickle cell Exclusion Zone have under gone a mutation in
anemia. What type of mutation causes the DNA molecule. However, with time the
development of this pathological condition? damaged interval of DNA molecule has been
A. Point mutation restored to its initial structure with a specific
B. Reading frame shift mutation enzyme. In this case the following occurred:
C. Deletion A. Repair
D. Inversion B. Replication
E. Chromosomal aberration C. Transcription
D. Reverse transcription
81. Part of the DNA chain turned 180 degree as E. Translation
a result of gamma radiation. What type of
mutation took place in the DNA chain? 86. Under the influence of physical factors there
A. Inversion can develop defect sin a DNA molecule.
B. Deletion Ultraviolet irradiation, for instance, can cause
C. Doubling development of dimers. Dimers are two adjacent

11
pyrimidine bases joined together. Name these 91. Fetal malformations may result from such
bases: maternal diseases as rubella, syphilis,
A. Thymine and cytosine toxoplasmosis, cytomegaly, herpes, chlamydia.
B. Adenine and thymine What is the form of variability relating to such
C. Guanine and cytosine malformations?
D. Adenine and guanine A. Modifiation
E. Guanine and thymine B. Mutational
C. Combinative
87. Patients suffering from xeroderma D. Genomic imprinting
pigmentosum have extremely photosensitive E. Epimutational
skin due to disrupted excision repair. Specify
the process that is affected in such patients: 92. As a result of iodine deficiency in
A. Repair of DNA molecule foodstuffs Transcarpathian people often have
B. Synthesis of iRNA endemic goiter. This disease is caused by the
C. Maturation of iRNA following type of variability:
D. Synthesis of protein primary structure A. Modification
E. Intron extraction and exon connection B. Mutational
Test items for licensing examination Krok 1 C. Combinative
«Stomatology». – 2016. - № 192. D. Ontogenetical
88. An employee of a chemical enterprise was E. Correlative
exposed to nitric acid and nitrite, which cause
cytosine deamination in the DNA molecule. 93. Lymphocyte is affected by HIV retrovirus
What enzyme initiates a chain of reparation (AIDS). In this case, the direction of
processes? information flow in the cell will be:
A. Uridine DNA glycosidase A. RNA → DNA → i-RNA → polypeptide
B. Citidine triphosphate synthetase B. DNA → i-RNA → polypeptide → DNA
C. Orotidyl monophosphate decarboxylase C. DNA → Polypeptide → i-RNA
D. DNA-dependent RNA polymerase D. i-RNA → polypeptide → DNA
E. Thymidylat synthase E. Polypeptide → RNA → DNA → i-RNA

89. Tetracycline taking in the first half of 94. A young family came for a genetic
pregnancy causes abnormalities of fetus organs counseling to identify the father of their child.
and systems, including tooth hypoplasia and The husband insists that the child does not
alteration of their colour. What type of resemble him at all and cannot possibly be his.
variability is the child’s disease related to? Polymerase chain reaction method
A. Modification for person identification is based on the
B. Combinative following:
C. Mutational A. Gene amplification
D. Hereditary B. Nucleotide deletion
E. Recombinan C. Genetic recombination
D. Missense mutation
90. The woman took antibiotics in the first half E. Transduction
of pregnancy. This led to the hypoplasia of the
teeth and the change in their color in the child. 95. A doctor was addressed by a 30-year old
The genotype has not changed. Set the type of man. There is a probability of the patient being
variability that underlies the disease: HIV-positive. To clarify the diagnosis the
A. Modification doctor proposed to perform polymerase chain
B. Combinative reaction. The basic process in this kind
C. Mutational of investigation is:
D. Correlative A. Gene amplification
E. Recombinative B. Transcription
C. Genetic recombination
D. Genomic mutation

12
E. Chromosome mutation A. Phenocopy
B. Mutation
96. It is known that the gene responsible for C. Recombination
development of blood groups according to AB0 D. Heterosis
system has three allele variants. Existence of the E. Replication
IV blood group can be explained by the
following variability form: 98. The doctor discovered a kid's disease due to
A. Combinative a lack of vitamin D, but in his manifestation
B. Mutational similar to hereditary vitamin-resistant rickets
C. Phenotypic (curvature of tubular bones, deformity of joints
D. Genocopy of lower extremities, dental abscesses). How
E. Phenocopy called defects of development, which resemble
hereditary, but not inherited?
97. A mother had taken synthetic hormones A. Phenocopy
during pregnancy. Her daughter was born with B. Genocopy
hirsutismformally resembling of adrenal C. Monosomies
syndrome. Such D. Trisomy
manifestation of variability is called: E. Gene diseases

13
Biochemistry of intercellular communications: hormones of protein-
peptide nature
1. Degeneration of glycogen in liver is B. Triamcinolone
stimulated by glucagon. What secondary C. Norepinephrine
messenger (mediator) is thus formed in the cell? D. Prenisolone
A. c-AMP E. Hydrocortisone
B. c-GMP
C. CO 6. Condition of a patient with diabetes mellitus
D. NO sharply deteriorated after a regular injection of
E. Triacylglycerol insulin. The patient became anxious and broke
out in cold sweat; tremor of the extremities,
2. The ions of metals take part in the regulation general weakness, and dizziness appeared. What
of physiological functions. One of them was medicine can remove these symptoms?
named the “king of messengers”. Such a A. Adrenaline
bioelement mediator is: B. Tolbutamide
A. Ca ++ C. Caffeine
B. Na + D. Noradrenaline
C. K + E. Glibutid(Buformin)
D. Fe +++
E. Zn ++ 7. Dentists widely apply local anaesthesia
adding adrenalin to an anaesthetic solution.
3. Tissue inosytol triphosphates are generated as What is the purpose of this method?
a result of the phosphatidyl inositol diphosphate A. Local vasoconstriction
hydrolysis and act as secondary agents B. Local vasodilatation
(mediators) in the mechanism of hormone C. Lowering of arterial pressure
action. Their effect in cells is directed at: D. Local reduction of vascular resistance
A. Calcium ion liberation from cellular depot E. Microcirculation improvement
B. Adenylate cyclase activation
C. Protein kinase A activation 8. A 50-year-old man declined anaesthesia
D. Phosphodiesterase inhibition during dental manipulations. Due to severe pain
E. Protein kinase A inhibition he developed anuria caused by acute increase in
production of:
4. The formation of a secondary mediatoris A. Adrenaline
obligatory in membrane-intracellular B. Renin
mechanism of hormone action. Point out the C. Thymosin
substance that is unable to be a secondary D. Thyroxin
mediator: E. Glucagon
A. Glycerol
B. Diacylglycerol 9. A sick woman after parenteral administration
C. Inositol-3,4,5-triphosphate of the hormone had an increase in blood
D. CAMP pressure, as well as increased levels of glucose
E. Ca2+ and lipids in the blood. What hormone was
introduced?
5. A patient with a diagnosis of diabetes, in the A. Adrenaline
morning on an empty stomach received a B. Glucagon
prescribed dose of insulin with prolonged C. Insulin
action. He missed another meal and soon felt D. Progesterone
weakness, pain, dizziness, sweating, trembling E. Folliculin
of the body, convulsions, feeling of hunger,
hypoglycemia. The use of glucose did not 10. A patient suffering from pheochromocytoma
alleviate the condition. What hormonal drug is complains of thirst, dry mouth, hunger. Blood
necessary to enter to stop this condition? test for sugar revealed hyperglycemia. What
A. Adrenaline type of hyperglycemia is it?
14
A. Adrenal
B. Hypercorticoid 15. Before the cells can utilize the glucoze, it is
C. Alimentary first transported from the extracellular space
D. Somatotropic through the plasmatic membrane inside theml.
E. Hypoinsulinemic This process is stimulated by the following
hormone:
11. Patient with diabetes mellitus experienced A. Insulin
loss of consciousness and convulsions after an B. Glucagon
injection of insulin. What might be the result of C. Thyroxin
biochemical blood analysis for concentration of D. Aldosterone
sugar? E. Adrenalin
A. 1,5 mmol/L
B. 8,0 mmol/L 16. After insulin injection, glucose levels fall
C. 10,0 mmol/L within a few seconds. This is due to the
D. 3,3 mmol/L activation of this process: A. Transport of
E. 5,5 mmol/L glucose into cells
B. Glycolysis
12. A patient with diabetes mellitus had an C. Synthesis of glycogen
insuline injection. It caused loss of D. Synthesis of lipids
consciousness and convulsions. What was the E. Pentose phosphate cycle
result of biochemic blood analysis on glucose
content? 17. Prior to glucose utilization in cells it is
A. 2,5 mmоle/l transported inside cells from extracellular space
B. 3,3 mmоle/l through plasmatic membrane. This process is
C. 8,0 mmоle/l stimulated by the following hormone:
D. 10 mmоle/l A. Insulin
E. 5,5 mmоle/l B. Glucagon
C. Thyroxin
13. A 16-year-old young man was taken to D. Aldosterone
hospital, the patient had insulin-dependent E. Adrenalin
diabetes mellitus. The patient's blood glucose
level was 1.8 mmol/l. The patient was given 18. A patient is in the state of hypoglycemic
insulin. Two hours later, the glucose level coma. What hormone can cause this condition if
decreased to 8.2 mmol/l, since insulin: overdosed?
A. Stimulates the conversion of glucose to A. Insulin
glycogen and TAG in the liver B. Progesterone
B. Stimulates glucose transport through plasma C. Cortisol
membranes in the brain and liver D. Somatotropin
C. It inhibits the synthesis of ketone bodies from E. Corticotropin
glucose
D. Stimulates the breakdown of glycogen in the 19. A patient during fasting developed
liver E. Stimulates the breakdown of glycogen ketoacidosis as a result of increased fatty acids
in the muscles decomposition. This decomposition can be
inhibited with:
14. A patient with insulin-dependent diabetes A. Insulin
had an insulin injection. Some time later he felt B. Glucagon
weakness, irritability, excessive sweating. What C. Adrenaline
is the main reason of these disorders? D. Thyroxin
A. Carbohydrate starvation of brain E. Cortisol
B. Intensified glycogenolysis
C. Intensified ketogenesis 20. Examination of a patient revealed
D. Intensified lypogenesis overgrowth of facial bones and soft tissues,
E. Reduced glyconeogenesis tongue enlargement, wide interdental spaces in

15
the enlarged dental arch. What changes of the 25. A student who passes an exam has a plasma
hormonal secretion are the most likely? glucose content of 8 mmol/l. Increased secretion
A. Hypersecretion of the somatotropic hormone which of hormones contributes to the
B. Hyposecretion of the somatotropic hormone development of hyperglycemia?
C. Hypersecretion of insulin A. Glucagon
D. Hyposecretion of thyroxin B. Insulin
E. Hyposecretion of insulin C. Thyroxine
D. Triiodothyronine
21. A 49-year-old patient was found to have a E. Aldosterone
disproportionate enlargement of hands, feet,
nose, ears, superciliary arches and cheek bones. 26. A patient with tress and painful sensation
Blood test revealed hyperglycemia, impaired before a visit to the dentist is accompanied by
glucose tolerance. What is the most likely cause anuria (lack of urination). This phenomenon is
of this pathology development? due to an increase in:
A. Hypersecretion of growth hormone A. Secretion of vasopressin and adrenaline
B. Posterior pituitary hormone hypersecretion B. Activity of the parasympathetic nervous
C. Insulin hyposecretion system
D. Vasopressin hyposecretion C. Activity antinociceptive system
E. Glucocorticoid hypersecretion D. Secretion of vasopressin and a decrease in
adrenaline
22. A woman after labor lost 20 kg of body E. Secretion of adrenaline and a decrease in
weight, her hair and teeth fall out, she has vasopressin
muscle atrophy (hypophysial cachexia).
Synthesis of what hypophysis hormone is 27. As a result of a home injury, a patient
disturbed? suffered a significant blood loss, which led to a
A. Somatotropic fall in blood pressure. Rapid blood pressure
B. Corticotrophic recovery after the blood loss is provided by the
C. Thyreotropic following hormones:
D. Gonadotropic A. Adrenaline, vasopressin
E. Prolactin B. Cortisol
C. Sex hormones
23. Adult height 100 cm with a proportional D. Oxytocin
body structure and normal mental development. E. Aldosterone
It might be caused by deficiency secretion of the
following hormone: 28. A severe injury in a 36-year-old patient
A. Somatotropic resulted in a significant blood loss which was
B. Gonadotropic accompanied by a blood pressure drop. What
C. Adrenocorticotropic hormones provide rapid recovery of blood
D. Thyroid-stimulating pressure after the blood loss?
E. Prolactin A. Adrenalin, vasopressin
B. Cortisol
24. Examination of a patient revealed C. Sex hormones
enlargement of some body parts (jaw, nose, D. Oxytocin
ears, feet, hands), but body proportions were E. Aldosterone
conserved. It might be caused by intensified
secretion of the following hormone: 29. A man has the increased volume of
A. Somatotropin circulating blood and reduced osmotic pressure
B. Somatostatin of plasma considerable. He has decrease in
C. Tetraiodothyronine diuresis. The primary cause of such diuresis
D. Triiodothyronine disorder is the hypersecretion of the following
E. Cortisol hormone:
A. Vasopressin
B. Aldosterone

16
C. Adrenalin C. Vasopressin
D. Renin D. Thyrocalcitonin
E. Natriuretic E. Glucagon

30. A 20 year old patient complains of excessive 35. A 25-year-old woman one month after
thirst and urinary excretion upto 10 L a day. The giving birth consulted a doctor about a decrease
level of glucose in blood is normal, there is no in milk production. What hormone deficiency
glucose in urine. What hormone deficit can led to this condition?
cause such changes? A. Prolactin
A. Vasopressin B. Somatostatin
B. Oxytocin C. Adrenocorticotropic hormone
C. Insulin D. Insulin
D. Triiodothyronine E. Glucagon
E. Cortisol
36. Products of some proteins hydrolysis and
31. A patient with pituitary tumor complains of modification are the biologically active
increased daily diuresis (polyuria). Glucose substances called hormones. Lipotropin,
concentration in blood plasma equals 4,8 corticotropin, melanotropin and endorphins are
mmol/l. What hormone can be the cause of this synthesized in the hypophysis of the following
if its secretion is disturbed? protein:
A. Vasopressin A. Proopiomelanocortin (POMC)
B. Aldosterone B. Neuroalbumin
C. Natriuretic hormone C. Neurostromin
D. Insulin D. Neuroglobulin
E. Angiotensin I E. Thyreoglobulin

32. Limiting water intake has led to dehydration 37. After a case of sepsis a 27-year-old woman
of the organism. What mechanism is activated developed ”bronzed” skin discoloration
under these conditions to preserve water in the characteristic of Addison’s disease.
body? Hyperpigmentation mechanismin this case is
A. Increased vasopressin secretion based on increased secretion of:
B. Increased somatostatin secretion A. Melanocyte-stimulating hormone
C. Reduced vasopressin secretion B. Somatotropin
D. Increased aldosterone secretion C. Gonadotropin
E. Reduced aldosterone secretion D. β-lipotropin
E. Thyroid-stimulating hormone
33. At ria of a test animal were super distended
with blood,which resulted in decreased 38. A 20-year-old woman came to the doctor
reabsorption of Na+ and water in renal with complaints of general weight loss, loss of
tubules.This can be explained by the effect of appetite, weakness, skin discoloration
the following factor on the kidneys: resembling bronze tan. In addtition to
A. Natriuretic hormone hyperpigmentation, examination in the hospital
B. Aldosterone revealed bilateral adrenal tuberculosis. What
C. Renin substance leads to skin hyperpigmentation,
D. Angiotensin when accumulated excessively?
E. Vasopressin A. Melanin
B. Bilirubin
34. A 32-year-old patient consulted a doctor C. Hemozoin
about the absence of lactation after parturition. D. Lipofuscin
Such disorder might be explained by the deficit E. Adrenochrome
of the following hormone:
A. Prolactin 39. A patient with gastric juice hypersecretion
B. Somatotropin has been recommended to exclude from the diet

17
rich broths and vegetable infused water. A
doctor recommended it, because these food 44. A 40-year-old patient complains of intensive
products stimulate production of the following heartbeats, sweating, nausea, visual impairment,
hormone: arm tremor, hypertension. From his anamnesis:
A. Production of gastrin 2 years ago he was diagnosed with
B. Taste Receptors pheochromocytoma. Hyperproduction of what
C. Mechano-receptors of the oral cavity hormones causes the given pathology?
D. Mechanoreceptors of the stomach A. Catecholamines
E. Formation of secretin B. Aldosterone
C. Glucocorticoids
40. During an acute experiment some of diluted D. ACTH
solution of hydrochloric acid was injected into E. Thyroidal hormones
the duodenal cavity of an experimental animal.
This will result in hypersecretion of the 45. A special diet has led to a decrease in Ca2+
following hormone: ions in the blood. What hormone will increase
A. Secretin the secretion?
B. Gastrin A. Parathormone
C. Motilin B. Thyrocalcitonin
D. Neurotensin C. Vasopressin
E. Histamine D. Somatotropin
E. Thyroxine
41. A pregnant woman with a low labor activity
entered the maternity ward. Assign a hormonal 46. A patient has the sudden decrease of Ca2+
remedy to enhance labor activity: content in blood. What hormone secretion will
A. Oxytocin increase?
B. Progesterone A. Parathormone
C. Methandrostenolone B. Thyrocalcitonin
D. Hydrocortisone C. Aldosterone
E. ACTH D. Vasopressin
E. Somatotropin
42. A 26-year-old woman at 40 weeks pregnant
has been delivered to the maternity ward. 47. The calcium ions in the blood plasma of a
Objectively: the uterine cervix is opened, but patient is increased, reduced - in the bones. The
the contractions are absent. The doctor has excessive secretion of which hormone may
administered her a hormonal drug to stimulate cause such changes?
the labor. Name this drug: A. Parathormone
A. Oxytocin B. Thyroxine
B. Hydrocortisone C. Triiodothyronine
C. Estrone D. Thyreocalcitonin
D. Testosterone E. Aldosteron
E. ACTH
48. Kidneys of a man under examination show
43. To stimulate the laboractivity aparturient increased resorbtion of calcium ions and
woman was prescribed a drug a posterior decreased resorbtion of phosphate ions. What
pituitary hormone that does not affect the blood hormone causes this phenomenon?
pressure. As the pregnancy progresses, the A. Parathormone
sensitivity to this hormone increases.Name the B. Thyrocalcitonin
prescribed drug: C. Hormonal form D3
A. Oxytocin D. Aldosterone
B. Dinoprostone E. Vasopressin
C. Dinoprost
D. Pituitrin 49. Following thyroid surgery, a 47-year old
E. Ergotal female patient had fibrillary twitching of

18
muscles in the arms, legs and face. These
disorders can be treated by the introduction of 54. A patient has hypocalcemia. What hormone
the following hormone: deficiency may be it’s cause?
A. Parathyroid hormone A. Parathormone
B. Triiodothyronine B. Thyrocalcitonin
C. Thyrotropin C. Aldosterone
D. Thyroxine D. Corticotropin
E. Thyroid-stimulating hormone E. Corticoliberin

50. During removal of the hyperplastic thyroid 55. A patient presents with osteoporosis;
gland of a 47-year-old woman, the parathyroid hypercalcemia and hypophosphatemia are
gland was damaged.One month after the surgery observed in the patient’s blood. What is the
the patient developed signs of cause of this condition?
hypoparathyroidism: frequent convulsions, A. Increased parathormone secretion
hyperreflexia, laryngospasm. What is the most B. Increased thyroxin secretion
likely cause of the patient’s condition? C. Inhibited parathormone secretion
A. Hypocalcemia D. Increased corticosteroid secretion
B. Hyponatremia E. Inhibited corticosteroid secretion
C. Hyperchlorhydria
D. Hypophosphatemia 56. A 5-month-old boy was hospitalized for
E. Hyperkalemia tonic convulsions. He has a lifetime history of
this disease. Examination revealed coarse hair,
51. A child has abnormal formation of tooth thinned and fragilenails, pale and dry skin. In
enamel and dentin as a result of low blood: calcium - 1,5 millimole/l, phosphor - 1,9
concentration of calcium ions in blood. Such millimole/l. These changes are associated with:
abnormalities might be caused by deficiency of A. Hypoparathyroidism
the following hormone: B. Hyperparathyroidism
A. Parathormone C. Hyperaldosteronism
B. Thyrocalcitonin D. Hypoaldosteronism
C. Thyroxin E. Hypothyroidism
D. Somatotropic hormone
E. Triiodothyronine 57. A 5-month-old boy was hospitalized for
tonic convulsions. Sick since birth. On
52. Owing to a pronounced decrease in plasma examination, the hair is hard, the nails are
calcium concentration, a 2-year-old child had thinned and brittle, the skin is pale and dry. In
tetanic reductions in the respiratory and the blood: calcium - 1.5 mmol / l, phosphorus -
pharyngeal muscles. Decreased secretion of 1.9 mmol / l. What are these changes related to?
what hormone can be the cause of this? A. Hypoparathyroidism
A. hormone B. Hyperparathyroidism
B. Calcitonin C. Hyperaldosteronism
C. Aldosterone D. Hypoaldosteronism
D. Somatotropin E. Hypothyroidism
E. Cortisol
58. Periodic renal colics attacks areobserved in
53. A 2-year-old child experienced convulsions a woman with primery hyperparathyroidizm.
because of lowering calciumions concentration Ultrasonic examination revealed small stones in
in the blood plasma. Function of what structure the kidneys. What is the most plausible reason
is decreased? of the stones’s formation?
A. Parathyroid glands A. Hypercalcemia
B. Hypophysis B. Hyperphosphatemia
C. Adrenal cortex C. Hypercholesterinemia
D. Pineal gland D. Hyperuricemia
E. Thymus E. Hyperkalemia

19
B. Uric acid
59. A 56-year-old man presents with C. Cystine
parathyroid tumor. The following is observed: D. Bilirubin
muscle weakness, osteoporosis, bone E. Cholesterol
deformation, nephroliths consisting of oxalates
and phosphates. The patient’s condition is 64. Parodontitis is treated with calcium
caused by: preparations and a hormone that stimulates
A. Increased secretion of parathyroid hormone tooth mineralization and inhibits tissue
B. Decreased secretion of parathyroid hormone resorption. What hormone is it?
C. Increased secretion of calcitonin A. Calcitonin
D. Decreased secretion of calcitriol B. Parathormone
E. Increased secretion of thyroxin C. Adrenalin
D. Aldosterone
60. Due to trauma the patient’s parathyroid E. Thyroxine
glands have been removed, which resulted in
inertness, thirst, sharp increase of 65. A patient with hypoparathyreosis has
neuromuscular excitability. Metabolism of the multiple carious lesions of teeth. This pathology
following substance is disturbed: is caused by insufficiency of the following
A. Calcium hormone:
B. Manganese A. Calcitonin
C. Chlorine B. Thyroxin
D. Molybdenum C. Triiodothyronine
E. Zinc D. Thyroid-stimulating hormone
E. Somatotropin
61. There is a 9 year old boy in endocrinological
department, who has already had a few fractures 66. A 46-year-old patient suffering from the
of extremeties caused by fragility of bones. diffuse toxic goiter underwent resection of the
Malfunction of what endocrinous glands (gland) thyroid gland. After the surgery the patient
takes place? presents with appetite loss, dyspepsia, increased
A. Parathyroid glands neuromuscular excitement. The body weight
B. Thyroid gland remained unchanged. Body temperature is
C. Thymus normal. Which of the following has caused such
D. Adrenal glands a condition in this patient?
E. Epiphysis A. Reduced production of parathormone
B. Increased production of thyroxin
62. A patient with signs of osteoporosis and C. Increased production of calcitonin
urolithiasis has been admitted to the D. Increased production of thyroliberin
endocrinology department. Blood test revealed E. Reduced production of thyroxin
hypercalcemia and hypophosphatemia. These
changes are associated with abnormal synthesis 67. During surgery on the thyroid gland due to
of the following hormone: the disease of the Basedow's disease, the
A. Parathyroid hormone parathyroid glands were mistakenly removed.
B. Calcitonin There were convulsions, tetany. Exchange of
C. Cortisol which item was broken?
D. Aldosterone A. Calcium
E. Calcitriol B. Magnesium
C. Potassium
63. Cardinal symptoms of primary D. Iron
hyperparathyroidism are osteoporosis andrenal E. Sodium
lesion along with development of urolithiasis.
What substance makes up the basis of these 68. Soldiers injured in the midst of a battle may
calculi in this disease? not feel pain before it is completed. What
A. Calcium phosphate

20
hormones of the opiate antinociceptive system C. Vasopresin
reduce the sensation of pain? D. Aldosterone
A. Endorphins E. Oxytocin
B. Serotonin

21
Biochemistry of intercellular communications: hormones of steroid and
thyroid nature
1. A girl is diagnosed with adrenogenital 6. Parents of a 10 y.o. boy consulted a doctor
syndrome (pseudohermaphroditism). This about extension of hair-covering, growth of
pathology was caused by hypersecretion of the beard and moustache, low voice. Intensified
following adrenal hormone: secretion of which hormone must be assumed?
A. Androgen A. Of testosterone
B. Estrogen B. Of somatotropin
C. Aldosterone C. Of oestrogen
D. Cortisol D. Of progesterone
E. Adrenalin E. Of cortisol

2. The secretion of which hypophysial 7. A female patient presents with endocrine


hormones will be inhibited after taking the oral dysfunction of follicular cells of the ovarian
contraceptives containing sexhormones? follicles resulting from an inflammation. The
A. Gonadotropic hormone synthesis of the following hormone will be
B. Vasopressin inhibited:
C. Thyrotrophic hormone A. Estrogen
D. Somatotropic hormone B. Progesterone
E. Ocytocin C. Lutropin
D. Follicle stimulating hormone
3. The woman was threatened with premature E. Follistatine
termination of pregnancy. This is most likely
due to insufficient secretion of such a hormone: 8. Intake of oral contraceptives containing sex
A. Progesterone hormones inhibits secretion of the hypophysiae
B. Estradiol hormones. Secretion of which of the indicated
C. Oxytocin hormones is inhibited while taking oral
D. Testosterone contraceptives with sexhormones?
E. Aldosterone A. Follicle-stimulating
B. Vasopressin
4. Testosterone and it’s analogs increase the C. Thyrotropic
mass of skeletal muscles that allows to use them D. Somatotropic
for treatment of dystrophy. Dueto interaction of E. Oxytocin
the hormone with what cell substrate is this
action caused? 9. Corticosteroid hormones regulate the
A. Nuclear receptors adaptation processes of the body as a whole to
B. Membrane receptors environmental changes and ensure the
C. Ribosomes maintenance of internal homeostasis. What
D. Chromatin hormone activates the hypothalamo-pituitary-
E. Proteins-activators of transcription adrenal axis?
A. Corticoliberin
5. A 30-year-old female exhibits signs of B. Somatoliberin
virilism (growth of body hair, balding temples, C. Somatostatin
menstrual disorders). This condition can be D. Corticostatin
caused by the over production of the following E. Thyroliberin
hormone:
A. Testosterone 10. A man prescribed by a doctor for a long
B. Oestriol time took the preparation of a group of
C. Relaxin glucocorticoid hormones. Which of the
D. Oxytocin following hormones will be depressed due to
E. Prolactin this?
A. Corticotropic
B. Somatotropic
22
C. Tyrotropic C. Muscle glycogenolysis
D. Sexual D. Intestinal glucose absorption
E. Mineralocorticoid E. Synthesis of glycogen

11. The release of adrenal hormones is regulated 16. A patient with Itsenko-Cushing syndrome
by ACTH of the adenohypophysis. What has persistent hyperglycemia and glycosuria,
hormones are secreted by the adrenal glands hypertension, osteoporosis, obesity. What
under the action of the latter? hormon’s synthesis and secretion are intensified
A. Glucocorticoids in this case?
B. Androgenes A. Cortisol
C. Catecholamines B. Adrenaline
D. Mineralocorticoid C. Glucagon
E. Prostaglandins D. Thyroxin
E. Aldosterone
12. A patient has been taking glucocorticoids
for a long time. Drug withdrowal caused acute 17. On examination of the patient, the doctor
attack of his disease, blood pressure reduction, suspected Itsenko-Cushing's syndrome.
weakness. What are these occurences connected Determination of what substance in the patient's
with? blood will confirm the doctor's presumption
A. Adrenal glands insufficiency A. Cortisol
B. Drug habituation B. Tocopherol
C. Sensibilisation C. Retinol
D. Hyperproduction of corticotroph hormone D. Adrenaline
E. Cumulation E. Cholesterol

13. Chronic overdosage of glucocorticoids leads 18. A patient with a diagnosis of Itsenko-
to the development of hyperglycemia. What Cushing's disease (hyperproduction of adrenal
process of carbohydrate metabolism is hormones) has an increased concentration of
responsible for this effect? glucose, ketone bodies, sodium in the blood .
A. Gluconeogenesis What is the biochemical mechanism leading to
B. Glycogenolysis the occurrence of hyperglycemia?
C. Aerobic glycolisis A. Gluconeogenesis
D. Pentose-phosphate cycle B. Glycogenesis
E. Glycogenesis C. Glycogenolysis
D. Glycolysis
14. A 40-year-old woman with Cushing’s E. Aerobic glycolysis
disease presents with steroid diabetes. On
biochemical examination she has hyperglycemia 19. The patient was found to have obesity,
and hypochloremia. What process activates in hirsutism, a “moon-shaped” face, scarring of a
the first place in such patients? purple color on the skin of the thighs. Blood
A. Gluconeogenesis pressure is 180/110 mm Hg, blood glucose is
B. Glycogenolysis 17.2 mmol / l. What change in the production of
C. Glucose reabsorption adrenal hormones is possible such a picture?
D. Glucose transportation into a cell A. Hyperproduction of glucocorticoids
E. Glycolysis B. Hypoproducts of glucocorticoids
C. Hyperproduction of mineralocorticoids
15. A 28-year-old patient with Itsenko-Cushing D. Hypoproducts of mineralocorticoids
syndrome Hyperglycemia, glycosuria were E. Hypoproduction of adrenaline
detected hyperglycemia, glycosuria. The main
mechanism of hyperglycemia in this patient is 20. A 44 year old woman complains of general
stimulation: weakness, heart pain, significant increase of
A. Gluconeogenesis body weight. Objectively: moon face, hirsutism,
B. Liver glycogenolysis AP is 165/100 mm Hg, height - 164 cm, weight

23
- 103 kg; the fat is mostly accumulated on her D. Diabetes mellitus
neck, thoracic girdle, belly. What is the main E. Essential hypertension
pathogenetic mechanism of obesity?
A. Increased production of glucocorticoids 25. A 38-year-old female patient complains of
B. Reduced production of thyroid hormones general weakness, cardiac pain, increased
C. Increased insulin production appetite, no menstruation. Objectively: the
D. Reduced glucagon production height is 166 cm, weight 108 kg, the patient has
E. Increased mineralocorticoid production moon-shaped face, subcutaneous fat is
deposited mainly in the upper body, torso and
21. A 16-year-old patient suffering from hips. There are also blood-red streaks. Ps-
Itsenko-Cushing's disease was counseled for 62/min, AP-160/105 mm Hg. Which of the
over weight. During the survey, it turned out following diseases is the described pattern of
that the energy value of the food consumed is obesity most typical for?
1700-1900 kcal / day. What is the leading cause A. Cushing pituitary basophilism
of obesity in this case? B. Alimentary obesity
A. Excess glucocorticoid C. Myxedema
B. Lack of insulin D. Insulinoma
C. Excess insulin E. Babinski-Frohlich syndrome
D. Lack of glucocorticoids
E. Hypodynamia 26. Examination of a patient with high blood
pressure revealed secondary arterial
22. A 29-year-old female patient has moon - hypertension. It was found that the cause of this
shaped face, upper body obesity, stretch marks condition of the patient is a hormonally active
on the abdomen, hirsutism; urine shows an tumor of the adrenal cortex. Hyperproduction of
increased rate of 17-oxy ketosteroids. What what hormone is the cause of secondary arterial
disease are these presentations typical for? hypertension in a patient?
A. Itsenko-Cushing syndrome A. Cortisol
B. Pheochromocytoma B. Adrenaline
C. Conn’s syndrome C. Thyroxine
D. Primary aldosteronism D. Insulin
E. Secondary aldosteronism E. Glucagon

23. The patient with complaints of permanent 27. Acceptance of corticosteroid analogues
thirst applied to the doctor. Hyperglycemia, causes the breakdown of muscle proteins to free
polyuria and increased concentration of 17- amino acids. In which process will amino acids
ketosteroids in the urine were revealed. What be involved in such conditions?
disease is the most likely? A. Gluconeogenesis in the liver
A. Steroid diabetes B. Muscle glycolysis
B. Insulin-dependent diabetes mellitus C. Synthesis of higher fatty acids
C. Myxoedema D. Glycogenolysis
D. Type I glycogenosis E. Decarboxylation
E. Addison’s disease
28. A 19-year-old male was found to have an
24. Examination of a 42 year old patient elevated level of potassium in the secondary
revealed a tumour of adenohypophysis. urine. These changes might have been caused
Objectively: the patient’s weight is 117 kg, he by the increase in the following hormone level:
has moon-like hyperemic face, redblue striae of A. Aldosterone
skin distension on his belly. Osteoporosis and B. Oxytocin
muscle dystrophy are present. AP is 210/140 C. Adrenaline
mm Hg. What is the most probable diagnosis? D. Glucagon
A. Cushing’s disease E. Testosterone
B. Cushing’s syndrome
C. Conn’s disease

24
29. A concentrated solution of sodium chloride 34. A humans are reduced diuresis,
was intravenously injected to ananimal. This hypernatremia and hypokalemia. What hormone
caused decreased reabsorption of sodium ions in hypersecretion can cause such changes?
the renal tubules. It is the result of the following A. Aldosterone
changes of hormonal secretion: B. Vasopressin
A. Aldosterone reduction C. Atrial Natriuretic Factor
B. Aldosterone increase D. Adrenaline
C. Vasopressin reduction E. Parathormone
D. Vasopressin increase
E. Reduction of atrial natriuretic factor 35. Atria of an experimental animal were
superdistended by blood that resulted in
30. A patient with adenoma of glomeral zone of decreased reabsorption of Na+ and water in
adrenal cortex (Conn’s disease) has arterial renal tubules. This can be explained by the
hypertension, convulsions, polyuria. What is the influence of the following factor upon kidneys:
main link in pathogenesis of these disorders? A. Natriuretic hormone
A. Aldosterone hypersecretion B. Aldosterone
B. Aldosterone hyposecretion C. Renin
C. Catecholamine hypersecretion D. Angiotensin
D. Glucocorticoid hypersecretion E. Vasopressin
E. Glucocorticoid hyposecretion
36. A patient has insufficient blood supply to
31. After a traffic accident a man presents with the kidneys, which has caused the development
severe blood loss, consciousness disturbance, of pressor effect due to the constriction of
low blood pressure, as well as compensatory arterial resistance vessels. This is the result of
activation of the reninangiotensin system, which the vessels being greately affected by the
results in: following substance:
A. Hyperproduction of aldosterone A. Angiotensin II
B. Increased blood coagulation B. Angiotensinogen
C. Intensification of erythropoiesis C. Renin
D. Hyperproduction of vasopressin D. Catecholamines
E. Intensification of heart contractions E. Norepinephrine

32. A patient has hyperkaliemia and 37. Examination of a patient with high blood
hyponatremia. Reduced secretion of what pressure showed him secondary arterial
hormone may cause such changes? hypertension. The cause of this condition is a
A. Aldosterone renin-producing tumor of the kidney. What is
B. Vasopressin the main link in the pathogenesis of secondary
C. Cortosol arterial hypertension in a patient?
D. Parathormone A. Angiotensin 2, aldosterone hyperproduction
E. Natriuretic hormone B. Cortisol hyperproduction
C. Insulin hyperproduction
33. People adapted to high external D. Insufficient vasopressin production
temperatures have such pecularity: profuse E. Insufficient catecholamines production
sweating isn’t accompanied by loss of large
volumes of sodium chloride.This is caused by 38. A patient with cirrhosis of the liver has
the effect of the following hormone upon the sustained arterial hypotension (BP is 90/50 mm
perspiratory glands: Hg). What is the reason for lowering blood
A. Aldosterone pressure in this pathology of the liver?
B. Vasopressin A. Reduced angiotensin synthesis
C. Cortisol B. Increased synthesis of Na-uretic hormone
D. Tgyroxin C. Excessive inactivation of vasopressin
E. Natriuretic D. Increasing the reflex influence from the
receptor zone of the aortic arch

25
E. Activation of the kallikrein-kinin system
44. Indirect calorimetry allowed to establish that
39. A patient with kidney disease has high blood a 30-year-old male patient had a 30% decrease
pressure, espesially the diastolic one. in basal metabolic rate. This might be caused by
Hypersecretion of what biologically active the reduced concentration of the following
substance causes blood pressure rise? hormones in blood plasma:
A. Renin A. Triiodothyronine, tetraiodothyronine
B. Adrenaline B. Thyrocalcitonin, parathormone
C. Noradrenaline C. Glucocorticoids
D. Vasopressin D. Catecholamines
E. Catecholamines E. Somatoliberin, somatostatin

40. Under the experimental conditions, a rabbit 45. A 37-year-old patient has has lost 5 kg in
was tied up the renal artery, it was resulted in a weight over the past three months, he complains
significant increase in blood pressure after 2 of hand tremor, excessive sweating,
weeks. As a result of an increase in the secretion exophthalmos, tachycardia. These changes
of a biologically active substance, this might have been caused by the increased
happened? secretion of the following hormone:
A. Renin A. Thyroxine
B. Adrenaline B. Cortisol
C. Vasopressin C. Insulin
D. Norepinephrine D. Glucagon
E. Natriuretic hormone E. Thyrocalcitonin

41. A patient who had been continuously taking 46. A 38-year-old woman complains of
drugs blocking the production of angiotensin II excessive sweating, palpitations, fever in the
developed bradycardia and arrhythmia. A likely evening hours. The main exchange is increased
cause of these disorders is: by 60%. The doctor decided the diagnosis -
A. Hyperkalemia thyrotoxicosis. What properties of thyroxin lead
B. Hypokalemia to increased heat production?
C. Hypernatremia A. Separates oxidative phosphorylation
D. Hypocalcemia B. Increases conjugation of oxidation and
E. Hypercalcemia phosphorylation
C. Reduces β-oxidation of fatty acids
42. A patient with kidney disease has high blood D. Reduces amino acid deamination
pressure, espesially the diastolic one. E. Promotes acetyl-coA accumulation
Hypersecretion of what biologically active
substance causes blood pressure rise? 47. A 40-year-old woman suffering from diffuse
A. Renin toxic goiter presents with constant increase of
B. Adrenaline her body temperature. What mechanism results
C. Noradrenaline in such clinical presentation?
D. Vasopressin A. Separation of oxidation and phosphorization
E. Catecholamines in cell mitochondria
B. Increased breakdown of glycogen inhepatic
43. A 40-year-old woman on examination cells
presents with intensified basal metabolic rate. C. Increased catabolism of protein incells
What hormone present in excess leads to such D. Increased excitability of nerve cells
condition? E. Increased cell sensitivity to catecholamines
A. Triiodothyronine
B. Thyrocalcitonin 48. A patient is followed up in an
C. Glucagon endocrinological dispensary on account of
D. Aldosterone hyperthyreosis. Weight loss, tachycardia, finger
E. Somatostatin tremor are accompanied by hypoxia symptoms -

26
headache, fatigue, eye flicker. What mechanism 53. A doctor has established significant growth
of thyroid hormones action underlies the retardation, disproportional body build, and
development of hypoxia? mental deficiency of a child. What is the most
A. Disjunction, oxydation and phosphorilation likely cause of this pathology?
B. Inhibition of respiratory ferment synthesis A. Hypothyroidism
C. Competitive inhibition of respiratory B. Insufficient nutrition
ferments C. Hyperthyroidism
D. Intensification of respiratory ferment D. Genetic defects
synthesis E. Hypopituitarism
E. Specific binding of active centres of
respiratory ferments 54. A 12-year-old child is of short stature, has
disproportionate body structure and mental
49. A 56 y.o. patient has been suffering from retardation. These characteristics might be
thyreotoxicosis for a long time.What type of caused by the hyposecretion of the following
hypoxia can be developed? hormone:
A. Tissue A. Thyroxine
B. Hemic B. Insulin
C. Circulatory C. Cortisol
D. Respiratory D. Somatotropin
E. Mixed E. Glucagon

50. The patient mistakenly took an excess dose 55. Inhabitants of territories with cold climate
of thyroxine. What changes in secretion of have high content of an adaptive
thyroliberin and thyrotropin will it lead to? thermoregulatory hormone. What hormone is
A. Secretion of hormones will decrease meant?
B. Secretion of hormones will increase. A. Thyroxin
C. Secretion of hormones will not change B. Insulin
D. Secretion of thyroliberin will increase, C. Glucagon
thyrotropin will decrease D. Somatotropin
E. Secretion of thyrotropin will increase, E. Cortisol
thyroliberin will decrease
56. The clinical examination of the patient
51. A 19-year-old female suffers from revealed an increase in the thyroid gland
tachycardia in rest condition, weight loss, (goiter), an increase in basal metabolism, loss of
excessive sweating, exophtalmos and body weight, an imbalance of heat, an increase
irritability. What hormone would you expect to in appetite, an increase in excitability and
find elevated in her serum? irritability, exophthalmos and tachycardia were
A. Thyroxine detected. What endocrine disruption leads to
B. Cortisol these symptoms?
C. Mineralocorticoids A. Hyperfunction of the thyroid gland
D. ACTH B. Hypofunction of the parathyroid glands
E. Insulin C. Pituitary Hyperfunction
D. Epiphysis hypofunction
52. A child presents with symptoms of psychic E. Hypofunction of the thyroid gland
and physical retardation (cretinism). It is usually
associated with the following hormone 57. It is known that steroid anti-inflammatory
deficiency: drugs inhibit the activity of phospholipase A2,
A. Thyroxin which is necessary for the synthesis of
B. Somatotropic prostaglandins. What substance is the precursor
C. Calcitonin of these inflammatory mediators?
D. Insulin A. Arachidonic acid
E. Testosterone B. Cholesterol
C. Tyrosine

27
D. Proopiomelanocortin 59. Utilization of arachidonic acid via
E. Palmitic acid cyclooxigenase pathway results in formation of
some bioactive substances. Name them:
58. A patient is followed up in the clinic on A. Prostaglandins
account of for pneumonia complicated by B. Thyroxine
pleurisy. He was given prednisolon as part of a C. Biogenic amins
combination therapy. The anti-inflammatory D. Somatomedins
effect of this synthetic glucocorticoid is E. Insulin-like growth factors
associated with blocking the release of
arachidonic acid by inhibiting: 60. Experimental studies revealed steroid
A. Phospholipase A2 hormones to have an effect on proteosynthesis.
B. Cyclo-oxygenase They influence synthesis of the following
C. Phospholipase C substances:
D. Lipoxygenase A. Specific messenger RNA
E. Peroxidase B. Adenosine triphosphate
C. Specific transfer RNA
D. Guanosine triphosphate
E. Specific ribosomal RNA

28
Biochemistry and pathobiochemistry of blood
1. A 7-year-old girl has signsof anemia. E. -
Laboratory examination revealed pyruvate
kinase deficiency in erythrocytes. What process 6. Sulfanilamides are applied as antimicrobal
disturbance plays the main role in agents in clinical practice.Sulfanilamide
anemiadevelopment? treatment, however, canresult in hemolytic
A. Anaerobic glycolysis anemia developmentin patients that suffer from
B. Oxidative phosphorylation geneticdefect of the following enzyme
C. Tissue respiration ofpentose phosphate metabolism inerythrocytes:
D. Peroxide decomposition A. Glucose-6-phosphate dehydrogenase
E. Aminoacids desamination B. Hexokinase
C. Transketolase
2. Human red blood cells contain no D. Transaldolase
mitochondria. What is the main pathway E. Pyruvate kinase
forATP production in these cells?
A. Anaerobic glycolysis 7. Biochemical analysis of an
B. Aerobic glycolysis infant’serythrocytes revealed evident
C. Oxidative phosphorylation glutathioneperoxidase deficiency and low
D. Creatine kinase reaction concentration of reduced glutathione. What
E. Cyclase reaction pathological condition can develop in this
infant?
3. Erythrocytes of the patient withhemolytic A. Hemolytic anemia
anemia present with significantdecrease of B. Pernicious anemia
pyruvate kinase activity. Whatmethabolic C. Megaloblastic anemia
process is disturbed in thiscase? D. Sicklemia
A. Glycolysis E. Iron-deficiency anemia
B. Glycogenolysis
C. Gluconeogenesis 8. A 20 year old patient complains ofgeneral
D. Pentose-phosphate pathway of weakness, dizziness, quick fatigability. Blood
glucoseoxidation analysis results: Hb-80g/l. Microscopical
E. Glycogen synthesis examination results:erythrocytes are of modified
form. Thiscondition might be caused by:
4. A 3 year old child with fever wasgiven A. Sickle-cell anemia
aspirin. It resulted in intensified erythrocyte B. Hepatocellular jaundice
haemolysis. Hemolyticanemia might have been C. Acute intermittent porphyria
caused bycongenital insufficiency of the D. Obturative jaundice
followingenzyme: E. Addison’s disease
A. Glucose 6-phosphate dehydrogenase
B. Glucose 6-phosphatase 9. With a number of hemoglobinopathies, amino
C. Glycogen phosphorylase acid substitutions occur in the α- and β-chains of
D. Glycerol phosphate dehydrogenase hemoglobin. Which of them is characteristic of
E. γ-glutamiltransferase Hb S (sickle cell anemia)?
A. Glutamate-valine
5. There is an increased tendency of B. Aspartate-lysine
erythrocytes to hemolysis in patients with a C. Alanin-serine
hereditary defect of glucose-6-phosphate D. Methionine-histidine
dehydrogenase. What metabolic process is E. Glycine-serine
impaired under these conditions?
A. Pentose phosphate pathway of glucose 10. Substitution of the glutamic acid onvaline
oxidation was revealed while examining initial molecular
B. Gluconeogenesis structure. For what inheritedpathology is this
C. Aerobic glucose oxidation symptom typical?
D. Synthesis of glycogen A. Sickle-cell anemia
29
B. Thalassemia vitamin deficiency leads to such changes in
C. Minkowsky-Shauffard disease blood?
D. Favism A. B12
E. Hemoglobinosis B. C
C. P
11. A patient suffers from mutation of a gene D. B6
that corresponds with hemoglobin synthesis. E. PP
This condition led to development of sicklecell
disease. Name the pathologicalhemoglobin 16. A 43-year-old patient with chronic atrophic
characteristic of this disease: gastritis and megaloblastic hyperchromic
A. HbS anemia excreted methylmalonic acid in the
B. HbA urine. What vitamin deficiency is caused by the
C. HbF occurrence of the specified symptom complex?
D. HbA1 A. В12
E. Bart-Hb B. В2
C. В3
12. Along with normal hemoglobin typesthere D. В5
can be pathological ones in the organism of an E. В6
adult. Name one of them:
A. HbS 17. A pregnant woman turned to the
B. HbF obstetrician-gynecologist, who was diagnosed
C. HbA1 with megaloblastic anemia. Which of the
D. HbA2 following drugs should be prescribed?
E. HbO2 A. Cyanocobalamin
B. Pentoxyl
13. After an extended treatment with C. Methyluracil
sulfanamides a patient has developedmacrocytic D. Glaucin
anemia. Production of activeforms of the E. Streptokinase
following vitamin is disruptedin such a
condition: 18. The patient was diagnosed with
A. Folic acid megaloblastic anemia. Specify a vitamin
B. Thiamine deficiency which can lead to the development of
C. Riboflavin this disease.
D. Pyridoxine A. Cyanocobalamin.
E. Cyanocobalamin B. Rutin.
C. Nicotinamide.
14. A 50-year-old patient has beenexamined by D. Thiamine.
a dentist and found tohave crimson smooth E. Cholecalciferol
tongue. Bloodanalysis revealed a decrease in
RBC leveland hemoglobin concentration, 19. In a 65-year-old patient with prolonged
symptoms of megaloblastichematopoiesis, complaints characteristic of chronic gastritis,
degenerative changes inWBCs. What blood megalocytes were found in peripheral blood,
disorder was found inthis patient? and megaloblastic erythropoiesis was found in
A. B12-folic-acid-deficiency anemia the bone marrow. What is the most likely
B. Iron deficiency anemia diagnosis?
C. Myeloid leukemia A. B12-folic deficiency anemia
D. Aplastic anemia B. Aplastic anemia
E. Hemolytic anemi C. Hypoplastic anemia
D. Hemolytic anemia
15. After removal in the patient 2/3 of the E. Iron deficiency anemia
stomach in the blood decreased the amount of
hemoglobin, the number of erythrocytes, A patient who had subtotal gastrectomy 5 years
increased the size of these blood counts. What ago developed B12 folic acid deficiency

30
anemia. What is the leading mechanism in the hemolysis of red blood cells and
development of such anemia? hemoglobinuria were detected. The action of
A. The absence of an internal factor Castle snake venom due to the presence of the enzyme
B. The absence of an external factor Castle in it:
C. Impaired absorption of vitamin B12 in the A. Phospholipase A2
small intestine B. Phospholipase A1
D. Folic acid deficiency C. Phospholipase C
E. Transcobalamin deficiency. D. Phospholipase D
E. Sphingomyelinase
20. Examination of a patient, suffering from
atrophic gastritis, revealedmegaloblastic 25. For people who permanently reside in highlands,
anemia. The anemia is likely to be caused by the adaptation to "oxygen starvation" is carried out by
deficiency of the following substance: facilitating the release of hemoglobin oxygen due to:
A. Gastromucoproteid A. Increased formation of 2,3-diphosphoglycerate in
B. Vitamin B6 erythrocytes
B. Reduced formation of 2,3-diphosphoglycerate in
C. Vitamin B1 erythrocytes
D. Iron C. The increase in CO2 partial pressure
E. Erythropoietins D. Increasing blood pH
E. Decreased blood temperature
21. Surgical removal of a part of
stomachresulted in disturbed absorption of 26. After the accident in the chemical industry,
vitamin B12, it is excreted with feces. the environment was polluted with nitro
Thepatient was diagnosed with anemia. compounds. The people living in this area, there
Whatfactor is necessary for absorption of was a sharp weakness, headache, shortness of
thisvitamin? breath, dizziness. What is the cause of hypoxia?
A. Gastromucoprotein A. Methemoglobin formation
B. Gastrin B. Inhibition of dehydrogenase
C. Hydrochloric acid C. Formation of carboxyhemoglobin
D. Pepsin D. Reduced function of flavin enzymes
E. Folic acid E. Inactivation of cytochrome oxidase

22. Examination of a 52-year-oldfemale patient 27. A 7-year-old child presents with marked
has revealed a decreasein the amount of red signs of hemolytic anemia. Biochemical
blood cells andan increase in free hemoglobin analysis of erythrocytes determined low
inthe blood plasma (hemoglobinemia).What concentration of NADPH and reduced
type ofanemia is being observed in the patient? glutathione. What enzyme is deficient in this
A. Acquired hemolytic case leading to the biochemical changes and
B. Hereditary hemolytic their clinical manifestations?
C. Acute hemorrhagic A. Glucose-6-phosphate dehydrogenase
D. Chronic hemorrhagic B. Hexokinase
E. Anemia due to diminishederythropoiesis C. Fructokinase
D. Pyruvate kinase
23. A patient with hemolytic anemia showed E. Lactate dehydrogenase
pyruvate kinase deficiency in erythrocytes. In these
conditions, the cause of hemolysis of red blood cells
28. A 38 year old patient takes aspirin and
are:
sulfanilamides. After their intake intensified
A. Reduced Na+, K+ -ATPases
B. Na+ deficiency in red blood cells erythrocyte haemolysis is observed which is
C. Excess K+ in red blood cells caused by deficiency of glucose 6-phosphate
D. Genetic defects of glycophorin A dehydrogenase. This pathologyis caused by
E. Spectrin deficiency failure of the following coenzyme:
A. NADP – H
24. During laboratory examination of the blood B. FAD - H2
of a person who was bitten by a snake, C. Pyridoxal phosphate

31
D. FMN - H2 B. Cytochrome oxidase
E. Ubiquinone C. Succinate dehydrogenase
D. Ketoglutarate dehydrogenase
29. A 22 year old woman has been taking E. Aconitase
sulfanilamides for a long time that led to
symptoms of hemolytic anaemia caused by 34. When periodontitis develops lipid
hereditary disturbance of synthesis of glucose 6- peroxidation in periodontal tissues, the content
phosphate dehydrogenase. This enzyme of of malondialdehyde, hydrogen peroxide
pentose-phosphate cycleis responsible for increases in the oral cavity. Which of the
generation of: following enzymes provide antioxidant
A. NADP - H2 protection?
B. NAD A. Superoxide dismutase, catalase
C. FAD B. Amylase, trypsin
D. FMN C. Maltase, chymotrypsin
E. ATP D. Lactase, lysozyme
E. Sucrase, prothrombin
30. It is known that pentose-phosphate pathway
actively functions in the erythrocytes. What is 35. Periodontitis induces the development of
the main function of this metabolic pathway in lipid peroxidation in the periodontal tissues, as
the erythrocytes? well as an increase in malondialdehyde and
A. Counteraction to lipid peroxidation hydrogen peroxide concentration in the oral
B. Activation of microsomal oxidation cavity. Which of the following enzymes
C. Neutralization of xenobiotics provides antioxidant protection?
D. Oxidation of glucose into lactate A. Catalase
E. Increase of lipid peroxidation B. Amylase
C. Maltase
31. In course of metabolic process active forms D. Lactase
of oxygen including superoxide anion radical E. Invertase
are formed in the human body. By means of
what enzyme is this anion inactivated? 36. Those organisms which in the process of
A. Superoxide dismutase evolution failed to develop protection from
B. Catalase H2O2 can exist only in anaerobic conditions.
C. Peroxidase Which of the following enzymes can break
D. Glutathioneperoxidase hydrogen peroxide down?
E. Glutathionereductase A. Peroxidase and catalase
B. Oxygenase and hydroxylase
32. Reduced activity of antioxidant enzymes C. Cytochrome oxidase, cytochrome B5
enhances peroxidation of cell membrane lipids. D. Oxygenase and catalase
The reduction of glutathione peroxidase activity E. Flavin-dependent oxidase
is caused by the following microelement
deficiency: 37. Patient with abscess of the cut wound
A. Selenium applied to the traumatological department. In
B. Molybdenum order to clean the wound from the pus doctor
C. Cobalt washed it with 3% hydrogen peroxide. Foam
D. Manganese was absent. What caused the absence of the
E. Copper drug activity?
A. Inherited insufficiency of catalase
33. In pathological processes accompanied by B. Low concentration H2O2
hypoxia, an incomplete reduction of the oxygen C. Inherited insufficiency of erythrocyte
molecule in the respiratory chain and phosphatdehydrogenase
accumulation of hydrogen peroxide occurs. D. Shallow wound
Specify the enzyme that ensures its destruction: E. Pus in the wound
A. Catalase

32
38. A worker has decreased buffer capacity of B. Ketone bodies
blood due to exhausting muscular work. The C. Acetyl CoA
influx of what acid substance in the blood can D. Glucose 6-phosphate
cause this symptom? E. Oxaloacetate
A. Lactate
B. Pyruvate 43. A 32-year-old female patient suffers from
C. 1,3-bisphosphoglycerate gingivitis accompanied by gum hypoxia. What
D. α-ketoglutarate metabolite of carbohydrate metabolism is
E. 3-phosphoglycerate produced inthe periodontium tissues more
actively in this case?
39. A child during the first 3 months after birth A. Lactate
developed a severe form of hypoxia, which was B. Ribose 5-phosphate
manifested by asphyxia and cyanosis of skin. C. Glycogen
The reason for this is a violation of the D. Glucose 6-phosphate
replacement of fetal hemoglobin on: E. NADPH-H
A. Hemoglobin A
B. Hemoglobin S 44. A 29-year-old patient was delivered to a
C. Glycated hemoglobin hospital because of intoxication with carbon
D. Methemoglobin monoxide. Objectively: the patient presents with
E. Hemoglobin M symptoms of severe hypoxia- evident dyspnea,
cyanosis, tachycardia. What compound is
40. Diseases of the respiratory system and produced as a result of intoxication with carbon
circulatory disorders impair the transport of monooxide?
oxygen, thus leading to hypoxia. Under these A. Carboxyhemoglobin
conditions the energy metabolism is carried out B. Methemoglobin
byanaerobic glycolysis. As a result, the C. Carbhemoglobin
following substance is generated and D. Sulfhemoglobin
accumulated in blood: E. Oxyhemoglobin
A. Lactic acid
B. Pyruvic acid 45. A man lost consciousness in a car with
C. Glutamic acid running engine where he had been waiting for a
D. Citric acid friend for a long time. What hemoglobin
E. Fumaric acid compaund can be found in the blood of the
patient?
41. A patient is followed up in an A. Carboxyhemoglobin
endocrinological dispensary on account of B. Deoxyhemoglobin
hyperthyreosis. Weight loss, tachycardia, finger C. Carbhemoglobin
tremor are accompanied by hypoxia symptoms - D. Methemoglobin
headache, fatigue, eye flicker. What mechanism E. Oxyhemoglobin
of thyroid hormones action underlies the
development of hypoxia? 46. A patient with respiratory failure has blood
A. Disjunction oxydation and phosphorilation pH of 7,35. pCO2 test revealed hypercapnia.
B. Inhibition of respiratory fermentsynthesis Urine pH test revealed an increase in the urine
C. Competitive inhibition of respiratoryferments acidity. What form of acid-base imbalance is the
D. Intensification of respiratory case?
fermentsynthesis A. Compensated respiratory acidosis
E. Specific binding of active centres B. Compensated metabolic acidosis
ofrespiratory ferments C. Decompensated metabolic acidosis
D. Compensated respiratory alkalosis
42. After a sprint an untrained person develops E. Decompensated respiratory alkalosis
muscle hypoxia. This leads to the accumulation
of the following metabolite in muscles: 47. An infant has apparent diarrhea resulting
A. Lactate from improper feeding. One of the main

33
diarrhea effects is plentiful excretion of sodium 52. Prophylactic examination of a patient
bicarbonate. What form of acid-base balance revealed hyperglycemia, ketonuria, polyuria,
disorder is the case? glycosuria. What form of acid-base balance
A. Metabolic acidosis disorder is the case?
B. Metabolic alkalosis A. Metabolic acidosis
C. Respiratory acidosis B. Gaseous acidosis
D. Respiratory alkalosis C. Nongaseous acidosis
E. No disorders of acid-base balance will be D. Gaseous alkalosis
observed E. Metabolic alkalosis

48. A 30-year-old man with diabetes mellitus 53. A patient with diabetes developed a diabetic
type I was hospitalised. The patient is comatose. coma due to the acid-base imbalance. Specify
Laboratory tests revealed hyperglycemia and the kind of this imbalance:
ketonemia. What metabolic disorder can be A. Metabolic acidosis
detected in this patient? B. Metabolic alkalosis
A. Metabolic acidosis C. Respiratory acidosis
B. Metabolic alkalosis D. Gaseous alkalosis
C. Respiratory acidosis E. Non-gaseous alkalosis
D. Respiratory alkalosis
E. Normal acid-base balance 54. Ketoacidosis that develops due to
accumulation of ketone bodies in blood serum is
49. A patient suffers from disrupted patency of a primary complication of diabetes mellitus.
the airways at the level of small and medium- What acid-base disbalance develops during this
sized bronchial tubes. What changes of acid- condition?
base balance can occur in the patient? A. Metabolic acidosis
A. Respiratory acidosis B. Metabolic alkalosis
B. Respiratory alkalosis C. Respiratory acidosis
C. Metabolic acidosis D. Respiratory alkalosis
D. Metabolic alkalosis E. –
E. Acid-base balance remains unchanged
55. Due to recurring vomiting a patient has lost
50. Diabetes mellitus causes ketosis as a result significant amount of gastric juice, which led to
of activated oxidation of fatty acids. What development of acidbase dysbalance. What type
disorders of acid-base equilibrium may be of acid-base dysbalance has developed?
caused by excessive accumulation of ketone A. Nongaseous alkalosis
bodies in blood? B. Gaseous acidosis
A. Metabolic acidosis C. Nongaseous acidosis
B. Metabolic alcalosis D. Gaseous alkalosis
C. Any changes woun’t happen E. Metabolic acidosis
D. Respiratory acidosis
E. Respiratory alcalosis 56. A mountaineer who rose to a height of
5200m developed gas alkalosis. What is the
51. Ketosis develops in the patients with cause of its development?
diabetes mellitus, as the result of activation of A. Hyperventilation
fatty acids oxidation processes. What acidbase B. Hypoventilation of the lungs
imbalance can result from accumulation of C. Hyperoxemia
excessive ketone bodies in the blood? D. Hypoxemia
A. Metabolic acidosis E. Reduced ambient temperature
B. Metabolic alkalosis
C. No imbalance occurs 57. In a laboratory study of the respiratory
D. Respiratory acidosis function of the blood, it has been established
E. Respiratory alkalosis that there is a deterioration of CO2 transport.

34
What enzyme deficiency in red blood cells can examination revealed anaemia and presence of
this be due to? the paraprotein in the zone of gamma-globulins.
A. Carboanhydrase To confirm the myeloma diagnosis it is
B. 2,3-diphosphoglycerate necessary to determine the following index in
C. Adenylate cyclase the patient’s urine:
D. Protein kinase A. Bence Jones protein
E. Phosphorylase B. Bilirubin
C. Haemoglobin
58. In the process of hemoglobin catabolismiron D. Ceruloplasmin
is released and then as a part of special transport E. Antitrypsin
protein is returned to the bonemarrow, to be
used again for hemoglobinsynthesis. Name this 63. A 16-year-old girl, who has been starving
transport protein: herself for a long time to lose weight, developed
A. Transferrin an edema. This phenomenon is mainly caused
B. Transcobalamin by:
C. Haptoglobin A. Hypoproteinemia due to protein synthesis
D. Ceruloplasmin disturbance
E. Albumin B. Hypoglycemia due to glycogen synthesis
disturbance
59. Hemoglobin catabolism results in release of C. Venous congestion and increased venous
iron which is transported to the bone marrow by pressure
a certain transfer protein and used again for the D. Deceleration of glomerular filtration rate
synthesis of hemoglobin. Specify this transfer E. Decreased production of vasopressin in the
protein: hypothalamus
A. Transferrin (siderophilin)
B. Transcobalamin 64. A 36-year-old female patient who has been
C. Haptoglobin limiting the number of food stuffs in her diet for
D. Ceruloplasmin 3 months presents with a decrease in body
E. Albumin weight, deterioration of physical and mental
health, face edemata. These changes may be
60. In the liver of a patient suffering from iron caused by the deficiency of the following
deficiency anemia, a violation of the synthesis nutrients:
of iron-containing protein, which is a source of A. Proteins
iron for heme synthesis, was found. What is the B. Vitamins
name of this protein? C. Fats
A. Ferritin D. Carbohydrates
B. Transferrin E. Micronutrients
C. Hemosiderin
D. Ceruloplasmin 65. Upon toxic damage of hepatic cells resulting
E. Hemoglobin in disruption of liver function the patient
developed edemas. What changes of blood
61. In men of 40 years as a result of enhanced plasma are the main cause of edema
hemolysis of erythrocytes, the iron content in development?
the blood plasma increased. What protein A. Decrease of albumin content
provides its deposition in the tissues? B. Increase of globulin content
A. Ferritin C. Decrease of fibrinogen content
B. Haptoglobin D. Increase of albumin content
C. Transferrin E. Decrease of globulin content
D. Transcortin
E. Albumin 66. In case of toxic damage to hepatocytes with
a violation of their protein synthesis function,
62. A patient complains about dyspnea the patient's content of albumin in the blood
provoked by the physical activity. Clinical plasma and oncotic pressure of the plasma

35
sharply decreased. What will be the result of A. γ-globulins
these changes? B. Albumins
A. The appearance of edema C. α1-globulins
B. Reduced diuresis D. α2-globulins
C. Reduced ESR E. β-globulins
D. Increased circulating blood volume
E. Increased blood viscosity 72. When the inflammatory process is activated,
some autoimmune and infectious diseases in the
67. Toxic affection of liver results in blood plasma sharply increase the level of
dysfunction of protein synthesis. It is usually proteins of the acute phase. Which of the
accompanied by the following kind of following proteins can form a gel when the
dysproteinemia: whey cools?
A. Absolute hypoproteinemia A. Cryoglobulin
B. Relative hypoproteinemia B. Haptoglobin
C. Absolute hyperproteinemia C. Ceruloplasmin
D. Relative hyperproteinemia D. C-reactive protein
E. Paraproteinemia E. α2 macroglobin

68. The concentration of albumins in human 73. A 49-year-old male patient with acute
blood sample is lower than normal. This leads to pancreatitis was likely to develop pancreatic
edema of tissues. What blood function is necrosis, while active pancreatic proteases were
damaged? absorbed into the blood stream and tissue
A. Maintaining the oncotic blood pressure proteins broke up. What protective factors of the
B. Maintaining the Ph level body can inhibit these processes?
C. Maintaining the body temperature A. α2-macroglobulin, α1-antitrypsin
D. Maintaining the blood sedimentation system B. Immunoglobulin
E. All answers are correct C. Cryoglobulin, interferon
D. Ceruloplasmin, transferrin
69. A 4 y.o. child with signs of durative E. Hemoplexin, haptoglobin
proteinic starvation was admitted to the hospital.
The signs were as follows: growth inhibition, 74. Wilson’s disease is a disorder of copper
anemia, edemata, mental deficiency. Choose a transport which leads to the accumulation of this
cause of edemata development: metal in brain and liver cells. It is associated
A. Reduced synthesis of albumins with a disturbance in the synthesis of the
B. Reduced synthesis of globulins following protein:
C. Reduced synthesis of hemoglobin A. Ceruloplasmin
D. Reduced synthesis of lipoproteins B. Metallothionein
E. Reduced synthesis of glycoproteins C. Transcobalamin
D. Haptoglobin
70. The prolonged action of a number of E. Siderophilin
antibiotics and sulfonamides is due to the fact
that they circulate in the blood for a long time in 75. Examination of a 27-year-old patient
combination with: revealed pathological changes in liver and brain.
A. Albumin Blood plasma analysis revealed an abrupt
B. Transferrin decrease in the copper concentration, urine
C. Hemoglobin analysis revealed an increased copper
D. Haptoglobin concentration. The patient was diagnosed with
E. Hemopexin Wilson’s degeneration. To confirm the
diagnosis it is necessary to study the activity of
71. Electrophoretic study of a bloodserum the following enzyme in blood serum:
sample, taken from the patient with pneumonia, A. Ceruloplasmin
revealed an increase in one of the protein B. Carbonic anhydrase
fractions. Specify this fraction: C. Xanthine oxidase

36
D. Leucine aminopeptidase B. Prothrombin
E. Alcohol dehydrogenase C. Fibrinogen
D. G immunoglobulin
76. Biochemical analysis of the blood serum of E. A immunoglobulin
a patient with hepatolenticular degeneration
(Wilson-Konovalov disease) revealed a 81. A 60-year-old man complains of joint pain.
decrease in the content of ceruloplasmin. In this An increase in the concentration of C-reactive
patient, the concentration of such ions in the protein and hydroxyproline was detected in the
serum will be increased: patient's serum. What disease are these
A. Copper symptoms characteristic of?
B. Calcium A. Rheumatism
C. Phosphorus B. Gout
D. Potassium C. Hepatitis
E. Sodium D. Jaundice
E. Diabetes
77. A patient suffering from hepatocerebral
degeneration has low concentration of 82. A 6-month-old child experienced frequent
ceruloplasmin in blood serum. What element and intense subcutaneous hemorrhages. The
accumulation will be observed in liver, appointment of a synthetic analogue of vitamin
cerebrum and kidneys of the patient? K (vikasol) gave a positive effect. In the γ-
A. Cuprum carboxylation of glutamic acid, what protein of
B. Calcium the blood coagulation system is this vitamin
C. Sodium taking part in?
D. Potassium A. Prothrombin
E. Ferrum B. Fibrinogen
C. Hageman factor
78. A 33-year-old woman suffers from D. Antihemophilic globulin A
hepatocerebral dystrophy (Wilson's disease). In E. Rosenthal factor
the blood - low content of ceruloplasmin. In the
urine - sharply elevated amino acids. What 83. After implantation of a cardiac valve a
process has increased these changes? young man constantly takes indirect
A. Complexation of amino acids with copper anticoagulants. His state was complicated by
B. Urea synthesis hemorrhage. What substance content has
C. Transamination of amino acids decreased in blood?
D. Disintegration of tissue proteins A. Prothrombin
E. Gluconeogenesis B. Haptoglobin
C. Heparin
79. A 38 year old patient suffers from D. Creatin
rheumatism in its active phase. What laboratory E. Ceruloplasmin
characteristic of blood serumis of diagnostic
importance in case of this pathology? 84. The patient complains of frequent bleeding
A. C-reactive protein from the gums. A deficiency of coagulation
B. Uric acid factor II (prothrombin) was found in the blood.
C. Urea What phase of blood coagulation is impaired in
D. Creatinine a person, above all?
E. Transferrin A. Thrombin formation
B. The formation of prothrombinase
80. Blood plasma of a healthy man contains C. Formation of fibrin
several dozens of proteins. During an illness D. Fibrinolysis
new proteins can originate, namely the protein E. Clot retraction
of "acute phase". Select such protein from the
listed below:
A. C-reactive protein

37
85. Inflammatory processes cause synthesis of 90. The student used canned donor blood to
protein of acute phase in an organism. What determine the time it was collected. However,
substances stumulate their synthesis? he could not get any positive result. The reason
A. Interleukin-1 for this is the lack of blood:
B. Immunoglobulins A. Ionized calcium
C. Interferons B. Hageman factor
D. Biogenic amins C. Thromboplastin
E. Angiotensin D. Fibrinogen
E. Vitamin K.
86. Pyrogenal administered to a rabbit, in the
course of an experiment, resulted in increase of 91. A 16 year old boy after an illness has
its body temperature. What substance of those diminished function of protein synthesis in liver
named below acts as a secondary pyrogen that is as a result of vitamin K deficiency. It will cause
a part of fever inducing mechanism? disturbance of:
A. Interleukin 1 A. Blood coagulation
B. Pseudomonas polysaccharide (Piromen) B. Erythrocyte sedimentation rate
C. Histamine C. Anticoagulant generation
D. Bradykinin D. Erythropoietin secretion
E. Immunoglobulin E. Osmotic blood pressure

87. A patient who suffers from pneumonia has 92. The patient has hemorrhages, the
high body temperature. What biologically active concentration of prothrombin is reduced in the
substance plays the leading part in origin of this blood. What vitamin deficiency led to a
phenomenon? violation of the synthesis of this clotting factor?
A. Interleukin-I A. K
B. Histamine B. A
C. Bradykinin C. D
D. Serotonin D. C
E. Leukotrienes E. E

88. After transfusion of 200 ml of blood apatient 93. A 37-year-old patient, with long-term use of
presented with body temperature rise up to antibiotics, has increased bleeding with minor
37,9oC. Which of the following substances is injuries. In the blood - a decrease in activity II,
the most likely cause of temperature rise? VII, X blood clotting factors; lengthening of
A. Interleukin-1 blood clotting time. What vitamin deficiency
B. Interleukin-2 caused these changes?
C. Tumour necrosis factor A. Vitamin K
D. Interleukin-3 B. Vitamin A
E. Interleukin-4 C. Vitamin C
D. Vitamin D
89. For the development of febrile conditions, E. Vitamin E
an increase in the level of the “acute phase”
proteins of ceruloplasmin, fibrinogen, and C- 94. Plasmic factors of blood coagulationare
reactive protein is characteristic. Specify the exposed to post-translational modification with
possible mechanism of this phenomenon: the participation of vitamin K. It is necessary as
A. Stimulating effect of IL-1 on hepatocytes a cofactor in the enzyme system of γ-
B. The destructive effect of temperature on the carboxylation of protein factors of blood
cells of the body coagulation due to the increased affinity of their
C. Proliferative effect of IL-2 on T-lymphocytes molecules with calcium ions. What amino acid
D. Degranulation of tissue basophils is carboxylated in these proteins?
E. - A. Glutamic
B. Valine
C. Serine

38
D. Phenylalanine likely to have a disorder of the following
E. Arginine biochemical process:
A. Production of gammacarboxyglutamate
95. As a result of posttranslative modifications B. Conversion of homocysteine to methionine
some proteins taking part in blood coagulation, C. Conversion of methylmalonyl CoA to
particularly prothrombin, become capable of succinyl CoA
calcium binding. The following vitamin takes D. Degradation of glutathione
part in this process: E. Hydroxylation of proline
A. K
B. C 100. Activation of a number of hemostatic
C. A factors occurs through their joining with
D. B1 calcium ions. What structural component allows
E. B2 for adjoining of calcium ions?
A. Gamma-carboxyglutamic acid
96. To prevent postoperative bleeding a 6 y.o. B. Gamma-aminobutyric acid
child was administered vicasol that is a C. Gamma-oxybutyric acid
synthetic analogue of vitamin K. Name post- D. Hydroxyproline
translational changes of blood coagulation E. Monoamine-dicarboxylic acids
factors that will be activated by vicasol:
A. Carboxylation of glutamin acid 101. A 46-year-old female patient hasa
B. Phosphorylation of serine radicals continuous history of progressive muscular
C. Partial proteolysis (Duchenne’s) dystrophy. Which blood enzyme
D. Polymerization changes will be of diagnostic value in this case?
E. Glycosylation A. Creatine phosphokinase
B. Lactate dehydrogenase
97. A few days before an operation apatient C. Pyruvate dehydrogenase
should be administered vitamin K or its D. Glutamate dehydrogenase
synthetic analogue Vicasol. Vitamin K takes E. Adenylate cyclase
part in the following posttranslational
modification of the II, VII, IX, X blood clotting 102. A 15-year-old boy has been diagnosedwith
factors: acute viral hepatitis. What blood values hould
A. Carboxylation be determined to confirm acute affection of
B. Decarboxylation hepatic cells?
C. Deamination A. Aminotransferase activity (AST, ALT)
D. Transamination B. Unconjugated and conjugated bilirubin
E. Glycosylation content
C. Erythrocytes sedimentation rate (ESR)
98. A patient, who has been suffering for a long D. Cholesterol content
time from intestine disbacteriosis, has increased E. Protein fraction content
hemorrhaging caused by disruption of
posttranslational modification of blood- 103. 12 hours after an accute attack of
coagulation factors II, VII, IХ, and Х in the retrosternal pain a patient presented a jump of
liver. What vitamin deficiency is the cause of aspartate aminotransferase activity in blood
this condition? serum. What pathology is this deviation typical
A. К for?
B. 12 A. Myocardium infarction
C. 9 B. Viral hepatitis
D. С C. Collagenosis
E. Р D. Diabetes mellitus
E. Diabetes insipidus
99. A newborn baby has numerous
hemorrhages. Blood coagulation tests reveal 104. A 43-year-old man after eating fatty foods
increased prothrombin time. The child is most and alcohol complains of severe abdominal

39
pain. The serum content of trypsin is 850 mmol D. Lungs
/ (h • l) (normal 60-240 mmol / (h • l)). What is E. Spleen
the most characteristic pathology of the
digestive system? 109. In the serum of the patient increased
A. Acute pancreatitis activity of hyaluronidase. Determining what
B. Dynamic intestinal obstruction biochemical index of serum will allow to
C. Mechanical intestinal obstruction confirm the assumption of connective tissue
D. Gastric ulcer pathology?
E. Hepatitis A. Sialic acids
B. Bilirubin
105. For biochemical diagnostics of myocardial C. Uric acid
infarction it is necessary to measure activity of a D. Glucose
number of enzymes and their isoenzymes. What E. Galactose
enzymatic test is considered to be the best to
prove or disprove the diagnosis of infarction in 110. Marked increase of activity of МВ forms
the early period after the chest pain is detected? of CPK (creatinephosphokinase) and LDH-1
A. Creatine kinase isoenzyme CK-MB was revealed by examination of the patient’s
B. Creatine kinase isoenzyme CK-MM blood. What is the most probable pathology?
C. LDH1 lactate dehydrogenaseisoenzyme A. Miocardial infarction
D. LDH2 lactate dehydrogenaseisoenzyme B. Hepatitis
E. Aspartate aminotransferasecytoplasmic C. Rheumatism
isoenzyme D. Pancreatitis
E. Cholecystitis
106. An increase in the activity of LDH4,5,
ALT, carbamoylornitene transferase was 111. An increase in the activity of LDH 1, LDH
detected in the patient's blood. In which body 2, AsAT, and creatine kinase was found in the
can you predict the development of the patient's blood. In which organ of the patient is
pathological process? the most likely development of the pathological
A. Liver (possible hepatitis) process?
B. Heart muscle (possible myocardial A. Heart
infarction) B. Pancreas
C. Skeletal muscle C. Liver
D. Kidneys D. Kidneys
E. Connective tissue E. Skeletal muscle

107. Albumin concentration in the patient’s 112. There is increased activity of AST, LDH1,
blood is 2.8 g / l, increased concentration of LDH2, and CPK in the patient’s blood.
lactate dehydrogenase 5 (LDH 5). What kind of Pathological process most likely occurs in the:
organ disease does this indicate? A. Heart
A. Liver B. Skeletal muscles
B. Kidney C. Kidneys
C. Heart D. Liver
D. Lung E. Adrenal glands
E. Spleen
113. The activity of the isoenzymes LDG1 and
108. Blood test of the patient revealed albumine LDG2 appeared in the patient's blood plasma.
content of 20 g/l and increased activity of On the pathology of a body that indicates?
lactate dehydrogenase isoenzyme 5 (LDH5). A. Myocardium
These results indicate disorder of the following B. Liver
organ: C. Kidney
A. Liver D. Brain
B. Kidneys E. Skeletal muscle
C. Heart

40
114. A patient presents high activity of LDH1,2, B. Skeletal muscle dystrophy
aspartate aminotransferase, creatine C. Diabetes mellitus
phosphokinase. In what organ (organs) is the D. Viral hepatitis
development of apathological process the most E. Acute pancreatitis
probable?
A. In the heart muscle (initial stage of 119. A 60-year-old man consulted a doctor
myocardium infarction) about an onset of chest pain. In blood serum
B. In skeletal muscles (dystrophy, atrophy) analysis showed a significant increase in the
C. In kidneys and adrenals activity of the following enzymes: creatine
D. In connective tissue kinase and its MB-isoform, aspartate
E. In liver and kidneys aminotransferase. These changes indicate the
development of the pathological process in the
115. A 49-year-old driver complains about following tissues:
unbearable constricting pain behind the A. Cardiac muscle
breastbone irradiating to the neck. The pain B. Lungs
arose 2 hours ago. Objectively: the patient’s C. Skeletal muscles
condition is grave, he is pale, heart tones are D. Liver
decreased. Laboratory studies revealed high E. Smooth muscles
activity of creatinekinase and LDH1. What
disease are these symptoms typical for? 120. Hereditary hyperlipoproteinemia type I is
A. Acute myocardial infarction caused by lipoprotein lipase deficiency.
B. Acute pancreatitis Increasing the level of some transport forms of
C. Stenocardia lipids in plasma even on an empty stomach is
D. Cholelithiasis characteristic?
E. Diabetes mellitus A. Chylomicrons
B. Low density lipoproteins
116. 6 hours after the myocardial infarction a C. Very low density lipoproteins
patient was found to have elevated level of D. High density lipoproteins
lactate dehydrogenase in blood. What E. Modified lipoproteins
isoenzyme should be expected in this case?
A. LDH1 121. Cholesterol content in blood serum of a 12-
B. LDH2 year-old boy is 25 mmol/l. Anamnesis states
C. LDH3 hereditary familial hypercholesterolemia caused
D. LDH4 by synthesis disruption of receptor-related
E. LDH5 proteins for:
A. Low-density lipoproteins
117. A 47-year-old male patient was diagnosed B. High-density lipoproteins
with myocardial infarction in the intensive care C. Chylomicrons
unit. Which of the lactate dehydrogenase (LDH) D. Very low-density lipoproteins
fractions will prevail in the blood serum during E. Middle-density lipoproteins
the first two days? М-а-2016-176
A. LDH 1
B. LDH 2 122. Examination of the patient revealed an
C. LDG 3 increase in the content of low-density
D. LDH 4 lipoproteins in the blood serum. What disease
E. LDG 5 can be assumed in this patient?
A. Atherosclerosis
118. The high level of Lactate Dehydrogenase B. Kidney damage
(LDH) isozymes concentration showed the C. Acute pancreatitis
increase of LDH-1 and LDH-2 in a patient’s D. Gastritis
blood plasma. Point out the most probable E. Inflammation of the lungs
diagnosis:
A. Myocardial infarction

41
123. A 58-year-old patient suffers from the
cerebral atherosclerosis. Examination revealed 128. During examination of a teenager with
hyperlipoidemia. What class of lipoproteins will xanthomatosis the family history of
most probably show increase in concentration in hypercholesterolemia is revealed. What
this patient’s blood serum? transportable lipids are increased in
A. Low-density lipoproteins concentration in case of such a disease?
B. High-density lipoproteins A. Low-density lipoproteins
C. Fatty acid complexes with albumins B. Chylomicrons
D. Chylomicrons C. Very low-density lipoproteins
E. Cholesterol D. High-density lipoproteins
E. Intermediate-density lipoproteins
124. Rabbits lived on food with addition of
cholesterol. Five months later the 129. The level of which plasma protein allows
atherosclerotic aorta changes were revealed. retrospectively (for the previous 4-8 weeks
Name the main cause of atherogenesis in this before the examination) to assess the level of
case: glycemia, if the patient has diabetes mellitus,
A. Exogenous hypercholesterolemia which is accompanied by fasting hyperglycemia
B. Overeating more than 7.2 mmol / l?
C. Hypodynamia A. Glycosylated hemoglobin.
D. Endogenous hypercholesterolemia B. Albumin.
E. – C. C-reactive protein.
D. Ceruloplasmin.
125. Examination of an ill child’s blood E. Fibrinogen.
revealed inherited hyperlipoproteinemia.
Genetic defect of what enzyme synthesis causes 130. A patient is diagnosed with pancreatic
this phenomenon? diabetes with associated hyperglycemia.
A. Lipoprotein lipase Glycemia rate can be assessed retrospectively
B. Glycosidase (4-8 weeks prior to examination) by measuring
C. Proteinase concentration of the following blood plasma
D. Hemsynthetase protein:
E. Phenylalanine hydroxylase A. Glycated hemoglobin
B. Albumin
126. In the study of the patient's blood plasma 4 C. Fibrinogen
hours after ingestion of fatty foods, it was found D. C-reactive protein
that it is cloudy. The most likely cause of this E. Ceruloplasmin
condition is an increase in plasma
concentration: 131. A patient is ill with diabetes mellitus that is
A. Chylomicrons accompanied by hyperglycemia of over 7,2
B. HDL millimole/l on an empty stomach. The level of
C. LDL what blood plasma protein allows to estimate
D. Cholesterol the glycemia rate retrospectively (4-8 weeks
E. Phospholipids before examination)?
A. Glycated hemoglobin
127. Blood serum of the patient has milky B. Albumin
appearance. Biochemical analysis revealed high C. Fibrinogen
content of triacylglycerols and chylomicrons. D. C-reactive protein
This condition is caused by hereditary defect of E. Ceruloplasmin
the following enzyme:
A. Lipoprotein lipase 132. A 42-year-old woman, who has been
B. Phospholipase keeping to a vegetarian diet for a long period of
C. Pancreatic lipase time, consulted a doctor. Examination revealed
D. Adipose tissue hormone-sensitive lipase negative nitrogen balance in the patient. What
E. Phosphodiesterase

42
factor is the most likely cause of such a D. Nitrogen balance unchanged
condition? E. Ketonemia
A. Insufficient amount of proteins in the diet
B. Insufficient amount of dietary fiber 138. A patient with primary nephrotic syndrome
C. Excessive amount of fats in the diet has the following content of whole protein: 40
D. Insufficient amount of fats in the diet g/l. What factor caused hypoproteinemia?
E. Decreased rate of metabolic processses A. Proteinuria
B. Transition of protein from vessels to tissues
133. Examination of a 45-year-old man whohad C. Reduced protein synthesis in liver
kept to a vegeterian diet for a long time revealed D. Increased proteolysis
negative nitrogen balance.Which peculiarity of E. Disturbance of intestinal protein absorption
his diet is the causeof this phenomenon?
A. Lack of proteins 139. Examination of a patient with chronicrenal
B. Lack of fats insufficiency revealed an increase in residual
C. Excess of water nitrogen concentration in blood up to 35
D. Excess of carbohydrates millimole/l, more than half of which is urea.
E. Lack of vitamins What type of hyperazotemia is it?
A. Retentional
134. A 14 year old child was found to have a B. Hepatic
positive nitrogen balance. Which of the C. Productional
following could be the reason for this? D. Residual
A. Body growth E. Combined
B. Fasting
C. Reduced protein in food 140. On the basis of laboratory analysis, the
D. Significant physical loads patient confirmed the diagnosis of gout. To
E. Emotional stress establish the diagnosis was carried out
determination of the content:
135. A month after a serious operation a 38- A. Urinary acid in the blood and urine
year-old patient has recovered and has now B. Creatinine in urine
positive nitrogen balance. Urine of this patient C. Residual nitrogen in the blood
may be found to have low concentration of the D. Urea in the blood and urine
following nitrogencontaining substance: E. Ammonia in the urine
A. Urea
B. Lactate 141. A young man of 18 years old was
C. Stercobilinogen diagnosed with muscular dystrophy. Is the
D. Galactose increase in the serum content of the substance
E. 17-ketosteroids most likely in this pathology?
A. Creatine
136. A chiled was diagnosed with acuterenal B. Myoglobin
failure. What biochemic saliva indices can C. Myosin
confirm this diagnosis? D. Lactate
A. Increased level of rest nitrogen E. Alanin
B. Increase of immunoglobuline A
C. Reduction of alkaline phosphatase 142. A 46-year-old patient complains of dry
D. Increase of alpha amylase mouth, thirst, frequent urination, and general
E. Decreased level of phosphate weakness. In the blood: hyperglycemia,
hyperketonemia. In the urine: glucose, ketone
137. A patient who goes out of a state of bodies. On ECG: diffuse changes in the
prolonged fasting has determined nitrogen myocardium. What is the most likely diagnosis?
metabolism. What result can you expect? A. Diabetes
A. Reducing nitrogen excretion B. Alimentary hyperglycemia
B. Increased nitrogen excretion C. Acute pancreatitis
C. Nitrogen equilibrium D. Diabetes mellitus

43
E. Coronary heart disease 147. A 48 year old patient complained about
intense pain, slight swelling and reddening of
143. The patient after suffering parotitis lost skin over the joints, temperature rise up to
weight, constantly feeling thirsty, drinks a lot of 38oC. Bloodanalysis revealed high
water, notes frequent urination, increased concentration ofurates. This condition might be
appetite, pruritus, weakness, furunculosis. In the caused by disturbed metabolism of:
blood: glucose - 16 mmol / l, ketone bodies 100 A. Purines
mkmol / l. What disease has the patient B. Collagen
developed? C. Cholesterol
A. Insulin-dependent diabetes mellitus D. Pyrimidines
B. Insulin independent diabetes E. Carbohydrates
C. Steroid diabetes
D. Diabetes mellitus 148. A patient has increased content of uric acid
E. Diabetes mellitus malnutrition in his blood that is clinically presented by pain
syndrome as a result of urate deposition in the
144. A patient was delivered to the hospital by joints. What process does this acid result from?
an emergency team. Objectively: grave A. Lysis of purine nucleotides
condition, unconscious, adynamy. Cutaneous B. Lysis of pyrimidine nucleotides
surfaces are dry, eyes are sunken, face is C. Heme catabolism
cyanotic. There is tachycardia and smell of D. Proteolysis
acetone from the mouth. Analysis results: blood E. Reutilization of purine bases
glucose -20,1 micromole/l (standard is 3,3-5,5
micromole/l), urine glucose - 3,5% (standardis - 149. A 1,7-year-old child with a developmental
0). What is the most probable diagnosis? delay and manifestations of self-agression has
A. Hyperglycemic coma the concentration of uric acid in blood at the rate
B. Hypoglycemic coma of 1,96 millimole/l. What metabolic disoder is
C. Acute heart failure this typical for?
D. Acute alcoholic intoxication A. Lesch-Nyhan syndrome
E. Anaphylactic shock B. Podagra
C. Acquired immunodeficiency syndrome
145. A 38-year-old patient was admitted to the D. Gierke’s disease
intensive care unit in an unconscious state. E. Cushing’s basophilism
Reflexes are absent. Blood sugar - 2.1 mmol / l.
In history - diabetes mellitus from 18 years. 150. M-r S presents all signs of the hepatic
What kind of coma does the patient have? coma: loss of consciousness, absence of
A. Hypoglycemic reflexes, cramps, convulsion, disorder of heart
B. Ketoacidotic activity, recurrent (periodical) respiration. What
C. Lacticidemic are cerebrotoxical substances which accumulate
D. Hyperosmolar in blood under hepar insufficiency?
E. Hyperglycemic A. Ammonia
B. IL-1
146. A patient with a diagnosis of Itsenko- C. Autoantibody
Cushing's disease (hyperproduction of adrenal D. Necrosogenic substances
hormones) in the blood has an increased E. Ketonic body
concentration of glucose, ketone bodies,
sodium. What is the biochemical mechanism 151. After a serious viral infection a 3-year-old
leading to the occurrence of hyperglycemia? child has repeated vomiting, loss of
A. Gluconeogenesis consciousness, convulsions. Examination
B. Glycogenesis revealed hyperammoniemia. What may have
C. Glycogenolysis caused changes of biochemical blood indices of
D. Glycolysis this child?
E. Aerobic glycolysis A. Disorder of ammonia neutralization in
ornithinic cycle

44
B. Activated processes of aminoacids E. Pancreas
decarboxylation
C. Disorder of biogenic amines neutralization 156. Laboratory examination of a child revealed
D. Increased purtefaction of proteins in increased concentration of leucine, valine,
intestines isoleucine and their ketoderivatives in blood and
E. Inhibited activity of transamination enzymes urine. Urine smelt of maple syrup. This disease
is characterized by the deficit of the following
152. A 2-year-old child presents with mental enzyme:
development retardation, intolerance of A. Dehydrogenase of branched amino acids
proteins, severe hyperammonemia against the B. Aminotransferase
background of low blood urea content. This C. Glucose-6-phosphatase
condition is caused by the congenital deficiency D. Phosphofructokinase
of the following mitochondrial enzyme: E. Phosphofructomutase
A. Carbamoyl phosphate synthetase
B. Citrate synthase 157. A sick child presents with high content of
C. Succinate dehydrogenase phenyl pyruvate in urine (normally it is
D. Malate dehydrogenase practically absent). Blood phenylalanine level is
E. Monoamine oxidase 350 mg/L (norm - 15 mg/L). What disease are
these symptoms characteristic of?
153. A newborn presents with weak suckling, A. Phenylketonuria
frequent vomiting, and hypotonia. Blood and B. Albinism
urine citrulline are very high. What metabolic C. Tyrosinosis
process is disturbed? D. Alkaptonuria
A. Ornithine cycle E. Gout
B. Tricarboxylic acid cycle
C. Glycolysis 158. A hospital admitted a 9 y.o. boy with
D. Gluconeogenesis mental and physical retardation. Biochemical
E. Cori cycle blood analysis revealed high content of
phenylalanine. Such condition may be caused
154. Nitrogen is being excreted from thebody by blocking of the following enzyme:
mainly as urea. When activity of acertain A. Phenylalanine-4-monooxigenase
enzyme in the liver is low, it resultsin inhibition B. Oxidase of homogentisic acid
of urea synthesis and nitrogenaccumulation in C. Glutamine transaminase
blood and tissues. Namethis enzyme: D. Aspartate aminotransferase
A. Carbamoyl phosphate synthetase E. Glutamate decarboxylase
B. Aspartate aminotransferase
C. Urease 159. Blood of the patients with diabetes mellitus
D. Amylase shows increased content of free fatty acids.
E. Pepsin Name the most likely cause of this:
A. Increased activity of adipose triglyceride
155. The patient has a reduced content of lipase
indican in the blood serum, and also his daily B. Accumulation of palmitoyl-CoA in cytosol
excretion with urine is reduced. Impairment of C. Activation of ketone bodies utilization
which organ is the cause of this? D. Activation of apoА1, apoА2, and apoА4
A. Liver apolipoprotein synthesis
B. Kidney E. Decreased activity of plasma
C. Heart phosphatidylcholine-cholesterolacyltransferase
D. Lungs

45
Biochemistry of immune system
1. Differentiation of B-lymphocytes into plasma 5. Bone marrow has been transplanted to the
cells leads to synthesis of immunoglobulins that liquidator of Chernobyl atomic power station
ensure specific immune response of the body. accident which was irradiated. The reaction “a
Differentiation of B-lymphocytes takes place in transplant against a host” development was
the following organ of immune system: diagnosed at the patient after operation. Which
A. Tonsils antigens are the reason of this reaction?
B. Red bone marrow A. Antigens of HLA system in the cells of
C. Liver liquidator’s organism
D. Thymus B. Antigens of Rh system in the erythrocytes of
E. Thyroid gland liquidator
C. Antigens HBs, HBc, HBe
2. Among lymphocytes, there are populations of D. Antigens of ABO system in the erythrocytes
cells with membrane receptors for IgM, they are of liquidator
activated by the action of specific antigens, E. Antigens of HLA system in the cells of
mitotically multiply, and differentiate into donor’s spinal cord
plasma cells that produce antibodies
(immunoglobulins). What are these cells called? 6. A male patient has been diagnosed with acute
A. B-lymphocytes poststreptococcal glomerulonephritis. It is most
B. Memory T lymphocytes likely that the lesion of the basement membrane
C. T-killer lymphocytes of renal corpuscles was caused by the following
D. T-suppressor lymphocytes allergic reaction:
E. - A. Immune complex
B. Anaphylactic
3. A 2-year-old boy often became ill with C. Cytotoxic
respiratory diseases, stomatitis, pustular skin D. Delayed
lesions. Even small damage to the gums and E. Stimulating
mucous membranes is complicated by
prolonged inflammation. It is established that 7. A 5-year-old child is diagnosed with Bruton
the blood of the child is practically absent syndrome (X-linked agammaglobulinemia) that
immunoglobulins of all classes. Reduction manifests itself in severe clinical course of
the functional activity of a cell population is the bacterial infections and absence of B
basis of the syndrome described? lymphocytes and plasma cells. What changes of
A. B-lymphocytes immunoglobulin content can be observed in
B. T lymphocytes blood serum of the child with
C. Neutrophils immunodeficiency?
D. Macrophages A. Decreased IgA, IgM
E. NK lymphocytes B. Increased IgA, IgM
C. Decreased IgD, IgE
4. A 13-year-old boy presents with eczematous D. Increased IgD, IgE
rashes on his shins and torso. Anamnesis states E. No changes
cases of otitis, pneumonia, and furuncles in the
patient. Blood test: platelets - 70 • 109/l, low 8. Examination of a child who frequently suffers
activity of T helper and T suppressor cells, low from infectious diseases revealed that IgG
IgM, with normal IgA and IgG. What concentration in blood serum was 10 times less
immunodeficient disease does this boy have? than normal, IgA and IgM concentration was
A.Wiskott-Aldrich syndrome also significantly reduced. Analysis showed also
B. Louis-Bar syndrome (Ataxiatelangiectasia) lack of B-lymphocytes and plasmocytes. What
C. Severe combined immunodeficiency (Swiss disease are these symptoms typical for?
type) A. Bruton’s disease
D. DiGeorge syndrome B. Swiss-type agammaglobulinemia
E. Chediak-Higashi syndrome C. Dysimmunoglobulinemia
D. Louis-Bar syndrome
46
E. Di George syndrome B. Combined Immunodeficiency Syndrome
C. Secondary immunodeficiency syndrome
9. Parents of 5-year-old child report him o have D. Acute lymphocytic leukemia
frequent colds that develop into pneumonias, E. Syndrome of chronic intoxication
presence of purulent rashes on the skin.
Laboratory tests have revealed the following: 13. During the examination of the patient a
absence of immunoglobulins of any type, and insufficient quantity of immunoglobulins was
naked cells are absent from the lymph nodes detected. What kind of immune cells do they
punctate. What kind of immune disorder is it? produce?
A. X-linked hypogammaglobulinemia A. Plasma cells
(Bruton type agammaglobulinemia) B. T-helper
B. Autosomal recessive agammaglobulinaemia C. T-killers
(Swiss type) D. T-suppressors
C. Hypoplastic anemia E. Plasmoblasty
D. Agranulocytosis
E. Louis-Barr syndrome 14. Following exposure to radiation a lot of
mutant cells appeared in a patient. Some time
10. A child with suspected tuberculosis was later most of them were detected and destroyed
given Mantoux test. After 24 hours the site of by the following cells of the immune system:
the allergen injection got swollen, hyperemic A. T-lymphocytes-killers
and painful.What are the main components that B. Plasmoblasts
determine such response of the body? C. T-lymphocytes-supressors
A. Mononuclear cells, T-lymphocytes and D. B-lymphocyte
lymphokines E. Stem cells
B. Granulocytes, T-lymphocytes and IgG
C. Plasma cells, T-lymphocytes and 15. Numerous plasma cells were found in the
lymphokines blood girl 16 years old, suffering from an
D. B-lymphocytes, IgM autoimmune inflammation of the thyroid gland.
E. Macrophages, B-lymphocytes and monocytes With the proliferation and differentiation of
what blood cells cause increase of plasmocyte?
11. A 27-year-old woman has dropped penicillin A B-lymphocytes
containing eye drops. In a few minutes there B T-helper
appeared feeling of itching, burning of the skin, C Mast cells
lips and eyelids D T-killer
edema, whistling cough, decrease of BP. What E T-suppressor
imunoglobulins take part in the development of
this allergic reaction? 16. The patient is diagnosed with ARVI. Class
A. IgE and IgG M immunoglobulins have been found in the
B. IgM and IgG serum. What is the period of the infection
C. IgA and IgM process in this case?
D. IgM and IgD A. Acute
E. IgG and IgD B. Prodromally
C. Incubation
12. A 3 year old child with multiple impaired D. Reconvalescence
development of the bones of the facial skull E. Micro carrier
died. The cause of death is sepsis, which
developed on the background of 17. A 34-year-old patient, after suffering an
bronchopneumonia. Blood content intestinal infection caused by Salmonella,
immunoglobulins within the physiological symptoms of the disease began to fade. What
norm. At the autopsy revealed the absence of class of immunoglobulins will be detected in the
the thymus. Name the main cause of the child’s patient's blood during the recovery period?
illness: A. Ig G
A. Syndrome of cellular immunity deficiency B. Ig A

47
C. Ig D hypotension. When this reaction is developed
D. Ig E and the allergen achieves tissue basophils, it
E. Ig M reacts with:
A. IgE
18. Preventive vaccination against polyomyelitis B. IgА
is made with inactivated vaccine introduced C. IgD
parenterally. What immunoglobulins create the D. IgМ
postvaccinal immunity in the case? E. T-lymphocytes
A. IgM, IgG
B. IgG, secretory IgA 23. A 34-year-old patient, after suffering an
C. IgM, secretory IgA intestinal infection caused by Salmonella,
D. Serum IgA, IgM symptoms of the disease began to fade. What
E. Ig E, IgM class of immunoglobulins will be detected in the
patient's blood during the recovery period?
19. A 37-year-old man was injected with A. Ig G
novocaine solution in the treatment of acute B. Ig A
pulpitis. A few minutes later the patient C. Ig D
developed an anaphylactic shock.What D. Ig E
immunoglobulin does the antigen mainly E. Ig M
interact with in the body during a given allergic
reaction? 24. One of the functions of saliva is protective,
A. IgE which is realized by the formation of local
B. IgM immunity of the mucous membrane due to the
C. IgA secretion of the parotid glands of such a protein:
D. IgD A. Secretory Immunoglobulin A
E. IgG B. Collagen
C. Elastin
20. After the introduction of lidocaine, a 25- D. Fibrinogen
year-old patient developed shortness of breath, E. Albumin
bronchospasm, and his blood pressure dropped
sharply, which required the use of immediate 25. Various cells of the oral mucous membrane
relief from a dentist doctor. What mechanism is and antimicrobial substances synthesized by
the basis of such phenomena? these cells play an important part in the local
A. Allergic reactions involving IgE immunity of the oral cavity. Specify the key
B. Allergic cytotoxic reactions factors for the local immunity:
C. The phenomenon of idiosyncrasy A. Secretory IgA
D. Hypersensitivity due to T-lymphocytes B. B-lymphocytes
E. Effects caused by T-killers C. IgG
D. Macrophages
21. Skin samples of a patient with bronchial E. Eosinophils
asthma revealed allergen sensitization of poplar
fuzz. What factor of immune system plays the 26. In our country, routine preventive
main part in vaccinations against poliomyelitis ivolve using
development of this immunopathological state? live vaccine that is administered orally. What
A. IgE immunoglobulins are responsible for the
B. IgD development of local post-vaccination immunity
C. IgM in this case?
D. Sensitized Т-lymphocytes A. Secretory IgA
E. – B. IgM
C. IgG
22. A teenger had his tooth extracted under D. Serum IgA
novocain anaesthesia. 10 minutes later he E. IgE
presented with skin pallor, dyspnea,

48
27. Throughout a year a 37-year-old woman B. DNA → i-RNA → polypeptide → DNA
periodically got infectious diseases of bacterial C. DNA → polypeptide → i-RNA
origin, their course was extremely lingering, D. i-RNA → polypeptide → DNA
remissions were short. Examination revealed E. Polypeptide → RNA → DNA → i-RNA
low level of major classes of immunoglobulins.
The direct cause of this phenomenon may be the 32. A patient with acquired immunodeficiency
following cell dysfunction: syndrome (СНІД) the immunological reactivity,
A. Plasmocytes manifested by the development of chronic
B. Phagocytes inflammatory processes, infectious diseases,
C. Neutrophils tumor growth, is significantly reduced. What
D. Macrophages types of blood cells damages HIV-infections,
E. Lymphocytes which results in reduced immune protection?
A. T4 helper
28. A patient has been hospitalized with B. Natural Kileri (NK)
provisional diagnosis of virus B hepatitis. C. T-suppressors
Serological reaction based on complementation D. T8-Effects
of antigen with antibody chemically bound to E. B-lymphocytes
peroxidase or alkaline phosphatase has been
used for disease diagnostics. What is the name 33. A 20-year-old patient, an AIDS diagnosis
of the applied serological reaction? was established. What cell populations are most
A. Immune-enzyme analysis sensitive to human immunodeficiency virus?
B. Radioimmunoassay technique A. T-helpers
C. Immunofluorescence test B. Hepatocytes
D. Bordet-Gengou test C. Endotheliocytes
E. Antigen-binding assay D. Epithelial cells
E. B lymphocytes
29. A doctor examined a patient with recurrent
aphthous stomatitis with concominant 34. A 1-year-old child often suffers from viral
candidosis and decided to eliminate a possibility and bacterial infections that are difficult to treat.
of HIV-infection. What examination can help to During the investigation the immunological
clear the situation up and make a provisional status revealed the absence of lymphocytes in
diagnosis? the blood providing cellular immunity. What
A. Immune-enzyme analysis immunodeficiency and what system is detected
B. Gel precipitation reaction in this child?
C. Reaction of hemagglutination inhibition A. In a T cell primary response
D. Reaction of hemagglutination B. In a T-cell secondary response
E. Phase-contrasr microscopy C. In a B cell primary response
D. In a macrophage system primary response
30. T-lymphocytes are determined to be affected E. In a microphages primary response
with HIV. In this case viral enzyme reverse
transcriptase (RNA-dependent DNA- 35. Blood serum of a newborn contains
polymerase) catalyzes the synthesis of: antibodies to measles virus. What kind of
A. DNA based on the viral RNA matrix immunity is this indicative of?
B. Viral RNA based on the DNA matrix A. Natural passive
C. Viral protein based on the viral RNA matrix B. Natural active
D. Viral DNA based on the DNA matrix C. Artificial passive
E. Informational RNA based on the viral protein D. Artificial active
matrix E. Heredoimmunity

31. Lymphocytes are affected by HIV retrovirus 36. A child was born with cleft palate.
(AIDS). In this case, the direction of Examination revealed aorta defects and reduced
information flow in the cell will be: number of T-lymphocytes in blood. What
A. RNA → DNA → i-RNA → Polypeptide immunodeficient syndrome is it?

49
A. Di George A. Interleukin-1
B.Wiskott-Aldrich B. Immunoglobulins
C. Chediak-Higashi C. Interferons
D. Louis-Bar D. Biogenic amins
E. Swiss-type E. Angiotensin

37. A female patient underwent liver 42. Donor skin transplantation was performed to
transplantation. 1,5 month after it her condition a patient with extensive burns. On the 8-th day
became worse because of reaction of transplant the graft became swollen and changed colour;
rejection. What factor of on the 11-th day graft rejection started.What
immune system plays the leading part in this cells take part in this process?
reaction? A. T-lymphocytes
A. T-killers B. Erythrocytes
B. Interleukin-1 C. Basophils
C. Natural killers D. Eosinophils
D. B-lymphocytes E. B-lymphocytes
E. T-helpers
43. The patient contacted the dermatologist with
38. A patient consulted an immunologist about complaints about eczematous lesions of the skin
diarrhea, weight loss within several months, that appeared after contact with the detergent
low-grade fever, enlarged lymph nodes. The "Lotus". The use of rubber gloves prevents this.
doctor suspected HIV infection. What The pathological reaction of the skin is caused
immunocompetent cells must be studied in the by activation:
first place? A. T-lymphocytes
A. Helper T-lymphocytes B. B-lymphocytes
B. Suppressor T-lymphocytes C. Monocytes
C. B-lymphocytes D. Neutrophils
D. Monocytes E. Basohiles
E. Plasma cells
44. The development of febrile states are
39. After transfusion of 200 ml of blood a characterized by an increase in the level “the
patient presented with body temperature rise up acute phase” proteins. There are ceruloplasmin,
to 37,9°C. Which of the following substances is fibrinogen, C-reactive protein. Enter the
the most likely cause of temperature rise? Possible mechanism of this phenomenon:
A. Interleukin-1 A. Stimulating effect of IL-1 on hepatocytes
B. Interleukin-2 B. The destructive effect of temperature on the
C. Tumour necrosis factor cells of the body
D. Interleukin-3 C. The proliferative effect of IL-2 on
E. Interleukin-4 T lymphocytes
D. Degranulation of tissue basophils
40. A patient with skin mycosis has disorder of E. -
cellular immunity. The most typical
characteristic of it is reduction of the following 45. A pregnant woman with the previous
index: diagnosis of toxoplasmosis has been
A. T-lymphocytes hospitalized. What serological reaction should
B. Immunoglobulin G be used to determine toxoplasmosis in test
C. Immunoglobulin E specimens?
D. B-lymphocytes A. Complement fixation test.
E. Plasmocytes B. Neutralization
C. Haemadsorption
41. Inflammatory processes cause synthesis of D. Agglutination
protein of acute phase in an organism. What E. Inhibition of hemagglutination
substances simulate their synthesis?

50
Biochemistry of liver
1. Disorderthe splitting of the lipids in the small achilic syndrome due to obturation of the biliary
intestine is due to a disorder of lipase activity. tract. Which components of food will be
Which of the following factors activates the mastered the most?
lipase? A. Fats
A. Bile acids B.Carbohydrates
B. Hydrochloric acid C. Proteins
C. Enterokinase D. Nucleic acids
D. Pepsin E. Electrolytes
E. Solts of Na+
7. The drug "Geptral", which is used in liver
2. Examination of a patient revealed that dental diseases contains S-adenosylmethionine. This
hypoplasia was caused by hypovitaminosis of active amino acid is involved in the synthesis
vitamins A and D. These vitamins were of:
administered perorally but they didn’t have any A. Phospholipids
medicinal effect. What is the probable cause of B. Fatty acids
disturbed vitamin assimilation? C. Triacylglycerols
A. Bile acid deficiency D. Cholesterol
B. Phospholipase A2 deficiency E. Heme
C. Cholesterolesterase deficiency
D. Colipase deficiency 8. The patient appointed a lipotropic drug a
E. Pancreatic lipase deficiency donor of methyl groups, to prevent a fatty liver
distrophia. This is sensible:
3. A coprological study found that the feces are A. S-Adenosylmethionine
discolored, there are found drops of neutral fat. B. Cholesterol
The most likely cause of this is a violation: C. Bilirubin
A. Flow of bile into the intestine D. Valine
B. pH of gastric juice E. Glucose
C. Secretions of pancreatic juice
D. Secretion of intestinal juice 9. Examination of cell culture gotfrom a patient
E. Absorption processes in the intestine with lysosomal pathology revealed
accumulation of great quantity of lipids in the
4. After eating fatty foods, the patient has lysosomes. What of the following diseases is
nausea and heartburn, steatorrhea occurs. The this disturbance typical for?
reason for such a state can be: A. Tay-Sachs disease
A. Lack of bile B. Gout
B. Increased lipase secretion C. Phenylketonuria
C. Violation of trypsin synthesis D.Wilson disease
D. Lack of amylase E. Galactosemia
E. Impaired phospholipase synthesis
10. Steatosis occurs as a result of the
5. Due to the blockage of the common bile duct accumulation of triacylglycerols in hepatocytes.
(which was radiographically confirmed), the One of the mechanisms of development of this
biliary flow to the duodenum was stopped. We disease is reduction of utilization neutral fat
should expect the impairment of: LDL. What lipotropic substances prevent the
A. Fat emulsification development of steatosis? A. Methionine, BC,
B. Protein absorption B12
C. Carbohydrate hydrolysis B. Arginine, B2, B3
D. Secretion of hydrochloric acid C. Alanine, B1, PP
E. Salivation inhibition D. Valine, B3, B2
E. Isoleucine, B1, B2
6. A 65-year-old patient suffers from
cholelithiasis. Recently, there were signs of
51
11. In an experimental animal, receiving non- E. Leucine
protein diet, fatty infiltration of the liver has
developed due to the deficiency of the 16. The gluconeogenesis is activated in the liver
methylating agents. The formation of which after intensive physical trainings. What substance is
metabolite is disturbed in the experimental utilized in gluconeogenesis first of all in this case:
animal? A. Lactate
A. Choline B. Pyruvate
C. Glucose
B. DOPA D. Glutamate
C. Cholesterol E. Alanine
D. Acetoacetate
E. Linolic acid 17. In patients with glycogenosis, that is von
Gierke’s disease, the conversion of glucose-6-
12. A dry cleaner’s worker has been found to phosphate in to glucose is inhibited, which is
have hepatic steatosis. This pathology can be accompanied by the improper breakdown of
caused by the disruption of synthesis of the glycogen in the liver. The cause of this
following substance: condition is the following enzyme deficiency:
A. Phosphatidylcholine A. Glucose-6-phosphatase
B. Tristearin B. Glycogen phosphorylase
C. Urea C. Glucose-6-phosphate dehydrogenase
D. Phosphatidic acid D. Phosphofructokinase
E. Cholic acid E. Phosphoglucomutase
13. In the 2-year-old boy, an increase in the size 18. A child has a history of hepatomegaly,
of the liver and spleen, cataract is observed. The hypoglycemia, seizures, especially on an empty
concentration of sugar is elevated in the blood, stomach and in stressful situations. The child is
but the test of glucose tolerance is normal. An diagnosed with Gierke disease. This disease is
hereditary violation of the metabolism of which caused by the genetic defect of the following
substance is the cause of this condition? enzyme:
A. Galactose A. Glucose-6-phosphatase
B. Fructose B. Amyloid-1,6-glycosidase
C. Glucose C. Phosphoglucomutase
D. Maltose D. Glycogen phosphorylase
E. Saccharose E. Glucokinase
14. During starvation normal rate ofglucose is
maintained by means ofactivation of 19. A patient with chronic hypoglycemia had
gluconeogenesis. Whatsubstance can be used as adrenaline introduction. After introduction
a substrate forthis process? blood test hasn’t changed essentially. Doctor
A. Alanine assumed liver pathology. What liver function
B. Ammonia may have been changed?
C. Adenine A. Function of glycogen depositing
D. Urea B. Function of cholesterin production
E. Guanine C. Ketogenic function
D. Glycolytic function
15. In a patient undergoing a course of medical E. Excretory function
starvation, the normal level of glucose in the
blood is maintained mainly due to 20. After the introduction of adrenaline in a
gluconeogenesis. From what amino acids at the patient with persistent hypoglycemia, blood test
same time in the human liver most actively has not changed significantly. In such
synthesized glucose? circumstances, there is a possibility of
A. Alanine disturbances in the liver. What function of the
B. Lysine liver is changed?
C. Valine A. Glycogendeposing.
D. Glutamic acid B. Glycolytic.
52
C. Excretory. B. Increase of globulin conten
D. Ketogenic. C. Decrease of fibrinogen conten
E. Cholesterol-forming. D. Increase of albumin conten
E. Decrease of globulin conten
21. In the human diet there are a large number
of carbohydrates. The number of which 26. A 25-year-old patient has been diagnosed
structures will increase in cytoplasm of with chronic hepatitis. The patient complains of
hepatocytes? 10 kg weight loss within 2 months. Objectively:
A. Glycogen granules the patient has dry peeling skin, pale with
B. Drops of fat yellow shade, petechial haemorrhages,
C. The lysosomes stomatorrhagia. Petechial haemorrhages and
D. Free ribosomes stomatorrhagia are caused by the disturbance of
E. Inclusion of lipofuscine the following hepatic function:
A. Protein synthesizing
22. The main part of nitrogen is taking out of B. Chromogenic
organism as element of urea. Reducing the C. Glycogen synthesizing
activity of what liver enzyme leads to inhibition D. Detoxication
of the synthesis of urea and increasing the E. Depositing
accumulation of ammonia in blood and tissues?
A. Carbamoyl phosphate synthase 27. Hepatitis has led to the development of
B. Aspartate aminotransferase hepatic failure. Mechanism of edemata
C. Urease formation is activated by the impairment of the
D. Amilaza following liver function:
E. Pepsin A. Protein-synthetic
B. Barrier
23. In the biosynthesis of urea in the liver, the C. Chologenetic
formation of ornithine and urea is stimulated. D. Antitoxic
What amino acid is the intermediate product of E. Glycogen-synthetic
this synthesis?
A. Аrginine 28. A patient being treated for viral hepatitis
B. Leicinc. type B got symptoms of hepatic insufficiency.
C. Сitrate. What blood changes indicative of protein
D. Valin. metabolism disorder will be observed in this
E. Tryptophan. case?
A. Absolute hypoalbuminemia
24. In a boy of 4 years after suffering from B. Absolute hyperalbuminemia
severe viral hepatitis, vomiting, episodes of C. Absolute hyperfibrinogenemia
nephropathy, seizures are observed. In the blood D. Proteinic blood composition is unchanged
there is hyperammonia. Violation of which of E. Absolute hyperglobulinemia
the biochemical process in the liver caused such
a condition of the patient? 29. In obstructive jaundice and bile ducts,
A. Disposal of ammonia. prothrombin insufficiency is often observed.
B. Decarboxylation of amino acids. With a deficiency in the organism of which
C. Disposal of Biogenic Amines. vitamin it is connected?
D. Synthesis of alphabets. A. K
E. Gluconeogenesis. B. B6
C. A
25. Upon toxic damage of hepatic cells resulting D. C
in disruption of liver function the patient E. E
developed edemas. What changes of blood
plasma are the main cause of edema 30. A patient has been admitted to the
development? contagious isolation ward with signs of jaundice
A. Decrease of albumin content caused by hepatitis virus.Which of the

53
symptoms given below is strictly specific for syndrome. Which type of jaundice is developed
hepatocellular jaundice? in this patient?
A. Increase of ALT, AST level A. Gallstone
B. Hyperbilirubinemia B. Hemolytic jaundice
C. Bilirubinuria C. Hepatitis
D. Cholemia D. Chronic gastritis
E. Urobilinuria E. Chronic colitis

31. A 15-year-old boy has been diagnosed with 36. The patient turned to doctor with
acute viral hepatitis. What blood value should disturbances in the right side of the subarea.
be determined to confirm acute affection of After examination of the patient, the doctor was
hepatic cells? found yellow sclera. Laboratory diagnostic
A. Aminotransferase activity (AST, ALT) revealed increased ALT, negative stercobilin in
B. Unconjugated and conjugated bilirubin feces. Which type of disease is developed these
content symptoms?
C. Erythrocytes sedimentation rate (ESR) A. Hepatitis
D. Cholesterol content B. Haemolytic jaundice
E. Protein fraction content C. Chronic gastroduodenitis
D. Chronic colitis
32. Blood analysis of a patient with jaundice E. Chronic gastritis
reveals conjugated bilirubinemia, increased
concentration of bile acids. There is no 37. The 20-years-old man is diagnosed with
stercobilinogen in urine. What type of jaundice heredity deficiency of UDP-
is it? glucuronosyltransferase. Change in which blood
A. Obstructive jaundice parameter can prove this diagnosis?
B. Hepatocellular jaundice A. Indirect (unconjugated) bilirubin
C. Parenchymatous jaundice B. Direct (conjugated) bilirubin
D. Hemolytic jaundice C. Urobilin
E. Cythemolytic jaundice D. Stercobilinogen
E. Animal indican
33. Encephalopathy has developed in a child
with hemolytic disease of the newborn. What 38. The preterm newborn has a jaundice. Which
substance had increased in the child’s blood, enzyme is probably deficient in this baby?
resulting in damage to the CNS? A. UDP-glucosyltransferase
A. Unconjugated bilirubin B. alkaline phosphatase
B. Bilirubin-albumin complex C. acidic phosphatase
C. Bilirubin glucuronide D. catalase
D. Verdohemoglobin E. NAD+-dehydrogenase
E. Bileacids
39. The born in time newborn has yellow skin
34. Barbiturates prescribed for jaundice and mucus color. The most probable reason of
treatment because it induces UDP- this condition is temporary insufissiency of the
glucuronosyltransferase synthesis. Which following enzyme:
substance formation provide therapeutic effect? A. UDP-glucosyltransferase
A. Direct (conjugated) bilirubin B. Glucuronosyltransferase
B. Indirect (unconjugated) bilirubin C. Heme synthase
C. Biliverdin D. Heme oxygenase
D. Protoporphyrin E. Biliverdin reductase
E. Heme
40. The newborn has a physiological jaundice.
35. The patient came to doctor with compliance The level of blood free bilirubin considerably
about yellow sclera and skin. No higher than normal. What enzyme deficiency
encephalopathy, cholestasis or acholic can cause this

54
A. UDP-glucosyltransferase bilirubin is normal. Which type of jaundice this
B. Transaminase patient is most probably develop?
C. Xanthine oxidase A. Hemolytic
D. Adenosine deaminase B. Parenchymatous (hepatic)
E. Heme oxygenase C. Mechanical
D. Newborn jaundice
41. Transfusion of Rh-incompatible blood E. Gilbert's syndrome
resulted in hemolytic jaundice development in
the patient. What laboratory blood value 46. The patient develops yellowish of skin and
confirms this type of jaundice? mucus membranes. Analysis of blood plasma
A. Accumulation of unconjugated bilirubin revealed increased total bilirubin, feces –
B. Reduction of unconjugated bilirubin increased stercobilin, urine – increased urobilin.
C. Accumulation of urobilinogen What type of jaundice this patient has?
D. Reduction of stercobilin A. Hemolytic
E. Reduction of conjugated bilirubin B. Gilbert's syndrome
C. Parenchymatous
42. The patient has a following symptoms: D. Obstructive
yellow skin, dark urine, yellow-dark feces. E. Cholestatic
Which substance you will predict to be
increased in blood serum? 47. After blood transfusion patient has
A. Free bilirubin yellowish of skin and mucus membrane,
B. Conjugated bilirubin increased total and indirect bilirubin, increased
C. Mesobilirubin urine urobilin and feces stercobilin. What kind
D. Verdoglobin of jaundice can be the reason?
E. Biliverdin A. Hemolytic jaundice.
B. Newborn jaundice.
43. A 46 year old woman suffering from C. Obstructive jaundice.
chololithiasis developed jaundice. Her urine D. Parenchymatous jaundice.
became dark-yellow and feces became E. Genetic jaundice.
colourless. Blood serum will have the highest
concentration of the following substance: 48. The patient who suffers from jaundice
A. Conjugated bilirubin because of increased bilirubin as part of its
B. Unconjugated bilirubin indirect fraction. Urine and feces have intensive
C. Biliverdin color. What is the most probable explanation to
D. Mesobilirubin these disruptions?
E. Urobilinogen A. Increased erythrocyte hemolysis
B. Disrupted liver urobilinogen transformation.
44. Enzymatic jaundices are accompanied by C. Disruption of direct bilirubin formation.
abnormal activity of UDPglucuronyl D. Liver parenchyma damage.
transferase. What compound is accumulated in E. Disruption of bile flow from liver
blood serum in case of these pathologies?
A. Unconjugated bilirubin 49. The 28-years-old woman came to infectious
B. Conjugated bilrubin department with yellow skin, sclera, mucus
C. Dehydrobilirubin membrane. Laboratory diagnosis revealed
D. Hydrobilirubin increased blood direct bilirubin; urobilinogen
E. Choleglobin and bilirubin in urine. Which of following
condition can lead to those symptoms?
45. The 20-years-old patient has a jaundice and A. Parenchymatous jaundice
the following laboratory parameter: increase B. Hemolytic jaundice
general blood bilirubin, predominantly by C. Kidney infarction
indirect (free) fraction; urine and feces has a D. Renal tuberculosis
high stercobilin; blood direct (conjugated) E. Mechanical jaundice

55
50. The 28-years old man came to doctor to 54. The patient has yellow skin, skin rash,
complaints about acute disturbances in the right general weakness. In the urine: there is no
side of the subarea, nausea, vomiting. urobilin. Which type of pathology this patient is
Objectively: jaundice of the skin and sclera; the most probably develop?
temperature of the body is elevated, the urine is A. Mechanical jaundice
dark color. Feces is hypochloric. The patient B. Parenchymal jaundice
have hyperbilirubinemia (bilateral and indirect C. Acute liver insufficiency
biliary), bilirubinuria, urobilinuria, D. Haemolytic jaundice
gipoproteinemiya, reduction of blood E. Chronic liver insufficiency
transfusion. Which of following condition can
lead to those symptoms? 55. The 43-years-old patient during 10 years has
A. Hemolitic jaundice a repetitive compliance about acute bellyache,
B. Subacute jaundice cramps, vision problems. His relatives have
C. Suprachondrial hemolytic jaundice Patient urine have red color. He was diagnosed
D. Acute cholecystitis with acute intermittent porphyria. The reason of
E. Acute pancreatitis he his disease is not being a vampire but
disrupted biosynthesis of:
51. The 48-years-old woman was delivered to A. Heme
hospital with compliance of general weakness, B. Bile acids
sleep disturbance. Objectively: skin and sclera C. Insulin
are yellow. Blood analysis has increased total D. Collagen
bilirubin with direct bilirubin predominance. E. Prostaglandin
Feces is acholic. Urine has dark color, because
of bile pigments. Which type of jaundice this 56. The patients with erythropoietic porphyria
patient has? (Gunther disease) have ultraviolet red-
A. Mechanical fluorescent tooth, sun-sensitive skin, red-
B. Hemolytic colored urine. Which enzyme deficiency cause
C. Parenchymatous this disease?
D. Gilbert's syndrome A. Uroporphyrinogen III synthase
E. Crigler–Najjar syndrome B. δ- aminolevulinic acid synthase
52. The patient was delivered to hospital with C. Uroporphyrinogen I synthase
complaints about general weakness and sleep D. Uroporphyrinogen decarboxylase
disturbance. Skin has yellow color. There is E. Ferrochelatase
increased blood direct bilirubin and bile acids.
Feces is acholic. Which type of disease can 57. The patient who suffers from anemia has
provoke those changes? increased protoporphyrin IX. Which mineral
A. Mechanical jaundice deficiency can lead to such pathology?
B. Hemolytic jaundice A. Iron
C. Pre-hepatic jaundice B. Potassium
D. Gilbert's syndrome C. Magnesium
E. Chronic cholecestitis D. Sodium
E. Phosphorus
53. The patient with jaundice has increased
direct bilirubin and bile acids in blood; no 58. The patient has increased photosensitivity,
stercobilinogen was revealed in urine. What after prolong sun exposure urine starts to get
type of jaundice can be the reason of these dark-red color. Which type of disease has this
symptoms? characteristic?
A. Mechanical A. Porphyria
B. Hepatic B. Alkaptonuria
C. Parenchymatous C. Albinism
D. Hemolytic D. Hemolytic jaundice
E. Post-hepatic E. Pellagra

56
59. The 33 years old patient worries 10 years.
Periodically, he addresses the doctor with 63. Detoxification of xenobiotics (drugs,
complaints of acute abdominal pain, epoxides, the arena oxides, aldehydes, nitro
convulsions, visual impairment. His relatives derivatives, etc.) and endogenous metabolites
are witnessing similar symptoms. Urine red. (estradiol, prostaglandins, leukotrienes) takes
Hospitalized with diagnoses - acute diarrhea place in the liver by conjugation with:
porphyria. The cause of the disease may be a A. Glutathione
violation of the biosynthesis of such a B. Aspartic acid
substance: C. Glycine
A. Gem D. S-Adenozylmethionin
B. Insulin E. Fosfoadenozyn
C.Chicken acids 64. For a patient suffering from chronic
D. Prostaglandins hepatitis, a load of sodium benzoate was carried
E. Collagen out to evaluate the degreasing function of the
liver. For the discharge of what substances with
60. A 43-year-old woman complains of general urine judge the neutralizing function of the
weakness, weight loss, apathy, and drowsiness. stomach?
Chronic lead intoxication confirmed laboratory - A. Hippuric acid
revealed hypochromic anemia. In the blood, an B. Phenylacetic acid
increased level of Zn-protoporphyrin and a C. Citric acid
reduced level of alpha-aminolevulinic acid, D. Valerian acid
indicating a violation of the synthesis: E. Oxalic acid
A. Heme
B. DNA 65. Patient has cirrhosis of the liver. The study
C. RNA of which of the listed substances that are
D. Protein excreted in the urine can characterize the state
E. Mevalonic acid of the antitoxic function of the liver?
A. Hippuric acid
61. A mother consulted a doctor about her 5- B. Ammonium salts
year-old child who develops erythemas, C. Creatinine
vesicular rash and skin itch under the influence D. Uric acid
of sun. Laboratory studies revealed decreased E. Amino acids
iron concentration in the blood serum, increased
uroporphyrinogen I excretion with the urine. 66. A woman of a chemical enterprise due to
What is the most likely inherited pathology in violation of the rules of safe work has suffered a
this child? toxic action of nitric acid and nitrites, which
A. Erythropoietic porphyria cause deamination of cytosine in the DNA.
B. Methemoglobinemia What enzyme initiates a chain of reparative
C. Hepatic porphyria processes?
D. Coproporphyria A. Uridine-DNA-glucosidase
E. Intermittent porphyria B. Cytidine triphosphate synthetase
C. Orthotidyl monophosphate-decarboxylase
62. A patient suffering from chronic hepatitis D. DNA-Dependent-RNA Polymerase
complains of increasing the sensitivity to E. Timidilatsintaza
barbiturates, which were earlier tolerated
without symptoms of intoxication. With the 67. In the 70's, scientists found that the cause of
disturbance of which of the liquor functions, severe jaundice of newborns is a disturbance of
this is connected at the highest possible level? the binding of bilirubin in hepatocytes. What
A. Detoxification substance is used to form a conjugate?
B. Hemopoietic A. Glucuronic acid
C. Hemodynamic B. Uric acid
D. Formation of bile C. Sulfuric acid
E. Phagocytic D. Lactic acid

57
E. Pyruvic acid
71. In the liver, detoxification of natural
68. A 50-year-old patient with food poisoning metabolites and xenobiotics is impaired. Name
was prescribed a dropper with a 10% glucose cytochrome, the activity of which can be
solution. It not only provides the energy needs reduced:
of the body, but also performs a detoxification A. Cytochrome P-450
function due to the formation of a metabolite B. Cytochrome oxidase
that participates in the conjugation reaction: C. Hemoglobin
A. Gluuronation D. Cytochrome b
B. Sulfonation E. Cytochrome c1
C. Methylation
D. Glycosylation 72. Study of conversion of a food colouring
E. Hydroxylation agent revealed that neutralization of this
xenobiotic takes place only in one phase -
69. To determine the antitoxic function of the microsomal oxydation. Name a component of
liver, the patient is assigned sodium benzonate, this phase:
which in the liver turns into hippuric acid. What A. Cytochrome Р-450
compound is used for this? B. Cytochrome B
A. Glycine C. Cytochrome C
B. Methionine D. Cytochrome A
C. Cysteine E. Cytochrome oxidase
D. FAFS
E. UDF-glucuronic acid 73. The universal biological system of oxidation
of nonpolar compounds (drugs, toxic
70. In a patient with chronic alcoholic liver compounds), steroid hormones, cholesterol is
disease, processes of xenobiotics and microsomal oxidation. What is the name of the
endogenous toxic compounds biotransformation cytochrome, which is part of the oxygenase
are disturbed. Reducing the activity of which chain of the microsome.
chromoprotein can be the cause of this? A. Cytochrome P 450.
A. Cytochrome P-450 B. Cytochrome a3.
B. Hemoglobin C. Cytochrome in.
C. Cytochrome Oxidase D. Cytochrome c.
D. Cytochrome b E. Cytochrome a.
E. Cytochrome c1

58
Pathologic urine components
1. A 13 year-old boy is complaining about A. Cystinuria.
general weakness, dizziness, fatigue. Mental B. Alpathururia.
retardation is detected. The examination C. Cystitis
revealed high concentrations of valine, D. Phenylketonuria.
isoleucine, and leucine in blood and urine. The E. Hartnup's disease.
urine has a specific smell. What is the most
possible diagnosis? 6. A 36-year-old female patient has a history of
A. Maple syrup urine disease collagen disease. Urine analysis is likely to
B. Addison’s disease reveal an increased concentration of the
C. Tyrosinosis following metabolite:
D. Histidinemia A. Oxyproline
E. Grave’s disease B. Indican
C. Creatinine
2. Excess of phenylpyruvate and phenylacetate D. Urea
was detected in the urine sample of a 6 day-old E. Urobilinogen
baby. Which amino acid metabolism is
disturbed? 7. A patient with systemic scleroderma has an
A. Phenylalanine intensified collagen destruction. Collagen
B. Tryptophan destruction will be reflected by intensified
C. Methionine urinary excretion of the
D. Histidine following amino acid:
E. Arginine A. Oxyproline
B. Alanine
3. In a sick child, a raised level of C. Tryptophane
phenylpyruvate was detected in the urine (in the D. Serine
norm practically absent). The amount of E. Phenylalanine
phenylalanine in the blood is 350 mg/l (about 15
mg/l). Which disease is characterized by the 8. A child has physical and mental retardation,
above symptoms? serious abnormalities in connective tissue of
A. Fenilketonuria. internal organs; urine contains keratan sulfates.
B. Albin. This is caused by metabolic disorder of the
C. Alpathonuria. following substance:
D. Gout A. Glycosaminoglycan
E. Tyrosinosis. B. Collagen
C. Elastin
4. A child with mental and physical D. Fibronectin
development retardation was delivered to the E. Hyaluronic acid
hospital. Phenylpyruvate was found in the urine.
Disturbance of which process is the reason for 9. A traumatology unit received a patient with
this pathology? crushed muscular tissue. What biochemical
A. Amino acid metabolism indicator of urine will be raised in this case?
B. Lipid metabolism A. Creatinine
C. Carbohydrate metabolism B. Total lipids
D. Water-salt balance regulation C. Glucose
E. Calcium and phosphate homeostasis D. Mineral salts
E. Uric acid
5. A 12-year-old boy in the urine revealed high
levels of all amino acids in the aliphatic row. In 10. A biochemical urine analysis has been
this case, the highest excretion of cysteine and performed for a patient with progressive
cysteine was noted. In addition, ultrasound of muscular dystrophy. In the given case muscle
the kidneys showed the presence of stones in disease can be confirmed by the high content of
them. Choose a possible pathology. the following substance in urine:
59
A. Creatine B. Phenylalanine
B. Porphyrin C. Alanine
C. Urea D. Methionine
D. Hippuric acid E. Asparagine
E. Creatinine
16. The 28-years old man came to doctor to
11. A biochemical urine analysis has been complaints about acute disturbances in the right
performed for a patient with progressive side of the subarea, nausea, vomiting.
muscular dystrophy. In the given case muscle Objectively: jaundice of the skin and sclera; the
disease can be confirmed by the high content of temperature of the body is elevated, the urine is
the following substance in urine: dark color. Feces is hypochloric. The patient
A. Creatine have hyperbilirubinemia (bilateral and indirect
B. Porphyrin biliary), bilirubinuria, urobilinuria,
C. Urea gipoproteinemiya, reduction of blood
D. Hippuric acid transfusion. Which of following condition can
E. Creatinine lead to those symptoms?
A. Hemolitic jaundice
12. A baby has coloured sclera and mucouses B. Subacute jaundice
and her urine is darkening when exposed to air. C. Suprachondrial hemolytic jaundice
Homogentisic acid was found in urine and blood D. Acute cholecystitis
samples. What is the reason for this condition? E. Acute pancreatitis
A Alcaptonuria
B. Albinism 17. A patient has an increased pyruvate
C. Galactosemia concentration in blood. A large amount of it is
D. Cystinuria excreted with the urine. What vitamin is lacking
E. Histidinemia in this patient?
A. B1
13. A 5-year old child has darkened urine but no B. E
bile enzymes were found in it. The child is C. B3
diagnosed with alcaptonuria. Which enzyme D. B6
deficiency is the reason for it? E. B2
A. Homogentisic acid oxydase
B. Phenylalanine hydroxylase 18. A patient, who has been subsisting
C. Tyrosinase exclusively on polished rice, has developed
D. Oxyphenylpyruvate oxydase polyneuritis due to thiamine deficiency. What
E. Phenylpyruvate decarboxylase substance is an indicator of such avitaminosis,
when it is excreted with urine?
14. A lab test revealed increased concentration A. Pyruvic acid
of leucine, valine, isoleucine and ketone B. Malate
derivatives in the blood and urine. The urine has C. Methylmalonic acid
a specific smell of maple syrup. Which enzyme D. Uric acid
deficiency is typical for this condition? E. Phenyl pyruvate
A. Amino acid dehydrogenase
B. Aminotranspherase 19. A patient 43 years old with chronic atrophic
C. Glucose-6-phosphatase gastritis and hyperchromic megaloblastic
D. Phosphofructokinase anemia increased methylmalonic acid excretion
E. Phosphofructomutase in the urine. The lack of which vitamin caused
the occurrence of this symptom complex?
15. Excretion of homogentisic acid with the A. В12
urine is the reason for the condition called B. В2
alcaptonuria. Disturbance of which amino acid C. В3
metabolism causes this condition? D. В5
A. Tyrosine E. В6

60
components are revealed in course of laboratory
20. A hereditary disease - homocystinuria - is examination of the patient’s urine?
caused by disturbed transformation of A. Glucose, ketone bodies
homocysteine in to methionine. Accumulated B. Protein, aminoacids
homocysteine forms its dimer (homocystine) C. Protein, creatine
that can be found in urine. What vitamin D. Bilirubin, urobilin
preparation can decrease homocysteine E. Blood
production?
A. Vitamin B12 25. A patient at the early stage of diabetes
B. Vitamin C mellitus was found to have polyuria.What is its
C. Vitamin B1 cause?
D. Vitamin B2 A. Hyperglycemia
E. Vitamin PP B. Ketonemia
C. Hypocholesterolemia
21. In a newborn child there is a decrease in D. Hypercholesterolemia
intensity of sucking, frequent vomiting, E. Hyperkaliemia
hypotension. In urine and blood, the
concentration of citrulline is significantly 26. At the reception to the therapist came a man
increased. What metabolic process is affected? of 37 years of age with complaints of periodic
A. Ornithine cycle intense pain in the joints of the thumb and their
В. Glycolysis swelling. In the urine: slightly acidic reaction
C. Core's cycle and pink color. With the presence of which
D. СТК substances can these changes be associated?
E. Gluconeogenesis A. Salt of uric acid
B. Chlorides
22. In the urine of the newborn, determined by C. Amoniev's salt
citrulline and high levels of ammonia. Indicate D. Phosphate calcium
which substance is most likely to be inflicted on E. Magnesium sulphate
a baby?
A. Urea. 27. On the basis of laboratory analysis, the
B. Bilirubin. patient was diagnosed with gout. Based on
C. Creatine. laboratory analysis, the patient confirmed the
D. Creatinine. diagnosis of gout. What was the analysis for the
E. Uric acid. diagnosis?
A. Determination of uric acid in the blood and
23. According to the results of glucose tolerance urine
test a patient has no disorder of carbohydrate B. Determination of urinary creatinine
tolerance. Despite that glucose is detected in the C. Determination of residual nitrogen in the
patients’s urine (5 mmol/l). The patient has been blood
diagnosed with renal diabetes. What renal D. The determination of urea in the blood and
changes cause glucosuria in this case? urine
A. Decreased activity of glucose reabsorption E. Determination of urine ammonia
enzymes
B. Increased activity of glucose reabsorption 28. A patient has been operated due to acute
enzymes abdomen. His urine is brown, concentration of
C. Exceeded glucose reabsorption threshold indican is higher than 93 mmol/day. What does
D. Increased glucose secretion it indicate?
E. Increased glucose filtration A. High intensity of protein decomposition
processes in the intestine
24. A 42 year old woman diagnosed with B. Decreased activity of the urea cycle enzymes
diabetes mellitus was admitted the C. Acceleration of aromatic amino acids
endocrinological department with complaints of desamination
thirst, excessive appetite. What pathological D. Disturbance of the kidney absorption

61
E. Decrease of ammonia detoxification
34. Analysis of a dentist’s urine obtained at the
29. A 43 year-old woman has been operated due end of his working day revealed protein
to acute abdomen. Her urine is brown and the concentration at the rate of 0,7 g/l. His morning
concentration of indican in the blood has urine hadn’t such changes. What is this
increased rapidly. What does it indicate? phenomenon called?
A.High intensity of protein decomposition A. Functional proteinuria
B. Acceleration of amino acids desamination B. Organic proteinuria
C. Supression of glomerular filtration C. Nonselective proteinuria
D. Decreased intensity of the urea cycle D. Extrarenal proteinuria
E. Inhibition of gluconeogenesis E. Hematuria

30. A 60 year-old man suffers from chronic 35. Chronic glomerulonephritis was diagnosed
intestinal obstruction. The processes of protein in a 34-year-old patient 3 years ago. Edema has
decomposition in the large intestine are developed within the last 6 monthes. What
intensified. What is the evidence of these caused the edema?
processes? A. Proteinuria
A. Indicanuria B. Hyperproduction of vasopressin
B. Bilirubinuria C. Liver disfunction of protein formation
C. Hyperuricosuria D. Hyperosmolarity of plasma
D. Creatinuria E. Hyperaldosteronism
E. Glucosuria
36. Glucose concentration in a patient’s blood is
31. A newborn child gains weight very slowly, 15 millimole/l (reabsorption threshold is 10
his urine contains too much orotic acid that is millimole/l). What effect can be expected?
indicative of disturbed synthesis of pyrimidine A. Glucosuria
nucleotides. What metabolite should be used in B. Diuresis reduction
order to normalize metabolism? C. Reduced glucose reabsorption
A. Uridine D. Reduced vasopressin secretion
B. Adenosine E. Reduced aldosterone secretion
C. Guanosine
D. Thymidine 37. A patient with rheumatoid arthritis has been
E. Histidine given hydrocortisone for a long time. He has
developed hyperglycemia, polyuria, glycosuria,
32. In orоtaciduria the release of Orotic acid is thirst. These complications of treatment result
many times higher than normal. Synthesis of from the activation of the following process:
what substances will be disturbed in this A. Gluconeogenesis
pathology? B. Glycogenolysis
A. Pyrimidine nucleotides. C. Glycogenesis
B. Biogenic Amines. D. Glycolysis
C. Purine nucleotides. E. Lipolysis
D. Urea
E. Uric acid 38. A 28-year-old patient with Itsenko-Cushing
syndrome Hyperglycemia, glycosuria were
33. The patient has increased photosensitivity, detected hyperglycemia, glycosuria. The main
after prolong sun exposure urine starts to get mechanism of hyperglycemia in this patient is
dark-red color. Which type of disease has this stimulation:
characteristic? A. Gluconeogenesis
A. Porphyria B. Liver glycogenolysis
B. Alkaptonuria C. Muscle glycogenolysis
C. Albinism D. Intestinal glucose absorption
D. Hemolytic jaundice E. Synthesis of glycogen
E. Pellagra

62
39. Fructosuria is known to be connected with E. Sphingomyelinase
inherited deficiency of fructose-1-phosphate
aldolase.What product of fructose metabolism 44. A patient presents with high content of
will accumulate in the organism resulting in vasopressin (antidiuretic hormone) in the blood.
toxicaction? What changes in the patient’s diuresis will
A. Fructose-1-phosphate occur?
B. Glucose-1-phosphate A. Oliguria
C. Glucose-6-phosphate B. Polyuria
D. Fructose-1,6-biphosphate C. Anuria
E. Fructose-6-phosphate D. Glycosuria
E. Natriuria
40. Prophylactic examination of a patient
revealed hyperglycemia, ketonuria, polyuria, 45. A patient with tress and painful sensation
glycosuria. What form of acidbase balance before a visit to the dentist is accompanied by
disorder is the case? anuria (lack of urination). This phenomenon is
A. Metabolic acidosis due to an increase in:
B. Gaseous acidosis A. Secretion of vasopressin and adrenaline
C. Nongaseous acidosis B. Activity of the parasympathetic nervous
D. Gaseous alkalosis system
E. Metabolic alkalosis C. Activity antinociceptive system
D. Secretion of vasopressin and a decrease in
41. Examination of a 2 year-old child with renal adrenaline
insufficiency revealed hyperoxaluria that caused E. Secretion of adrenaline and a decrease in
depositing of calcium oxalate stones in the vasopressin
kidneys. The reason for this condition is a
disturbance of metabolism of the following 46. A patient with pituitary tumor complains of
aminoacid: increased daily diuresis (polyuria). Glucose
A. Glycine concentration in blood plasma equals 4,8
B. Lysine mmol/l. What hormone can be the cause of this
C. Methionine if its secretion is disturbed?
D. Arginine A. Vasopressin
E. Histidine B. Aldosterone
C. Natriuretic hormone
42. Cardinal symptoms of primary D. Insulin
hyperparathyroidism are osteoporosis andrenal E. Angiotensin I
lesion along with development of urolithiasis.
What substance makes up the basis of these 47. After a person had drunk 1,5 liters of water,
calculi in this disease? the amount of urine increased significantly, and
A. Calcium phosphate its relative density decreased to 1,001. These
B. Uric acid changes are a result of decreased water
C. Cystine reabsorption in the distal nephron portion due to
D. Bilirubin reduced secretion of:
E. Cholesterol A. Vasopressin
B. Aldosterone
43. In the laboratory examination of blood of a C. Angiotensin II
person who was bitten by a change, hemolysis D. Renin
of erythrocytes, hemoglobinuria was revealed. E. Prostaglandins
The action of a mixed poison is due to the
presence of an enzyme in it: 48. A man has a considerable decrease in
A. Phospholipase A2 diuresis as a result of 1,5 l blood loss. The
B. Phospholipase D primary cause of such diuresis disorder is the
C. Phospholipase S. hypersecretion of the following hormone:
D. Phospholipase A1 A. Vasopressin

63
B. Corticotropin level of glucose in blood is normal, there is no
C. Natriuretic glucose in urine. What hormone deficit can
D. Cortisol cause such changes?
E. Parathormone A. Vasopressin
B. Oxytocin
49. Analysis of urine from a 24-year-old man C. Insulin
revealed the following changes: daily diuresis - D. Triiodothyronine
10 l, relative density - 1,001, qualitative E. Cortisol
alterations are absent. A patient complains of
excessive thirst, frequent urination. What is the 53. A 19-year-old male was found to have an
most likely cause of this disease? elevated level of potassium in the secondary urine.
A. Vasopressin hyposecretion These changes might have been caused by the
B. Glucocorticoid hypersecretion increase in the following hormone level:
C. Vasopressin hypersecretion A. Aldosterone
D. Relative insulin insufficiency B. Oxytocin
C. Adrenaline
E. Aldosteron hypersecretion D. Glucagon
E. Testosterone
50. A 50 year old patient complains of excessive
thirst, drinks a lot of water; expressed polyuria. 54. A concentrated solution of sodium chloride
Blood glucose - 4.8 mmol / l. There is no was intravenously injected to an animal. This
glucose and acetone bodies in urine, colorless caused decreased reabsorption of sodium ions in
urine, specific gravity - 1.002 - 1.004. What is the renal tubules. It is the result of the following
the cause of polyuria? changes of hormonal secretion:
A. Lack of vasopressin A. Aldosterone reduction
B. Hypothyroidism B. Aldosterone increase
C. Insulin Deficiency C. Vasopressin reduction
D. Aldosteronism D. Vasopressin increase
E. Thyrotoxicosis E. Reduction of atrial natriuretic factor
51. A patient with damage to the posterior lobe 55. At ria of a test animal were super distended
of the pituitary gland has increase in daily with blood,which resulted in decreased
diuresis to 10-15 liters What is the main reabsorption of Na+ and water in renal
mechanism in the development of polyuria? tubules.This can be explained by the effect of
A. Vasopressin deficiency the following factor on the kidneys:
B. Excess vasopressin A. Natriuretic hormone
C. Excess aldosterone B. Aldosterone
D. Excess natriuretic factor C. Renin
E. Corticotropin deficiency D. Angiotensin
E. Vasopressin
52. A 20 year old patient complains of excessive
thirst and urinary excretion upto 10 L a day. The

64
Biochemistry of nervous tissue
1. A substance that is synthesized in the central A. Hyperpolarization
nervous system and simulates the effects of B. Depolarization
morphine can be used for analgesia. Specify the C. Action potential
following substance: D. Local response
A. β-endorphin E. There will be no changes
B. Oxytocin
C. Vasopressin 7. A 50-year-old man came to a hospital with
D. Calcitonin complaints of memory disorders, painful
E. Somatoliberin sensations along the nerve trunks, decreased
mental ability, circulatory disorders and
2. Cell membrane restpotential changed from-85 dyspepsia. Anamnesis states excessive alcohol
to-90mV. It can becaused by activation of the consumption. What vitamin deficiency can
following cell membrane channels: result in such symptoms?
A. Potassium A. Thiamine
B. Sodium B. Niacin
C. Potassium and sodium C. Retinol
D. Calcium E. Potassium and calcium D. Calciferol
E. Riboflavin
3. The speed of the excitation nerve fibers is
120 m / s. Which of the above factors, above all, 8. Decarboxylation of glutamate induces
provides such speed? production of gamma aminobutyric acid
A. Presence of myelin sheath (GABA) neurotransmitter. After breakdown,
B. Great resting potential GABA is converted into a metabolite of the
C. Large amplitude potential effects citric acid cycle, that is:
D. Low threshold of depolarization A. Succinate
E. A big factor of reliability B. Citric acid
C. Malate
4. The patient is trembling hands, which is D. Fumarate
associated with Parkinson's disease. What E. Oxaloacetate
mediator deficiency in striopalidar structures
leads to such symptoms? 9. Degenerative changes in posterior and lateral
A. Dopamine columns of spinal cord (funicular myelosis)
B. GABA caused by methylmalonic acid accumulation
C. Substance P occur in patients with B12-deficiency anemia.
D. Norepinephrine This results in synthesis disruption of the
E. Serotonin following substance:
A. Myelin
5. An 84-year-old patient suffers from B. Acetylcholine
parkinsonism. One of the pathogenetic C. Norepinephrine
development elements of this disease is D. Dopamine
deficiency of a certain mediator in some of the E. Serotonin
brain structures. Name this mediator:
A. Dopamine 10. After pharmacological blockade ion
B. Adrenaline channels of the nerve fiber membrane resting
C. Noradrenaline potential decreased from -90 to -80 mV. What
D. Histamine channels were blocked?
E. Acetylcholine A. Potassium
B. Sodium
6. In the experiment, the permeability of the C. Calcium
excitable cell membrane to potassium ions was D. Magnesium
magnified. What changes in the electrical state E. Chloric
of the membrane will occur?
65
11. Disruption of nerve fiber myelinogenesis concentration increases in the brain under the
causes neurological disorders and mental action of antidepressants?
retardation. These symptoms are typical for A. Serotonin
hereditary and acquired alterations in the B. Dopamine
metabolism of: C. Glycine
A. Sphingolipids D. Taurine
B. Neutral fats E. Norepinephrine
C. Higher fatty acids
D. Cholesterol 16. Brain cells are very sensitive to energy
E. Phosphatidic acid deficiency, which can be caused by a high
content of ammonia, which stimulates the
12. A patient complained about dizziness, outflow of alpha-ketoglutarate from:
memory impairment, periodical convulsions. It A. Citric acid cycle
was revealed that these changes were caused by B. Ornithine cycle
a product of decarboxylation of glutamic acid. C. Glycolysis
Name this product: D. Glycogenolysis
A. GABA E. Pentose phosphate pathway
B. Pyridoxal phosphate
C. TDP 17. Depressions and emotional insanities result
D. ATP from the deficit of noradrenalin, serotonin and
E. UDP other biogenic amines in the brain. Their
concentration in the synapses can be increased
13. A patient presents with dysfunction of the by means of the antidepressants that inhibit the
cerebral cortex accompanied by epileptic following enzyme:
seizures. He has been administered a biogenic A. Monoamine oxidase
amine synthetized from glutamate and B. Diamine oxidase
responsible for central inhibition. What C. L-amino-acid oxidase
substance is it? D. D-amino-acid oxidase
A. γ-aminobutyric acid E. Phenylalanine-4-monooxygenase
B. Serotonin C.
С. Dopamine 18. Pharmacological effects of antidepressants
D. Acetylcholine are based upon blocking (inhibiting) the enzyme
E. Histamine that acts as a catalyst for the breakdown of
biogenic amines noradrenalin and serotonin in
14. An unconscious patient was taken by the mitochondria of cephalic neurons. What
ambulance to the hospital. On objective enzyme takes part in this process?
examination the patient was found to have no A. Monoamine oxidase
reflexes, periodical convulsions, irregular B. Transaminase
breathing. After laboratory examination the C. Decarboxylase
patient was diagnosed with hepatic coma. D. Peptidase
Disorders of the central nervous system develop E. Lyase
due to the accumulation of the following
metabolite: 19. A 9-month-old infant is fed with artificial
A. Ammonia formulas with unbalanced vitamin B6
B. Urea concentration. The infant presents with pellagral
C. Glutamine dermatitis, convulsions, anaemia. Convulsion
D. Bilirubin development might be caused by the disturbed
E. Histamine formation of:
A. GABA
15. For the treatment of depression of different B. Histamine
genesis, antidepressants are used, which are C. Serotonin
monoamine oxidase inhibitors. What substance D. DOPA
is a "neurotransmitter of well-being" and its E. Dopamine

66
inhibitor aldehyde dehydro- nazy The increase
20. Cerebral trauma caused increase of in blood of what tabolita aversion to al to
ammonia formation. What aminoacid takes part someone?
in removal of ammonia from A. Acetaldehyde
cerebral tissue? B. Ethanol
A. Glutamic C. Malonic aldehyde
B. Tyrosine D. Propionic aldehyde
C. Valine E. Methanol
D. Tryptophan
E. Lisine 25. Soldiers who were injured in the midst of
the battle may not be aware of the pain until it is
21. Ammonia is a very toxic substance, completed. What hormones are opiate
especially for nervous system. What substance antinociceptive system reduce the sensation of
takes the most active part in ammonia pain?
detoxication in brain tissues? A. Endorfines
A. Glutamic acid B. Serotonines
B. Lysine C. Vasopressin
C. Proline D. Aldosterone
D. Histidine E. Oxytocine
E. Alanine
26. In an excitable cell the ion channels were
22. A patient presented to a hospital with blocked. It hasn’t changed essentially the value
complaints about quick fatigability and of rest potential, but the cell lost its ability to
significant muscle weakness. Examination generate AP (action potential). What channels
revealed an autoimmune disease that were blocked?
causes functional disorder of receptors in the A. Natrium
neuromuscular synapses. This will result in the B. Potassium
disturbed activity of the following mediator: C. Natrium and potassium
A. Acetylcholine D. Chloric
B. Noradrenaline E. Calcium
C. Dopamine
D. Serotonin 27. The infant has epileptiform convulsions
E. Glycine caused by vitamin B6 deficiency. This is due to
a decrease in the nervous tissue of the inhibitory
23. Glutamate decarboxylation results in mediator, γ-aminobutyric acid. What is the
formation of inhibitory transmitter in CNS. enzyme's activity reduced in this case?
Name it: A. Glutamate decarboxylase
A. GABA B. Alanine aminotransferase
B. Glutathione C. Glutamate dehydrogenase
C. Histamine D. Pyridoxal kinase
D. Serotonin E. Glutamate Synthetase
E. Asparagine

24. In medical practice for alcoholism


prevention widely used teturam, which is

67
Biochemistry of muscle tissue
1. Buffer capacity of a worker’s blood was
decreased due to exhausting muscular work. By 6. A 35-year-old man developed acute heart
coming of what acid substance in the blood can failure while running for a long time. What
this state be explained? changes in ionic composition can be observed in
A. Lactate the cardiac muscle?
B. Pyruvate A. Accumulation of Na+ and Ca2+ ions in the
C. 1,3-bisphosphoglycerate myocardium cells
D. α-ketoglutarate B. Accumulation of K+ and Mg2+ ions in the
E. 3-phosphoglycerate myocardium cells
C. Reduction of Na+ and Ca2+ ions in the
2. A biochemical urine analysis has been myocardium cells
performed for a patient with progressive D. Reduction of K+ and Mg2+ ions in the
muscular dystrophy. In the given case muscle extracellular space
disease can be confirmed by the high content of E. Reduction of Na+ and Ca2+ ions in the
the following substance in urine: extracellular space
A. Creatine
B. Porphyrin 7. A patient is diagnosed with cardiac infarction.
C. Urea Blood test for cardiospecific enzymes activity
D. Hippuric acid was performed. Which of the enzymes has three
E. Creatinine isoforms?
A. Creatine kinase
3. A considerable increase of activity of МВ- B. Lactate dehydrogenase
forms of CPK (creatinephosphokinase) and C. Aspartate transaminase
LDH-1 was revealed on the examination of D. Alanine transaminase
patient’s blood. What is the most likely E. Pyruvate kinase
pathology?
A. Miocardial infarction 8. A 50-year-old woman diagnosed with cardiac
B. Hepatitis infarction has been delivered into an intensive
C. Rheumatism care ward. What enzyme will be the most active
D. Pancreatitis during the first two days?
E. Cholecystitis A. Aspartate aminotransferase
B. Alanineaminotransferase
4. With intense physical work, muscle tissue C. Alanineaminopeptidase
accumulates lactic acid, which diffuses into the D. LDH4
blood and is absorbed by the liver and the heart. E. LDH5
What process ensures the recovery of glycogen
stores in the muscles? 9. Untrained people often have muscle pain
A. The Cori Cycle after sprints as a result of lactate accumulation.
B. Citric Acid Cycle This might be caused by intensification of the
C. Urea Cycle following biochemical process:
D. A Cycle of Tricarboxylic Acids A. Glycolysis
E. Hexose Monophosphate Pathway B. Gluconeogenesis
C. Pentose phosphate pathway
5. A sportsman needs to improve his sporting D. Lipogenesis
results. He was recommended to take a E. Glycogenesis
preparation that contains carnitine. What
process is activated the most by this compound? 10. A patient with suspected diagnosis
A. Fatty acids transporting "progressing muscular dystrophy"got his urine
B. Amino acids transporting tested.What compound will confirm this
C. Calcium ions transporting diagnosis if found in urine?
D. Glucose transporting A. Kreatine
E. Vitamin K transporting B. Collagen
68
C. Porphyrin C. Acetyl CoA
D. Myoglobin D. Glucose 6-phosphate
E. Calmodulin E. Oxaloacetate

11. Rheography of an 18 year old student during 16. A 49-year-old driver complains about
exercise showed redistribution of blood flow unbearable constricting pain behind the
between organs. The peak blood flow will be breastbone irradiating to the neck. The pain
observed in the following vessels: arose 2 hours ago. Objectively: the patient’s
A. Skeletal muscles condition is grave, he is pale, heart tones are
B. Liver decreased. Laboratory studies revealed high
C. Cerebrum activity of creatine kinase and LDH1. What
D. Kidneys disease are these symptoms typical for?
E. Gastrointestinal tract A. Acute myocardial infarction
B. Acute pancreatitis
12. A 46-year-old female patient has continuous C. Stenocardia
history of progressive muscular (Duchenne’s) D. Cholelithiasis
dystrophy. Which blood enzyme changes will E. Diabetes mellitus
be of diagnostic value in this case?
A. Creatine phosphokinase 17. Cytoplasm of the myocytes contains a lot of
B. Lactate dehydrogenase dissolved metabolites resulting from glucose
C. Pyruvate dehydrogenase oxidation. Name the metabolite that turns
D. Glutamate dehydrogenase directly into lactate:
E. Adenylate cyclase A. Pyruvate
B. Oxaloacetate
13. A 50-year-old patient complains about C. Glycerophosphate
general weakness, appetite loss and cardiac D. Glucose-6-phosphate
arrhythmia. The patient presents with muscle E. Fructose-6-phosphate
hypotonia, flaccid paralyses, weakened
peristaltic activity of the bowels. Such condition 18. A patient came to the hospital complaining
might be caused by: about quick fatigability and apparent muscle
A. Hypokaliemia weakness. Examination revealed an
B. Hypoproteinemia autoimmune disease that causes disorder of
C. Hyperkaliemia functional receptor condition in neuromuscular
D. Hypophosphatemia synapses. What transmitter will be blocked?
E. Hyponatremia A. Acetylcholine
B. Noradrenalin
14. Some students developed myodynia after C. Dopamine
continuous physical activity during physical D. Serotonin
education. The reason for such condition was E. Glycine
accumulation of lactic acid in the skeletal
muscles. It was generated in the students’ 19. During processing of the atypical
bodies after activation of the following process: cardiomyocytes of the synaptic node with a
A. Glycolysis biologically active substance, an increase in
B. Gluconeogenesis their membrane potential was recorded due to
C. Pentose-phosphate cycle increased permeability for potassium ions. What
D. Lipolysis biologically active substance affects
E. Glycogeny cardiomyocytes?
A. Acetylcholine
15. After a sprint an untrained person develops B. Adrenalin
muscle hypoxia. This leads to the accumulation C. Noradrenaline
of the following metabolite in muscles: D. Thyroxine
A. Lactate E. Atriopeptide
B. Ketone bodies

69
20. 12 hours after an accute attack of A. Transporting of fatty acids to mitochondrions
retrosternal pain a patient presented a jump of B. Regulation of Ca2+ rate in mitochondrions
aspartate aminotransferase activity in blood C. Substrate phosphorylation
serum. What pathology is this deviation typical D. Lactic acid utilization
for? E. Actin and myosin synthesis
A. Myocardium infarction
B. Viral hepatitis 24. After a sprint an long distantion the skeletal
C. Collagenosis muscle training person uses glucose for the
D. Diabetes mellitus purpose of obtaining energy ATP for muscle
E. Diabetes insipidus contraction. Specify basic glucose utilization
process in these conditions:
21. A patient presents high activity of LDH 1,2, A. Aerobic glycolysis
aspartate aminotransferase, creatine B. Anaerobic glycolysis
phosphokinase. In what organ (organs) is the C. Glycogenolysis
development of a pathological process the most D. Gluconeogenesis
probable? E. Glycogenesis
A. In the heart muscle (initial stage of
myocardium infarction) 25. The gluconeogenesis is activated in the liver
B. In skeletal muscles (dystrophy, atrophy) after intensive physical trainings.What
C. In kidneys and adrenals substance is utilized in gluconeogenesis first of
D. In connective tissue all in this case:
E. In liver and kidneys A. Lactate
B. Pyruvate
22. A traumatology unit received a patient with C. Glucose
crushed muscular tissue. What biochemical D. Glutamate
indicator of urine will be raised in this case? E. Alanine
A. Creatinine
B. Total lipids 26. 6 hours after the myocardial infarction a
C. Glucose patient was found to have elevated level of
D. Mineral salts lactate dehydrogenase in blood. What is enzyme
E. Uric acid should be expected in this case?
A. LDH1
23. A 1 y.o. child with symptoms of muscle B. LDH2
affection was admitted to the hospital. C. LDH3
Examination revealed carnitine deficit D. LDH4
inmuscles. Biochemical base of this pathology E. LDH5
is disturbed process of:

70
Biochemistry of connective tissue
1. Examination of a patient revealed typical C. Nucleotides
presentations of collagenosis. This pathology is D. Vitamins
characterized by increase of the following urine E. Phospholipids
index:
A. Hydroxyproline 6. A patient in the dental department has been
B. Arginine diagnosed with Paget's disease, which is
C. Glucose accompanied by collagen degradation. The
D. Mineral salts decisive fact for the diagnosis was the detection
E. Ammonium salts in the patient's urine of an elevated level:
A. Hydroxproline
2. A child has physical and mental retardation, B. Arginine
serious abnormalities in connective tissue of C. Tryptophan
internal organs; urine contains keratan sulfates. D. Serina
This is caused by metabolic disorder of the E. Alanina
following substance:
A. Glycosaminoglycan 7. A 60 year old man complains about in the
B. Collagen joints. In the serum of the patient, the
C. Elastin concentration of C-reactive protein and
D. Fibronectin oxyproline was found to increase. what disease
E. Hyaluronic acid these symptoms can be associated with:
A. Rheumatism
3. Inherited diseases, such as B. Gout
mucopolysaccharidoses, are manifested in C. Hepatitis
metabolic disorders of connective tissue, bone D. Jaundice
and joint pathologies. The sign of this disease is E. Diabetes
the excessive urinary excretion of the following
substance: 8. A 36-year-old female patient has a history of
A. Glycosaminoglycans collagen disease. Urine analysis is likely to
B. Amino acids reveal an increased concentration of the
C. Glucose following metabolite:
D. Lipids A. Oxyproline
E. Urea B. Indican
C. Creatinine
4. A 28 year old pregnant woman had the D. Urea
enzymes in the cells of amniotic fluid analyzed. E. Urobilinogen
The analysis revealed insufficient activity of β-
glucuronidase. What pathological process is it? 9. A 30-year-old woman first developed pain,
A. Mucopolysaccharidosis swelling, and skin redness in the area of joints
B. Glycogenosis about a year ago. Provisional diagnosis is
C. Aglycogenosis rheumatoid arthritis. One of the likely causes of
D. Collagenosis this disease is change in the structure of the
E. Lipidosis following connective tissue protein:
A. Collagen
5. The five year old boy was observed small B. Mucin
stature, mental retardation, limited movement, C. Myosin
rough facial features. These features have D. Ovalbumin
become noticeable from 18 months of age. He E. Troponin
was diagnosed with a deficiency of L-
iduronidase. The exchange of which compounds 10. Collagenosis patients typically present with
is broken? connective tissue destruction processes.The
A. Glycosminoglycans presence of the seprocesses can beconfirmed by
B. Proteins the increasein:
71
A. Blood oxyproline and oxylysine alkaline phosphatase in the intercellular
B. Blood creatine and creatinine substance. What cell produces this enzyme?
C. LDH-isoenzyme activity in the blood A. Osteoblast
D. Transaminase activity in the blood B. Osteocyte
E. Blood urates C. Osteoclast
D. Chondroblast
11. A 53-year-old male patient is diagnosed E. Chondrocyte
with Paget’s disease. The concentration of
oxyproline in daily urine is sharply increased, 16. When a wound heals, a scar takes its place.
which primarily means intensified disintegration What substance is the main component of its
of: connective tissue?
A. Collagen A. Collagen
B. Keratin B. Elastin
C. Albumin C. Keratan sulfate
D. Hemoglobin D. Chondroitin sulfate
E. Fibrinogen E. Hyaluronic acid

12. A 63-year-old woman developed symptoms 17. A patient with systemic scleroderma has an
of rheumatoid arthritis. Their increase of which intensified collagen destruction. Collagen
blood values indicators could be the most destruction will be reflected by intensified
significant in proving the diagnosis? urinary excretion of the following amino acid:
A. Additive glycosaminoglycans A. Oxyproline
B. Lipoproteids B. Alanine
C. Acid phosphatase C. Tryptophane
D. General cholesterol D. Serine
E. R-glycosidase E. Phenylalanine

13. Osteolaterism is charcterized by a decrease 18. A patient who is ill with scurvy displays
in collagen strength caused by much less disturbed processes of connective tissue
intensive formation of crosslinks in collagen formation that leads to loosening and falling of
fibrils. This phenomenon is caused by the low teeth. Disturbed activity of what enzyme causes
activity of the following enzyme: these symptomps?
A. Lysyl oxidase A. Lisilhydroxylase
B. Monoamino-oxidase B. Glycosiltransferase
C. Prolyl hydroxylase C. Elastase
D. Lysyl hydroxylase D. Procollagenpeptidase of N-terminal peptide
E. Collagenase E. Procollagenpeptidase of C-terminal peptide

14. A 34-year-old patient has a history of 19. In spring a patient experiences petechial
periodontitis. As a result of increased collagen haemorrhages, loosening of teeth, high liability
degradation, there is a significantly increased to colds. A doctor supposes hypobitaminosis C.
urinary excretion of one of the amino acids. In this respect loosening of teeth can be
Which one? explained by:
A. Hydroxyproline A. Structural failure of collagen in the
B. Valine periodontal ligaments
C. Alanine B. Structural change of glycosaminoglycan
D. Glycine C. Increased permeability of periodont
E. Serine membranes
D. Mechanical damage of teeth
15. Calcification of the intercellular substance E. Disturbed oxidation-reduction process in the
of bone tissue is accompanied by the deposition periodont
of hydroxyapatite crystals along the collagen
fibers. This process requires the presence of

72
20. Increased fragility of vessels, enamel and B. Formation of polypeptide chains
dentine destruction resulting from scurvy are C. Glycosylation of hydroxylysine residues
caused by disorder of collagen maturation. What D. Removal of C-ended peptide from
stage of procollagen modification is disturbed procollagen
under this avitaminosis? E. Detaching of N-ended peptide
A. Hydroxylation of proline

73
Biochemistry of tooth and saliva
1. What enzyme has a demineralizing effect - 6. The periodontal dentist must be assessed in
enhances the cleavage of the mineral the patient by factors of non-specific resistance
components of the tissues in the tooth? of saliva and excretory mucous membrane of
A. Sulfate phosphatase the oral cavity. What factor of non-specific
B. Alkaline phosphatase resistance should first be studied in the
C. Glucose-6-phosphatase investigated material?
D. Glucogenphosphorylase A. Lysozyme
E. Phosphotransferase B. Secretory IgA
C. Properdine
2. The hydroxyapatite crystals are deposited D. Interferon
along the collagen fibers in the process of E. Complement
grafting the intercellular substance of the tissue.
In order to realize this process, the presence of 7. What substance gives slime a viscous mucous
intercellular alkaline phosphatase must be character, performs a protective role and
present. What kind of creatine produces this including from mechanical damage to the oral
enzyme? mucosa?
A. Osteoblast A. Mutsin
B. Osteocyte B. Glucose
C. Osteoclasts C. Kalikrein
D. Hondroblaste D. Amilaza
E. Chondrocyte E. Lysozyme

3. The pH of saliva is 6.4 - 7.8 it is in the norm. 8. Some beards of saliva perform a protective
What changes in the enamel leads to a shift in function. Which one protects the oral mucosa
the pH of saliva in the sour beet (less than 6.2)? from mechanical damage?
A. Demineralization A. Mutsin
B. Calcification B. Lysozyme
C. Fluorosis C. Catalase
D. Mineralization D. Peroxidase
E. Strengthening of stability E. Renin

4. Cationic glycoproteins are the main 9. The patient with chronic inflammation of
components of salivation of the parotid glands. submaxilar salivary glands have observed
What Amino Acids cause their positive charge? giposalivation. What violation of the increment
A. Lizin, arginine, hististine of which biologically active substance is
B. Aspartate, glutamate, glucine observed at this process?
C. Aspartate, arginine, glutamate A. Parotin
D. Glutamate, valine, leucine B. Calcitonin
E. Cysteine, glucine, proline C. Paratyrin
D. Gluckagon
5. The protective function of saliva is due to E. Somatostatin
several mechanisms, including the presence of
an enzyme that has bactericidal action, causes 10. Patient with symptoms of reduced excretory
lysis of the polysaccharide complex of the function of the kidneys, an unpleasant smell
staphylococcal shell, streptococci. Indicate this from the mouth is indicated. What substance are
enzyme: increased excretion of the salivary glands is the
A. Lysozyme cause of this?
B. α-amylase A. Urea
C. Oligo-1,6-glucosidase B. Alpha amylase
D. Collagenase C. Lysozyme
E. β-glucuronidase D. Phosphatase
E. Mutsin
74
16. A 60 year old patient was found to have a
11. The child has acute renal insufficiency. dysfunction of main digestive enzyme of saliva.
What biochemical indicators of saliva can be This causes the disturbance of primary
confirmed? hydrolysis of:
A. Increasing the level of residual nitrogen A. Carbohydrates
B. Increase in immunoglobulin A. B. Fats
C. Reduction of alkaline phosphatase C. Proteins
D. Increase in alpha amylase D. Cellulose
E. Reducing the phosphate level E. Lactose

12. Periodontitis is accompanied by activation 17. It is necessary to evaluate the digestible


of proteolysis in periodontal tissues. The properties of saliva. With what substrate for this
increase of which oral fluid component testifies it needs to be shifted?
to the activation of proteolysis? A. Starch
A. Amino acids B. Casein
B. Organic acids C. Fat
C. Glucose D. DNA
D. Biogenic Amines E. RNA
E. Cholesterol
18. When investigating human saliva it is
13. When treated with hydrogen peroxide in the necessary to assessits hydrolytic properties.
mucous membrane of the patient's oral cavity, What substance should be used as a substrate in
the blood was painted in a brown color instead the process?
of pene formation. When the concentration of A. Starch
any of the enzymes listed below is reduced, is it B. Proteins
possible? C. Fats
A. Catalase D. Fiber
B. Pseudocholinesterase E. Amino acids
C. Glucose-6-phosphate dehydrogenase
D. Acetyltransferase 19. Afther using cookies, sweets in mixed
E. Methemoglobin reductase saltines temporarily increases the level of
lactate. The activation of which biomic process
14. In periodontites, lipid peroxidation develops leads to this?
in the periodontal tissues, and the content of A. Anaerobic digestion
malonic dialdehyde, hydrogen peroxide B. Tissue breathing
increases in the oral cavity. Which of the C. Aerobic Glucose
enzymes is antioxidant protection? D. Gluconeogenesis
A. Superoxide dismutase, catalase E. Microsomal oxidation
B. Amilase, trypsin
C. Maltase, Chymotrypsin 20. In the saliva of patient is increased content
D. Lactase, lysozyme of lactate was detected. Activation of a process
E. Saharaza, prothrombin. is the main reason for the increase of lactate?
A. Anaerobic decomposition of glucose
15. With age, the activity of the peripheral B. Aerobic decomposition of glucose
salivary glands decreases. Which does an C. Decomposition of glycogen
enzyme's activity decrease in slime? D. Hydrolysis of carbohydrates
A. Amilaza E. Glucose-lactate cycle
B. Lysozyme
C. Phosphatase 21. To form a mineral matrix of hard tooth
D. Geksokinase tissues, a high concentration of phosphate ions
E. Malthus is required, which is formed during the process
of hydrolysis of phosphorous-bonded bonds
with the participation of alkaline phosphatase.

75
What ions of the metal are the triggers of this D. Calcium chloride
process: E. Kalia bromide
A. Zinc
B. Magnium 27. A patient with renal insufficiency developed
C. Iron osteodystrophy, which is accompanied by
D. Calcium intensive demineralization of the hips. Which
E. Natrium active form of vitamin violation’s of the
formation is the cause of this complication?
22. In economically developed countries, A. Calcipherol
common caries is a cureus of teeth. This disease B. Retinol
affects more than 95% of the population. What C. Thiamin
role plays in the demineralization of hard tooth D. Naftohinon
tissues in caries? E. Ryboflavin
A. Organic acids
B. Malnutrition 28. The enamel is characterized by high stability
C. Extreme effects on the organism to the action of various mechanical and
D. Disturbance of regulation of metabolism chemical factors. Which component is synthesis
E. Insufficient vitamin C of provides such resistance?
A. Fluorapatite
23. The high excess concentration of glucose in B. Hydroxyapatite
oral solution in diabetes leads to development: C. Chlorapatite
A. Multiple caries D. Collagen
B. Hyperplasia enamel E. Carbonate apatite
C. Hypoplasia of enamel
D. Fluorosis 29. Calcification of dental tissues is
E. Enhanced calcification of enamel significantly influenced by osteocalcin protein
which has an ability to bind calcium ions due to
24. In the Transcarpathian settlement, after the presence of the following modified amino
prophylactic examination of many children have acid residues in the polypeptide chain:
found a multiple of caries. With mineral of the A. γ-carbon glutamine
insufficiency in food you can link development B. Alanine
of careers? C. γ-aminobutyric
A. Fluoride D. Carboxy aspargine
B. Iodine E. δ-aminopropionic
C. Molibden
D. Iron 30. A 35-year-old patient consulted a dentist
E. Cobalt about low density of dental tissues, increased
fragility of teeth on eating solid food. In order to
25. A child has disturbed processes of determine Ca/P relation a scrape of enamel was
ossification and "punctate"enamel. What sent to the laboratory. What value of this index
microelement metabolism is disturbed? is suggestive of intensified demineralization?
A. Fluorine A. 0,9
B. Iron B. 1,67
C. Zinc C. 1,85
D. Chromium D. 2,5
E. Copper E. 1,5

26. A saline metal was assigned for salvage 31. The preparation comlex for periodontitis
therapy at the initial caries of the teeth. Which treatment includes the medicine from the group
drug is determinated of this process? of water soluble vitamins, bioflavonide
A. Sodium fluoride derivative, which is prescribed together with
B. Sodium bromide ascorbic acid. This preparation has anti-
C. Sodium chloride

76
oxidative properties, decreases gingival E. Nicotinamide
hemorrhage. What preparation is meant?
A. Rutin 36. The child has a delayed teething, their
B. Calcium pantothenate wrong arrangement, upon examination
C. Calcium panganate noticeable dryness of the oral cavity, in the
D. Cyanocobalamin corners of the mouth there are cracks with
E. Folic acid suppuration. With the lack of the vitamin this
condition may be related?
32. Examination of a child who hasn’t got fresh A. Vitamin D.
fruit and vegetables during winter revealed B. Vitamin C.
numerous subcutaneous hemorrhages, C. Vitamin E.
gingivitis, carious cavities in teeth. What D. Vitamin K.
vitamin combination should be prescribed in E. Vitamin A.
this case?
A. Ascorbic acid and rutin 37. In a child of the first year of life is observed
B. Thiamine and pyridoxine an increase in the size of the head and abdomen,
C. Folic acid and cobalamin late teething, violation of the enamel structure.
D. Riboflavin and nicotinamide The consequence of vitamin deficiencies are
E. Calciferol and ascorbic acid these changes?
A. Hypovitaminosis D
33. The patient has an increase in the B. Hypovitaminosis C
permeability of the walls of blood vessels with C. Hypovitaminosis A
the development of increased bleeding and the D. Hypovitaminosis B1
appearance of small point hemorrhages on the E. Hypovitaminosis B2
skin, tooth loss. How does the vitamin exchange
disorder explain these symptoms? 38. A child with renal insufficiency exhibits
A. Hypovitaminosis C delayed teeth eruption. This is mostlikely
B. Hypervitaminosis D caused by the abnormal formationof the
C. Hypervitaminosis C following substance:
D. Hypovitaminosis D A. 1,25 (OH)2D3
E. Hypovitaminosis A B. Glycocyamine
C. Glutamate
34. In examining the oral cavity of the patient, D. α-ketoglutarate
the dentist paid attention to the presence of an E. Hydroxylysine
inflammatory and dystrophic process in the
mucous membrane (Hunter’s glossitis, atrophic 39. A patient has enamel erosion. What vitamin
stomatitis). A blood test revealed hyperchromic should be administered for itstreatment?
anemia. What factor is the cause of this disease? A. D3
A. Hypovitaminosis B12 B. C
B. Hypovitaminosis B1 C. K
C. Hypovitaminosis B6 D. B1
D. Increasing pH of the gastric juice E. PP
E. Hypovitaminosis A
40. A 5 years old child has insufficient
35. 10 month old child has high excitability, calcification of enamel, tooth decay. Which
sleep disturbance, amyotonia, retarded dentition, vitamin hypovitaminosis leads to the
teeth erupt withinadequate enamel calcification. development of this process?
Thesechanges are caused by deficiency of A. Calciferol
thefollowing vitamin: B. Tocopherol
A. Cholecalciferol C. Biotin
B. Riboflavin D. Nicotinic acid
C. Thiamine E. Folic acid
D. Retinol

77
41. Hormonal form of a certain vitamin induces B. Calciferol
genome level synthesis of Ca binding proteins C. Thiamine
and enterocytes thus regulating the intestinal D. Biotin
absorption of Ca2+ ions required for dental E. Phyloquinone
tissuedevelopment. What vitamin is it?
A. D3 44. There are various diseases that cause sharp
B. A increase of active oxygen, leading tocell
C. B1 membranes destruction. Antioxidantsare used to
D. E prevent it from happening. Themost potent
E. K natural antioxidant is:
A. Alpha-tocopherol
42. While the examination of patient’s oral B. Glycerol
cavity the dentist found xerostomia, numerous C. Vitamin D
erosions. What vitamin deficit caused this D. Fatty acids
effect? E. Glucose
A. Vitamin A
B. Vitamin K 45. When treating sialadenitis (inflammation of
C. Vitamin P the salivary glands), preparations of vitamins
D. Vitamin H are used. Which of the following vitamins plays
E. Vitamin PP an important role in antioxidant defense?
A. Tocopherol
43. In order to prevent gum inflammation and to B. Pantothenic acid
improve regeneration of epithelial periodontium C. Riboflavin
cells manufacturers add to the tooth pastes one D. Thiamine
of the following vitamins: E. Pyridoxine
A. Retinol

78
1. Buffer capacity of a worker’s blood was decreased due to
exhausting muscular work. By coming of what acid substance
in the blood can this state be explained?

• alpha-ketoglutarate
• 1,3-bisphosphoglycerate
• 3-phosphoglycerate
• Pyruvate
• Lactate

2. A biochemical urine analysis has been performed for a


patient with progressive muscular dystrophy. In the given case
muscle disease can be confirmed by the high content of the
following substance in urine:

• Hippuric acid
• Creatine
• Porphyrin
• Urea
• Creatinine

3. A considerable increase of activity of ??-forms of CPK


(creatinephosphokinase) and LDH-1 was revealed on the
examination of patient’s blood. What is the most likely
pathology?

• Miocardial infarction
• Pancreatitis
• Cholecystitis
• Rheumatism
• Hepatitis
4. With intense physical work, muscle tissue accumulates
lactic acid, which diffuses into the blood and is absorbed by
the liver and the heart. What process ensures the recovery of
glycogen stores in the muscles?

• Urea Cycle
• The Cori Cycle
• Citric Acid Cycle
• A Cycle of Tricarboxylic Acids
• Hexose Monophosphate Pathway

5. A sportsman needs to improve his sporting results. He was


recommended to take a preparation that contains carnitine.
What process is activated the most by this compound?

• Amino acids transporting


• Glucose transporting
• Vitamin K transporting
• Fatty acids transporting
• Calcium ions transporting

6. A 35-year-old man developed acute heart failure while


running for a long time. What changes in ionic composition
can be observed in the cardiac muscle?

• Reduction of K+ and Mg2+ ions in the extracellular space


• Reduction of Na+ and Ca2+ ions in the myocardium cells
• Accumulation of K+ and Mg2+ ions in the myocardium cells
• Reduction of Na+ and Ca2+ ions in the extracellular space
• Accumulation of Na+ and Ca2+ ions in the myocardium cells

7. A patient is diagnosed with cardiac infarction. Blood test for


cardiospecific enzymes activity was performed. Which of the
enzymes has three isoforms?
• Aspartate transaminase
• Alanine transaminase
• Creatine kinase
• Pyruvate kinase
• Lactate dehydrogenas

8. A 50-year-old woman diagnosed with cardiac infarction has


been delivered into an intensive care ward. What enzyme will
be the most active during the first two days?

• Alanineaminotransferase
• Alanineaminopeptidase
• Aspartate aminotransferase
• LDH5
• LDH4

9. Untrained people often have muscle pain after sprints as a


result of lactate accumulation. This might be caused by
intensification of the following biochemical process:

• Glycogenesis
• Gluconeogenesis
• Pentose phosphate pathway
• Lipogenesis
• Glycolysis

10. A patient with suspected diagnosis "progressing muscular


dystrophy"got his urine tested.What compound will confirm
this diagnosis if found in urine?

• Myoglobin
• Collagen
• Porphyrin
• Calmodulin
• Kreatine

11. Rheography of an 18 year old student during exercise


showed redistribution of blood flow between organs. The peak
blood flow will be observed in the following vessels:

• Gastrointestinal tract
• Skeletal muscles
• Cerebrum
• Liver
• Kidneys

12. A 46-year-old female patient has continuous history of


progressive muscular (Duchenne’s) dystrophy. Which blood
enzyme changes will be of diagnostic value in this case?

• Lactate dehydrogenase
• Pyruvate dehydrogenase
• Glutamate dehydrogenase
• Creatine phosphokinase
• Adenylate cyclase

13. A 50-year-old patient complains about general weakness,


appetite loss and cardiac arrhythmia. The patient presents with
muscle hypotonia, flaccid paralyses, weakened peristaltic
activity of the bowels. Such condition might be caused by:

• Hyponatremia
• Hypoproteinemia
• Hyperkaliemia
• Hypophosphatemia
• Hypokaliemia
14. Some students developed myodynia after continuous
physical activity during physical education. The reason for
such condition was accumulation of lactic acid in the skeletal
muscles. It was generated in the students’ bodies after
activation of the following process:

• Pentose-phosphate cycle
• Glycogeny
• Glycolysis
• Gluconeogenesis

• Lipolysis

15. After a sprint an untrained person develops muscle


hypoxia. This leads to the accumulation of the following
metabolite in muscles:

• Oxaloacetate
• Acetyl CoA
• Glucose 6-phosphate
• Lactate
• Ketone bodies

16. A 49-year-old driver complains about unbearable


constricting pain behind the breastbone irradiating to the neck.
The pain arose 2 hours ago. Objectively: the patient’s condition
is grave, he is pale, heart tones are decreased. Laboratory
studies revealed high activity of creatine kinase and LDH1.
What disease are these symptoms typical for?

• Acute myocardial infarction


• Stenocardia
• Acute pancreatitis
• Diabetes mellitus
• Cholelithiasis

17. Cytoplasm of the myocytes contains a lot of dissolved


metabolites resulting from glucose oxidation. Name the
metabolite that turns directly into lactate:

• Glycerophosphate
• Oxaloacetate
• Pyruvate
• Glucose-6-phosphate
• Fructose-6-phosphate

18. A patient came to the hospital complaining about quick


fatigability and apparent muscle weakness. Examination
revealed an autoimmune disease that causes disorder of
functional receptor condition in neuromuscular synapses.
What transmitter will be blocked?

• Glycine
• Noradrenalin
• Dopamine
• Serotonin
• Acetylcholine

19. During processing of the atypical cardiomyocytes of the


synaptic node with a biologically active substance, an increase
in their membrane potential was recorded due to increased
permeability for potassium ions. What biologically active
substance affects cardiomyocytes?

• Acetylcholine
• Noradrenaline
• Thyroxine
• Atriopeptide
• Adrenalin

20. 12 hours after an accute attack of retrosternal pain a patient


presented a jump of aspartate aminotransferase activity in
blood serum. What pathology is this deviation typical for?

• Myocardium infarction
• Diabetes mellitus
• Diabetes insipidus
• Collagenosis
• Viral hepatitis

21. A patient presents high activity of LDH 1,2, aspartate


aminotransferase, creatine phosphokinase. In what organ
(organs) is the development of a pathological process the most
probable?

• In connective tissue
• In the heart muscle (initial stage of myocardium infarction)
• In liver and kidneys
• In kidneys and adrenals
• In skeletal muscles (dystrophy, atrophy)

22. A traumatology unit received a patient with crushed


muscular tissue. What biochemical indicator of urine will be
raised in this case?

• Total lipids
• Glucose
• Creatinine
• Uric acid
• Mineral salts
23. A 1 y.o. child with symptoms of muscle affection was
admitted to the hospital. Examination revealed carnitine deficit
inmuscles. Biochemical base of this pathology is disturbed
process of:

• Regulation of Ca2+ rate in mitochondrions


• Transporting of fatty acids to mitochondrions
• Lactic acid utilization
• Actin and myosin synthesis
• Substrate phosphorylation

24. After a sprint an long distantion the skeletal muscle training


person uses glucose for the purpose of obtaining energy ATP
for muscle contraction. Specify basic glucose utilization
process in these conditions:

• Glycogenesis
• Glycogenolysis
• Aerobic glycolysis
• Anaerobic glycolysis
• Gluconeogenesis

25. The gluconeogenesis is activated in the liver after intensive


physical trainings.What substance is utilized in
gluconeogenesis first of all in this case:

• Pyruvate
• Glutamate
• Lactate
• Alanine
• Glucose
26. 6 hours after the myocardial infarction a patient was found
to have elevated level of lactate dehydrogenase in blood. What
is enzyme should be expected in this case?

• LDH3
• LDH4
• LDH5
• LDH2
• LDH1

1. A substance that is synthesized in the central nervous


system and simulates the effects of morphine can be used for
analgesia. Specify the following substance:

• Vasopressin
• beta-endorphin
• Oxytocin
• Calcitonin
• Somatoliberin

2. Cell membrane restpotential changed from-85 to-90mV. It


can becaused by activation of the following cell membrane
channels:

• Potassium and calcium


• Potassium
• Sodium
• Calcium
• Potassium and sodium

3. The speed of the excitation nerve fibers is 120 m / s. Which


of the above factors, above all, provides such speed?
• A big factor of reliability
• Large amplitude potential effects
• Low threshold of depolarization
• Presence of myelin sheath
• Great resting potential

4. The patient is trembling hands, which is associated with


Parkinson's disease. What mediator deficiency in striopalidar
structures leads to such symptoms?

• GABA
• Serotonin
• Substance P
• Norepinephrine
• Dopamine

5. An 84-year-old patient suffers from parkinsonism. One of the


pathogenetic development elements of this disease is
deficiency of a certain mediator in some of the brain
structures. Name this mediator:

• Noradrenaline
• Histamine
• Acetylcholine
• Dopamine
• Adrenaline

6. In the experiment, the permeability of the excitable cell


membrane to potassium ions was magnified. What changes in
the electrical state of the membrane will occur?

• Local response
• There will be no changes
• Hyperpolarization
• Depolarization
• Action potential

7. A 50-year-old man came to a hospital with complaints of


memory disorders, painful sensations along the nerve trunks,
decreased mental ability, circulatory disorders and dyspepsia.
Anamnesis states excessive alcohol consumption. What
vitamin deficiency can result in such symptoms?

• Riboflavin
• Calciferol
• Thiamine
• Retinol
• Niacin

8. Decarboxylation of glutamate induces production of gamma


aminobutyric acid (GABA) neurotransmitter. After breakdown,
GABA is converted into a metabolite of the citric acid cycle,
that is:

• Fumarate
• Citric acid
• Malate
• Oxaloacetate
• Succinate

9. Degenerative changes in posterior and lateral columns of


spinal cord (funicular myelosis) caused by methylmalonic acid
accumulation occur in patients with B12-deficiency anemia.
This results in synthesis disruption of the following substance:

• Serotonin
• Dopamine
• Acetylcholine
• Norepinephrine
• Myelin

10. After pharmacological blockade ion channels of the nerve


fiber membrane resting potential decreased from -90 to -80 mV.
What channels were blocked?

• Magnesium
• Potassium
• Sodium
• Chloric
• Calcium

11. Disruption of nerve fiber myelinogenesis causes


neurological disorders and mental retardation. These
symptoms are typical for hereditary and acquired alterations in
the metabolism of:

• Phosphatidic acid
• Higher fatty acids
• Neutral fats
• Sphingolipids
• Cholesterol

12. A patient complained about dizziness, memory impairment,


periodical convulsions. It was revealed that these changes
were caused by a product of decarboxylation of glutamic acid.
Name this product:

• ATP
• TDP
• UDP
• GABA
• Pyridoxal phosphate
13. A patient presents with dysfunction of the cerebral cortex
accompanied by epileptic seizures. He has been administered
a biogenic amine synthetized from glutamate and responsible
for central inhibition. What substance is it?

• Serotonin C.
• Histamine
• C. Dopamine
• gamma-aminobutyric acid
• Acetylcholine

14. An unconscious patient was taken by ambulance to the


hospital. On objective examination the patient was found to
have no reflexes, periodical convulsions, irregular breathing.
After laboratory examination the patient was diagnosed with
hepatic coma. Disorders of the central nervous system develop
due to the accumulation of the following metabolite:

• Bilirubin
• Glutamine
• Histamine
• Urea
• Ammonia

15. For the treatment of depression of different genesis,


antidepressants are used, which are monoamine oxidase
inhibitors. What substance is a "neurotransmitter of well-
being" and its concentration increases in the brain under the
action of antidepressants?

• Taurine
• Serotonin
• Dopamine
• Glycine
• Norepinephrine

16. Brain cells are very sensitive to energy deficiency, which


can be caused by a high content of ammonia, which stimulates
the outflow of alpha-ketoglutarate from:

• Glycogenolysis
• Pentose phosphate pathway
• Ornithine cycle
• Glycolysis
• Citric acid cycle

17. Depressions and emotional insanities result from the deficit


of noradrenalin, serotonin and other biogenic amines in the
brain. Their concentration in the synapses can be increased by
means of the antidepressants that inhibit the following
enzyme:

• Monoamine oxidase
• D-amino-acid oxidase
• Phenylalanine-4-monooxygenase
• L-amino-acid oxidase
• Diamine oxidase

18. Pharmacological effects of antidepressants are based upon


blocking (inhibiting) the enzyme that acts as a catalyst for the
breakdown of biogenic amines noradrenalin and serotonin in
the mitochondria of cephalic neurons. What enzyme takes part
in this process?

• Lyase
• Decarboxylase
• Transaminase
• Monoamine oxidase
• Peptidase

19. A 9-month-old infant is fed with artificial formulas with


unbalanced vitamin B6 concentration. The infant presents with
pellagral dermatitis, convulsions, anaemia. Convulsion
development might be caused by the disturbed formation of:

• Histamine
• Serotonin
• GABA
• DOPA
• Dopamine

20. Cerebral trauma caused increase of ammonia formation.


What aminoacid takes part in removal of ammonia from
cerebral tissue?

• Valine
• Lisine
• Tryptophan
• Glutamic
• Tyrosine

21. Ammonia is a very toxic substance, especially for nervous


system. What substance takes the most active part in ammonia
detoxication in brain tissues?

• Histidine
• Alanine
• Proline
• Glutamic acid
• Lysine
22. A patient presented to a hospital with complaints about
quick fatigability and significant muscle weakness.
Examination revealed an autoimmune disease that causes
functional disorder of receptors in the neuromuscular
synapses. This will result in the disturbed activity of the
following mediator:

• Dopamine
• Noradrenaline
• Serotonin
• Glycine
• Acetylcholine

23. Glutamate decarboxylation results in formation of inhibitory


transmitter in CNS. Name it:

• Histamine
• Serotonin
• Glutathione
• Asparagine
• GABA

24. In medical practice for alcoholism prevention widely used


teturam, which is inhibitor aldehyde dehydro- nazy The
increase in blood of what tabolita aversion to al to someone?

• Acetaldehyde
• Ethanol
• Methanol
• Malonic aldehyde
• Propionic aldehyde
25. Soldiers who were injured in the midst of the battle may not
be aware of the pain until it is completed. What hormones are
opiate antinociceptive system reduce the sensation of pain?

• Vasopressin
• Endorfines
• Serotonines
• Oxytocine
• Aldosterone

26. In an excitable cell the ion channels were blocked. It hasn’t


changed essentially the value of rest potential, but the cell lost
its ability to generate AP (action potential). What channels were
blocked?

• Natrium
• Potassium
• Calcium
• Natrium and potassium
• Chloric

27. The infant has epileptiform convulsions caused by vitamin


B6 deficiency. This is due to a decrease in the nervous tissue
of the inhibitory mediator, ?-aminobutyric acid. What is the
enzyme's activity reduced in this case?

• Glutamate decarboxylase
• Glutamate dehydrogenase
• Glutamate Synthetase
• Alanine aminotransferase
• Pyridoxal kinase
1.Human red blood cells do not contain mitochondri= What is the main pathway for ATP
production in these cells?
Select one:
a. Anaerobic glycolysis
b. Oxidative phosphorylation
c. Creatine kinase reaction
d. Aerobic glycolysis
e. Cyclase reaction

2.Biochemical analysis of an infant’s erythrocytes revealed evident glutathione peroxidase


deficiency and low concentrati¬on of reduced glutathion~ What pathologi¬cal condition can
develop in this infant?
Select one:
a. Hemolytic anemia
b. Iron-deficiency anemia
c. Megaloblastic anemia
d. Pernicious anemia
e. Sicklemia

3.A patient with respiratory failure has blood pH of 7,35. pCO2 test revealed hypercapni=
Urine pH test revealed an increase in the urine acidity.What form of acid-base imbalance is
the case?
Select one:
a. Compensated respiratory alkalosis
b. Decompensated respiratory alkalosis
c. Compensated respiratory acidosis
d. Decompensated metabolic acidosis
e. Compensated metabolic acidosis

4.An infectious disease unit admitted a patient with signs of jaundice caused by hepatitis
virus. Select an indicator that is specific only for parenchymatous jaundice:
Select one:
a. Increase in ALT and AST rate
b. Urobilinuria
c. Cholaemia
d. Hyperbilirubinemia
e. Bilirubinuria

5.A 62 year old woman complains of frequent pain attacks in the area of her chest and
backbone, rib fractures. Her doctor suspected myeloma (plasmocytoma). What of the
following laboratory characteristics will be of the greatest diagnostic importance?
Select one:
a. Hypoproteinemia
b. Hypoglobulinemia
c. Hyperalbuminemia
d. Proteinuria
e. Paraproteinemia

6.Toxic affection of liver results in dysfunction of protein synthesis. It is usually accompanied


by the following kind of dysproteinemia:
Select one:
a. Paraproteinemia
b. Relative hyperproteinemia
c. Absolute hyperproteinemia
d. Absolute hypoproteinemia
e. Relative hypoproteinemia

7.A 28-year-old patient undergoing treatment in the pulmonological department has been
diagnosed with pulmonary emphysema caused by splitting of alveolar septum by tissular
tripsin. The disease is cased by the congenital deficiency of the following protein:
Select one:
a. α1-proteinase inhibitor
b. Haptoglobin
c. Transferrin
d. Cryoglobulin
e. α2-macroglobulin

8.
Question text
A month after surgical constriction of rabbit’s renal artery the considerable increase of
systematic arterial pressure was observe~ What of the following regulation mechanisms
caused the animal’s pressure change?
Select one:
a. Noradrenaline
b. Serotonin
c. Angiotensin-II
d. Vasopressin
e. Adrenaline

9.A newborn child with pylorostenosis has often repeating vomiting accompanied by apathy,
weakness, hypertonicity, sometimes convulsions. What disorder form of acid-base balance is
it?
Select one:
a. Gaseous acidosis
b. Excretory acidosis
c. Metabolic acidosis
d. Gaseous alkalosis
e. Nongaseous alkalosis

10.A patient underwent a course of treatment for atherosclerosis. Laboratory tests revealed
an increase in the antiatherogenic lipoprotein fraction in the blood plasm= The treatment
efficacy is confirmed by the increase in:
Select one:
a. HDL
b. Chylomicrons
c. IDL
d. LDL
e. VLDL

11.A patient who had been continuously taking drugs blocking the production of angiotensin
II developed bradycardia and arrhythmi= A likely cause of these disorders is:
Select one:
a. Hypokalemia
b. Hypercalcemia
c. Hypernatremia
d. Hyperkalemia
e. Hypocalcemia

12.A patient complains about dyspnea provoked by the physical activity. Clinical examination
revealed anaemia and presence of the paraprotein in the zone of gamma-globulins. To
confirm the myeloma diagnosis it is necessary to determine the following index in the
patient’s urine:
Select one:
a. Ceruloplasmin
b. Bence Jones protein
c. Haemoglobin
d. Antitrypsin
e. Bilirubin

13.Increased HDL levels decrease the risk of atherosclerosis. What is the mechanism of
HDL anti-atherogenic action?
Select one:
a. They activate the conversion of cholesterol to bile acids
b. They supply tissues with cholesterol
c. They promote absorption of cholesterol in the intestine
d. They are involved in the breakdown of cholesterol
e. They remove cholesterol from tissues

14.In human organism significant blood loss leads to decreased blood pressure, tachycardia,
and weakness. Eventually the sensation of thirst appears. What hormone participates in the
development of this sensation?
Select one:
a. Angiotensin 2
b. Dopamine
c. Cortisol
d. Serotonin
e. Adrenalin
15.A patient consulted a doctor with complaints of dyspnea occurring after physical exertion.
Physical examination revealed anemia, paraprotein was detected among gamma globulin.
What value should be determined in the patient’s urine to confirm the diagnosis of myeloma?
Select one:
a. Antitrypsin
b. Bence Jones protein
c. Ceruloplasmin
d. Hemoglodin
e. Bilirubin

16.Increased HDL levels decrease the risk of atherosclerosis. What is the mechanism of
HDL anti-atherogenic action?
Select one:
a. They promote absorption of cholesterol in the intestine
b. They remove cholesterol from tissues
c. They supply tissues with cholesterol
d. They are involved in the breakdown of cholesterol
e. They activate the conversion of cholesterol to bile acids

17.For biochemical diagnostics of myocardial infarction it is necessary to measure activity of


a number of enzymes and their isoenzymes. What enzymatic test is considered to be the
best to prove or disprove the diagnosis of infarction in the early period after the chest pain is
detected?
Select one:
a. LDH2 lactate dehydrogenase isoenzyme
b. LDH1 lactate dehydrogenase isoenzyme
c. Creatine kinase isoenzyme CK-MB
d. Aspartate aminotransferase cytoplasmic isoenzyme
e. Creatine kinase isoenzyme CK-MM

18.A patient suffers from disrupted patency of the airways at the level of small and medium –
sized bronchial tubes. What changes of acid-base balance can occur in the patient?
Select one:
a. Respiratory acidosis
b. Acid-base balance remains unchanged
c. Respiratory alkalosis
d. Metabolic acidosis
e. Metabolic alkalosis

19.An infant has pylorospasm, weakness, hypodynamia, convulsions as a result of frequent


vomiting. What kind of acid-base disbalance is it?
Select one:
a. Metabolic acidosis
b. Exogenous nongaseous acidosis
c. Excretory acidosis
d. Excretory alkalosis
e. Gaseous alkalosis

20.Toxic damage to hepatic cells resulted in discruption of the patients liver function and the
patient developed edemas. What changes of blood plasma are the main cause of edema
development?
Select one:
a. Decrease of globulin content
b. Increase globulin content
c. Decrease of fibrinogen content
d. Increase of albumin content
e. Decrease of albumin content

21.Hemoglobin catabolism results in release of iron which is transported to the bone marrow
by a certain transfer protein and used again for the synthesis of hemoglobin. Specify this
transfer protein:
Select one:
a. Transcobalamin
b. Transferrin (siderophilin)
c. Ceruloplasmin
d. Albumin
e. Haptoglobin

22.A patient with diabetes developed a diabetic coma due to the acid-base imbalanc~
Specify the kind of this imbalance:
Select one:
a. Metabolic acidosis
b. Non-gaseous alkalosis
c. Metabolic alkalosis
d. Respiratory acidosis
e. Gaseous alkalosis

23.A 49-year-old male patient with acute pancreatitis was likely to develop pancreatic
necrosis, while active pancreatic proteases were absorbed into the blood stream and tissue
proteins broke up.What protective factors of the body can inhibit these processes?
Select one:
a. Ceruloplasmin, transferrin
b. Cryoglobulin, interferon
c. Hemoplexin, haptoglobin
d. α2-macroglobulin, α1-antitrypsin
e. Immunoglobulin

24.A 30-year-old man with diabetes mellitus type I was hospitalise~ The patient is comatos~
Laboratory tests revealed hyperglycemia and ketonemi= What metabolic disorder can be
detected in this patient?
Select one:
a. Metabolic acidosis
b. Normal acid-base balance
c. Respiratory alkalosis
d. Respiratory acidosis
e. Metabolic alkalosis

25.A patient has insufficient blood supply to the kidneys, which has caused the development
of pressor effect due to constriction of arterial resistance vessels. This condition results from
the vessels being strongly affected by the following substance:
Select one:
a. Angiotensinogen
b. Catecholamines
c. Renin
d. Norepinephrine
e. Angiotensin II

26.A 28-year-old patient undergoing treatment in a pulmonological department has been


diagnosed with pulmonary emphysema caused by splitting of alveolar septum by tissular
tripsin. The disease is caused by the congenital deficiency of the following protein:
Select one:
a. Haptoglobin
b. α1-proteinase inhibitor
c. α2-macroglobulin
d. Transferrin
e. Cryoglobulin

27.
Question text
A 15-year-old boy has been diagnosed with acute viral hepatitis. What blood value should be
determined to confirm acute affection of hepatic cells?
Select one:
a. Cholesterol content
b. Aminotransferase activity (AST, ALT)
c. Protein fraction content
d. Erythrocytes sedimentation rate (ESR)
e. Unconjugated and conjugated bilirubin content

28.A 12-year-old patient was found to have blood serum cholesterol at the rate of 25 mmol/l.
The boy has a history of hereditary familial hypercholesterolemia, which is caused by the
impaired synthesis of the following protein receptors:
Select one:
a. Low density lipoproteins
b. Very low density lipoproteins
c. High density lipoproteins
d. Intermediate density lipoproteins
e. Chylomicrons
29.A patient has been admitted to the contagious isolation ward with signs of jaundice
caused by hepatitis virus. Which of the symptoms given below is strictly specific for
hepatocellular jaundice?
Select one:
a. Bilirubinuria
b. Cholemia
c. Urobilinuria
d. Increase of ALT, AST level
e. Hyperbilirubinemia

30.Deficiency of linoleic and linolenic acids in the body leads to the skin damage, hair loss,
delayed wound healing, thrombocytopenia, low resistance to infections. These changes are
most likely to be caused by the impaired synthesis of the following substances:
Select one:
a. Catecholamines
b. Interleukins
c. Interferons
d. Eicosanoids
e. Corticosteroids
V ebf68bc9

Krok \ Dentistry \ Krok 1 \ Bases \ Biochemistry \ 2010


1 Roentgenological examination of a patient revealed a cyst enclosing a tooth in its cavity in
the area of the premolar. Microscopical examination revealed that the cyst wall consisted of
connective tissue and was lined with multilayer squamous epithelium. What is the most
probable diagnosis?

A Follicular cyst
B Radicular cyst
C Primordial cyst
D Eosinophilic granuloma
E Epulis

2 A dentist was examining oral cavity of a 9 year old child in the buccal surface of gingiva in
the area of the lower canine he revealed a red, soft, node-like formation 1 cm in diameter
that started immediately bleeding when touched. Microscopical examination revealed that
this formation consisted of many small vessels like venules and capillaries separated by thin
layers of connective tissue, with focal infiltration by lymphoid and plasmatic cells. Such
changes are typical for:

A Angiomatous epulis
B Capillary hemangioma
C Radicular granuloma
D Fibrous epulis
E Papilloma

3 Analysis of urine from a 24-year-old man revealed the following changes: daily diuresis -
10 l, relative density - 1,001, qualitative alterations are absent. A patient complains of
excessive thirst, frequent urination. What is the most likely cause of this disease?

A Vasopressin hyposecretion
B Glucocorticoid hypersecretion
C Vasopressin hypersecretion
D Relative insulin insufficiency
E Aldosteron hypersecretion

4 A patient suffering from syphilis was prescribed a drug the action of which based upon
disturbed generation of murein leading to death of the causative agent. What drug is it?

A Benzylpenicillin sodium salt


B Bijochinol
C Ciprofloxacin
D Azithromycin
E Doxycycline hydrochloride
V ebf68bc9

5 Osteolathyrism is characterized by a loss of tensile strength of collagen, which is induced


by a significant decrease in the formation of cross-links in collagen fibrils. The cause for it is
the reduced activity of:

A Lysyl oxidase
B Monoamine oxidase
C Prolyl hydroxylase
D Lysyl hydroxylase
E Collagenase

6 Periodontitis is accompanied by activation of proteolysis in the periodontium tissues. The


evidence of proteolysis activation is increase of the following component of oral liquid:

A Amino acids
B Organic acids
C Glucose
D Biogenic amines
E Cholesterol

7 Two weeks after hemotransfusion a patient developed fever. What protozoal disease can
be suspected?

A Malaria
B Toxoplasmosis
C Leishmaniasis
D Amebiasis
E Trypanosomiasis

8 A patient applied to a doctor complaining about dizziness, memory impairment,


periodical convulsions. It was found out that such changes were caused by a product of
glutamic acid decarboxylation. What product is meant?

A GABA
B Pyridoxalphosphate
C Thymidine diphosphate
D ATP
E Tetrahydrofolate

9 Laboratory examination of a child revealed high content of leucine, valine, isoleucine and
their ketoderivates in blood and urine. Urine had the typical smell of maple syrup. This
disease was caused by deficiency of the following enzyme:

A Dehydrogenase of branched amino acids


B Aminotransferase
C Glucose-6-phosphatase
D Phosphofructokinase
V ebf68bc9

E phosphofructomutase

10 Hepatic disfunctions accompanied by insufficient inflow of bile to the bowels result in


coagulation failure. This phenomenon can be explained by:

A Vitamin K deficiency
B Iron deficiency
C Thrombocytopenia
D Erythropenia
E Leukopenia

11 A child has disturbed processes of ossification and punctate enamel. What microelement
metabolism is disturbed?

A Fluorine
B Chromium
C Copper
D Iron
E Zinc

12 During examination of a 36-year-old woman a dentist revealed a formation in form of a


nodule up to 0,8 cm in diameter, of dark brown-red colour, soft, on a wide base. The
formation was found on the buccal surface of gum in the region of the 2nd molar.
Histological examination revealed that the formation had plenty of sinusoid vessels and a
lot of roundish mononuclear and big multinuclear cells; in some parts accumulations of
hemosiderin granules could be found. What is the most likely diagnosis?

A Giant-cell epulis
B Root granuloma
C Angiomatous epulis
D Ameloblastoma
E Mandibular osteoclastoma

13 While on holiday in the countryside a boy found a spider with the following
morphological peculiarities: body length at the rate of 2 cm, round black abdomen with two
rows of red dots on its dorsal surface, four pairs of segmented extremities covered with tiny
black hairs. Identify this arthropod:

A Steppe spider (Latrodectus tredeci-mguttatus)


B Scorpion
C Solifugae
D Mite
E Tarantula

14 A girl who was provisionally diagnosed with Turner’s syndrome came to a genetic
consultation. The diagnosis can be specified by means of the following genetic method:
V ebf68bc9

A Sex chromatin test


B Genealogical
C Hybridological
D Biochemical
E Dermatoglyphics

15 Cyanide poisoning causes immediate death. What is the mechanism of cyanide effect at
the molecular level?

A They inhibit cytochromoxidase


B They bind substrates of tricarboxylic acid cycle
C They block succinate dehydrogenase
D They inactivate oxygene
E They inhibit cytochrome B

16 A blood smear of a patient who has recently recovered from flu contains 10% of
roundish cells 4,5-7 micrometer large with a big round nucleus and basophi-lically stained
cytoplasm in form of a narrow border around the nucleus. What blood status are they
typical for?

A Lymphocytopenia
B Thrombopenia
C Leukopenia
D Lymphocytosis
E Monocytopenia

17 A patient was diagnosed with seborrheic dermatitis associated with vitamin H (biotin)
deficiency. The patient has disturbed activity of the following enzyme:

A Acetyl-CoA-carboxylase
B Pyruvate decarboxylase
C Alcohol dehydrogenase
D Amino transferase
E Carbomoyl phosphate synthetase

18 Active physical work induces rise of concentration of carbonic acid in blood. This causes
deepening and acceleration of respiration thus reducing concentration of carbonic acid and
hydrogen ions in blood. This maintains the following process:

A Homeostasis
B Immunity
C Ontogenesis
D Orthobiosis
E Anabiosis
V ebf68bc9

19 The activity of parotides reduces with age. Activity of what enzyme in saliva will be
reducing?

A Amylase
B Lysozime
C Phosphatase
D Hexokinase
E Maltase

20 Chronic overdosage of glucocorticoids leads to the development of hyperglycemia. What


process of carbohydrate metabolism is responsible for this effect?

A Gluconeogenesis
B Glycogenolysis
C Aerobic glycolisis
D Pentose-phosphate cycle
E Glycogenesis

21 A patient has mental retardation, small height, brachydactyly, mongoloid slant. Analysys
of his karyotype revealed trisomy 21. What chromosomal anomaly is it?

A Down’s disease
B Klinefelter’s syndrome
C Turner’s syndrome
D Trisomy X
E Specific fetopathy

22 Examination of urine in a newborn revealed presence of citrulline and high ammonia


concentration. This baby is most likely to have the disorder of the following substance
production:

A Urea
B Uric acid
C Ammonia
D Creatinine
E Creatine

23 In the mountains some clinically healthy people present with anaemia symptoms. Blood
test can reveal sickle cells. What is the genotype of such people?

A Aa
B aa
C AA
D XcXc
E XCXc
V ebf68bc9

24 To what total ATP quantity is the full glucose oxidation and its linking with
phosphorylation equivalent?

A 38
B8
C 12
D 52
E 58

25 While examining a blood smear taken form a patient and stained by Romanovsky’s
method a doctor revealed some protozoa and diagnozed the patient with Chagas disease.
What protozoan is the causative agent of this disease?

A Trypanosoma cruzi
B Toxoplasma gondii
C Leishmania donovani
D Leishmania tropica
E Trypanosoma brucei

26 A 10 year old child underwent Mantoux test (with tuberculin). 48 hours later there
apperaed a papule up to 8 mm in diameter on the site of tuberculin injection. Tuberculin
injection caused the following hypersensitivity reaction:

A IV type hypersensitivity reaction


B Arthus reaction
C Seroreaction
D Atopic reaction
E II type hypersensitivity reaction

27 A chiled was diagnosed with acute renal failure. What biochemic saliva indices can
confirm this diagnosis?

A Increased level of rest nitrogen


B Increase of immunoglobuline A
C Reduction of alkaline phosphatase
D Increase of alpha amylase
E Decreased level of phosphate

28 A 28 year old pregnant woman had the enzymes in the cells of amniotic fluid analyzed.
The analysis revealed insufficient activity of beta-glucuronidase. What pathological process
is it?

A Mucopolysaccharidosis
B Glycogenosis
C Collagenosis
D Lipidosis
V ebf68bc9

E Aglycogenosis

29 A patient with chronic hypoglycemia had adrenaline introduction. After introduction


blood test has not changed essentially. Doctor assumed liver pathology. What liver function
may have been changed?

A Function of glycogen depositing


B Ketogenic function
C Function of cholesterin production
D Glycolytic function
E Excretory function

30 In order to speed up healing of the thermal injury it is required to prescribe a drug that
facilitates epithelization of skin and mucous membranes. What drug is it?

A Retinol acetate
B Tocopherol acetate
C Nicotinic acid
D Ergocalciferol
E Ascorbic acid

31 A child has abnormal formation of tooth enamel and dentin as a result of low
concentration of calcium ions in blood. Such abnormalities might be caused by deficiency of
the following hormone:

A Parathormone
B Thyrocalcitonin
C Thyroxin
D Somatotropic hormone
E Triiodothyronine

32 Microscopical examination of discharges from the gums of a patient ill with paradontosis
revealed some protozoan pear-shaped organisms 6-13 micrometer long. The parasite has
one nucleus and undulating membrane, there are four flagella at the front of its body. What
protozoan were found?

A Trichomonads
B Leishmania
C Amoebae
D Balantidia
E Lamblia

33 A patient has painfulness along big nerve trunks and excessive content of pyruvate in
blood. What vitamin deficit may cause such changes?

ABi
B B2
V ebf68bc9

C PP
D Pantothenic acid
E Biotin

34 A patient with diabetes mellitus had an insuline injection. It caused loss of consciousness
and convulsions. What was the result of biochemic blood analysis on glucose content?

A 2,5 mmole/l
B 3,3 mmole/l
C 8,0 mmole/l
D 10 mmole/l
E 5,5 mmole/l

35 Examination of a 6 days old infant revealed phenyl pyruvate and phenyl acetate excess
in his urine. What ami-noacid metabolism is disturbed in the child’s organism?

A Phenylalanine
B Tryptophan
C Methionine
D Histidine
E Arginine

36 For assessment of the neutralizing function of liver a patient with chronic hepatitis went
through a test with natrium benzoate load. The excretion of what acid with urine will
characterize the neutralizing function of liver?

A Hippuric acid
B Phenylacetic acid
C Citric acid
D Valeric acid
E Oxalic acid

37 A 42 year old woman diagnosed with diabetes mellitus was admitted the
endocrinological department with complaints of thirst, excessive appetite. What
pathological components are revealed in course of laboratory examination of the patients
urine?

A Glucose, ketone bodies


B Protein, aminoacids
C Bilirubin, urobilin
D Blood
E Protein, creatine

38 A 2 year old child suffers from intestinal dysbacteriosis that lead to the development
hemorrhagic syndrome. The most probable cause of hemorrhage is:

A Vitamin K deficiency
V ebf68bc9

B Activation of tissue thromboplastin


C Hypovitaminosis PP
D Fibrinogen deficiency
E Hypocalcemia

39 Decreased ratio of adenylic nucleotides ATP/ADP results in intensified glycolysis in


parodentium tissues in hypoxic conditions. What reaction is activated in this case?

A Phosphofructokinase
B Lactate dehydrogenase
C Triosophosphate isomerase
D Aldolase
E Enolase

40 Helminthological examination of patient’s feces revealed oval brown eggs with tuberous
external membrane. Name the type of helminth:

A Ascarid
B Pinworm
C Whipworm
D Dwarf tapeworm
E Broad tapeworm

41 A patient has the folowing changes: disorder of twilight vision, drying out of conjunctiva
and cornea. Such disorders may be caused by deficiency of vitamin:

A Vitamin A
B Vitamin B
C Vitamin C
D Vitamin D
E Vitamin B12

42 Examination of a 10 y.o. child revealed on the alveolar submandibular process a fixed


tumourous mass 1,5 cm in diameter closing premolar crown on the vestibular side. Mucous
membrane of its surface is reddish-brown, it bleeds as a reaction to a slight mechanical
intervention. Biopsy results: the mass consists of small size vessels separated by thin layers
of connective tissue and infiltrated by plasmocytes, mucous membrane is here and there
ulcered. What is the most probable diagnosis?

A Angiomatous form of epulis


B Gingival fibromatosis
C Giant cell form of epulis
D Hypertrophic gingivitis
E Fibrous form of epulis
V ebf68bc9

43 A non trained man has usually muscular hypoxy after a sprint. What metabolite
accumulates in the muscles as a result of it?

A Lactate
B Ketone bodies
C Glucose 6-phosphate
D Oxaloacetate
E-

44 A man got poisoned with mushrooms. They contain muscarine that stimulates
muscarinic cholinoreceptors. What symptom is typical for poisoning with inedible
mushrooms?

A Miosis
B Mydriasis
C Bronchi dilation
D Heart rate rise
E Arterial pressure rise

45 Myocyte cytoplasm contains a big number of dissolved metabolites of glucose oxidation.


Name one of them that turns directly into lactate:

A Pyruvate
B Oxaloacetate
C Glycerophosphate
D Glucose 6-phosphate
E Fructose 6-phosphate

46 Roentgenological examination of mandible of a 27 year old man revealed a focus of


osseous tissue destruction. Histological examination revealed a tumour consisting of
odontogenous epithelium cords, immature connective tissue and dysplastic dentin rests.
What tumour is it?

A Dentinoma
B Ameloblastic fibro-odontoma
C Odontoameloblastoma
D Odontogenous fibroma
E Complex odontoma

47 Patient with pigmentary xeroderma are characterized by anamalously high sensivity to


ultraviolet rays that causes skin cancer as a result of enzyme systems incapability to restore
damages of hereditary apparatus of cells. What process abnormality is this pathology
connected with?

A DNA reparation
B Genetic conversion
V ebf68bc9

C DNA recombination
D Genetic complementation
E DNA reduplication

48 A patient has the sudden decrease of Са2+ content in blood. What hormone secretion
will increase?

A Parathormone
B Somatotropin
C Aldosterone
D Thyrocalcitonin
E Vasopressin

49 A female patient has symptoms of inflammation of urogenital tracts. A smear from the
vaginal mucous membrane contained big unicellular pyriform organisms with a sharp spike
on the back end of their bodies; big nucleus and undulating membrane. What protozoa were
revealed in the smear?

A Trichomonas vaginalis
B Trichomonas hominis
C Trichomonas buccalis
D Trypanosoma gambiense
E Lamblia intestinalis

50 A patient has increased content of uric acid in his blood that is clinically presented by
pain syndrome as a result of urate deposition in the joints. What process does this acid
result from?

A Lysis of purine nucleotides


B Lysis of pyrimidine nucleotides
C Heme catabolism
D Proteolysis
E Reutilization of purine bases

51 Among public catering workers examined by doctors of sanitary-and-epidemiologic


station often occur asymptomatic parasite carriers. This means that a healthy person
carries cysts that infect other people. Such parasitizing is impossible for the following
causative agent:

A Dysenteric amoeba
B Malarial plasmodium
C Intestinal trichomonad
D Dermatotropic leishmania
E Viscerotropic leishmania
V ebf68bc9

52 The 16th tooth of a patient is missing. X-ray picture shows in the depth of alveolar
process rarefaction of bone and a well-defined cavity that contained the underdeveloped
tooth crown. Microscopical examination revealed that the cavity wall was lined with
stratified squamous epithelium and enclosed within a fibrous capsule. Make a diagnosis:

A Follicular gnathic cyst


B Radicular gnathic cyst
C Cyst of the incisive canal
D Cystic ameloblastoma of jaw
E Primordial gnathic cyst

53 A patient has increased permeability of blood-vessel walls, increased gingival


hemorrhage, small punctate hematomas on his skin, falling of teeth. What disturbance of
vitamun metabolism can account for these symptoms?

A Hypovitaminosis C
B Hypervitaminosis D
C Hypervitaminosis C
D Hypovitaminosis D
E Hypovitaminosis A

54 A patient has liver cirrhosis. Analysis of what substance excreted with urine may
characterize the state of antitoxic liver function?

A Hippuric acid
B Ammonian salts
C Creatinine
D Uric acid
E Amino acids

55 A sportsman needs to improve his sporting results. He was recommended a drug


containing carnitine. What process is activated by this compound in the first place?

A Transport of fatty acids


B Transport of amino acids
C Transport of calcium ions
D Transport of glucose
E Transport of vitamin K

56 A child has disturbed enamel and dentine formation as a result of decreased content of
calcium ions in his blood. What hormone deficiency may cause such changes?

A Thyreocalcitonin
B Somatotropin
C Thyroxin
D Parathormone
V ebf68bc9

E Triiodothyronine

57 A hospital admitted a patient with complaints about abdominal swelling, diarrhea,


meteorism after consumption of food rich in proteins. It is indicative of disturbed protein
digestion and their intensified decaying. What substance is the product of this process in the
bowels?

A Indole
B Bilirubin
C Cadaverine
D Agmatine
E Putrescine

58 A 38 year old patient takes aspirin and sulfanilamides. After their intake intensified
erythrocyte haemolysis is observed which is caused by deficiency of glucose 6-phosphate
dehydrogenase. This pathology is caused by failure of the following coenzyme:

A NADP - H
B FAD - H2
C Pyridoxal phosphate
D FMN - H2
E Ubiquinone

59 Up to 50% of world population aged above thirty is affected by paradontosis. The


leading part in pathogenesis of this disease is played by:

A Neurodystrophic factor
B Parodontium tissues damaged by kalli-krein
C Parodontium damaged by active cells
D Dental calculus caused by microflora
E Immune damage of tissues

60 A 57 year old patient with diabetes mellitus was developed ketoacedosis. Biochemical
base of this condition is decrease of acetyl-CoA utilization. What cell compound deficit
causes this effect?

A Oxaloacetate
B Glutamate
C 2-oxoglutarate
D Aspartate
E Succinate

61 A patient was taken to the hospital with preliminary diagnosis progressive muscle
distrophy. What substance will be excessively contained in urine and confirm this
diagnosis?

A Creatine
V ebf68bc9

B Pyruvate
C Carnosine
D Troponine
E Hydroxiproline

62 Microscopical examination of a surgical specimen (an ulcered part of a lip) revealed in


the connective tissue of mucous membrane near the borders and under the floor of the
ulcer some epithelial complexes consisting of atypic multi-stratal epithelium with
accumulations of bright pink concentric formations. What pathology is it?

A Squamous cell keratinous carcinoma


B Squamous cell nonkeratinous carcinoma
C Transitional cell carcinoma
D Basal cell carcinoma
E Papilloma

63 Examination of a child who hasn’t got fresh fruit and vegetables during winter revealed
numerous subcutaneous hemorrhages, gingivitis, carious cavities in teeth. What vitamin
combination should be prescribed in this case?

A Ascorbic acid and rutin


B Thiamine and pyridoxine
C Folic acid and cobalamin
D Riboflavin and nicotinamide
E Calciferol and ascorbic acid

64 Coprological examination of a patient’s feces revealed small operculate eggs. It is known


from the anamnesis that the patient often consumes fish. What fluke parasitizes in the
patient’s organism?

A Cat liver fluke


B Blood fluke
C Lung fluke
D Liver fluke
E Lancet fluke

65 While the examination of patient’s oral cavity the dentist found xerostomia, numerous
erosions. What vitamin deficit caused this effect?

A Vitamin A
B Vitamin K
C Vitamin P
D Vitamin H
E Vitamin PP
V ebf68bc9

66 A sportsman was recommended to take a preparation with carnitine in order to improve


his achievements. What process is activated by carnitine to the most extent?

A Transporting of fatty acids to the mi-tochondrions


B Synthesis of steroid hormones
C Synthesis of ketone bodies
D Lypide synthesis
E Tissue respiration

67 After implantation of a cardiac valve a young man constantly takes indirect


anticoagulants. His state was complicated by hemorrhage. What substance content has
decreased in blood?

A Prothrombin
B Haptoglobin
C Heparin
D Creatin
E Ceruloplasmin

68 A 28 year old woman consulted a doctor about sterility. Examination revealed


underdeveloped ovaries and uterus, irregular menstrual cycle. Study of sex chromatin
revealed 2 Barr’s bodies in most somatic cells. What chromosome disease is the most
probable in this case?

A Triplo-X syndrome
B Edwards’ syndrome
C Patau’s syndrome
D Klinefelter’s syndrome
E Turner’s syndrome

69 A patient who is ill with scurvy displays disturbed processes of connective tissue
formation that leads to loosening and falling of teeth. Disturbed activity of what enzyme
causes these symptomps?

A Lisilhydroxylase
B Glycosiltransferase
C Elastase
D Procollagenpeptidase of N-terminal peptide
E Procollagenpeptidase of C-terminal peptide

70 During physical exercise people are less sensitive to pain. The reason for it is the
activation of:

A Antinociceptive system
B Nociceptive system
C Thyroid gland functions
V ebf68bc9

D Sympathoadrenal system
E Adrenal gland functions

71 A one year old child has enlarged head and belly, retarded cutting of teeth, destruction of
enamel structure. What hypovitaminosis causes these changes?

A Hypovitaminosis D
B Hypovitaminosis C
C Hypovitaminosis A
D Hypovitaminosis B 1
E Hypovitaminosis B2

72 A sample taken from the pharynx of a patient with angina was inoculated on the blood-
tellurite agar. This resulted in growth of grey, radially striated (in form of rosettes) colonies
up to 4-5 mm in diameter. Microscopically there can be seen gram-positive rods with club-
shaped ends arranged in form of spread fingers. What microorganisms are these?

A Corynebacteria diphtheriae
B Clostridium botulinum
C Diphtheroids
D Streptococci
E Streptobacilli

73 A patient with high obesity was recommended to take carnitine as a food additive for
better fat burning. What function is fulfilled by carnitine in the process of fat oxidation?

A Transport of fatty acids from the cytosol to the mitochondria


B Transport of fatty acids from the fat depots to the tissues
C Participation in one of the reactions of beta-oxidation of fatty acids
D Fatty acid activation
E Intracellular lipolysis activation

74 Laboratory of extremely dangerous infections received a sample taken from a patient


with assumed cholera. What express-diagnostics method can confirm this diagnosis?

A Immunofluorescence test
B Complement binding reaction
C Agglutination test
D Precipitation reaction
E Hemagglutination reaction

75 A patient with focal tuberculosis of superior lobe of his right lung takes isoni-azid as a
part of combined therapy. After a time he started complaining of muscular weakness,
decrease of skin sensitivity, sight and movement coordination disorder. What vitamin
preparation will be right for elimination of these occurances?

A Vitamin B6
V ebf68bc9

B Vitamin A
C Vitamin D
D Vitamin B12
E Vitamin C

76 The first grade pupils went through a medical examination aimed at selection of children
needing tuberculosis revaccination. What test was applied?

A Mantoux test
B Schick test
C Supracutaneous tularin test
D Burne test
E Anthracene test

77 A 15 year old girl was delivered to the hospital with inflammation of vermiform
appendix. Blood analysis revealed signs of anaemia. Her feces contained lemon-shaped
helminthic eggs (50x30 micrometer) with "plugs" on the poles. What type of helminth is it?

A Trichuris
B Hookworm
C Hymenolepis nana
D Echinococcus
E Pinworm

78 A patient has roundish ulcers on his face, inflammation and enlargement of lymph nodes.
These symptoms turned up as a result of mosquito bites. Laboratory examination of
discharge from the ulcers revealed unicellular aflagellar organisms. What is the most
probable diagnosis?

A Dermatotropic leishmaniasis
B Toxoplasmosis
C Scabies
D Trypanosomiasis
E Myasis

79 A patient has deformation of jaw bones. Histological examination revealed there growth
of fibrocellular tumourlike ill-defined tissue with primitive osteogenesis. What disease are
these presentations typical for?

A Fibrous dysplasia
B Ameloblastoma
C Osteosarcoma
D Eosinophilic granuloma
E Parathyroid osteodystrophy
V ebf68bc9

80 Cytogenetic examination of a patient with dysfunction of the reproductive system


revealed normal karyotype 46,XY in some cells, but most cells have Klinefelter’s syndrome
karyotype - 47,XXY Such phenomenon of cell inhomogeneity is called:

A Mosaicism
B Inversion
C Transposition
D Duplication
E Heterogeneity

81 Examination of an ill child’s blood revealed inherited hyperlipoproteinemia. Genetic


defect of what enzyme synthesis causes this phenomenon?

A Lipoprotein lipase
B Glycosidase
C Proteinase
D Hemsynthetase
E Phenylalanine hydroxylase

82 Dehelmintization of a patient revealed some long fragments of a helminth with


segmented structure. Mature segments were rectangular, 30x12 mm large, closed-type
matrix was in form of a stem with 17-35 lateral branches. Specify this helminth:

A Hookless tapeworm
B Alveococcus
C Echinococcus
D Dwarf tapeworm
E Armed tapeworm

83 Examination of a 30-year-old man mandible revealed in the region of his molar a dense
tumour-like formation that significantly deformed the mandible. The formation wasnt fully
detached from the bone tissue. Microscopical examination of a tissue sampling revealed that
stroma had some cords and follicles with odontogenous cylindric epithelial cells in
peripheria and stellate cells resembling of the enamel organ pulp in the centre. What is the
most likely diagnosis?

A Ameloblastoma
B Primary intraosteal cancer
C Adenomatoid tumour
D Adenocarcinoma
E Osteoclastoma

84 Histological examination of a micro specimen presenting a malignant lung tumour


revealed that the tumor consisted of lymphocyte-like cells forming any structures. Stroma is
mildely marked, there are a lot of mitoses and necroses. What tumour is it?
V ebf68bc9

A Small cell carcinoma


B Squamous cell nonkeratinous carcinoma
C Fibroma
D Squamous cell keratinous carcinoma
E Adenocarcinoma

85 A 40-year-old male patient had a tumour-like formation 8x7 cm large on his neck. A
surgeon removed it only partially because of close connection with large vessels.
Microscopical examination revealed marked cellular and tissue atypism, lipoblast-type cells
in different stages of maturity, with polymorphism and nuclear hyperchromia, pathological
mitoses, necrosis foci. Specify the histological form of the tumour:

A Liposarcoma
B Lipoma
C Fibroma
D Fibrosarcoma
E Hibernoma

86 In compliance with the clinical presentations a man was prescribed pyri-


doxalphosphate. What processes are corrected by this preparation?

A Transamination and decarboxylation of amino acids


B Oxidative decarboxilation of keto acids
C Desamination of purine nucleotides
D Synthesis of purine and pyrimidine bases
E Protein synthesis
V 02b91030

Krok \ Dentistry \ Krok 1 \ Bases \ Biochemistry \ 2010


1 Active physical work induces rise of concentration of carbonic acid in blood. This causes
deepening and acceleration of respiration thus reducing concentration of carbonic acid and
hydrogen ions in blood. This maintains the following process:

A Orthobiosis
B Homeostasis
C Ontogenesis
D Anabiosis
E Immunity

2 The first grade pupils went through a medical examination aimed at selection of children
needing tuberculosis revaccination. What test was applied?

A Anthracene test
B Supracutaneous tularin test
C Mantoux test
D Schick test
E Burne test

3 Cyanide poisoning causes immediate death. What is the mechanism of cyanide effect at
the molecular level?

A They inhibit cytochrome B


B They bind substrates of tricarboxylic acid cycle
C They block succinate dehydrogenase
D They inhibit cytochromoxidase
E They inactivate oxygene

4 Periodontitis is accompanied by activation of proteolysis in the periodontium tissues. The


evidence of proteolysis activation is increase of the following component of oral liquid:

A Glucose
B Organic acids
C Amino acids
D Biogenic amines
E Cholesterol

5 A 40-year-old male patient had a tumour-like formation 8x7 cm large on his neck. A
surgeon removed it only partially because of close connection with large vessels.
Microscopical examination revealed marked cellular and tissue atypism, lipoblast-type cells
in different stages of maturity, with polymorphism and nuclear hyperchromia, pathological
mitoses, necrosis foci. Specify the histological form of the tumour:

A Fibroma
V 02b91030

B Liposarcoma
C Fibrosarcoma
D Hibernoma
E Lipoma

6 A child has disturbed processes of ossification and punctate enamel. What microelement
metabolism is disturbed?

A Chromium
B Zinc
C Fluorine
D Iron
E Copper

7 Two weeks after hemotransfusion a patient developed fever. What protozoal disease can
be suspected?

A Leishmaniasis
B Trypanosomiasis
C Amebiasis
D Malaria
E Toxoplasmosis

8 In compliance with the clinical presentations a man was prescribed pyri-doxalphosphate.


What processes are corrected by this preparation?

A Transamination and decarboxylation of amino acids


B Desamination of purine nucleotides
C Oxidative decarboxilation of keto acids
D Protein synthesis
E Synthesis of purine and pyrimidine bases

9 The activity of parotides reduces with age. Activity of what enzyme in saliva will be
reducing?

A Lysozime
B Maltase
C Phosphatase
D Amylase
E Hexokinase

10 Laboratory examination of a child revealed high content of leucine, valine, isoleucine and
their ketoderivates in blood and urine. Urine had the typical smell of maple syrup. This
disease was caused by deficiency of the following enzyme:

A Glucose-6-phosphatase
B Phosphofructokinase
V 02b91030

C Dehydrogenase of branched amino acids


D phosphofructomutase
E Aminotransferase

11 In the mountains some clinically healthy people present with anaemia symptoms. Blood
test can reveal sickle cells. What is the genotype of such people?

A AA
B aa
C XcXc
D XCXc
E Aa

12 Examination of a 10 y.o. child revealed on the alveolar submandibular process a fixed


tumourous mass 1,5 cm in diameter closing premolar crown on the vestibular side. Mucous
membrane of its surface is reddish-brown, it bleeds as a reaction to a slight mechanical
intervention. Biopsy results: the mass consists of small size vessels separated by thin layers
of connective tissue and infiltrated by plasmocytes, mucous membrane is here and there
ulcered. What is the most probable diagnosis?

A Gingival fibromatosis
B Angiomatous form of epulis
C Giant cell form of epulis
D Fibrous form of epulis
E Hypertrophic gingivitis

13 Examination of a 6 days old infant revealed phenyl pyruvate and phenyl acetate excess
in his urine. What ami-noacid metabolism is disturbed in the child’s organism?

A Phenylalanine
B Histidine
C Methionine
D Tryptophan
E Arginine

14 Laboratory of extremely dangerous infections received a sample taken from a patient


with assumed cholera. What express-diagnostics method can confirm this diagnosis?

A Hemagglutination reaction
B Agglutination test
C Precipitation reaction
D Complement binding reaction
E Immunofluorescence test
V 02b91030

15 A man got poisoned with mushrooms. They contain muscarine that stimulates
muscarinic cholinoreceptors. What symptom is typical for poisoning with inedible
mushrooms?

A Heart rate rise


B Arterial pressure rise
C Bronchi dilation
D Mydriasis
E Miosis

16 Helminthological examination of patient’s feces revealed oval brown eggs with tuberous
external membrane. Name the type of helminth:

A Whipworm
B Broad tapeworm
C Dwarf tapeworm
D Ascarid
E Pinworm

17 A girl who was provisionally diagnosed with Turner’s syndrome came to a genetic
consultation. The diagnosis can be specified by means of the following genetic method:

A Genealogical
B Sex chromatin test
C Dermatoglyphics
D Hybridological
E Biochemical

18 While examining a blood smear taken form a patient and stained by Romanovsky’s
method a doctor revealed some protozoa and diagnozed the patient with Chagas disease.
What protozoan is the causative agent of this disease?

A Trypanosoma brucei
B Toxoplasma gondii
C Leishmania tropica
D Trypanosoma cruzi
E Leishmania donovani

19 A 28 year old pregnant woman had the enzymes in the cells of amniotic fluid analyzed.
The analysis revealed insufficient activity of beta-glucuronidase. What pathological process
is it?

A Lipidosis
B Collagenosis
C Mucopolysaccharidosis
D Aglycogenosis
V 02b91030

E Glycogenosis

20 A patient has painfulness along big nerve trunks and excessive content of pyruvate in
blood. What vitamin deficit may cause such changes?

ABi
B Pantothenic acid
C PP
D Biotin
E B2

21 A 15 year old girl was delivered to the hospital with inflammation of vermiform
appendix. Blood analysis revealed signs of anaemia. Her feces contained lemon-shaped
helminthic eggs (50x30 micrometer) with "plugs" on the poles. What type of helminth is it?

A Hymenolepis nana
B Trichuris
C Echinococcus
D Hookworm
E Pinworm

22 A sample taken from the pharynx of a patient with angina was inoculated on the blood-
tellurite agar. This resulted in growth of grey, radially striated (in form of rosettes) colonies
up to 4-5 mm in diameter. Microscopically there can be seen gram-positive rods with club-
shaped ends arranged in form of spread fingers. What microorganisms are these?

A Diphtheroids
B Corynebacteria diphtheriae
C Streptococci
D Streptobacilli
E Clostridium botulinum

23 A patient was diagnosed with seborrheic dermatitis associated with vitamin H (biotin)
deficiency. The patient has disturbed activity of the following enzyme:

A Alcohol dehydrogenase
B Carbomoyl phosphate synthetase
C Pyruvate decarboxylase
D Amino transferase
E Acetyl-CoA-carboxylase

24 Roentgenological examination of mandible of a 27 year old man revealed a focus of


osseous tissue destruction. Histological examination revealed a tumour consisting of
odontogenous epithelium cords, immature connective tissue and dysplastic dentin rests.
What tumour is it?

A Odontogenous fibroma
V 02b91030

B Ameloblastic fibro-odontoma
C Complex odontoma
D Dentinoma
E Odontoameloblastoma

25 A patient suffering from syphilis was prescribed a drug the action of which based upon
disturbed generation of murein leading to death of the causative agent. What drug is it?

A Benzylpenicillin sodium salt


B Doxycycline hydrochloride
C Azithromycin
D Ciprofloxacin
E Bijochinol

26 A hospital admitted a patient with complaints about abdominal swelling, diarrhea,


meteorism after consumption of food rich in proteins. It is indicative of disturbed protein
digestion and their intensified decaying. What substance is the product of this process in the
bowels?

A Bilirubin
B Putrescine
C Indole
D Agmatine
E Cadaverine

27 A patient has roundish ulcers on his face, inflammation and enlargement of lymph nodes.
These symptoms turned up as a result of mosquito bites. Laboratory examination of
discharge from the ulcers revealed unicellular aflagellar organisms. What is the most
probable diagnosis?

A Toxoplasmosis
B Trypanosomiasis
C Scabies
D Myasis
E Dermatotropic leishmaniasis

28 A patient has increased permeability of blood-vessel walls, increased gingival


hemorrhage, small punctate hematomas on his skin, falling of teeth. What disturbance of
vitamun metabolism can account for these symptoms?

A Hypovitaminosis A
B Hypovitaminosis D
C Hypervitaminosis C
D Hypovitaminosis C
E Hypervitaminosis D
V 02b91030

29 Hepatic disfunctions accompanied by insufficient inflow of bile to the bowels result in


coagulation failure. This phenomenon can be explained by:

A Vitamin K deficiency
B Thrombocytopenia
C Leukopenia
D Iron deficiency
E Erythropenia

30 Analysis of urine from a 24-year-old man revealed the following changes: daily diuresis -
10 l, relative density - 1,001, qualitative alterations are absent. A patient complains of
excessive thirst, frequent urination. What is the most likely cause of this disease?

A Vasopressin hyposecretion
B Vasopressin hypersecretion
C Relative insulin insufficiency
D Aldosteron hypersecretion
E Glucocorticoid hypersecretion

31 Dehelmintization of a patient revealed some long fragments of a helminth with


segmented structure. Mature segments were rectangular, 30x12 mm large, closed-type
matrix was in form of a stem with 17-35 lateral branches. Specify this helminth:

A Dwarf tapeworm
B Hookless tapeworm
C Armed tapeworm
D Alveococcus
E Echinococcus

32 Coprological examination of a patient’s feces revealed small operculate eggs. It is known


from the anamnesis that the patient often consumes fish. What fluke parasitizes in the
patient’s organism?

A Liver fluke
B Lancet fluke
C Blood fluke
D Cat liver fluke
E Lung fluke

33 Examination of a child who hasn’t got fresh fruit and vegetables during winter revealed
numerous subcutaneous hemorrhages, gingivitis, carious cavities in teeth. What vitamin
combination should be prescribed in this case?

A Thiamine and pyridoxine


B Calciferol and ascorbic acid
C Folic acid and cobalamin
V 02b91030

D Ascorbic acid and rutin


E Riboflavin and nicotinamide

34 A patient with high obesity was recommended to take carnitine as a food additive for
better fat burning. What function is fulfilled by carnitine in the process of fat oxidation?

A Transport of fatty acids from the fat depots to the tissues


B Intracellular lipolysis activation
C Transport of fatty acids from the cytosol to the mitochondria
D Participation in one of the reactions of beta-oxidation of fatty acids
E Fatty acid activation

35 A one year old child has enlarged head and belly, retarded cutting of teeth, destruction of
enamel structure. What hypovitaminosis causes these changes?

A Hypovitaminosis C
B Hypovitaminosis A
C Hypovitaminosis B 1
D Hypovitaminosis D
E Hypovitaminosis B2

36 A non trained man has usually muscular hypoxy after a sprint. What metabolite
accumulates in the muscles as a result of it?

A Glucose 6-phosphate
B Ketone bodies
C Lactate
D Oxaloacetate
E-

37 During physical exercise people are less sensitive to pain. The reason for it is the
activation of:

A Nociceptive system
B Antinociceptive system
C Adrenal gland functions
D Sympathoadrenal system
E Thyroid gland functions

38 A patient applied to a doctor complaining about dizziness, memory impairment,


periodical convulsions. It was found out that such changes were caused by a product of
glutamic acid decarboxylation. What product is meant?

A ATP
B GABA
C Thymidine diphosphate
D Tetrahydrofolate
V 02b91030

E Pyridoxalphosphate

39 A patient has liver cirrhosis. Analysis of what substance excreted with urine may
characterize the state of antitoxic liver function?

A Uric acid
B Creatinine
C Hippuric acid
D Amino acids
E Ammonian salts

40 A child has abnormal formation of tooth enamel and dentin as a result of low
concentration of calcium ions in blood. Such abnormalities might be caused by deficiency of
the following hormone:

A Thyroxin
B Triiodothyronine
C Somatotropic hormone
D Thyrocalcitonin
E Parathormone

41 While the examination of patient’s oral cavity the dentist found xerostomia, numerous
erosions. What vitamin deficit caused this effect?

A Vitamin H
B Vitamin P
C Vitamin PP
D Vitamin K
E Vitamin A

42 A 42 year old woman diagnosed with diabetes mellitus was admitted the
endocrinological department with complaints of thirst, excessive appetite. What
pathological components are revealed in course of laboratory examination of the patients
urine?

A Blood
B Glucose, ketone bodies
C Protein, aminoacids
D Bilirubin, urobilin
E Protein, creatine

43 Microscopical examination of a surgical specimen (an ulcered part of a lip) revealed in


the connective tissue of mucous membrane near the borders and under the floor of the
ulcer some epithelial complexes consisting of atypic multi-stratal epithelium with
accumulations of bright pink concentric formations. What pathology is it?

A Squamous cell keratinous carcinoma


V 02b91030

B Basal cell carcinoma


C Papilloma
D Transitional cell carcinoma
E Squamous cell nonkeratinous carcinoma

44 A patient has the folowing changes: disorder of twilight vision, drying out of conjunctiva
and cornea. Such disorders may be caused by deficiency of vitamin:

A Vitamin C
B Vitamin D
C Vitamin B12
D Vitamin B
E Vitamin A

45 A chiled was diagnosed with acute renal failure. What biochemic saliva indices can
confirm this diagnosis?

A Decreased level of phosphate


B Increased level of rest nitrogen
C Reduction of alkaline phosphatase
D Increase of alpha amylase
E Increase of immunoglobuline A

46 For assessment of the neutralizing function of liver a patient with chronic hepatitis went
through a test with natrium benzoate load. The excretion of what acid with urine will
characterize the neutralizing function of liver?

A Valeric acid
B Hippuric acid
C Oxalic acid
D Citric acid
E Phenylacetic acid

47 Microscopical examination of discharges from the gums of a patient ill with paradontosis
revealed some protozoan pear-shaped organisms 6-13 micrometer long. The parasite has
one nucleus and undulating membrane, there are four flagella at the front of its body. What
protozoan were found?

A Amoebae
B Trichomonads
C Balantidia
D Leishmania
E Lamblia

48 A female patient has symptoms of inflammation of urogenital tracts. A smear from the
vaginal mucous membrane contained big unicellular pyriform organisms with a sharp spike
V 02b91030

on the back end of their bodies; big nucleus and undulating membrane. What protozoa were
revealed in the smear?

A Trichomonas buccalis
B Trypanosoma gambiense
C Lamblia intestinalis
D Trichomonas vaginalis
E Trichomonas hominis

49 A 57 year old patient with diabetes mellitus was developed ketoacedosis. Biochemical
base of this condition is decrease of acetyl-CoA utilization. What cell compound deficit
causes this effect?

A 2-oxoglutarate
B Succinate
C Aspartate
D Glutamate
E Oxaloacetate

50 A sportsman needs to improve his sporting results. He was recommended a drug


containing carnitine. What process is activated by this compound in the first place?

A Transport of vitamin K
B Transport of calcium ions
C Transport of amino acids
D Transport of glucose
E Transport of fatty acids

51 Patient with pigmentary xeroderma are characterized by anamalously high sensivity to


ultraviolet rays that causes skin cancer as a result of enzyme systems incapability to restore
damages of hereditary apparatus of cells. What process abnormality is this pathology
connected with?

A DNA reparation
B DNA reduplication
C Genetic complementation
D DNA recombination
E Genetic conversion

52 Among public catering workers examined by doctors of sanitary-and-epidemiologic


station often occur asymptomatic parasite carriers. This means that a healthy person
carries cysts that infect other people. Such parasitizing is impossible for the following
causative agent:

A Malarial plasmodium
B Viscerotropic leishmania
V 02b91030

C Dermatotropic leishmania
D Intestinal trichomonad
E Dysenteric amoeba

53 A sportsman was recommended to take a preparation with carnitine in order to improve


his achievements. What process is activated by carnitine to the most extent?

A Transporting of fatty acids to the mi-tochondrions


B Synthesis of steroid hormones
C Tissue respiration
D Lypide synthesis
E Synthesis of ketone bodies

54 The 16th tooth of a patient is missing. X-ray picture shows in the depth of alveolar
process rarefaction of bone and a well-defined cavity that contained the underdeveloped
tooth crown. Microscopical examination revealed that the cavity wall was lined with
stratified squamous epithelium and enclosed within a fibrous capsule. Make a diagnosis:

A Cystic ameloblastoma of jaw


B Follicular gnathic cyst
C Primordial gnathic cyst
D Radicular gnathic cyst
E Cyst of the incisive canal

55 A 38 year old patient takes aspirin and sulfanilamides. After their intake intensified
erythrocyte haemolysis is observed which is caused by deficiency of glucose 6-phosphate
dehydrogenase. This pathology is caused by failure of the following coenzyme:

A NADP - H
B Ubiquinone
C FAD - H2
D Pyridoxal phosphate
E FMN - H2

56 A patient has the sudden decrease of Са2+ content in blood. What hormone secretion
will increase?

A Somatotropin
B Vasopressin
C Parathormone
D Aldosterone
E Thyrocalcitonin

57 Examination of urine in a newborn revealed presence of citrulline and high ammonia


concentration. This baby is most likely to have the disorder of the following substance
production:
V 02b91030

A Creatinine
B Uric acid
C Creatine
D Ammonia
E Urea

58 Histological examination of a micro specimen presenting a malignant lung tumour


revealed that the tumor consisted of lymphocyte-like cells forming any structures. Stroma is
mildely marked, there are a lot of mitoses and necroses. What tumour is it?

A Squamous cell keratinous carcinoma


B Adenocarcinoma
C Small cell carcinoma
D Squamous cell nonkeratinous carcinoma
E Fibroma

59 A patient has mental retardation, small height, brachydactyly, mongoloid slant. Analysys
of his karyotype revealed trisomy 21. What chromosomal anomaly is it?

A Trisomy X
B Down’s disease
C Turner’s syndrome
D Klinefelter’s syndrome
E Specific fetopathy

60 To what total ATP quantity is the full glucose oxidation and its linking with
phosphorylation equivalent?

A 38
B8
C 58
D 52
E 12

61 A 2 year old child suffers from intestinal dysbacteriosis that lead to the development
hemorrhagic syndrome. The most probable cause of hemorrhage is:

A Vitamin K deficiency
B Hypovitaminosis PP
C Fibrinogen deficiency
D Hypocalcemia
E Activation of tissue thromboplastin

62 A patient who is ill with scurvy displays disturbed processes of connective tissue
formation that leads to loosening and falling of teeth. Disturbed activity of what enzyme
causes these symptomps?
V 02b91030

A Procollagenpeptidase of N-terminal peptide


B Elastase
C Glycosiltransferase
D Procollagenpeptidase of C-terminal peptide
E Lisilhydroxylase

63 A patient has increased content of uric acid in his blood that is clinically presented by
pain syndrome as a result of urate deposition in the joints. What process does this acid
result from?

A Heme catabolism
B Lysis of purine nucleotides
C Lysis of pyrimidine nucleotides
D Proteolysis
E Reutilization of purine bases

64 A child has disturbed enamel and dentine formation as a result of decreased content of
calcium ions in his blood. What hormone deficiency may cause such changes?

A Thyroxin
B Parathormone
C Triiodothyronine
D Thyreocalcitonin
E Somatotropin

65 While on holiday in the countryside a boy found a spider with the following
morphological peculiarities: body length at the rate of 2 cm, round black abdomen with two
rows of red dots on its dorsal surface, four pairs of segmented extremities covered with tiny
black hairs. Identify this arthropod:

A Mite
B Steppe spider (Latrodectus tredeci-mguttatus)
C Tarantula
D Solifugae
E Scorpion

66 Examination of a 30-year-old man mandible revealed in the region of his molar a dense
tumour-like formation that significantly deformed the mandible. The formation wasnt fully
detached from the bone tissue. Microscopical examination of a tissue sampling revealed that
stroma had some cords and follicles with odontogenous cylindric epithelial cells in
peripheria and stellate cells resembling of the enamel organ pulp in the centre. What is the
most likely diagnosis?

A Osteoclastoma
B Adenomatoid tumour
C Ameloblastoma
V 02b91030

D Primary intraosteal cancer


E Adenocarcinoma

67 Roentgenological examination of a patient revealed a cyst enclosing a tooth in its cavity


in the area of the premolar. Microscopical examination revealed that the cyst wall consisted
of connective tissue and was lined with multilayer squamous epithelium. What is the most
probable diagnosis?

A Follicular cyst
B Primordial cyst
C Radicular cyst
D Eosinophilic granuloma
E Epulis

68 Examination of an ill child’s blood revealed inherited hyperlipoproteinemia. Genetic


defect of what enzyme synthesis causes this phenomenon?

A Lipoprotein lipase
B Phenylalanine hydroxylase
C Proteinase
D Glycosidase
E Hemsynthetase

69 Osteolathyrism is characterized by a loss of tensile strength of collagen, which is induced


by a significant decrease in the formation of cross-links in collagen fibrils. The cause for it is
the reduced activity of:

A Prolyl hydroxylase
B Lysyl hydroxylase
C Collagenase
D Lysyl oxidase
E Monoamine oxidase

70 Up to 50% of world population aged above thirty is affected by paradontosis. The


leading part in pathogenesis of this disease is played by:

A Immune damage of tissues


B Parodontium tissues damaged by kalli-krein
C Parodontium damaged by active cells
D Dental calculus caused by microflora
E Neurodystrophic factor

71 A 10 year old child underwent Mantoux test (with tuberculin). 48 hours later there
apperaed a papule up to 8 mm in diameter on the site of tuberculin injection. Tuberculin
injection caused the following hypersensitivity reaction:

A II type hypersensitivity reaction


V 02b91030

B Arthus reaction
C Seroreaction
D Atopic reaction
E IV type hypersensitivity reaction

72 During examination of a 36-year-old woman a dentist revealed a formation in form of a


nodule up to 0,8 cm in diameter, of dark brown-red colour, soft, on a wide base. The
formation was found on the buccal surface of gum in the region of the 2nd molar.
Histological examination revealed that the formation had plenty of sinusoid vessels and a
lot of roundish mononuclear and big multinuclear cells; in some parts accumulations of
hemosiderin granules could be found. What is the most likely diagnosis?

A Giant-cell epulis
B Angiomatous epulis
C Mandibular osteoclastoma
D Ameloblastoma
E Root granuloma

73 A patient with chronic hypoglycemia had adrenaline introduction. After introduction


blood test has not changed essentially. Doctor assumed liver pathology. What liver function
may have been changed?

A Excretory function
B Function of cholesterin production
C Glycolytic function
D Ketogenic function
E Function of glycogen depositing

74 A 28 year old woman consulted a doctor about sterility. Examination revealed


underdeveloped ovaries and uterus, irregular menstrual cycle. Study of sex chromatin
revealed 2 Barr’s bodies in most somatic cells. What chromosome disease is the most
probable in this case?

A Turner’s syndrome
B Patau’s syndrome
C Edwards’ syndrome
D Klinefelter’s syndrome
E Triplo-X syndrome

75 A patient with focal tuberculosis of superior lobe of his right lung takes isoni-azid as a
part of combined therapy. After a time he started complaining of muscular weakness,
decrease of skin sensitivity, sight and movement coordination disorder. What vitamin
preparation will be right for elimination of these occurances?

A Vitamin A
B Vitamin D
V 02b91030

C Vitamin C
D Vitamin B6
E Vitamin B12

76 A dentist was examining oral cavity of a 9 year old child in the buccal surface of gingiva
in the area of the lower canine he revealed a red, soft, node-like formation 1 cm in diameter
that started immediately bleeding when touched. Microscopical examination revealed that
this formation consisted of many small vessels like venules and capillaries separated by thin
layers of connective tissue, with focal infiltration by lymphoid and plasmatic cells. Such
changes are typical for:

A Fibrous epulis
B Papilloma
C Capillary hemangioma
D Radicular granuloma
E Angiomatous epulis

77 After implantation of a cardiac valve a young man constantly takes indirect


anticoagulants. His state was complicated by hemorrhage. What substance content has
decreased in blood?

A Ceruloplasmin
B Heparin
C Prothrombin
D Creatin
E Haptoglobin

78 Decreased ratio of adenylic nucleotides ATP/ADP results in intensified glycolysis in


parodentium tissues in hypoxic conditions. What reaction is activated in this case?

A Lactate dehydrogenase
B Phosphofructokinase
C Enolase
D Triosophosphate isomerase
E Aldolase

79 A patient has deformation of jaw bones. Histological examination revealed there growth
of fibrocellular tumourlike ill-defined tissue with primitive osteogenesis. What disease are
these presentations typical for?

A Ameloblastoma
B Fibrous dysplasia
C Osteosarcoma
D Parathyroid osteodystrophy
E Eosinophilic granuloma
V 02b91030

80 In order to speed up healing of the thermal injury it is required to prescribe a drug that
facilitates epithelization of skin and mucous membranes. What drug is it?

A Ascorbic acid
B Retinol acetate
C Tocopherol acetate
D Nicotinic acid
E Ergocalciferol

81 A patient with diabetes mellitus had an insuline injection. It caused loss of consciousness
and convulsions. What was the result of biochemic blood analysis on glucose content?

A 3,3 mmole/l
B 10 mmole/l
C 2,5 mmole/l
D 5,5 mmole/l
E 8,0 mmole/l

82 A blood smear of a patient who has recently recovered from flu contains 10% of
roundish cells 4,5-7 micrometer large with a big round nucleus and basophi-lically stained
cytoplasm in form of a narrow border around the nucleus. What blood status are they
typical for?

A Lymphocytopenia
B Leukopenia
C Monocytopenia
D Thrombopenia
E Lymphocytosis

83 A patient was taken to the hospital with preliminary diagnosis progressive muscle
distrophy. What substance will be excessively contained in urine and confirm this
diagnosis?

A Troponine
B Hydroxiproline
C Carnosine
D Creatine
E Pyruvate

84 Chronic overdosage of glucocorticoids leads to the development of hyperglycemia. What


process of carbohydrate metabolism is responsible for this effect?

A Aerobic glycolisis
B Glycogenolysis
C Gluconeogenesis
D Pentose-phosphate cycle
V 02b91030

E Glycogenesis

85 Cytogenetic examination of a patient with dysfunction of the reproductive system


revealed normal karyotype 46,XY in some cells, but most cells have Klinefelter’s syndrome
karyotype - 47,XXY Such phenomenon of cell inhomogeneity is called:

A Duplication
B Heterogeneity
C Transposition
D Mosaicism
E Inversion

86 Myocyte cytoplasm contains a big number of dissolved metabolites of glucose oxidation.


Name one of them that turns directly into lactate:

A Fructose 6-phosphate
B Oxaloacetate
C Glucose 6-phosphate
D Pyruvate
E Glycerophosphate
V 02b91030

Keys: 1 B, 2 C, 3 D, 4 C, 5 B, 6 C, 7 D, 8 A, 9 D, 10 C, 11 E, 12 B, 13 A, 14 E, 15 E, 16 D, 17 B,
18 D, 19 C, 20 A, 21 B, 22 B, 23 E, 24 D, 25 A, 26 C, 27 E, 28 D, 29 A, 30 A, 31 B, 32 D, 33 D,
34 C, 35 D, 36 C, 37 B, 38 B, 39 C, 40 E, 41 E, 42 B, 43 A, 44 E, 45 B, 46 B, 47 B, 48 D, 49 E,
50 E, 51 A, 52 E, 53 A, 54 B, 55 A, 56 C, 57 E, 58 C, 59 B, 60 A, 61 A, 62 E, 63 B, 64 D, 65 B,
66 C, 67 A, 68 A, 69 D, 70 E, 71 E, 72 A, 73 E, 74 E, 75 D, 76 E, 77 C, 78 B, 79 B, 80 B, 81 C,
82 A, 83 D, 84 C, 85 D, 86 D,
Krok \ Dentistry \ Krok 1 \ Bases \ Biochemistry \ 2010
V 98705547
1 A child has disturbed processes of ossification and punctate enamel. What microelement metabolism is
disturbed?
A Iron
B Zinc
C Fluorine
D Chromium
E Copper

2 A patient applied to a doctor complaining about dizziness, memory impairment, periodical convulsions.
It was found out that such changes were caused by a product of glutamic acid decarboxylation. What
product is meant?
A Pyridoxalphosphate
B GABA
C Tetrahydrofolate
D Thymidine diphosphate
E ATP

3 Laboratory examination of a child revealed high content of leucine, valine, isoleucine and their
ketoderivates in blood and urine. Urine had the typical smell of maple syrup. This disease was caused by
deficiency of the following enzyme:
A Aminotransferase
B Phosphofructokinase
C Glucose-6-phosphatase
D Dehydrogenase of branched amino acids
E phosphofructomutase

4 A 40-year-old male patient had a tumour-like formation 8x7 cm large on his neck. A surgeon removed it
only partially because of close connection with large vessels. Microscopical examination revealed
marked cellular and tissue atypism, lipoblast-type cells in different stages of maturity, with
polymorphism and nuclear hyperchromia, pathological mitoses, necrosis foci. Specify the histological
form of the tumour:
A Lipoma
B Fibrosarcoma
C Liposarcoma
D Hibernoma
E Fibroma

5 A non trained man has usually muscular hypoxy after a sprint. What metabolite accumulates in the
muscles as a result of it?
A Oxaloacetate
B Ketone bodies
C Lactate
D Glucose 6-phosphate
E-

6 A 10 year old child underwent Mantoux test (with tuberculin). 48 hours later there apperaed a papule
up to 8 mm in diameter on the site of tuberculin injection. Tuberculin injection caused the following
hypersensitivity reaction:
A Arthus reaction
B Seroreaction
C II type hypersensitivity reaction
D Atopic reaction
E IV type hypersensitivity reaction

7 A patient with high obesity was recommended to take carnitine as a food additive for better fat
burning. What function is fulfilled by carnitine in the process of fat oxidation?
A Fatty acid activation
B Participation in one of the reactions of beta-oxidation of fatty acids
C Transport of fatty acids from the fat depots to the tissues
D Intracellular lipolysis activation
E Transport of fatty acids from the cytosol to the mitochondria

8 A blood smear of a patient who has recently recovered from flu contains 10% of roundish cells 4,5-7
micrometer large with a big round nucleus and basophi-lically stained cytoplasm in form of a narrow
border around the nucleus. What blood status are they typical for?
A Leukopenia
B Lymphocytosis
C Monocytopenia
D Thrombopenia
E Lymphocytopenia

9 Hepatic disfunctions accompanied by insufficient inflow of bile to the bowels result in coagulation
failure. This phenomenon can be explained by:
A Erythropenia
B Vitamin K deficiency
C Thrombocytopenia
D Leukopenia
E Iron deficiency

10 A patient has increased content of uric acid in his blood that is clinically presented by pain syndrome
as a result of urate deposition in the joints. What process does this acid result from?
A Proteolysis
B Heme catabolism
C Lysis of pyrimidine nucleotides
D Lysis of purine nucleotides
E Reutilization of purine bases

11 Examination of urine in a newborn revealed presence of citrulline and high ammonia concentration.
This baby is most likely to have the disorder of the following substance production:
A Creatinine
B Uric acid
C Creatine
D Urea
E Ammonia

12 A patient has liver cirrhosis. Analysis of what substance excreted with urine may characterize the
state of antitoxic liver function?
A Uric acid
B Creatinine
C Hippuric acid
D Amino acids
E Ammonian salts

13 While the examination of patient’s oral cavity the dentist found xerostomia, numerous erosions. What
vitamin deficit caused this effect?
A Vitamin PP
B Vitamin K
C Vitamin P
D Vitamin A
E Vitamin H

14 A patient has increased permeability of blood-vessel walls, increased gingival hemorrhage, small
punctate hematomas on his skin, falling of teeth. What disturbance of vitamun metabolism can account
for these symptoms?
A Hypervitaminosis D
B Hypovitaminosis A
C Hypovitaminosis C
D Hypovitaminosis D
E Hypervitaminosis C

15 The activity of parotides reduces with age. Activity of what enzyme in saliva will be reducing?
A Lysozime
B Maltase
C Hexokinase
D Amylase
E Phosphatase

16 Analysis of urine from a 24-year-old man revealed the following changes: daily diuresis - 10 l, relative
density - 1,001, qualitative alterations are absent. A patient complains of excessive thirst, frequent
urination. What is the most likely cause of this disease?
A Aldosteron hypersecretion
B Relative insulin insufficiency
C Vasopressin hyposecretion
D Glucocorticoid hypersecretion
E Vasopressin hypersecretion

17 A female patient has symptoms of inflammation of urogenital tracts. A smear from the vaginal
mucous membrane contained big unicellular pyriform organisms with a sharp spike on the back end of
their bodies; big nucleus and undulating membrane. What protozoa were revealed in the smear?
A Trichomonas buccalis
B Trypanosoma gambiense
C Trichomonas hominis
D Trichomonas vaginalis
E Lamblia intestinalis

18 A patient suffering from syphilis was prescribed a drug the action of which based upon disturbed
generation of murein leading to death of the causative agent. What drug is it?
A Azithromycin
B Bijochinol
C Ciprofloxacin
D Benzylpenicillin sodium salt
E Doxycycline hydrochloride

19 Decreased ratio of adenylic nucleotides ATP/ADP results in intensified glycolysis in parodentium


tissues in hypoxic conditions. What reaction is activated in this case?
A Lactate dehydrogenase
B Triosophosphate isomerase
C Phosphofructokinase
D Aldolase
E Enolase

20 Examination of a 6 days old infant revealed phenyl pyruvate and phenyl acetate excess in his urine.
What ami-noacid metabolism is disturbed in the child’s organism?
A Tryptophan
B Histidine
C Phenylalanine
D Methionine
E Arginine

21 Examination of a child who hasn’t got fresh fruit and vegetables during winter revealed numerous
subcutaneous hemorrhages, gingivitis, carious cavities in teeth. What vitamin combination should be
prescribed in this case?
A Ascorbic acid and rutin
B Folic acid and cobalamin
C Thiamine and pyridoxine
D Riboflavin and nicotinamide
E Calciferol and ascorbic acid

22 Two weeks after hemotransfusion a patient developed fever. What protozoal disease can be
suspected?
A Trypanosomiasis
B Amebiasis
C Leishmaniasis
D Toxoplasmosis
E Malaria

23 Osteolathyrism is characterized by a loss of tensile strength of collagen, which is induced by a


significant decrease in the formation of cross-links in collagen fibrils. The cause for it is the reduced
activity of:
A Lysyl oxidase
B Prolyl hydroxylase
C Collagenase
D Monoamine oxidase
E Lysyl hydroxylase

24 A sportsman needs to improve his sporting results. He was recommended a drug containing
carnitine. What process is activated by this compound in the first place?
A Transport of glucose
B Transport of vitamin K
C Transport of amino acids
D Transport of calcium ions
E Transport of fatty acids

25 While on holiday in the countryside a boy found a spider with the following morphological
peculiarities: body length at the rate of 2 cm, round black abdomen with two rows of red dots on its
dorsal surface, four pairs of segmented extremities covered with tiny black hairs. Identify this arthropod:
A Scorpion
B Tarantula
C Mite
D Solifugae
E Steppe spider (Latrodectus tredeci-mguttatus)

26 A patient has deformation of jaw bones. Histological examination revealed there growth of
fibrocellular tumourlike ill-defined tissue with primitive osteogenesis. What disease are these
presentations typical for?
A Eosinophilic granuloma
B Parathyroid osteodystrophy
C Ameloblastoma
D Osteosarcoma
E Fibrous dysplasia

27 During physical exercise people are less sensitive to pain. The reason for it is the activation of:
A Antinociceptive system
B Sympathoadrenal system
C Thyroid gland functions
D Adrenal gland functions
E Nociceptive system

28 To what total ATP quantity is the full glucose oxidation and its linking with phosphorylation
equivalent?
A 12
B 58
C 52
D8
E 38

29 A 2 year old child suffers from intestinal dysbacteriosis that lead to the development hemorrhagic
syndrome. The most probable cause of hemorrhage is:
A Hypovitaminosis PP
B Activation of tissue thromboplastin
C Hypocalcemia
D Fibrinogen deficiency
E Vitamin K deficiency

30 A patient with chronic hypoglycemia had adrenaline introduction. After introduction blood test has
not changed essentially. Doctor assumed liver pathology. What liver function may have been changed?
A Ketogenic function
B Excretory function
C Glycolytic function
D Function of cholesterin production
E Function of glycogen depositing

31 Patient with pigmentary xeroderma are characterized by anamalously high sensivity to ultraviolet
rays that causes skin cancer as a result of enzyme systems incapability to restore damages of hereditary
apparatus of cells. What process abnormality is this pathology connected with?
A Genetic conversion
B DNA reduplication
C Genetic complementation
D DNA reparation
E DNA recombination

32 A 57 year old patient with diabetes mellitus was developed ketoacedosis. Biochemical base of this
condition is decrease of acetyl-CoA utilization. What cell compound deficit causes this effect?
A Glutamate
B Oxaloacetate
C 2-oxoglutarate
D Succinate
E Aspartate

33 Microscopical examination of a surgical specimen (an ulcered part of a lip) revealed in the connective
tissue of mucous membrane near the borders and under the floor of the ulcer some epithelial complexes
consisting of atypic multi-stratal epithelium with accumulations of bright pink concentric formations.
What pathology is it?
A Squamous cell nonkeratinous carcinoma
B Transitional cell carcinoma
C Papilloma
D Basal cell carcinoma
E Squamous cell keratinous carcinoma

34 Active physical work induces rise of concentration of carbonic acid in blood. This causes deepening
and acceleration of respiration thus reducing concentration of carbonic acid and hydrogen ions in blood.
This maintains the following process:
A Ontogenesis
B Orthobiosis
C Immunity
D Homeostasis
E Anabiosis

35 A patient who is ill with scurvy displays disturbed processes of connective tissue formation that leads
to loosening and falling of teeth. Disturbed activity of what enzyme causes these symptomps?
A Lisilhydroxylase
B Elastase
C Glycosiltransferase
D Procollagenpeptidase of N-terminal peptide
E Procollagenpeptidase of C-terminal peptide

36 Helminthological examination of patient’s feces revealed oval brown eggs with tuberous external
membrane. Name the type of helminth:
A Ascarid
B Broad tapeworm
C Dwarf tapeworm
D Whipworm
E Pinworm

37 A dentist was examining oral cavity of a 9 year old child in the buccal surface of gingiva in the area of
the lower canine he revealed a red, soft, node-like formation 1 cm in diameter that started immediately
bleeding when touched. Microscopical examination revealed that this formation consisted of many small
vessels like venules and capillaries separated by thin layers of connective tissue, with focal infiltration by
lymphoid and plasmatic cells. Such changes are typical for:
A Fibrous epulis
B Angiomatous epulis
C Radicular granuloma
D Papilloma
E Capillary hemangioma

38 In the mountains some clinically healthy people present with anaemia symptoms. Blood test can
reveal sickle cells. What is the genotype of such people?
A XCXc
B XcXc
C AA
D Aa
E aa

39 A 15 year old girl was delivered to the hospital with inflammation of vermiform appendix. Blood
analysis revealed signs of anaemia. Her feces contained lemon-shaped helminthic eggs (50x30
micrometer) with "plugs" on the poles. What type of helminth is it?
A Echinococcus
B Trichuris
C Hookworm
D Pinworm
E Hymenolepis nana

40 Coprological examination of a patient’s feces revealed small operculate eggs. It is known from the
anamnesis that the patient often consumes fish. What fluke parasitizes in the patient’s organism?
A Lung fluke
B Lancet fluke
C Cat liver fluke
D Liver fluke
E Blood fluke

41 Dehelmintization of a patient revealed some long fragments of a helminth with segmented structure.
Mature segments were rectangular, 30x12 mm large, closed-type matrix was in form of a stem with 17-
35 lateral branches. Specify this helminth:
A Armed tapeworm
B Echinococcus
C Dwarf tapeworm
D Alveococcus
E Hookless tapeworm

42 Roentgenological examination of a patient revealed a cyst enclosing a tooth in its cavity in the area of
the premolar. Microscopical examination revealed that the cyst wall consisted of connective tissue and
was lined with multilayer squamous epithelium. What is the most probable diagnosis?
A Radicular cyst
B Epulis
C Primordial cyst
D Eosinophilic granuloma
E Follicular cyst

43 Up to 50% of world population aged above thirty is affected by paradontosis. The leading part in
pathogenesis of this disease is played by:
A Parodontium tissues damaged by kalli-krein
B Dental calculus caused by microflora
C Neurodystrophic factor
D Parodontium damaged by active cells
E Immune damage of tissues

44 Cytogenetic examination of a patient with dysfunction of the reproductive system revealed normal
karyotype 46,XY in some cells, but most cells have Klinefelter’s syndrome karyotype - 47,XXY Such
phenomenon of cell inhomogeneity is called:
A Duplication
B Inversion
C Transposition
D Mosaicism
E Heterogeneity

45 A patient has the folowing changes: disorder of twilight vision, drying out of conjunctiva and cornea.
Such disorders may be caused by deficiency of vitamin:
A Vitamin C
B Vitamin A
C Vitamin B12
D Vitamin D
E Vitamin B

46 In compliance with the clinical presentations a man was prescribed pyri-doxalphosphate. What
processes are corrected by this preparation?
A Synthesis of purine and pyrimidine bases
B Protein synthesis
C Oxidative decarboxilation of keto acids
D Desamination of purine nucleotides
E Transamination and decarboxylation of amino acids

47 In order to speed up healing of the thermal injury it is required to prescribe a drug that facilitates
epithelization of skin and mucous membranes. What drug is it?
A Retinol acetate
B Ascorbic acid
C Nicotinic acid
D Tocopherol acetate
E Ergocalciferol

48 A sportsman was recommended to take a preparation with carnitine in order to improve his
achievements. What process is activated by carnitine to the most extent?
A Synthesis of ketone bodies
B Lypide synthesis
C Synthesis of steroid hormones
D Transporting of fatty acids to the mi-tochondrions
E Tissue respiration

49 Examination of a 10 y.o. child revealed on the alveolar submandibular process a fixed tumourous
mass 1,5 cm in diameter closing premolar crown on the vestibular side. Mucous membrane of its surface
is reddish-brown, it bleeds as a reaction to a slight mechanical intervention. Biopsy results: the mass
consists of small size vessels separated by thin layers of connective tissue and infiltrated by plasmocytes,
mucous membrane is here and there ulcered. What is the most probable diagnosis?
A Angiomatous form of epulis
B Gingival fibromatosis
C Hypertrophic gingivitis
D Giant cell form of epulis
E Fibrous form of epulis

50 Chronic overdosage of glucocorticoids leads to the development of hyperglycemia. What process of


carbohydrate metabolism is responsible for this effect?
A Aerobic glycolisis
B Glycogenolysis
C Pentose-phosphate cycle
D Glycogenesis
E Gluconeogenesis

51 While examining a blood smear taken form a patient and stained by Romanovsky’s method a doctor
revealed some protozoa and diagnozed the patient with Chagas disease. What protozoan is the causative
agent of this disease?
A Toxoplasma gondii
B Leishmania tropica
C Leishmania donovani
D Trypanosoma brucei
E Trypanosoma cruzi

52 A patient has mental retardation, small height, brachydactyly, mongoloid slant. Analysys of his
karyotype revealed trisomy 21. What chromosomal anomaly is it?
A Trisomy X
B Down’s disease
C Klinefelter’s syndrome
D Specific fetopathy
E Turner’s syndrome

53 Histological examination of a micro specimen presenting a malignant lung tumour revealed that the
tumor consisted of lymphocyte-like cells forming any structures. Stroma is mildely marked, there are a
lot of mitoses and necroses. What tumour is it?
A Squamous cell keratinous carcinoma
B Squamous cell nonkeratinous carcinoma
C Adenocarcinoma
D Small cell carcinoma
E Fibroma

54 A child has abnormal formation of tooth enamel and dentin as a result of low concentration of calcium
ions in blood. Such abnormalities might be caused by deficiency of the following hormone:
A Thyrocalcitonin
B Parathormone
C Triiodothyronine
D Thyroxin
E Somatotropic hormone

55 A child has disturbed enamel and dentine formation as a result of decreased content of calcium ions
in his blood. What hormone deficiency may cause such changes?
A Thyroxin
B Triiodothyronine
C Thyreocalcitonin
D Parathormone
E Somatotropin

56 Microscopical examination of discharges from the gums of a patient ill with paradontosis revealed
some protozoan pear-shaped organisms 6-13 micrometer long. The parasite has one nucleus and
undulating membrane, there are four flagella at the front of its body. What protozoan were found?
A Trichomonads
B Leishmania
C Balantidia
D Lamblia
E Amoebae

57 Examination of a 30-year-old man mandible revealed in the region of his molar a dense tumour-like
formation that significantly deformed the mandible. The formation wasnt fully detached from the bone
tissue. Microscopical examination of a tissue sampling revealed that stroma had some cords and follicles
with odontogenous cylindric epithelial cells in peripheria and stellate cells resembling of the enamel
organ pulp in the centre. What is the most likely diagnosis?
A Adenomatoid tumour
B Ameloblastoma
C Adenocarcinoma
D Osteoclastoma
E Primary intraosteal cancer

58 For assessment of the neutralizing function of liver a patient with chronic hepatitis went through a
test with natrium benzoate load. The excretion of what acid with urine will characterize the neutralizing
function of liver?
A Valeric acid
B Citric acid
C Phenylacetic acid
D Oxalic acid
E Hippuric acid

59 A patient has the sudden decrease of Са2+ content in blood. What hormone secretion will increase?
A Parathormone
B Aldosterone
C Vasopressin
D Thyrocalcitonin
E Somatotropin

60 A sample taken from the pharynx of a patient with angina was inoculated on the blood-tellurite agar.
This resulted in growth of grey, radially striated (in form of rosettes) colonies up to 4-5 mm in diameter.
Microscopically there can be seen gram-positive rods with club-shaped ends arranged in form of spread
fingers. What microorganisms are these?
A Diphtheroids
B Streptococci
C Streptobacilli
D Clostridium botulinum
E Corynebacteria diphtheriae

61 Periodontitis is accompanied by activation of proteolysis in the periodontium tissues. The evidence of


proteolysis activation is increase of the following component of oral liquid:
A Cholesterol
B Amino acids
C Organic acids
D Glucose
E Biogenic amines

62 Roentgenological examination of mandible of a 27 year old man revealed a focus of osseous tissue
destruction. Histological examination revealed a tumour consisting of odontogenous epithelium cords,
immature connective tissue and dysplastic dentin rests. What tumour is it?
A Ameloblastic fibro-odontoma
B Odontoameloblastoma
C Odontogenous fibroma
D Dentinoma
E Complex odontoma

63 Cyanide poisoning causes immediate death. What is the mechanism of cyanide effect at the molecular
level?
A They bind substrates of tricarboxylic acid cycle
B They block succinate dehydrogenase
C They inhibit cytochrome B
D They inactivate oxygene
E They inhibit cytochromoxidase

64 A 28 year old pregnant woman had the enzymes in the cells of amniotic fluid analyzed. The analysis
revealed insufficient activity of beta-glucuronidase. What pathological process is it?
A Mucopolysaccharidosis
B Collagenosis
C Glycogenosis
D Lipidosis
E Aglycogenosis

65 The first grade pupils went through a medical examination aimed at selection of children needing
tuberculosis revaccination. What test was applied?
A Mantoux test
B Anthracene test
C Supracutaneous tularin test
D Burne test
E Schick test

66 A patient was taken to the hospital with preliminary diagnosis progressive muscle distrophy. What
substance will be excessively contained in urine and confirm this diagnosis?
A Troponine
B Carnosine
C Creatine
D Pyruvate
E Hydroxiproline

67 A patient has roundish ulcers on his face, inflammation and enlargement of lymph nodes. These
symptoms turned up as a result of mosquito bites. Laboratory examination of discharge from the ulcers
revealed unicellular aflagellar organisms. What is the most probable diagnosis?
A Toxoplasmosis
B Scabies
C Myasis
D Dermatotropic leishmaniasis
E Trypanosomiasis

68 A patient with focal tuberculosis of superior lobe of his right lung takes isoni-azid as a part of
combined therapy. After a time he started complaining of muscular weakness, decrease of skin
sensitivity, sight and movement coordination disorder. What vitamin preparation will be right for
elimination of these occurances?
A Vitamin D
B Vitamin C
C Vitamin B6
D Vitamin A
E Vitamin B12

69 A girl who was provisionally diagnosed with Turner’s syndrome came to a genetic consultation. The
diagnosis can be specified by means of the following genetic method:
A Sex chromatin test
B Dermatoglyphics
C Genealogical
D Hybridological
E Biochemical

70 A patient has painfulness along big nerve trunks and excessive content of pyruvate in blood. What
vitamin deficit may cause such changes?
A PP
B Pantothenic acid
C B2
D Biotin
EBi

71 Myocyte cytoplasm contains a big number of dissolved metabolites of glucose oxidation. Name one of
them that turns directly into lactate:
A Glucose 6-phosphate
B Pyruvate
C Fructose 6-phosphate
D Oxaloacetate
E Glycerophosphate

72 A hospital admitted a patient with complaints about abdominal swelling, diarrhea, meteorism after
consumption of food rich in proteins. It is indicative of disturbed protein digestion and their intensified
decaying. What substance is the product of this process in the bowels?
A Bilirubin
B Agmatine
C Indole
D Putrescine
E Cadaverine

73 A 38 year old patient takes aspirin and sulfanilamides. After their intake intensified erythrocyte
haemolysis is observed which is caused by deficiency of glucose 6-phosphate dehydrogenase. This
pathology is caused by failure of the following coenzyme:
A FAD - H2
B NADP - H
C Pyridoxal phosphate
D Ubiquinone
E FMN - H2

74 The 16th tooth of a patient is missing. X-ray picture shows in the depth of alveolar process
rarefaction of bone and a well-defined cavity that contained the underdeveloped tooth crown.
Microscopical examination revealed that the cavity wall was lined with stratified squamous epithelium
and enclosed within a fibrous capsule. Make a diagnosis:
A Primordial gnathic cyst
B Follicular gnathic cyst
C Radicular gnathic cyst
D Cyst of the incisive canal
E Cystic ameloblastoma of jaw

75 A chiled was diagnosed with acute renal failure. What biochemic saliva indices can confirm this
diagnosis?
A Increase of alpha amylase
B Reduction of alkaline phosphatase
C Decreased level of phosphate
D Increase of immunoglobuline A
E Increased level of rest nitrogen

76 A 42 year old woman diagnosed with diabetes mellitus was admitted the endocrinological department
with complaints of thirst, excessive appetite. What pathological components are revealed in course of
laboratory examination of the patients urine?
A Protein, aminoacids
B Bilirubin, urobilin
C Glucose, ketone bodies
D Blood
E Protein, creatine

77 A man got poisoned with mushrooms. They contain muscarine that stimulates muscarinic
cholinoreceptors. What symptom is typical for poisoning with inedible mushrooms?
A Heart rate rise
B Bronchi dilation
C Miosis
D Mydriasis
E Arterial pressure rise

78 Examination of an ill child’s blood revealed inherited hyperlipoproteinemia. Genetic defect of what
enzyme synthesis causes this phenomenon?
A Phenylalanine hydroxylase
B Lipoprotein lipase
C Hemsynthetase
D Proteinase
E Glycosidase

79 After implantation of a cardiac valve a young man constantly takes indirect anticoagulants. His state
was complicated by hemorrhage. What substance content has decreased in blood?
A Ceruloplasmin
B Haptoglobin
C Creatin
D Heparin
E Prothrombin

80 A 28 year old woman consulted a doctor about sterility. Examination revealed underdeveloped
ovaries and uterus, irregular menstrual cycle. Study of sex chromatin revealed 2 Barr’s bodies in most
somatic cells. What chromosome disease is the most probable in this case?
A Turner’s syndrome
B Triplo-X syndrome
C Patau’s syndrome
D Edwards’ syndrome
E Klinefelter’s syndrome

81 Laboratory of extremely dangerous infections received a sample taken from a patient with assumed
cholera. What express-diagnostics method can confirm this diagnosis?
A Hemagglutination reaction
B Precipitation reaction
C Complement binding reaction
D Immunofluorescence test
E Agglutination test
82 Among public catering workers examined by doctors of sanitary-and-epidemiologic station often
occur asymptomatic parasite carriers. This means that a healthy person carries cysts that infect other
people. Such parasitizing is impossible for the following causative agent:
A Intestinal trichomonad
B Viscerotropic leishmania
C Dermatotropic leishmania
D Malarial plasmodium
E Dysenteric amoeba

83 During examination of a 36-year-old woman a dentist revealed a formation in form of a nodule up to


0,8 cm in diameter, of dark brown-red colour, soft, on a wide base. The formation was found on the
buccal surface of gum in the region of the 2nd molar. Histological examination revealed that the
formation had plenty of sinusoid vessels and a lot of roundish mononuclear and big multinuclear cells; in
some parts accumulations of hemosiderin granules could be found. What is the most likely diagnosis?
A Angiomatous epulis
B Root granuloma
C Mandibular osteoclastoma
D Giant-cell epulis
E Ameloblastoma

84 A patient was diagnosed with seborrheic dermatitis associated with vitamin H (biotin) deficiency. The
patient has disturbed activity of the following enzyme:
A Pyruvate decarboxylase
B Amino transferase
C Alcohol dehydrogenase
D Carbomoyl phosphate synthetase
E Acetyl-CoA-carboxylase

85 A patient with diabetes mellitus had an insuline injection. It caused loss of consciousness and
convulsions. What was the result of biochemic blood analysis on glucose content?
A 10 mmole/l
B 2,5 mmole/l
C 3,3 mmole/l
D 8,0 mmole/l
E 5,5 mmole/l

86 A one year old child has enlarged head and belly, retarded cutting of teeth, destruction of enamel
structure. What hypovitaminosis causes these changes?
A Hypovitaminosis B 1
B Hypovitaminosis B2
C Hypovitaminosis D
D Hypovitaminosis C
E Hypovitaminosis A
V 98705547

Keys: 1 C, 2 B, 3 D, 4 C, 5 C, 6 E, 7 E, 8 E, 9 B, 10 D, 11 D, 12 C, 13 D, 14 C, 15 D, 16 C, 17 D, 18 D, 19
C, 20 C, 21 A, 22 E, 23 A, 24 E, 25 E, 26 E, 27 A, 28 E, 29 E, 30 E, 31 D, 32 B, 33 E, 34 D, 35 A, 36 A,
37 B, 38 D, 39 B, 40 C, 41 E, 42 E, 43 C, 44 D, 45 B, 46 E, 47 A, 48 D, 49 A, 50 E, 51 E, 52 B, 53 D, 54
B, 55 C, 56 A, 57 B, 58 E, 59 A, 60 E, 61 B, 62 D, 63 E, 64 A, 65 A, 66 C, 67 D, 68 C, 69 A, 70 E, 71 B,
72 C, 73 B, 74 B, 75 E, 76 C, 77 C, 78 B, 79 E, 80 B, 81 D, 82 E, 83 D, 84 E, 85 B, 86 C,
Krok \ Dentistry \ Krok 1 \ Bases \ Biochemistry \ 2010
V efc41401
1 Roentgenological examination of a patient revealed a cyst enclosing a tooth in its cavity in the area of
the premolar. Microscopical examination revealed that the cyst wall consisted of connective tissue and
was lined with multilayer squamous epithelium. What is the most probable diagnosis?
A Follicular cyst
B Radicular cyst
C Primordial cyst
D Eosinophilic granuloma
E Epulis

2 A dentist was examining oral cavity of a 9 year old child in the buccal surface of gingiva in the area of
the lower canine he revealed a red, soft, node-like formation 1 cm in diameter that started immediately
bleeding when touched. Microscopical examination revealed that this formation consisted of many small
vessels like venules and capillaries separated by thin layers of connective tissue, with focal infiltration by
lymphoid and plasmatic cells. Such changes are typical for:
A Angiomatous epulis
B Capillary hemangioma
C Radicular granuloma
D Fibrous epulis
E Papilloma

3 Analysis of urine from a 24-year-old man revealed the following changes: daily diuresis - 10 l, relative
density - 1,001, qualitative alterations are absent. A patient complains of excessive thirst, frequent
urination. What is the most likely cause of this disease?
A Vasopressin hyposecretion
B Glucocorticoid hypersecretion
C Vasopressin hypersecretion
D Relative insulin insufficiency
E Aldosteron hypersecretion

4 A patient suffering from syphilis was prescribed a drug the action of which based upon disturbed
generation of murein leading to death of the causative agent. What drug is it?
A Benzylpenicillin sodium salt
B Bijochinol
C Ciprofloxacin
D Azithromycin
E Doxycycline hydrochloride

5 Osteolathyrism is characterized by a loss of tensile strength of collagen, which is induced by a


significant decrease in the formation of cross-links in collagen fibrils. The cause for it is the reduced
activity of:
A Lysyl oxidase
B Monoamine oxidase
C Prolyl hydroxylase
D Lysyl hydroxylase
E Collagenase

6 Periodontitis is accompanied by activation of proteolysis in the periodontium tissues. The evidence of


proteolysis activation is increase of the following component of oral liquid:
A Amino acids
B Organic acids
C Glucose
D Biogenic amines
E Cholesterol

7 Two weeks after hemotransfusion a patient developed fever. What protozoal disease can be suspected?
A Malaria
B Toxoplasmosis
C Leishmaniasis
D Amebiasis
E Trypanosomiasis

8 A patient applied to a doctor complaining about dizziness, memory impairment, periodical convulsions.
It was found out that such changes were caused by a product of glutamic acid decarboxylation. What
product is meant?
A GABA
B Pyridoxalphosphate
C Thymidine diphosphate
D ATP
E Tetrahydrofolate

9 Laboratory examination of a child revealed high content of leucine, valine, isoleucine and their
ketoderivates in blood and urine. Urine had the typical smell of maple syrup. This disease was caused by
deficiency of the following enzyme:
A Dehydrogenase of branched amino acids
B Aminotransferase
C Glucose-6-phosphatase
D Phosphofructokinase
E phosphofructomutase

10 Hepatic disfunctions accompanied by insufficient inflow of bile to the bowels result in coagulation
failure. This phenomenon can be explained by:
A Vitamin K deficiency
B Iron deficiency
C Thrombocytopenia
D Erythropenia
E Leukopenia

11 A child has disturbed processes of ossification and punctate enamel. What microelement metabolism
is disturbed?
A Fluorine
B Chromium
C Copper
D Iron
E Zinc

12 During examination of a 36-year-old woman a dentist revealed a formation in form of a nodule up to


0,8 cm in diameter, of dark brown-red colour, soft, on a wide base. The formation was found on the
buccal surface of gum in the region of the 2nd molar. Histological examination revealed that the
formation had plenty of sinusoid vessels and a lot of roundish mononuclear and big multinuclear cells; in
some parts accumulations of hemosiderin granules could be found. What is the most likely diagnosis?
A Giant-cell epulis
B Root granuloma
C Angiomatous epulis
D Ameloblastoma
E Mandibular osteoclastoma

13 While on holiday in the countryside a boy found a spider with the following morphological
peculiarities: body length at the rate of 2 cm, round black abdomen with two rows of red dots on its
dorsal surface, four pairs of segmented extremities covered with tiny black hairs. Identify this arthropod:
A Steppe spider (Latrodectus tredeci-mguttatus)
B Scorpion
C Solifugae
D Mite
E Tarantula

14 A girl who was provisionally diagnosed with Turner’s syndrome came to a genetic consultation. The
diagnosis can be specified by means of the following genetic method:
A Sex chromatin test
B Genealogical
C Hybridological
D Biochemical
E Dermatoglyphics

15 Cyanide poisoning causes immediate death. What is the mechanism of cyanide effect at the molecular
level?
A They inhibit cytochromoxidase
B They bind substrates of tricarboxylic acid cycle
C They block succinate dehydrogenase
D They inactivate oxygene
E They inhibit cytochrome B

16 A blood smear of a patient who has recently recovered from flu contains 10% of roundish cells 4,5-7
micrometer large with a big round nucleus and basophi-lically stained cytoplasm in form of a narrow
border around the nucleus. What blood status are they typical for?
A Lymphocytopenia
B Thrombopenia
C Leukopenia
D Lymphocytosis
E Monocytopenia

17 A patient was diagnosed with seborrheic dermatitis associated with vitamin H (biotin) deficiency. The
patient has disturbed activity of the following enzyme:
A Acetyl-CoA-carboxylase
B Pyruvate decarboxylase
C Alcohol dehydrogenase
D Amino transferase
E Carbomoyl phosphate synthetase

18 Active physical work induces rise of concentration of carbonic acid in blood. This causes deepening
and acceleration of respiration thus reducing concentration of carbonic acid and hydrogen ions in blood.
This maintains the following process:
A Homeostasis
B Immunity
C Ontogenesis
D Orthobiosis
E Anabiosis

19 The activity of parotides reduces with age. Activity of what enzyme in saliva will be reducing?
A Amylase
B Lysozime
C Phosphatase
D Hexokinase
E Maltase

20 Chronic overdosage of glucocorticoids leads to the development of hyperglycemia. What process of


carbohydrate metabolism is responsible for this effect?
A Gluconeogenesis
B Glycogenolysis
C Aerobic glycolisis
D Pentose-phosphate cycle
E Glycogenesis

21 A patient has mental retardation, small height, brachydactyly, mongoloid slant. Analysys of his
karyotype revealed trisomy 21. What chromosomal anomaly is it?
A Down’s disease
B Klinefelter’s syndrome
C Turner’s syndrome
D Trisomy X
E Specific fetopathy

22 Examination of urine in a newborn revealed presence of citrulline and high ammonia concentration.
This baby is most likely to have the disorder of the following substance production:
A Urea
B Uric acid
C Ammonia
D Creatinine
E Creatine

23 In the mountains some clinically healthy people present with anaemia symptoms. Blood test can
reveal sickle cells. What is the genotype of such people?
A Aa
B aa
C AA
D XcXc
E XCXc

24 A hospital admitted a patient with complaints about abdominal swelling, diarrhea, meteorism after
consumption of food rich in proteins. It is indicative of disturbed protein digestion and their intensified
decaying. What substance is the product of this process in the bowels?
A Indole
B Bilirubin
C Cadaverine
D Agmatine
E Putrescine

25 A 38 year old patient takes aspirin and sulfanilamides. After their intake intensified erythrocyte
haemolysis is observed which is caused by deficiency of glucose 6-phosphate dehydrogenase. This
pathology is caused by failure of the following coenzyme:
A NADP - H
B FAD - H2
C Pyridoxal phosphate
D FMN - H2
E Ubiquinone

26 Up to 50% of world population aged above thirty is affected by paradontosis. The leading part in
pathogenesis of this disease is played by:
A Neurodystrophic factor
B Parodontium tissues damaged by kalli-krein
C Parodontium damaged by active cells
D Dental calculus caused by microflora
E Immune damage of tissues

27 A 57 year old patient with diabetes mellitus was developed ketoacedosis. Biochemical base of this
condition is decrease of acetyl-CoA utilization. What cell compound deficit causes this effect?
A Oxaloacetate
B Glutamate
C 2-oxoglutarate
D Aspartate
E Succinate

28 A patient was taken to the hospital with preliminary diagnosis progressive muscle distrophy. What
substance will be excessively contained in urine and confirm this diagnosis?
A Creatine
B Pyruvate
C Carnosine
D Troponine
E Hydroxiproline

29 To what total ATP quantity is the full glucose oxidation and its linking with phosphorylation
equivalent?
A 38
B8
C 12
D 52
E 58

30 Microscopical examination of a surgical specimen (an ulcered part of a lip) revealed in the connective
tissue of mucous membrane near the borders and under the floor of the ulcer some epithelial complexes
consisting of atypic multi-stratal epithelium with accumulations of bright pink concentric formations.
What pathology is it?
A Squamous cell keratinous carcinoma
B Squamous cell nonkeratinous carcinoma
C Transitional cell carcinoma
D Basal cell carcinoma
E Papilloma

31 Examination of a child who hasn’t got fresh fruit and vegetables during winter revealed numerous
subcutaneous hemorrhages, gingivitis, carious cavities in teeth. What vitamin combination should be
prescribed in this case?
A Ascorbic acid and rutin
B Thiamine and pyridoxine
C Folic acid and cobalamin
D Riboflavin and nicotinamide
E Calciferol and ascorbic acid

32 Coprological examination of a patient’s feces revealed small operculate eggs. It is known from the
anamnesis that the patient often consumes fish. What fluke parasitizes in the patient’s organism?
A Cat liver fluke
B Blood fluke
C Lung fluke
D Liver fluke
E Lancet fluke
33 While the examination of patient’s oral cavity the dentist found xerostomia, numerous erosions. What
vitamin deficit caused this effect?
A Vitamin A
B Vitamin K
C Vitamin P
D Vitamin H
E Vitamin PP

34 While examining a blood smear taken form a patient and stained by Romanovsky’s method a doctor
revealed some protozoa and diagnozed the patient with Chagas disease. What protozoan is the causative
agent of this disease?
A Trypanosoma cruzi
B Toxoplasma gondii
C Leishmania donovani
D Leishmania tropica
E Trypanosoma brucei

35 A 10 year old child underwent Mantoux test (with tuberculin). 48 hours later there apperaed a papule
up to 8 mm in diameter on the site of tuberculin injection. Tuberculin injection caused the following
hypersensitivity reaction:
A IV type hypersensitivity reaction
B Arthus reaction
C Seroreaction
D Atopic reaction
E II type hypersensitivity reaction

36 A chiled was diagnosed with acute renal failure. What biochemic saliva indices can confirm this
diagnosis?
A Increased level of rest nitrogen
B Increase of immunoglobuline A
C Reduction of alkaline phosphatase
D Increase of alpha amylase
E Decreased level of phosphate

37 A sportsman was recommended to take a preparation with carnitine in order to improve his
achievements. What process is activated by carnitine to the most extent?
A Transporting of fatty acids to the mi-tochondrions
B Synthesis of steroid hormones
C Synthesis of ketone bodies
D Lypide synthesis
E Tissue respiration

38 After implantation of a cardiac valve a young man constantly takes indirect anticoagulants. His state
was complicated by hemorrhage. What substance content has decreased in blood?
A Prothrombin
B Haptoglobin
C Heparin
D Creatin
E Ceruloplasmin

39 A 28 year old woman consulted a doctor about sterility. Examination revealed underdeveloped
ovaries and uterus, irregular menstrual cycle. Study of sex chromatin revealed 2 Barr’s bodies in most
somatic cells. What chromosome disease is the most probable in this case?
A Triplo-X syndrome
B Edwards’ syndrome
C Patau’s syndrome
D Klinefelter’s syndrome
E Turner’s syndrome

40 A patient who is ill with scurvy displays disturbed processes of connective tissue formation that leads
to loosening and falling of teeth. Disturbed activity of what enzyme causes these symptomps?
A Lisilhydroxylase
B Glycosiltransferase
C Elastase
D Procollagenpeptidase of N-terminal peptide
E Procollagenpeptidase of C-terminal peptide

41 During physical exercise people are less sensitive to pain. The reason for it is the activation of:
A Antinociceptive system
B Nociceptive system
C Thyroid gland functions
D Sympathoadrenal system
E Adrenal gland functions

42 A one year old child has enlarged head and belly, retarded cutting of teeth, destruction of enamel
structure. What hypovitaminosis causes these changes?
A Hypovitaminosis D
B Hypovitaminosis C
C Hypovitaminosis A
D Hypovitaminosis B 1
E Hypovitaminosis B2

43 A sample taken from the pharynx of a patient with angina was inoculated on the blood-tellurite agar.
This resulted in growth of grey, radially striated (in form of rosettes) colonies up to 4-5 mm in diameter.
Microscopically there can be seen gram-positive rods with club-shaped ends arranged in form of spread
fingers. What microorganisms are these?
A Corynebacteria diphtheriae
B Clostridium botulinum
C Diphtheroids
D Streptococci
E Streptobacilli

44 A patient with high obesity was recommended to take carnitine as a food additive for better fat
burning. What function is fulfilled by carnitine in the process of fat oxidation?
A Transport of fatty acids from the cytosol to the mitochondria
B Transport of fatty acids from the fat depots to the tissues
C Participation in one of the reactions of beta-oxidation of fatty acids
D Fatty acid activation
E Intracellular lipolysis activation

45 A 28 year old pregnant woman had the enzymes in the cells of amniotic fluid analyzed. The analysis
revealed insufficient activity of beta-glucuronidase. What pathological process is it?
A Mucopolysaccharidosis
B Glycogenosis
C Collagenosis
D Lipidosis
E Aglycogenosis

46 Laboratory of extremely dangerous infections received a sample taken from a patient with assumed
cholera. What express-diagnostics method can confirm this diagnosis?
A Immunofluorescence test
B Complement binding reaction
C Agglutination test
D Precipitation reaction
E Hemagglutination reaction

47 A patient with focal tuberculosis of superior lobe of his right lung takes isoni-azid as a part of
combined therapy. After a time he started complaining of muscular weakness, decrease of skin
sensitivity, sight and movement coordination disorder. What vitamin preparation will be right for
elimination of these occurances?
A Vitamin B6
B Vitamin A
C Vitamin D
D Vitamin B12
E Vitamin C

48 A patient with chronic hypoglycemia had adrenaline introduction. After introduction blood test has
not changed essentially. Doctor assumed liver pathology. What liver function may have been changed?
A Function of glycogen depositing
B Ketogenic function
C Function of cholesterin production
D Glycolytic function
E Excretory function

49 In order to speed up healing of the thermal injury it is required to prescribe a drug that facilitates
epithelization of skin and mucous membranes. What drug is it?
A Retinol acetate
B Tocopherol acetate
C Nicotinic acid
D Ergocalciferol
E Ascorbic acid

50 A child has abnormal formation of tooth enamel and dentin as a result of low concentration of calcium
ions in blood. Such abnormalities might be caused by deficiency of the following hormone:
A Parathormone
B Thyrocalcitonin
C Thyroxin
D Somatotropic hormone
E Triiodothyronine

51 The first grade pupils went through a medical examination aimed at selection of children needing
tuberculosis revaccination. What test was applied?
A Mantoux test
B Schick test
C Supracutaneous tularin test
D Burne test
E Anthracene test

52 Microscopical examination of discharges from the gums of a patient ill with paradontosis revealed
some protozoan pear-shaped organisms 6-13 micrometer long. The parasite has one nucleus and
undulating membrane, there are four flagella at the front of its body. What protozoan were found?
A Trichomonads
B Leishmania
C Amoebae
D Balantidia
E Lamblia

53 A 15 year old girl was delivered to the hospital with inflammation of vermiform appendix. Blood
analysis revealed signs of anaemia. Her feces contained lemon-shaped helminthic eggs (50x30
micrometer) with "plugs" on the poles. What type of helminth is it?
A Trichuris
B Hookworm
C Hymenolepis nana
D Echinococcus
E Pinworm

54 A patient has painfulness along big nerve trunks and excessive content of pyruvate in blood. What
vitamin deficit may cause such changes?
ABi
B B2
C PP
D Pantothenic acid
E Biotin

55 A patient has roundish ulcers on his face, inflammation and enlargement of lymph nodes. These
symptoms turned up as a result of mosquito bites. Laboratory examination of discharge from the ulcers
revealed unicellular aflagellar organisms. What is the most probable diagnosis?
A Dermatotropic leishmaniasis
B Toxoplasmosis
C Scabies
D Trypanosomiasis
E Myasis

56 A patient has deformation of jaw bones. Histological examination revealed there growth of
fibrocellular tumourlike ill-defined tissue with primitive osteogenesis. What disease are these
presentations typical for?
A Fibrous dysplasia
B Ameloblastoma
C Osteosarcoma
D Eosinophilic granuloma
E Parathyroid osteodystrophy

57 A patient with diabetes mellitus had an insuline injection. It caused loss of consciousness and
convulsions. What was the result of biochemic blood analysis on glucose content?
A 2,5 mmole/l
B 3,3 mmole/l
C 8,0 mmole/l
D 10 mmole/l
E 5,5 mmole/l

58 Examination of a 6 days old infant revealed phenyl pyruvate and phenyl acetate excess in his urine.
What ami-noacid metabolism is disturbed in the child’s organism?
A Phenylalanine
B Tryptophan
C Methionine
D Histidine
E Arginine

59 For assessment of the neutralizing function of liver a patient with chronic hepatitis went through a
test with natrium benzoate load. The excretion of what acid with urine will characterize the neutralizing
function of liver?
A Hippuric acid
B Phenylacetic acid
C Citric acid
D Valeric acid
E Oxalic acid

60 Cytogenetic examination of a patient with dysfunction of the reproductive system revealed normal
karyotype 46,XY in some cells, but most cells have Klinefelter’s syndrome karyotype - 47,XXY Such
phenomenon of cell inhomogeneity is called:
A Mosaicism
B Inversion
C Transposition
D Duplication
E Heterogeneity

61 Examination of an ill child’s blood revealed inherited hyperlipoproteinemia. Genetic defect of what
enzyme synthesis causes this phenomenon?
A Lipoprotein lipase
B Glycosidase
C Proteinase
D Hemsynthetase
E Phenylalanine hydroxylase

62 Dehelmintization of a patient revealed some long fragments of a helminth with segmented structure.
Mature segments were rectangular, 30x12 mm large, closed-type matrix was in form of a stem with 17-
35 lateral branches. Specify this helminth:
A Hookless tapeworm
B Alveococcus
C Echinococcus
D Dwarf tapeworm
E Armed tapeworm

63 A 42 year old woman diagnosed with diabetes mellitus was admitted the endocrinological department
with complaints of thirst, excessive appetite. What pathological components are revealed in course of
laboratory examination of the patients urine?
A Glucose, ketone bodies
B Protein, aminoacids
C Bilirubin, urobilin
D Blood
E Protein, creatine

64 Examination of a 30-year-old man mandible revealed in the region of his molar a dense tumour-like
formation that significantly deformed the mandible. The formation wasnt fully detached from the bone
tissue. Microscopical examination of a tissue sampling revealed that stroma had some cords and follicles
with odontogenous cylindric epithelial cells in peripheria and stellate cells resembling of the enamel
organ pulp in the centre. What is the most likely diagnosis?
A Ameloblastoma
B Primary intraosteal cancer
C Adenomatoid tumour
D Adenocarcinoma
E Osteoclastoma

65 Histological examination of a micro specimen presenting a malignant lung tumour revealed that the
tumor consisted of lymphocyte-like cells forming any structures. Stroma is mildely marked, there are a
lot of mitoses and necroses. What tumour is it?
A Small cell carcinoma
B Squamous cell nonkeratinous carcinoma
C Fibroma
D Squamous cell keratinous carcinoma
E Adenocarcinoma

66 A 40-year-old male patient had a tumour-like formation 8x7 cm large on his neck. A surgeon removed
it only partially because of close connection with large vessels. Microscopical examination revealed
marked cellular and tissue atypism, lipoblast-type cells in different stages of maturity, with
polymorphism and nuclear hyperchromia, pathological mitoses, necrosis foci. Specify the histological
form of the tumour:
A Liposarcoma
B Lipoma
C Fibroma
D Fibrosarcoma
E Hibernoma

67 In compliance with the clinical presentations a man was prescribed pyri-doxalphosphate. What
processes are corrected by this preparation?
A Transamination and decarboxylation of amino acids
B Oxidative decarboxilation of keto acids
C Desamination of purine nucleotides
D Synthesis of purine and pyrimidine bases
E Protein synthesis

68 A 2 year old child suffers from intestinal dysbacteriosis that lead to the development hemorrhagic
syndrome. The most probable cause of hemorrhage is:
A Vitamin K deficiency
B Activation of tissue thromboplastin
C Hypovitaminosis PP
D Fibrinogen deficiency
E Hypocalcemia

69 Decreased ratio of adenylic nucleotides ATP/ADP results in intensified glycolysis in parodentium


tissues in hypoxic conditions. What reaction is activated in this case?
A Phosphofructokinase
B Lactate dehydrogenase
C Triosophosphate isomerase
D Aldolase
E Enolase

70 Helminthological examination of patient’s feces revealed oval brown eggs with tuberous external
membrane. Name the type of helminth:
A Ascarid
B Pinworm
C Whipworm
D Dwarf tapeworm
E Broad tapeworm

71 A patient has the folowing changes: disorder of twilight vision, drying out of conjunctiva and cornea.
Such disorders may be caused by deficiency of vitamin:
A Vitamin A
B Vitamin B
C Vitamin C
D Vitamin D
E Vitamin B12

72 Examination of a 10 y.o. child revealed on the alveolar submandibular process a fixed tumourous
mass 1,5 cm in diameter closing premolar crown on the vestibular side. Mucous membrane of its surface
is reddish-brown, it bleeds as a reaction to a slight mechanical intervention. Biopsy results: the mass
consists of small size vessels separated by thin layers of connective tissue and infiltrated by plasmocytes,
mucous membrane is here and there ulcered. What is the most probable diagnosis?
A Angiomatous form of epulis
B Gingival fibromatosis
C Giant cell form of epulis
D Hypertrophic gingivitis
E Fibrous form of epulis

73 A non trained man has usually muscular hypoxy after a sprint. What metabolite accumulates in the
muscles as a result of it?
A Lactate
B Ketone bodies
C Glucose 6-phosphate
D Oxaloacetate
E-

74 A man got poisoned with mushrooms. They contain muscarine that stimulates muscarinic
cholinoreceptors. What symptom is typical for poisoning with inedible mushrooms?
A Miosis
B Mydriasis
C Bronchi dilation
D Heart rate rise
E Arterial pressure rise

75 Myocyte cytoplasm contains a big number of dissolved metabolites of glucose oxidation. Name one of
them that turns directly into lactate:
A Pyruvate
B Oxaloacetate
C Glycerophosphate
D Glucose 6-phosphate
E Fructose 6-phosphate

76 Roentgenological examination of mandible of a 27 year old man revealed a focus of osseous tissue
destruction. Histological examination revealed a tumour consisting of odontogenous epithelium cords,
immature connective tissue and dysplastic dentin rests. What tumour is it?
A Dentinoma
B Ameloblastic fibro-odontoma
C Odontoameloblastoma
D Odontogenous fibroma
E Complex odontoma

77 Patient with pigmentary xeroderma are characterized by anamalously high sensivity to ultraviolet
rays that causes skin cancer as a result of enzyme systems incapability to restore damages of hereditary
apparatus of cells. What process abnormality is this pathology connected with?
A DNA reparation
B Genetic conversion
C DNA recombination
D Genetic complementation
E DNA reduplication

78 A patient has the sudden decrease of Са2+ content in blood. What hormone secretion will increase?
A Parathormone
B Somatotropin
C Aldosterone
D Thyrocalcitonin
E Vasopressin

79 A female patient has symptoms of inflammation of urogenital tracts. A smear from the vaginal
mucous membrane contained big unicellular pyriform organisms with a sharp spike on the back end of
their bodies; big nucleus and undulating membrane. What protozoa were revealed in the smear?
A Trichomonas vaginalis
B Trichomonas hominis
C Trichomonas buccalis
D Trypanosoma gambiense
E Lamblia intestinalis

80 A patient has increased content of uric acid in his blood that is clinically presented by pain syndrome
as a result of urate deposition in the joints. What process does this acid result from?
A Lysis of purine nucleotides
B Lysis of pyrimidine nucleotides
C Heme catabolism
D Proteolysis
E Reutilization of purine bases

81 Among public catering workers examined by doctors of sanitary-and-epidemiologic station often


occur asymptomatic parasite carriers. This means that a healthy person carries cysts that infect other
people. Such parasitizing is impossible for the following causative agent:
A Dysenteric amoeba
B Malarial plasmodium
C Intestinal trichomonad
D Dermatotropic leishmania
E Viscerotropic leishmania

82 The 16th tooth of a patient is missing. X-ray picture shows in the depth of alveolar process
rarefaction of bone and a well-defined cavity that contained the underdeveloped tooth crown.
Microscopical examination revealed that the cavity wall was lined with stratified squamous epithelium
and enclosed within a fibrous capsule. Make a diagnosis:
A Follicular gnathic cyst
B Radicular gnathic cyst
C Cyst of the incisive canal
D Cystic ameloblastoma of jaw
E Primordial gnathic cyst

83 A patient has increased permeability of blood-vessel walls, increased gingival hemorrhage, small
punctate hematomas on his skin, falling of teeth. What disturbance of vitamun metabolism can account
for these symptoms?
A Hypovitaminosis C
B Hypervitaminosis D
C Hypervitaminosis C
D Hypovitaminosis D
E Hypovitaminosis A

84 A patient has liver cirrhosis. Analysis of what substance excreted with urine may characterize the
state of antitoxic liver function?
A Hippuric acid
B Ammonian salts
C Creatinine
D Uric acid
E Amino acids

85 A sportsman needs to improve his sporting results. He was recommended a drug containing
carnitine. What process is activated by this compound in the first place?
A Transport of fatty acids
B Transport of amino acids
C Transport of calcium ions
D Transport of glucose
E Transport of vitamin K

86 A child has disturbed enamel and dentine formation as a result of decreased content of calcium ions
in his blood. What hormone deficiency may cause such changes?
A Thyreocalcitonin
B Somatotropin
C Thyroxin
D Parathormone
E Triiodothyronine
MINISTRY OF PUBLIC HEALTH OF UKRAINE
NATIONAL O.O.BOGOMOLETS MEDICAL UNIVERSITY

Department of Bioorganic and Biological Chemistry

"Approved"
at the methodical council of the department
bioorganic and biological chemistry

Head of Department
d.m.s professor ___________ L.V. Gayova
"___" ______________ 2019

METHODOLOGICAL RECOMMENDATIONS
TRAINING FOR PRACTICAL LESSON

Studying discipline Bioorganic and Biological Chemistry


Module №3 Molecular biology. Biochemistry of
physiological functions.
Semantic module Fundamentals of molecular biology.
Topic:3 Hemoglobin biosynthesis. Porphyria.
Heme catabolism. Jaundice
pathochemistry
Course II
Faculty Medical
Number of hours 3 hours

Prepared by PhD Ischenko A.A.


Translated by PhD Mazur Iu.

Kyiv -2019
Hemoglobin biosynthesis. Porphyria. Heme catabolism. Jaundice pathochemistry

Theoritical questions for the class


1. Porphirin synthesis:
• substrates for heme synthesis;
• δ-aminolevulenic acid synthase role;
• uroporphirinogene III formation;
• protoporphirinogene convertion into protoporphitrin III (IX);
• heme synthase role;
• cheme of heme biosyntesis reactions.
2. Heridaty disease of porphirin metabolism:
• porphirias clinical sympoms клінічні: photosensitivity, neurological disorders;
• porphirias classification – erythropoietic porphiria (Gunther disease), hepatic
porphyrias.
3. Catabolism of hemoglobin and its prosthetic group (heme):
• formation of verdoglobin as a result of tetrapyrrole ring breakup;
• disintegration of verdoglobin to biliverdin;
• conversion of biliverdin to bilirubin
• bilirubin transport from mononuclear phagocyte (reticuloendothelial) system to
liver;
• bilirubin glucuronide formation;
• bilirubin glucuronide biotransformation in the intestines;
• recirculation of biotransformation products.
4. Jaundice pathochemistry:
• Pre-hepatic/ hemolytic jaundice;
• Hepatic/ hepatocellular jaundice;
• Post-Hepatic/ cholestatic jaundice;
• enzymatic jaundice.
5. Clinical and biochemical characteristic of jaundice by following parameters: indirect
bilirubin, direct bilirubin, bilirubin in urine, urobilinogen in urine, stercobilin in feces.
6. Biochemical parameters which used for liver disease diagnosis: ALA, AST, glutamate
dehydrogenase activity, level of direct, indirect bilirubin, functional probe.

2
Case studies

Case 1. Patient diagnosed with jaundice. Total bilirubin is 80 M/l, direct bilirubin is
normal, indirect is increased, urine bilirubin is not revealed, urobilinogen in urine is increased.
Feces has dark color, stercobilinogen is increased. What type of jaundice it is? Prove your
answer.
Etalon of answer: hemolytic jaundice.
Erythrocyte hemolysis and intensive heme degradation cause hyperbilirubinemia mainly
by indirect bilirubin. Increased bilirubin flow in intestines is a source of stercobilinogen which
secreted as stercobilin with feces (dark, almost black hypercholic feces) and as urobilin with
urine (urine intensive color) after its absorption in large intestine.

Case 2. The 48-old-patient come to hospital with compliance of general weakness,


irritability, sleep disturbance. Her skin and sclera has a yellow color. Blood parameters:
increased total bilirubin with prevalence of direct. Feces is acholic. Urine has a dark color
because of bile pigment. Which type of jaundice this patient has? Prove your answer.
Etalon of answer: cholestatic jaundice.
When bile ducts are blocked cholestatic jaundice is develop. Overwhelming bile ducts is
damaged and leak bilirubin in blood capillaries. Thus, huge quantity of direct bilirubin appears
in blood. Its excreted by urine which gives it golden-brownish color (“urine of bear color”).
Direct bilirubin as a part of bile couldn’t be transported into intestine, thus, feces does not
contain stercobilinogen (acholic feces).

Case 3. Patients with erythropoietic protoporphyria (Gunther disease) has a red-


fluorescent tooth in ultra-wave, their skin is sensitive to light, urine has red color. Which
enzyme deficiency cause this disease? Explain your answer
Etalon of answer: uroporphyrinogen III synthase .
Erythropoietic protoporphyria (Gunther disease) is a pathology which develops in case of
uroporphyrinogen III synthase disrupted synthesis. This biochemical defect cause formation
of non-physiological isomer of uroporphyrinogen I. This disease is characterized by red urine
color (in some cases bones and tooth) because of uroporphyrinogen I accumulation in kidney
which finally converts to uroporphyrin I..

Recommended literature.
Main:
1. Біологічна і біоорганічна хімія: у 2 книгах: підручник. Кн. 2. Біологічна хімія (ВНЗ
ІV р. а.) / за ред. Ю.І. Губського, І.В. Ніженковської. Вид.: ВСВ "Медицина", 2016. –
544 с.
2. Скляров О.Я. Біологічна хімія: підруч. для студентів стоматол. ф-тів вищ. мед. навч.
закл. IV рівня акредитації / О. Я. Скляров, Н. В. Фартушок, Т. І. Бондарчук.–
Тернопіль: ТДМУ: Укр медкнига, 2015. – 705 с.

3
3. Біологічна і біоорганічна хімія: у 2 кн.: підручник. Кн. 1. Біоорганічна хімія (ВНЗ ІV
р. а.) / за ред. Б.С. Зіменковського, І.В. Ніженковської. Вид.: ВСВ "Медицина", 2014.
– 272 с.
Additional:
1. Гонський Я.І., Максимчук Т.П., Калинський М.І. Біохімія людини. Підручник. –
Тернопіль: Укрмедкнига, 2013. –744 с.
2. Функціональна біохімія: навчальний посібник для студ. вищого фарм. навч. закл.
ІVрівня акредитації / А.Л.Загайко [та ін.]. – Харків: НФаУ, 2010. – 219 с.
3. Біологічна хімія з біохімічними методами дослідження: підручник для студ. вищих
мед. навч. закл. І-ІІ рівнів акредитації / О.Я. Скляров.– К.: Медицина, 2009. – 352 с.
4. Біологічна хімія : підруч. для студ. вищ. мед. навч. закл. IV рівня акредитації / Ю. І.
Губський. – Вид. 2-ге. – К.; Вінниця: НОВА КНИГА, 2009. – 664 с.

4
Hemoglobin biosynthesis. Porphyria. Heme catabolism. Jaundice pathochemistry
In “KROK-1” MCQ old terminology may appear e.g. obstructive jaundice, mechanical jaundicem
conjugated bilirubin, unconjugated bilirubin
MCQ Correct answer
1. Barbiturates prescribed for jaundice treatment because Correct answer: Direct (conjugated) bilirubin.
it induces UDP-glucuronosyltransferase synthesis. In endoplasmic reticulum of erythrocytes bilirubin is
Which substance formation provide therapeutic effect? conjugated with glucuronic acid (UDP-glucoronate)
A. *Direct (conjugated) bilirubin and form water-soluble product (bilirubin mono- or
B. Indirect (unconjugated) bilirubin diglucoronid, e.g. conjugated, direct bilirubin). This
C. Biliverdin compound is hydrophilic and has low toxicity.
D. Protoporphyrin Described process is catalyzed by UDP-
E. Heme glucuronosyltransferase. Expression of
Test question collection for licensed exam Krok-1 «General medical preparation». glucuronosyltransferase can be increased with such
– 2010. – №82.
drugs as phenobarbital.
2. The patient came to doctor with compliance about Correct answer: Gallstone
yellow sclera and skin. No encephalopathy, cholestasis When gallstone moves through gall duct, bile flow is
or acholic syndrome. Which type of jaundice is blocked, person experience acute pain attack, after
developed in this patient? some time jaundice develops which reminds
A. *Gallstone obstructive by following parameters: conjugated
B. Hemolytic jaundice bilirubin cannot with the bile move to the intestine,
C. Hepatitis thus no stercobilin is produced. Bile stasis in the bile
D. Chronic gastritis capillaries leads to hepatocytes necrosis,
E. Chronic colitis consequently some enzymes are released in blood,
including ALA
Test question collection for licensed exam Krok-1 «General medical preparation».
– 2015. – №187.; – 2016. – №183.

3. The 53-years-old man compliance about acute pain in Correct answer: Gallstone
right under rib area. Doctor noticed yellow sclera. When gallstone moves through gall duct, bile flow is
Laboratory diagnostic revealed increased ALA, negative blocked, person experience acute pain attack, after
stercobilin in feces. Which disease can be diagnosed with some time jaundice develops which reminds
these symptoms? obstructive by following parameters: conjugated
A. * Gallstone bilirubin cannot with the bile move to the intestine,
B. Hemolytic jaundice thus no stercobilin is produced. Bile stasis in the bile
C. Hepatitis capillaries leads to hepatocytes necrosis,
D. Chronic colitis consequently some enzymes are released in blood,
E. Chronic gastritis including ALA
Test question collection for licensed exam Krok-1 «General medical preparation»..
– 2013. – №116.; – 2014. – №101.

4. The born in time newborn has yellow skin and mucus Correct answer: UDP-glucosyltransferase.
color. The most probable reason of this condition is The newborn jaundice characterized by high
temporary insufissiency of the following enzyme: conjugated bilirubin concentration. This condition is
A. *UDP-glucosyltransferase cause by immature enzyme system, specially UDP-
B. Glucuronosyltransferase glucosyltransferase which catalyze conjugation of
C. Heme synthase free bilirubin with glucuronic acid.
D. Heme oxygenase
E. Biliverdin reductase
Test question collection for licensed exam Krok-1 «General medical preparation»..
– 2017. – №15.
5. The preterm newborn has a jaundice. Which enzyme Correct answer: UDP-glucosyltransferase.
is probably deficient in this baby? The newborn jaundice characterized by high
A. * UDP-glucosyltransferase conjugated bilirubin concentration. This condition is
B. alkaline phosphatase cause by immature enzyme system, specially UDP-

5
C. acidic phosphatase glucosyltransferase which catalyze conjugation of
D. catalase free bilirubin with glucuronic acid.
E. NAD+-dehydrogenase
Test question collection for licensed exam Krok-1 «General medical preparation».
– 2013. – №173.; – 2014. – №165.
6. The newborn has a physiological jaundice. The level Correct answer: UDP-glucosyltransferase.
of blood free bilirubin considerably higher than normal. Newborn jaundice characterized by considerably
What enzyme deficiency can cause this high unconjugated bilirubin. As usual the main cause
A. * UDP-glucosyltransferase is immature enzyme system, specially UDP-
B. Transaminase glucosyltransferase which catalyze bilirubin
C. Xanthine oxidase conjugation with glucuronic acid.
D. Adenosine deaminase
E. Heme oxygenase
Test question collection for licensed exam Krok-1 «General medical preparation».
– 2006. – №123.

7. The patient has a following symptoms: yellow skin, Correct answer: Free bilirubin.
dark urine, yellow-dark feces. Which substance you will Increased concentration of free (indirect,
predict to be increased in blood serum? unconjugated) bilirubin in blood cause rise in free
A. *Free bilirubin bilirubin flow to the intestine, thus, formation of
B. Conjugated bilirubin stercobilinogen which excreted with feces (black,
C. Mesobilirubin almost hypercholic feces) and its absorption in large
D. Verdoglobin intestine and urine excretion (intensive urine color).
E. Biliverdin
Test question collection for licensed exam Krok-1 «General medical preparation».
– 2008. – №21.; – 2017. – №31.
8. The 20-years-old patient has a jaundice and the Correct answer: Hemolytic.
following laboratory parameter: increase general blood Intensive erythrocyte hemolysis and heme breakup in
bilirubin, predominantly by indirect (free) fraction; urine blood plasma cause hyperbilirubinemia mostly
and feces has a high stercobilin; blood direct because of indirect (unconjugated) bilirubin fraction.
(conjugated) bilirubin is normal. Which type of jaundice High bilirubin amount in the intestine leads to
this patient is most probably develop? massive stercobilinogen production which excreted
A. *Hemolytic as stercobilin with feces (dark, almost black
B. Parenchymatous (hepatic) hyperholic feces) or absorbed in large intestines and
C. Mechanical excreted with urine (intensive urine color).
D. Newborn jaundice
E. Gilbert's syndrome
Test question collection for licensed exam Krok-1 «General medical preparation».
– 2011. – №79.; – 2012 – №65.; – 2013. – №50.; – 2015. – №29.; – 2016. – №147.;
– 2017. – №59.
9. The patient develops yellowish of skin and mucus Correct answer: Hemolytic.
membranes. Analysis of blood plasma revealed increased Intensive erythrocyte hemolysis and heme breakup in
total bilirubin, feces – increased stercobilin, urine – blood plasma cause hyperbilirubinemia mostly
increased urobilin. What type of jaundice this patient because of indirect (unconjugated) bilirubin fraction.
has? High bilirubin amount in the intestine leads to
A. *Hemolytic massive stercobilinogen production which excreted
B. Gilbert's syndrome as stercobilin with feces (dark, almost black
C. Parenchymatous hyperholic feces) or absorbed in large intestines and
D. Obstructive excreted with urine (intensive urine color).
E. Cholestatic
Test question collection for licensed exam Krok-1 «General medical preparation».
– 2009. – №141.; – 2010. – №123.
10. The patient with jaundice has increased direct Correct answer: Mechanical.
bilirubin and bile acids in blood; no stercobilinogen was Primary feature of mechanical (obstructive) jaundice
revealed in urine. What type of jaundice can be the reason is untypical yellow color of skin, mucus membranes,
of these symptoms? sclera. Bile which contains bile acids as well as direct

6
A. *Mechanical bilirubin formed from indirect in hepatocytes cannot
B. Hepatic pass to duodenum because of bile duct blockage.
C. Parenchymatous Consequently, as direct bilirubin is not able to be
D. Hemolytic transported to intestine as part of bile, feces and urine
E. Post-hepatic do not contain stercobilinogen.
Test question collection for licensed exam Krok-1 «General medical preparation».
– 2010. – №91.

11. The 20-years-old man is diagnosed with heredity Correct answer: Indirect (unconjugated)
deficiency of UDP-glucuronosyltransferase. Change in bilirubin.
which blood parameter can prove this diagnosis? In normal conditions UDP-glucuronosyltransferase
A. Indirect (unconjugated) bilirubin catalyze transfer of glucuronic acid to bilirubin with
B. Direct (conjugated) bilirubin formation of conjugated bilirubin. In case of
C. Urobilin disrupted synthesis of this enzyme conjugation
D. Stercobilinogen process is impossible which cause accumulation of
E. Animal indican unconjugated (indirect) bilirubin in blood
Test question collection for licensed exam Krok-1 «General medical preparation».
– 2016. – №67.
12. The 28-years-old woman came to infectious Correct answer: Parenchymatous jaundice.
department with yellow skin, sclera, mucus membrane. In case of parenchymatous jaundice hepatocytes and
Laboratory diagnosis revealed increased blood direct bile ducts are damaged which enable liver to
bilirubin; urobilinogen and bilirubin in urine. Which of transport conjugated bilirubin from hepatocytes to
following condition can lead to those symptoms? bile against concentration gradient. Formed
A. * Parenchymatous jaundice conjugated bilirubin partly get into big blood circle,
B. Hemolytic jaundice while its concentration in the intestines considerably
C. Kidney infarction decreased. Mesobilinogen quantity decline, thus
D. Renal tuberculosis feces in parenchymatous jaundice are hypocholyc
E. Mechanical jaundice (almost colorless) and urine has intensive color
Test question collection for licensed exam Krok-1 «General medical preparation».
– 2012. – №137.
because of mesobilinogen (urobilinogen) and
conjugated hemoglobin.
13. The 48-years-old woman was delivered to hospital Correct answer: Mechanical.
with compliance of general weakness, sleep disturbance. In case of bile duct stasis and obstruction of bile
Objectively: skin and sclera are yellow. Blood analysis outflow jaundice develops. The bile capillaries are
has increased total bilirubin with direct bilirubin distended by the back pressure of bile; they rupture
predominance. Feces is acholic. Urine has dark color, and the bilirubin escapes into the blood capillaries.
because of bile pigments. Which type of jaundice this Huge amount of direct (conjugated) bilirubin appears
patient has? in blood, which excreted with urine and coloring it in
A. *Mechanical golden-brown color (“urine of bear color”).
B. Hemolytic
C. Parenchymatous
D. Gilbert's syndrome
E. Crigler–Najjar syndrome
Test question collection for licensed exam Krok-1 «General medical preparation».
– 2007. – №59.; – 2009. – №74.
14. The patient was delivered to hospital with complaints Correct answer: Mechanical.
about general weakness and sleep disturbance. Skin has Primary feature of mechanical (obstructive) jaundice
yellow color. There is increased blood direct bilirubin is untypical yellow color of skin, mucus membranes,
and bile acids. Feces is acholic. Which type of disease sclera. Bile, which contains bile acids as well as
can provoke those changes? direct bilirubin formed from indirect in hepatocytes,
A. *Mechanical jaundice cannot pass to duodenum because of bile duct
B. Hemolytic jaundice blockage. Consequently, as direct bilirubin is not
C. Pre-hepatic jaundice able to be transported to intestine as part of bile, feces
D. Gilbert's syndrome and urine do not contain stercobilinogen.
E. Chronic cholecestitis

7
Test question collection for licensed exam Krok-1 «General medical preparation».
– 2011. – №172.
15. After blood transfusion patient has yellowish of skin Correct answer: Hemolytic jaundice.
and mucus membrane, increased total and indirect Incompatible blood transfusion (by blood type or
bilirubin, increased urine urobilin and feces stercobilin. Rhesus factor) provoke intensive erythrocyte
What kind of jaundice can be the reason? hemolysis, consequently heme degradation and
A. *Hemolytic jaundice. increased formation of indirect bilirubin. Liver is
B. Newborn jaundice. enable to conjugate it all, thus, blood characterized
C. Obstructive jaundice. by increased total bilirubin (mainly by indirect).
D. Parenchymatous jaundice. Increased bilirubin in intestine leads to intensified
E. Genetic jaundice. stercobilinogen synthesis which excreted as
Test question collection for licensed exam Krok-1 «General medical preparation». stercobilin in feces (dark, almost black hypercholic
– 2005. – № 121.
feces) and as urobilin in urine (intensive urine color).
16. The patient who suffers from jaundice because of Correct answer: Increased erythrocyte hemolysis.
increased bilirubin as part of its indirect fraction. Urine Incompatible blood transfusion (by blood type or
and feces have intensive color. What is the most probable Rhesus factor) provoke intensive erythrocyte
explanation to these disruptions? hemolysis, consequently heme degradation and
A. *Increased erythrocyte hemolysis increased formation of indirect bilirubin. Liver is
B. Disrupted liver urobilinogen transformation. enable to conjugate it all, thus, blood characterized
C. Disruption of direct bilirubin formation. by increased total bilirubin (mainly by indirect).
D. Liver parenchyma damage. Increased bilirubin in intestine leads to intensified
E. Disruption of bile flow from liver stercobilinogen synthesis which excreted as
Test question collection for licensed exam Krok-1 «General medical preparation». stercobilin in feces (dark, almost black hypercholic
– 2010. – №142.
feces) and as urobilin in urine (intensive urine color).
17. The 43-years-old patient during 10 years has a Correct answer: Heme.
repetitive compliance about acute bellyache, cramps, Genetic diseases of heme biosynthesis (enzymes
vision problems. His relatives have Patient urine have red deficiency) leads to accumulation of interment
color. He was diagnosed with acute intermittent products of synthesis – porphyrins. This cause acute
porphyria. The reason of he his disease is not being a bellyache, vision problems, urine gets dark-red color.
vampire but disrupted biosynthesis of:
A. *Heme
B. Bile acids
C. Insulin
D. Collagen
E. Prostaglandin
Test question collection for licensed exam Krok-1 «General medical preparation».
–2011. – № 58.
18. The patients with erythropoietic porphyria (Gunther Correct answer: Uroporphyrinogen III synthase.
disease) have ultraviolet red-fluorescent tooth, sun- Erythropoietic porphyria (Gunther disease) is a
sensitive skin, red-colored urine. Which enzyme pathology of disrupted uroporphyrinogen III
deficiency cause this disease? synthase synthesis. This biochemical defect leads to
A. * Uroporphyrinogen III synthase formation of non-physiological isomer of
B. - aminolevulinic acid synthase uroporphyrinogen I. This substance is accumulated
C. Uroporphyrinogen I synthase in kidneys where converted to uroporphyrin I and
D. Uroporphyrinogen decarboxylase colored urine in red (in some cases it also colors teeth
E. Ferrochelatase and bones).
Test question collection for licensed exam Krok-1 «General medical preparation».
– 2010. – № 182.
19. The patient who suffers from anemia has increased Correct answer: Iron.
protoporphyrin IX. Which mineral deficiency can lead to During heme biosynthesis heme synthetase attaches
such pathology? iron to protoporphyrin IX and form heme which is a
A. *Iron part of hemoglobin. Disruption of its synthesis cause
B. Potassium anemia.
C. Magnesium
8
D. Sodium
E. Phosphorus
Test question collection for licensed exam Krok-1 «General medical preparation».
–2007. – № 117
20. The patient has increased photosensitivity, after Correct answer: Porphyria.
prolong sun exposure urine starts to get dark-red color. Genetic diseases of heme biosynthesis (enzymes
Which type of disease has this characteristic? deficiency) leads to accumulation of interment
A. *Porphyria products of synthesis – porphyrins. This cause
B. Alkaptonuria increased photosensitivity, urine gets dark-ted color.
C. Albinism
D. Hemolytic jaundice
E. Pellagra
Test question collection for licensed exam Krok-1 «General medical preparation».
– 2009. – № 43.

9
MINISTRY OF HEALTHCARE OF UKRAINE
O.O. BOHOMOLETS NATIONAL MEDICAL UNIVERSITY

Department of bioorganic and biological chemistry

“Approved”
on the methodical meeting of the bioorganic and
biological chemistry department

Head of Department
M.D. Professor of ___________L.V. Gayova
„___”______________2019 р.

GUIDLINE FOR PRACTICAL CLASSES

Course unit Biological and bioorganic chemistry


Modul 3 3

Content module.

Subject: Investigation of xenobiotics and endogenous toxins


biotransformation processes. Microsomal oxidation,
cytochrome P 450
Course 2
Faculty
Number of hours 3 hours

Kyiv-2019
Investigation of xenobiotics and endogenous toxins biotransformation processes.
Microsomal oxidation, cytochrome P-450

Theoretical questions:

1. Detoxicational function of the liver: biotransformation of xenobiotics and endogenous toxins.


2. Basic stages of biotransformation of foreign chemical compounds in the liver:
• first stage - oxidation-reduction and hydrolytic reactions;
• second stage - synthesis reactions, or conjugation.
3. Microsomal oxidation reactions:
• microsomal oxygenase (monooxygenase);
• Electron transport chains of the endoplasmic reticulum membranes;
• cytochrome P 450;
• catalytic cycle of the cytochrome P 450 functioning.
4. Genetic polymorphism and inducibility of biosynthesis of cytochrome P 450.
5. The origin and nature of the tolerance to medicinal products development.
6. Reaction of conjugation in hepatocytes:
• glucuronidation reactions involving UDF-glucuronic acid;
• sulfation reaction involving 3'-phospho-adenosine-5'-phosphosulfate;
• methylation reaction involving S-adenosylmethionine;
• acetylation reaction involving acetyl-S-CoA;
• conjugation reactions with glycine.
7. Examples of endogenous toxins detoxification reactions in the liver:
• neutralization of bilirubin - the product of heme decomposition;
• neutralizing the scotol and indole products of the transformation of tryptophan.

Recommended literature.

Basic:

1. Richard A. Harvey, Denise R. Ferrier, / Lippincott’s Illustrated Reviews: Biochemistry 5-th


edition // Lippincott Williams & Wilkins, Walters Kluwer business. - 2011. - 522 P.
2. Robert K. Murray, Daryl K. Granner, Peter A. Mayes e.a. / Harper’s Illustrated Biochemistry,
Twenty-Sixth Edition // McGraw-Hill Companies, Inc. - 2003. - 702 P.
3. Voet D. Biochemistry / D. Voet, J. Voet. – Wiley, 2004. – 1514 p.
Additional:
1. Lehninger Principles of Biochemistry 6th Edition // by David L. Nelson, Michael M. Cox –
2012. – 1340 p.

2. Jan Koolman, Klaus-Heinrich Roehm / Color Atlas of Biochemistry Second edition, revised
and enlarged // ThiemeStuttgart New York. - 2005. - 467 P.
Investigation of xenobiotics and endogenous toxins biotransformation processes.
Microsomal oxidation, cytochrome P 450
Test questions Answers
1. Detoxification of xenobiotics (drugs, epoxides, the Correct answer: Glutathione.
arena oxides, aldehydes, nitro derivatives, etc.) and Biotransformation of xenobiotics and endogenous
endogenous metabolites (estradiol, prostaglandins, toxic compounds occurs in the liver. This process
leukotrienes) takes place in the liver by conjugation consists of two stages - microsomal oxidation and
with: conjugation reactions, the consistent functioning
A. *Glutathione of which ensures the elimination of a number of
B. Aspartic acid foreign substances of all chemical classes.
C. Glycine The neutralization of various xenobiotic structures
D. S-Adenozylmethionin (medicines, epoxides, aroxides, aldehydes, nitro-
E. Fosfoadenozyn trophic, etc.) and endogenous metabolites
(estradiol, prostaglandins, leukotrienes) takes
place in the liver through their conjugation with
glutathione.
This process involves a number of steps. Initially,
glutathione transferase catalyzes the interaction of
the substrate (RX) with reduced glutathione.
Glutathione conjugates are sequentially secreted
glutamic acid and glycine. The generated
conjugates of xenobiotics with cysteine may be
excreted in the urine or converted into
mercaptanic acids, which are also excreted in the
urine, in the acetylation reaction.

2. In the 70's, scientists found that the cause of severe Correct answer: Glucuronic acid.
jaundice of newborns is a disturbance of the binding Jaundicinous neonates are characterized by
of bilirubin in hepatocytes. What substance is used to excessive concentration of unconjugated bilirubin
form a conjugate? in the blood. The reason for this is usually
A. * Glucuronic acid immaturity of enzyme systems, including
B. Uric acid insufficient function UDF-glucuronyltransferase,
C. Sulfuric acid which provides conjugation of indirect bilirubin
D. Lactic acid with glucuronic acid.
E. Pyruvic acid
3. Patient has cirrhosis of the liver. The study of Correct answer: Hippuric acid.
which of the listed substances that are excreted in the Hyppuric acid (benzoylglycine) is formed due to
urine can characterize the state of the antitoxic interaction in the liver (and partly in the kidneys)
function of the liver? of benzoic acid with glycine. In clinical practice,
A. * Hippuric acid for the purpose of studying the detoxifying
B. Ammonium salts function of the liver, a Quicks’ test is performed:
C. Creatinine the amount of hippuric acid that is excreted in the
D. Uric acid urine after 4 hours after per os administration of
E. Amino acids the standard dose of sodium benzoate under the
normal conditions should be 2.0 - 2.5 g. Reduction
of this amount indicates a disturbance of the liver
detoxification function.

4. For a patient suffering from chronic hepatitis, a Correct answer: Hippuric acid.
load of sodium benzoate was carried out to evaluate Hyppuric acid (benzoylglycine) is formed due to
the degreasing function of the liver. For the discharge interaction in the liver (and partly in the kidneys)
of what substances with urine judge the neutralizing of benzoic acid with glycine. In clinical practice,
function of the stomach? for the purpose of studying the detoxifying
A. * Hippuric acid function of the liver, a Quicks’ test is performed:
B. Phenylacetic acid the amount of hippuric acid that is excreted in the
C. Citric acid urine after 4 hours after per os administration of
D. Valerian acid the standard dose of sodium benzoate under the
E. Oxalic acid normal conditions should be 2.0 - 2.5 g. Reduction
of this amount indicates a disturbance of the liver
detoxification function.

5. To determine the antitoxic function of the liver, the Correct answer: Glycine.
patient is assigned sodium benzonate, which in the Hippuric acid (benzoylglycine) is formed due to
liver turns into hippuric acid. What compound is used interaction in the liver and in the kidneys of
for this? benzoic acid with glycine.
A. * Glycine
B. Methionine
C. Cysteine
D. FAFS
E. UDF - glucuronic acid

6. In a patient with chronic alcoholic liver disease, Correct answer: Cytochrome P-450.
processes of xenobiotics and endogenous toxic Biotransformation of xenobiotics and endogenous
compounds biotransformation are disturbed. toxic compounds occurs in the liver and consists
Reducing the activity of which chromoprotein can be of two stages - microsomal oxidation and
the cause of this? conjugation reactions, the consistent functioning
A. * Cytochrome P-450 of which provides for the elimination of a number
B. Hemoglobin of foreign substances of all chemical classes. The
C. Cytochrome Oxidase main component of the microsomal system is the
D. Cytochrome b chromoprotein cytochrome P-450, which as a
E. Cytochrome c1 prosthetic group contains heme and has binding
sites for oxygen and substrate.

7. A patient suffering from chronic hepatitis Correct answer: Detoxification.


complains of increasing the sensitivity to In normal conditions, barbiturates undergo
barbiturates, which were earlier tolerated without biochemical modification in the liver
symptoms of intoxication. With the disturbance of (hydroxylation in reactions of microsomal
which of the liquor functions, this is connected at the oxidation and subsequent glucuronidation in the
highest possible level? conjugation reaction), resulting in the formation of
A. * Detoxification polar, water soluble metabolites (glucuronide
B. Hemopoietic barbiturates) which are easily derived from the
C. Hemodynamic body in urine. In chronic hepatitis the
D. Formation of bile detoxification function of hepatocytes decreases,
E. Phagocytic as a result of which the toxicity of xenobiotics
(including barbiturates) increases, causing
poisoning (intoxication).

8. The universal biological system of oxidation of Correct answer: Cytochrome P450.


nonpolar compounds (drugs, toxic compounds), The main component of the microsomal system is
steroid hormones, cholesterol is microsomal the Cytochrome P-450. It belongs to
oxidation. What is the name of the cytochrome, hemoproteins, as a prosthetic group contains hem
which is part of the oxygenase chain of the and has ligands for oxygen and substrate
microsome. (xenobiotics, steroids, fatty acids, etc.).
A. * Cytochrome P 450.
B. Cytochrome a3.
C. Cytochrome in.
D. Cytochrome c.
E. Cytochrome a.
9. Excretion of animal indican in the urine is Correct answer: Tryptophan.
observed at decay products of detoxification of the Free indole is formed in the colon as a result of the
certain amino acid in the colon. Name this amino action of enzymes of microorganisms on L-
acid. tryptophan, then by microsomal oxidation (phase I
A. * Tryptophan. of biotransformation) in the liver turns into indoxil
B. Valin. and gose to the conjugation reacts with FAFS
C. Glycine. (Phase II of biotransformation). The indoxil
D. Serin. sulfate formed during the reaction is excreted in
E. Cysteine the form of potassium salt in urine, which was
named "animal indican".

10. Study of conversion of a food colouring agent Correct answer: Cytochrome P-450.
revealed that neutralization of this xenobiotic takes Biotransformation of xenobiotics and endogenous
place only in one phase - microsomal oxydation. toxic compounds occurs in the liver and consists
Name a component of this phase: of two stages - microsomal oxidation and
A. *Cytochrome Р-450 conjugation reactions, the consistent functioning
B. Cytochrome B of which provides for the elimination of a number
C. Cytochrome C of foreign substances of all chemical classes. The
D. Cytochrome A main component of the microsomal system is the
E. Cytochrome oxidase chromoprotein cytochrome P-450, which as a
prosthetic group contains heme and has binding
sites for oxygen and substrate.

11. Test Kvika was administered for Male 32 years Correct answer: Hippuric acid.
with hepatic impairment to understand detoxication Hyppuric acid (benzoylglycine) is formed due to
ability. Results of urine show low levels of: interaction in the liver (and partly in the kidneys)
A. *Hippuric acid of benzoic acid with glycine. In clinical practice,
B. Hydroxyproline for the purpose of studying the detoxifying
C. Sodium benzoate function of the liver, a Quicks’ test is performed:
D. Creatinine the amount of hippuric acid that is excreted in the
E. Amino acids urine after 4 hours after per os administration of
the standard dose of sodium benzoate under the
normal conditions should be 2.0 - 2.5 g. Reduction
of this amount indicates a disturbance of the liver
detoxification function.
.
Investigation of xenobiotics and endogenous toxins biotransformation processes.
Microsomal oxidation, cytochrome P 450

1. Detoxification of xenobiotics (drugs, epoxides, the arena oxides, aldehydes, nitro derivatives,
etc.) and endogenous metabolites (estradiol, prostaglandins, leukotrienes) takes place in the liver
by conjugation with:
A. *Glutathione
B. Aspartic acid
C. Glycine
D. S-Adenozylmethionin
E. Fosfoadenozyn
F.
2. In the 70's, scientists found that the cause of severe jaundice of newborns is a disturbance of
the binding of bilirubin in hepatocytes. What substance is used to form a conjugate?
A. * Glucuronic acid
B. Uric acid
C. Sulfuric acid
D. Lactic acid
E. Pyruvic acid

3. Patient has cirrhosis of the liver. The study of which of the listed substances that are excreted
in the urine can characterize the state of the antitoxic function of the liver?
A. * Hippuric acid
B. Ammonium salts
C. Creatinine
D. Uric acid
E. Amino acids

4. For a patient suffering from chronic hepatitis, a load of sodium benzoate was carried out to
evaluate the degreasing function of the liver. For the discharge of what substances with urine
judge the neutralizing function of the stomach?
A. * Hippuric acid
B. Phenylacetic acid
C. Citric acid
D. Valerian acid
E. Oxalic acid

5. To determine the antitoxic function of the liver, the patient is assigned sodium benzonate,
which in the liver turns into hippuric acid. What compound is used for this?
A. * Glycine
B. Methionine
C. Cysteine
D. FAFS
E. UDF - glucuronic acid

6. In a patient with chronic alcoholic liver disease, processes of xenobiotics and endogenous
toxic compounds biotransformation are disturbed. Reducing the activity of which chromoprotein
can be the cause of this?
A. * Cytochrome P-450
B. Hemoglobin
C. Cytochrome Oxidase
D. Cytochrome b
E. Cytochrome c1

7. A patient suffering from chronic hepatitis complains of increasing the sensitivity to


barbiturates, which were earlier tolerated without symptoms of intoxication. With the
disturbance of which of the liquor functions, this is connected at the highest possible level?
A. * Detoxification
B. Hemopoietic
C. Hemodynamic
D. Formation of bile
E. Phagocytic

8. The universal biological system of oxidation of nonpolar compounds (drugs, toxic


compounds), steroid hormones, cholesterol is microsomal oxidation. What is the name of the
cytochrome, which is part of the oxygenase chain of the microsome.
A. * Cytochrome P 450.
B. Cytochrome a3.
C. Cytochrome in.
D. Cytochrome c.
E. Cytochrome a.

9. Excretion of animal indican in the urine is observed at decay products of detoxification of the
certain amino acid in the colon. Name this amino acid.
A. * Tryptophan.
B. Valin.
C. Glycine.
D. Serin.
E. Cysteine

10. Study of conversion of a food colouring agent revealed that neutralization of this xenobiotic
takes place only in one phase - microsomal oxydation. Name a component of this phase:
A. *Cytochrome Р-450
B. Cytochrome B
C. Cytochrome C
D. Cytochrome A
E. Cytochrome oxidase

11. Test Kvika was administered for Male 32 years with hepatic impairment to understand
detoxication ability. Results of urine show low levels of:
A. *Hippuric acid
B. Hydroxyproline
C. Sodium benzoate
D. Creatinine
E. Amino acids
MODULE 1
General Pharmacology

1. A patient with angina pectoris was helping himself with


nitroglycerin. One day the patient used nitroglycerin several times because of
frequent episodes of pain. So as the last sublingual administration of
nitroglycerin did not relieve pain, the patient addressed his doctor. Doctor
advised him to withdraw nitroglycerin for 8 hours and prescribed another drug.
After 8 hours the effect of nitroglycerin was restored. Which phenomenon did
take place in this case?
A. Increase in nitroglycerin metabolism
B. Increase in protein binding
C. Decrease of absorption
D. Drug dependence
E. Tolerance*.

2. Metabolism can render pharmacological activity to some initially


inert substances (pro-drug). Point out the substance, which is pro-drug, used for
treatment of hypertension.
A. Levodopa*
B. Salbutamol
C. Verapamil
D. Reserpine
E. Propranolol.

3. A patient who has been treated with diazepam on account of


neurosis complains of toothache. Doctor administered him an analgetic, but its
dose was lower than average therapeutic dose. What phenomenon did the doctor
take into account while prescribing the patient an underdose?
A. Summation
B. Potentiation*
C. Cumulation
D. Tolerance
E. Drug dependence.

4. A patient with frequent attacks of stenocardia was prescribed


sustak-forte to be taken one tablet twice a day. At first the effect was positive
but on the second day stenocardia attacks resumed. What can explain
inefficiency of the prescribed drug?
A. Tachyphylaxis*
B. Sensibilization
C. Dependence
D. Cumulation
E. Idiosyncrasy.

5. A man who has been taking a drug for a long time cannot withhold
it because this causes impairment of psychic, somatic and vegetative functions.
Name the syndrome of different disturbances caused by drug discontinuation:
A. Idiosyncrasy
B. Sensibilization
C. Tachyphylaxis
D. Abstinence*
E. Cumulation.

6. What is the mechanism of absorption of drug which is lipophilic


and unionized?
A. Active transport
B. Passive diffusion*
C. Filtration
D. Pinocytosis
E. Binding to protein.

7. Which route of administration results in 100% bioavailability of


drug?
A. Oral
B. Subcutaneous
C. Inhalational
D. Intravenous*
E. Intramuscular.

8. Chloramphenicol is highly toxic in newborn. What is the cause of


such toxicity?
A. Intensive binding to protein
B. Deficiency of metabolizing enzymes in liver*
C. Penetration into the cell
D. Inhibition of protein synthesis
E. Inhibition of nucleic acid synthesis.

9. What is the result of tubular reabsorption of drug?


A. Increase in duration of action*
B. Increase in excretion
C. Increase in distribution
D. Decrease in duration of action
E. Loss of pharmacologic activity.

10. Anticoagulant drug warfarin is bounding to plasma albumin. How


its effect can be changed in case of hypoalbuminemia?
A. Effect unchanged
B. Increased*
C. Decrease
D. Significant delayed onset of effect
E. Abolishing of effect .

11. Passive reabsorption from tubular urine (pH of 5) of a drug


administered IV would be reduced if the drug were
A. A weak base*
B. A weak acid
C. Water soluble
D. Protein bound
E. None of the above.

12. Patient suffering from angina pectoris was helping himself by


sublingual usage of nitroglycerin. One day he used the drug several times. The
last use did not relieve his pain. What phenomenon does occur?
A. Antagonism
B. Tachyphylaxis*
C. Hypersensitivity
D. Cumulation
E. Binding to protein.

13. Point out the kind of drug interaction that permits to use one of
them in lower dose.
A. Antagonism
B. Cumulation
C. Potentiation*
D. Tolerance
E. Additive synergism.

14. Patient with infection of GIT was treated with antibiotic. Which
type of therapy is it?
A. Symptomatic
B. Pathogenetic
C. Ethiotropic*
D. Replacement
E. Stimulating.

15. Alongside with therapeutic effect drugs can cause side effects. In
which dose they can occur?
A. In overdose
B. In therapeutic dose*
C. In threshold dose
D. In subthreshold dose
E. At any dose.

16. Patient suffering from gastric peptic ulcer was treated with M-
cholinoblocker. What is the mechanism of this drug action?
A. Enzymatic
B. Receptor*
C. Direct chemical
D. Physicochemical
E. Action on ion channels of membrane.
17. A patient with chronic cardiac insufficiency was prescribed digoxin
in average therapeutic dose. After 2 weeks of treatment the symptoms of
intoxication (bradycardia, extrasystole, nausea) were developed. Point out the
phenomenon responsible for intoxication.
A. Tachyphilaxis
B. Material cumulation*
C. Idiosyncrasy
D. Functional cumulation
E. Tolerance.

18. The patient with heart failure was using digitoxin. His condition
initially improved, but with time the manifestations of heart failure have
increased again, and symptoms of glycoside intoxication apeared. What is the
cause of patient's condition worsening?
A. Potentiation
B. Cumulation*
C. Tachyphylaxis
D. Sensitization
E. Idiosyncrazy.

19. Analgesic effect of methamizole(analgin) in the dental pain


develops in 0.5 hour and lasts for 1.5-2 hours. Tablets "Piraminal" in the same
situation are faster and longer acting. What is the phenomenon responsible for
difference in the action of these drugs?
A. Chemical antagonism
B. Functional antagonism
C. Undirectional antagonism
D. Potentiation
E. Summation*.

20. In the elderly patient with symptoms of senile atonic constipation


long-term treatment with drug buckthorn has led to a weaking effect. What is
this phenomenon?
A. Tolerance*
B. Cumulation
C. Sensibilization
D. Desensitization
E. Tachyphylaxis.

21. During the barbituric anesthesia in a patient respiratory depression


occurred. After intravenous injection of 10 ml 0.5% solution of bemegride the
patient's condition improved, the volume of pulmonary ventilation inceased.
Point out the phenomenon responsible for curative effect of bemegride in this
case
A. Direct synergisnm
B. Undirectional antagonism
C. Direct antagonism*
D. Indirect antagonism
E. Indirect synergism.

22. A child of 2 years admitted to the infectious deseases hospital with


a diagnosis of bacillary dysentery. What a way of enteral administration of
antimicrobial agent polymyxin is the most rational, if child has uncontrollable
vomiting?
A. Subcutaneous
B. Intravenous
C. Oral
D. Rectal*
E. Inhaled.

23. In a patient with traumatic brain injury there are such symptoms:
the obscured consciousness, inpaired respiration, filiform pulse, absence of
reflexes. Which route of drug administration for urgent aid is the most
appropriate in this case?
A. Rectal
B. Intravenous*
C. Subcutaneous
D. Oral
E. Inhaled.

24. In poisoning with mushrooms containing muscarine atropine is used


as an antidote. Which of the following mechanisms, typical for atropine, leads to
the neutralization of the poison mushroom - muscarine?
A. Antimetabolic
B. Physical and chemical
C. Antienzymatic
D. Receptor*
E. Enzymatic.

25. Prolonged use of certain drugs before pregnancy can disrupt fetal
development. What is the name of this action?
A. Fetotoxic
B. Embriotoxic
C. Teratogenic
D. Mutagenic*
E. Carcinogenic.

26. With long-term treatment by antipsychotic drugs at therapeutic


doses in patient, along with the restoration of the psychic function, some
undesired effects such as tension in the skeletal muscles, stiffness of gait, and
some autonomic disorders occurred. What the type of drug’s effects should
include these?
A. Aftereffect
B. Overdose
C. The main therapeutic effect
D. Intoxication
E. Side effects*.

27. After clinical and laboratory examination of a patient with diabetes


mellitus insulin was appointed for treatment.What is the type of this therapy?
A. Etiotropic
B. Replacement*
C. Pathogenetic
D. Symptomatic
E. Functional.

28. In order to reduce toothache a patient was recommended to take


methamisol (analgin). What the type of pharmacotherapy was used?
A. Functional
B. Pathogenetic
C. Causal
D. Substitution
E. Symptomatic*.

29. In 18 years old patient with shoulder phlegmon intramuscular


injection of penicillin was made, after which he developed tachycardia, filiform
pulse, fall of blood pressure to 80/60 mm Hg. What type of pathological
response was originated?
A. Potentiation
B. Anaphylaxis*
C. Tolerance
D. Antagonism
E. Tachyphylaxis.
Local anesthetics

30. To lance abscess doctor performed local anesthesia with therapeutic


dose of anesthetic, but it was not sufficient. What was the cause of decrease in
anesthetic effect?
A. Quick absorption of drug
B. Quick metabolism of anesthetic
C. Alkaline medium
D. Acidic pH of tissues*
E. Neutral medium.

31. Surgeon is going to perform the operation under local anaesthesia.


The probable duration of operation is more than 2 hours. In past patient was
complaining for ventricular extrasystole. Which anaesthetic is reasonable to
chose for local anaesthesia?
A. Tetracaine(decaine)
B. Procaine(novocaine)
C. Benzocaine(anaesthesine)
D. Trimecaine*
E. Cocaine.

32. Dentist revealed that his patient is suffering from hypersensitivity to


procaine. Which of the following agents may be used for local anaesthesia?
A. Benzocaine(anaesthesine)
B. Piromecaine
C. Dicaine(tetracaine)
D. Lidocaine*
E. Cocaine.

33. Which of the listed drugs is usually added to local anaesthetic to


prolong its action and to decrease toxicity?
A. Noradrenaline
B. Ephedrine
C. Adrenaline*
D. Reserpine
E. Atropine.

34. A 25-year-old woman with red and itchy eczematoid dermatitis


visits your office, She had a dental procedure one day earlier with administration
of a local anesthetic. There were no other findings, although she indicated that
she had a history of allergic reactions. Which of the following drugs is most
likely involved?
A. Procaine*
B. Etidocaine
C. Bupivacaine
D. Cocaine
E. Lidocaine.

35. For the patient with gastroenteritis doctor prescribed preparation of


tannin derivative with anti-inflammatory effect on the basis of the astringent
mechanism. Which of the following drugs was prescribed?
A. Bismuth subnitrat
B. Tannin
C. Xeroform
D. Tanalbin*
E. De-nol.

36. Patient 75 years old suffers from chronic constipation. What the
drug of a group of emollients can be assigned to this patient?
A. Senna leaf*
B. Phenolphtalein
C. Neostigmine
D. Magnesium sulfate (enterally)
E. Camphor spirit.

37. Bed sores appeared in patient on strict bed regiment. What the
irritating drug was prescribed by physician to improve blood circulation in the
area of bed sores
A. Camphor spirit*
B. Vishnevsky’s ointment
C. Wilkinson's ointment
D. A solution of potassium permanganate
E. Talc.

38. A child with acute poisoning with datura seeds doctor prescribed
oral usage of activated charcoal. Specify the mechanism of its action.
A. Adsorptions alkaloids on the surface*
B. Insoluble salts formation
C. Block of sensory receptors of the stomach
D. Precipitation of proteins on the surface of gastric mucous
membrane
E. Formation of protective layer of the colloid on the mucous
membranes.

39. The drug from the group covering agents with antacid effect was
prescribed to patients with gastric ulcer. Which of the following medications
was prescribed?
A. Tannin
B. Decoction of oak bark
C. Almagel*
D. Infusion of sage leaves
E. Bismuth subnitrate.
40. Athlete has a sharp muscular pain in the back after training. What
the irritating drug is helpful in this case?
A. The turpentine oil*
B. A solution of menthol
C. Ammonia
D. Validol
E. Chloroform.

41. Gastric lavage with a solution of tannin was performed to the


patient with alkaloid poisoning. What the concentration of tannin solution
should be used for this purpose?
A. 0.5%*
B. 0.1%
C. 0,01
D. 0,05
E. 0,1.

42. The patient suffers from hemorrhoids. For pain relief doctor
prescribed rectal suppositories with local anesthetic. Point out anesthetic used in
this dosage form.
A. Cocaine
B. Tetracaine
C. Novocaine
D. Trimekaine
E. Benzocaine (Anestesin)*.

43. Doctor applied paste with benzocaine to reduce the itching of the
forearm skin. Due to which of the following properties of benzocaine it is used
in soft dosage form?
A. Good absorption
B. Poor solubility*
C. The high degree of distribution
D. The good solubility
E. Inadequate absorption.

44. The patient will have surgery on the limbs of more than 2 hours
duration, which anesthetist plans to hold under local anesthesia. Which of the
following drugs should be selected for long-term conduction anesthesia?
A. Novocaine
B. Cocaine
C. Tetracaine
D. Benzocaine
E. Lidocaine*.

Cholinomimetics
45. Patient with complaints of dryness in the mouth, photophobia and
disturbances of vision was admitted to the reception-room. Skin is hyperemic
and dry, pupils are dilated, tachycardia. Poisoning with belladonna alkaloids was
diagnosed on further examination. What medicine should be prescribed as
antidote?
A. Diazepam
B. Prozerin (Neostigmine)*
C. Atropine
D. Pilocarpine
E. Dipyroxim.

46. Patient with pin-point pupils, sweating, salivation, abdominal


cramps, disturbances in respiration was admitted to the emergency room.
Poisoning with what substance is the most likely occurred?
A. Non-narcotic analgesics
B. Sedatives
C. Narcotic analgesics
D. М-cholinergic antagonists
E. Muscarine*.

47. In the patient, after the stroke, paralysis of hand and leg appeared.
To restore the movements of paralyzed extremities patient was treated with
physical exercises and cholinomimetics. Which of listed drugs was used for this
purpose?
A. Aceclidine
B. Pilocarpine
C. Physostigmine
D. Ambenonium chloride (oxazyl)*
E. Lobeline.

48. A known patient of myasthenia gravis came to you with his disease
well controlled by neostigmine; but he asked you to give him drug that needs
more rare usage. Choose the necessary drug.
A. Physostigmine
B. Atropine
C. Ambenonium chloride*
D. Armin
E. Pralidoxime.

49. A49-year old frequent business traveler presents to his physician


requesting medication to help him with nausea and dizziness that he gets during
turbulent flights. A scolopamine patch is prescribed for his motion sickness.
Which of the following is the most likely side effects from this patch?
A. Bradycardia
B. Diaphoresis
C. Diarrhea
D. Salivation
E. Urinary retention*.

50. In complex treatment of a child, suffering from cerebral palsy, a


doctor decided to include anticholinesterase drug moderately improving mental
development. Choose this drug.
A. Physostigmine
B. Neostigmine (Proserinum)
C. Galanthamine*
D. Pilocarpine
E. Lobeline.

51. Analeptical remedy of reflective type from the N-cholinomimetics


group was given to the patient for restoration of breathing after poisoning with
carbon monoxide, What medicine was prescribed to the patient?
A. Adrenalin hydrochloride
B. Atropine sulphate
C. Mesaton
D. Pentamin
E. Lobeline hydrochloride*.

52. A patient with complaints of dryness in the mouth, photophobia and


vision impairment was admitted in the reception-room. Skin is hyperemic, dry,
pupils, are dilated, tachycardia. Poisoning with belladonna alkaloids was
diagnosed on further examination. What medicine should be prescribed?
A. Armine
B. Dipiroxim
C. Pilocarpine
D. Diazepam
E. Prozerin*.

53. On the 2-3rd day after stomach resection intestinal peristalsis wasn't
restored. What is to be administered for stimulation of gastrointestinal tract?
A. Atropine sulfate
B. Prasosin
C. Acetylcholine
D. Proserin(Neostigmine)*
E. Cyclodole.

54. The patient developed postoperative intestinal atony. After injection


of M-cholinomimetic the patient's condition was improved. Point out the drug
that was used in this cvase
A. Arminum
B. Aceclidine*
C. Ambenonium chloride (Oxazylum)
D. Atropine
E. Platyphylline.

55. After completion of abdominal surgery conducted with the use of


nondepolarizing myoreolaxant, the patient did not restored spontaneous
breathing. What kind of drug is able to restore breathing in this case?
A. Scopolamine
B. Aceclidine
C. Pilocarpine
D. Neostigmine*
E. Izonitrozinum.

56. A child poisoned by mushrooms containing muscarine was brought


to the toxicological department Which drug should be used for emergency
assistance in the first place?
A. Papaverine
B. Sodium thiosulfate
C. Atropine*
D. Unitiol
E. Dipiroxim.

57. A patient with signs of poisoning by organophosphorus insecticides


was delivered to the hospital emergency department What medication should be
applied as a first-aid?
A. Atropine*
B. Unitiol
C. Panangin
D. Chlorpromazine
E. Glucose.

58. The patient was taken to the emergency department with complaints
of dryness in the mouth, photophobia and blurred vision. On examination
hyperemic dry skin, dilated pupils, tachycardia were revealed, and poisoning
with belladonna alkaloids was diagnosed. Which of the drugs is advisable to
apply?
A. Neostigmine*
B. Pilocarpine
C. Dipiroxim
D. Diazepam
E. Arminum.

59. The patient in the postoperative period to stimulate intestinal


motility and tone of the bladder has been appointed agent of a group of
anticholinesterase. Identify it among the following products:
A. Propranolol (Anaprilin)
B. Mannitol
C. Reserpine
D. Dichlotiazid
E. Neostigmine*.

60. The patient poisoned with carbon monoxide to restore breathing


was treated with analeptic of a reflex type from group of N-cholinomimetics.
Which drug was assigned to the patient?
A. Adrenalin hydrochloride
B. Pentamin
C. Lobeline hydrochloride*
D. Mezaton
E. Atropine sulfate.

61. Specify the M-cholinomimetic used for treatment of glaucoma:


A. Neostigmine methylsulfate
B. Pilocarpine*
C. Adrenaline
D. Norepinephrine
E. Phenylephrine (Mesatonum).

62. Which of the following medicines is anticholinesterase agent:


A. Clonidine
B. Atropine
C. Adrenaline
D. Acetylcholine
E. Neostigmine*.

63. Which group is pilocarpine?


A. M-cholinomimetics*
B. Anticholinesterase
C. Adrenomimetics
D. Adrenoblockers
E. Cholinoblockers.

64. Enter the N-cholinomimetic, which stimulates the reflex of breath:


A. Neostigmine
B. Pilocarpine
C. Lobeline*
D. Atropine
E. Adrenaline.

Cholinoblockers

65. Teenager with fracture of hand was presented to the emergency


room. To perform reposition of fractured bones it was necessary to relax skeletal
muscle. For this purpose myorelaxant causing sustained depolarization of the
post junctional membrane was administered. Which of the following agents was
used?
A. Pancuronium
B. Atracurium
C. D-tubocurarine
D. Succinylcholine*
E. Millectine.

66. Patient suffering from gastric peptic ulcer was treated with
cholinotropic drug selectively blocking M1 cholinoceptors of parietal gastric
glands. Point out this drug?
A. Atropine
B. Plathyphylline
C. Metacine
D. Pirenzepine*
E. Hyoscine (scopolamine).

67. Young boy with burns is brought to emergency ward. There is


history of death of one of his family members due to succinylcholine induction.
So we will avoid succinylcholine because it
A. Causes hyperkalemia
B. Cause hepatotoxicity
C. Causes muscle pains
D. Can trigger malignant hyperthermia*
E. Can cause spasm of masseter muscle.

68. A patient given succinylcholine for skeletal muscle relaxation


during an operation, is not recovered for the last 1 hour. What must be used to
restore spontaneous respiration?
A. Carbogen
B. Lobelin
C. Neostigmine
D. Blood transfusion*
E. Caffeine.

69. A 59 year old man develops excessive salivation and sweating,


diarrhea, and bradycardia while being treated with neostigmine for myasthenia
gravis. Which of the following is the most appropriate therapy for these
symptoms and signs?
A. Atropine*
B. Carbachol
C. Edrophonium
D. Epinephrine
E. Pralidoxime.
70. A patient with symptoms of phosphorganic poisoning was taken to
emergency department. Which of the following drugs mast be used as a drug of
first aid?
A. Unithyolum
B. Potassium chloride
C. Atropine*
D. Chlorpromazine (Aminazine)
E. Glucose.

71. After abdominal surgery performed under general anaesthesia with


non- depolarizing myorelaxant spontaneous respiration was not restored. Which
of the following drugs is to be used as an antidote of non-depolarizing
myorelaxant?
A. Aceclidinum
B. Pilocarpin
C. Neostigmine (Proserin)*
D. Hyascine
E. Isonitrosine.

72. A 5 years old child was taken to toxicological department with


Belladonna poisoning. Alongside with peripheral manifestations the CNS
symptoms were displayed. Which of the following agents has to be administered
as an antidote?
A. Aceclidin
B. Pilocarpin
C. Neostigmine (Proserin)
D. Adrenaline
E. Galanthamine hydrobromide*.

73. For testing refraction of eye atropine was instilled into conjunctival
sac. On completion of the procedure another cholinergic drug was used to
counteract mydriasis and cycloplegia, caused by atropine. Point this drug.
A. Pilocarpine*
B. Lobeline
C. Hyoscine (Scopolamine)
D. Phenylephrine (Mesatonum)
E. Pirenzepine.

74. Administration of pirenzepine in patients with gastric peptic ulcer


is not accompanied by numerous side effects characteristic for atropine and
other M-cholinoblckers due to
A. Inability to penetrate through blood brain barrier
B. Selective inhibition of M1 –cholinoceptors*
C. Inhibition of all types of M –cholinoceptors
D. Inhibition of cholinesterase
E. Significant protein binding.
75. It is known that administration of tubocurarine during surgery can
enhance the development of postoperative paralytic ileus. What is the
mechanism of this side effect of tubocurarine?
A. Inhibition of M –cholinoceptors
B. Inhibition of cholinesterase
C. Ganglion blocking activity*
D. Stimulation of M –cholinoceptors
E. Stimulation of both M – and N-cholinoceptors.

76. Which of the following conditions is alleviated by centrally acting


anti-cholinergic drugs?
A. Depression
B. Multiple sclerosis
C. Mania
D. Parkinsonism*
E. Psychosis.

77. A 50-year-old male farm worker has been brought to the emergency
room. He was found confused in the orchard and since then has remained
unconscious. His heart rate is 45 and his blood pressure is 80/40 mm Hg. He is
sweating and salivating profusely. Which of the following should be prescribed?
A. Pentamine
B. Norepinephrine
C. Physostigmine
D. Atropine*
E. Proserine.

78. Introduction of a pharmaceutical substance to an experimental


animal resulted in reduction of salivation, pupil mydriasis. Next intravenous
introduction of acetylcholine didn't lead to any significant changes of heart rate.
Name this substance:
A. Proserin
B. Atropine*
C. Propranolol
D. Adrenaline
E. Salbutamol.

79. A patient had to go through an operation. Doctors introduced him


dithylinum (listenone) and performed intubation. After the end of operation and
cessation of anesthesia the independent respiration wasn't restored. Which
enzyme deficit prolongs the action of muscle relaxant?
A. Pseudocholinesterase*
B. K-Na-adenosine triphosphatase
C. N - acetyltransferase
D. Succinate dehydrogenase
E. Carbanhydrase.
80. A patient with fracture of his lower jaw was admitted to the
maxillofacial department. It was decided to fix his bones surgically under
anaesthetic. After intravenous introduction of muscle relaxant there arose short
fibrillar contractions of the patient's facial muscles. What muscle relaxant was
applied?
A. Dithylinum*
B. Pipecuronium bromide
C. Diazepam
D. Melictine
E. Tubocurarin chloride.

81. If a man has an attack of bronchiospasm it is necessary to reduce


the effect of vagus on smooth muscles of bronchi. What membrane
cytoreceptors should be blocked for this purpose?
A. M-cholinoreceptors*
B. beta-adrenoreceptors
C. alfa- and beta-adrenoreceptors
D. alfa-adrenoreceptors
E. N-cholinoreceptors.

82. The patient with bronchial asthma uses inhaled form M -


holinoblocker. In this case, as a rule, characteristic side effects such as blurred,
constipation tachycardia do not appear. Point out the drug that patient uses.
A. Platifillin
B. Atropine
C. Metacin
D. Ipratropium Bromide*
E. Pirenzepine.

83. For the treatment of patients with essential hypertension doctor


prescribed the drug from the group M – cholinoblocker. Enter the drug.
A. Platyphylline*
B. Atropine
C. Scopolamine
D. Metacin
E. Gastrotsepin.

84. The drug selectively blocking M1 - cholinergic receptors was


assigned to patient suffering from gastric peptic ulcer. What this drug is?
A. Pirenzepine*
B. Atropine
C. Platyphylline
D. Methacinum
E. Scopolamine.

85. A patient with a fracture of the mandible was delivered to the


Maxillofacial department. It was decided to connect the bones surgically under
general anesthesia. After intravenous injection of muscle relaxant transient
fibrillar contractions of facial muscles were observed. What kind of muscle
relaxant was used?
A. Dithylinum*
B. Tubocurarine chloride
C. Pipecuroniub bromide
D. Mellictinum
E. Diazepam.

86. Dentist in order to reduce salivation during the filling of the tooth
prescribed the drug to the patient. What the medicine was prescribed?
A. Adrenalin hydrochloride
B. Phenylephrine (Mezaton)
C. Neostigmine
D. Pilocarpine hydrochloride
E. Atropine sulfate*.

87. Iintroduction of the drug in experimental animals decreases


salivation, dilates pupils, and on the subsequent intravenous injection of
acetylcholine heart rate did not change significantly. Specify the name of the
substance:
A. Propranolol (Anaprilin)
B. Neostigmine methylsulfate
C. Adrenaline
D. Salbutamol
E. Atropine*.

88. Dithylinum was injected to the patient before surgery and


intubation was performed. After the operation, and termination of anesthesia
spontaneous breathing was not restored. Deficiency of which enzyme is
responsible for prolongation of this muscle relaxant action?
A. Carbonic anhydrase
B. Pseudocholinesterase*
C. Succinate dehydrogenase
D. K-Na-ATPase
E. N-acetyltransferase.

89. The preparation of the depolarizing neuromuscular blocking agents


should be assign to the patient with a broken limb for short-lasting surgical
procedure. Choose the drug.
A. Atropine sulfate
B. Pentamin
C. Dithylinum*
D. Cytitonum
E. Tubocurarine chloride.
90. In a patient with myasthenia after the treatment with neostigmine
nausea, diarrhea, and hypersalivation appeared. Point out the drug to eliminate
these side effects.
A. Atropine Sulfate*
B. Phenylephrine (Mezaton)
C. Physostigmine
D. Isoprenaline
E. Pyridostigmine bromide.

91. In poisoned patient dryness of mucous membranes of the mouth and


dilated pupils were observed. What is the mechanism of these symptoms
development?
A. Stimulation of N-cholinergic receptors
B. Stimulation of the M-cholinergic receptors
C. Adrenoceptor blockade
D. M-cholinergic receptor blockade*
E. Stimulation of adrenergic receptors.

92. In the patient with a forearm injury during the bone repositioning
Dithylinum was used for muscle relaxation. Full recovery of muscle tone and
function took more than an hour. What can be the cause of significant
lengthening of the drug’s action?
A. Genetic deficiency of monoamine oxidase
B. Genetic deficiency of butyrylcholinesterase*
C. Inhibition of microsomal oxidation
D. Genetic deficiency of hydroxylase
E. Formation of the active metabolite.

Adrenomimetics and sympathomimetics

93. A patient with bronchial asthma is administered inhalation of 0,5


percent isadrine solution. Bronchospasm was relieved but the patient began
complaining of pain in the heart region and palpitation. It is connected with the
stimulation of :
A. alpha one – adrenoreceptros
B. Beta two – adrenoreceptros
C. Beta one – adrenoreceptros*
D. Acetylchloline synthesis
E. M-chlolinoceptors.

94. Patient with bronchial asthma was taking tablets, which caused
insomnia, headache, increased blood pressure. What medicine can cause such
complications?
A. Chromolin sodium
B. Adrenaline
C. Euphyline
D. Ephedrine*
E. Izadrine.

95. Spasm of smooth muscle of bronchi developed in the patient. Usage


of activators of what membrane cytoreceptors is physiologically valid to
decrease attack?
A. alpha-аdrenoreceptors
B. beta-adrenoreceptors*
C. alpha-and- beta-аdrenoreceptors
D. N-cholinoreceptors
E. М-cholinoreceptors.

96. In the treatment of patient suffering from congestive heart failure


resistant to digitalis adrenomimetic was used as inotropic agent. Point out which
of the following drugs was administered.
A. Phenylephrine
B. Partusisten
C. Dobutamine*
D. Xylometazoline
E. Naphazoline.

97. 63 years patient with symptoms of collapse was admitted into


emergency department. Doctor has chosen noradrenaline to overcome collapse.
What mechanism is responsible for therapeutic effect of this drug?
A. Activation of alfa1 receptors*
B. Activation of beta adrenoceptors
C. Activation of dopaminergic receptors
D. Blockade of M –cholinoceptors
E. Activation of serotonin receptors..

98. Dopamine increases urine production in a shocked patient because


it:
A. Increase cardiac output
B. Decreases aldosterone release
C. Decreases ADH release
D. Causes peripheral vasodilation
E. Causes renal arterial dilation*.

99. To perform fundoscopy ophthalmologist instilled in the eye an


agent capable of causing mydriasis without cycloplegia. Point out this agent.
A. Phenylephrine (Mesatonum)*
B. Noradrenaline
C. Atropine
D. Pilocarpine
E. Isoprenaline (Isadrinum).
100. A patient with bronchial asthma addresses his doctor with
complaints about unpleasant palpitations that occur after usage of inhalation
form of isoprenaline (isadrinum).What is the cause of this side effect?
A. stimulation of alfa1– adrenoceptros
B. stimulation of alfa2– adrenoceptros
C. stimulation of beta1– adrenoceptros*
D. stimulation of beta2– adrenoceptros
E. inhibition of M-chlolinoceptors.

101. A patient with moderate hypertension was treated with the drug that
realizes its effect by decrease in noradrenaline release from sympathetic nerve
endings. While treatment patient experienced drowsiness. What the drug was
used?
A. Prazosin
B. Reserpin*
C. Drotaverine
D. Atenolol
E. Dichlorothiazide.

102. Surgical treatment of benign tumor of prostate in 67- year patient


was postponed instead pharmacotherapy with adrenergic drag was started. Point
out which of the drugs was most probably used.
A. Adrenaline
B. Ephedrine
C. Prazosine*
D. Propranolol (Anaprilinum)
E. Atenolol .

103. A patient has got a spasm of smooth muscles of bronchi. Blockers


of what membrane cytoreceptors are physiologically reasoned to stop an attack?
A. beta-adrenoreceptors
B. M-cholinoreccptors*
C. alpha- and beta-adrenoreceptors
D. H-cholinoreceptors
E. alpha -adrenoreceptors.

104. A patient ill with bronchial asthma didn't inform his doctor that he
had attacks of stenocardia. Doctor administered him a medication, which taking
resulted in less frequent attacks of bronchial asthma. but stenocardia attacks
became more frequent. What medication was administered?
A. Cromolyn sodium
B. Salbutamol
C. Aminophylline
D. Phenotherol
E. Isadrin*.
105. An ophthalmologist used a 1% mesaton solution for the diagnostic
purpose (pupil dilation for eve-ground examination). What is the cause of
mydriasis induced by the drug?
A. Activation of alfa1 adrenoreceptors*
B. Activation of beta1 adrenoreceptors
C. Activation of M-cholinoreceptors
D. Block of alfa1 adrenoreceptors
E. Activation of alfa2 adrenoreceptors.

106. At the doctor's the dentist- surgeon injected 1% - procaine for


conduction anesthesia. It caused the change in patient’s condition: the skin was
covered with sticky and cold sweat, the pulse became almost impalpable,
breathing became rare and weak. Realizing that anaphylactic shock is originated,
a doctor immediately introduced intravenously drug that promptly improved the
patient's condition. Which of the following drugs can be used for first aid in
severe allergic reactions?
A. Ipratropium bromide
B. Labetalol
C. Fenoterol
D. Adrenaline*
E. Dobutamine.

107. A patient with heart failure, resistant to cardiac glycosides was


admitted to hospital. A doctor appointed adrenomimetic as inotropic
(cardiotonic) drug. Which of these adrenomimetic has inotropic (cardiotonic)
action?
A. Dobutamine*
B. Phenylephrine (Mezaton)
C. Naphazoline
D. Fenoterol
E. Halazolin.

108. In the maternity ward there is a pregnant woman with the danger of
miscarriage. Which of the following agents can be used to save the pregnancy?
A. Reserpine
B. Isoprenalin
C. Nicergoline
D. Ephedrine
E. Fenoterol (Partusisten)*.

109. The patient went to the doctor for medical examination. In the
queue while waiting an attack of asthma originated. Which group of drugs is
advisable to use to provide first aid?
A. N - cholinomimetics
B. Sympathomimetics
C. Beta-adrenomimetics*
D. Beta-adrenoblockers
E. M - cholinomimetics.

110. In anesthetic practice adrenaline is usually added to the local


anesthetics to prolong analgesia and reduce side effects. What the effect of
adrenaline contributes to this?
A. Reducing the sensitivity of pain receptors
B. Constriction of blood vessels*
C. Increase in peripheral resistance
D. Stimulation of the antinociceptive system
E. CNS depression.

111. In the course of therapy of a patient a number of complications such


as mydriasis, tachycardia, and increased blood pressure have occurred. Which of
the listed drugs can cause such side effects?
A. Propranolol (Anaprilin)
B. Adrenaline*
C. Phentolamine
D. Isoprenaline
E. Norepinephrine.

112. A patient made an inhalation of medication to relieve an attack of


asthma it quickly improved patient condition but caused tachycardia. Which of
the following agents can lead to such a complication?
A. Fenoterol
B. Atenolol
C. Metoprolol
D. Isoprenaline*
E. Salbutamol.

113. A pregnant woman with a danger of abortion is admitted to


obstetric department. Which of these drugs should be applied to save the
pregnancy?
A. Salbutamol*
B. Oxytocin
C. Neostigmine methylsulfate
D. Dinoproston
E. Quinine.

114. An ambulance was called to women 28 years old in connection with


an attack of asthma. After intramuscular injection of ephedrine her condition
was improved. However, after 49 minutes the attack was repeated, ephedrine
injected once more acted weaker, in the third attack of bronchospasm that
occurred in half an hour, ephedrine did not cause any effect.What is the name of
this phenomenon?
A. Addiction
B. Biotransformation
C. Elimination
D. Adsorption
E. Tachyphylaxis*.

115. In a patient during a visit to the dentist an acute hypotension


occurred. Which of the drugs that stimulate adrenergic structures should be used
to normalize blood pressure?
A. Naphazoline
B. Xylometazoline
C. Phenylephrine (Mezaton)*
D. Ergotamine
E. Doxozasin.

116. In the experimental modeling of mechanism of inotropic effect of


adrenaline activation of one of listed receptors was revealed. Point out these
receptors.
A. M-Cholinergic receptors
B. Beta-1 - adrenoceptors*
C. Beta 2 - adrenergic receptors
D. Alpha 2 - adrenergic receptors
E. Alpha 1 - adrenergic receptors.

117. A patient with acute rhinitis was often applying intranasally 5%


solution of ephedrine hydrochloride. The effectiveness of the drug was declining
with each usage and on the third day disappeared entirely? What is the cause of
rapid decrease in drug’s efficacy (tachyphylaxis)?
A. The progressive decrease in noradrenaline storage in sympathetic
nerve endings*
B. Violation of the synthesis of norepinephrine in the sympathetic
nerve endings
C. The increase in the reuptake of neurotransmitter
D. Adrenoceptor blockade
E. Violation of the process of neurotransmitter release from the depot.

Antiadrenergic drugs sympatholytics

118. The patient with a history of asthma have developed atrial


fibrillation, that requires agent appointment. Which of the listed
antiarrhythmic drug is contraindicated in this case?
A. Nifedipine
B. Verapamil
C. Propranolol (Anaprilin)*
D. Procainainamide
E. Ajmaline.
119. A patient with fibrillation, who has had bronchial asthma in his
anamnesis, should be administered antiarrhythmic drug. Which preparation from
the suggested group is contraindicated for this patient?
A. Ajimaline
B. Verapamil
C. Novocainamide
D. Nifedipine
E. Propranolol (Anapriline )*.

120. 72-year woman suffering from angina pectoris was treated by


propranolol (anaprilinium). On the background of the treatment the cough with
bronchospastic components was developed. Doctor replaced propanolol by
atenolol. It results in abolishing of respiratory disturbances. What is the reason
of beneficial effect of atenolol ?
A. Block of M-cholinoreceptors
B. Absence of action upon beta2- adrenoceptors*
C. Stimulating of M-cholonoreceptorsd
D. Inhibition of m-cholinireceptors
E. Absence of action on alfa- adrenoceptors.

121. Doctor prescribed selective alfa1-adrenoceptor blocker for


treatment of hypertensive patient. Which of the following drugs belongs to this
group?
A. Metoprolol
B. Phentolamine
C. Cordarone
D. Tropaphan
E. Doxazosine*.

122. Patient with angina pectoris was treated with drug decreasing
oxygen demands of myocardium. While treatment, patient starts to complain of
extremities coldness especially fingers. Which of the drug was used?
A. Atenolol
B. Propanol (anaprilin)*
C. Reserpine
D. Prazosin
E. Labetalol.

123. Patient with Raynaud disease was treated with phentolamine. What
is the mechanism of action of this drug?
A. Blockade of B-adrenoceptors
B. Blockade of alpha adrenoceptors*
C. Decrease in neurotransmitter amount
D. D Stimulation of central alpha2 adrenoceptors
E. Blockade of central M-cholinoceptors.
124. Patient with initial form of hypertension complains of pain in heart
and tachycardia. Which of the following drugs has to be used in the treatment of
this patient
A. Phentolamine
B. Dihydroergotamine
C. Propranolol (Anaprilin)*
D. Prazosin
E. Guanethidine(Octadinum).

125. Patient having in the past episodes of bronchial asthma addressed


doctor with angina pectoris. Doctor decided to treat patient with B-
adrenoblocker. Which of B-adrenoblocker is not contraindicated in this case?
A. Anapriline(Propranolol)
B. Atenolol*
C. Oxprenolol
D. Pindolol
E. Nadolol.

126. Middle aged woman addressed doctor complaining on the attack of


periodical headache. On examination migraine was diagnosed. Doctor
prescribed anti-adrenergic drug. Point out the most suitable drug.
A. Phentolamine
B. Dihydoergotamine*
C. Atenolol
D. Dopamine
E. Reserpine.

127. For treatment of hypertensive patient doctor prescribed


antiadrenergic drug, first administration of which in therapeutic dose caused
“first dose syncope”. Which drug was most probably used?
A. Metoprolol
B. Atenolol
C. Prazosin*
D. Reserpin
E. Propranolol.

128. A 43 year-old male patient is suffering from hypertension. His


blood pressure was successfully controlled by monotherapy with adrenoblocker.
With time treatment was complicated with diarrhoea and impotence. The patient
addressed his doctor and the drug was changed for another adrenoblocker.
Which of adrenoblockers can cause above listed side effects?
A. Metoprolol
B. Salbutamol
C. Propranolol
D. Dobutamine
E. Prazosin*.
129. In the presence of a beta-blocker, administration of adrenaline is
most likely to cause:
A. Hypoglycemia
B. Hypokalemia
C. Hypertension*
D. Hyperglycemia
E. Hypotension..

130. Doctor prescribed selective alfa1-adrenoceptor blocker for


treatment of hypertensive patient. Which of the following drugs belongs to this
group?
A. Metoprolol
B. Phentolamine
C. Cordarone
D. Tropaphan
E. Prazosin*.

131. A 62-year-old patient, prone to bronchoconstriction, addressed his


doctor with complaints of tachyarrhythmia troubling him several times a day.
The doctor prescribed anaprilin (propranolol). Administration of drug provoked
severe attack of bronchospasm. What is the mechanism of this side effect?
A. Stimulation of M1- cholinoceptors
B. Inhibition of alfa-adrenoceptors
C. Inhibition of beta2 - adrenoceptors*
D. Increase in n. vagus tone
E. Stimulation of leukotriene release.

132. A young woman with 6 months pregnancy visited her doctor


complaining of periodical slight contractions of uterus. To prevent premature
labour doctor prescribed partusisten (fenoterol). What is the mechanism of
action of this drug?
A. Stimulation of beta2 - adrenoceptors*
B. Inhibition of beta2 - adrenoceptors
C. Stimulation of alfa1 - adrenoceptors
D. Inhibition of alfa1 - adrenoceptors
E. Stimulation of M-cholinoceptors.

133. The patient with the initial form of hypertension complains of pain
in the heart and tachycardia. Which of the following drugs should be used in the
treatment of this patient?
A. Prazosin
B. Phentolamine
C. Dihydroergotamine
D. Propranolol (Anaprilin)*
E. Reserpine.
134. A doctor prescribed the drug to the patient with hypertension. The
drug is acting by selective block of alpha 1 - adrenergic receptors. What the drug
was prescribed?
A. Metoprolol
B. Doxazosin*
C. Phentolamine
D. Propranolol (Anaprilin)
E. Reserpine.

135. The patient in case of angina chest pain regularly takes


nitroglycerin, however, recently angina pectoris complicated by extrasystoles
and episodes of bronchospasm. Which of beta - blockers better to appoint for
such a patient?
A. Timolol
B. Propranolol (Anaprilin)
C. Oxprenolol
D. Atenolol*
E. Nadolol.

136. Patients with essential hypertension (stage II) used one of


antihypertensive drugs. After a while blood pressure decreased, but the patient
began to complain of fatigue, sleepiness, apathy. Later on, a pain in the stomach
occurred, and gastric peptic ulcer was diagnosed. What the drug was used?
A. Reserpine*
B. Verapamil
C. Captopril
D. Dibazol
E. Furosemide.

137. A patient with ischemic heart disease was admitted to cardiology


department. To prevent attacks of angina pains drug of beta-1adrenoblockers
was assigned. Name this drug:
A. Metoprolol*
B. Morphine hydrochloride
C. Furosemide
D. Oxytocin
E. Atropine sulfate.

138. Hypertensive patients is taking sympatholytic. Enter the drug.


A. Propranolol (Anaprilin)
B. Reserpine*
C. Adrenaline
D. Atropine
E. Phenylephrine (Mesatonum).

139. Note the non-selective beta-blocker.


A. Mezatonum
B. Atenolol
C. Adrenaline
D. Salbutamol
E. Propranolol (Anaprilin)*.

140. The patient with hypertension was appointed cardioselective beta-


blocker. Which drug is assigned to the patient?
A. Reserpine
B. Propranolol (Anaprilin)
C. Metoprolol*
D. Atropine
E. Adrenaline.

141. To patient with a tachyarrhythmia doctor prescribed a beta-blocker.


Enter the drug.
A. Adrenaline
B. Propranolol (Anaprilin)*
C. Atropine
D. Isoprenaline
E. Salbutamol.

142. The patient suffering from angina takes a selective beta-


adrenoblocker. Enter the drug.
A. Metoprolol*
B. Prazosin
C. Atropine
D. Pilocarpine
E. Propranolol (Anaprilin).

143. Non selective alpha-adrenoblocker is prescribed for diagnostic


purposes to a woman with a suspected pheochromocytoma. Point out this drug
A. Fentolamin*
B. Propranolol (Anaprilin)
C. Salbutamol
D. Phenylephrine (Mesatonum)
E. Clonidine.

The drugs for general anaesthesia

144. To anaesthetize the manipulation connected with burn surface


treatment, a patient was intravenously injected a medication for short-acting
narcosis.1 minute later the patient being under anesthesia had elevated blood
pressure, tachycardia, increased tone of skeletal muscles; the reflex were
reversed. After awakening the patient had disorientation and the visual
hallucination. What medicine was injected to the patient?
A. Sombrevin
B. Thiopental sodium
C. Nitrous oxide
D. Ketamine*
E. Diethyl ether.

145. The anaesthetic used along with halothane to give excellent


analgesia and rapid induction is
A. Nitrous oxide*
B. Thiopental
C. Desflurane
D. Propofol
E. Choroform.

146. To anaesthetize the manipulation related to burn surface treatment,


a patient was intravenously injected a medication for short-acting narcosis. 1
minute later the patient being under anaesthesia had increased blood pressure,
tachycardia, increased tone of sceletal muscles reflexes remained. After
awakening the patient had desorientation and visual hallucinations. What
medication was the patient injected?
A. Nitrous oxide
B. Thiopental sodium
C. Ketamine*
D. Diethyl ether
E. Sombrevin.

147. Dental surgeon made tooth extraction in a patient under general


anesthesia. Which of the following medications can be used in this case for
short-term noninhalation general anesthesia?
A. Ketamine*
B. Predion
C. Thiopental sodium
D. Halothane
E. Nitrogen oxide.

148. To make the treatment of burn surface painless intravenous general


anesthetic for short-term anesthesia was injected to patient. Anesthesia
developed in 1 minute during anesthesia increase in blood pressure, tachycardia,
increase in tone of skeletal muscles have occurred. On the recovery from
anesthesia patient experienced disorientation and visual hallucinations. What is
the drug entered the patient?
A. Ether for anesthesia
B. Sombrevin
C. Ketamine*
D. Thiopental sodium
E. Nitrous oxide.
149. To do a biopsy of breast tumors in 35 years old woman analgesia is
required. What type anesthesia should be used?
A. Potentiated anesthesia with muscle relaxants
B. Combined mixed anesthesia
C. Neurolept-analgesia*
D. Basic anesthesia (Na-hydroxybutyrate)
E. Inhalation anesthesia.

150. A 28 years old woman addressed the dentist about dental


prosthetics. Because she is allergic to local anesthetics, dental treatment was
necessary to carry out under general anesthesia. Which general anesthetic drug
should be used for this purpose, taking into consideration short duration of
manipulation?
A. Sodium hydroxybutyrate
B. Ketamine*
C. Thiopental
D. Halothane
E. Hexobarbital (Hexenalum).

151. To anesthetize the procedure of taking a biopsy in patient a drug


that causes a fast anesthesia lasting about 5 minutes was injected intravenously.
During anesthesia there were involuntary movements, slight decrease in blood
pressure, short-term respiratory arrest, which was quickly restored. What kind of
drug was used?
A. Halothane
B. Ketamine
C. Predion
D. Propanidid*
E. Sodium hydroxybutyrate.

152. Generalized tonic-clonic seizures with loss of consciousness were


periodically occurring in a patient after an injury; these seizures were followed
by a general depression of central nervous system. What the drug should be
assign to prevent seizures?
A. Trihexyphenidyl (Cyclodolum)
B. Midantan
C. Levodopa
D. Disilfiram (Teturam)
E. Phenobarbital*.

153. Identify the drug that has sedative, hypnotic, anesthetic, anti-
hypoxic, myorelaxant actions and it is used as general anesthetic and hypnotic.
A. Sodium hydroxybutyrate*
B. Hexobarbital
C. Chloral hydrate
D. Diazepam
E. Thiopental sodium.
154. The patient takes disulfiram (teturam). What is the indication for
use of this drug?
A. Psychosis
B. Preparation for surgery
C. Fear and anxiety
D. Neurosis
E. Alcoholism*.

Hypnotic, antiepileptic and antiparkinsonian drugs

155. A 5 years old child from time to time have seizures with loss of
consciousness. The doctor diagnosed epilepsy (large seizures). Which drug is
advisable to appoint a patient?
A. Phenobarbital*
B. Benactizin(Amizulum)
C. Trihexyphenidyl (Cyclodolum)
D. Ethosuximide
E. Levodopa.

156. A man aged 68 with trembling hands and incoordination diagnosed


with Parkinson's disease. Which drug is advisable to appoint a patient?
A. Phenytoin
B. Finlepsin
C. Ethosuximide
D. Phenobarbital
E. Trihexyphenidyl (Cyclodolum)*.

157. 40 years woman addressed doctor with complaints of psychic


disturbances appeared with long-term usage of phenobarbital as a hypnotic.
What the drug should be used instead of phenobarbital?
A. Sodium hydroxybutyrate
B. Bromizoval
C. Chloral hydrate
D. Nitrazepam*
E. Noxiron.

158. Due to emotional stress a man permanently is in a state of nervous


tension and has poor sleep. The doctor gave him a diazepam. Which of the
following effects of diazepam is the base for its clinical application in this case?
A. Hypotensive
B. Anxiolytic (Tranquilizing)*
C. Analgesic
D. Anticonvulsant
E. Antidepressant.
159. A patient suffering from insomnia addressed his doctor. The main
problem was in the difficulties with sleep onset. Doctor prescribed zolpidem.
Which of the following characteristics is true for this agent?
A. It belongs to benzodiazepine derivatives
B. It belongs to barbiturates
C. It has the ability to cause quick onset of sleep*
D. It’s poorly absorbed from GIT
E. It’s poorly bind to protein.

160. A 46- year-old patient with a history of epilepsy (grand mal)


developed status epilepticus. Choose the drug to arrest convulsions.
A. Sodium valproate
B. Diazepam*
C. Levodopa
D. Chlorpromazine
E. Droperidol.

161. An antiepileptic drug, which prevents metabolism of GABA, was


included in complex therapy of patient with epilepsy (grand mal). Point out this
drug
A. Phenytoin
B. Phenobarbitone
C. Carbamazepine
D. Ethosuximide
E. Sodium valproate*.

162. Patient with phenobarbital poisoning was presented to emergency


department. To promote kidney excretion of poison sodium bicarbonate was
administered intravenously. What mechanism is responsible for an increase in
barbiturate excretion?
A. Increase in polarity of barbiturate*
B. Increase in fat solubility of barbiturate
C. Increase in barbiturate reabsorption
D. Increase in rate of glomerular filtration
E. Decrease in protein binding.

163. Deficiency in dopamine production by neurons of strio-pallidal


system is considered to be one of the main causes of Parkinson’s disease. Point
out the drug used for replacement therapy of this disease
A. Atropine
B. Dopamine
C. Levodopa*
D. Carbidopa
E. Amantadine.
164. The patient was taken the mixture prescribed by neuropathologist
for neurasthenia for 2 weeks. Patient felt better but developed coryza,
conjunctivitis, rash, inertia, decrease of memory. Bromizm was diagnosed.
What should be prescribed to decrease symptoms?
A. Glucose solution 5%
B. Calcium chloride
C. Natrium chloride*
D. Polyglucin
E. Asparcam.

165. The antiepileptic drug which causes nystagmus, ataxia and gum
hypertrophy is
A. Phenytoin*
B. Phenobarbitone
C. Carbamazepine
D. Ethosuximide
E. Valproic acid.

166. A 16 years boy is a known as patient suffering from epilepsy.


Following several years of a drug therapy, gingival hyperplasia developed. For
which drug this side effect is characteristic?
A. Alprazolam
B. Carbamazepine
C. Valproic acid
D. Ethosuximide
E. Phenytoin*.

167. A woman, which is on phenytoin, wants to conceive. What advice


you can give to her about drug?
A. Increase the dose of phenytoin
B. Change to valproic acid
C. Addition of valproic acid
D. Change to phenobarbitone*
E. Decrease the dose of phenobarbitone.

168. An old epileptic patient on phenytoin is having status eplilepticus.


What will you give to this patient in emergency?
A. Phenytoin
B. Carbamazepine
C. Diazepam*
D. Valproic acid
E. Primidone.

169. Disulfiram is widely used in medical practice to prevent


alcocholism. It inhibits aldehyde dehydrogenase. Increased level of what
metabolite causes aversion to alcochol?
A. Propionic aldehyde
B. Methanol
C. Ethanol*
D. Acetaldehyde
E. Malonyc aldehyde.

Pharmacology of opioid analgesics

170. Very painful uterine contractions have been occurring in a pregnant


woman at normal term. To relieve the woman’s condition during labour, a
doctor decided to use opioid (narcotic) analgesic. Which of the listed drugs can
be used in this case?
A. Trimeperidine (Promedolum)*
B. Morphine
C. Metamizole (Analginum)
D. Papaverine
E. Codeine.

171. A 60-year-old male is brought to emergency room. He is comatose


and his pupils are constricted. Physician suspects opium overdose. What is the
best drug to be administered?
A. Flumazanil
B. Calcium carbonate
C. Sodium bicarbonate
D. Naloxone*
E. Atropine.

172. A young man was taken to emergency department with numerous


traumas of chest and head. Anesthesiologist proposed to inject morphine to
relieve patient’s condition, but traumatologist rejected proposition of his
colleague. Why morphine is contraindicated in this case?
A. It increases intracranial pressure*
B. It stimulates vagal center
C. It decreases intraocular pressure
D. It causes miosis
E. It depresses center of cough reflex.

173. It is known that morphine can cause a shift of blood from


pulmonary to systemic circuit. It results in lowering of resistance in pulmonary
vascular bed. In which case in may be used?
A. Bronchitis
B. Pneumonia
C. Pulmonary oedema*
D. Bronchiectatic disease
E. Emphysema.
174. Patient in unconscious state was admitted to the emergency room.
Skin is cold, pupils are pinpoint like, breathing is heavy, with cycles of the
Cheyne- Stokes type, blood pressure is decreased, urinary bladder is overloaded.
Poisoning with what substance is the most likely?
A. Neosigmin (Proserinum)
B. Sedatives
C. M-cholinergic antagonists
D. Nor-narcotic analgesics
E. Narcotic analgesics*.

175. Examination of a patient revealed extremely myotic pupils,


sleepiness, infrequent Chain-Stoke's respiration. urinary retention, slowing-
down of heart rate, enhancement of spinal reflexes. What substance caused the
poisoning?
A. Atropine
B. Phosphacole
C. Caffeine
D. Morphine*
E. Barbital.

176. A patient with acute morphine poisoning was delivered to a


hospital. What specific narcotic antagonist should be chosen in this case?
A. Unithiol
B. Paracetamol
C. Digoxin
D. Methacin
E. Naloxone*.

177. A young man with an addiction to opioid drugs denies drug


dependence. Introduction of one of these drugs will help to prove drug
dependence. Point out this drug.
A. Morphine
B. Codeine
C. Trimeperidine (Promedol)
D. Fentanyl
E. Pentazocine*.

178. A 30 years old patient was delivered to emergency department with


a fractured thigh bone after a car accident. The patient had dramatically reduced
blood pressure to 70/40 mm Hg, weak pulse, enhanced pain response to touch to
damaged area. What should be used for prevention of traumatic shock in
patient?
A. Metamizol (Analgin)
B. Papaverine
C. Morphine*
D. Paracetamol
E. Pentazocine.
179. The patient with inoperative lung cancer metastasizing in the spine
was suffering from severe pain. To relieve patient's condition analgesic is
required. Which drug should be administered?
A. Analgine
B. Codeine
C. Paracetamol
D. Morphine*
E. Papaverine.

180. On arrival of ambulance a 25 years old patient was unconscious and


15 vials of morphine solution were found near him patient had periodic
breathing, slowed heart rate, suppressed reflexes except for the knee, and
narrowed pupils he was taken to the emergency department. What measure
should be applied in the first place?
A. Gastric lavage*
B. Diuretic
C. Antiarrhythmic drug
D. Enema
E. Induction of vomiting.

181. A child of 4 years is hospitalized to the orthopedic department with


a fractured ankle with displacement. The reposition of bone fragments required
analgesia. What medication should be used?
A. Analgin
B. Promedol*
C. Morphine hydrochloride
D. Panadol
E. Omnopon.

182. Choose a specific antagonist of narcotic analgesics.


A. Caffeine-sodium benzoate
B. Ethimizol
C. Bemegride
D. Strychnine
E. Naloxone*.

183. A patient with symptoms of renal colic is admitted to the urology


department. Which of these drugs should be advised to provide him first aid?
A. Morphine
B. Trimeperidine (Promedol)*
C. Pentazocine
D. Fentanyl
E. Tramadol.
184. A patient with dislocation of shoulder was admitted to
traumatologic department. The reduction of dislocation should be performed
under neuroleptanalgesia. Chose the opioid analgesic used for this purpose.
A. Naloxone
B. Cordiamin
C. Morphine
D. Ethimizol
E. Fentanyl*.

Pharmacology of nonopioid analgesics

185. A 67 year-old male patient suffering from arthritis was treating


himself with acetyl salicylic acid (aspirin). In 2 weeks patient addressed doctor
complaining of pain in epigastric area. On examination of the patient doctor
prescribed anti-inflammatory agent, selectively inhibiting COX–2. Point out this
drug.
A. Paracetamole
B. Diclophenac-sodium
C. Celecoxib*
D. Metamisole (Analginum)
E. Indomethacine.

186. In complex treatment of a patient with angina pectoris non- narcotic


analgesic exerting antiplatelet activity was included. Point out this agent.
A. Meperidine
B. Morphine
C. Tramadol
D. Metamizole
E. Aspirin*.

187. Due to inhibition of COX aspirin exerts all the following effects
except
A. Anti-inflammatory
B. Antiplatelet
C. Increase in warfarin efficacy*
D. Antipyretic
E. Analgesic.

188. A patient with toothache was relieving his pain with help of
metamizole (analginum). Point out another useful effect of this drug that
contributes to the improvement of patient’s condition
A. Sedative effect
B. Anti-inflammatory effect*
C. AntIplatelet effect
D. Antioxidative effect
E. Antimicrobial effect.

189. The usage of acetylsalicylic acid resulted in occurrence of


epigastric pain in patient. On examination of the patient gastric ulcer was
revealed. What the mechanism is responsible for ulcerogenity of this drug?
A. Stimulation of pepsin secretion
B. Inhibition of prostaglandin synthesis*
C. Increase in bile production and its reflux
D. Spasm of vessels
E. Spasm of gastric smooth muscles.

190. Doctor visited a 5-year-old child with fever and complaints of


headache. Doctor suspected the onset of viral infection. To relieve patient’s
condition doctor prescribed non-opioid analgesic, which additionally exerts
antipyretic effect but lacks anti-inflammatory action. Point out the prescribed
drug
A. Metamizole (Analginum)
B. Celecoxib
C. Aspirin
D. Acetaminophen (Paracetamol)*
E. Diclophenac-sodium.

191. After the usage of acetylsalicylic acid a patient developed epigastric


pain because of exacerbation of his ulcer. What is the mechanism of this drug
ulcerogenity?
A. Stimulation of pepsin secretion
B. Antiprostagladinic effect*
C. Cholagogic effect
D. Spasm of vessels
E. Immunodepressive effect.

192. Point out non- narcotic analgesic, which exerts antiplatelet activity.
A. Meperidine
B. Morphine
C. Tramadol
D. Metamizole
E. Aspirin*.

193. A 2-year-old child is often catching a cold with fever. Which


antipyretic drug is safer at this age than acetylsalicylic acid?
A. Analgin
B. Indomethacin
C. Paracetamol*
D. Phenylbutazone
E. Phenylbutasone.
194. A 7 year child have caught cold, he was shivering, fever up to 39,2
° C, his respiratory rate - 45 per minute, pulse - 105 beats per minute. The doctor
diagnosed an acute viral infection, and appointed a baby aspirin. By means of
what mechanism the drug realized its antipyretic effect in this case?
A. The overall calming effect
B. Slowing down heartbeat
C. Increase in heat output*
D. Slowing of the respiratory rhythm
E. Reduction of heat production.

Neuroleptics, antianxiety, and sedative agents

195. A patient was treated for psychosis for 2 weeks. The psychic
patient's condition improved but rigidity, tremor, and hypokinesia developed.
Which of the drugs can cause such complications?
A. Diazepam
B. Chlorpromazine (Aminazine)*
C. Amytriptiline
D. Piracetam
E. Chlordiazepoxide.

196. A 60 years old woman addressed her doctor complaining of side


effects which appeared while treatment with chlorpromazine (aminazinum). She
was troubled with tremor and disturbances of movements. What is the
mechanism of this side effect?
A. Activation of hyppocampus
B. Inhibition of reticular formation (alfa1 - adrenoceptors)
C. Inhibition of neostriatum (D2 receptors)*
D. Inhibition of hypothalamus
E. Inhibition of hyppocampus.

197. A 50 years old patient with chronic alcoholism became aggressive.


To abolish aggression, chlorpromazine was administered intramuscularly. The
patient’s attempt to rise soon after an injection resulted in loss of consciousness.
What is the probable cause of such complication?
A. Blockade of reticular formation
B. Disturbance of coronary blood circulation
C. Blockade of alfa- adrenoceptors*
D. Inhibition of hypothamus
E. Blockade of M- cholinoceptors.

198. After emotional stress, patient was permanently in a condition of


nervous tension and had poor sleep. Doctor prescribed him diazepam. Which
of the listed effects of this drug is responsible for its clinical use?
A. Hypotensive effect
B. Analgesic effect
C. Anticonvulsant
D. Tranquilizing*
E. Anti psychotic.

199. Point out the neuroleptic that is most commonly used in


neuroleptanalgesia.
A. Chlorpromazine
B. Droperidol*
C. Diazepam
D. Chlorprotixene
E. Sulpiride.

200. A patient on a particular psychotropic medication complains of


severe dizziness. His blood pressure in a supine position is 115/80 mm hg on
standing it drops to 82/50 mm hg. Which of the following drugs is most likely
responsible for these symptoms?
A. Carbamazepine
B. Chlorprozamine*
C. Chlordiazepoxide
D. Cortisone
E. Ibuprofen.

201. A patient has been taking a mixture prescribed by neuropathologist


for neurasthenia for two weeks. The patient feels better but has developed
coryza, conjunctivitis, rash, inertia, decrease of memory. She is diagnosed with
bromizm. What should be prescribed to decrease the
A. Glucose solution 5%
B. Polyglucin
C. Asparcam
D. Natrium chloride*
E. Kalium chloride.

202. A patient who has been treated in a neural clinic and has been
taking a sedative for a long time got the following complication: cough, rhinitis,
epiphora. What drug caused these disturbances?
A. Sodium bromide*
B. Valerian
C. Phenazepam
D. Reserpine
E. Diazepam.

203. Select the most appropriate drug for the systematic treatment of a
patient with severe productive psychotic symptoms (delirium and
hallucinations).
A. Trifluoperazine (Triftazin)*
B. Clozapine
C. Medazepam
D. Amitriptyline
E. Buspirone.

204. Neuroleptics have the properties of eliviating delusions,


hallucinations, reducing of aggression and psychomotor agitation. This action is
called:
A. Antipsychotic*
B. Hypodynamic
C. Antihysteric
D. Anxiolytic
E. Antineurotic.

205. A 40 years patient delivered to psychiatric clinic in a state of


agitation, aggression, delusions. What medication should be administered?
A. Sodium bromide
B. Chlorpromazine*
C. Reserpine
D. Tincture of valerian
E. Amitriptyline.

206. The patient with schizophrenia was appointed chlorpromazine.


What the pharmacological effect is the basis for assigning it to the patient?
A. Hypothermic
B. Hypotensive
C. Antipsychotic*
D. Antiemetic
E. Myorelaxant.

207. A patient addressed a doctor complaining of anxiety, fear,


uneasiness, inner tension. Which drug should be assigned?
A. Tincture of valerian
B. Pavlov Medicine
C. Diazepam*
D. Tincture of leonuri
E. Etaminal sodium.

208. On examination of the patient, unstable blood pressure, irritability,


emotional imbalance, poor sleep were revealed, and doctor has prescribed
chlordiazepoxide (elenium). Which of the following statements is ground for
prescription elenium in this case?
A. Psychosis
B. Cardiopathy
C. Hypertention
D. Coronary artery disease
E. Neurosis*.
209. The presence of which syndrome is the basis for the appointment of
diazepam?
A. Lingering paralysis
B. Inadequate perceptions (hallucinations)
C. Euphoric state of pathological origin
D. Anxiety*
E. Pathological thought disorder.

210. Patient 38 years old suffering from schizophrenia for a long time is
on the psychotropic drugs. He addressed the doctor with complaints of
disturbances of motor coordination, hand tremor, drowsiness. Which group of
drugs can cause such a condition?
A. Antipsychotics*
B. Antidepressants
C. Analgesics
D. Adaptogens
E. Psychomotor stimulants.

211. A single parenteral administration of the drug in the patient with


schizophrenia caused severe hypotension. To which of these groups does drug
belong?
A. Nootropics
B. Antipsychotics*
C. Tranquilizers
D. Antidepressants
E. CNS stimulants.

212. A patient with acute myocardial infarction experienced persistent


chest pain. The ineffectiveness of drugs previously designated gave the grounds
for doctor to use neuroleptanalgesia. Which antipsychotic agent is used for this
type of anesthesia?
A. Chlorpromazine
B. Triftazinum
C. Metaperazine
D. Droperidol*
E. Thioridazine.

213. A patient with schizophrenia is taking typical neuroleptic. Which of


the effects is the basis for its application in schizophrenia?
A. Antipsychotic*
B. Antiemetic
C. Hypothermic
D. Myorelaxant
E. Hypotensive.

214. A patient addressed the doctor with complaints of poor health,


depression, skin rashes. An interview revealed that the patient within three
weeks was taking sedative medicine, prescribed by a neurologist. With the
admission of which drugs complaints of patient were associated?
A. Bromides*
B. Lithium
C. Tranquilizers
D. Antipsychotics
E. Preparations of valerianae.

Psychotropic stimulating agents. Antidepressants.


Psychomotostimulants. Analeptics. Nootropic drugs

215. A patient with poisoning caused by carbon monoxide was


administered directly acting analeptic drug. What medicine was used?
A. Codeine
B. Ephedrine
C. Caffeine*
D. Atropine sulfate
E. Lobeline.

216. A premature newborn was apnoic. Directly acting analeptic drug


was given to restore breathing. What medicine was most probably administered
to the patient?
A. Ethymisolum*
B. Phenylephrine (Mesatonum)
C. Adrenaline hydrochloride
D. Atropine sulfate
E. Lobeline hydrochloride.

217. A psychiatrist was invited to a patient who made an attempt to


commit suicide. Doctor diagnosed endogenous depression. Choose the most
suitable drug for treatment of this patient.
A. Nootropil (piracetam)
B. Amytriptyline*
C. Sydnocarbum
D. Ethymisolum
E. Caffeine.

218. A patient had a stroke. Which of the listed drugs is necessary to


include in the complex therapy in order to improve mental performance?
A. Caffeine
B. Piracetam*
C. Diazepam
D. Phenazepam
E. Amitryptiline.
219. What is the main mechanism by which amitriptyline increases
amount of catecholamines in CNS synapses?
A. Increase in catecholamines release from presynaptic membrane
B. Increase in catecholamines synthesis in presynaptic membrane
C. Prevention of catecholamines degradation in the synapse
D. Inhibition of neuronal re uptake of catecholamines*
E. Inhibition of MAO.

220. Analeptical directly acting drug was given to the patient for
restoration of breathing after poisoning with carbon monoxide. What medicine
was prescribed to the patient?
A. Pentamin
B. Mesaton
C. Caffeine*
D. Atropine sulphate
E. Lobeline hydrochloride.

221. Which of the following characteristics of amphetamines is most


likely to be responsible for increasing blood pressure?
A. Indirect release of endogenous catecholamines*
B. Inhibition of catecholamines metabolism
C. Metabolism to false neuro-chemical transmitters
D. Potent alpha1 adrenergic transmitters
E. Potent beta-2 adrenergic agonist.

222. The CNS stimulation produced by methylxanthines, such as


caffeine, is most likely due to the antagonism of one of the following recertors:
A. Glycine receptors
B. Adenosine receptors*
C. Glutamate receptors
D. GABA receptors
E. Cholinergic muscarinic receptors.

223. A patient who has been treated with diazepam on account of


neurosis complains of toothache. Doctor administered him an analgetic, but its
dose was lower then average therapeutic dose. What phenomenon did the doctor
take into account while prescribing the patient an underdose?
A. Drug dependence
B. Cumulation
C. Tolerance
D. Summation
E. Potentation*.

224. Pharmacological effects of antidepressants are connected with


inhibitionof an enzyme catalyzing biogenic amines noradrenaline and serotonine
in the mitochondrions of cerebral neurons. What enzyme participates in this
process?
A. Lyase
B. Monoamine oxidase*
C. Transaminase
D. Peptidase
E. Decarboxylase.

225. A psychiatrist was called for the woman who tried to commit
suicide on examination the state of endogenous depression was diagnosed.
Which drug is most efficient for treatment of this patient?
A. Piracetam (Nootropil)
B. Amitriptyline*
C. Sydnokarb
D. Aethimizolum
E. Caffeine.

226. Which agent will you choose for increase in mental performance
and decrease of tiredness?
A. Buspirone
B. Nialamide
C. Sidnokarb
D. Imipramine
E. Caffeine*.

227. Doctor recommended taking caffeine the patient with pathological


sleepiness.Which of the following mechanisms of action provides its therapeutic
effect in this disease?
A. The concentration and increased inhibition in the cerebral cortex
B. Increased synthesis of dopamine
C. Enhancement the processes of excitation in the brain*
D. Inhibition of the midbrain reticular formation
E. The weakening of the excitation in the brain.

228. Point out preferable analeptic in acute barbiturate poisoning


A. Cordiaminum
B. Caffeine
C. Camphor
D. Bemegride*
E. Aethimizolum.

229. The patient had a stroke. Which of the following drugs should be
included in the combined therapy to improve the metabolism of the brain?
A. Piracetam*
B. Caffeine
C. Diazepam
D. Phenazepam
E. Amitriptyline.
230. Which the drug from the group of psychomotor stimulants can
cause the development of drug addiction?
A. Amphetamin (Phenaminum)*
B. Meridil
C. Sidnokarb
D. Bemithylum
E. Caffeine.

231. Which of the following effects of piracetam is used for chronic


vascular disorders of the brain?
A. Anticonvulsant activity
B. Improvement of mental performance*
C. Antiwithdrawal effect
D. Improvement of physical activity
E. Antipsychotic effect.

232. The patient complains of fatigue, general weakness, decrease in


visual acuty, low blood pressure. Which drug is helpful in this case?
A. Liquid extract of Eleutherococcus*
B. Phenibut
C. Piracetam
D. Dry extract of Valeriana
E. Amitriptyline.

233. The primary goal of medical aid in vascular collapse is an increase


in blood pressure. Which of the following medications can be used for this
purpose?
A. Aethimizolum
B. Diphenhydramine
C. Bemegride
D. Caffeine-sodium benzoate*
E. Ketotifen.

234. A patient admitted to the neurological department complains of


memory loss and cognitive disability after trauma of the head. What kind of
medication is needed to improve the metabolism of the brain?
A. Caffeine
B. Metamizole (Analginum)
C. Piracetam (nootropil)*
D. Sidnokarb
E. Meridil.

235. A psychiatrist was called for the woman who tried to commit
suicide on examination the state of endogenous depression was diagnosed.
Which drug is most efficient for treatment of this patient?
A. Piracetam (Nootropil)
B. Amitriptyline*
C. Sydnokarbum
D. Aethimizolum
E. Caffeine.

236. Which agent will you choose for increase in mental performance
and decrease of tiredness?
A. Nialamide
B. Diazepam
C. Imipramine
D. Buspirone
E. Caffeine*.

237. When should you expect for a specific therapeutic effect of


pantocrinum in its systematic application?
A. After 9-15 days*
B. After 30-60 minutes
C. 1-3 days
D. After 30-45 days
E. After 3-6 hours.

238. Doctor recommended taking caffeine the patient with pathological


sleepiness. Which of the following mechanisms of action provides its
therapeutic effect in this disease?
A. Enhancement the processes of excitation in the brain*
B. Increased synthesis of dopamine
C. The concentration and increased inhibition in the cerebral cortex
D. Inhibition of the midbrain reticular formation
E. The weakening of the excitation in the brain.

239. Point out preferable analeptic in acute barbiturate poisoning


A. Caffeine
B. Bemegride*
C. Camphor
D. Cordiaminum
E. Aethimizolum.

240. The patient had a stroke. Which of the following drugs should be
included in the combined therapy to improve the metabolism of the brain?
A. Caffeine
B. Diazepam
C. Piracetam*
D. Phenazepam
E. Amitriptyline.

241. The primary goal of medical aid in vascular collapse is an increase


in blood pressure. Which of the following medications can be used for this
purpose?
A. Diphenhydramine
B. Bemegride
C. Caffeine-sodium benzoate*
D. Aethimizolum
E. Ketotifen.

242. A patient admitted to the neurological department complains of


memory loss and cognitive disability after trauma of the head. What kind of
medication is needed to improve the metabolism of the brain?
A. Piracetam (nootropil)*
B. Metamizole (Analginum)
C. Caffeine
D. Sydnokarbum
E. Meridil.

243. Which of the following tonics is derived from animal products?


A. Tincture ginseng
B. Pantocrine*
C. Saparalum
D. Ecdistenum
E. Securininum.

244. Among the antidepressant drugs there are those that inhibit
neuronal norepinephrine reuptake. Point out such drug.
A. Fluoxetine
B. Nialamide
C. Chlorprothixene
D. Chlorpromazine
E. Maprotilime*.

245. Mark pharmacological effect, limiting the use of strychnine as


analeptic.
A. Stiffness in skeletal muscle, convulsions*
B. Hypercatharsis
C. Increased metabolism
D. Strengthening of sight and hearing
E. Impact on hematopoiesis.

246. What drug from the group of analeptics is preferable to use in


newborn asphyxia?
A. Caffeine
B. Aethimizolum*
C. Camphor
D. Cordiaminum
E. Corazolum.
247. A 19 years old boy addressed doctor complaining of discomfort in
the heart area. Doctor asked him about smoking and other bad habits. Patient
denied all of mentioned bad habits, and sad he likes coffee and uses up to 8
glasses of strong drink a day. Why coffee containing caffeine can worsen heart
function?
A. It causes tachycardia, increases in oxygen demand of the heart*
B. It slows impulse conduction in the heart
C. It weakens the force of heart contractions
D. It causes a narrowing of the coronary vessels
E. It decreases automaticity of the heart.

248. What is the mechanism of action of antidepressant pirlindol


(pirazidol)?
A. The reversible inhibition of MAO*
B. Irreversible inhibition of MAO
C. Selective inhibition of neuronal capture of catecholamines
D. Non-selective inhibition of neuronal capture catecholamines
E. Inhibitor of vesicular capture catecholamines.

249. A patient with symptoms of mental depression in the drug treatment


did not keep the doctor recommended a diet that caused the rise in blood
pressure. Which group of antidepressants requires compliance in this condition?
A. Tricyclic antidepressant
B. Nonselective MAO inhibitors*
C. Selective inhibitors of catecholamines reuptake
D. Lithium preparations

THE AGENTS INFLUENCE THE EFFECTORY ORGANS

CARDIAC GLYCOSIDES AND OTHER CARDIOTONIC DRUGS

A patient suffers from chronic left-ventricular insufficiency. What


medication should be administered?

A *Digoxin

B Bemegride

C Etimizole

D Vinpocetine

E Pyracetam
A patient that entered the admission office had the following signs of
acute cardiac insufficiency: paleness, acrocyanosis, frequent shallow
respiration. What drug is indicated in this case?

A *Corglycon

B Digitoxin

C Cordiamin

D Nitroglycerine

E Adrenaline hydrochloride

Patient complaines of weakness, dyspnea, low extremities oedema.


Diagnosis: chronic cardiac insufficiency. What medicine should be
prescribed first of all?

A *Digitoxin

B Caffeine

C Papaverine

D Propranolol

E Raunatin

A patient suffering from chronic cardiac insufficiency was recommended


to undergo a prophylactic course of treatment with a cardiological drug
from the group of cardiac glycosides that is to be taken enterally. What
drug was recommended?

A *Digoxin

B Strophanthine

C Corglycon

D Cordiamin

E Cordarone

A 50 y.o. patient with chronic cardiac insufficiency and


tachyarrythmia was prescribed a cardiotonic drug. What drug was
prescribed?

A *Digoxin

B Dopamine
C Dobutamine

D Amyodarone

E Mildronate

1. A patient with pulmonary edema caused by acute left ventricular


insufficiency patient was treated with cardiac glycoside. In 10-15 min, his
condition improved and maximal effect - was in 1-1,5 hours, after that the
action gradually decreased. What drug has been injected?

A. * Strophanthin

В. Digoxin

C. Celanidum

D. Digltoxin

E. Adonisidum

2. A patient had been suffering from heart insufficiency was treated with
digoxinum. He took diuretic Dichlothiazidum without the doctor's
knowledge and after 2 days he felt worse and was obliged to address this
matter to the doctor. Doctor administered several drugs to improve
patient's state and among these drugs was Unithiolum. Indicate the
mechanism of action of this agent

A. *It has got free sulfhydril groups which bind to the molecules of
digoxinum

B. Promotes excretion of calcium ions from the organism

С. Promotes retention of potassium ions in the organism

D. Decrease oxygen demand of the myocardium

E. Creates complexes with calcium ions

3. Specify the cardiac glycoside which posseses the fastest onset of the
action.

A. *Strophanthin

В. Celanidum (lanatoside)

С. Digitoxin
D. Digoxinum

E. Adonisidum

4. What effect of cardiac glycosides is of greatest importance?

A. *Increase of myocardium contractility

B. Increase of myocardium excitability

C. Decrease of myocardium automatism

D. Decrease of myocardium conductivity

E. Increase of diuresis and elimination of edemas

8. Indicate the mechanism of cardiotonic action glycosides

A. *Inhibition of Na-K-ATPase of cardiomyocyte membranes

B. Excitation of B-adrenoceptors of myocardium

С Activation of calcium channels of cardiomyocyte membranes

D Oppression of phosphodiesterase of cardiomyocytes

E. Activation of potassium channels of cardiomyocytes

9. Indicate the group of drugs which is the most expedient for treatment of
ciliary arrhythmia?

A. *Cardiac glycosides

B. M -cholinoblockers

C. Na-channels blockers

D. Beta-adrenomimetics

E. Alfa-adrenoblockers

10. Indicate the group of drugs, overdosage of which is accompanied by


following signs: nausea, vomiting, diarrhea, infringement of heart activity
(extrasystoles, delay of atrioventricular conductivity), headache, vision
impairment (xanthopsia, diplopia).

A. * Cardiac glycosides

B. Organic nitrates

С Ca-channels blockers
D. Beta-adrenoblockers

E. Angiotensin converting enzyme inhibitors

11. Why do strophanthin and corglycon possess fast action after the
introduction into the organism?

A. * They have low affinity to plasma proteins

B. They have high molecular weight

С. They have low molecular weioht

D. They have high affinity to plasma proteins

E. They have short half-life period

ANTIARRHYTHMIC AGENTS.

16. Indicate the group of drugs which is used for treatment of


atrioventricular blockade

A. * M-cholinoblockers

B. Ca-channels blockers

С. Local anaesthetics

D. Beta-adrenoblockers

E. Potassium containing remedies

15. A patient has lengthening of P-Q interval on the electrocardiogram


under the treatment with an antiarrhythmic drug. What agent could cause
it?

A. *Atenolol

B. Prazosin

С. Atropine

D. Lidocaine

E. Plathyphyllin

14. It is necessary to appoint an anti-arrhythmic agent to a patient with


ciliary arrhythmia accompanying by bronchial asthma. What drug from
listed below is contraindicated to this patient?
A. *Anaprilinum (propranolol)

B. Verapamil

С. Aimalin

D. Digoxine

E. Novocainamidum (procainamide)

12. Indicate the mechanism of action of Verapamil

A * Blockade of calcium channels

B. Inhibition of Na-K-ATPase

С Activation of beta-adrenoceptors

D. Activation of M-cholinoceptors

E. Blockade of beta-adrenoceptors

13. A patient suffers from allergic reaction to iodine. Indicate an


antiarrhythmic agent, which is absolutely contraindicated to him.

A. *Amiodarone

В. Verapamil

С. Novocainamidum (procainamide)

D. Ornidum (bretytium)

E. Quinidine sulfate

1. In a clinic, the patient complained of unpleasant sensations in the heart


region, and attacks of weakness and loss of consciousness. Inspection of
the patient's electrocardiogram had revealed the presence of II degree
atrioventricular block. Specify a drug which should be used in this
situation.

A. * Isadrinum

B. Novocainamide

C. Nitroglycerine

D. Stropnanthin

E. Anaplilinum
Indicate the mechanism of anti arrhythmic action of quinidine sulphate

A. *Blockade of sodium channels of the cardiomyocyte membranes

B. Blockade of calcium channels of the cardiomyocyte membranes

С. Blockade of β-adrenoceptors of the myocardium

D. Blockade о α- and β-adrenoceptors of the myocardium

E. Blockade of M-cholinoreceptors of the myocardium

5. Specify the mechanism of anti arrhythmical action of verapamil

A. *Blockade of calcium channels of the cardiomyocyte membranes

B. Blockade of sodium channels of the cardiomyocyte membranes

C. Blockade or β-adrenoceptors of the myocardium

D Blockade of α- and β-adrenoceptors of the myocardium

E. Blockade of М-сholinoreceptors of the myocardium

ANTIANGINAL DRUGS

A patient with coronary artery disease was admitted to the cardiological


department. For stenocardia prevention a drug from the group of beta-
adrenoceptor blockers was administered. What drug is it?

A *Metoprolol

B Atropine sulfate

C Morphine hydrochloride

D Oxytocin

E Furosemide

After a tooth extraction a patient felt persistent pain behind his breast
bone. After sublingual intake of an antianginal drug the pain behind the
breast bone disappeared, but the patient complained of headache and
dizziness. What drug are these properties typical

for?

A *Nitroglycerin
B Propranolol

C Metoprolol

D Validol

E Verapamil

A patient who had myocardial infarction was administered 75 mg of


acetylsalicinic acid a day. What is the purpose of this administration?

A *Reduction of thrombocyte aggregation

B Inflammation reduction

C Pain relief

D Temperature reduction

E Coronary vessel dilatation

A patient suffers from stenocardia and takes isosorbide mononitrate. He


was prescribed a complementary drug with disaggregating effect. What
drug is it?

A *Acetylsalicinic acid

B Nitroglycerine

C Propranolol

D Nifedipine

E Validol

A patient suffering from coronary artery disease had taken a certain drug
many times a day in order to arrest stenocardia attacks. Overdose of this
drug finally caused intoxication. Objectively: cyanotic skin and mucous
membranes, dramatic fall in the arterial pressure, tachycardia, respiration
inhibition. Blood has increased concentration of methemoglobin. The
drug the patient had taken relates to the following group:

A *Organic nitrates

B alpha-adrenoceptor blockers

C Calcium channel blockers


D Adenosine drugs

E Myotropic spamolytics

Anapriline therapy caused positive effect in the dynamic of the disease of


a 44-year-old woman suffering from stenocardia. What is the main
mechanism of the effect of this medicine?

A *Blockade of beta-adrenoreceptors and decrease myocardial


requirements to the oxygen.

B Decrease of oxidative exchange in myocardium due to enzyme


blockade of Krebs' cycle

C Decreased power inputs of myocardium due to reduced loading

D Increased oxygen supply to the myocardium

E Decreased need in increasing of oxygen supply to the myocardium

1. Determine an antianginal agent by its pharmacological effect. It


insignificantly improves coronary circulation (especially in
subendocardial layers), but the main cause of elimination of stenocardia is
dilation of peripheral veins and arteries, that leads to decrease of cardiac
work and myocardial oxygen demand. Besides, it oppresses the central
links of cardiostimulating reflexes.

A. * Nitroglycerine

B. Phenihydinum

С Amiodaronum

D. Validolum

E. Anaplilinum

2. A patient who had been suffering from stenocardia accompanied by


cardiac arrhythmia (paroxysmal tachycardia) and arterial hypertension
was admitted to the hospital. Specify antianginal drug and the group
which it belongs to, which should be admin¬istered taking into account
the patient's diseases.

A. *β-adrenoblocker - anaprilinum

B. Organic nitrate - nitroglycerinum


C. Myotropic spasmolytic agent – No-spa

D. Calcium antagonist - nifedipin

E. Potassium channel activator nicorandil

3. Indicate the nitroglycerin's drug with prolonged action

A. *Sustac

B. Validolum

С. Amylnitrite

D. Natrium nitroprussid

E. Dipiridamolum

4. Indicate the drug which exerts antianginal action because of decrease of


oxygen demand and increase of oxygen delivery to the myocardium.

A. *Nitroglycerinum

B. Anaprilinum

C. Dipiridamolum

D. Talinololum

E. Carbocromenum

5. Indicate the state which requires administration of nitroglycerine

A. *An attack of stenocardia

B. Acute cardiac failure

С Hypertensive crisis

D. Chronic cardiac insufficiency

E. Endarteritis obliterans

6. Indicate the mechanism of action of nitroglycerine

A. * Release of NO groups which activate guanylyl cyclase

B. Blockade of calcium channels

C. Blockade of adenosine receptors

D. Activation ol adenylyl cyclase


E. Inhibition of phosphodiesterase

7. А patient with ischemic heart disease complaints of worsening of his


state that is caused by overdosage of antianginal agent. What group of
drugs can cause this state it it is known that concentration of
methemoglobin in patient’s blood is increased?

A. *Organic nitrates

B. Beta-adrenoblockers

С. Blockators of calcium channels

D. Activators of potassium channels

E. Myotropic coronary dilators

8. After sublingual introduction of nitroglycerine its maximal


concentra¬tion in blood is developed in:

A. * 4-5minutes

B. 15minutes

С. 1 minute

D. 30 minutes

E. 1 hour

9. Why are the tablets of nitroglycerine introduced sublingually only?

A. *The substance is being considerably destroyed during its first passage


through the liver

B. The substance is badly absorbed in the gastrointestinal tract

С The 'substance operates reflexly from oral cavity receptors

D. The substance is destroyed under the action of gastric Juice

E. It causes less side-effects in such way of introduction

10. Determine an antianginal agent according to its pharmacological


effects: dilating coronary arteries it increases myocardial blood supply,
dilating peripheral veins it decreases myocardial preload, dilating
perypheral arteries it decrease myocardial postload, besides it oppresses
the central links of coronaroconstrictive and cardiostimulatory reflexes:

A. *Nitroglycerine
B. Fenigidinum

С. Amiodarone

D. Validolum

E. Anaprilinum

11. Introduction of an antianginal drug to a patient with stenocardia


caused improvement of patient's state and also arterial hypotension,
tachycardia and throbbing headache. Indicate this drug.

A. *Nitroglycerine

B. Carbocromen

С. Dipyridamole

D. Mildronate

E. Verapamil

12. A 50 years old patient has suffered from angina pectoris for several
months. As a rule he has successfully used a tablet of validolum during
the attack but last 2 weeks this remedy hasn’t been effective. What drug
should be administered to the patient for elimination of the attack?

A. * Nitroglycerine

B. Nifedipine

С Verapamil

D. Anaprilinum (propranolol)

E. Isosorbide mononitrate

13. The calcium channels of cardiomyocytes have been blocked on an


isolated rabbit's heart. What changes in the heart's activity can happen as a
result?

A. *Decrease rate and force of heart beat

B. Heart stops in systole

C. Decrease of heart beat rate

D. Decrease force of the contraction

E. Heart stops in diastole


HYPO- AND HYPERTENSIVE DRUGS

A patient with hypertensic crisis was admitted to the cardiological


department, he was injected intravenously with an antihypertensive drug -
salt of an alkaline-earth metal. What drug was injected?

A *Magnesium sulfate

B Potassium chloride

C Sodium hydrocarbonate

D Calcium lactate

E Benzohexamethonium

A patient with II stage hypertension has been taking one of hypotensive


medications for the purpose of treatment. After a time arterial pressure
decreased, but the patient started complaining of flaccidity, sleepiness,
indifference. A bit later he felt stomach pain. He was diagnosed with
ulcer. What hypotensive medication has the patient been taking?

A *Reserpine

B Dibazole

C Furosemide

D Verapamil

E Captopril

A 66 year old female patient got intravenous injection of magnesium


sulfate solution for the purpose of elimination of hypertensive crisis. But
arterial pressure didn't go down and after repeated introduction of the
same preparation there appered sluggishness, slow response, inhibition of
consciousness and respiration. What preparation is antagonist of
magnesium sulfate and can eliminate symptoms of its overdose?

A Calcium chloride

B Potassium chloride

C Sodium chloride

D Activated carbon
E Potassium permanganate

A 63 y.o. man with collapse symptoms was delivered to the


emergency hospital. A doctor

chose noradrenaline in order to prevent hypotension. What is the


action mechanism of this

medication?

A *Activation of alpha-1-adrenoreceptors

B Activation of serotonin receptors

C Activation of beta-adrenoreceptors

D Activation of dopamine receptors

E Block of M-cholinoreceptors

1. The patient suffering from arterial hypertension with hyperkinetic type


of circulation and the high contents of rennin, accompanied by stenocardia
and sinus tachycardia has been treating for 10 years. Indicate the group of
drugs should be administered in this situation.

A. * Beta-adrenoblockers

B. Drugs of nitroglycerine

C. Alfa-adrenoblockers

D. Sympatholytics

E. Ganglioblockers

2. A 45 year old patient, who had been suffering from idiopathic


hyper¬tension, was treated by an antihypertensive drug. After 4 days his
arterial pressure decreased, but he complained of sleepiness and
psychological suppresion. With which drug was the patient treated?

A * Clophelinum

B. Prazozinum

С. Captopril
D. Enalapril

E. Apressine

3. A patient who had been suffering from hypertonic disease had been
treated for a long time with the drug from the group of Rauwolf ia
alkaloids and began to complain of heartburn, pain in the epigastrial area
and bad mood. Indicate the drug which caused these complications.

A. *Reserpinum

В. Octadinum

С. Clophelinum

D. Papaverinum

E. Dlbazolum

4. A patient who had been suffering from arterial hypertension had taken a
hypotensive drug, but in an hour his blood pressure increased and 2 hours
after it decreased. Indicate this antihypertensive agent.

A *Octadimlm

B. Prazosinum

С. Captoprii

D. Anaprilinum

E. Nifedipinum

5. A patient had been suffering from hypertonic disease accompanied by


chronic bronchitis with asthmatical component. Indicate the drug which is
contraindicated due to it's action on the bronchi.

A *Anaprillinum

B. Captopril

С. Prazosinum

D. Nifedipine

E. Dichlothiazidum

6. A doctor has administered to a patient clonidine (clophelinum) for


elimination of hypertensive crisis. What class of hypotensive drugs does
the named agent belong to?
A * Central neurotropic

В. Peripheral neurotropic

С. Diuretics

D. Drugs affecting the renin-angiotensin system

E. Myotropic (vasotropic) hypotensive agents

7. A patient with hypertensive disease caused by raised sympathoadrenal


system activity requires administration of a drug reducing neurogenic tone
of vessels. What is the most suitable agent to be administered?

A. *Clophelinum

B. Losartane

С. Verapamil

D. Hydrochlorthiazide

E. Apressinum (hydralazine)

8. Hypertensic crisis characterized by sharp headache, dizziness,


hyperemia of face, pains in the region of heart, rapid pulse, arterial
pressure of 220/110 mm Hg has developed in a patient suffering from
essential hypertension during the visit to the dentist. What agent is it
necessary to introduce to the patient?

A. *Clophelinum (clonidine)

В. Pinlenum fpempidine)

С. Timolol

D. Moxonidine

E. Anaprilinum fpropranolol)

9. Indicate the antihypertensive agent which can cause such side-effects as


dryness in the mouth, constipation and retention of water in the organism

A. * Clophelinum (clonidine)

B. Coraiaminum (nikethamide)

С. Verapamil

D. Anaprilinum fpropranolol)
E. Nifedipine

10. Stable arterial hypertension was arisen in the patient who had been
suffering from chronic glomerulonephritis. Indicate the most effective
group of drugs to treat this patient.

A. *Angiotensin converting enzyme inhibitors

B. Ganglion blockers

С. α-adrenoblockers

D. Myotropic spasmolytics

E. Calcium antagonists

THE AGENTS USED IN INSUFFICIENCY OF CEREBRAL


CIRCULATION. HYPOLIPIDEMIC AGENTS

1. The patient who had been treated with a vitamin drug for the
prophylaxis of brain vessel constriction complained of unpleasant
sensations: blushing of upper part of the body, vertigo, flushing of blood
to the head. Which drug exerts this effect

A. * Nicotinic acid

В. Tocopherol acetate

C. Riboflavin

D. Thiamini bromidum

E. Calcium pangamate

2. A patient suffering from atherosclerosis is treated with lovastatinum


0,04 g PO before sleep. Why is this drug administered once a day and
before sleep?

A. *Cholesterol is synthesized only at night

B. Development of sleepiness in the action of the drug.

С. In the evening the drug is better absorbed.

D. The cataboiism of a cholesterol goes mainly at night.

E. The cholesterol is excreted from an organism mainly at night


3. A patient admitted to the neurology department with complaints of
severe headache, nausea, vomiting, feeling of numbness and weakness of
the right arm, disorders of speech. BP – 220/130 mm Hg. During 15 years
he had been suffering from arterial hypertension. After the examination
the following diagnosis was made: ische¬mia of the left hemisphere due
to vascular spasm and impairment of venous outflow. Specify the drug
which is the most preferable for the improvement of cerebral blood supply
due to decrease of hypercoagulation.

A. *Xanthinoli nicotinas

В. Sydnophenum

С. Coffetnum

D. Meridilum

E. Aminalonum

4. A patient admitted to a hospital with complaints of decrease of


memory, feeling of "noise" in the head. The diagnosis which had
estab¬lished after examination was atherosclerosis of brain blood
vesseles. Indicate the agent which can be administered to the patient.

A. *Clofibratum

B. Euphyllinum

С Dibazolum

D. Nimodipin

E. Minoxidilum

5. A 58 year old woman suffered from cerebral atherosclerosis. The


complex therapy administered by the physician included vitamins E and
C. Indicate the role of these drugs in the treatment of atherosclerosis.

A. *Inhibition of lipids' peroxydation

B. increase of release of the pituitary gonadotropic hormones

С. Decrease of release of glucocorticoids in adrenal cortex

D. Activation of the antitoxic function of the liver

E. Improvement of coronary circulation


6. A patient was admitted to the clinic of nervous diseases with increased
arterial pressure (220/130 mm Hg). It was diagnosed that he had ischemia
of the left hemisphere of the brain as a result of vessel spasm and
impairment of venous outflow. Choose the preparation from the listed
ones which is preferable for improvement of brain blood supply and
which removes hypercoagulation in an acute period of the illness:

A. * Xantinol nicotinate

B. Caffeine (coffeinum)

С. Meridilum (methylpnenidate)

D. Aminalonum (gamma-aminobutyric acid)

E. Sydnophenum (pheprosidine)

7. Indicate the main effect of Piracetam

A. * Improves memory and cognition

B. Decreases the integrating processes in the brain

С. Slows down synthesis of GABA in the brain

D. Reduces resistance of the brain tissue to hypoxia

E. increases brain necessity in oxygen

8. A patient was admitted to the neurological department complaining of


memory impairment and decrease of intellectual capacity after the car
crash head trauma. Offer the remedy for improvement of metabolism in
the brain:

A. * Pyracetam (Nootropil)

B. Nifedipinum

С. Sydnocarbum

D. Caffeine (coffeinum)

E. Analginum (metamizole)

9. What class of lipoproteins is the most atherogenic?

A. * Low density lipoproteins

B. Chylomicrons
С. High density lipoproteins

D. Very low density lipoproteins

E. Intermediate density lipoproteins

10. Indicate the principle of an antiatherosclerotic action of Lovastati-num

A. *Oppression of endogenous cholesterol synthesis in the liver

B. Inhibition of peroxide radicals formation

С. Infringement of exogenous cholesterol absorption

D. Inhibition of lipolysis in fatty tissue

E. Prevention or penetration of atherogenic lipoproteins in tunica intima


of vessels

11. A patient with atherosclerosis of vessels was treated with one of the
hypolipidemic drugs which reduces cholesterol synthesis due to inhibition
of enzyme 3-hydroxy-3-methylglutaryl-coenzyme A reductase. Indicate
the drug

A. * Lovastatinum

B. Cholestyramine

С. Clofibrate

D. Nicotinic acid

E. Probucol

12. Among special hypolipidemic agents the most effective ones are those
which block synthesis of endogenous cholesterol in the liver. What drug
from listed below has such mechanism of action?

A. *Lovastatinum

B. Clofibrate

С. Cholestyramine

D. Parmidinum

E. Probucol
13. A patient has the family hypercholesterolemia. Indicate the drug
which may be used due to ability to inhibit the main enzyme of
cholesterol synthesis?

A. * Lovastatinum

B. Colestipol

С. Cholestyramine

D. Nicotinic acid

E. Probucol

15. Indicate the hypolipidemic agent which may be used in atherosclerosis


of brain arteries

A. *Lovastatinum

B. Cinnarizine

С. Pyracetam

D. Tocopherol acetate

E. Ascorbinic acid

16. Specify the principle of antihy-perlipidemic action of lovastatinum.

A. *Inhibition of synthesis of endogenous cholesterol in the liver

B. Impairment of creation of superoxide radicals

С. Impairment of absorption of cholesterol in the intestine

D. Impairment of lipolysis in the fatty tissue

E. Impairment of binding of atherogenous lipoproteins with


endotheliocytes

DIURETIC AGENTS. DRUGS USED IN GOUT.

A patient takes digoxin for treatment of cardiac insufficiency. What


diuretic may increase digoxin toxicity due to the intensified excretion of
K+ions?

A *Hydrochlorothiazide

B Spironolactone
C Panangine

D Siliborum

E Lisinopril

A patient with chronic cardiac insufficiency has been treated with


cardiotonic drugs and a thiazide diuretic, but in spite of it there are still
edemata and risk of ascites. What medication should be prescribed to
amplify diuretic effect of the applied drugs?

A *Spironolactone

B Furosemide

C Amyloride

D Clopamide

E Manitole

The alternate usage of dichlotiazide, etacrin acid and lasix didn't cause
marked diuretic effect in patient with marked peripheral edema. Increased
amount of aldosterone is in the blood. Indicate the medicine to be
prescribed?

A *Spironolacton

B Mannit

C Clopamid

D Urea

E Amilorid

Diuretic drug was prescribed to the patient with hypertension in the course
of complex treatment. In a few days BP decreased but signs of
hypokaliemia developed. What drug could cause such complications?

A *Lasix

B Spironolactone

C Clophelin

D Triamterene

E Enalapril
A 65 year old female patient suffers from chronic renal insufficiency
accompanied by evident edemata caused by chronic glomerulonephritis.
What diuretic should be administered for forced diuresis?

A *Furosemide

B Hydrochlorothiazide

C Chlorthalidone

D Cyclometazide

E Acetazolamide

1.The diuretic agent in dosage 0,025 g 2 times a day had been prescribed
to the patient with the beginning stage of idiopathic hypertension. In 7-8
days, the arterial pressure had slightly decreased, but he began to
complain of pain in the heart region, muscle weakness, and tremor. The
analysis of blood has revealed hypokalemia. Which from the listed drugs
may cause this side effect

A. * Hydrochlorthiazide

B. Spironolactonum

С Triamterenum

D. Amilorid

E. Mannit

3. A patient had taken celanidum for long time due to chronic heart
failure. The physician administered to him dichlothiazidum to eliminate
leg edemas. Which drug should be taken together with the diuretic to
prevent hypokalemia?

A. *Kalii chloridum

B. Calcii chloridum

С. Unithiolum

D. Natrii sulfas

E. Magnii sulfas
4. A patient has been treated for a long time with cardiac glycoside
digoxinum in connection with congestive heart failure. Now the patient's
state is stable, but there are remaining edemas on the legs and face. What
diuretic should be taken to avoid side-effects caused by simultaneous
administration of cardiac glycosides and diuretics?

A. *Spironolactonum

B. Oxodolinum

C. Dlchlothiazidum

D. Diacarbum

E. Cyclomethiazidum

6. The usage of dichlotiazide, etacrinic acid and furosemide did not cause
marked diuretic effect in the patient with marked peripheral edemas. The
aldosterone level in the blood is increased. Indicate which medicine
should be prescribed:

A. *Spironolactone

B. Mannit

C. Amilorid

D. Clopamid

E. Urea

7. A doctor administered Allopurinol to a 26-year-old man with the


symptoms of gout. What pharmacol¬ogical action of Allopurinol ensures
therapeutical effect?

A. *Inhibition of uric acid synthesis

B. Increase of uric acid excretion

C. Inhibition of leucocyte migration into the joint

D. Analgesic effect

E. Antinflammatory effect

8. Indicate the diuretic agent which should be used to treat pulmonary


edema

A. *Furosemide
B. Hydrochlorthiazide

С. Triamteren

D. Spironolactone

E. Acetazolamide (diacarbum)

THE AGENTS AFFECTING THE MYOMETRIUM TONE AND


CONTRACTILE ABILITY.

A 45 year old woman had referred to her gynaecologist with complaints


of menorragia. The doctor had administered ergometrinum, which helped
the woman. Specify the main effect of the drug.

A. * Produces spastic contraction of the myometrium

B. Accelerates process of coagulation

С Oppresses proliferation of the endometrium cells

D Produces vasoconstriction endotheliocytes

E. Produces rhythmic contractions of the myometrium

5. 1 ml of an agent causing contraction of the myometrium was introduced


intramuscularly to a woman after abortion. In few minutes she began to
complain of headache. AP -160l100 mm Hg. Earlier she had initial stage
of arterial hypertension. Choose among the following drugs which is
preferred in this case, taking into account the woman had inclina¬tion to
arterial hypertension

A. *Oxytocin

B. Pituitrinum

С. Serotoninl adlpinas

D. Hyphitocinum

E. Mammophysinum

THE AGENTS AFFECTING FUNCTIONS OF THE RESPIRATORY


SYSTEM
A patient suffering from chronic bronchitis takes a synthetic mucolytic
drug that facilitates the sputum thinning. What drug is it?

A *Acetylcysteine

B Diazepam

C Heparin

D Furosemide

E Enalapril

A 70-year-old man, who suffered from chronic bronchitis, was prescribed


medicine for the cough - codeine. What is the mechanism of anticoughing
effect?

A *Central

B Reflex

C Competitive

D Local effect

E Peripheral effect

1. А patient had been suffering from chronic bronchitis, was treated with
an expectorant drug. In a week the symptoms of rhinitis, tearing, itching
of the skin and rashes appeared. What agent may cause these side effects?

A. * Potassium iodide

B. Terpinhydratum

С Acetylcysteinum

D. Infusum herbae Thermopsidis

E. Alatriihydrocarbonas

2. A 40 year old patient has been suffering from bronchial asthma,


accompanied with cardiac arrythmia (tachycardia) for 10 years. Indicate
adrenomimetic which should be administered for treatment taking into
account accompanied disease.

A. *Salbutamol
B. Adrenaline

С. Isadrinum

D. Orciprenalin

E. Ephedrine

3. Indicate broncholytic which should be administered to a patient


suffering from bronchial asthma accompanied with stenocardia.

A *Salbutamol

B. Ephedrine

С. Isadrinum

D. Orciprenalin

E. Adrenaline

4. A child was born with asphyxia. What agent is it necessary to introduce


for stimulation of breath?

А. Promedolum

B.*Aetlumlzole

C. Prazozin

D. Atropine

E. Proserinum

5. To specify an analeptic which possesses sedative activity and can be


used as desensibilizing agent in bronchial asthma

A. *Aethimizole

B. Camphor

С. Cordiaminum

D. Carbogen

E. Dimedrolum

6. Indicate antitussive agent possessing properties of opioid І analgesics

A.*Codeine

В. Libexinum
C. Tussuprex

D. Glaucine

E. Falimlnt

7. An antitussive agent (1 tablet 3 times a day) was administered to a


patient. Cough has decreased but the patient has started complaining of
dizziness, general weakness and arterial hypotension has been' revealed.
Indicate the drug.

A. *Glaucine

В. Codeine

С Libexinum (prenoxdiazine)

D. Oxeladine

E. Faiimint

8. Indicate the drug oppressing a peripheral link of cough reflex

A. * Libexinum

В. Codeine phosphate

С. Ethylmorphine hydrochloride

D. Bromhexinum

E. Atropine suifate

9. Thes mechanism of expectorant' action of Thermopsis herb infusion is:

A. *It stimulates bronchial secretion reflexively irritating the stomach


receptors

B. Directly stimulates peristalsis of bronchial smooth muscles

С. It destroys proteins of sputum

D. Irritates bronchial glands during excretion that leads to stimulation of


their secretion

E. It inhibits the cough center

10. Indicate an expectorant agent possessing the reflex type of action

A, * Thermopsis herb infusion


B. Bromhexinum

С. Acetylcysteine

D. Trypsine

E. Potassium iodide

11. A patient with acute bronchitis I suffers from intolerable dry cough.
What from enumerated below agents 'can transform dry cough into wet
cough?

A. *Thermopsis grass infusion

B. Codeine phosphate

С. Libexinum

D. Glaucine hydrochloride

E. Falimint

12. Indicate an expectorant agent that is an inorganic substance and is


usually used orally as a solution, rarely as an inhalation, ft exerts direct
irritating action on bronchial glands.

A. *Kalium iodide

В. Bromhexinum

C. Trypsin

D. Acetylcysteine

E. Libexinum

13. Mark the group of drugs used for elimination of bronchial asthma
attacks

A. *Beta-adrenomimetics

B. M-chotinomimetics

С. Sympatholytics

D. Beta-adrenoblockers

E. M-chotinomimetics
14. A patient with bronchial asthma was treated with the combined drug
in tablets that caused insomnia, irritability, headache and rise of arterial
pressure. What agent could cause these side-effects?

A. *Ephedrine

B. Adrenaline

C. Libexinum

D. Euphyliinum

E. Furosemide

15. A patient suffering from bronchial asthma was treated with the drug
which caused in several days insomnia and tachycardia. Indicate this drug.

A. * Ephedrine

B. Plathyphylline

С. Adrenaline

D. Euphyllinum (aminiphylline)

E. Salbutamolum

16. Indicate the mechanism broncholytic effect of adrenaline

A. * Stimulation of beta2-adrenoceptors

B. Stimulation of beta1 and beta2-adnenoceptors

С Blockade of beta2-adrenoreceptors

D. Stimulation of alfa1 and alfa2-adrenoceptors

E. Blockade of N-cholinoceptors

17. Indicate the diuretic agent which should be used to treat pulmonary
edema

A. *Furosemide

B. Hydrochlorthiazide

С. Triamteren

D. Spironolactone

E. Acetazolamide (diacarbum)
18. Indicate the drug used for elimination of pulmonary edema caused by
systemic arterial hypertension.

A. * Pentaminum

B. Strophanthin

С Bemegridum

D. Cordiaminum

E. Spiritus aethylicus

19. Indicate the drug used in pulmonary edema accompanying by


formation of foam to decrease superficial tension of bubbles to turn foam
into

A. *Spiritus aethylicus

B. Pentaminum

С. Strophanthin

D. Bemegridum

E Cordiaminum (nikethamide)

THE AGENTS INFLUENCING FUNCTIONS OF THE DIGESTIVE


SYSTEM

A 37-year-old man was admitted to the surgical department with


symptoms of acute pancreatitis: vomiting, diarrhea, bradycardia,
hypotention, weakness, dehydration of the organism. What medicine
should be used first of all?

A *Contrycal

B No-spa

C Platyphylline

D Etaperazine

E Ephedrine

A patient suffering from chronic hyperacidic gastritis takes an antacid


drug for heartburn elimination. After its ingestion the patient feels better
but at the same time he has a sensation of stomach swelling. Which of the
following drugs might be the cause of such side effect?
A *Sodium hydrocarbonate

B Magnesium oxide

C Magnesium trisilicate

D Aluminium hydrooxide

E Pepsin

A liquidator of a breakdown at a nuclear power plant who was irradiated


complained about vomiting that occurs all of a sudden. What medication
should be prescribed?

A *Metoclopramide

B Reserpine

C Atropine

D Aeron

E De-Nol

1. The patient was admitted to the hospital with the diagnosis: peptic ulcer
of the duodenum bulbus. Analysis of his gastric juice revealed increased
acidity. Choose the agent which decreases the secretory ability of gastric
glands due to blockade of H2-histaminic receptors.

A. *Panitidinum

B. Extract of belladonna

C. Atropinum

D. Methacinum

E. Platyphytlinum

2. A 25 year-old man, suffering from peptic ulcer of the stomach, has


been treated with omeprasole. In 3 weeks the ulcer was healed. What
mechanism of action does this drug produce?

A. * Blockade of H+-K+-ANP-ase (the proton pump)

B. Blockade of M-cholinoceptors

С. Blockade of synthesis of Gastrin

D. Blockade of H+-K+-ANP аsе


E. Blockade of H1 histaminic receptors

3. Patient who had been suffering from chronic gastritis was treated with
an antacidic drug, after introduction of which he felt better however at the
same time he experienced bloating of the stomach together with
eructation. Indicate the drug which might cause this side effect.

A. *Natrii hydrocarbonas

В. Magnesu trisilicate

С. Magnesii oxydum

D. Almagel

E Aluminii hydroxydum

4. Indicate the drug to stimulate appetite, mechanism of action of which is


associated with irritation of the mucus membrane of the oral cavity, that
leads to reflex excitation of the hunger center in the hypothalamus.

A. *Absinthium tincture

В. Phepranonum

C. Desopimonum

D. Fenfluramine

E. Insulin

5. Indicate the drug which increases appetite due to decrease of glucose


concentration in the blood

A. *Insulin

B. Mazindolum

С. Fenfluramine

D. Absinthium tincture

E. Phepranonum

6. A 32 year old patient had been suffering from the ulcer of the
duode¬nal bulb was treated with Farmotidin which caused him to feel
better. Indicate the mechanism of action of this agent.

A. *Blockade of Н2-histaminic receptors


B. Inhibition of gastrin release

С. Suppression of the function of the gastric mucousal cells

D. Decrease of release of hydrochloric acid

E. Decrease of pepsin release

7. A patient suffering from chronic hypoacidic gastritis with remained


secretory function requires administration of an agent which is
physiological stimulant of the gastric glands. Indicate this agent

A. *Carbonaceous mineral water

B. Pepsin

С. Histamine

D. Diluted hydrochloric acid

E. Natural gastric juice

8. A patient with essential hypertension was treated for a long period of


time with preparations containing reserpin. During last 2-3 months he
started to suffer from pains in the region of stomach, heartburn and
nausea. The diagnosis of hyperacidic gastritis was made after the clinical
examination. Indicate the group of drugs which possesses etiotropic
curative action in this case.

A. *M-cholinoblockers

B. Astringent agents

С. Antiacidic agents

D. Inhibitors of proton pump

E. H2-histamine receptors blockers

9. Indicate the remedy increasing bile secretion:

A. *Oxaphenamidum

B. Apomorphine

С. Cimetidine

D. Almagel

E. No-Spa (drotaverine)
10. Indicate the agent which stimulates contraction of gall bladder smooth
muscle and causes evacuation of bile into the intestine?

A. *Magnesium sulfate in enteral introduction

B. Magnesium sulfate in parenteral introduction

С. Dehydrocholic acid

D. Legaion (silimar in)

E. No-Spa (drotaverine)

11. Indicate the agent which may be used in an attack of biliary colic to
relax smooth muscles?

A. *Platyphyllinum

B. Paracetamolum

С. Analginum (metamizole)

D. Pentazocine

E. Morphine

12. A 40 years old patient was admitted to the hospital with the biliary
colic attack. What agent should be administered in this case?

A. *No-spa (drotaverine)

B. Almagel

С Pancreatin

D. Contrycal (aprotinine)

E. Metoclopramide

13. Indicate the drug from the group of myotropic spasmolytics which is
suitable to eliminate pain in intestinal colic

A. *Papaverine

B. Neostigmine (proserinum)

C. Piridostigmine

D. Pilocarpine

E. Prazosine
14. Indicate a cholagogue agent used for treatment of chronic cholecystitis

A. *Allocnolum

B. Absinthium (sagebrush) tincture

С. Metoclopramide

D. Almagel

E. Plathyphyllin

15. Specify an agent from the group of hepatoprotectors which restores


normal structure and function of hepatocytes, used in different liver
deseases.

A. * Essentiale

B. Tetracycline

С. Cholenzymum

D. Tocopherol acetate

E. Allocholum

16. Indicate the drug, which is used in chronic pancreatitis, accompa¬nied


by enzymesT insufficiency, for improvement of digestion processes.

A. *Festalum

B. Pepsin

С Acidin-pepsinum

D. Natural gastric Juice

E. Diluted nydrocnloric acid

17. Why is contrycal (aprotinine) used in the case of acute pancreatitis?

A. *It inactivates trypsin which causes autolysis of pancreas

B. It opens Oddies sphincter

С. It reduces the activity of hyaluronidase

D. It impairs secretion of trypsinogen

E. It oppresses secretion of bile


18. Indicate an anti-enzymatic agent inhibiting activity of trypsin,
kallikrein and fibrinolysis

A. *Confrykal (aprotinine)

B. Cholenzymum

E. Pancreatin

D. Pancreozymin

E. Festal

19. A 37-year-old man was admitted to the surgical department with the
symptoms of pancreatitis: vomiting, diarrhea, bradycardia, hypotension,
weakness, dehydration. What medicine should be used first of all?

A. *Contrycal

B. Etaperazine

C. No-spa

D. Platyphylline

E. Ephedrine

THE AGENTS INFLUENCING THE BLOOD COAGULATION,


ERYTHRO- AND LEUCOPOIESIS.

Before tooth extraction a patient was advised to take a certain drug for
haemorrhage prevention. What drug was advised?

A *Vicasolum

B Heparin

C Asparcam

D Magnesium sulfate

E Dimedrol

A patient ill with essential hypertension was recommended a drug that


prevents thrombosis. It is to be taken parenterally. What drug is it?

A *Heparin

B Amben
C Protamine sulfate

D Neodicumarin

E Syncumar

1. The patient was admitted to the traumatologic department due to


fracture of chin bones, damages of soft tissues and massive bleeding.
Examination revealed paleness of the skin, pain in palpation of area of
trauma, swelling of the skin, bleeding on the whole surface of the wound.
Specify a drug for local use to stop the bleeding.

A. * Thrombinum

B. Calcii chloridum

С. Vikasolum

D. Aminocapronic acid

E. Ambenum

2. Drugs delaying blood coagulation (anticoagulants) are used for


prevention and treatment of thrombosis. Specify an anticoagulant which
antagonist is protamine sulfate.

A. * Heparinum

B. Meodicoclmarin

C. Syncumarum

D. Phenilinum

E. Sodium hydrocitrate

3. A patient was delivered to a hospital with complaints of loss of


appetite, decrease of body weight, fatigue, pain around the epigastric area.
Examination of the blood revealed megaloblastic anemia. Specify the
main agent for the treat¬ment of this disease.

A. *Cyanocobalaminum

B. Ferri lastas

C. Folic acid

D. Fercovenum

E. Coamidum
4. Specify the antagonist of the anticoagulants with indirect action.

A. *Vikasolum

B. Fercovenum

С Pentoxylum

D. Protamini sulfas

E. Contrykalum

5. Specify the coagulant agent available for local use only (to stop
bleedings from small blood vessels).

A. *Hemostatic sponge

B. Vikasolum

С Calcii chtoridum

D. Fibrinogen

E. Aminocapronic acid

6. Specify the thrombolytic agent which belongs to with direct action


plasma proteins.

A. *Fibrinolysin

В. Streptokinase

C. Contrycalum

D. Urokinase

E. Streptodecase

7. Inhibition of leukopoiesis is observed in a 43 years old roentgenologist.


The amount of leukocytes 3,5*109/l. Specify the agent to be used for
correction of leukopoiesis.

A. *Pentoxylum

B. Ferroplexum

С. Hemostimulinum

D. Cvanocobalaminum

E. Ascorbic acid
VITAMINS

In order to accelerate healing of a radiation ulcer a vitamin drug was


administered. What drug is it?

A *Retinol acetate

B Retabolil

C Prednisolone

D Levamisole

E Methyluracil

A 64 year old woman has impairment of twilight vision (hemeralopy).


What vitamin should be recommended in the first place?

A *Vitamin A

B Vitamin B2

C Vitamin E

D Vitamin C

E Vitamin B6

A patient presents with twilight vision impairment. Which of the


following vitamins should be administered?

A *Retinol acetate

B Cyanocobalamin

C Pyridoxine hydrochloride

D Ascorbic acid

E Nicotinic acid

A patient suffers from vision impairment - hemeralopy (night


blindness). What vitamin

preparation should be administered the patient in order to restore his


vision?

A *Retinol acetate
B Vicasol

C Pyridoxine

D Thiamine chloride

E Tocopherol acetate

1. A 64 year old woman suffers from night blindness (hemeralopia). What


vitamin drug should be recommended?

A. * Retinolum

B. Riboflavin

С. Tocopherolum

D. Pyridoxinum

E. Ascorbic acid

2. A 58 year old woman suffers from cerebral atherosclerosis. Complex of


her pharmacotherapy includes drug of vitamin E & С. Which
pharmacological effect of these drugs is the main for treatment of this
disease?

A. * Inhibition of free-radical oxidation of lipids

B. Increase of gonadotropic hormones synthesis in the pituitary body

С. Decrease of glucocorticoids release by adrenals

D. Strengthening of the antitoxic function of the liver

E. Improvement of the coronary circulation

3. A patient who had been taking л vitamin drug for the prevention of
cerebrovascular spastic reactions began to complain of unpleasnt feelings:
hyperemia of the face and upper part of the body, vertigo, feeling of blood
influx into the head. What drug may cause these side-effects

A. *Nicotinic acid

В. Tocopheroli acetas

C. Nicotinamidum

D. Thiamin/ bromidum
E. Calcii pangamas

4. A patient had been taking vitamin D for a long time for treatment of
rickets. Soon the signs of vitamin D intoxication developed: loss of
appetite, nausea, headache, fatigue, increase of body temperature, etc.
What vitamin decreasing the toxic influence of vitamin D should be
administered?

A *Vitamin A

B. Vitamin С

C. Vitamin В12

D. Vitamin PP

E. Vitamin B2

5. A 55 year old patient was admitted to the haemotological department


with signs of acute anemia. After laboratory examination megaloblastic
hyperchromic anemia was diagnosed, which drug must be administered to
the patient first of all?

A. *Cyanocobalaminum

B. Hemostimulinum

C. Ferroplexum

D. Ferrum-Lek

E. Folic acid

6. A 55 year old patient suffering from hyperchromic anemia obtained


long-term treatment with vitamin В12 parenterally. Why the parenteral
way of introduction of vitamin B12 is prefered more than the peroral
way?

A.*It isn't absorbed in the intestine in oral introduction due to deficit of


gastromucoproiein

B. It is faster absored

С It is longer circulated in the blood

D. It is faster penetrated to the bone marrow

E. it isn't destroied in the liver


7. A patient has diarrhoea, dementia and dermatitis. What vitamin has to
be included into the complex pharmacotherapy?

A. *Nicotinic acid

B. Thiaminum

C. Cyanocobalaminum

D. Panthotenic acid

E. Riboflavinum

8.There is an inhibited coagulation in the patient with bile ducts


obstruction, bleeding due to the low level of absorbtion of a vitamin.
What vitamin is in deficiency?

A. *Vitamin К

B. Vitamin D

С. Carotene

D. Vitamin A

E. Vitamin E.

9. A patient with diabetes mellitus is treated by injections of vitamin B1 to


eliminate metabolic acidosis. Which biochemical mechanism of action of
vitamin B1 ensures the positive effect?

A. * Activation of dehydrogenases of the Krebs cycle

B. Activation of adenylate cyclase

С. Blockade or phosphodiesterase

D. Acceleration of acetylcholine synthesis

E. Acceleration of adrenaline synthesis

10. Metabolic acidosis arose in a patient due to impairment of


carbohy¬drate metabolism and accumulation of ketoacids in the organism,
indicate the vitamin drug which promotes its elimination due to decrease
concentration of ketoacids.

A. *Thiamine

B. Pyridoxins
С. Folic acid

D. Riboflavine

E. Ascorbic acid

11. A patient suffers from chronic alcoholism with the following


symptoms: pain in arms and legs, impairment of skin sensitivity, muscle
weakness, edemas and increased amount of pyruvate. Which vitamin drug
should be prescribed to the patient?

A. * Thiamine

B. Ergocalciferol

C. Retinol

D. Rutin

E. Vikasoium (Menadione)

12. 55 years old patient was admitted to the hematological department


with acute anemia: RBCs - 1,5 x 1012/litre; Hb - 80g%, colour index -
1,3. Hyperchromic anemia was diagnosed. Which drug should be
administered for treatment of this disease?

A. *Cyanocobalamine

B. Ferroplexum

С. Hemostimuline

D. Ferrum-lek

E. Folic acid

13.Which of the acids below decreases permeability of connective tissue


structures, possesses antioxidant activity due to ability to be transformed
from the oxydized form into reduced and on the contrary?

A. * Ascorbic acid

B. Hydrochloric acid

C. Mefenamic acid

D. Aspirin (acetylsalicylic acid)

E. Aminocaproic acid
14. For synthesis of the basic substances of connective tissue
(mucopolysaccharides and collagen) an essential agent is:

A. * Ascorbic acid

B. Nicotinic acid

С. Folic acid

D. Salicylic acid

E. Acetylsalicylic acid

15. Radiation therapy is performed to the patient. What vitamin drug with
antioxidant properties is necessary for administration to increase stability
of tissues in this case?

A. * Ascorutinum

B. Vitamin B6

С Thiamine chloride

D. Cyanocobaiamine

E. Folic acid

16. The woman of 25 years who anted to get thin was on a diet hich
consisted of 2 cups of coffee ithout sugar, 3 crackers from white bread, 2
sausages or 2 eggs in a day within 1,5 months. The mass of a body has
dropped on 5 kg, but there were developed strong headaches, often nasal
bleedings, bleeding, sickness of gums, shaking of teeth, -.helling of sfcin
and shedding of hair.What vitamin drug is expedient in this (ase?

A. *Ascorbic acid

B. Folic acid

C. Retinol acetate

D. Cyancobalamine

E. Vikasolum (Menadione)

17. The patient who was treated by a vitaminic drug for prophylaxis of
vasospasms of the brain, has developed complaints of the unpleasant
sensations related to taking of this medicine: reddening of the face and the
upper half of a trunk, giddiness, sence of flush of blood to a head. For
what drug the specified side-effects are characteristic?
A. *'Nicotinic acid

B. Nicotinamidum

C. Thiamine bromide

D. Tocoferol acetate

E. Calcium pangamate

18. Bioflavonoids (rutin, quercetin) possess all listed below


pharmacodynamic effects, except for:

A. * Dilation of capillaries

B. Antioxidant activity

C. Inhibition of hyaluronidase activity

D. Decrease of permeability of capillary wall

E. Protections of ascorbic acid, assistance to its transport and


accumulation

19. 64 years old woman suffers from hemeralopia (disturbance of vision


in darkness). What vitaminic drug should be recommended her first of all?

A. *Retinoli acetate

B. Tocoferol acetate

C. Pyridoxin

D. Ascorbic acid

E. Riboflavin

20. What vitamin promotes growth and development of epithelial cells,


including epidermal ones?

A. *Retinol

B. Ergocalciferol

C. Ascorbic acid

D. Nicotinic acid

E. Lipoic acid
21. 39 years old patient suffers from hyperkeratosis, disturbance of vision
in darkness, frequent infectious diseases. What vitaminic drug should be
administered for treatment .

A. *Retinol acetate

B. Pyridoxin

С. Riboflavin

D. Ergocalciferol

E. Tocoferol acetate

22. What vitamin is formed in skin under influence of ultraviolet


radiation?

A. *Cholecalciferol

B. Ascorbinic acid

C. Retinole acetate

D. Calcium pantotenate

E. Riboflavin

23. To the child with signs of rachitis the pediatrist and the "dentist
administered a liposoluble vitamin which influences an exchange of
phosphorus and calcium in an organism, promotes sedimentation of
calcium in bone tissue and dentins. Determine a drug.

A. *Ergocalciferol

B. Tocoferol acetate

С. Retinoli acetate

D. Vikasolum (Menadione)

E. Thyreoidinum

24. Parasthesia, xeroderma arid sticking out of fontanel are observsed at


the 6 months child under the treatment by a vitaminic drug. Specify this
drug.

A. *Ergocalciferol

B. Pyridoxine
С. Riboflavin

D. Retinoli acetate

E. Tocoferol acetate

25. The dentist administered to his patient liposoluble vitamin with


antioxidant activity for treatment of parodontitis. Specify this vitaminic
drug.

A. * Tocoferol acetate

B. Ascorbic acid

C. Rutin

D. Nicotinic acid

E. Ergocalciferol

26.A doctor administered tocoferol acetate to a patient with ischemic heart


disease. What effect of a drug does the doctor expect?

A. *Antioxidant

B. Spasmolytic

С. Hypotensive

D. Increase of oxygen delivery to myocardium

E. Positive inotropic

27. What enzymatic drug is used with the purpose of dropping of density
and rising of permeability of connective tissue structures?

A. *Lidase

B. Lipase

С. Cocarboxylase

D. Cholines terase

E. Amylase

PLASMA SUBSTITUTES. AGENTS FOR CORRECTION OF WATER-


SALT BALANCE

1. A patient was admitted to the intensive care unit after taking a large
dose of Phenobarbitalum with the purpose of suicide. Investigation of the
patient revealed respiratory olic acidosis. What drug should be used for
the correction of acid-base state?

A * Trisaminum

B. Sodium chloride

C. Calcium chloride

D. Ammonium chloride

E. Potassium chloride

2. During treatment of the patient with digitoxin, extrasystoles, muscle


weakness, diarrhea, vomiting, and impairment of vision developed. What
drugs should be used to eliminate these signs of intoxication?

A. * Drugs of potassium

B. Drugs of calcium

С. Iron preparations

D. Drugs of sodium

E. Drugs of magnesium

3. What agent acts as magnesium ions' antagonist and is used in


overdosage of parenteral introduction of magnesium sulphatis?

A. *Calcium

B. Potassium

C. Sodium

D. Iron

E. Bromine

4. Specify the drug which eliminate both intra- and extracellular acidosis.

A. *Trisaminum

B. Natrii hydrocarbonas

С. Natrii lactas

D. Ammonii chioridum

E. Natrii hydroxydum
5. Specify the drug of first choice to be administered in a 7-year old child
with multiple caries.

A. *Calcii glycerophosphas

B. Calcii gluconas

С. Calcii chloridurn

D. Calcii hydroxydum

E. Calmecinum

6. The worker who several days ago started working in foundry, was
addressed to the doctor with complaints of headache, nausea. Due to
excessive diaphoresis he drank nearly 5 liters of tap water per day. What
drug will promptly and effec¬tively eliminate the specified signs and
normalize state of the worker?

A. * Sodium chloride

B. Decamevitum

С Aspirin (acetylsalicylic acid)

D. Analginum (methamizole)

E. Pentalginum

7. Ketoacidosis and dyspnoe are observed at the patient with non-


compensated diabetes mellitus. Which drug should be used for
normalization of patient’s state?

A. *Sodium hydrocarbonate

B. Naloxone

C. Bemegride

D. Pananginum

E. Ammonium chloride

8. It is necessary to eliminate ketoacidosis at the patient with sugar


diabetes complicated by hyperglycemic coma. What solution, being the
intra¬cellular buffer, can be administered?

A. * Trisamine (trometamol)

B. Solution of Sodium hydrocarbonate


С. Solution of Sodium lactate

D. Neohemodesum

E. Ringer’s solution

9.Ths patient suffering from ollagenosis has been treated for a long time
by Prednisolone in a dose of 30 mq per day. Recently he has started to
complain of painful convulsions of skeletal muscles of lower extremities.
What agent may be used or improvement of patient's state?

A * Panangin

В. Ergocatciferol

С. Calcitonin

D Diazepam

E. Aminazine (chlorpromazine)

10. The patient was admitted to the hospital with signs of dehydration.
The doctor has immediately administered him intravenous infusion of
Sodium chloride. At what situation is necessary to use this drug?

A. *Cholera

B. Toxicosis of pregnancy

C. Osteoporosis

D. Edemas

E. Arthritis

11. Convulsions and laryngospasm have developed at the patient with


hypoparathyrosis. Laboratory examina¬tion revealed significant decrease
of calcium ions concentration in the blood and slight elevation of pH.
Which drug should be introduced for correction of metabolic alkalosis?

A. *Ammonium chloride

B. Trisamine

C. Magnesium oxide

D. Aluminium hydroxide

E. Sodium hydrocarbonate
12. Which of plasma substitutes listed below circulates in the blood for a
long time?

A. * Rheopolyglucinum

B. 5% glucose solution

С. 0.9% solution of Sodium chloride

D. Ringer-Locke solution

E. Polydesum

PRINCIPLES OF TREATMENT OF ACUTE POISONINGS BY


DRUGS

1. A 38 year-old patient was admitted to a hospital due to acute poisoning


with mercury dichloride. Indicate the antidote which should be introduced
to the patient immediately.

A. * Unithiolum

B. Dipyroximum

С. Atropinum

D. Nalorphinum

E. Isonitrosinum

2. A patient was admitted to a hospital by the ambulance due to acute


poisoning with morphine. Indicate the agent which is used for gastric
lavage in this situation.

A. *Potassium permanganate

B. Natrii hydrocarbonas

С. Furacilinum

D. Tanninum

E. Boric acid

3. In a patient with syphilis there are grey stains on the oral cavity mucosa
and the signs of nephropathy developed during treatment with bismuth
compounds. Specify the agent which is used as antidote in poisonings by
bismuth compounds,

A. *Unithiolum
В. Nalorphine

С. Bemegridum

D. Naloxone

E. Methylen blue

4. A patient after drinking unknown liquid was admitted to a hospital with


complaints of stomachache, pain in gullet and diarrhea with little amount
of blood. Under examination hyperemia of throat mucous membranes,
gums' bleeding, lymphatic nodes enlargement and metallic aftertaste were
revealed. Within 2-3 days renal insufficiency was developed. Determine
the agent which caused named above symptoms:

A. *Salts of heavy metals

B. Furacilinum

С. Ethylic alcohol

D. Boric acid

E. Atropine

5. After treatment with bismuth preparation, a patient with syphilis


developed symptoms of kidney insuffuciency and gray strains on mouth
mucous membranes. Indicate a drug a doctor has to use as specific
antidote in the case:

A. *Unithiol

B. Bemegrid

С. Nalorphine

D. Naloxone

E. Methylenum coeruleum (methylene blue)

6. A 38 years-old male was admitted to ICU of a hospital with severe


poisoning by mercury chloride. Determine specific antidote:

A. *Unithiol

B. Dipiroxime

С. Atropine

D. Nalorphine
E. Isonitrosine

CHEMOTHERAPEUTICAL DRUGS

ANTISEPTICS AND DESINFECTANTS

Burned skin surface was treated with a certain preparation. Its antiseptic
properties are provided by atomic oxygen that is formed in presence of
organic substances. What preparation was applied?

A *Potassium permanganate

B Furacillin

C Chlorhexidine bigluconate

D Alcoholic iodine solution

E Sodium hydrocarbonate

Patient with abscess of the cut wound applied to the traumatological


department. Doctor for

the cleaning of the wound from the pus washed it with 3%


hydrogen peroxide. Foam was

absence. What caused the absents on the drug activity?

A *Inherited insufficiency of catalase

B Low concentration $H_{2}O_{2}$

C Inherited insufficiency phosphatdehydrogenase of erythrocyte

D Shallow wound

E Pus in the wound

For the preparation of a patient's burn skin surface a certain


medication was used. Its

antiseptic action is provided by free oxygen that segregates in


presence of organic

substances. Choose the right answer:

A *Potassium permanganate

B Furacilin

C Chlorhexidine
D Boric acid

E Sodium bicarbonate

1. The patient addressed to the doctor in relation with trauma of the foot.
The foot was bandaged with a dirty gauze bandage, impregnated with
purulent discharges. Attempt to take off a bandage for survey and
processings of a wound invoked an acute pain as the bandage had stuck to
wound surface. Choose an antiseptic which will facilitate taking off of a
bandage and will mechanically clear a wound of mud and pus.

A. * Hydrogen peroxide

В. Aethacridinum lactate

С. Aethonium

P. Potassium permanganate

E. Furacilinum

2. The patient addressed to the doctor with complaints of pustular pimples


on the skin of the face. In bacteriological analysis of contents of pustules
staphylococcus aureus was found and the diagnosis of staphylococcal
pyodermia was given. Choose the most efficient drug from the listed
antiseptics for local use in pustular pimples.

A.* Brilliant green

B. Ethyl alcohol

С. Chlorhexidinum

P. Potassium permanganate

E. Aethacridinum lactate

3. A patient addressed to г ophthalmologist with complaints of eye


discomfort, discharge of purulent exudate, disorders of vision. Specify the
antiseptic available for rinsing of the eyes.

A. *Silver nitrate

B. Lugol's solution

С Potassium permanganate
P. Salicylic acid

E. Ammonium solution

4. In a patient with varicose dilation of veins the trophic ulcer of the leg
developed. The bacteriological examination of the ulcer discharge
revealed Staphylococcus infection. For the local treatment of the ulcer an
antiseptic in the form of ointment from the group of detergents was
administered. Specify it.

A. *Ethonium

B. Brilliant green

С Furacil/inum

D. Potassium permanganate

E. Ethacridini iactas

5. Which acid possesses the properties of an antiseptic?

A. *Boric acid

B. Nicotinic acid

C. Folic acid

D. Ascorbic acid

E. Dehydrocholic acid

6. Specify the antiseptic which is used for disinfection of operation field


and surgeon's hands.

A. *70% solution of ethyl alcohol

B. Furacilinum

C. Ethonium

D. Potassium permanganate

E. 95% solution of ethyl alcohol

7. Determine the following drug: it contains an halogen, exerts


antimicrobial and deodorizing action, is used for disinfection of non-metal
instruments, as an antiseptic - for processing of hands

A.*Chloraminum
B. Hydregen peroxide

C. Formaldehyde

D. Phenol

E. Resorcinum

8. A patient was admitted into the emergency department in relation with


acute poisoning - by mistake he drank mercury dichloride solution. The
patient complained of severe pain in the oral cavity, along the esophagus
and in the epigastric area, hypersalivation, fatigue, tachycardia. Specify
the agent which would neutralize the absorbed mercury binding to it.

A. *5% solution of unithiolum intramuscularly

B. Methylene blue with 5% glucose solution intravenously solution of


sodium

C. 4% solution of sodium carbonate intravenously

D. 2% solution of sodium nitrite intravenously

E. 2% solution of furosemide

9. All antiseptics possess ail following properties except:

A *Selective antimicrobic action

B. Versatile antimicrobic action

С. Bactericidal action

D. Highly toxic for human

E. Are not introduced parenterally

10. Chloramine possesses all following effects, except:

A. *Antiallergical

B. Deodorization

C. Antiseptic

D. Spermicidal

E. Fading

11. Formaldehyde solution is used for disinfection of non-metallic


surgical tools. Indicate the correct name of group of formaldehyde:
A. * Aliphatic agents

B. Aromatic agents

C. Spirits

D. Halogen-maintained agents

E. Detergents

12. 70% solution of aethyl spirit was used by a surgeon for cleaning his
hands before operation. Explain the mechanism of action of the antiseptic
drug:

A. *Protein dehydration of microbes protoplasm

B. Blockade of sulfhydryl groups of enzymes

C. Oxidation of organic components of microbes 'protoplasm

D. Interaction with aminogroups of protoplasm proteins of microbes

E. Interaction with hydroxilic groups of microbes enzymes

13. A doctor used 5% spirituous solution of iodine for cleaning of


operation field. Indicate its mechanism of action:

A. *Interaction with amino groups of microbes 'proteins that disposes to


their denaturation

B. Dehydration protoplasm's proteins

C. Bound to enzymes' sulfhydric groups

D. Formation of albuminates

E. inhibition of dehydrogenase

14. A female suffers from varicose veins dilatation of lower extremity


which is complicated by ulceration on ankle. The ulcer is accompanied by
local hyperemia and itching around and discharges pus with
staphylococci. An antiseptic ointment from the group of detergents was
administered for treatment. After treatment all of the symptoms are
diminished. Indicate the drug:

A. * Ethonium

B. Furacilhium fnitrofuran)

С Viride nitens (brilliant green)


D. Aethacrydine lactate

E. Potassium permanganate

SULFONAMIDES AND OTHER SYNTHETIC ANTIBACTERIAL


AGENTS

Patient with pneumonia has intolerance to antibiotics. Which of the


conbined sulfanilamide

medicines should be prescribed to the patient?

A *Biseptol

B Aethazol

C Natrium sulfacyl

D Streptocid

E Sulfadimethoxine

A 7 year old child is ill with bronchitis. It is necessary to administer


him an antibacterial

drug. What drug of fluoroquinolone group is \textbf{CONTRA-


INDICATED} at this age?

A *Cyprofloxacin

B Ampicillin

C Amoxicillin

D Sulfadimethoxine

E Ampiox

Gonorrhoea was revealed in the patient on bacterioscopy of the smear


from urethra. Taking into account that medecines for gonorrhoea are
fluorquinolones, patient should be prescribed:

A *Ciprofloxacin

B Furazolidone

C Fluorouracil

D Urosulfan
E Cefazoline

1. The doctor administered Sulfadimezinum in tablets to the patient with


bacterial infection, and advised to take the drug with alkaline mineral
water. Indicate the purpose of the given reference.

A. * For prophylaxis of crystallization of acetylated derivants of the drug


in renal tubules

B. For prolongation of action

С For reducing of the irritative action on the stomach

D. For neutralization of HCl of a gastric juice

E. For shift of blood pH in the alkaline side

2. A 37 year-old patient was admitted to an infectious diseases hospital


with the diagnosis of dysentery. Indicate the drug which should be
appointed to the patient?

A. * Ciprofloxacin

B. Erythromycin

С. Oxacillinum

D. Phenylsalicylate

E. Imodium

3. Specify the sulfonamide drug which is poorly absorbed in the intestine


and is used for the treatment of intestinal infections.

A. *Phthalazolum

B. Ethazolum

С. Sulfadimethoxinum

D. Sulfadimezinum

E. Sulfacylum-natrium

4. Specify the sulfonamide agent for the treatment of conjunctivitis.

A. *Sulfacylum-natrium

B. Phthalazolum
С. Urosulfanum

D. Sulfadimezinum

E. Biseptoium

5.Specify the combined sulfonamide agent.

A. *Biseptolum

B. Ethazolum

С. Sulfacylum-natrium

D. Sulfadimethoxinum

E. Streptocidum

6. A patient visited a physician with complaints of painful and frequent


urination, pain in the lower part of the back. After laboratory and
bacteriologic examination of the urine (it revealed gram-positive cocci,
Proteus, acute cystitis and urethritis were diagnosed. Specify the agent
that should be administered taking into account the localization of its
action.

A. *Nitroxlinum

B. Ethazolum

C. Biseptoium

D. Sulfadimezinum

E. Furasolidonum

6. Specify the antimicrobial drug from the group of 8-oxiquinolirte


derivatives.

A. *Nitroxolinum

B. Biseptoium

C. Nalidixic acid

D. Ciprofloxacinum

E. Furaginum

7. Specify the antimicrobial drug from the group of nitrofurans.

A. *Furaginum
B. Biseptoium

С Nalidixic acid

D. Nitroxolinum

E. Ciprofloxacinum

8. Specify the antimicrobal drug from the group of fluoroquinolones.

A. *Ciprofloxacinum

B. Biseptoium

С. Nalidixic acid

D. Nitroxolinum

E. Furaginum

9. During examination in out-patient department a physician identified


pneumonia and- prescribed in-patient treatment by ampicillin and
cefalexin. However, the patient started treatment at home with the same
antibiotics, dosage and timing prescribed by the doctor. Within three days
the sick person felt better, fever and cough reduced. The treatment was
discontinued and the patient turned back to work. Next day he/she felt
much worse, fever and cough were developed again, that is why the
patient had to be examined by the physician, indicate please which
principle of chemotherapy was disobeyed by the patient:

A. *Duration of treatment

B. Combined usage of agents

C. The earliest chemotherapy beginning

D. Effective agent choice based on clinical and bacteriological diagnosis

E. Optimal selection of dosage, timing and introduction ways of agent

10. Patient with pneumonia was treated by injections of antibiotic.


Determine the type of chemotherapy:

A. *Causal treatment

B. Substitute treatment

C. Preventive treatment

D. Symptomatic treatment
E. Pathogenic treatment

11. Duration of sulfonamide agents' activity depends on:

A. *Affiliate activity with proteins of blood or/and reabsorption in renal


canaliculi

B. Aptitude of enterohepatic circulation

С. Speed of absorption in GIT

D. Level and speed of metabolic biotransformation in liver

E. All named above

12. After antimicrobial treatment of pneumonia within 8 days patient


developed painful urination, pain in kidneys area, the urine has brown.
Indicate the drugs group that used in the case:

A. *Sulfonlamide

B. Lincomycin

C. Β- lactam antibiotic

D. Aminoglycosides

E. Cephaiosporines

13. After long-term treatment by antibiotics at the in-patient department a


patient developed dyspeptic syndrome. Investigation of stool revealed
diminished amount of Bifidobacterium and Bacillus coli. What is the
reason of present illness?

A. *Disbacteriosis

B. Enteric colibacillosis

C. Pseudomembranous enterocolitis

D. Toxic action of the agents

E. Acquisition of nosocomial infection

14. Indicate an antibacterial agent from the group of fluoroquinolone:

A. *Ciprofloxacin

B. Nalidix acid

С. Nitroxolin
D. Furosemide

E. Biceptol

ANTIBIOTICS (I)

A 60-year-old patient was admitted to the surgical department


because of infection caused

by blue pus bacillus (Pseudomonas aeruginosa) which is sensative


to penicillin antibiotics.

Indicate which of the given penicillins has marked activity to the


Pseudomonas

aeruginosa?

A *Carbenicillin disodium

B Benzylpenicillin

C Phenoxymethylpenicillin

D Oxacillin

E Methicillin

A 19 year old woman suffers from primary syphilis. Doctor


administered her complex

therapy that includes benzylpenicillin sodium salt. What is the


mechanism of action of this

drug?

A *It blocks synthesis of peptidoglycan of microbal membrane

B It blocks synthesis of cytoplasm proteins

C It blocks thiol enzymes

D It blocks RNA synthesis

E It blocks DNA synthesis

A patient with bacterial pneumonia was prescribed benzylpenicillin.


What is the mechanism

of its antibacterial effect?


A *Inhibition of synthesis of microorganism wall

B Inhibition of intracellular protein synthesis

C Abnormal permeability of cytoplasmic membrane

D Inhibition of SH-groups of microorganism enzymes

E Antagonism with p-amino-benzoic acid

3. A patient was delivered to the surgical department with anaerobic


gangrene. Specify the antibiotic of first choice for the treatment of this
infection.

A. *Benzylpenicillinum natrium

В. Tetracyclinum

С. Clindamycinum

D. Cefazolinum

E. Chloramphenicol

4. Specify the main antibiotic for the treatment of diphtheria.

A. *Erythromycinum

B. Laevomycetinum

С. Cefazolinum

D. Gentamycinum

E. Tetracyclinum

7. Specifу the group of antibiotics whose mechanism of action is


connected with inhibition of synthesis of bacterial ceil wall.

A. *Penicillins

B. Macrolides

C. Tetracyclines

D. Aminoglycosides

E. Lincosamides

8. Specify the most typical side-effect of penicillins.


A.*Allergic reactions

B. Agranulocytosis

C. Anemia

D. Decrease of audition

E. Hepatotoxic influence

9. Specify the antibiotic from the group of semisynthetic penicillins.

A. *Ampiciilinum

B. Phenoxymethylpenici/linum

C. Benzylpenicillinum natrium

D. Benzylpenicillinum kalium

E. Benzylpenicillinum novocainum

14. Indicate a drug group which oppresses synthesis of cell membrane


components:

A. *Penicillines

B. Tetracyclines

C. Aminoglycosides

D. Lincosamides

E. Macrolides

15. Drug with β-lactam ring was prescribed to a patient with streptococci
gums inflammation. Indicate this drug:

A. *Benzylpenicin

B. Rifampicine

C. Erythromycin

D. Streptomycin sulfate

E. Laevomycetine

16. A patient was admitted to a hospital with diagnosis: gaseous gangrene.


Drugs for its treatment are divided on two groups: basic and reserve.
Indicate the basic antibiotic:
A. *Benzylpenicillin natrium

B. Tetracycline

C. Laevomycetine

D. Clindamycin

E. Cefazolin

17. Cephalosporines possess following properties, except:

A. *Detergent activity

B. Mechanism of action linked to infringement of microbe s


membrane synthesis

C. Bactericidal activity

D. Distinguished from penicillines by higher persistence toward β-


lactamase

E. Distinguished from penicillines by spectrum of antimicrobial


activity

18. 56-years old male was admit¬ted to a hospital with pneumonia. It is


known he suffers from hay fever and seasonal vasomotor rhinitis. What
drug should be administered in the case?

A. *Cefazolin

B. Benzylpenicillin

С. Bicillin

D. Oxacillin

E. Ampiciliin

19. 14-years old boy developed acute pneumonia in low lobe of the right
lung. The agent in sputum analysis was resistant to penicillin. Choose the
drug for treatment in this case:

A. Gentamycin

B. Laevomycetine

С. Streptomycin

D. Tetracycline
E. *Cefazolin

20. Determine drug by following: it oppresses of protein synthesis by


microbes ribosomes because of inhibition of peptidtranslocase, belongs to
reserve macrolide, causes side effects relatively seldom.

A. *Erythromycin

B. Sygmamycin

С. Tetraolean

D. Azithromycin

E. Tetracycline

23. A patient with diminished hearing has severe bacillary infection.


Which drug group is contradicted to the patient?

A. *Aminoglycosides

B. Peniciltines

C. Cephalosporines

D. Tetracyclines

E. Rifampicines

24. Patient with acute appendicitis, was admitted to a surgical department.


Appendectomy was performed. During ten days after operation patient
received an antibiotic. After a while lowering of hearing were revealed.
Indicate drug group with the same side effects:

A. *Aminoglycosides

B. Tetracyclines

C. Polymyxines

D. Macrolides

E. Penicillines

25. Determine the drug for treatment infections of bones that able to
penetrate to bone tissue and bone marrow:

A *Lincomycin

В. Benzylpenicillin
С. Bicitlin-3

D. Gentamycin

E. Synthomycinum

29. A patient with diagnosed streptococcal bronchopneumonia after


treatment with an antibiotic suffers from allergic symptoms. Determine
the drug:

A. *Benzylpenicillin-natrium (penicillin G sodium)

B. Tetracycline

C. Gentamycin

D. Laevomycetineum (chloramphenicol)

E. Doxicycline

30. Infectious agent determined by lab tests is known to be sensitive to


third generation cephalosporins. Choose the drug for treatment:

A. Cefazolin

B. Cefalexin

С Cefalotin

D. *Cephtriaxone

E. Cefaloridin

31. A patient with bacterial pneumonia was treated by the erythromycin


which acts on microbes by interaction with their free 50S subunits of
ribosomes. What process does this drug block?

A. *Proteins' synthesis

B. RNA synthesis

C. DNA synthesis

D. Lyposynthesis

E. Polysaccharides' synthesis

26. An antibiotic with ability to penetrate to bones tissue was prescribed


to 30 years old patient with osteomyelitis. After three weeks of using it
the patient felt much better. Determine the drug:
A. Lincomycin

B. Bicilline-3

C. Benzylpenicillin

D. Polymixine M

E. Ampicillin

2. A patient started to complaint of worsening of audition after treatment


with antibiotic because of purulent complication after the surgical
operation. Specify the group of antibiotics which posses ototoxic activity.

A. * Aminoglycosides

В Penicillins

С. Tetracyclines

D. Polymyxins

E. Macrolids

ANTIBIOTICS (II). ANTIFUNGAL AGENTS

A patient underwent appendectomy. In the postoperative period he has


been taking an

antibiotic. The patient complains about hearing impairment and


vestibular disorders. What

group of antibiotics has such by-effects?

A *Aminoglycosides

B Penicillins

C Tetracyclines

D Macrolides

E Cephalosporins

A patient has herpetic rash. What medication should be administered?

A *Acyclovir
B Gentamycin

C Clotrimazole

D Benzylpenicillin sodium salt

E Biseptol

Patient was admitted to the infection unit with diagnosis of bacterial


dysentery. On laboratory studies it was revealed that causative element is
sensative to the many antimicrobial medicines, but patient has anemia.
What medicine is contra-indicated to the patient?

A *Levomycetin

B Phthalazol

C Furazolidone

D Enteroseptol

E Ampicillin

A patient suffers from severe postoperative pseudomonadous infection.


What of the following antibiotics should be administered in this case?

A *Amicacin sulfate

B Benzylpenicillin

C Cephazolin

D Erythromycin

E Doxycycline

1. The patient with the diagnosis of cholera was admitted to the infection
diseases hospital. Specify a group of antibiotics of the first choice for
treatment of this disease

A. *Tetracyclines

B. Aminoglycosides

С. Penicillins

D. Macrolids
E. Cephalosporines

5. A woman addressed to a dentist with complaints of teeth destruction in


her little child. It was revealed that during pregnancy the woman took
antibiotics. Specify the group of antibiotics that could cause these side-
effects.

A. *Tetracyclines

B. Macrolides

C. Penicillins

D. Cephalosporins

E. Aminoglycosides

6. An antibiotic was administered to a patient suffering from abdominal


typhoid. Soon there was general improvement, but on the 2nd week after
the treatment the patient had elevation of body temperature, signs of
tonsillitis, and rashes on mucous membranes of lips & nose. In laboratory
examination of discharges, Candida fungi were found. The blood analysis
revealed leukopenia and agranulocytosis. Which antibiotic could cause
these complications?

A. *Laevomycetinum

B. Tetracyclinum

С. Polymyxins

D. Gentamycinum

E. Cefazolinum

10. Specify the group of antibiot ics whose mechanism of action involves
inhibition of protein synthesis by microorganisms.

A. *Tetracyclines

B. Penicillins

С. Cephalosporins

D. Monobactams

E. Polymyxins
11. In the treatment with wide-spectrum antibiotics some complicaions,
including candidiasis may occur. Specify the agent for the treatment of
candidiasis.

A. *Ketoconazole

В Amphotericinum B

C. Griseofulvinum

D. Gramicidinum

E. Undecinum

12. A patient with dermatomycosis took antifungal agent which was able
to be accumulated within the cells producing keratin (skin, nails, hairs), in
several clays the patient visited the physician complaining of headache,
desorientation. Specify the appointed antibiotic.

A. *Griseofulvinum

B. Levorinum

С. Amphotericinum В

D. Mycogeptinum

E. Nystatinum

13. After long-term treatment with tetracyclinum a patient was


hospitalized in relation, with aphthous stomatitis. During laboratory
examination the Candida fungi were identified. Specify the agent
available for the treatment of candidiasis.

A. *Nystatinum

В. Furazolidonum

С. Griseofuivinum

D. Amicazolum

E. Cefalexinum

21. Determine drug with wide spectrum of antibiotic activity, a basic


antibiotic agent of treatment enteric fever and other salmonellosises and
possesses following side effects: oppresses of bone marrow activity,
disbacteriosis and dyspeptic disorders:
A. *Laevomycetine

B. Phthalazolum

С. Benzylpenicillin natrium

D. Neomycin sulfate

E. Tetracycline

22. Which drug is used for treatment of enteric fever?

A. *Laevomycetine

B. Ampicillin

C .Cefalexin

D. Benzylpenicillin

E. Erythromycin

27. Deteirmine drug for treatment of candidiasis:

A. *Nystatin

B. Kanamycin

C. Tetracycline

D. Erythromycin

E. Benzylpenicillin

28. Considerable number of Candida albicans was revealed on cytological


investigation of smear of 25 years-old woman with exacerbation of
chronicle vaginitis. Which drug should be prescribed?

A. *Nystatin

B. Amphotericine

C. Miconazole

D. Clotrimazole

E. Metronidazole

32. An antibiotic for treatment of enteric fever was administered to a


patient. Clinical recovery was achieved, but within 2 weeks the patient
developed symptoms of quinsy, fever, rashes at mucous membranes of
lips and nose. Blood test revealed diminished amount of WBC and
granulocytopenia. Choose an antibiotic which can cause these side effects:

A. Tetracycline

B. *Laevomycetinum (chloramphenicol)

С. Роlymyxine М sulfate

D. Cefazoline

E. Gentamycine

33. Tetracycline was administered PO for treatment of acute purulent


sinusitis. What antimycotic drug should be administered to a patient to
prevent candidiasis?

A. Griseofulvin

B. Levamisole

C. Furazolidone

D. Ciprofloxacin

E. *Nystatin

34. After taking tetracycline for a long period of time, patient developed
candidiasis of mucous membranes of mouth. Which drug should be used
for treatment?

A. *Nystatin

B. Griseofulvin

С. Nitrofungine

D. Nitroxoffne

E. Furadon/ne

ANTITUBERCUFOUS, ANTIVIRAL AND ANTISYPHILITIC


AGENTS.

A patient suffers from pulmonary tuberculosis. During treatment neuritis


of visual nerve

arose. What drug has caused this by-effect?


A *Isoniazid

B Ethambutol

C Kanamycin

D Rifampicin

E Streptomycin

A patient was diagnosed with active focal pulmonary tuberculosis. What


drug should be

prescribed in the first place?

A *Isoniazid

B Sulfalen

C Cyclocerine

D Ethionamide

E Ethoxide

1. The patient who had been suffering from tuberculosis was treated with
Isoniazidum. After a while the patient began to complaint of muscle
weekness, decrease of skin sensitivity, impairment of vision and motor
discoordination. Indicate the vitamin's drug which should be administered
to eliminate the specified phenomena?

A. * Pyridoxin (B6)

B. Retinol (A)

С. Ergocalciterol (D)

D. Cyanocobatamin (B12)

E. Ascorbinic acid (C)

2. Indicate the drug which is used for intranasal dropping with the purpose
of prophylaxis of infuenza.

A. * Interferon

B. Remantadinum

С. Ampicillinum

D. Aciclovir
E. Paracetamolum

3. Specify the antibiotic available for the treatment of tuberculosis.

A.*Rifampicinum

B. Tetracyclinum

С. Ampici/linum

D. Erythromycinum

E. Lincomycinum

4. Specify the antituberculous agent which inhibits synthesis of mycolic


acids by Mycobacterium tuberculosis.

A. *Isoniazidum

B. Ethambutolum

C. Streptomycinum

D. Cycloserinum

E. PAS

5. Indicate the most effective synthetic antituberculous drug.

A. Kanamycinum

B. Streptomycinum

С. Rifampicinum

D. PAS

E. *Isoniazidum

6. A patient, 60 years old had been treated for tuberculosis for a long time.
Recently he began to omplain of decrease in audition, which drug should
be contraindicated?

A. *Streptomycinum

B. Ftivazidum

С Ethambutolum

D. Isoniazidum

E. Rifampicinum
7. Drugs of which group must be administered first of all to a girl 1.5
years old in relation with acute herpetic, stomatitis during rash period?

A. *Antiviral agents

B. Antiallergic agents

С. Antibiotics

D. Antiseptics

E. Keratoplasty

8. Specify the agent which could be used for the prevention of influenza
during epidemic period.

A. *Remantadinum

B. Biseptolum

С. Ampicillinum

D. Anaiginum

E. Paracetamolum

9. In the newborn department of a hospital there was sudden increase of


acute respiratory disease caused by venous types of viruses. To prevent
spread of the infection it was recommended to use human leukocytic
interferons. Specify the available way of introduction in this case.

A. *Intranasal

B. Subcutaneous.

С Intramuscular

D. Peroral

E. Inhaled.

10. A woman 25 years old was hospitalized for treatment of syphilis.


Specify one of the main antibiotics for treatment of this disease.

A. *Benzylpenicillinum natrium

В Erythromycinum

C. Tetracyclinum

D. Lincomycinum
E. Vancomycinum

11. A 35-year-old man under the treatment for pulmonary tuberculosis


suffers acute pain of the right big toe, accompanied by swelling and slight
fever. The gouty arthritis was diagnosed and high serum uric acid level
was found. Which of the following antituberculos drug is known for
causing high uric acid levels?

A. *Pyrasinamide

B. Cycloserine

С. Rifampicine

D. Thioacetazone

E. Aminosalicylic acid

12. Patient has inherited of acetyl-transferase insufficiency. Which drugs


can cause severe intoxication in this case?

A. *Hydrazids of isonicotinic acid

B. Barbiturates

С. Antibiotics-tetracyc/ines

D. β-adrenoblockers

E. Nitrates

13.Which antituberculous drug from the following oppresses transcription


DNA to RNA?

A. *Rifampicine

B. Isoniazid

С. Streptomycin

D. Ethionamide

E. PAS

14. Patient with leprosies developed hypopigmentic rash with absence of


perception in its location. An antibiotic that is the basic antituberculous
agent was prescribed. Indicate this drug:

A. *Rifampicine
B. Amoxicillin

С. Erythromycin

D. Nitroxoli

E. Cefazolin

15. After treatment of patient suffering from tuberculosis, his vision


worsened rapidly, visual fields were narrowed. Determine the drug which
caused these side effects:

A *Ethambutol

B. Isoniazid

C. Kanamycin sulfate

D. Ethionamide

E. Rifampicine

16. 60-Years old male, with diagnosed tuberculosis long time ago, timely
receives antituberculous treatment. He developed neuritis of nervous
facials. Which drug can cause this side effect and should be accepted from
the therapy:

A *Ethambutol

B. Isoniazid

С. Rifampicine

D. Streptomycin

E. PAS

17. After treatment by antituberculous drugs during three month, a patient


developed daltonism, reduced ability to distinguish red and green colors.
Which antituberculous agent can cause this side effect?

A *Ethambutol

B. Streptomycin

С. PAS

D. Rifampicine

E. Cycloserine
18. Determine drug for AIDS treat¬ment with following mechanism of
action: it is able to be phosphorilizated in cells and transformed to
triphosphate, and then it inhibits viral

transcriptase and impede of DNA synthesis from viral RNA.

A. *Zidovudine

B. Saquinavir

С. Indinavir

D. Ritonavir

E. Virasept

19. After treatment by the antiretroviral drug from the group of


nucleosides within six month a patient with AIDS developed reducing
amount of RBC, neutrophilic cells and platelet cells in the blood. Indicate
the agent that is able to cause these side effects?

A. *Zidovudine

B. Sackvinovir

С. Acilovir

D. Valaciclovir

E. Remantadine

20. 19-Years old patient with primary syphilis receives complex treatment
by benzylpenicillin natrium. What is its mechanism of action?

A. *Blockade of murein synthesis in cell walls

B. Blockade of protein synthesis in cytoplasm

С. Blockade of tiotic groups of enzymes

D. Blockade of RNA synthesis

E. Blockade of DNA synthesis

21. A patient has been suffering from tuberculosis associated with


intracellular location of mycobacterium for a long period of time. What
drug must be included in complex treatment?

A. Rifampicine
B. *Isoniazid

C. Ethambutol

D. Ethionamide

E. Natrium paraaminosalicilate

22. A patient known to be treated of pulmonary tuberculosis noticed that


his lachrymal liquid and urine became red. What drug is able to develop
such side effect?

A. *Rifampicine

B. Isoniazid

C. Ethionamide

D. Streptomycin sulfas

E. Ethambutol

23. 39-years-old patient with pulmonary tuberculosis received effective


complex treatment composed of 3 antituberculous agents including
Steptomycin sulfate. What is its mechanism of action?

A. *Blockade of proteins synthesis

B. PABA anti-metabolite

C. Blockade of RNA synthesis

D. Inhibition of DNA replication

F. Inhibition of mycolic acids synthesis

24. A patient with primary syphilis has allergy to benzylpenicillin. What


drug can be prescribed in the case?

A. *Erythromycin

B. Amoxicillin

С. Amoxiclav

D. Carbenicillin

E. Lincomycin

30. 25-years old woman was admitted to a hospital with diagnosed


syphilis. Indicate the main antibiotic for her treatment:
A. *Benzylpenicillin natrium (penicillin G sodium)

B. Tetracycline

С. Lincomycin

D. Vancomycin

E. Erythromycin

ANTIPROTOZOAL AND ANTHELMINTIC AGENTS.

A patient consulted a stomatologist about purulent inflammation of


his gums. What drug will

be the most effective if it is suspected that a causative agent is an


anaerobe?

A *Metronidazole

B Gentamicin

C Oxacillin sodium

D Co-trimoxazole

E Nitroxoline

A 30-year-old patient complains about having abdominal pain and


diarrhea for five days;

body temperature rise up to $37,5^oC$ along with chills. The day


before a patient had

been in a forest and drunk from an open water reservoir. Laboratory


analyses enabled to

make the following diagnosis: amebic dysentery. What is the drug


of choice for its

treatment?

A *Metronidazole

B Furazolidonum

C Levomycetin
D Phthalazol

E Emetine hydrochloride

Systemic amebiasis with involvment of intestines, liver, lungs was


diagnosed in a

52-year-old patient. What drug should be prescribed?

A *Metronidasol

B Quiniofone

C Tetracycline

D Quingamine

E Enteroseptol

A patient consulted a doctor about bowels disfunction. The doctor


established symptoms of

duodenitis and enteritis. Laboratory examination helped to make


the following diagnosis:

lambliosis. What medication should be administered?

A *Metronidazole

B Erythromycin

C Monomycin

D Chingamin

E Tetracycline

A patient ill with amebiasis was prescribed a certain drug. The use
of alcohol together with

this drug is contra-indicated because the drug inhibits metabolism


of ethyl alcohol. What

drug is it?

A *Metronidazole

B Reserpine

C Clonidine
D Diazepam

E Aminazine

1. A female patient addressed to gynecologist with complaints of undant


discharges from vagina with pleasant smell. After bacteriological
vestigation the diagnosis of trichmo-•sis has been given. Specify the ug
which should be administered.

A. * Metronidazolum (Trichopolum)

В. Sulfadimezinum

C. Chingaminum

D. Chloridinum

E. Monomycinum

2. Mother addressed to the pediatrician with the child who complained of


strong itch in the region around the anus, pain intensified at night. After
investigation of feaces the diagnosis of enterobiasis was given. Indicate
the drug which should be administered.

A. *Levamisolum

B. Trichlorophenum

С. Phenasalum

D. Ditrazinum

E. Aminoacrichinum

3. Indicate the antimalarial agent which is active against paraerythrocytic


forms of Plasmodium.

А *Рrimachinum

В Chingaminum

С. Galochinum

D. Hydroxychlorochinum

E. Aminoacrichinum

4. Specify the drug which is used in amebiasis of any localization of


pathological process.
A. *Metronidazolum (Trichopolum)

B. Chingaminum

C. Emetinum hydrochloridum

D. Chiniophonum

E. Tetracyciinum

5. A patient visited a physician with complaints of bowel dysfunction.


After laboratory examination the diagnosis of lambliasis was made.
Specify the drug that should be used.

A. *Metronidazolum (Trichopolum)

B. Tetracycilinum

С. Trichomonacid

D. Monomycinun

E. Chingaminum

6. During summer vacations a student from tropical country developed


tertian malaria. After recovery he turned back to Ukraine for study
extension. In January an exacerbation was developed, it is known from
past history of disease that drug acting on paraerythrocytic plasmodium
malariae for prevention of relapse was not prescribed. Indicate the drug:

A. Chingaminum

B. Halochin

С. Hydroxychloroquine

D. Amodiaquine

E. *Primaquine

7. Patient addressed to a physician to get a drug for prevention of malaria.


Indicate the drug:

A. *Primaquine

B. Clotrimazole

С. Mebendazole

D. Furazolidone
E. Fenasal

8. What is the mechanism of anthelmintic action of levamisole?

A. *Oppression ot succinate dehydrogenase, ATPase

В. Oppression of MAO

C. DNA synthesis damage

D. Cholinesterase activation

E. Oppression of N-acetyltransferase

9. A drug is administered for prevention and treatment of malaria,


treatment of amebiasis and diseases of connective tissue. Indicate the
drug.

A. *Chingaminum (chloroquine)

B. Tetracycline

C. Metronidazole

D. Erythromycinum

E. Quinine

10. A drug was administered to a patient with ascaridosis. It is known to


have influence on immune system, and is used as immunological
modulator. Indicate the drug!

A. *Levamisole

B. Piperazine

С. Pyrantel

D. Phenasaium (niclosamide)

E. Praziquantel

11. A patient complaints of nausea, vomiting, loss of appetite. After


investigation of stool ascaridosis was revealed. A drug with immune
modulation activity was prescribed for single usage. Indicate the drug:

A. *Levamisole

B. Mebendazole

С. Pyrantel
D. Naphtamonum

E. Piperazine

ANTITUMOAL AGENTS

1. The antitumoral agent from the group of antimetabolites


(antagonists of folic acid) was administered to the patient with acute
leucosis. Indicate this drug.

A. *Methotrexatum

B. Fluorouracil

C. Myelosanum

D. Mercaptopurine

E. Hexestrolum

2. Determine a drug for treatment of lympholeukosis:

A. *Embichinum

B. Phthoruracilum

C. Depostat

D. Diethylstilbestrol

E. Phenobolinum

3. A drug belongs to the group of anti-metabolites being an antagonis


causes impairment of purines' synthesis, and thus lead to diminishing of
nucleic acids' synthesis. Determine the drug:

A. *Methotrexate

B. Mercaptopurine

C. Phthoruracilum (fluorouracil)

D. Cytarabine

E. Cispiatine
4. Antitumoral drug from the group of antimetabolites is used for
treatment of leucosis of children and cancer of adults. Determine the drug:

A. *Methotrexate

B. Sarcolysinum

С. Colchamine

D. Rubomycin

E. Prednisolonum

THE TREATMENT OF ACUTE POISONONGS BY DRUGS

A patient with acute morphine poisoning was delivered to a


hospital. What specific narcotic

antagonist should be chosen in this case?

A *Naloxone

B Paracetamol

C Methacin

D Digoxin

E Unithiol

Patient with mercury poisoning was admitted to the toxicological


department from the

chemical industry. What medicine should be used?

A *Unithiol

B Isonitrozin

C Naloxone

D Activated carbon

E Enterosorbent

A patient suffering from syphilis has been treated with bismuth


preparations. As a result of
it some grey spots turned up on the mucous membrane of the oral
cavity; nephropathy

symptoms were also present. What drug should be used for


treatment of bismuth

intoxication?

A *Unithiol

B Methylene blue

C Naloxone

D Bemegride

E Nalorphine

Март 2016
General Pharmacology

1. A patient with angina pectoris was helping himself with


nitroglycerin. One day the patient used nitroglycerin several times because of
frequent episodes of pain. So as the last sublingual administration of
nitroglycerin did not relieve pain, the patient addressed his doctor. Doctor
advised him to withdraw nitroglycerin for 8 hours and prescribed another drug.
After 8 hours the effect of nitroglycerin was restored. Which phenomenon did
take place in this case?
A. Increase in nitroglycerin metabolism
B. Increase in protein binding
C. Decrease of absorption
D. Drug dependence
E. Tolerance*.

2. Metabolism can render pharmacological activity to some initially


inert substances (pro-drug). Point out the substance, which is pro-drug, used for
treatment of hypertension.
A. Enalapril*
B. Salbutamol
C. Verapamil
D. Reserpine
E. Propranolol.

3. A patient who has been treated with diazepam on account of


neurosis complains of toothache. Doctor administered him an analgetic, but its
dose was lower than average therapeutic dose. What phenomenon did the doctor
take into account while prescribing the patient an underdose?
A. Summation
B. Potentiation*
C. Cumulation
D. Tolerance
E. Drug dependence.

4. A patient with frequent attacks of stenocardia was prescribed


sustak-forte to be taken one tablet twice a day. At first the effect was positive
but on the second day stenocardia attacks resumed. What can explain
inefficiency of the prescribed drug?
A. Tachyphylaxis*
B. Sensibilization
C. Dependence
D. Cumulation
E. Idiosyncrasy.

5. A man who has been taking a drug for a long time cannot withhold
it because this causes impairment of psychic, somatic and vegetative functions.
Name the syndrome of different disturbances caused by drug discontinuation:
A. Idiosyncrasy
B. Sensibilization
C. Tachyphylaxis
D. Abstinence*
E. Cumulation.

6. What is the mechanism of absorption of drug which is lipophilic


and unionized?
A. Active transport
B. Passive diffusion*
C. Filtration
D. Pinocytosis
E. Binding to protein.

7. Which route of administration results in 100% bioavailability of


drug?
A. Oral
B. Subcutaneous
C. Inhalational
D. Intravenous*
E. Intramuscular.

8. Chloramphenicol is highly toxic in newborn. What is the cause of


such toxicity?
A. Intensive binding to protein
B. Deficiency of metabolizing enzymes in liver*
C. Penetration into the cell
D. Inhibition of protein synthesis
E. Inhibition of nucleic acid synthesis.
9. What is the result of tubular reabsorption of drug?
A. Increase in duration of action*
B. Increase in excretion
C. Increase in distribution
D. Decrease in duration of action
E. Loss of pharmacologic activity.

10. Anticoagulant drug warfarin is bounding to plasma albumin. How


its effect can be changed in case of hypoalbuminemia?
A. Effect unchanged
B. Increased*
C. Decrease
D. Significant delayed onset of effect
E. Abolishing of effect .

11. Passive reabsorption from tubular urine (pH of 5) of a drug


administered IV would be reduced if the drug were
A. A weak base*
B. A weak acid
C. Water soluble
D. Protein bound
E. None of the above.

12. Patient suffering from angina pectoris was helping himself by


sublingual usage of nitroglycerin. One day he used the drug several times. The
last use did not relieve his pain. What phenomenon does occur?
A. Antagonism
B. Tachyphylaxis*
C. Hypersensitivity
D. Cumulation
E. Binding to protein.

13. Point out the kind of drug interaction that permits to use one of
them in lower dose.
A. Antagonism
B. Cumulation
C. Potentiation*
D. Tolerance
E. Additive synergism.

14. Patient with infection of GIT was treated with antibiotic. Which
type of therapy is it?
A. Symptomatic
B. Pathogenetic
C. Ethiotropic*
D. Replacement
E. Stimulating.

15. Alongside with therapeutic effect drugs can cause side effects. In
which dose they can occur?
A. In overdose
B. In therapeutic dose*
C. In threshold dose
D. In subthreshold dose
E. At any dose.

16. Patient suffering from gastric peptic ulcer was treated with M-
cholinoblocker. What is the mechanism of this drug action?
A. Enzymatic
B. Receptor*
C. Direct chemical
D. Physicochemical
E. Action on ion channels of membrane.

17. A patient with chronic cardiac insufficiency was prescribed digoxin


in average therapeutic dose. After 2 weeks of treatment the symptoms of
intoxication (bradycardia, extrasystole, nausea) were developed. Point out the
phenomenon responsible for intoxication.
A. Tachyphilaxis
B. Material cumulation*
C. Idiosyncrasy
D. Functional cumulation
E. Tolerance.

18. The patient with heart failure was using digitoxin. His condition
initially improved, but with time the manifestations of heart failure have
increased again, and symptoms of glycoside intoxication apeared. What is the
cause of patient's condition worsening?
A. Potentiation
B. Cumulation*
C. Tachyphylaxis
D. Sensitization
E. Idiosyncrazy.

19. Analgesic effect of methamizole(analgin) in the dental pain


develops in 0.5 hour and lasts for 1.5-2 hours. Tablets "Piraminal" in the same
situation are faster and longer acting. What is the phenomenon responsible for
difference in the action of these drugs?
A. Chemical antagonism
B. Functional antagonism
C. Undirectional antagonism
D. Potentiation
E. Summation*.
20. In the elderly patient with symptoms of senile atonic constipation
long-term treatment with drug buckthorn has led to a weaking effect. What is
this phenomenon?
A. Tolerance*
B. Cumulation
C. Sensibilization
D. Desensitization
E. Tachyphylaxis.

21. During the barbituric anesthesia in a patient respiratory depression


occurred. After intravenous injection of 10 ml 0.5% solution of bemegride the
patient's condition improved, the volume of pulmonary ventilation inceased.
Point out the phenomenon responsible for curative effect of bemegride in this
case
A. Direct synergisnm
B. Undirectional antagonism
C. Direct antagonism*
D. Indirect antagonism
E. Indirect synergism.

22. A child of 2 years admitted to the infectious deseases hospital with


a diagnosis of bacillary dysentery. What a way of enteral administration of
antimicrobial agent polymyxin is the most rational, if child has uncontrollable
vomiting?
A. Subcutaneous
B. Intravenous
C. Oral
D. Rectal*
E. Inhaled.

23. In a patient with traumatic brain injury there are such symptoms:
the obscured consciousness, inpaired respiration, filiform pulse, absence of
reflexes. Which route of drug administration for urgent aid is the most
appropriate in this case?
A. Rectal
B. Intravenous*
C. Subcutaneous
D. Oral
E. Inhaled.

24. In poisoning with mushrooms containing muscarine atropine is used


as an antidote. Which of the following mechanisms, typical for atropine, leads to
the neutralization of the poison mushroom - muscarine?
A. Antimetabolic
B. Physical and chemical
C. Antienzymatic
D. Receptor*
E. Enzymatic.

25. Prolonged use of certain drugs before pregnancy can disrupt fetal
development. What is the name of this action?
A. Fetotoxic
B. Embriotoxic
C. Teratogenic
D. Mutagenic*
E. Carcinogenic.

26. With long-term treatment by antipsychotic drugs at therapeutic


doses in patient, along with the restoration of the psychic function, some
undesired effects such as tension in the skeletal muscles, stiffness of gait, and
some autonomic disorders occurred. What the type of drug’s effects should
include these?
A. Aftereffect
B. Overdose
C. The main therapeutic effect
D. Intoxication
E. Side effects*.

27. After clinical and laboratory examination of a patient with diabetes


mellitus insulin was appointed for treatment.What is the type of this therapy?
A. Etiotropic
B. Replacement*
C. Pathogenetic
D. Symptomatic
E. Functional.

28. In order to reduce toothache a patient was recommended to take


methamisol (analgin). What the type of pharmacotherapy was used?
A. Functional
B. Pathogenetic
C. Causal
D. Substitution
E. Symptomatic*.

29. In 18 years old patient with shoulder phlegmon intramuscular


injection of penicillin was made, after which he developed tachycardia, filiform
pulse, fall of blood pressure to 80/60 mm Hg. What type of pathological
response was originated?
A. Potentiation
B. Anaphylaxis*
C. Tolerance
D. Antagonism
E. Tachyphylaxis.
Local anesthetics

30. To lance abscess doctor performed local anesthesia with therapeutic


dose of anesthetic, but it was not sufficient. What was the cause of decrease in
anesthetic effect?
A. Quick absorption of drug
B. Quick metabolism of anesthetic
C. Alkaline medium
D. Acidic pH of tissues*
E. Neutral medium.

31. Surgeon is going to perform the operation under local anaesthesia.


The probable duration of operation is more than 2 hours. In past patient was
complaining for ventricular extrasystole. Which anaesthetic is reasonable to
chose for local anaesthesia?
A. Tetracaine(decaine)
B. Procaine(novocaine)
C. Benzocaine(anaesthesine)
D. Trimecaine*
E. Cocaine.

32. Dentist revealed that his patient is suffering from hypersensitivity to


procaine. Which of the following agents may be used for local anaesthesia?
A. Benzocaine(anaesthesine)
B. Piromecaine
C. Dicaine(tetracaine)
D. Lidocaine*
E. Cocaine.

33. Which of the listed drugs is usually added to local anaesthetic to


prolong its action and to decrease toxicity?
A. Noradrenaline
B. Ephedrine
C. Adrenaline*
D. Reserpine
E. Atropine.

34. A 25-year-old woman with red and itchy eczematoid dermatitis


visits your office, She had a dental procedure one day earlier with administration
of a local anesthetic. There were no other findings, although she indicated that
she had a history of allergic reactions. Which of the following drugs is most
likely involved?
A. Procaine*
B. Etidocaine
C. Bupivacaine
D. Cocaine
E. Lidocaine.

35. For the patient with gastroenteritis doctor prescribed preparation of


tannin derivative with anti-inflammatory effect on the basis of the astringent
mechanism. Which of the following drugs was prescribed?
A. Bismuth subnitrat
B. Tannin
C. Xeroform
D. Tanalbin*
E. De-nol.

36. Patient 75 years old suffers from chronic constipation. What the
drug of a group of emollients can be assigned to this patient?
A. Senna leaf*
B. Phenolphtalein
C. Neostigmine
D. Magnesium sulfate (enterally)
E. Camphor spirit.

37. Bed sores appeared in patient on strict bed regiment. What the
irritating drug was prescribed by physician to improve blood circulation in the
area of bed sores
A. Camphor spirit*
B. Vishnevsky’s ointment
C. Wilkinson's ointment
D. A solution of potassium permanganate
E. Talc.

38. A child with acute poisoning with datura seeds doctor prescribed
oral usage of activated charcoal. Specify the mechanism of its action.
A. Adsorptions alkaloids on the surface*
B. Insoluble salts formation
C. Block of sensory receptors of the stomach
D. Precipitation of proteins on the surface of gastric mucous
membrane
E. Formation of protective layer of the colloid on the mucous
membranes.

39. The drug from the group covering agents with antacid effect was
prescribed to patients with gastric ulcer. Which of the following medications
was prescribed?
A. Tannin
B. Decoction of oak bark
C. Almagel*
D. Infusion of sage leaves
E. Bismuth subnitrate.
40. Athlete has a sharp muscular pain in the back after training. What
the irritating drug is helpful in this case?
A. The turpentine oil*
B. A solution of menthol
C. Ammonia
D. Validol
E. Chloroform.

41. Gastric lavage with a solution of tannin was performed to the


patient with alkaloid poisoning. What the concentration of tannin solution
should be used for this purpose?
A. 0.5%*
B. 0.1%
C. 0,01
D. 0,05
E. 0,1.

42. The patient suffers from hemorrhoids. For pain relief doctor
prescribed rectal suppositories with local anesthetic. Point out anesthetic used in
this dosage form.
A. Cocaine
B. Tetracaine
C. Novocaine
D. Trimekaine
E. Benzocaine (Anestesin)*.

43. Doctor applied paste with benzocaine to reduce the itching of the
forearm skin. Due to which of the following properties of benzocaine it is used
in soft dosage form?
A. Good absorption
B. Poor solubility*
C. The high degree of distribution
D. The good solubility
E. Inadequate absorption.

44. The patient will have surgery on the limbs of more than 2 hours
duration, which anesthetist plans to hold under local anesthesia. Which of the
following drugs should be selected for long-term conduction anesthesia?
A. Novocaine
B. Cocaine
C. Tetracaine
D. Benzocaine
E. Lidocaine*.

Cholinomimetics
45. Patient with complaints of dryness in the mouth, photophobia and
disturbances of vision was admitted to the reception-room. Skin is hyperemic
and dry, pupils are dilated, tachycardia. Poisoning with belladonna alkaloids was
diagnosed on further examination. What medicine should be prescribed as
antidote?
A. Diazepam
B. Prozerin (Neostigmine)*
C. Atropine
D. Pilocarpine
E. Dipyroxim.

46. Patient with pin-point pupils, sweating, salivation, abdominal


cramps, disturbances in respiration was admitted to the emergency room.
Poisoning with what substance is the most likely occurred?
A. Non-narcotic analgesics
B. Sedatives
C. Narcotic analgesics
D. М-cholinergic antagonists
E. Muscarine*.

47. In the patient, after the stroke, paralysis of hand and leg appeared.
To restore the movements of paralyzed extremities patient was treated with
physical exercises and cholinomimetics. Which of listed drugs was used for this
purpose?
A. Aceclidine
B. Pilocarpine
C. Physostigmine
D. Ambenonium chloride (oxazyl)*
E. Lobeline.

48. A known patient of myasthenia gravis came to you with his disease
well controlled by neostigmine; but he asked you to give him drug that needs
more rare usage. Choose the necessary drug.
A. Physostigmine
B. Atropine
C. Ambenonium chloride*
D. Armin
E. Pralidoxime.

49. A49-year old frequent business traveler presents to his physician


requesting medication to help him with nausea and dizziness that he gets during
turbulent flights. A scolopamine patch is prescribed for his motion sickness.
Which of the following is the most likely side effects from this patch?
A. Bradycardia
B. Diaphoresis
C. Diarrhea
D. Salivation
E. Urinary retention*.

50. In complex treatment of a child, suffering from cerebral palsy, a


doctor decided to include anticholinesterase drug moderately improving mental
development. Choose this drug.
A. Physostigmine
B. Neostigmine (Proserinum)
C. Galanthamine*
D. Pilocarpine
E. Lobeline.

51. Analeptical remedy of reflective type from the N-cholinomimetics


group was given to the patient for restoration of breathing after poisoning with
carbon monoxide, What medicine was prescribed to the patient?
A. Adrenalin hydrochloride
B. Atropine sulphate
C. Mesaton
D. Pentamin
E. Lobeline hydrochloride*.

52. A patient with complaints of dryness in the mouth, photophobia and


vision impairment was admitted in the reception-room. Skin is hyperemic, dry,
pupils, are dilated, tachycardia. Poisoning with belladonna alkaloids was
diagnosed on further examination. What medicine should be prescribed?
A. Armine
B. Dipiroxim
C. Pilocarpine
D. Diazepam
E. Prozerin*.

53. On the 2-3rd day after stomach resection intestinal peristalsis wasn't
restored. What is to be administered for stimulation of gastrointestinal tract?
A. Atropine sulfate
B. Prasosin
C. Acetylcholine
D. Proserin(Neostigmine)*
E. Cyclodole.

54. The patient developed postoperative intestinal atony. After injection


of M-cholinomimetic the patient's condition was improved. Point out the drug
that was used in this cvase
A. Arminum
B. Aceclidine*
C. Ambenonium chloride (Oxazylum)
D. Atropine
E. Platyphylline.

55. After completion of abdominal surgery conducted with the use of


nondepolarizing myoreolaxant, the patient did not restored spontaneous
breathing. What kind of drug is able to restore breathing in this case?
A. Scopolamine
B. Aceclidine
C. Pilocarpine
D. Neostigmine*
E. Izonitrozinum.

56. A child poisoned by mushrooms containing muscarine was brought


to the toxicological department Which drug should be used for emergency
assistance in the first place?
A. Papaverine
B. Sodium thiosulfate
C. Atropine*
D. Unitiol
E. Dipiroxim.

57. A patient with signs of poisoning by organophosphorus insecticides


was delivered to the hospital emergency department What medication should be
applied as a first-aid?
A. Atropine*
B. Unitiol
C. Panangin
D. Chlorpromazine
E. Glucose.

58. The patient was taken to the emergency department with complaints
of dryness in the mouth, photophobia and blurred vision. On examination
hyperemic dry skin, dilated pupils, tachycardia were revealed, and poisoning
with belladonna alkaloids was diagnosed. Which of the drugs is advisable to
apply?
A. Neostigmine*
B. Pilocarpine
C. Dipiroxim
D. Diazepam
E. Arminum.

59. The patient in the postoperative period to stimulate intestinal


motility and tone of the bladder has been appointed agent of a group of
anticholinesterase. Identify it among the following products:
A. Propranolol (Anaprilin)
B. Mannitol
C. Reserpine
D. Dichlotiazid
E. Neostigmine*.

60. The patient poisoned with carbon monoxide to restore breathing


was treated with analeptic of a reflex type from group of N-cholinomimetics.
Which drug was assigned to the patient?
A. Adrenalin hydrochloride
B. Pentamin
C. Lobeline hydrochloride*
D. Mezaton
E. Atropine sulfate.

61. Specify the M-cholinomimetic used for treatment of glaucoma:


A. Neostigmine methylsulfate
B. Pilocarpine*
C. Adrenaline
D. Norepinephrine
E. Phenylephrine (Mesatonum).

62. Which of the following medicines is anticholinesterase agent:


A. Clonidine
B. Atropine
C. Adrenaline
D. Acetylcholine
E. Neostigmine*.

63. Which group is pilocarpine?


A. M-cholinomimetics*
B. Anticholinesterase
C. Adrenomimetics
D. Adrenoblockers
E. Cholinoblockers.

64. Enter the N-cholinomimetic, which stimulates the reflex of breath:


A. Neostigmine
B. Pilocarpine
C. Lobeline*
D. Atropine
E. Adrenaline.

Cholinoblockers

65. Teenager with fracture of hand was presented to the emergency


room. To perform reposition of fractured bones it was necessary to relax skeletal
muscle. For this purpose myorelaxant causing sustained depolarization of the
post junctional membrane was administered. Which of the following agents was
used?
A. Pancuronium
B. Atracurium
C. D-tubocurarine
D. Succinylcholine*
E. Millectine.

66. Patient suffering from gastric peptic ulcer was treated with
cholinotropic drug selectively blocking M1 cholinoceptors of parietal gastric
glands. Point out this drug?
A. Atropine
B. Plathyphylline
C. Metacine
D. Pirenzepine*
E. Hyoscine (scopolamine).

67. Young boy with burns is brought to emergency ward. There is


history of death of one of his family members due to succinylcholine induction.
So we will avoid succinylcholine because it
A. Causes hyperkalemia
B. Cause hepatotoxicity
C. Causes muscle pains
D. Can trigger malignant hyperthermia*
E. Can cause spasm of masseter muscle.

68. A patient given succinylcholine for skeletal muscle relaxation


during an operation, is not recovered for the last 1 hour. What must be used to
restore spontaneous respiration?
A. Carbogen
B. Lobelin
C. Neostigmine
D. Blood transfusion*
E. Caffeine.

69. A 59 year old man develops excessive salivation and sweating,


diarrhea, and bradycardia while being treated with neostigmine for myasthenia
gravis. Which of the following is the most appropriate therapy for these
symptoms and signs?
A. Atropine*
B. Carbachol
C. Edrophonium
D. Epinephrine
E. Pralidoxime.
70. A patient with symptoms of phosphorganic poisoning was taken to
emergency department. Which of the following drugs mast be used as a drug of
first aid?
A. Unithyolum
B. Potassium chloride
C. Atropine*
D. Chlorpromazine (Aminazine)
E. Glucose.

71. After abdominal surgery performed under general anaesthesia with


non- depolarizing myorelaxant spontaneous respiration was not restored. Which
of the following drugs is to be used as an antidote of non-depolarizing
myorelaxant?
A. Aceclidinum
B. Pilocarpin
C. Neostigmine (Proserin)*
D. Hyascine
E. Isonitrosine.

72. A 5 years old child was taken to toxicological department with


Belladonna poisoning. Alongside with peripheral manifestations the CNS
symptoms were displayed. Which of the following agents has to be administered
as an antidote?
A. Aceclidin
B. Pilocarpin
C. Neostigmine (Proserin)
D. Adrenaline
E. Galanthamine hydrobromide*.

73. For testing refraction of eye atropine was instilled into conjunctival
sac. On completion of the procedure another cholinergic drug was used to
counteract mydriasis and cycloplegia, caused by atropine. Point this drug.
A. Pilocarpine*
B. Lobeline
C. Hyoscine (Scopolamine)
D. Phenylephrine (Mesatonum)
E. Pirenzepine.

74. Administration of pirenzepine in patients with gastric peptic ulcer


is not accompanied by numerous side effects characteristic for atropine and
other M-cholinoblckers due to
A. Inability to penetrate through blood brain barrier
B. Selective inhibition of M1 –cholinoceptors*
C. Inhibition of all types of M –cholinoceptors
D. Inhibition of cholinesterase
E. Significant protein binding.
75. It is known that administration of tubocurarine during surgery can
enhance the development of postoperative paralytic ileus. What is the
mechanism of this side effect of tubocurarine?
A. Inhibition of M –cholinoceptors
B. Inhibition of cholinesterase
C. Ganglion blocking activity*
D. Stimulation of M –cholinoceptors
E. Stimulation of both M – and N-cholinoceptors.

76. Which of the following conditions is alleviated by centrally acting


anti-cholinergic drugs?
A. Depression
B. Multiple sclerosis
C. Mania
D. Parkinsonism*
E. Psychosis.

77. A 50-year-old male farm worker has been brought to the emergency
room. He was found confused in the orchard and since then has remained
unconscious. His heart rate is 45 and his blood pressure is 80/40 mm Hg. He is
sweating and salivating profusely. Which of the following should be prescribed?
A. Pentamine
B. Norepinephrine
C. Physostigmine
D. Atropine*
E. Proserine.

78. Introduction of a pharmaceutical substance to an experimental


animal resulted in reduction of salivation, pupil mydriasis. Next intravenous
introduction of acetylcholine didn't lead to any significant changes of heart rate.
Name this substance:
A. Proserin
B. Atropine*
C. Propranolol
D. Adrenaline
E. Salbutamol.

79. A patient had to go through an operation. Doctors introduced him


dithylinum (listenone) and performed intubation. After the end of operation and
cessation of anesthesia the independent respiration wasn't restored. Which
enzyme deficit prolongs the action of muscle relaxant?
A. Pseudocholinesterase*
B. K-Na-adenosine triphosphatase
C. N - acetyltransferase
D. Succinate dehydrogenase
E. Carbanhydrase.
80. A patient with fracture of his lower jaw was admitted to the
maxillofacial department. It was decided to fix his bones surgically under
anaesthetic. After intravenous introduction of muscle relaxant there arose short
fibrillar contractions of the patient's facial muscles. What muscle relaxant was
applied?
A. Dithylinum*
B. Pipecuronium bromide
C. Diazepam
D. Melictine
E. Tubocurarin chloride.

81. If a man has an attack of bronchiospasm it is necessary to reduce


the effect of vagus on smooth muscles of bronchi. What membrane
cytoreceptors should be blocked for this purpose?
A. M-cholinoreceptors*
B. beta-adrenoreceptors
C. alfa- and beta-adrenoreceptors
D. alfa-adrenoreceptors
E. N-cholinoreceptors.

82. The patient with bronchial asthma uses inhaled form M -


holinoblocker. In this case, as a rule, characteristic side effects such as blurred,
constipation tachycardia do not appear. Point out the drug that patient uses.
A. Platifillin
B. Atropine
C. Metacin
D. Ipratropium Bromide*
E. Pirenzepine.

83. For the treatment of patients with essential hypertension doctor


prescribed the drug from the group M – cholinoblocker. Enter the drug.
A. Platyphylline*
B. Atropine
C. Scopolamine
D. Metacin
E. Gastrotsepin.

84. The drug selectively blocking M1 - cholinergic receptors was


assigned to patient suffering from gastric peptic ulcer. What this drug is?
A. Pirenzepine*
B. Atropine
C. Platyphylline
D. Methacinum
E. Scopolamine.

85. A patient with a fracture of the mandible was delivered to the


Maxillofacial department. It was decided to connect the bones surgically under
general anesthesia. After intravenous injection of muscle relaxant transient
fibrillar contractions of facial muscles were observed. What kind of muscle
relaxant was used?
A. Dithylinum*
B. Tubocurarine chloride
C. Pipecuroniub bromide
D. Mellictinum
E. Diazepam.

86. Dentist in order to reduce salivation during the filling of the tooth
prescribed the drug to the patient. What the medicine was prescribed?
A. Adrenalin hydrochloride
B. Phenylephrine (Mezaton)
C. Neostigmine
D. Pilocarpine hydrochloride
E. Atropine sulfate*.

87. Iintroduction of the drug in experimental animals decreases


salivation, dilates pupils, and on the subsequent intravenous injection of
acetylcholine heart rate did not change significantly. Specify the name of the
substance:
A. Propranolol (Anaprilin)
B. Neostigmine methylsulfate
C. Adrenaline
D. Salbutamol
E. Atropine*.

88. Dithylinum was injected to the patient before surgery and


intubation was performed. After the operation, and termination of anesthesia
spontaneous breathing was not restored. Deficiency of which enzyme is
responsible for prolongation of this muscle relaxant action?
A. Carbonic anhydrase
B. Pseudocholinesterase*
C. Succinate dehydrogenase
D. K-Na-ATPase
E. N-acetyltransferase.

89. The preparation of the depolarizing neuromuscular blocking agents


should be assign to the patient with a broken limb for short-lasting surgical
procedure. Choose the drug.
A. Atropine sulfate
B. Pentamin
C. Dithylinum*
D. Cytitonum
E. Tubocurarine chloride.
90. In a patient with myasthenia after the treatment with neostigmine
nausea, diarrhea, and hypersalivation appeared. Point out the drug to eliminate
these side effects.
A. Atropine Sulfate*
B. Phenylephrine (Mezaton)
C. Physostigmine
D. Isoprenaline
E. Pyridostigmine bromide.

91. In poisoned patient dryness of mucous membranes of the mouth and


dilated pupils were observed. What is the mechanism of these symptoms
development?
A. Stimulation of N-cholinergic receptors
B. Stimulation of the M-cholinergic receptors
C. Adrenoceptor blockade
D. M-cholinergic receptor blockade*
E. Stimulation of adrenergic receptors.

92. In the patient with a forearm injury during the bone repositioning
Dithylinum was used for muscle relaxation. Full recovery of muscle tone and
function took more than an hour. What can be the cause of significant
lengthening of the drug’s action?
A. Genetic deficiency of monoamine oxidase
B. Genetic deficiency of butyrylcholinesterase*
C. Inhibition of microsomal oxidation
D. Genetic deficiency of hydroxylase
E. Formation of the active metabolite.

Adrenomimetics and sympathomimetics

93. A patient with bronchial asthma is administered inhalation of 0,5


percent isadrine solution. Bronchospasm was relieved but the patient began
complaining of pain in the heart region and palpitation. It is connected with the
stimulation of :
A. alpha one – adrenoreceptros
B. Beta two – adrenoreceptros
C. Beta one – adrenoreceptros*
D. Acetylchloline synthesis
E. M-chlolinoceptors.

94. Patient with bronchial asthma was taking tablets, which caused
insomnia, headache, increased blood pressure. What medicine can cause such
complications?
A. Chromolin sodium
B. Adrenaline
C. Euphyline
D. Ephedrine*
E. Izadrine.

95. Spasm of smooth muscle of bronchi developed in the patient. Usage


of activators of what membrane cytoreceptors is physiologically valid to
decrease attack?
A. alpha-аdrenoreceptors
B. beta-adrenoreceptors*
C. alpha-and- beta-аdrenoreceptors
D. N-cholinoreceptors
E. М-cholinoreceptors.

96. In the treatment of patient suffering from congestive heart failure


resistant to digitalis adrenomimetic was used as inotropic agent. Point out which
of the following drugs was administered.
A. Phenylephrine
B. Partusisten
C. Dobutamine*
D. Xylometazoline
E. Naphazoline.

97. 63 years patient with symptoms of collapse was admitted into


emergency department. Doctor has chosen noradrenaline to overcome collapse.
What mechanism is responsible for therapeutic effect of this drug?
A. Activation of alfa1 receptors*
B. Activation of beta adrenoceptors
C. Activation of dopaminergic receptors
D. Blockade of M –cholinoceptors
E. Activation of serotonin receptors..

98. Dopamine increases urine production in a shocked patient because


it:
A. Increase cardiac output
B. Decreases aldosterone release
C. Decreases ADH release
D. Causes peripheral vasodilation
E. Causes renal arterial dilation*.

99. To perform fundoscopy ophthalmologist instilled in the eye an


agent capable of causing mydriasis without cycloplegia. Point out this agent.
A. Phenylephrine (Mesatonum)*
B. Noradrenaline
C. Atropine
D. Pilocarpine
E. Isoprenaline (Isadrinum).
100. A patient with bronchial asthma addresses his doctor with
complaints about unpleasant palpitations that occur after usage of inhalation
form of isoprenaline (isadrinum).What is the cause of this side effect?
A. stimulation of alfa1– adrenoceptros
B. stimulation of alfa2– adrenoceptros
C. stimulation of beta1– adrenoceptros*
D. stimulation of beta2– adrenoceptros
E. inhibition of M-chlolinoceptors.

101. A patient with moderate hypertension was treated with the drug that
realizes its effect by decrease in noradrenaline release from sympathetic nerve
endings. While treatment patient experienced drowsiness. What the drug was
used?
A. Prazosin
B. Reserpin*
C. Drotaverine
D. Atenolol
E. Dichlorothiazide.

102. Surgical treatment of benign tumor of prostate in 67- year patient


was postponed instead pharmacotherapy with adrenergic drag was started. Point
out which of the drugs was most probably used.
A. Adrenaline
B. Ephedrine
C. Prazosine*
D. Propranolol (Anaprilinum)
E. Atenolol .

103. A patient has got a spasm of smooth muscles of bronchi. Blockers


of what membrane cytoreceptors are physiologically reasoned to stop an attack?
A. beta-adrenoreceptors
B. M-cholinoreccptors*
C. alpha- and beta-adrenoreceptors
D. H-cholinoreceptors
E. alpha -adrenoreceptors.

104. A patient ill with bronchial asthma didn't inform his doctor that he
had attacks of stenocardia. Doctor administered him a medication, which taking
resulted in less frequent attacks of bronchial asthma. but stenocardia attacks
became more frequent. What medication was administered?
A. Cromolyn sodium
B. Salbutamol
C. Aminophylline
D. Phenotherol
E. Isadrin*.
105. An ophthalmologist used a 1% mesaton solution for the diagnostic
purpose (pupil dilation for eve-ground examination). What is the cause of
mydriasis induced by the drug?
A. Activation of alfa1 adrenoreceptors*
B. Activation of beta1 adrenoreceptors
C. Activation of M-cholinoreceptors
D. Block of alfa1 adrenoreceptors
E. Activation of alfa2 adrenoreceptors.

106. At the doctor's the dentist- surgeon injected 1% - procaine for


conduction anesthesia. It caused the change in patient’s condition: the skin was
covered with sticky and cold sweat, the pulse became almost impalpable,
breathing became rare and weak. Realizing that anaphylactic shock is originated,
a doctor immediately introduced intravenously drug that promptly improved the
patient's condition. Which of the following drugs can be used for first aid in
severe allergic reactions?
A. Ipratropium bromide
B. Labetalol
C. Fenoterol
D. Adrenaline*
E. Dobutamine.

107. A patient with heart failure, resistant to cardiac glycosides was


admitted to hospital. A doctor appointed adrenomimetic as inotropic
(cardiotonic) drug. Which of these adrenomimetic has inotropic (cardiotonic)
action?
A. Dobutamine*
B. Phenylephrine (Mezaton)
C. Naphazoline
D. Fenoterol
E. Halazolin.

108. In the maternity ward there is a pregnant woman with the danger of
miscarriage. Which of the following agents can be used to save the pregnancy?
A. Reserpine
B. Isoprenalin
C. Nicergoline
D. Ephedrine
E. Fenoterol (Partusisten)*.

109. The patient went to the doctor for medical examination. In the
queue while waiting an attack of asthma originated. Which group of drugs is
advisable to use to provide first aid?
A. N - cholinomimetics
B. Sympathomimetics
C. Beta-adrenomimetics*
D. Beta-adrenoblockers
E. M - cholinomimetics.

110. In anesthetic practice adrenaline is usually added to the local


anesthetics to prolong analgesia and reduce side effects. What the effect of
adrenaline contributes to this?
A. Reducing the sensitivity of pain receptors
B. Constriction of blood vessels*
C. Increase in peripheral resistance
D. Stimulation of the antinociceptive system
E. CNS depression.

111. In the course of therapy of a patient a number of complications such


as mydriasis, tachycardia, and increased blood pressure have occurred. Which of
the listed drugs can cause such side effects?
A. Propranolol (Anaprilin)
B. Adrenaline*
C. Phentolamine
D. Isoprenaline
E. Norepinephrine.

112. A patient made an inhalation of medication to relieve an attack of


asthma it quickly improved patient condition but caused tachycardia. Which of
the following agents can lead to such a complication?
A. Fenoterol
B. Atenolol
C. Metoprolol
D. Isoprenaline*
E. Salbutamol.

113. A pregnant woman with a danger of abortion is admitted to


obstetric department. Which of these drugs should be applied to save the
pregnancy?
A. Salbutamol*
B. Oxytocin
C. Neostigmine methylsulfate
D. Dinoproston
E. Quinine.

114. An ambulance was called to women 28 years old in connection with


an attack of asthma. After intramuscular injection of ephedrine her condition
was improved. However, after 49 minutes the attack was repeated, ephedrine
injected once more acted weaker, in the third attack of bronchospasm that
occurred in half an hour, ephedrine did not cause any effect.What is the name of
this phenomenon?
A. Addiction
B. Biotransformation
C. Elimination
D. Adsorption
E. Tachyphylaxis*.

115. In a patient during a visit to the dentist an acute hypotension


occurred. Which of the drugs that stimulate adrenergic structures should be used
to normalize blood pressure?
A. Naphazoline
B. Xylometazoline
C. Phenylephrine (Mezaton)*
D. Ergotamine
E. Doxozasin.

116. In the experimental modeling of mechanism of inotropic effect of


adrenaline activation of one of listed receptors was revealed. Point out these
receptors.
A. M-Cholinergic receptors
B. Beta-1 - adrenoceptors*
C. Beta 2 - adrenergic receptors
D. Alpha 2 - adrenergic receptors
E. Alpha 1 - adrenergic receptors.

117. A patient with acute rhinitis was often applying intranasally 5%


solution of ephedrine hydrochloride. The effectiveness of the drug was declining
with each usage and on the third day disappeared entirely? What is the cause of
rapid decrease in drug’s efficacy (tachyphylaxis)?
A. The progressive decrease in noradrenaline storage in sympathetic
nerve endings*
B. Violation of the synthesis of norepinephrine in the sympathetic
nerve endings
C. The increase in the reuptake of neurotransmitter
D. Adrenoceptor blockade
E. Violation of the process of neurotransmitter release from the depot.

Antiadrenergic drugs sympatholytics

118. The patient with a history of asthma have developed atrial


fibrillation, that requires agent appointment. Which of the listed
antiarrhythmic drug is contraindicated in this case?
A. Nifedipine
B. Verapamil
C. Propranolol (Anaprilin)*
D. Procainainamide
E. Ajmaline.
119. A patient with fibrillation, who has had bronchial asthma in his
anamnesis, should be administered antiarrhythmic drug. Which preparation from
the suggested group is contraindicated for this patient?
A. Ajimaline
B. Verapamil
C. Novocainamide
D. Nifedipine
E. Propranolol (Anapriline )*.

120. 72-year woman suffering from angina pectoris was treated by


propranolol (anaprilinium). On the background of the treatment the cough with
bronchospastic components was developed. Doctor replaced propanolol by
atenolol. It results in abolishing of respiratory disturbances. What is the reason
of beneficial effect of atenolol ?
A. Block of M-cholinoreceptors
B. Absence of action upon beta2- adrenoceptors*
C. Stimulating of M-cholonoreceptorsd
D. Inhibition of m-cholinireceptors
E. Absence of action on alfa- adrenoceptors.

121. Doctor prescribed selective alfa1-adrenoceptor blocker for


treatment of hypertensive patient. Which of the following drugs belongs to this
group?
A. Metoprolol
B. Phentolamine
C. Cordarone
D. Tropaphan
E. Doxazosine*.

122. Patient with angina pectoris was treated with drug decreasing
oxygen demands of myocardium. While treatment, patient starts to complain of
extremities coldness especially fingers. Which of the drug was used?
A. Atenolol
B. Propanol (anaprilin)*
C. Reserpine
D. Prazosin
E. Labetalol.

123. Patient with Raynaud disease was treated with phentolamine. What
is the mechanism of action of this drug?
A. Blockade of B-adrenoceptors
B. Blockade of alpha adrenoceptors*
C. Decrease in neurotransmitter amount
D. D Stimulation of central alpha2 adrenoceptors
E. Blockade of central M-cholinoceptors.
124. Patient with initial form of hypertension complains of pain in heart
and tachycardia. Which of the following drugs has to be used in the treatment of
this patient
A. Phentolamine
B. Dihydroergotamine
C. Propranolol (Anaprilin)*
D. Prazosin
E. Guanethidine(Octadinum).

125. Patient having in the past episodes of bronchial asthma addressed


doctor with angina pectoris. Doctor decided to treat patient with B-
adrenoblocker. Which of B-adrenoblocker is not contraindicated in this case?
A. Anapriline(Propranolol)
B. Atenolol*
C. Oxprenolol
D. Pindolol
E. Nadolol.

126. Middle aged woman addressed doctor complaining on the attack of


periodical headache. On examination migraine was diagnosed. Doctor
prescribed anti-adrenergic drug. Point out the most suitable drug.
A. Phentolamine
B. Dihydoergotamine*
C. Atenolol
D. Dopamine
E. Reserpine.

127. For treatment of hypertensive patient doctor prescribed


antiadrenergic drug, first administration of which in therapeutic dose caused
“first dose syncope”. Which drug was most probably used?
A. Metoprolol
B. Atenolol
C. Prazosin*
D. Reserpin
E. Propranolol.

128. A 43 year-old male patient is suffering from hypertension. His


blood pressure was successfully controlled by monotherapy with adrenoblocker.
With time treatment was complicated with diarrhoea and impotence. The patient
addressed his doctor and the drug was changed for another adrenoblocker.
Which of adrenoblockers can cause above listed side effects?
A. Metoprolol
B. Salbutamol
C. Propranolol
D. Dobutamine
E. Prazosin*.
129. In the presence of a beta-blocker, administration of adrenaline is
most likely to cause:
A. Hypoglycemia
B. Hypokalemia
C. Hypertension*
D. Hyperglycemia
E. Hypotension..

130. Doctor prescribed selective alfa1-adrenoceptor blocker for


treatment of hypertensive patient. Which of the following drugs belongs to this
group?
A. Metoprolol
B. Phentolamine
C. Cordarone
D. Tropaphan
E. Prazosin*.

131. A 62-year-old patient, prone to bronchoconstriction, addressed his


doctor with complaints of tachyarrhythmia troubling him several times a day.
The doctor prescribed anaprilin (propranolol). Administration of drug provoked
severe attack of bronchospasm. What is the mechanism of this side effect?
A. Stimulation of M1- cholinoceptors
B. Inhibition of alfa-adrenoceptors
C. Inhibition of beta2 - adrenoceptors*
D. Increase in n. vagus tone
E. Stimulation of leukotriene release.

132. A young woman with 6 months pregnancy visited her doctor


complaining of periodical slight contractions of uterus. To prevent premature
labour doctor prescribed partusisten (fenoterol). What is the mechanism of
action of this drug?
A. Stimulation of beta2 - adrenoceptors*
B. Inhibition of beta2 - adrenoceptors
C. Stimulation of alfa1 - adrenoceptors
D. Inhibition of alfa1 - adrenoceptors
E. Stimulation of M-cholinoceptors.

133. The patient with the initial form of hypertension complains of pain
in the heart and tachycardia. Which of the following drugs should be used in the
treatment of this patient?
A. Prazosin
B. Phentolamine
C. Dihydroergotamine
D. Propranolol (Anaprilin)*
E. Reserpine.
134. A doctor prescribed the drug to the patient with hypertension. The
drug is acting by selective block of alpha 1 - adrenergic receptors. What the drug
was prescribed?
A. Metoprolol
B. Doxazosin*
C. Phentolamine
D. Propranolol (Anaprilin)
E. Reserpine.

135. The patient in case of angina chest pain regularly takes


nitroglycerin, however, recently angina pectoris complicated by extrasystoles
and episodes of bronchospasm. Which of beta - blockers better to appoint for
such a patient?
A. Timolol
B. Propranolol (Anaprilin)
C. Oxprenolol
D. Atenolol*
E. Nadolol.

136. Patients with essential hypertension (stage II) used one of


antihypertensive drugs. After a while blood pressure decreased, but the patient
began to complain of fatigue, sleepiness, apathy. Later on, a pain in the stomach
occurred, and gastric peptic ulcer was diagnosed. What the drug was used?
A. Reserpine*
B. Verapamil
C. Captopril
D. Dibazol
E. Furosemide.

137. A patient with ischemic heart disease was admitted to cardiology


department. To prevent attacks of angina pains drug of beta-1adrenoblockers
was assigned. Name this drug:
A. Metoprolol*
B. Morphine hydrochloride
C. Furosemide
D. Oxytocin
E. Atropine sulfate.

138. Hypertensive patients is taking sympatholytic. Enter the drug.


A. Propranolol (Anaprilin)
B. Reserpine*
C. Adrenaline
D. Atropine
E. Phenylephrine (Mesatonum).

139. Note the non-selective beta-blocker.


A. Mezatonum
B. Atenolol
C. Adrenaline
D. Salbutamol
E. Propranolol (Anaprilin)*.

140. The patient with hypertension was appointed cardioselective beta-


blocker. Which drug is assigned to the patient?
A. Reserpine
B. Propranolol (Anaprilin)
C. Metoprolol*
D. Atropine
E. Adrenaline.

141. To patient with a tachyarrhythmia doctor prescribed a beta-blocker.


Enter the drug.
A. Adrenaline
B. Propranolol (Anaprilin)*
C. Atropine
D. Isoprenaline
E. Salbutamol.

142. The patient suffering from angina takes a selective beta-


adrenoblocker. Enter the drug.
A. Metoprolol*
B. Prazosin
C. Atropine
D. Pilocarpine
E. Propranolol (Anaprilin).

143. Non selective alpha-adrenoblocker is prescribed for diagnostic


purposes to a woman with a suspected pheochromocytoma. Point out this drug
A. Fentolamin*
B. Propranolol (Anaprilin)
C. Salbutamol
D. Phenylephrine (Mesatonum)
E. Clonidine.

The drugs for general anaesthesia

144. To anaesthetize the manipulation connected with burn surface


treatment, a patient was intravenously injected a medication for short-acting
narcosis.1 minute later the patient being under anesthesia had elevated blood
pressure, tachycardia, increased tone of skeletal muscles; the reflex were
reversed. After awakening the patient had disorientation and the visual
hallucination. What medicine was injected to the patient?
A. Sombrevin
B. Thiopental sodium
C. Nitrous oxide
D. Ketamine*
E. Diethyl ether.

145. The anaesthetic used along with halothane to give excellent


analgesia and rapid induction is
A. Nitrous oxide*
B. Thiopental
C. Desflurane
D. Propofol
E. Choroform.

146. To anaesthetize the manipulation related to burn surface treatment,


a patient was intravenously injected a medication for short-acting narcosis. 1
minute later the patient being under anaesthesia had increased blood pressure,
tachycardia, increased tone of sceletal muscles reflexes remained. After
awakening the patient had desorientation and visual hallucinations. What
medication was the patient injected?
A. Nitrous oxide
B. Thiopental sodium
C. Ketamine*
D. Diethyl ether
E. Sombrevin.

147. Dental surgeon made tooth extraction in a patient under general


anesthesia. Which of the following medications can be used in this case for
short-term noninhalation general anesthesia?
A. Ketamine*
B. Predion
C. Thiopental sodium
D. Halothane
E. Nitrogen oxide.

148. To make the treatment of burn surface painless intravenous general


anesthetic for short-term anesthesia was injected to patient. Anesthesia
developed in 1 minute during anesthesia increase in blood pressure, tachycardia,
increase in tone of skeletal muscles have occurred. On the recovery from
anesthesia patient experienced disorientation and visual hallucinations. What is
the drug entered the patient?
A. Ether for anesthesia
B. Sombrevin
C. Ketamine*
D. Thiopental sodium
E. Nitrous oxide.
149. To do a biopsy of breast tumors in 35 years old woman analgesia is
required. What type anesthesia should be used?
A. Potentiated anesthesia with muscle relaxants
B. Combined mixed anesthesia
C. Neurolept-analgesia*
D. Basic anesthesia (Na-hydroxybutyrate)
E. Inhalation anesthesia.

150. A 28 years old woman addressed the dentist about dental


prosthetics. Because she is allergic to local anesthetics, dental treatment was
necessary to carry out under general anesthesia. Which general anesthetic drug
should be used for this purpose, taking into consideration short duration of
manipulation?
A. Sodium hydroxybutyrate
B. Ketamine*
C. Thiopental
D. Halothane
E. Hexobarbital (Hexenalum).

151. To anesthetize the procedure of taking a biopsy in patient a drug


that causes a fast anesthesia lasting about 5 minutes was injected intravenously.
During anesthesia there were involuntary movements, slight decrease in blood
pressure, short-term respiratory arrest, which was quickly restored. What kind of
drug was used?
A. Halothane
B. Ketamine
C. Predion
D. Propanidid*
E. Sodium hydroxybutyrate.

152. Generalized tonic-clonic seizures with loss of consciousness were


periodically occurring in a patient after an injury; these seizures were followed
by a general depression of central nervous system. What the drug should be
assign to prevent seizures?
A. Trihexyphenidyl (Cyclodolum)
B. Midantan
C. Levodopa
D. Disilfiram (Teturam)
E. Phenobarbital*.

153. Identify the drug that has sedative, hypnotic, anesthetic, anti-
hypoxic, myorelaxant actions and it is used as general anesthetic and hypnotic.
A. Sodium hydroxybutyrate*
B. Hexobarbital
C. Chloral hydrate
D. Diazepam
E. Thiopental sodium.
154. The patient takes disulfiram (teturam). What is the indication for
use of this drug?
A. Psychosis
B. Preparation for surgery
C. Fear and anxiety
D. Neurosis
E. Alcoholism*.

Hypnotic, antiepileptic and antiparkinsonian drugs

155. A 5 years old child from time to time have seizures with loss of
consciousness. The doctor diagnosed epilepsy (large seizures). Which drug is
advisable to appoint a patient?
A. Phenobarbital*
B. Benactizin(Amizulum)
C. Trihexyphenidyl (Cyclodolum)
D. Ethosuximide
E. Levodopa.

156. A man aged 68 with trembling hands and incoordination diagnosed


with Parkinson's disease. Which drug is advisable to appoint a patient?
A. Phenytoin
B. Finlepsin
C. Ethosuximide
D. Phenobarbital
E. Trihexyphenidyl (Cyclodolum)*.

157. 40 years woman addressed doctor with complaints of psychic


disturbances appeared with long-term usage of phenobarbital as a hypnotic.
What the drug should be used instead of phenobarbital?
A. Sodium hydroxybutyrate
B. Bromizoval
C. Chloral hydrate
D. Nitrazepam*
E. Noxiron.

158. Due to emotional stress a man permanently is in a state of nervous


tension and has poor sleep. The doctor gave him a diazepam. Which of the
following effects of diazepam is the base for its clinical application in this case?
A. Hypotensive
B. Anxiolytic (Tranquilizing)*
C. Analgesic
D. Anticonvulsant
E. Antidepressant.
159. A patient suffering from insomnia addressed his doctor. The main
problem was in the difficulties with sleep onset. Doctor prescribed zolpidem.
Which of the following characteristics is true for this agent?
A. It belongs to benzodiazepine derivatives
B. It belongs to barbiturates
C. It has the ability to cause quick onset of sleep*
D. It’s poorly absorbed from GIT
E. It’s poorly bind to protein.

160. A 46- year-old patient with a history of epilepsy (grand mal)


developed status epilepticus. Choose the drug to arrest convulsions.
A. Sodium valproate
B. Diazepam*
C. Levodopa
D. Chlorpromazine
E. Droperidol.

161. An antiepileptic drug, which prevents metabolism of GABA, was


included in complex therapy of patient with epilepsy (grand mal). Point out this
drug
A. Phenytoin
B. Phenobarbitone
C. Carbamazepine
D. Ethosuximide
E. Sodium valproate*.

162. Patient with phenobarbital poisoning was presented to emergency


department. To promote kidney excretion of poison sodium bicarbonate was
administered intravenously. What mechanism is responsible for an increase in
barbiturate excretion?
A. Increase in polarity of barbiturate*
B. Increase in fat solubility of barbiturate
C. Increase in barbiturate reabsorption
D. Increase in rate of glomerular filtration
E. Decrease in protein binding.

163. Deficiency in dopamine production by neurons of strio-pallidal


system is considered to be one of the main causes of Parkinson’s disease. Point
out the drug used for replacement therapy of this disease
A. Atropine
B. Dopamine
C. Levodopa*
D. Carbidopa
E. Amantadine.
164. The patient was taken the mixture prescribed by neuropathologist
for neurasthenia for 2 weeks. Patient felt better but developed coryza,
conjunctivitis, rash, inertia, decrease of memory. Bromizm was diagnosed.
What should be prescribed to decrease symptoms?
A. Glucose solution 5%
B. Calcium chloride
C. Natrium chloride*
D. Polyglucin
E. Asparcam.

165. The antiepileptic drug which causes nystagmus, ataxia and gum
hypertrophy is
A. Phenytoin*
B. Phenobarbitone
C. Carbamazepine
D. Ethosuximide
E. Valproic acid.

166. A 16 years boy is a known as patient suffering from epilepsy.


Following several years of a drug therapy, gingival hyperplasia developed. For
which drug this side effect is characteristic?
A. Alprazolam
B. Carbamazepine
C. Valproic acid
D. Ethosuximide
E. Phenytoin*.

167. A woman, which is on phenytoin, wants to conceive. What advice


you can give to her about drug?
A. Increase the dose of phenytoin
B. Change to valproic acid
C. Addition of valproic acid
D. Change to phenobarbitone*
E. Decrease the dose of phenobarbitone.

168. An old epileptic patient on phenytoin is having status eplilepticus.


What will you give to this patient in emergency?
A. Phenytoin
B. Carbamazepine
C. Diazepam*
D. Valproic acid
E. Primidone.

169. Disulfiram is widely used in medical practice to prevent


alcocholism. It inhibits aldehyde dehydrogenase. Increased level of what
metabolite causes aversion to alcochol?
A. Propionic aldehyde
B. Methanol
C. Ethanol
D. Acetaldehyde*
E. Malonyc aldehyde.

Pharmacology of opioid analgesics

170. Very painful uterine contractions have been occurring in a pregnant


woman at normal term. To relieve the woman’s condition during labour, a
doctor decided to use opioid (narcotic) analgesic. Which of the listed drugs can
be used in this case?
A. Trimeperidine (Promedolum)*
B. Morphine
C. Metamizole (Analginum)
D. Papaverine
E. Codeine.

171. A 60-year-old male is brought to emergency room. He is comatose


and his pupils are constricted. Physician suspects opium overdose. What is the
best drug to be administered?
A. Flumazanil
B. Calcium carbonate
C. Sodium bicarbonate
D. Naloxone*
E. Atropine.

172. A young man was taken to emergency department with numerous


traumas of chest and head. Anesthesiologist proposed to inject morphine to
relieve patient’s condition, but traumatologist rejected proposition of his
colleague. Why morphine is contraindicated in this case?
A. It increases intracranial pressure*
B. It stimulates vagal center
C. It decreases intraocular pressure
D. It causes miosis
E. It depresses center of cough reflex.

173. It is known that morphine can cause a shift of blood from


pulmonary to systemic circuit. It results in lowering of resistance in pulmonary
vascular bed. In which case in may be used?
A. Bronchitis
B. Pneumonia
C. Pulmonary oedema*
D. Bronchiectatic disease
E. Emphysema.
174. Patient in unconscious state was admitted to the emergency room.
Skin is cold, pupils are pinpoint like, breathing is heavy, with cycles of the
Cheyne- Stokes type, blood pressure is decreased, urinary bladder is overloaded.
Poisoning with what substance is the most likely?
A. Neosigmin (Proserinum)
B. Sedatives
C. M-cholinergic antagonists
D. Nor-narcotic analgesics
E. Narcotic analgesics*.

175. Examination of a patient revealed extremely myotic pupils,


sleepiness, infrequent Chain-Stoke's respiration. urinary retention, slowing-
down of heart rate, enhancement of spinal reflexes. What substance caused the
poisoning?
A. Atropine
B. Phosphacole
C. Caffeine
D. Morphine*
E. Barbital.

176. A patient with acute morphine poisoning was delivered to a


hospital. What specific narcotic antagonist should be chosen in this case?
A. Unithiol
B. Paracetamol
C. Digoxin
D. Methacin
E. Naloxone*.

177. A young man with an addiction to opioid drugs denies drug


dependence. Introduction of one of these drugs will help to prove drug
dependence. Point out this drug.
A. Morphine
B. Codeine
C. Trimeperidine (Promedol)
D. Fentanyl
E. Pentazocine*.

178. A 30 years old patient was delivered to emergency department with


a fractured thigh bone after a car accident. The patient had dramatically reduced
blood pressure to 70/40 mm Hg, weak pulse, enhanced pain response to touch to
damaged area. What should be used for prevention of traumatic shock in
patient?
A. Metamizol (Analgin)
B. Papaverine
C. Morphine*
D. Paracetamol
E. Pentazocine.
179. The patient with inoperative lung cancer metastasizing in the spine
was suffering from severe pain. To relieve patient's condition analgesic is
required. Which drug should be administered?
A. Analgine
B. Codeine
C. Paracetamol
D. Morphine*
E. Papaverine.

180. On arrival of ambulance a 25 years old patient was unconscious and


15 vials of morphine solution were found near him patient had periodic
breathing, slowed heart rate, suppressed reflexes except for the knee, and
narrowed pupils he was taken to the emergency department. What measure
should be applied in the first place?
A. Gastric lavage*
B. Diuretic
C. Antiarrhythmic drug
D. Enema
E. Induction of vomiting.

181. A child of 4 years is hospitalized to the orthopedic department with


a fractured ankle with displacement. The reposition of bone fragments required
analgesia. What medication should be used?
A. Analgin
B. Promedol*
C. Morphine hydrochloride
D. Panadol
E. Omnopon.

182. Choose a specific antagonist of narcotic analgesics.


A. Caffeine-sodium benzoate
B. Ethimizol
C. Bemegride
D. Strychnine
E. Naloxone*.

183. A patient with symptoms of renal colic is admitted to the urology


department. Which of these drugs should be advised to provide him first aid?
A. Morphine
B. Trimeperidine (Promedol)*
C. Pentazocine
D. Fentanyl
E. Tramadol.
184. A patient with dislocation of shoulder was admitted to
traumatologic department. The reduction of dislocation should be performed
under neuroleptanalgesia. Chose the opioid analgesic used for this purpose.
A. Naloxone
B. Cordiamin
C. Morphine
D. Ethimizol
E. Fentanyl*.

Pharmacology of nonopioid analgesics

185. A 67 year-old male patient suffering from arthritis was treating


himself with acetyl salicylic acid (aspirin). In 2 weeks patient addressed doctor
complaining of pain in epigastric area. On examination of the patient doctor
prescribed anti-inflammatory agent, selectively inhibiting COX–2. Point out this
drug.
A. Paracetamole
B. Diclophenac-sodium
C. Celecoxib*
D. Metamisole (Analginum)
E. Indomethacine.

186. In complex treatment of a patient with angina pectoris non- narcotic


analgesic exerting antiplatelet activity was included. Point out this agent.
A. Meperidine
B. Morphine
C. Tramadol
D. Metamizole
E. Aspirin*.

187. Due to inhibition of COX aspirin exerts all the following effects
except
A. Anti-inflammatory
B. Antiplatelet
C. Increase in warfarin efficacy*
D. Antipyretic
E. Analgesic.

188. A patient with toothache was relieving his pain with help of
metamizole (analginum). Point out another useful effect of this drug that
contributes to the improvement of patient’s condition
A. Sedative effect
B. Anti-inflammatory effect*
C. AntIplatelet effect
D. Antioxidative effect
E. Antimicrobial effect.

189. The usage of acetylsalicylic acid resulted in occurrence of


epigastric pain in patient. On examination of the patient gastric ulcer was
revealed. What the mechanism is responsible for ulcerogenity of this drug?
A. Stimulation of pepsin secretion
B. Inhibition of prostaglandin synthesis*
C. Increase in bile production and its reflux
D. Spasm of vessels
E. Spasm of gastric smooth muscles.

190. Doctor visited a 5-year-old child with fever and complaints of


headache. Doctor suspected the onset of viral infection. To relieve patient’s
condition doctor prescribed non-opioid analgesic, which additionally exerts
antipyretic effect but lacks anti-inflammatory action. Point out the prescribed
drug
A. Metamizole (Analginum)
B. Celecoxib
C. Aspirin
D. Acetaminophen (Paracetamol)*
E. Diclophenac-sodium.

191. After the usage of acetylsalicylic acid a patient developed epigastric


pain because of exacerbation of his ulcer. What is the mechanism of this drug
ulcerogenity?
A. Stimulation of pepsin secretion
B. Antiprostagladinic effect*
C. Cholagogic effect
D. Spasm of vessels
E. Immunodepressive effect.

192. Point out non- narcotic analgesic, which exerts antiplatelet activity.
A. Meperidine
B. Morphine
C. Tramadol
D. Metamizole
E. Aspirin*.

193. A 2-year-old child is often catching a cold with fever. Which


antipyretic drug is safer at this age than acetylsalicylic acid?
A. Analgin
B. Indomethacin
C. Paracetamol*
D. Phenylbutazone
E. Phenylbutasone.
194. A 7 year child have caught cold, he was shivering, fever up to 39,2
° C, his respiratory rate - 45 per minute, pulse - 105 beats per minute. The doctor
diagnosed an acute viral infection, and appointed a baby aspirin. By means of
what mechanism the drug realized its antipyretic effect in this case?
A. The overall calming effect
B. Slowing down heartbeat
C. Increase in heat output*
D. Slowing of the respiratory rhythm
E. Reduction of heat production.

Neuroleptics, antianxiety, and sedative agents

195. A patient was treated for psychosis for 2 weeks. The psychic
patient's condition improved but rigidity, tremor, and hypokinesia developed.
Which of the drugs can cause such complications?
A. Diazepam
B. Chlorpromazine (Aminazine)*
C. Amytriptiline
D. Piracetam
E. Chlordiazepoxide.

196. A 60 years old woman addressed her doctor complaining of side


effects which appeared while treatment with chlorpromazine (aminazinum). She
was troubled with tremor and disturbances of movements. What is the
mechanism of this side effect?
A. Activation of hyppocampus
B. Inhibition of reticular formation (alfa1 - adrenoceptors)
C. Inhibition of neostriatum (D2 receptors)*
D. Inhibition of hypothalamus
E. Inhibition of hyppocampus.

197. A 50 years old patient with chronic alcoholism became aggressive.


To abolish aggression, chlorpromazine was administered intramuscularly. The
patient’s attempt to rise soon after an injection resulted in loss of consciousness.
What is the probable cause of such complication?
A. Blockade of reticular formation
B. Disturbance of coronary blood circulation
C. Blockade of alfa- adrenoceptors*
D. Inhibition of hypothamus
E. Blockade of M- cholinoceptors.

198. After emotional stress, patient was permanently in a condition of


nervous tension and had poor sleep. Doctor prescribed him diazepam. Which
of the listed effects of this drug is responsible for its clinical use?
A. Hypotensive effect
B. Analgesic effect
C. Anticonvulsant
D. Tranquilizing*
E. Anti psychotic.

199. Point out the neuroleptic that is most commonly used in


neuroleptanalgesia.
A. Chlorpromazine
B. Droperidol*
C. Diazepam
D. Chlorprotixene
E. Sulpiride.

200. A patient on a particular psychotropic medication complains of


severe dizziness. His blood pressure in a supine position is 115/80 mm hg on
standing it drops to 82/50 mm hg. Which of the following drugs is most likely
responsible for these symptoms?
A. Carbamazepine
B. Chlorprozamine*
C. Chlordiazepoxide
D. Cortisone
E. Ibuprofen.

201. A patient has been taking a mixture prescribed by neuropathologist


for neurasthenia for two weeks. The patient feels better but has developed
coryza, conjunctivitis, rash, inertia, decrease of memory. She is diagnosed with
bromizm. What should be prescribed to decrease the
A. Glucose solution 5%
B. Polyglucin
C. Asparcam
D. Natrium chloride*
E. Kalium chloride.

202. A patient who has been treated in a neural clinic and has been
taking a sedative for a long time got the following complication: cough, rhinitis,
epiphora. What drug caused these disturbances?
A. Sodium bromide*
B. Valerian
C. Phenazepam
D. Reserpine
E. Diazepam.

203. Select the most appropriate drug for the systematic treatment of a
patient with severe productive psychotic symptoms (delirium and
hallucinations).
A. Trifluoperazine (Triftazin)*
B. Clozapine
C. Medazepam
D. Amitriptyline
E. Buspirone.

204. Neuroleptics have the properties of eliviating delusions,


hallucinations, reducing of aggression and psychomotor agitation. This action is
called:
A. Antipsychotic*
B. Hypodynamic
C. Antihysteric
D. Anxiolytic
E. Antineurotic.

205. A 40 years patient delivered to psychiatric clinic in a state of


agitation, aggression, delusions. What medication should be administered?
A. Sodium bromide
B. Chlorpromazine*
C. Reserpine
D. Tincture of valerian
E. Amitriptyline.

206. The patient with schizophrenia was appointed chlorpromazine.


What the pharmacological effect is the basis for assigning it to the patient?
A. Hypothermic
B. Hypotensive
C. Antipsychotic*
D. Antiemetic
E. Myorelaxant.

207. A patient addressed a doctor complaining of anxiety, fear,


uneasiness, inner tension. Which drug should be assigned?
A. Tincture of valerian
B. Pavlov Medicine
C. Diazepam*
D. Tincture of leonuri
E. Etaminal sodium.

208. On examination of the patient, unstable blood pressure, irritability,


emotional imbalance, poor sleep were revealed, and doctor has prescribed
chlordiazepoxide (elenium). Which of the following statements is ground for
prescription elenium in this case?
A. Psychosis
B. Cardiopathy
C. Hypertention
D. Coronary artery disease
E. Neurosis*.
209. The presence of which syndrome is the basis for the appointment of
diazepam?
A. Lingering paralysis
B. Inadequate perceptions (hallucinations)
C. Euphoric state of pathological origin
D. Anxiety*
E. Pathological thought disorder.

210. Patient 38 years old suffering from schizophrenia for a long time is
on the psychotropic drugs. He addressed the doctor with complaints of
disturbances of motor coordination, hand tremor, drowsiness. Which group of
drugs can cause such a condition?
A. Antipsychotics*
B. Antidepressants
C. Analgesics
D. Adaptogens
E. Psychomotor stimulants.

211. A single parenteral administration of the drug in the patient with


schizophrenia caused severe hypotension. To which of these groups does drug
belong?
A. Nootropics
B. Antipsychotics*
C. Tranquilizers
D. Antidepressants
E. CNS stimulants.

212. A patient with acute myocardial infarction experienced persistent


chest pain. The ineffectiveness of drugs previously designated gave the grounds
for doctor to use neuroleptanalgesia. Which antipsychotic agent is used for this
type of anesthesia?
A. Chlorpromazine
B. Triftazinum
C. Metaperazine
D. Droperidol*
E. Thioridazine.

213. A patient with schizophrenia is taking typical neuroleptic. Which of


the effects is the basis for its application in schizophrenia?
A. Antipsychotic*
B. Antiemetic
C. Hypothermic
D. Myorelaxant
E. Hypotensive.

214. A patient addressed the doctor with complaints of poor health,


depression, skin rashes. An interview revealed that the patient within three
weeks was taking sedative medicine, prescribed by a neurologist. With the
admission of which drugs complaints of patient were associated?
A. Bromides*
B. Lithium
C. Tranquilizers
D. Antipsychotics
E. Preparations of valerianae.

Psychotropic stimulating agents. Antidepressants.


Psychomotostimulants. Analeptics. Nootropic drugs

215. A patient with poisoning caused by carbon monoxide was


administered directly acting analeptic drug. What medicine was used?
A. Codeine
B. Ephedrine
C. Caffeine*
D. Atropine sulfate
E. Lobeline.

216. A premature newborn was apnoic. Directly acting analeptic drug


was given to restore breathing. What medicine was most probably administered
to the patient?
A. Ethymisolum*
B. Phenylephrine (Mesatonum)
C. Adrenaline hydrochloride
D. Atropine sulfate
E. Lobeline hydrochloride.

217. A psychiatrist was invited to a patient who made an attempt to


commit suicide. Doctor diagnosed endogenous depression. Choose the most
suitable drug for treatment of this patient.
A. Nootropil (piracetam)
B. Amytriptyline*
C. Sydnocarbum
D. Ethymisolum
E. Caffeine.

218. A patient had a stroke. Which of the listed drugs is necessary to


include in the complex therapy in order to improve mental performance?
A. Caffeine
B. Piracetam*
C. Diazepam
D. Phenazepam
E. Amitryptiline.
219. What is the main mechanism by which amitriptyline increases
amount of catecholamines in CNS synapses?
A. Increase in catecholamines release from presynaptic membrane
B. Increase in catecholamines synthesis in presynaptic membrane
C. Prevention of catecholamines degradation in the synapse
D. Inhibition of neuronal re uptake of catecholamines*
E. Inhibition of MAO.

220. Analeptical directly acting drug was given to the patient for
restoration of breathing after poisoning with carbon monoxide. What medicine
was prescribed to the patient?
A. Pentamin
B. Mesaton
C. Caffeine*
D. Atropine sulphate
E. Lobeline hydrochloride.

221. Which of the following characteristics of amphetamines is most


likely to be responsible for increasing blood pressure?
A. Indirect release of endogenous catecholamines*
B. Inhibition of catecholamines metabolism
C. Metabolism to false neuro-chemical transmitters
D. Potent alpha1 adrenergic transmitters
E. Potent beta-2 adrenergic agonist.

222. The CNS stimulation produced by methylxanthines, such as


caffeine, is most likely due to the antagonism of one of the following recertors:
A. Glycine receptors
B. Adenosine receptors*
C. Glutamate receptors
D. GABA receptors
E. Cholinergic muscarinic receptors.

223. A patient who has been treated with diazepam on account of


neurosis complains of toothache. Doctor administered him an analgetic, but its
dose was lower then average therapeutic dose. What phenomenon did the doctor
take into account while prescribing the patient an underdose?
A. Drug dependence
B. Cumulation
C. Tolerance
D. Summation
E. Potentation*.

224. Pharmacological effects of antidepressants are connected with


inhibitionof an enzyme catalyzing biogenic amines noradrenaline and serotonine
in the mitochondrions of cerebral neurons. What enzyme participates in this
process?
A. Lyase
B. Monoamine oxidase*
C. Transaminase
D. Peptidase
E. Decarboxylase.

225. A psychiatrist was called for the woman who tried to commit
suicide on examination the state of endogenous depression was diagnosed.
Which drug is most efficient for treatment of this patient?
A. Piracetam (Nootropil)
B. Amitriptyline*
C. Sydnokarb
D. Aethimizolum
E. Caffeine.

226. Which agent will you choose for increase in mental performance
and decrease of tiredness?
A. Buspirone
B. Nialamide
C. Sidnokarb
D. Imipramine
E. Caffeine*.

227. Doctor recommended taking caffeine the patient with pathological


sleepiness.Which of the following mechanisms of action provides its therapeutic
effect in this disease?
A. The concentration and increased inhibition in the cerebral cortex
B. Increased synthesis of dopamine
C. Enhancement the processes of excitation in the brain*
D. Inhibition of the midbrain reticular formation
E. The weakening of the excitation in the brain.

228. Point out preferable analeptic in acute barbiturate poisoning


A. Cordiaminum
B. Caffeine
C. Camphor
D. Bemegride*
E. Aethimizolum.

229. The patient had a stroke. Which of the following drugs should be
included in the combined therapy to improve the metabolism of the brain?
A. Piracetam*
B. Caffeine
C. Diazepam
D. Phenazepam
E. Amitriptyline.
230. Which the drug from the group of psychomotor stimulants can
cause the development of drug addiction?
A. Amphetamin (Phenaminum)*
B. Meridil
C. Sidnokarb
D. Bemithylum
E. Caffeine.

231. Which of the following effects of piracetam is used for chronic


vascular disorders of the brain?
A. Anticonvulsant activity
B. Improvement of mental performance*
C. Antiwithdrawal effect
D. Improvement of physical activity
E. Antipsychotic effect.

232. The patient complains of fatigue, general weakness, decrease in


visual acuty, low blood pressure. Which drug is helpful in this case?
A. Liquid extract of Eleutherococcus*
B. Phenibut
C. Piracetam
D. Dry extract of Valeriana
E. Amitriptyline.

233. The primary goal of medical aid in vascular collapse is an increase


in blood pressure. Which of the following medications can be used for this
purpose?
A. Aethimizolum
B. Diphenhydramine
C. Bemegride
D. Caffeine-sodium benzoate*
E. Ketotifen.

234. A patient admitted to the neurological department complains of


memory loss and cognitive disability after trauma of the head. What kind of
medication is needed to improve the metabolism of the brain?
A. Caffeine
B. Metamizole (Analginum)
C. Piracetam (nootropil)*
D. Sidnokarb
E. Meridil.

235. A psychiatrist was called for the woman who tried to commit
suicide on examination the state of endogenous depression was diagnosed.
Which drug is most efficient for treatment of this patient?
A. Piracetam (Nootropil)
B. Amitriptyline*
C. Sydnokarbum
D. Aethimizolum
E. Caffeine.

236. Which agent will you choose for increase in mental performance
and decrease of tiredness?
A. Nialamide
B. Diazepam
C. Imipramine
D. Buspirone
E. Caffeine*.

237. When should you expect for a specific therapeutic effect of


pantocrinum in its systematic application?
A. After 9-15 days*
B. After 30-60 minutes
C. 1-3 days
D. After 30-45 days
E. After 3-6 hours.

238. Doctor recommended taking caffeine the patient with pathological


sleepiness. Which of the following mechanisms of action provides its
therapeutic effect in this disease?
A. Enhancement the processes of excitation in the brain*
B. Increased synthesis of dopamine
C. The concentration and increased inhibition in the cerebral cortex
D. Inhibition of the midbrain reticular formation
E. The weakening of the excitation in the brain.

239. Point out preferable analeptic in acute barbiturate poisoning


A. Caffeine
B. Bemegride*
C. Camphor
D. Cordiaminum
E. Aethimizolum.

240. The patient had a stroke. Which of the following drugs should be
included in the combined therapy to improve the metabolism of the brain?
A. Caffeine
B. Diazepam
C. Piracetam*
D. Phenazepam
E. Amitriptyline.

241. The primary goal of medical aid in vascular collapse is an increase


in blood pressure. Which of the following medications can be used for this
purpose?
A. Diphenhydramine
B. Bemegride
C. Caffeine-sodium benzoate*
D. Aethimizolum
E. Ketotifen.

242. A patient admitted to the neurological department complains of


memory loss and cognitive disability after trauma of the head. What kind of
medication is needed to improve the metabolism of the brain?
A. Piracetam (nootropil)*
B. Metamizole (Analginum)
C. Caffeine
D. Sydnokarbum
E. Meridil.

243. Which of the following tonics is derived from animal products?


A. Tincture ginseng
B. Pantocrine*
C. Saparalum
D. Ecdistenum
E. Securininum.

244. Among the antidepressant drugs there are those that inhibit
neuronal norepinephrine reuptake. Point out such drug.
A. Fluoxetine
B. Nialamide
C. Chlorprothixene
D. Chlorpromazine
E. Maprotilime*.

245. Mark pharmacological effect, limiting the use of strychnine as


analeptic.
A. Stiffness in skeletal muscle, convulsions*
B. Hypercatharsis
C. Increased metabolism
D. Strengthening of sight and hearing
E. Impact on hematopoiesis.

246. What drug from the group of analeptics is preferable to use in


newborn asphyxia?
A. Caffeine
B. Aethimizolum*
C. Camphor
D. Cordiaminum
E. Corazolum.
247. A 19 years old boy addressed doctor complaining of discomfort in
the heart area. Doctor asked him about smoking and other bad habits. Patient
denied all of mentioned bad habits, and sad he likes coffee and uses up to 8
glasses of strong drink a day. Why coffee containing caffeine can worsen heart
function?
A. It causes tachycardia, increases in oxygen demand of the heart*
B. It slows impulse conduction in the heart
C. It weakens the force of heart contractions
D. It causes a narrowing of the coronary vessels
E. It decreases automaticity of the heart.

248. What is the mechanism of action of antidepressant pirlindol


(pirazidol)?
A. The reversible inhibition of MAO*
B. Irreversible inhibition of MAO
C. Selective inhibition of neuronal capture of catecholamines
D. Non-selective inhibition of neuronal capture catecholamines
E. Inhibitor of vesicular capture catecholamines.

249. A patient with symptoms of mental depression in the drug treatment


did not keep the doctor recommended a diet that caused the rise in blood
pressure. Which group of antidepressants requires compliance in this condition?
A. Tricyclic antidepressant
B. Nonselective MAO inhibitors*
C. Selective inhibitors of catecholamines reuptake
D. Lithium preparations
E. Nonselective inhibitors of neuronal reuptake of catecholamine
Drugs acting on respiratory system

250. Analeptic remedy of reflective type from the N-cholinomemetics


group was given to the patient for restoration of breathing after poisoning
with carbon monoxide. What medicine was prescribed to the patient?
A. Adrenalin hydrochloride
B. Lobeline hydrochloride*
C. Mesaton
D. Pentamin
E. Atropine sulfate.

251. In complex therapy of bronchial asthma myotropic spasmolytic


drug was added. Soon after its usage, nervousness and sleep disturbances
occurred. Which of the following drugs can cause above-mentioned side
effect?
A. Papaverine
B. Platyphilline
C. Aminophylline *
D. Drotaverine
E. Ephedrine .

252. A 5 year-old –child was suffering from bronchitis, accompanied


with a productive cough with thick sputum. In addition to antimicrobial
treatment a mucolytic drug was prescribed. This drug was especially
beneficial for the child due to its ability to stimulate production of
surfactant. Point out this drug
A. Codeine
B. Ambroxolum*
C. Oxeladine
D. Ethylmorphine
E. Potassium iodide.

253. Severe attack of bronchospasm and cough occurred in a patient with


bronchial asthma. Choose the best drug for quick relief of the patient
condition.
A. Ipratropium bromide
B. Orciprenaline sulfate
C. Atropine
D. Adrenaline*
E. Noradrenaline.

254. A patient with bronchial asthma had been taking tablets which
caused insomnia, headache, increased blood pressure. What medecine can
cause such complications?
A. Ephedrine*
B. Chromolin sodium
C. Euphyline
D. Adrenaline
E. Izadrine.

255. A 13-year-old girl with history of asthma complained of cough,


dyspnea and wheezing. Her symptoms became so severe that her parents
brought her to the emergency room. Physical examination revealed
diaphoresis, dyspnea, tachycardia and tachypnea. Her respiratory rate was
42/min. pulse rate was 110 beats per minute, and blood pressure was
l30/70 mm Hg. Choose from the following list the most appropriate drug
to reverse the bronchoconstriction rapidly:
A. Methylprednidsolone
B. Salbutamol*
C. Beclomethasone
D. Cromolyn
E. Ipratropium.

256. Apnoic newborn was resuscitated by administration of respiratory


stimulant.Which agent is the drug of first choice in this case?
A. Bemegridum
B. Aethimisolum*
C. Lobeline
D. Codeine
E. Camphor.

257. In complex treatment of patient with bronchial asthma cromolyn-


sodium was included as antiallergic agent. What is the mechanism of this
drug action?
A. Inhibition of histaminic H1 receptors
B. Inhibition of histaminic H2 receptors
C. Prevention of mast cell degranulation*
D. Inhibition of antigen-antibody interaction
E. Blockade of D2 receptors.

258. Severe attack of bronchospasm and cough occur in patient with


bronchial asthma. Choose the best drug for quick relieve of patient
condition.
A. Ipratropium bromide
B. Orciprenaline sulfate
C. Atropine
D. Adrenaline*
E. Noradrenaline.

259. Patient with bronchial asthma was taking tablets, which caused
insomnia, headache, increased blood pressure. What medicine can cause
such complications?
A. Izadrine
B. Adrenaline
C. Chromolin sodium
D. Euphyline
E. Ephedrine*.

260. In complex therapy of bronchial asthma myotropic spasmolytic


drug was included. Soon after its usage nervousness and sleep
disturbances occurred. Which of the following drugs can cause above-
mentioned side effect?
A. Papaverine
B. Platyphilline
C. Aminophylline*
D. Drotaverine
E. Ephedrine.

261. A patient suffering from chronic bronchitis takes a synthetic


mucolytic drug that facilitates the sputum thinning. What drug is it?
A. Acetylcysteine*
B. Diazepam
C. Heparin
D. Furosemide
E. Enalapril.

262. A patient suffering from bronchial asthma didn't inform his doctor
that he had attacks of angina pectoris(stenocardia). Doctor administered
him a drug, which usage resulted in less frequent attacks of bronchial
asthma, but stenocardia (anginal) attacks became more frequent. What
medication was administered?
A. Isadrin (Isoprenalin)*
B. Salbutamol
C. Aminophylline
D. Cromolyn sodium
E. Phenotherol.

263. A 70-year-old man, who suffered from chronic bronchitis, was


prescribed medicine for the cough - codeine. What is the mechanism of
antitussive effect?
A. Central*
B. Reflex
C. Competitive
D. Local effect
E. Peripheral effect.

264. In long lusting usage of fenoterol in bronchial asthma its


bronchodilating effect is gradually decreasing. What is probable cause of
tolerance development?
A. Worsening of absorption
B. Decrease in numbers of beta-adrenoceptors*
C. Enhancement of binding with plasma albumin
D. Enhancement of elimination
E. Decrease in numbers of Gs-proteins.

265. To terminate an attack of asthma patients used inhalation of the


drug that is used by inhalation as bronchodilators, and in injections it is
used in obstetric practice to prevent miscarriages. What drug was used by
patient?
A. Fenoterol*
B. Atenolol
C. Adrenaline
D. Ephedrine
E. Euphylline.

266. In patient with bronchial asthma attacks usually occur at night, they
are accompanied by bradycardia, spasmodic pains in the intestines,
diarrhea. Preparation of which group can eliminate these symptoms?
A. H2-histaminoblockers
B. N-cholinoblockers
C. M cholinoblockers*
D. Alpha-blockers
E. Beta-blockers.

267. A patient with respiratory depression was admitted to hospital.


Drugs of which pharmacological group stimulate breathing in such case?
A. Antidepressants
B. Tranquilizers
C. Antipsychotics
D. Analeptics*
E. Analgesics.

268. Asthmatic patient was assigned to inhalation of 0.5% solution


(isoprenalin) isadrinum. Bronchospasm was relieved, but the patient
began to complain of pain in the heart and heartbeat. This is due to the
stimulation of:
A. Beta2-adrenergic
B. Alpha1-adrenoceptor
C. M-cholinergic
D. Beta1-adrenergic*
E. Synthesis of acetylcholine.

269. Patient suffering from bronchitis with poorly separated sputum


started taking acetylcysteine. Name the mechanism of mucolytic action of
the drug.
A. Reflex stimulates the secretion of bronchial glands
B. Stimulates the cough center
C. Increases the secretion of bronchial mucus
D. Depolymerizes sputum*
E. Increases bronchial contraction.

270. A patient with severe attacks of asthma that was not stopped by
earlier effective beta-adrenoceptor agonists was admitted to the intensive
care unit Diagnosis: asthma, asthmatic status. What drug should be
administered the patient in the first place?
A. Adrenaline*
B. Acyclovir
C. Prednisone
D. Famotidine
E. Fenoterol.

271. A 25 y.o. patient in the dental office suddenly developed an attack


of asthma. The doctor gave the patient salbutamol to inhale. What is the
mechanism of action of this drug?
A. Stimulates the alpha-adrenergic
B. Blocks histamine H1 receptors
C. Blocks M-cholinergic
D. Stimulates beta2-adrenergic receptors*
E. Blocks phosphodiesterase.

272. For patient suffering of pneumonia and cough with thick poorly
separated sputum doctor prescribed expectorant drug that activates the
synthesis of surfactant in the lungs. What the drug was prescribed?
A. Acetylcysteine
B. Infusion of herb Thermopsis
C. Glaucin
D. Trypsin
E. Lasolvan (Ambroxol)*.

273. To suppress dry cough in patient with chronic bronchitis doctor


prescribed antitussive - glaucine hydrochloride. What is the mechanism of
action of this drug?
A. Resorptive central*
B. M-cholinergic blockade
C. Reflex
D. Antienzymatic
E. Peripheral.

274. Exhausting cough with fetid odor appeared in patient with lobar
pneumonia. Which of the following agents should be used in this case?
A. Famotidine
B. Mesaton
C. Fenoterol
D. Acyclovir
E. Trypsin*.

Cardiac glycosides and other inotropic drugs. Antiarrhythmic drugs.

275. Which of the following drugs is most likely to produce positive


inotropic and negative chronotropic effects?
A. Nitroglycerin
B. Furosemide
C. Diltiazem
D. Procainamide
E. Digoxin.*.

276. Choose the inotropic drug for treatment of patient suffering from
congestive heart failure, not responding to digitalis.
A. Phenylephrine
B. Dobutamine*
C. Clonidine
D. Noradrenaline
E. Salbutamol.

277. A patient who has been suffering from cardiac insufficiency for
several months has been taking digoxin on an outpatient basis. At a
certain stage of treatment there appeared symptoms of drug overdose.
What phenomenon underlies the development of this complication?
A. Functional cumulation
B. Tachyphylaxis
C. Habituation
D. Sensibilization
E. Material cumulation*.

278. A 50 y.o. patient with chronic cardiac insufficiency and


tachyarrythmia was prescribed cardiotonic drug. What drug was
prescribed?
A. Dobutamine
B. Mildronate
C. Amyodarone
D. Digoxin*
E. Dopamine.

279. A patient suffering from chronic cardiac insufficiency was


recommended to undergo a prophylactic course of treatment with a
cardiological drug from the group of cardiac glycosides that is to be taken
enterally. What drug was recommended?
A. Corglycon
B. Cordiamin
C. Strophanthine
D. Cordarone
E. Digoxin*.

280. A patient that entered the admission office had the following signs
of acute cardiac insuffiency: paleness, acrocyanosis, frequent shallow
respiration. What drug is indicated in this case?
A. Corglycon*
B. Adrenaline hydrochloride
C. Nitroglycerine
D. Cordiamin
E. Digitoxin.

281. To maintain normal sinus rhythm in patient with atrial fibrillation


the doctor prescribed an antiarrhythmic drug. After laboratory testing of
thyroid function administration of this drug was withdrawn. What drug
was initially prescribed?
A. Verapamil
B. Amiodarone*
C. Quinidine
D. Procainamide
E. Lidocaine.

282. Patient suffering from tachyarrhythmia was treated with calcium


channel blocker. What drug was used?
A. Niphedipine
B. Verapamil*
C. Labetolol
D. Alprenolol
E. Nicardipine.

283. A 52 year old patient with supraventricular tachyarrhythmia was


using antiarrhythmic drug of class III. For a long period it maintained the
normal cardiac rhythm, but with time dispnoe started to occur while going
upstairs. Patient addressed doctor. The examination revealed pulmonary
sclerosis. Which of the antiarrhythmic drug can cause such side effect?
A. Verapamil
B. Amiodarone*
C. Bretylium
D. Lidocaine
E. Procainamide.

284. Digoxin in tablets was prescribed to patient with chronic congestive


heart failure. After 1 month of treatment decrease in heart rate was noted
doctor advised the patient to continue treatment with lower dose of
digoxin. In spite of this the bradycardia was soon transformed into A-V-
block. Choose the necessary drug for abolishing of this manifestation of
glycoside toxicity.
A. Potassium chloride
B. Phenytoin
C. Atropine*
D. Lidocaine
E. Propranolol.

285. A pregnant woman addressed her doctor complying of frequent


heartbeats. Which of antiarrhythmic agents is contraindicated in this
case?
A. Atenolol
B. Propranolol*
C. Metoprolol
D. Verapamil
E. Diltiazem.

286. Patient with chronic congestive heart developed tachyarrhythmia.


Which of the listed drugs is contraindicated in this case?
A. Quinidine*
B. Lidocaine
C. Timecaine
D. Phenytoin
E. Ajmaline.

287. In the treatment of patient suffering from congestive heart failure


resistant to digitalis adrenomimetic was used as inotropic agent. Point out
which of the following drugs was administered.
A. Phenylephrine
B. Partusisten
C. Dobutamine*
D. Xylometazoline
E. Naphazoline

288. A 62-year-old patient, prone to bronchoconstriction, addressed his


doctor with complaints of tachyarrhythmia troubling him several times a
day. The doctor prescribed anaprilin (propranolol). Administration of drug
provoked severe attack of bronchospasm. What is the mechanism of this
side effect?
A. Stimulation of M1- cholinoceptors
B. Inhibition of α-adrenoceptors
C. Inhibition of β2 - adrenoceptors*
D. Increase in n. vagus tone
E. Stimulation of leukotriene release

289. A patient with fibrillation, who has had bronchial asthma in his
anamnesis, should be administered antiarrhythmic drug. Which
preparation from the suggested drugs is contraindicated for this patient?
A. Ajimaline
B. Verapamil
C. Novocainamide
D. Nifedipine
E. Anapriline (Propranolol)*

290. A 55-year-old patient with persisting ventricular arrhythmia was


admitted to the hospital. The patient is taking Timolol drops for
glaucoma, daily insulin injections for diabetes mellitus, and an ACE
inhibitor for hypertension. You have decided to use phenytoin instead of
procainamide. What is the reason?
A. The total anesthetic effect of procaninamide would aggravate the
hypertension
B. The anticholinergic effect of procainamide would aggravate
glaucoma*
C. The local aneathetic effect of procainamaide would potentiate
diabetes
D. The hypertensive effects of procainamide would aggravate the
hypertension
E. The cholinergic effects of procainamide would aggravate the diabetes

291. A patient with chronic cardiac insufficiency has been treated with
cardiotonic drugs and a thiazide diuretic, but in spite of it there are still
edemata and risk of ascites. What medication should be prescribed to
amplify diuretic effect of the applied drugs?
A. Furosemide
B. Spironolactone*
C. Amyloride
D. Clopamide
E. Mannitol

292. A patient takes digoxin for treatment of cardiac insufficiency. What


diuretic may increase digoxin toxicity due to the intensified excretion of
K+ ions?
A. Spironolactone
B. Hydrochlorothiazide*
C. Panangine
D. Siliborum
E. Lisinopril

293. A patient suffers from chronic left-ventricular insufficiency. What


medication should be administered?
A. Digoxin*
B. Bemegride
C. Aethimizolum
D. Vinpocetine
E. Pyracetam

294. Patient suffering from acute cardiac failure was treated with
nonglycoside inotropic drug inhibiting phosphodiesterase III. Which drug
was used?
A. Mildronate
B. Dopamine
C. Dobutamine
D. Amyodarone
E. Amrinone*

295. Patient complains of weakness, dyspnea, low extremities oedema.


Diagnosis: chronic cardiac insufficiency. What medicine should be
prescribed first of all?
A. Caffeine
B. Digitoxin*
C. Papaverine
D. Propranolol
E. Raunatin

Diuretics.

296. Point out antihypertensive agent that increases salts and H2O
excretion and can causes hyperglycemia and uric acid retention?
A. Nifedipine
B. Propranolol
C. Prazosin
D. Hydrochlorothiazide*
E. Acetazoleamide (Diacarbum).

297. A 67- year-old man treated with hydrochlorothiazide for control of


mild hypertension visited his doctor complaining of pains in joints. Which
of the listed side effects is the most probable cause of patient’s
complains?
A. Hyperglycemia
B. Hyperuricemia*
C. Drug hypersensitivity
D. Hyperkalemia
E. Hypokalemia.

298. A patient suffering from hypertension was treated with


hydrochlorothiazide. With time he addressed doctor complaining of
worsening of his condition. On examination of the patient hypokalemia
was diagnosed. Choose the drug, which has to be added aiming at
enhancement of diuretic effect and abolishing of hypokalemia.
A. Acetazolamide
B. Furosemide
C. Ethacrynic acid
D. Spironolactone*
E. Chlorothiazide.

299. A patient with acute poisoning was admitted to emergency


department. It was established that poisonous substance was excreted by
kidney. Choose the best diuretic for forced diuresis.
A. Acetazolamide
B. Amiloride
C. Spironolactone
D. Furosemide*
E. Hydrochlorothiazide.

300. A 42-year-old man suffering from gout has increased level of


urinary acid in blood. Allopurinol was prescribed to decrease the level of
urinary acid. Competitive inhibitor of what enzyme is allopurinol?
A. Xanthine oxidase*
B. Guanine deaminase
C. Adenosine deaminase
D. Hypoxanthine phosphoribosiltransferase
E. Adenine phosphoribosiltransferase.

301. A doctor administered Allopurinol to a 26-year-old young man with


the symptoms of gout. What pharmacological action of Allopurinol
ensures therapeutical effect?
A. By inhibiting leucocyte migration into the joint
B. By general analgetic effect
C. By increasing uric acid excretion
D. By general anti-inflammatory effect
E. By inhibiting uric acid synthesis*.

302. The alternate usage or dichlotiazide, etacrin acid and lasex did not
influence diuretically upon the patient with marked peripheral edemata.
The aldosterone rate in the blood is increased. Indicate which medicine
should be prescribed:
A. Urea
B. Amilorid
C. Clopamid
D. Spironolacton*
E. Mannit.

303. A patient with chronic cardiac insufficiency has been treated with
cardiotonic drugs and a thiazide diuretic, but in spite of it there are still
edemata and risk of ascites. What medication should be prescribed to
amplify diuretic effect of the applied drugs?
A. Spironolactone*
B. Furosemide
C. Manitole
D. Clopamide
E. Amyloride.

304. A patient takes digoxin for treatment of cardiac insufficiency. What


diuretic may increase digoxin toxicity due to the intensified excretion of
K+ ions?
A. Lisinopril
B. Panangine
C. Siliborum
D. Spironolactone
E. Hydrochlorothiazide*

305. Highly potent diuretic drug was prescribed to the patient with
pulmonary edema in the course of complex treatment. In a few days signs
of hypokaliemia were developed. What drug could cause such
complications?
A. Furosemide (Lasix)*
B. Spironolactone
C. Acetazolamide
D. Triamterene
E. Enalapril

306. For treatment of hypertensive patient doctor prescribed diuretic


agent. It resulted in sufficient control of BP, but soon patient addressed
doctor, complaining of pain in joints. Which diuretic was used?
A. Spironolactone
B. Hydrochlorothiazide*
C. Clonidine
D. Triamterene
E. Enalapril

307. A patient with chronic cardiac insufficiency has been treated with
cardiotonic drugs and a thiazide diuretic, but in spite of it there are still
edema and risk of ascite. What medication should be prescribed to
amplify diuretic effect of the applied drugs?
A. Spironolactone*
B. Furosemide
C. Amyloride
D. Clopamide
E. Mannitol

308. Diuretic drug was prescribed to the patient with hypertension in the
course of complex treatment. In a few days BP decreased but signs of
hypokaliemia developed. What drug could cause such complications?
A. Spironolactone
B. Clophelin
C. Lasix (Furosemide)*
D. Triamterene
E. Enalapril

309. A 65 year old female patient suffers from chronic renal


insufficiency accompanied by evident edemata caused by chronic
glomerulonephritis. What diuretic should be administered for forced
diuresis?
A. Furosemide*
B. Hydrochlorothiazide
C. Chlorthalidone
D. Cyclometazide
E. Acetazolamide

Antihypertensive drugs. Antihyperlilpidemic agents

310. A hypertensive patient was treated with the drug suppressing the
formation of angiotensin II and preventing degradation of bradykinine.
Point out the drug realizing antihypertensive effect by these mechanisms.
A. Nifedipine
B. Guanethidine
C. Clonidine
D. Enalapril*
E. Propranolol.

311. A hypertensive patient was fond of meat dishes and did not keep
proper diet. In the course of complex antihypertensive treatment an acute
attack of gout developed. Which of the following agents is responsible for
worsening of patient condition?
A. Hydrochlorothiazide*
B. Prazosin
C. Allopurinol
D. Acetazolamide
E. Propranolol.

312. The blood pressure of hypertensive patient is successfully


controlled by administration of enalapril. Which mechanism is
responsible for antihypertensive effect of this drug?
A. Inhibition of peripheral alpha1 - adrenoceptors
B. Blockade of angiotensin II receptors
C. Inhibition of renin release
D. Inhibition of kallikrein – kinin system
E. ACE inhibition and stimulation of kalliкrein – kinin system*.

313. Metabolism can render pharmacological activity to some initially


inert substances (pro-drug). Point out the substance, which is pro-drug,
used for treatment of hypertension.
A. Levodopa
B. Enalapril*
C. Verapamil
D. Reserpine
E. Propranolol.

314. Hypertensive patient was treated with the drug that decreases
vascular tone. His treatment was complicated by persistent dry cough.
Which drug most probably was used?
A. Papaverine
B. Phentolamine
C. Lisinopril*
D. Prazosin
E. Clonidine.

315. For treatment of hypertensive patient doctor prescribed captopril,


but soon patient addressed doctor, complaining of dry cough and rashes.
Doctor substituted captopril for losartan. What is the mechanism of the
last drug action?
A. Inhibition of renin release
B. Inhibition of ACE
C. Inhibition of angiotensinogen conversion to angiotensin I
D. Blockade of angiotensin II receptors*
E. Increase in bradykinin level.

316. A patient with II stage hypertension has been taking one of


hypotensive medications for the purpose of treatment. After a time arterial
pressure decreased, but the patient started complaining of flaccidity,
sleepiness, indifference. A bit later he felt stomach pain. He was
diagnosed with ulcer. What hypotensive medication has the patient been
taking?
A. Captopril
B. Dibazoie
C. Reserpine*
D. Furosemide
E. Verapamil.

317. A patient taking clonidine for essential hypertension treatment was


using alcohol that caused intense inhibition of central nervous system.
What may it be connected with?
A. Intoxication
B. Idiosyncrasy
C. Cumulation
D. Effect potentiating*
E. Effect summation.

318. A patient with hypertensic crisis was admitted to the cardiological


department, he was injected intravenously with an antihypertensive drug -
salt of an alkaline-earth metal. What drug was injected?
A. Benzohexamethonium
B. Potassium chloride
C. C Sodium hydrocarbonate
D. Magnesium sulfate*
E. Caleium lactate.

319. For treatment of hypertensive patient doctor prescribed


antiadrenergic drug, first administration of which in therapeutic dose
caused “first dose syncope”. Which drug was most probably used?
A. Metoprolol
B. Atenolol
C. Prazosin*
D. Reserpin
E. Propranolol

320. A 43 year-old male patient is suffering from hypertension. His


blood pressure was successfully controlled by monotherapy with
adrenoblocker. With time treatment was complicated with diarrhea and
impotence. The patient addressed his doctor and the drug was changed for
another adrenoblocker. Which of adrenoblockers can cause above listed
side effects?
A. Metoprolol
B. Salbutamol
C. Propranolol
D. Dobutamine
E. Prazosin*

321. Which of the following antiadrenergic drugs used in the treatment


of hypertension is contraindicated in patients prone to bronchospasm?
A. Prazosin
B. Metoprolol
C. Reserpine
D. Atenolol
E. Propranolol (Anaprilinum)*

322. Point out antihypertensive agent that increases salts and H2O
excretion and can cause hyperglycemia and uric acid retention?
A. Nifedipine
B. Propranolol
C. Prazosin
D. Hydrochlorothiazide*
E. Acetazoleamide (Diacarbum)

323. A patient suffering from hypertension was treated with


hydrochlorothiazide. With time he addressed doctor complaining of
worsening of his condition. On examination of the patient hypokalemia
was diagnosed. Choose the drug, which has to be added aiming at
enhancement of diuretic effect and abolishing of hypokalemia.
A. Acetazolamide
B. Furosemide
C. Ethacrynic acid
D. Spironolactone*
E. Chlorothiazide

324. A patient with moderate hypertension was treated with the drug that
realizes its effect by decrease in noradrenaline release from sympathetic
nerve endings. While treatment patient experienced drowsiness. What the
drug was used?
A. Prazosin
B. Reserpin*
C. Drotaverine
D. Atenolol
E. Dichlorothiazide

325. Phenofibrate was prescribed for a patient with a history of angina


pectoris and high plasma cholesterol level. What is the mechanism of
action of this drug?
A. Activation of lipoprotein lipolysis*
B. Inhibition of lipoprotein lipase
C. Inhibition of hydroxymethyl glutaryl coenzyme A
D. Inhibition of DL oxidation
E. Bile acid sequestration

326. In complex treatment of hypertensive patient antiatherogenic agent


that decreases synthesis of cholesterol was included. Point out this agent.
A. Cholestyramine
B. Colestipol
C. Fenofibrate
D. Simvastatin*
E. Probucol

327. Point out the most atherogenic lipoproteins


A. Chylomicrons (Chy)
B. Low density lipoproteins (LDL)*
C. Very low-density lipoproteins (VLDL)
D. High-density lipoproteins (HDL)
E. Intermediate density lipoproteins (IDL)

328. A 64 –year-old patient addressed doctor complaining of vertigo and


ringing in ears. On examination the disturbance of brain blood circulation
was diagnosed. Doctor prescribed the drug that is vinca alkaloid
derivative. Which drug was prescribed?
A. Cinnarizine
B. alfa-Tokopherol
C. Vinpocetine*
D. Xantinol nicotinate
E. Parmidin

329. Point out the class of lipoprotein that facilitates the removal of
cholesterol from tissues
A. Chylomicrons (Chy)
B. Low density lipoproteins (LDL)
C. Very low-density lipoproteins (VLDL)
D. High-density lipoproteins (HDL)*
E. Intermediate density lipoproteins (IDL)

330. A 60 year-old patient with angina pectoris visited her doctor for
routine examination. Blood sampling revealed elevated level of
cholesterol in the plasma. Due to this antisclerotic drug was added to
antianginal therapy. The effect of prescribed drug is based upon the
inhibition of cholesterol synthesis by suppression of HMG-CoA reductase
activity. What the drug was prescribed?
A. Nicotinic acid
B. Probucol
C. Lovastatin*
D. Clofibrate
E. Colestipol

331. Patient with moderate hypertension was treated with the drug,
decreasing heat work and renin release, and causing coldness of hands as
a side effect. What the drug was used?
A. Prazosin
B. Propranolol*
C. Spironolacton
D. Atenolol
E. Dichlorothiazide

332. Blood pressure of hypertensive patient is successfully controlled by


administration of enalapril. Which mechanism is responsible for
antihypertensive effect of this drug?
A. Inhibition of peripheral alpha –1 adrenoceptors
B. Blockade of angiotensin II receptors
C. Inhibition of renin release
D. Inhibition of kallikrein – kinin system
E. ACE inhibition and enhancement of kalliкrein – kinin system*

333. A 60 year-old patient suffering from hypertension was prescribe


drug inhibiting RAS that inhibits rennin directly. Point out this drug.
A. Enalapril
B. Verapamil
C. Reserpine
D. Propranolol
E. Aliskiren*

334. A 66 year old female patient got intravenous injection of


magnesium sulfate solution for the purpose of elimination of hypertensive
crisis. But arterial pressure didn't go down and after repeated
introduction of the same preparation there appered sluggishness, slow
response, inhibition of consciousness and respiration. What preparation
is antagonist of magnesium sulfate and can eliminate symptoms of its
overdose?
A. Calcium chloride*
B. Potassium chloride
C. Sodium chloride
D. Activated carbon
E. Potassium permanganate
Antianginal drugs

335. A patient with angina pectoris was helping himself with


nitroglycerin. One day the patient used nitroglycerin several times
because of frequent episodes of pain. So as the last sublingual
administration of nitroglycerin did not relieve pain, the patient addressed
his doctor. Doctor advised him to withdraw nitroglycerin for 8 hours and
prescribed another drug. After 8 hours the effect of nitroglycerin was
restored. Which phenomenon did take place in this case?
A. Increase in nitroglycerin metabolism
B. Increase in protein binding
C. Decrease of absorption
D. Drug dependence
E. Tolerance *.

336. A patient with Raynaud disease was treated with niphedipine.


What is the mechanism of action of this drug?
A. Blockade of B-adrenoceptors
B. Blockade of alpha adrenoceptors
C. Decrease in neurotransmitter amount
D. Calcium channels’ blockade*
E. Blockade of central M-cholinoceptors.
337. To prevent the onset of myocardial infarction in a patient with
angina pectoris aspirin was used. Due to which of listed effects aspirin is
beneficial in this case?
A. Antiinflammatory
B. Antiplatelet*
C. Analgesic
D. Antipyretic
E. Ulcerogenic.

338. After a tooth extraction a patient felt persistent pain behind his
breast bone. After sublingual intake of an antianginal drug the pain behind
the breast bone disappeared, but the patient complained of headache and
dizziness. What drug are these properties typical for?
A. Validol
B. Verapamil
C. Nitroglycerin*
D. Metoprolol
E. Propranolol.

339. In complex treatment of a patient with angina pectoris non- narcotic


analgesic exerting antiplatelet activity was included. Point out this agent.
A. Meperidine
B. Morphine
C. Tramadol
D. Metamizole
E. Aspirin*

340. A 72-year woman suffering from angina pectoris was treated by


propranolol (anaprilinium). On the background of the treatment cough
with bronchospastic components was developed. Doctor replaced
propanolol with metoprolol. This resulted in abolishing of respiratory
disturbances. What is the reason for the beneficial effect of metoprolol?
A. Block of M-cholinoceptors
B. Absence of action upon β2- adrenoceptors*
C. Stimulating of M-cholinoceptors
D. Inhibition of m-cholinireceptors
E. Absence of action on α- adrenoceptors

341. A patient with myocardial infarction was treated with


intravenous infusion of heparin. What is the goal of this drug
administration?
A. To cause the lysis of thrombus directly
B. To transform plasminogen into plasmin
C. To prevent further thrombosis*
D. To prevent platelets activation
E. To decrease the area of necrosis
342. It is necessary to prevent the onset of myocardial infarction in
patient with angina pectoris. Choose the drug with antiplatelet and
vasodilating effects.
A. Ticlopidine
B. Dipiridamole*
C. Aspirin
D. Nifedipine
E. Verapamil

343. Patient complains of weakness, dyspnea, low extremities


oedema. Diagnosis: chronic cardiac insufficiency. What medicine
should be prescribed first of all?
A. Digitoxin*
B. Caffeine
C. Papaverine
D. Propranolol
E. Raunatin

344. Myocardial infarction patient was treated with streptokinase. What


is the mechanism of thrombolytic action of this drug?
A. Formation of active complex with proactivator of plasminogen*
B. Direct activation of plasminogen
C. Prevention of prothrombin conversion to thrombin
D. Prevention of fibrin formation
E. Direct lysis of thrombus

345. 72-year-old patient with angina pectoris was prescribed


dipyridamole. Administration of drug resulted in worsening of patient
condition and doctor substituted dipyridamole by long acting organic
nitrate. What was the most probable cause of worsening?
A. Lack of coronary vessels dilation
B. Increase in oxygen demand
C. “Coronary steal phenomenon”*
D. Decrease in arterial pressure
E. Decrease in venous pressure

346. A patient who had myocardial infarction was prescribed 75 mg of


acetylsalicylic acid a day. What is the purpose of this administration?
A. Reduction of platelet (thrombocyte) aggregation *
B. Inflammation reduction
C. Pain relief
D. Temperature reduction
E. Coronary vessel dilatation

347. Anapriline (Propranolol) therapy caused positive effect in the


dynamic of the disease of a 44-year-old woman suffering from
stenocardia (angina pectoris). What is the main mechanism of the effect
of this medicine?
A. Blockade of β-adrenoreceptors and decrease myocardial requirements
to the oxygen.*
B. Decrease of oxidative exchange in myocardium due to enzyme
blockade of Krebs' cycle
C. Decreased power inputs of myocardium due to reduced loading
D. Increased oxygen supply to the myocardium
E. Decreased need in increasing of oxygen supply to the myocardium

348. A patient suffering from coronary artery disease had taken a certain
drug many times a day in order to arrest stenocardia attacks. Overdose of
this drug finally caused intoxication. Objectively: cyanotic skin and
mucous membranes, dramatic fall in the arterial pressure, tachycardia,
respiration inhibition. Blood has increased concentration of
methemoglobin. The drug the patient had taken relates to the following
group:
A. Organic nitrates*
B. Alpha-adrenoceptor blockers
C. Calcium channel blockers
D. Adenosine drugs
E. Myotropic spamolytics

349. A patient suffers from stenocardia and takes isosorbide


mononitrate. He was prescribed a complementary drug with
disaggregating effect. What drug is it?
A. Acetylsalicylic acid*
B. Nitroglycerine
C. Propranolol
D. Nifedipine
E. Validol

350. A patient with coronary artery disease was admitted to the


cardiological department. For stenocardia (angina pectoris) prevention a
drug from the group of beta-adrenoceptor blockers was administered.
What drug is it?
A. Metoprolol*
B. Atropine sulfate
C. Morphine hydrochloride
D. Oxytocin
E. Furosemide

351. A patient with frequent attacks of stenocardia was prescribed


sustak-forte to be taken one tablet twice a day. At first the effect was
positive but on the second day stenocardia attacks resumed. What can
explain inefficiency of the prescribed drug?
A. Tachyphylaxis*
B. Cumulation
C. Sensibilization
D. Idiosyncrasy
E. Dependence

Drugs affecting GIT

352. Patient with gastric peptic ulcer addressed his doctor. After
examination doctor prescribed famotidine aiming at inhibition of gastric
acid secretion. What is the mechanism of this drag action?
A. Inhibition of proton pump
B. Neutralization of gastric acid
C. Prevention of histamine release
D. Blockade of H1 histamine receptors
E. Blockade of H2 histamine receptors*.

353. A young woman with breast cancer after surgery was treated with a
chemotherapeutic complex. At the very beginning of treatment the patient
experienced much distressing nausea and vomiting. Choose the best agent
to relieve patient’s condition
A. Hyoscine (Aeronum)
B. Diphenhydramine (Dimedrolum)
C. Validolum
D. Chlorpromazine (Aminazinum)
E. Metoclopramide*.

354. Patient with gastric peptic ulcer was treated with omeprazole. This
agent is capable of decreasing in HCl production because it
A. Neutralizes gastric acid
B. Blocks H2 –histamine receptors
C. Inhibits the H+, K+-ATPase proton pump*
D. Coats gastric mucosa
E. Blocks muscarinic receptors.

355. A 45-year-old woman presents to the emergency room with severe


epigastric pain. On the base of patient examination and laboratory
findings an acute pancreatitis is diagnosed. Which of the drugs is to be
administered in this case?
A. Pancreatin
B. Mezym forte
C. Contrycal*
D. Morphine
E. Fentanyl.
356. A patient addressed doctor with complaints of heartburn often
troubling him. On examination ulcerative esophagitis was diagnosed and
inhibitor of proton pomp was prescribed. Which of the following drugs
was prescribed?
A. Cimetidine
B. Ranitidine
C. Omeprazole*
D. Misoprostol
E. Metronidazole.

357. A 37-year-old man was admitted to the surgical department with the
symptoms of acute pancreatitis: vomiting, diarrhea, bradycardia,
hypotention, weakness, dehydration of the organism. What medicine
should be used first of all?
A. Ephedrine
B. Platyphylline
C. No-spa
D. Etaperazine
E. Contrycal*.

358. A liquidator of a breakdown at a nuclear power plant who was


irradiated complained about vomiting that occurs all of a sudden. What
medication should be prescribed?
A. Metoclopramide*
B. Aeron
C. Reserpine
D. De-Nol
E. Atropine.

359. Examination of a 35 year old patient revealed high acidity of gastric


juice. What receptors should be blocked in order to reduce it?
A. alpha 1-adrenoreceptors
B. beta2-adrenoreceptors
C. beta1 -adrenoreceptors
D. alpha2-adrenoreceptors
E. Histamine*.

360. For treatment of biliary tract dyskinesia ( hypokinetic type) doctor


prescribed cholagogue, referring to the group holekinetics. Indicate the
drug
A. Magnesium sulfate (gavage)*
B. Alloholum
C. Cholenzymum
D. Oxafenamid
E. Cholosasum.
361. Aged patient is suffering from constipation for a long time. Which
synthetic laxatives should be proposed in this case?
A. Atropine
B. Bisacodyl*
C. Castor oil
D. Liquid paraffin
E. Magnesium sulfate.

362. A patient suffering from gastric ulcer with hyperacidic syndrome


was admitted to hospital. The drug of which group should be used in the
treatment of the patient?
A. Nonsteroidal anti-inflammatory drugs
B. Calcium channel blockers
C. Steroidal anti-inflammatory drugs
D. Blockers H1 histamine receptor
E. H2 histamine receptors lockers*.

363. For patients with peptic ulcer the drug inhibiting the growth and
reproduction of H.pylori was prescribed as a causal treatment. Name the
drug:
A. Corglyconum
B. Glauvent
C. Metronidazole*
D. Furosemide
E. Prazosin.

364. A 46 years old patient was complained of stomach pain, heartburn.


During examination increased gastric acidity was revealed. What should
be assigned to neutralize the acidity of gastric juice?
A. Papaverine
B. Atropine Sulfate
C. Almagel*
D. Neostigmine
E. Benzohexonium.

365. Patients with peptic ulcer of the duodenum uses famotidine. What is
the group of this drug?
A. Beta-agonists
B. Alpha-adrenergic agonists
C. Antiholinesterase
D. H2-blockers of histamine receptors*
E. Irritant.

366. A patient with exacerbation of duodenal peptic ulcer was admitted


to hospital The analysis showed an increased gastric secretion and acid-
forming function of the stomach. Choose a product that will reduce the
secretory function of the stomach by blocking histamine H2-receptor?
A. Famotidine*
B. Atropine Sulfate
C. Plathyphylline
D. Dry extract of belladonna
E. Metacin.

367. A pregnant woman is suffering from heartburn. Which agent is the


best?
A. Ranitidine
B. Sodium bicarbonate
C. Omeprazole
D. Almagel*
E. Bismuth sulfate.

368. The doctor prescribed bisacodyl for patient suffering from chronic
constipation. What pharmaceutical group is the drug?
A. Narcotic analgesics
B. Laxatives*
C. Antihypertensives
D. Blockers H2-histamine receptor
E. Diuretics.

369. A 45 –year-old women is suffering from chronic pancreatitis.


Doctor prescribed her complex preparation containing pancreatic enzymes
and dried bile. Point out prescribed drug.
A. Allocholum
B. Pancreatin
C. Aprotinin
D. Festal*
E. Cholenzymum.

370. A patient with cholecystitis relieves periodical attacks of pain with


myotropic spasmolytic drug. Which of listed agents is used?
A. Drotaverine(No-spa)*
B. Ephedrine
C. Pancreatin
D. Aprotinin
E. Atropine.

371. Patient addressed doctor complaining of periodically occurring


dyspepsia. Laboratory tests did not reveal pathogens that can be
responsible for diarrhoea. Choose the drug the most beneficial in this
case.
A. Atropine
B. Platyphylline
C. Papaverine
D. Bisacodyl
E. Loperamide*.

372. Patient with chronic hepatitis was prescribed complex preparation


with hepatoprotecting properties. Preparation contains essential
phospholipids and vitamins. Point out this preparation
A. Legalon
B. Essentiale*
C. Liv-52
D. Siliborum
E. Papaverine.

373. 25 year-old man was hospitalized to gastroenterology department


with complaints of "hungry" pain, a feeling of heaviness in the stomach,
heartburn. Doctor prescribed gastrocepin (pirenzepine). What effect of
gastrocepin (pirenzepine) was the base for choice of a physician?
A. Relaxation of biliary tract muscle
B. Relaxation of smooth muscles of the stomach
C. Reduction of gastric acid secretion and gastrin*
D. Destruction of Helycobacter pylori
E. Reduction of trypsin secretion .

374. Usage of sodium bicarbonate in peptic ulcer with high acidity gives
temporary relief. What is the cause of short lasting effect of drug in this
case?
A. The formation of carbon dioxide stimulating a secondary
hypersecretion*
B. Neutralization of hydrochloric acid
C. Stimulation of histamine H2-receptors
D. Stimulation of the gastric M-cholinergic receptors
E. Accumulation of histamine.

375. The patient has hyperacidic gastritis. Specify the drug with the
mechanism realized by blocking of proton pump (H +-K +-ATPase) used
in the combined therapy.
A. Gastrocepin (pirenzepine)
B. Omeprazole*
C. Misoprostol
D. Atropine sulfate
E. Almagel.

376. A patient with chronic hyperacidic gastritis to relieve heartburn is


using antacid, it gives prompt effect, but it is accompanied by unpleasant
feeling of fullness in the stomach. Which of the drugs can cause this side
effect?
A. Magnesium trisilicate
B. Aluminum hydroxide
C. Pepsin
D. Magnesium oxide
E. Sodium bicarbonate*.

377. A complex treatment of gastric peptic ulcers includes famotidine.


What is the mechanism of this drug action?
A. M-cholinergic blockade
B. The blockade of proton pump (H +, K +-ATPase)
C. Blockade of H2 histamine receptors*
D. Antacid effect
E. Local anesthetic effect on the mucous membrane of the stomach..

378. A patient has complex treatment of gastric peptic ulcers;


concomitant disease is glaucoma. Drugs of which group in this connection
should not be included in antiulcer therapy?
A. Myotropic antispasmodics
B. Blockers H2-histamine receptor
C. Antacid
D. Cholinoblockers*
E. Gastroprotectors.

379. Almagel was prescribed for patient with gastric ulcer. What is the
mechanism of drug action?
A. The neutralization of hydrochloric acid*
B. Local anesthetic action
C. The blockade of the H2 histamine receptor
D. M-cholinergic blockade
E. Adsorption of alkaline foods.

Drugs acting on haemopoises and haemostasis

380. The patient was using levomycetin for a long time without control,
his examination revealed a decrease in leukocyte count in the blood.
Which drug should be used for stimulation leukopoiesis?
A. Methotrexate
B. Prednisolone
C. Pentoxyluv*
D. Mercaptopurine
E. Cyanocobalamin.

381. Severe anemia occurred in patients after gastric resection on


account of ulceration. What medicine has to be assigned along with
improvement of nutrition?
A. Ferroceronum
B. Cyanocobalamin*
C. Methyluracilum
D. Iron sulfate
E. Coamidum.

382. Blood test revealed anemia in the patient with megaloblastic forms
of red cells, due to this cyanocobalamin was assigned for treatment. At
what type of anemia this vitamin preparation is indicated?
A. Hyperchromic anemia*
B. Hypoplastic
C. Hemolytic
D. Aplastic anemia
E. Hypochromic anemia.

383. Hypochromic anemia appeared in patient due to internal bleeding.


Which of these drugs can provide rapid recovery of red blood cells?
A. Cobamamid
B. Heparin
C. Sirepar
D. Furosemide
E. Ferum-Lek*.

384. After radiation therapy the myelo depressive reaction in the form of
leukopenia appeared in patient. Which of the following drugs should be
applied to stimulate leukopoiesis?
A. Vitogepat
B. Ferropleks
C. Hemostimulin
D. Sodium nucleinat*
E. Nicotinic acid.

385. In the patient that had long history of stomach disease


hyperchromic anemia was found. Which of the following agents is to be
prescribed for this disease?
A. Ascorbic acid
B. Cyanocobalamin*
C. Unithiolum
D. Ferum-Lek
E. Oxyferriscorbone sodium.

386. Iron deficiency anemia is very common among pregnant women,


complicating pregnancy, childbirth, adversely affects the fetus and
newborn. Propose for the treatment such anemia preparation of ferrous
iron for oral administration.
A. Ferkoven
B. Ferrum-lek
C. Hemostimulin
D. Koamidum
E. Iron ferrous lactate*.
387. Patients suffering from angina and receiving isosorbide
mononitrate, was appointed an additional drug with antiplatelet effect.
Identify the drug:
A. Validolum
B. Nitroglycerine
C. Nifedipine
D. Aspirin*
E. Inderal.

388. The child emerged bleeding from the nose. Which of the following
drugs can stop it?
A. Thrombin*
B. Fibrinolysin
C. Vicasolum (Menadione)
D. Potassium chloride
E. Ethamzylatum.

389. Massive bleeding began in woman after separation of the placenta.


Which of the drug you need to help in this case?
A. Thrombin
B. Fibrinolysin
C. Aminocapronic acid*
D. Fepromaronum
E. Vicasolum (Menadion).

390. Gastric bleeding appeared in the patient. Which of these drugs can
stop it?
A. Folic acid
B. Thrombin
C. Sodium citrate
D. Sodium oxalate
E. Fibrinogen*.

391. Anticoagulant therapy with heparin was performed in patient with


myocardial infarction. What preparation should be used to continue
treatment after withdrawal of heparin to prevent recurrence of
thrombosis?
A. Neodikumarinum*
B. Calcium chloride
C. Furosemide
D. Fibrinolysin
E. Sodium hydrocitrate.

392. Analysis of the patient's blood showed an increased number of red


blood cells and hemoglobin that is characteristic of polycythemia. Which
of these drugs should be prescribed for a patient?
A. Kobamamid
B. Contrycal
C. Vitogepatum
D. Imiphosum*
E. Pentoxylum.

393. To stop heavy uterine bleeding ergot alkaloid that increases


myometrium tone was selected. Which of listed agents is of this group?
A. Oxytocin
B. Ergometrine*
C. Neostigmin
D. Aminocapronic acid
E. Fenoterol.

394. Direct acting anticoagulant was prescribed for patients prone to


thrombosis. Point out the drug:
A. Aminocaproic acid
B. Vicasolum (Menadione)
C. Neodikumarin
D. Sinkumar
E. Heparin*.

395. Before tooth extraction a patient was advised to take a certain drug
for hemorrhage prevention. What drug was advised?
A. Heparin
B. Vicasolum (Menadione)*
C. Asparcam
D. Magnesium sulfate
E. Dimedrolum.

396. A patient with essential hypertension was recommended a drug that


prevents thrombosis. This drug exerts antiplatelet and coronary dilating
effects. What drug is it?
A. Heparin
B. Dipiridamole*
C. Protamine sulfate
D. Neodicumarin
E. Syncumar.

397. In acute thrombosis intravenous heparin rlieves the patient's


condition. Which of these mechanisms is responsible for effect of
heparin?
A. Dissolution of fresh thrombus
B. Inhibition of the transformation of prothrombin to thrombin in the
blood*
C. Inhibition of the synthesis of prothrombin in the liver
D. Reduction of the number of calcium ions in the blood
E. Influence on the retraction of the clot.

398. In the laboratory experimental studies in vitro and in vivo


antagonist of direct anticoagulant was selected for the clinical use. What
could be the preparation of these?
A. Protamine sulfate*
B. Sodium citrate
C. Contrycal
D. Menadione
E. Calcium chloride.

399. Massive uterine bleeding in woman was stopped by introducing


aminocaproic acid. Which of listed mechanisms contributed to hemostatic
effect of this drug?
A. Inhibition of fibrinolysis*
B. Activation of fibrinolysis
C. Increased prothrombin in the blood
D. Increasing calcium in the blood
E. Increased synthesis of prothrombin in the liver.

400. 8 year-old child is preparing for tonsillectomy. Blood analysis


showed clotting time increased to 7 minutes. What medication should be
included in a complex of drugs for preparation period of 5 days before
surgery in the first place?
A. Etamsylate
B. Vicasolum*
C. Calcium chloride
D. Fibrinogen
E. Aminocapronic acid.

401. For the prevention of myocardial infarction doctor prescribed


nonsteroidal anti inflammatory drug with antiplatelet effect - aspirin.
Choose the dose and regimen at which the drug will exert antiplatelet
effect.
A. 0.1 three times a day
B. 0.25 three times a day
C. 1.0 three or four times a day
D. 0.5 three times a day
E. 0.1-0.2 once a day for a long time*.

402. For the treatment of hypochromic anemia patient takes iron ferrous
sulfate. Specify one of the side effects of this drug.
A. The darkening of the teeth*
B. Dyspepsia
C. Allergy
D. Itching
E. Dizziness.
403. Vicasolum (Menadione) has been prescribed for patient a week
before tonsillectomy to prevent bleeding. Specify the mechanism of action
of this drug.
A. Inhibits fibrinolysis
B. Activates fibrinolysis
C. Increases the synthesis of clotting factors in the liver*
D. Inhibits ACE
E. Causes vasoconstriction.

404. Indirect acting anticoagulant warfarin was prescribed for patient to


prevent thrombosis. What is the mechanism of action of this drug?.
A. Inhibition of fibrinolysis
B. Activation of fibrinolysis
C. Decrease in the synthesis of clotting factors in the liver*
D. Inhibition of ACE
E. Inhibition of platelets aggregation.

405. On discontinuation of heparin treatment of patient with myocardial


infarction administration of warfarin was started. This drug exerts its
anticoagulant effect by
A. Blocking calcium binding to clotting factors
B. Forming a complex with clotting factors
C. Breaking down thrombin
D. Inhibiting of pro-clotting factor synthesis in the liver*
E. Depolymerization of fibrin.

406. A patient with myocardial infarction was treated with intravenous


infusion of heparin. What is the goal of this drug administration?
A. To cause the lysis of thrombus directly
B. To transform plasminogen into plasmin
C. To prevent further thrombosis*
D. To prevent platelets activation
E. To decrease the area of necrosis.

407. A patient with a history of angina pectoris and high plasma


cholesterol level was prescribed phenofibrate. What is the mechanism of
action of this drug?
A. Activation of lipoprotein lipase*
B. Inhibition of lipoprotein lipase
C. Inhibition of hydroxymethyl glutaryl coenzyme A
D. Inhibition of LDL oxidation
E. Bile acid sequestration.

408. Patient suffering from pernicious anemia is treated with folic acid.
Which agent is to be added to make effective the treatment of this
disease?
A. Retinoli acetas
B. Cyancobalamin*
C. Thiamine bromide
D. Retabolil
E. Insulin.

409. A patient ill with essential hypertension was recommended a drug


that prevents thrombosis. It is to be taken parenterally. What drug is it?
A. Protamine sulfate
B. Amben
C. Heparin*
D. Neodicumarin
E. Syncumar.

Drugs acting on myometrium. Antigout drugs.

410. Choose the hormonal drug for enhancement of uterine contraction


during labor
A. Dinoprost (PGF2)
B. Oxytocin*
C. Ergometrine
D. Salbutamol
E. Partusisten

411. To stop postpartum bleeding doctor prescribed ergot alkaloid


derivative. Point out this drug
A. Oxytocin
B. Methylergometrine*
C. Neostigmin
D. Dinoprostone (PGE2)
E. Qinine

412. Pregnant woman developed rhythmic uterine contractions at the


physiological term of delivery. But contractions were not progressing in
force and frequency. Cervix of uterus was neither softening nor opening.
Chose the drug, which is the most beneficial in this case.
A. Oxytocin
B. Pituitrin
C. Ergometrine
D. Dinoprostone*
E. Partusisten

413. A pregnant woman developed rhythmic uterine contractions at the


physiological term of delivery. But contractions were not progressing in
force and frequency. Choose the drug for stimulation of uterine
contraction.
A. Oxytocin*
B. Ergometrine
C. Partusisten
D. Physostigmine
E. Castor oil.

414. Intake of oral contraceptives containing sex hormones inhibits


secretion of the hypophysiae hormones. Secretion of which of the
indicated hormones is inhibited while taking oral contraceptives with sex
hormones?
A. Somatotropic
B. Oxytocin
C. Vasopressin
D. Follicle-stimulating*
E. Thyrotropic.

Vitamins

415. There is an inhibited coagulation in the patients with bile ducts


obstruction, bleeding due to the low level of absorbtion of a vitamin.
What vitamin is in deficiency?
A. Carotene
B. E
C. K*
D. A
E. D.

416. A patient suffers from vision impairment - hemeralopy (night


blindness). What vitaminpreparation should be administered the patient in
order to restore his vision?
A. Thiamine chloride
B. Tocopherol acetate
C. Retinol acetate*
D. Vicasol
E. Pyridoxine.
417. Patients after long-term complex treatment that included vitamin
drug began to complain of muscular weakness, loss of appetite, nausea,
and diarrhea. On examination tachycardia, hypertension, weakening of the
heart sounds were revealed. In the urine protein, red blood cells,
hypercalciuria were revealed, in the blood - increasing the concentration
of Ca and P was found. What phenomenon was observed in a patient?
A. Hypovitaminosis vit. C
B. Hypervitaminosis vit. A
C. Hypervitaminosis vit. D*
D. Hypovitaminosis vit. A
E. Hypervitaminosis vit. K.

418. The retardation of fontanelle closure and teeth appearance were


observed in child. The increase in activity of alkaline phosphatase and
decrease in the concentration of citrate in the blood were found. This
violation is related to deficiency of vitamin:
A. Vitamin D*
B. Vitamin A
C. Ascorbic acid
D. Vitamin E
E. Vitamin K.

419. The patient with myocardiodystrophy appointed vitamin


preparation with a potent antioxidant effect. Which of these drugs has
such action?
A. Rutin
B. Pyridoxine
C. Menadione
D. Riboflavin
E. Tocopherol*.

420. The patient has dementia, diarrhea and dermatitis. Which of the
following drugs should be included in complex treatment of this patient?
A. Nicotinamide*
B. Cobalamin
C. Calcium pantothenate
D. Riboflavin
E. Thiamine nitrophosphate.

421. In order to accelerate healing of a radiation ulcer a vitamin drug


was administered. What drug was it?
A. Levamisole
B. Retabolil
C. Prednisolone
D. Retinol acetate*
E. Methyluracil .

Hormonal, enzymatic and antienzymatic drugs

422. A person has reduced diuresis, hypernatremia, hypokalemia.


Hypersecretion of what hormone can cause such changes?
A. Parathormone
B. Aldosterone*
C. Adrenalin
D. Vasopressin
E. Auricular sodiumuretic factor.

423. A patient with infectious mononucleosis had been taking


glucocorticoids for two weeks. He was brought into remission, but he fell
ill with acute attack of chronic tonsillitis. What action of glucocorticoids
caused this complication?
A. Antiallergic
B. Immunosuppressive*
C. Anti-inflammatory
D. Antishock
E. Antitoxic.

424. A patient had been taking glucocorticoids for a long time. When the
preparation was withdrawn he developed the symptoms of disease
aggravation, decreased blood pressure and weakness. What is the reason
of this condition?
A. Appearance of adrenal insufficiency*
B. Hyperproduction of ACTH
C. Cumulation
D. Sensibilization
E. Habituation.

425. Examination of a 60 y.o. patient revealed hyperglycemia and


glucosuria. A doctor administered him a medication for internal use. What
medication is it?
A. Oxytocin
B. Corglycon
C. Furosemide
D. Pancreatine
E. Glibenclamid*.

426. A patient ill with collagenosis has been taking prednisolone for a
long time. Hypokaliemia development caused spastic pain of skeletal
muscles. What medication should be used in order to correct potassium
exchange?
A. Dithylinum
B. Panangin*
C. Thyrocalcitonin
D. Diazepam
E. Noshpa.

427. In course of histidine catabolism a biogenic amin is formed that has


powerful vasodilatating effect. Name it:
A. Histamine*
B. Dopamine
C. Dioxyphenylalanine
D. Noradrenalin
E. Serotonin.

428. Continious taking of a drug can result in osteoporosis. erosion of


stomach mucous membrane, hypokaliemia, retention of sodium and
water, reduced content of corticotrophin in blood. Name this drug:
A. Digoxin
B. Indometacin
C. Reserpine
D. Prednisolone*
E. Hydrochlorothiazide.

429. Examination of a 70 year old patient revealed non insulin-


dependent diabetes. What drug should be administered?
A. Glibenclamid*
B. Parathyroidin
C. Insulin
D. Mercazolilum
E. Cortisone.

430. Examination of a 60 year old patient revealed hyperglycemia and


glucosuria. A doctor administered him a medication forinternal use. What
medication is it?
A. Pancreatine
B. Furosemide
C. Oxytocin
D. Glibenclamid*
E. Corglycon.

431. Doctor prescribed preparation of posterior pituitary for the woman


with the uterine inertia. Choose the preparation:
A. Dihydroergotamine
B. Vasopressin
C. Oxytocin*
D. Adrenocorticotropin
E. Dinoprost.

432. The levorotatory isomer of thyroid hormone was assigned to the


patient with diagnosed myxedema. What drug was assigned?
A. Levodopa
B. L-thyroxine*
C. L-asparaginase
D. L-carnitine
E. Parathyroid hormone.
433. Doctor prescribed the inhalation of drug with proteolytic action to
improve sputum discharge. Mark drug that has proteolytic effect.
A. Trypsin*
B. Lidaza
C. Pepsin
D. Streptokinase
E. Festal.

434. Choose the drug that is a hormone produced by alpha-cells of


Langerhan’s islets of the pancreas.
A. Insulin
B. Glucagon*
C. Somatostatin
D. Natriuretic hormone
E. Testosterone.

435. The patient complained of a acute girdle pain in area of pancreas


projection. Which of the following drugs should be selected to provide
emergency aid to him?
A. Potassium chloride
B. Contrykal (Aprotinin)*
C. Platyphylline hydrotartrate
D. Pancreatin
E. Bendazol.

436. The patient with severe chest pain was admitted to the infarction
department. On the ECG extensive myocardial infarction caused by
thrombosis of the left coronary artery was revealed, and therefore the
administration of thrombolytic agent was started immediately. What the
drug was used in the treatment of this patient?
A. Collagenase
B. Hyaluronidase
C. Trypsin
D. Streptokinase*
E. Ribonuclease.

437. The patient was diagnosed a myocardial infarction with thrombosis


of the left coronary artery. Which group of drugs is used to restore blood
flow?
A. Nonopioid аnalgesics
B. Fibrinolysis activators*
C. Beta-blockers
D. ACE inhibitors
E. Glucocorticoids.
438. Doctor prescribed enzymatic preparation to the patient with
hypoacidic gastritis. Which of these preparations will improve digestion
in the stomach?
A. Pepsin*
B. Pancreatin
C. Trypsin
D. Medaza
E. Contrykal (Aprotinin).

439. Replacement hormonal treatment was assigned for patient with


hypothyrosis. Which drug should be selected for treatment?
A. Merkazolil (Thiamazole)
B. Parathyreoidinum (Parathyreocrinum)
C. Dihydrotachysterol
D. Potassium perchlorate (Kalii perchloridi)
E. Thyroidin*.

440. Patients aged 73 years has slovly healing fracture of the femoral
neck. What of hormones with anabolic effect can be assigned to him?
A. Parathyreoidin (Parathormon, Parathyreocrinum)
B. Prednisolone
C. Insulin
D. Retabolil (nandrolone)*
E. Thyroxine.

441. Blood pressure and blood glucose level were increased in female
patients with low blood pressure after parenteral administration of the
hormone. Which hormone was injected?
A. Folliculin (Estrone)
B. Adrenaline*
C. Glucagon
D. Progesterone
E. Insulin.

442. Hemostatic therapy that was assigned to a patient with a bleeding


gastric ulcer did not give desirable effect. The activation of the
fibrinolytic system was found in the patien’t blood. The administration of
what drug is beneficial in this case?
A. Aminocaproic acid*
B. Contrykal (Aprotinin)
C. Medical gelatin
D. Captopril
E. Fibrinogen.

443. To remove necrotic tissue of wound the surgeon made a wound


dressing with chymotrypsin. Explain the mechanism of drug action.
A. Inhibits protease
B. Depolymerization of nucleic acids
C. Depolymerization of hyaluronic acid
D. Breaks peptide bonds in proteins
E. Breaks bonds formed by aromatic residues and / acids*.

444. A woman addressed the pediatrician concerning illness of the 8


months child manifested by sweating, increased size of the crown,
retarded dentition (only 2 of the tooth), and anxiety. What medication
should be assigned at the first place?
A. Calcium pangamas
B. Folic acid
C. Cyanocobalamin
D. Thiamin
E. Ergocalciferol*.

445. The woman had several spontaneous miscarriages. The lack of


which vitamin can cause this?
A. Thiamine bromide
B. Vitamin D
C. Vitamin K
D. Vitamin E*
E. Ascorbic acid.

446. The patient complains of increased vascular fragility (touch to the


skin are bruises), bleeding from the gums. Appointment of ascorbic acid
reduced these symptoms. Specify the mechanism of action of the drug on
vascular permeability.
A. Stimulates the synthesis of corticosteroids
B. Stabilizes the cell membrane
C. Improves oxidation-inflammatory processes
D. Increases collagen production*
E. Stimulates the conversion of folic acid to folinic.

447. The patient developed a hypersensitivity to common cold after


long-term use of one of the therapeutic agents. Which of these drugs
could reduce immunity?
A. Sustak forte (Nitroglycerin)
B. Prednisone*
C. Reserpine
D. Tincture Ginsengi
E. Clonidine.

448. 16 years old young man was suffering from diabetes mellitus for 10
years. He heard about the replacement of insulin by glibenclamide tablets.
However, the doctor refused to change insulin for glibenclamide tablets.
Why glibenclamide is not assigned in this case?
A. Rapidly degraded in the liver.
B. Deplete the function of beta cells of the pancreas.*
C. Causes hyperproduction of hydrocortisone.
D. Increased allocation of adrenaline.
E. Stimulates the alpha cells of the pancreas..

449. Dermatologist for topical treatment of eczema selected ointment


containing fluorinated glucocorticoid, which provides a more expressed
antiinflammatory effect. Which of the following dermatological ointments
was applied?
A. Hydrocortisone ointment
B. Prednizolone ointment
C. Ointment "Sinaflan"*
D. Diclophenac ointment
E. Heparin ointment .

450. Patient was on glucocorticoids for a long time, discontinuation of


usage caused exacerbation of the illness, decreased BP, weakness. How
can you explain it?
A. Insufficiency of adrenal glands*
B. Adaptation to the medicine
C. Sensitization
D. Hyperproduction of ACTH
E. Cumulation.

451. Testosterone and it's analogs increase the mass of skeletal muscles
that allows to use them for treatment of dystrophy. Due to interaction of
the hormone with what cell substance is this action caused?
A. Proteins- activators of transcription
B. Membrane receptors
C. Ribosomes
D. Chromatin
E. Nuclear receptors*.

452. A patient ill with neurodermatitis has been taking prednisolone for a
long time. Examination revealed high level of sugar in his blood. This
complication is caused by the drug influence
A. Glycogenogenesis activation
B. Gluconeogenesis activation*
C. Intensification of glucose absorption in the bowels
D. Activation of insulin decomposition
E. Inhibition of glycogen synthesis.

453. Continious taking of a drug can result in osteoporosis, erosion of


stomach mucous membrane, hypokaliemia, retention of sodium and
water, reduced content of corticotropin in blood. Name this drug:
A. Digoxin
B. Hydrochlorothiazide
C. Prednisolone*
D. Indometacin
E. Reserpine .

454. A patient suffers from diabetes melitus. After the regular insulin
injection his condition grew worse: there appeared anxiety, cold sweat,
tremor of limbs, general weakness, dizziness. What preparation can
eliminate these symptoms?
A. Adrenaline hydrochloride*
B. Butamide
C. Caffeine
D. Noradrenaline
E. Glibutide .

455. Examination of a 60 y.o. patient revealed hyperglycemia and


glucosuri A doctor administered him a medication for internal use. What
medication is it?
A. Corglycon
B. Furosemide
C. Oxytocin
D. Pancreatine
E. Glibenclamid*.

456. An elderly female patient suffers from the type 2 diabetes mellitus
accompanied by obesity,
A. Insulin
B. Glibenclamid*
C. Retabolil
D. Lovastatin
E. Amlodipine .

Anti-inflammatory and antiallergic drugs. Immunomodulators

457. A patient with continious bronchopneumonia was admitted to the


therapeutic department. Antibiotic therapy didn't give much effect. What
medication for improvement of immune state should be added to the
complex treatment of this patient?
A. Timaline*
B. Analgin
C. Sulfocamphocaine
D. Paracetamol
E. Benadryl.
458. A patient with rheumatoid arthritis who had been treated with
indometacin has got signs of gastropathy. What activity of the drug can
this complication be connected with?
A. Anticyclooxygenase*
B. Antiserotonin
C. Antihistamine
D. Locally irritating
E. Antikinine.

459. A 45-year-old woman suffers from seasonal allergic rhinitis caused


by the ambrosia blossoming. What medicine from the stabilizer of the
adipose cells group can be used for prevention of this disease?
A. Diazoline
B. Tavegyl
C. Phencarol
D. Dimedrol
E. Kromolin sodium*.

460. A doctor administered a patient with allergic dermatitis a H1 -


histamine blocker as a part of complex treatment. Name this medication:
A. Prednisolone
B. Loratadine*
C. Cromolyn sodium
D. Hydrocortisone
E. Adrenaline.

461. A 12 year old child has intolerance to some foodstuffs. Their


consumption causes an allergic reaction in form of itching skin eruptions.
What antihistaminic drug should be admistered so that the child could
attend school?
A. Loratadine*
B. Dimedrol
C. Ephedrine
D. Aminophylline
E. Diclofenac.

462. Signs of gastropathy occurred in the patient with rheumatoid


arthritis who was treated with indometacin. With what activity of the drug
can this complication be connected?
A. Antikinine
B. Antiserotonin
C. Antihistamine
D. Anticyclooxygenase*
E. Local irritating.

463. A female patient consulted a doctor about pain and limited


movements in the knee joints. Which of the following nonsteroid anti-
inflammatory drugs should be administered taking into consideration that
the patient has a history of chronic gastroduodenitis?
A. Celecoxib*
B. Diclofenac sodium
C. Promedol
D. Acetylsalicilic acid
E. Butadiounum .

464. The patient suffering from arthritis was treated with prednisolone
for long period. Doctor decided to discontinue prednisolone
administration and change it for another drug. What is the right way of
this drug discontinuation?
A. It must be preceded by corticotropin administration*
B. It must be done in a vary short time
C. It must be preceded by administration of ADH
D. It must be performed on the background of mineralocortcoid
administration
E. It must be preceded by growth hormone administration.

465. A 45-year-old woman suffers from allergic seasonal coryza caused


by the ambrosia blossoming. What medicine from the stabilizer of the
mast cells group can be used for prevention of this disease?
A. Tavegyl
B. Diazoline
C. Phencarol
D. Ketotifen*
E. Dimedrolum.

466. Patient was on glucocorticoids for a long time, discontinuation of


usage caused exacerbation of the illness, decreased BP, weakness. How
can you explain it?
A. Sensitization
B. Adaptation to the medicine
C. Hyperproduction of ACTH
D. Insufficiency of adrenal glands*
E. Cumulation.

467. Patient suffering from rheumatoid arthritis was treated with potent
nonsteroidal anti-inflammatory drug. The treatment was complicated by
gastric bleeding. What drug was used?
A. Paracetamol
B. Indomethacin*
C. Analgin
D. Prednisolone
E. Dexametasone.
468. In complex treatment of patient with bronchial asthma cromolyn-
sodium was included as antiallergic agent. What is the mechanism of this
drug action?
A. Inhibition of histaminic H1 receptors
B. Inhibition of histaminic H2 receptors
C. Prevention of mast cell degranulation*
D. Inhibition of antigen-antibody interaction
E. Blockade of D2 receptors.

469. A patient who had myocardial infarction was administrated 75 mg


of acetylsalicylic acid a day. What is the purpose of this administration?
A. Coronary vessels dilatation
B. Temperature reduction
C. Inflammation reduction
D. Pain relief
E. Redaction of thrombocyte aggregation*.

470. Continuous use of certain drug may cause osteoporosis, erosion of


gastric mucosa, hypokalemia, retention of sodium and water in the
organism, decreased concentration of corticotropin in blood. What drug is
it?
A. Reserpine
B. Digoxine
C. Hypothiazide
D. Prednisolone*
E. Indomethacin.

471. A 33 –year-old female patient, who long treated for chronic


polyarthritis, complains of high blood pressure, changes in distribution of
body fat, menstrual disorders. Admission of which drug is related to these
complaints?
A. Prednisolone*
B. Beclomethasone
C. Phenylbutazone
D. Sinaflan
E. Indomethacin.

472. Prolonged use of steroid and non-steroid anti-inflammatory drugs in


patients with rheumatoid arthritis has not given effect. The doctor
changed non-steroid anti-inflammatory drugs for the preparation of gold.
Find this preparation among the listed drugs.
A. Krizanol*
B. Hingamin
C. Prednisolone
D. Plaquenil
E. D-penicillamine.
473. What pharmacological property of dimedrolum (diphenhydramine)
is used in the hives treatment?
A. Interference with formation of the antigen-antibody complex
B. Elimination of histamine action on the cell*
C. Sedative effect on the central nervous system
D. Violation of antibody synthesis
E. Inhibition of biologically active substances release by mast cell.

474. The patient who received steroid anti-inflammatory drugs had a


complication in the form of hypokalemia. Which of these drugs should be
used for correction of potassium level?
A. Sodium chloride
B. Magnesium sulfate
C. Panangin*
D. Calcium lactate
E. Calcium chloride.

475. Immune stimulant of animal origin was included in the complex


treatment of patients with severe bronchopneumonia. Which of these
drugs belong to this group of drugs?
A. Timalin*
B. Pirogenal
C. Prodigiozan
D. Levamisole
E. Sodium nucleinat.

476. Patient has got severe trauma in car accident due to this kidney
transplant was performed in emergency procedure. Which of the
following drugs should be included in the post-operative therapy for
successful engraftment of the transplanted organ?
A. Pirogenal
B. Levamisole
C. Timalin
D. Prodigiozan
E. Azathioprine*.

477. Anaphylactic shock developed in patient after administration of


lidocaine. Which of the following medications must choose a doctor to
bring the patient out of shock?
A. Acetylsalicylic acid
B. Corticotropin
C. Azathioprine
D. Adrenaline*
E. Aminopyrine.

478. A patient suffering from rheumatism was treated with prednisolone


for two months. After the abrupt discontinuation of the drug
administration patient’s blood pressure dropped dramatically, there was a
severe weakness, tachycardia, reappeared joint pain. Which condition was
developed in patient due to prednisolone discontinuation?
A. Orthostatic collapse
B. Withdrawal*
C. Allergic reaction
D. Anaphylactic shock
E. Hyperglycemic coma.

479. A patient with rheumatoid arthritis was on non-steroidal anti-


inflammatory drug - diclofenac sodium. After some time worsening of
associated disease occurred in the patient that required withdrawal of
medication. What disease was associated with rheumatoid arthritis?
A. Coronary artery disease
B. Bronchial asthma
C. Gastric ulcer*
D. Diabetes mellitus
E. Hypertention.

480. A patient with rheumatoid arthritis was on diclofenac sodium. After


blood test the doctor canceled the drug administration. What the
abnormality in blood test was the base for diclofenac sodium
discontinuation?
A. Enhancement of hemocoagulation
B. Eosinophilia
C. Leukopenia
D. Leukocytosis
E. Decrease in blood coagulation*.

481. The 12-year-old child has intolerance of some food stuffs. Their use
causes an allergic reaction in the form of itchy skin rash. What
antihistamines should be appointed, not to interfere with school activities
of the child (not cause drowsiness)?
A. Loratadine*
B. Mezaton
C. Diclofenac sodium
D. Aminophylline
E. Diphenhydramine.

482. To prevent transplant rejection after organ transplantation


immunosuppressive hormone therapy is compulsory. What group of
hormones is used for this purpose?
A. Catecholamines
B. Thyroid
C. Glucocorticoids*
D. Mineralocorticoids
E. Sex hormones.
483. Student addressed the doctor to appoint a drug for the treatment of
allergic rhinitis occurred in lime blossoming. Which drug can be used?
A. Loratadine*
B. Losartan
C. Noradrenaline gidrotartrat
D. Inderal
E. Ambroxol.

484. In the complex therapy of patient with allergic dermatitis doctor


prescribed H1-histamine blockers. Point out the drug.
A. Hydrocortisone
B. Prednisolone
C. Cromolyn sodium
D. Loratadine*
E. Adrenaline.
Antiseptics and disinfectants

485. For the preparation of a patient`s burn skin surface a certain


medication was used. Its antiseptic action is provided by free oxygen that
segregates in presence of organic substances. Choose the right answer:
A. Potassium permanganate*
B. Chlorhexidine
C. Sodium bicarbonate
D. Boric acid
E. Furacilin.

486. Patient suffering from tonsillitis was recommended to gargle throat


with preparation of nitrofurans. What drug was recommended?
A. A solution of hydrogen peroxide,
B. Solution of furacilinum,*
C. Solution of silver nitrate,
D. Chlorhexidine
E. Alcoholic solution of iodine.

487. Burned skin surface was treated with antimicrobial preparation. Its
antiseptic properties are provided by atomic oxygen that is formed in
presence of organic substances. What preparation was applied?
A. Potassium permanganate*
B. Furacillin
C. Chlorhexidine bigluconate
D. Alcoholic iodine solution
E. Boric acid .

488. Patient with abscess of the cut wound applied to the traumatological
department. Doctor for the cleaning of the wound from the pus washed it
with 3% hydrogen peroxide. Foam was absence. What is the cause of
absence of the drug activity?
A. Pus in the wound
B. Low concentration of H2O2
C. Inherited insufficiency phosphatdehydrogenase of erythrocyte
D. Shallow wound
E. Inherited insufficiency of catalase*.

489. A group of students were traveling by countryside collecting


medicinal plants. It was a hot day and the students spent the entire
drinking water. The team leader offered the water of the pond, using the
chlorine containing tablet for its disinfection. Point the drug.
A. Chloramine-B
B. Chlorhexidine
C. Halozan*
D. Brilliant Green
E. Furacilinum.
490. A nurse washed the patient’s contaminated wound with 3%
hydrogen peroxide solution that causes formation a lot of foam. What is
the mechanism of antiseptic effect of this drug?
A. The formation of molecular oxygen*
B. The formation of atomic form of oxygen
C. The interaction of hydrogen peroxide with fibrinolysine
D. Aggressive action of hydrogen peroxide on tissues
E. Formation of albuminates.

491. Patient with burns was treated by application of antiseptic


containing halogen and polyvinyl pirrolidon. Point out this preparation.
A. Iodovidon*
B. Halozan
C. Chlorhexidine
D. Chloramine
E. Pantocide.

492. The patient was admitted to the infectious disease clinic with
complaints of vomiting, diarrhea, fever, after errors in diet. What
antiseptic should be used for gastric lavage (washing out)?
A. Ethanol
B. Hydrogen peroxide
C. Methylene blue
D. Activated char coal
E. Potassium permanganate*.

493. A patient has purulent wound with necrotic content. What


medication should be used to cleanse the wound?
A. Hydrogen peroxide*
B. Furacilinum (Nitrofurazone)
C. Sulfacyl sodium
D. Alcohol solution of iodine
E. Aetacridini lactas.

494. In the patient with syphilis treated by, bismuth preparations gray
patches on the mucous membrane of the mouth and symptoms of kidney
disturbances appeared. Which agent is advisable for the treatment of
bismuth poisoning?
A. Naloxone
B. Unitiol*
C. Methylene blue
D. Nalorphine
E. Bemegride.

240
495. Dentist handled herpetic lesions of 7 years old child with halogen
antiseptic that has antimicrobial, antifungal and antiviral properties. Name
the drug.
A. Alcohol solution of iodine*
B. Potassium permanganate
C. Formaldehyde
D. Silver nitrate
E. Brilliant Green.

496. The patient with gingivitis after using applications was assigned to
rinse oral cavity with antiseptic agent realizing its effect by atomic
oxygen drug exerts deodorant and astringent effects. Identify the drug.
A. Ethanol
B. Chlorhexidine bigluconate
C. Hydrogen peroxide
D. Sodium bicarbonate
E. Potassium permanganate*
F. Hydrogen peroxide.

497. To prepare the operative field surgeon used a solution of iodine in


alcohol. What is the chemical group of this antiseptic preparation?
A. Heavy metals
B. Halogen-containing compounds*
C. Alcohols
D. Detergents
E. Aliphatic substance.

498. For disinfecting of metallic instruments in the surgical department


the formaldehyde solution is used. What is the chemical group of this
antiseptic preparation?
A. Halogenated compounds
B. Aromatic substance
C. Detergents
D. Aliphatic agent*
E. Alcohols.

499. After extirpation of the tooth in the patient bleeding from the hole
emerged. What drug from the group of antiseptics should be used in this
case?
A. A solution of hydrogen peroxide*
B. Solution of epinephrine hydrochloride
C. Solution of brilliant green
D. Iodinol
E. A solution of ethyl alcohol.

241
500. Nitrofurane antiseptic was prescribed for mouth wash to patient
with stomatitis. Point out this agent
A. Hexamethylen tetraminum
B. Aethonium
C. Furacilinum*
D. Boric acid
E. Aethacridini lactates.

501. Teenager addressed dermatologist complaining of acne. Doctor


prescribed him an antiseptic agent that gradually releases oxygen and is
traditionally used locally for treatment of acne. Point out prescribed drug.
A. Hydrogen peroxide
B. Potassium permanganate
C. Benzoyl peroxide*
D. Boric acid
E. Methylenum coeruleum.

502. To prevent pyodermia in 4-year-old girl with chickenpox mother


was recommended to apply a preparation of a group of dyes. Point out
this preparation.
A. Chlorhexidine
B. Alcoholic solution of iodine
C. Furacilinum
D. Alcoholic solution of of Brilliant Green*
E. Chloramine.

503. Before the operation surgeon used degmicidum for disinfecting his
hands. Point the group of this antimicrobial agent
A. Group of detergent*
B. Group of dyes
C. Halogen containing antiseptic
D. Oxidizing agent
E. Group of nitrofuranes .

504. Doctor prescribed oral rinse for patient with stomatitis. Which
antiseptic of oxidazinggroup is most suitable for this?
A. Boric acid
B. Potassium permanganate*
C. Alcoholic solution of iodine
D. Chloramine
E. Ethanol.

505. Gastric lavage with a solution of potassium permanganate was


performed in patient due to food poisoning. Which group of antiseptics
does this drug belong to?
242
A. Aromatic substances
B. Halogens
C. Acid
D. Oxidizing agents*
E. Detergents.

506. Solution of carbolic acid was used for disinfecting of instruments.


To which group does this antiseptic belong?
A. Oxidizing agents
B. Phenols*
C. Halogens
D. Acid
E. Detergents.

507. For disinfecting of hand before operation surgeon used chlorine


containing antiseptic, that also widely used in dentistry. Point out this
agent.
A. Potassium permanganate
B. Boric acid
C. Alcoholic solution of iodine
D. Chlorhexidine*
E. Ethanol.

508. Surgeon was urgently summoned to the operating room. Choose the
antiseptic used for surgeon hands in emergency.
A. Boric acid
B. Ethyl alcohol 96%*
C. Ethacridine lactate
D. Hydrogen peroxide
E. Ethyl alcohol 70%.

Synthetic antimicrobial drugs. Fluoroquinolones. Sulfonamides.

509. A 7 vear old child is ill with bronchitis. It is necessary to administer


him an antibacterial drug. What drug of fluoroquinolone group is
CONTRA-INDICATED at this age?
A. Cyprofloxacin*
B. Sulfadimethoxine
C. Ampiox
D. Ampicillin
E. Amoxicillin.

243
510. Patient with pneumonia has intolerance to antibiotics. Which of the
combined sulfanilamide medicines should be prescribed to the patient?
A. Sulfacyl sodium
B. Aethazol
C. Biseptol*
D. Streptocid
E. Sulfadimethoxine.

511. A patient consulted a stomatologist about purulent inflammation of


his gums. What drug will be the most effective if it is suspected that a
causative agent is an anaerobe?
A. Co-trimoxazole
B. Gentamicin
C. Oxacillin sodium
D. Metronidazole*
E. Nitroxoline .

512. A patient with pneumonia had a complex treatment that included


sulfonamide preparation. What should doctor recommend to prevent
cristaluria?
A. Decrease the dose of sulfonamide
B. Drinking of alkaline solution*
C. Administration of drugs before meal
D. Drinking of acidic solution
E. Use of vitamins.

513. Gonorrhoea was revealed in the patient on bacterioscopy of the


smear from urethra. Taking into account that medicines for gonorrhea are
fluorquinolones, patient should be prescribed:
A. Ciprofloxacin*
B. Furazolidone
C. Fluorouracil
D. Urosulfan
E. Cefazoline.

514. Patient with pneumonia has intolerance to antibiotics. Which of the


combined sulfanilamide medicine should be prescribed to the patient?
A. Trimethoprim
B. Aethazol
C. Natrium sulfacyl
D. Streptocid
E. Biseptol*.

515. A 30-year-old patient complains about having abdominal pain and


diarrhea for 5 days body temperature rise up to 37,5o C along with chills.
244
The day before patient had been in forest and drunk from open water
reservoir. Laboratory analysis enabled to make the diagnosis of amebic
dysentery. What is the drug of choice for its treatment?
A. Phthalazol
B. Furazolidonum
C. Emetine hydrochloride
D. Metronidazole*
E. Levomycetin.

516. What is the mechanism of antimicrobial effect of sulfonamide?


A. Inhibition of protein synthesis
B. Inhibition of bacterial cell wall synthesis
C. Competitive antagonism with para-aminobensoic acid*
D. Inhibition of cell membrane synthesis
E. Inhibition of nucleic acid synthesis.

517. Combined sulfonamide preparation with bactericidal effect was


prescribed for treatment of sore throat in 13 years old child. Point out this
drug.
A. Biseptol*
B. Eneroceptol
C. Sulfalen
D. Aethasolum
E. Urosulfanum.

518. A HIV patient has bacterial dysentery. On the result of


antimicrobial sensitivity test doctor prescribed him cotrimoxazol
(bactrim,biseptol). What type of antibacterial action has this drug?
A. Virostatic
B. Bacteriostatic
C. Fungicidal
D. Fungistatic
E. Bactericidal*.

519. A Bactrim was prescribed for 35-years old patient with bronchitis.
What is the mechanism of action of this drug?
A. Sequential block of two stages of bacterial folate metabolism*
B. Inhibition of bacterial cell wall synthesis
C. Inhibition of protein synthesis
D. Inhibition of cell membrane synthesis
E. Inhibition of nucleic acid synthesis.

520. For treatment of typhoid fever ciprofloxacin was prescribed. What


is mechanism of this drug action?
A. Inhibition of bacterial cell wall synthesis
245
B. Inhibition of DNA gyrase*
C. Inhibition of protein synthesis
D. Competitive antagonism with paraaminobenzoic acid
E. Inhibition of nucleic acid synthesis.

521. Due to which side effect ciprofloxacin is contraindicated in


children?
A. Hepatotoxicity
B. Glucosuria
C. Cartilage damage*
D. Nausea
E. Insomnia.

522. A doctor prescribed sulfonamide drug to patient with acute


bronchitis. After urinalyses he changed it for antibiotic of penicillin
group. The possibility of which side effect of sulfonamide was the base
for changing it by other drug?
A. Mental confusion
B. Hyperglycemia
C. Allergic reaction
D. Crystalurea*
E. Glucosurea .

523. Urinalyses revealed crystaluria in patient who used synthetic


antimicrobial drug for treatment of bronchitis. Which group of
antimicrobial drug can cause such side effect?
A. Sulfonamides*
B. Fluorochinolons
C. Tetracyclines
D. Penicillins
E. Macrolydes.

524. Sulfonamides are the drugs of broad antimicrobial spectrum that


includes some protozoa. For complex treatment of which protozoal
infection is it used?
A. Tuberculosis
B. Malaria*
C. Amebiasis
D. Otitis
E. Bronchitis.

525. Why procaine is not compatible with sulfonamides?


A. It binds with sulfonamide
B. It forms paraaminobenzoic acid when metabolized.*
C. It inhibits bacterial cell wall synthesis
246
D. It interferes with sulfonamide absorption
E. It interfere with sulfonamide distribution.

526. Synthetic antimicrobial drug of quinolones with moderate activity


against mycobacteria tuberculosis was recently included in second-line
antitubercular drugs. Point this drug.
A. Furozalidone
B. Sulfadimetoxine
C. Biseptol
D. Isoniazid
E. Ciprofloxacin*.

Antibiotics I

527. A 60-year-old patient was admitted to the surgical department


because of infection caused by blue pus bacillus (Pseudomonas
aeruginosa) which is sensitive to penicillin antibiotics. Indicate which of
the given penicillins has marked activity to the Pseudomonas aeruginosa?
A. Methicillin
B. Phenoxymethylpenicillin
C. Carbenicillin disodium*
D. Oxacillin
E. Benzylpeniciilin.

528. A 50-year-old patient with typhoid fever was treated with


Levomycetin, next day his condition became worse, temperature rose to
39,6°C. What caused the complication?
A. The effect of endotoxin agent*
B. Reinfection
C. Secondary infection addition
D. Irresponsiveness of an agent to the levomycetin
E. Allergic reaction.

529. A patient suffers from severe postoperative pseudomonadous


infection. What of the following antibiotics should be administered in this
case?
A. Erythromycin
B. Doxycycline
C. Cephazolin
D. Benzylpenicillin
E. Amicacin sulfate*.

247
530. Car driver has got into accident and was admited to emergancy
department. Surgical invasion in abdominal cavity was successfuly
performed, but postoperative period was complicated by peritonitis.
Bacterial analysis revealed Pseudomonas aeruginosa. For treatment of
patient you may propose any antibiotic leasted below with axception
A. Azlocillin
B. Genamycine
C. Cefotaxim
D. Imipenem (Tienam)
E. Tetracycline*.

531. A 6-year boy was admitted to hospital with pneumonia. Treatment


with amoxycycline was not effective. Bacterial analysis revealed
Micoplasma pneumoniae. Choose the most suitable drug for treatment of
this child.
A. Tetracycline
B. Azithromycine*
C. Bicillin 5
D. Nystatin
E. Oxacillin.

532. A 20 years old man with gonorrhea was treated with penicillin G in
combination with probenecid. His state was improved, but 1 week later
the patient was still complaining of a persistent urethral discharge and
pain on urination. Laboratory test reveals chlamidia in discharge. Which
of the listed drugs effective both in gonorrhea and chlamidial infection
must be used for further treatment
A. Amoxicillin
B. Doxycycline*
C. Oxacillincin
D. Gentamycin
E. Streptomycin.

533. Patient with tuberculosis was on the complex treatment including


antibiotics. At the end of first course doctor noticed significant
decrease in patient hearing. Which of the following drugs can cause such
side effect?
A. Isoniazid
B. Rifampicin
C. Streptomycin*
D. Paraaminosalicylic acid
E. Pirazinamide .

248
534. Patient, with severe pneumonia was treated with the III generation
cephalosporin. Doctor warned him about the danger of alcohol usage
while treatment. What the drug was used for treatment of patient?
A. Carbenicilin
B. Cefotaxim*
C. Cefalexin
D. Gentamycin
E. Doxycycline.

535. Patient was admitted to the infection unit with diagnosis of bacterial
dysentery. On laboratory studies it was revealed that causative element is
sensitive to the many antimicrobial medicines, but patient has anemia.
What medicine is contra-indicated to the patient?
A. Levomycetin*
B. Phthalazol
C. Enteroseptol
D. Furazolidone
E. Ampicillin.

536. Patient with pneumonia, prone to use alcohol, was treated with
antibiotic of wide spectrum. On the third day of treatment after alcohol
usage severe antabus-like reaction occurs. What was the group of
antibiotic used for treatment of patient?
A. Biosynthetic penicillin
B. Aminoglycoside
C. 3rd generation cephalosporin*
D. Penicilinse resistant penicillin
E. Tetracycline.

537. Patient condition after long term atimicrobial treatment was


complicated by pseudomembranous enterocolitis. For it treatment
vancomycin was prescribed. What is the mechanism of its antimicrobial
effect?
A. Disturbances of structure and function of cell membrane
B. Inhibition of cell wall synthesis*
C. Inhibition of protein synthesis
D. Inhibition of nucleic acid synthesis
E. Inhibition of folic acid metabolism.

538. Patient was admitted to the infection unit with diagnosis of bacterial
dysentery. On laboratory studies it was revealed that causative element is
sensitive to the many antimicrobial medicines, but patient has anemia.
What antibiotic can be recommended for this patient?
A. Levomycetin
B. Amoxicillin*
249
C. Erythromycin
D. Cotrimoxazol
E. Oxacillin.

539. A 5-year-old child has been diagnosed with acute right distal
pneumonia. Sputum inoculation revealed that the causative agent is
resistant to penicillin, but it is senstive to macrolides. What drug should
be prescribed?
A. Azithromycin*
B. Tetracycline
C. Gentamycin
D. Streptomycin
E. Ampicillin .

540. A 19 year old woman suffers from primary syphilis. Doctor


administered her complex therapy that includes benzylpenicillin sodium
salt. What is the mechanism of action of this drug?
A. It blocks RNA synthesis
B. It blocks synthesis of cytoplasm proteins
C. It blocks thiol enzymes
D. It blocks synthesis of peptidoglycan of microbal wall*
E. It blocks DNA synthesis .

541. A patient underwent appendectomy. In the postoperative period he


has been taking an antibiotic. The patient complains about hearing
impairment and vestibular disorders. What group of antibiotics has such
side-effects?
A. Cephalosporins
B. Penicillins
C. Tetracyclines
D. Macrolides
E. Aminoglycosides*.

542. A patient with bacterial pneumonia was prescribed benzylpenicillin.


What is the mechanism of its antibacterial effect?
A. Abnormal permeability of cytoplasmic membrane
B. Inhibition of intracellular protein synthesis
C. Inhibition of synthesis of microbial cell wall*
D. Inhibition of SH-groups of microorganism enzymes
E. Antagonism with p-amino-benzoic acid .

543. A patient with gastric peptic ulcer was on complex treatment that
includes antibiotic of macrolide group. Point out this antibiotic.
A. Clarithromycin*
B. Tetracycline
250
C. Gentamycin
D. Streptomycin
E. Ampicillin .

544. Antibiotic used for the treatment of typhoid fever is:


A. Oxacillin
B. Erythromycin
C. Cephalexin
D. Benzylpenicillin
E. Levomicetin*.

545. Patient has primary syphilis. What is the most effective antibiotic in
this case?
A. Penicillin*
B. Nystatin
C. Amphotericin
D. Streptomycin
E. Kanamycin.

546. Why tetracycline is not recommended in pregnant women?


A. Due to ototoxicity
B. Due to the depressing effect on the respiration of the fruit
C. Due to the ability of the drug to increase uterine tone
D. Due to the teratogenic effect of the drug*
E. Due to the ability to cause anemia in pregnant.

547. In the patient, who has long used tetracycline, there was mucosal
candidiasis. What medication should be appointed to treat it?
A. Nystatin*
B. Nizoral
C. Griseofulvin
D. Furadonin
E. Nitrofungin.

548. Mother of 2 years old child addressed dentist with tooth


abnormality in her child. On child examination the destruction of the
incisors, yellow enamel, the brown rim on the necks of the teeth ware
revealed. Mother informed doctor that she had infection and used
antibiotics. Which of the drugs has a strong teratogenic effect, disrupting
the development of teeth?
A. Polymyxin
B. Erythromycin
C. Tetracycline*
D. Cefrenol
E. Ampiox.
251
549. The patient of 42 years old for the treatment of bacterial pneumonia
was appointed ampicillin. Specify the mechanism of bactericidal action of
the drug
A. Inhibition of intracellular protein synthesis
B. Inhibition of the synthesis of the cell wall of the microorganism*
C. Violation of the permeability of the cytoplasmic membrane
D. Inhibition of SH - groups of enzymes of microorganisms
E. Antagonism with paraaminobenzoic acid.

550. Infectious patient is sensitized to benzylpenicillin. Which of the


following antibiotics would be the safest in this case?
A. Amoxicillin
B. Ampicillin
C. Erythromycin*
D. Oxacillin
E. Bicillin 1.

551. A 6 year child was admitted to the hospital with diagnosis of


candidiasis caused by Candida albicans, the mucous membrane of cheeks,
palate and tongue was cowered by fur of white and yellow color. Which
of the drugs can be used to treat the child?
A. Gentamicin
B. Ketoconazole*
C. Tetracycline
D. Cefran
E. Penicillin.

552. A patient with impaired hearing has severe bacillary infection.


Which group of antibiotics is contraindicated in this case?
A. Aminoglycosides*
B. Penicillins
C. Cephalosporins
D. Tetracyclines
E. Rifamycins.

553. A patient diagnosed with purulent pleurisy caused by penicillin


resistant staphylococcus. What preparation of penicillin resistant to beta-
lactamase may be used for the treatment of this patient?
A. Ampicillin
B. Benzylpenicillin
C. Phenoxymethylpenicillin
D. Augmentin*
E. Carbenicillin.

252
554. Use of which broad-spectrum antibiotic is contraindicated in liver
disease?
A. Polymyxin
B. Lincomycin
C. Tetracycline*
D. Oxacillin
E. Penicillin G.

555. Patient with staphylococcal sepsis was treated with


benzylpenicillin. Therapy was ineffective. What preparation of penicillin
group should be prescribed for the patient to continue treatment?
A. Oxacillin*
B. Polymyxin
C. Erythromycin
D. Phtalazol
E. Aztreonam.

556. Child of 5 years with a staphylococcus infection was assigned


penicillin antibiotic without testing pathogen susceptibility to drugs of this
group. Which drug of penicillin group is resistant to beta-lactamase and
effectively influences on the penicillin-resistant staphylococci?
A. Carbenicillin
B. Ampicillin
C. Bicillin-5
D. Phenoxymethylpenicillin
E. Oxacillin*.

Antituberculous drugs, antiviral, antispirochetous drugs.


Antimycotic agents

557. A 35-year-old man under the treatment for pulmonary tuberculosis


has acute-onset of right big toe pain, swelling, and low-grade fever. The
gouty arthritis was diagnosed and high serum uric acid level was found.
Which of the following antituberculosis drugs are known for causing high
uric acid levels?
A. Rifampicin
B. Aminosalicylic acid
C. Thiacetazone
D. Pyrazinamide*
E. Cycloserine.

558. A patient was diagnosed with active focal pulmonary tuberculosis.


What drug should be prescribed in the first place?
253
A. Ethionamide
B. Isoniazid*
C. Ethoxide
D. Sulfalen
E. Cyclocerine.

559. A patient suffers from pulmonary tuberculosis. During treatment


neuritis of visual nerve occurred. What drug has caused this side effect?
A. Isoniazid
B. Ethambutol*
C. Kanamycin
D. Rifampicin
E. Streptomycin .

560. Patient suffering from tuberculosis was treated with isoniazid. In


the course of treatment peripheral neuritis was developed. What is the
mechanism of this side effect?
A. Direct toxic effect of drug on peripheral nerves
B. Inhibition of myoneural junctions
C. Interference with pyridoxine metabolism*
D. Interference with folic acid synthesis
E. Inhibition of sodium channels.

561. Patient with tuberculosis was on the complex treatment including


antibiotics. At the end of first course doctor noticed significant decrease in
patient hearing. Which of the following drugs can cause such side effect?
A. Isoniazid
B. Rifampicin
C. Streptomycin*
D. Paraaminosalicylic acid
E. Pirazinamide .

562. One of the problems in the treatment of tuberculosis is caused by


existence of metabolically dormant forms of mycobacterium (persister).
Which of the drugs is effective against all subpopulations of Mycobacteria
tuberculosis and causes sterilizing effect?
A. Streptomycin
B. Isoniazid
C. Kanamycin
D. Rifampicin*
E. Paraaminosalicylic acid.

563. The patient suffering from tuberculosis was treated with


combination of antituberculous drugs, including isoniazid. Which drug
should be added to prevent neurological side effects of isoniazid?
254
A. Streptomycin
B. PASA
C. Rifampicin
D. Pyridoxine*
E. Ascorbic acid.

564. A patient has herpetic rash. What medication should be


administered?
A. Acyclovir*
B. Gentamycin
C. Clotrimazole
D. Benzylpenicillin sodium salt
E. Biseptol.

565. Point out the antiviral agent used in influenza.


A. Oseltamivir.*
B. Streptomycine
C. Gentamycin
D. Clotrimazole
E. Pyridoxine.

566. A patient with lung tuberculosis is treated with a drug that has a
wide antimicrobial spectrum; mycobacteria of tuberculosis are highly
sensitive to it. Its effect is realized by depression of bacterial RNA
synthesis. Drug is staining body liquids (urine, sputum, tears) in red color.
What the drug is used?
A. Streptomycin
B. Isoniazid
C. Ethambutol
D. Rifampicin*
E. Paraaminosalicylic acid.

567. Treatment of tuberculosis is realized by means of combination


chemotherapy, including agents with different mechanisms of action.
Which of the antituberculosis drugs inhibit the transcription of DNA into
RNA of mycobacteria?
A. Rifampicin*
B. Isoniazid
C. Streptomycin
D. Ethionamide
E. PAS.

568. The patient went to a doctor complaining of red staining of urine


and tear. From his case history it is known that he is treated on pulmonary

255
tuberculosis. Which of anti-TB drugs is responsible for this
phenomenon?
A. Isoniazid
B. Ethionamide
C. Ethambutol
D. Rifampicin*
E. Streptomycin .

569. How do you explain the fact that the in treatment of tuberculosis
with isoniazid dose is adjusted individually, with mandatory measurement
of its concentration in the urine after the first drug?
A. Genetically determined differences in rate of drug acetylating*
B. Occurrence of hyperglycemia as a side effect
C. The development of renal failure
D. Irritant action of the drug
E. Development of hemolytic anemia.

570. TB patient after prolonged treatment with effective anti-TB drug


called attention to breast enlargement. On this occasion he went to a
doctor, who confirmed gynecomastia. Identify the drug, which he used.
A. Cycloserine
B. Ethambutol
C. Isoniazid*
D. Rifampicin
E. Streptomycin sulfate .

571. As a result of prolonged use of broad spectrum antibiotics intestinal


candidamycosis was developed. What should be prescribed to treat it?
A. Interferon
B. Remantadin
C. Itraconazole*
D. Rifampicin
E. Gramicidin.

572. A patient with a diagnosis of AIDS was admitted to the infectious


disease hospital. To improve the immune status of patient, stabilizing
body weight and improwement of his condition, antiviral agent - an
antagonist of thymidine, which blocks the DNA polymerase of HIV was
prescribed. Select the designated agent.
A. Azidothymidine*
B. Acyclovir
C. Interferon
D. Rimantadine
E. Midantan.

256
573. Patients with herpes doctor prescribed an antiviral drug, which the
main mechanism of action is inhibition of viral DNA polymerase, and the
ability to be integrated instead of deoxyguanosine into viral DNA. Name
the drug.
A. Interferon
B. Azidothymidine
C. Acyclovir*
D. Midantan
E. Ethambutol.

574. A 20 years old patient with primary syphilis is receiving combined


therapy, which includes the sodium salt of benzylpenicillin. What is the
mechanism of action of this drug?
A. Blockade of DNA synthesis
B. RNA synthesis blockade
C. Blockade of the SH- groups of enzymes
D. Blockade of the synthesis murein of cell wall of microorganisms*
E. Blockade of protein synthesis in cytoplasm.

575. A patient addressed doctor with complaints on the damage of skin


between the fingers of foot, exudation and itching. Doctor prescribed
paste containing antifungal agent of wide spectrum. Point out this drug
A. Nystatin
B. Clotrimazole*
C. Streptomycin
D. Co-trimoxazol
E. Levorin.

576. This drug is used for treatment of tuberculosis, rate of its


inactivation in the liver is genetically determined and is different in
different persons. Point out this drug
A. Acyclovir
B. Azidothymidine
C. Ciprofloxacin
D. Isoniazid*
E. Rifampicin.

Antiprotozoal drugs

257
577. To prevent of 4-day malaria a 42-year-old patient was prescribed
primaquine. On the 3-rd day from the begin of treatment there appeared
stomach and heart pains, dyspepsia, general cyanosis, hemoglobinuria.
What is the cause of side effects of the preparation?
A. Drug potentiation by other preparations
B. Cumulation of the preparation
C. Delayed urinary excretion of the preparation
D. Decreased activity of microsomal liver enzymes
E. Genetic insufficiency of glucose 6-phosphate dehydrogenase*.

578. A 52-year-old patient has the following diagnosis: systemic


amebiasis with involvement of intestines, liver, lungs. What drug should
be prescribed?
A. Enteroseptol
B. Tetracycline
C. Quiniofone
D. Quingamine
E. Metronidasol*.

579. A patient consulted a doctor about bowels disfunction. The doctor


established symptoms of duodenitis and enteritis. Laboratory examination
helped to make the following diagnosis: lambliosis. What medication
should be administered?
A. Tetracycline
B. Monomycin
C. Metronidazole*
D. Erythromycin
E. Chingamin.

580. A patient consulted a stomatologist about purulent inflammation of


his gums. What drug will be the most effective if it is suspected that a
causative agent is an anaerobe?
A. Oxacillin sodium
B. Gentamicin
C. Nitroxoline
D. Co-trimoxazole
E. Metronidazole*.

581. A patient ill with amebiasis was prescribed a certain drug. The use
of alcohol together with this drug is contra-indicated because the drug
inhibits metabolism of ethyl alcohol. What drug is it?
A. Clonidine
B. Metronidazole*
C. Reserpine
D. Diazepam
258
E. Aminazine.

582. Before the doctor’s business trip to foreign country he was


proposed histoschizontocidal antimalarial drug for the personal prevention
of malaria. Which drug has got a specialist?
A. Mefloquine
B. Chloroquine
C. Quinine
D. Doxycycline
E. Primaquine*.

583. The patient with lovered immunity was examined concerning


helminthiasis. Laboratory studies have found ascariasis. Which drug
should be prescribed?
A. Niclosamide
B. Piperazine
C. Furazolidone
D. Gentamicin
E. Levamisole*.

584. The patient addresed her gynecologist complaining of heavy


vaginal discharge with an unpleasant odor. After the smears bacterioscopy
trichomoniasis was diagnosed. What the drug is indicated in this case?
A. Sulfadimezin
B. Chloroquine
C. Pyrimethamine
D. Metronidazole*
E. Monomecinum.

585. The patient has a mixed helminthic invasion: intestinal ascariasis


and liver trematodes. What anthelminthic drugis the most advisable for
the treatment of this patient?
A. Mebendazole*
B. Pyrantel B.
C. Chloxylum
D. Piperazine adipate
E. Levamisole.

586. The drug has a devastating effect on the erythrocytic forms of


malaria parasites, dysenteric amoeba. It is used to treat and prevent
malaria, the treatment of amoebiasis and connective tissue diseases.
Identify the drug.
A. Emetine hydrochloride
B. Chloroquine*
C. Tetracycline
259
D. Erythromycin
E. Quinine.

587. The patient addressed doctor complaining of epigastric discomfort,


nausea, loss of appetite. The study of duodenal content revealed Giardia
resistant to metronidazole. Which drug should be appointed?
A. Rifampicin
B. Chloroquine*
C. Metronidazole
D. Isoniazid
E. Acyclovir .

588. Antiprotozoal agent of wide spectrum was included in complex


treatment of patient with gastric peptic ulcer. Doctor warned patient about
prohibition of alcohol usage during treatment with this drug. What the
drug was used?
A. Chloramphenicol
B. Metronidazole*
C. Cotrimoxazole
D. Chloroquine
E. Praziquantel.

589. Patient suffering from severe malaria caused by plasmodium


falciparum resistant to chloroquine was treated with the oldest
antimalarial drug, obtained from the bark of cinchona tree. Point out this
drug.
A. Primaquine
B. Proguanil
C. Pyrimethamine
D. Quinine*
E. Emetine.

590. Point out the drug of first choice for treatment of patient suffering
from kala-azar (visceral leishmaniasis)
A. Metronidazole
B. Sodium stibogluconate*
C. Tetracycline
D. Pyrimethamine
E. Nystatin.

591. For treatment of neck phlegmon caused by anaerobic infection


antimicrobial drug of imidasole group was included. Point out this drug
A. Norfloxacin
B. Clindamycin
C. Metronidazole*
260
D. Tetracycline
E. Ampicillin.

592. A 30-year-old patient complains about having abdominal pain and


diarrhea for five days body temperature rise up to 37,5oC along with
chills. The day before a patient had been in a forest and drunk from an
open water reservoir. Laboratory analyses enabled to make the following
diagnosis: amebic dysentery. What is the drug of choice for its treatment?
A. Metronidazole*
B. Furazolidonum
C. Levomycetin
D. Phthalazol
E. Emetine hydrochloride .

593. Amebiasis of hepatic localization was diagnosed in patient. Choose


the drug for treatment of patient.
A. Norfloxacin
B. Clindamycin
C. Chloroquine*
D. Tetracycline
E. Ampicillin.

594. The drug with histoschizontocydal and gametocydal action was


used for causal prophylaxis of malaria. Point out this drug.
A. Metronidasole
B. Primaquine*
C. Chloroquine
D. Tetracycline
E. Emetine hydrochloride .

595. Point out the antimalarial drug that exert erythrocytic schizontocidal
effect by changing pH in parasitic vesicles and prevention of haeme
transformation to hemozoin.
A. Primaquine
B. Chloroquine*
C. Metronidazole
D. Sulfadoxine
E. Pyrimethamine.

Anticancer drugs. Preparations of acids, bases and salts. Basic


principles of acute poisoning treatment

261
596. A patient who was previously ill with mastectomy as a result of
breast cancer was prescribed radiation therapy. What vitamin preparation
has marked radioprotective action caused by antioxidant activity?
A. Ergocalciferol
B. Tocopherol acetate*
C. Thiamine chloride
D. Ribof1avin
E. Folic acid.

597. In order to accelerate healing of a radiation ulcer a vitamin drug


was administered. What drug is it?
A. Methyluracil
B. Retinol acetate*
C. Levamisole
D. Prednisolone
E. Retabolil.

598. An oncological patient was prescribed methotrexate. With the lapse


of time target cells of the tumour lost susceptibility to this drug. There is
change of gene expression of the folowing enzyme:
A. Thiaminase
B. Deaminase
C. Dehydrofolate reductase*
D. Folate decarboxylase
E. Folate oxidase.

599. Patient after surgery for breast cancer chemotherapy with


antiestrogen agent was prescribed. Which of the following drugs belongs
to antiestrogen with anticancer activity?
A. Tamoxifen*
B. Cyclophosphamide
C. Chlorbutin
D. Cisplatin
E. Fosfestrol.

600. What is the basic mechanism of anticancer action of ethyleneimine


derivative?
A. Alkylation of RNA and DNA of tumor cells*
B. Inhibition of cell division in metaphase
C. Formation of stable complexes with DNA of tumor cells
D. Competitive inhibition of nucleic acid metabolism in tumors
E. Cytorecptors blockade on the cell membrane.

262
601. In the intensive care unit a child was enrolled with pronounced
signs of acidosis, to relieve his condition immediate drip of infusion was
started. Which of the following drugs should be used in acidosis?
A. Potassium Chloride
B. Sodium Chloride
C. Sodium bicarbonate*
D. Glucose
E. Magnesium sulfate.

602. Patients with chronic heart failure was using digoxin. To increase
the result of treatment additional drug was included in patient therapy. It
resulted in the development of symptoms of intoxication. Which drug can
cause increase in cardiac glycoside toxicity?
A. Potassium chloride
B. Magnesium chloride
C. Asparkam
D. Glucose
E. Calcium chloride*.

603. A patient with acute poisoning was admitted to emergency


department. It was established that poisonous substance was excreted by
kidney. Choose the best diuretic for forced diuresis.
A. Acetazolamide
B. Amiloride
C. Spironolactone
D. Furosemide*
E. Hydrochlorothiazide.

604. Choose the most beneficial agent for stomach wash in patient
poisoned with alkaloid
A. Physiological solution
B. Chlorhexidine
C. Sodium bicarbonate
D. Potassium permanganate*
E. 5% solution of glucose.

605. Point out the antidote used in poisoning with salts of heavy metals
A. Penicillamine
B. Unithiolum*
C. Acetylcysteine
D. Magnesium sulfate
E. Atropine.

606. A patient in comatose state was admitted to emergency department.


His respiration was abnormal (Cheyn-Stoke’s respiration). Poisoning with
263
opioid analgesic was diagnosed. After intravenous administration of
antidote respiration was restored. What agent was used as antidote?
A. Pralidoxime
B. Naloxone*
C. Ethanol
D. Acetazolamide
E. Unithiolum.
607. A patient with myocardium infarction was treated with heparin. On
6th day of treatment nasal bleeding occurred. Overdose of heparin was
diagnosed. Point out the antidote of heparin
A. Acetylcysteine
B. Protamine sulfate*
C. Magnesium sulfate
D. Unithiolum
E. Deferoxamine.

608. Patient suffering from cancer of skin is treated with anticancer


antibiotic. Point out this drug
A. Neomycin
B. Danazol
C. Mercaptopurine
D. Phtoruracil
E. Bleomycin*.
609. Radio protector was given to a worker of a nuclear power station.
What mechanism is considered to be responsible for radioprotection?
A. Increasing of respiration
B. Inhibiton of free radicals formation*
C. Activation of oxidation reactions
D. Prevention of tissue's hypoxia
E. Increasing of tissue blood supply.
610. Patient suffering from acute leucosis was treated with agent that is
competitive antagonist of folic acid. Point out this agent
A. Phtoruracil
B. Methotrexate*
C. Mercaptopurine
D. Phtorafur
E. Cytarabine.

611. Patient with gastric cancer is treated with drug that interferes with
synthesis of nucleic acids due to its resemblance with pirimidine. Point
out this drug
A. Mercaptopurine
B. Methotrexate
264
C. Phluorouracil (Phtoruracil)*
D. Neomycin
E. Bleomycin.

612. Highly dehydrated patient with cholera was admitted to specialized


department. Which agent is necessary to use to normalize the volume of
circulating blood?
A. Solution KCl 4%
B. Solution of calcium chloride 10%
C. Solution of glucose 40%
D. Solution calcium gluconate
E. Solution of NaCl 0,9%*.
613. A patient working at a chemical plant was admitted to the
toxicological department with mercury poisoning. What medicine should
be used?
A. Isonitrozin
B. Naloxone
C. Enterosorbent
D. Unithiol*
E. Activated carbon.

265
Дополнения базы тестов 2015-2016 гг.

614. After surgical removal of the thyroid gland for patients assigned L-
thyroxine. What type of pharmacotherapy achieved in such treatment?
A. Replacement*

B. Recovery

C. Etiological

D. Functional

E. Symptomatic

615. In the treatment of patients with antibiotics frequently occurs an


allergy. What is the mechanism lies at the basis of this complication?

A. Immune*
B. Enzyme
C. Metabolic
D. Physico-chemical
E. Receptor

616. Tobaco smoking - a consequence of nicotine addiction.


From what reaction begins its development?

A. Euphoria*
B. Passion
C. Addiction
D. Abstinence
E. Synergy

617. Emergency care in drugs overdose including primarily maintenance


of specific antidotes. What type of drug interaction provides their application?

A. Functional antagonism*
B. Summation
C. Incompatibility
D. Drug synergy
E. Compatibility

266
618. Long-term use of chlorpromazine in patients with schizophrenia,
along with the restoration of mind, led to the development of Parkinson's disease
symptoms. For what type of drug action should be attributed these
manifestations?

A. Complication*
B. Allergic reaction
C. The main medicinal effect
D. Sensitization
E. Addiction

619. The young scientist after working in the archive library there was
allergic to book dust. What H1 histaminoblokator without long-acting hypnotic
effect should apply?

A. Loratadine*
B. Depot medroxyprogesterone acetate
C. Metylmetyonynsulfonyya chloride
D. Enoxaparin calcium
E. Ropivakayin

620. For the treatment of arthritis celecoxib doctor has ordered. What
group of NSAIDs owned this vehicle?

A. Koksyby*
B. Adaptogens
C. Oxidizing
D.Trombolytic
E. Complexons

621. The 3-year old child with a fever over chickenpox. What antipyretic
medication is allowed in this case to avoid the development of Reye syndrome?

A. Paracetamol*
B. Menthol
C. Castor Oil
D. Ringer-Locke solution
E. Fish Oil

267
622. In patients after long-term use of aspirin as antiplatelet agents was
dyspnea. How is this difficulty?

A. Apirin asthma*
B. Euphoria
C. Functional cumulation
D. Dysbacteriosis
E. Idiosyncrasy

623. After myocardial infarction, the doctor scheduled the patient anty-
atherosclerotic drug that blocks the synthesis of cholesterol in liver and reduces
the risk of ischemic complications. Define a drug?

A. Atorvastatin*
B. Lobeline
C. Clonidine
D. Reserpine
E. Xylometazoline

624. In the intensive care unit delivered a man with a diagnosis of


cardiogenic shock. The introduction of any drug would be appropriate in this
situation

A. Dobutamine*
B. Novocaine
C. Lovastatin
D. Phenytoin
E. Ranitidine

625. Determine the drug that the doctor prescribed for the treatment of a
patient who has history of angina, tachycardia and asthma?

A. Metoprolol*
B. Haloperidol
C. Bromocriptine
D. Tannin
E. Metronidazole

626. Select the drug has a hypotensive effect, reduces blood volume and
displays ions of sodium, potassium and magnesium from the body.
A. Furosemide*
B. Caffeine-sodium benzoate
C. Bemegride
D. Pantotsid
E. Magnesium sulfate
268
627. Elderly man suffering from bronchitis, was appointed drug with
mucolytic mucokinetic action. It is known that in addition to the drug
expectorant action promotes the synthesis of endogenous surfactant. What is this
medicines?

A. Ambroxol*
B. Naphthyzinum
C. Omnoponum
D. Tubokurarіn
E. Rofecoxib

628. Patients with cystitis doctor appointed antimicrobial agent with


broad-spectrum fluoroquinolone group, mechanism of action is associated with
blocking the enzyme DNA gyrase? What kind of preparation we talking about?

A. Ciprofloxacinum*
B. Arbіdol
C. Furacillіnum
D. Holenzіmum
E. Tanalbіnum

629. The patient to complex treatment of hypertension was included long-


acting medicine (24 hour), mechanism of action is associated with the blocking
an enzyme that promotes the synthesis of angiotensin II. What is this drug?

A. Lizinopril*
B. Ergocalciferol
C. Mercaptopurine
D. Lavomax
E. Bromhexinum

630. After tooth extraction doctor prescribed to the patient for mouthwash
procedure an antiseptic drug from halogencontaining group. Determinate the
drug?
А. Chlorhexidine *
B. Ampicillin
C. Fraxiparyn
D. Lobeline
E. Benzocaine

631. During stomatolodical procedure under conductive anesthesia in


patient with hypertension, blood pressure increased. Which of the following
drugs should be used to decrease blood pressure?

269
А. Magnesium sulfate *
B. Fizostihmin
C. Nafazolin
D. Haloperidol
E. Tramadol

632. During the root canal filling it’s important to irrigate a canal from
necrotic residual dentin by antiseptic solution. Determine which of the following
agents will contribute to mechanical cleaning.
А. Hydrogen peroxide *
B. Festal
C. Prazosin
D. Amoxiclav
E. Almagel

633. Dentist decided to conduct short-term anesthesia


(neuroleptanalgesia). Which of the psychotropic drugs in it combination?
A. Droperidol *
B. Sulpiride
C. Trioxazyn
D. Reserpine
E. Neuleptil
634. Before stomatological treatment, patient has got the tranquilizer –
diazepam. Identify the mechanism of its anxiolytic action.
А. Benzodiazepine receptor agonist *
B. M-cholinergic receptors agonist
C. β-adrenoceptor blocker
D. Histamine receptor blocker
E. Stimulator of redox processes

635. Anaphylactic shock appeared in patient after injection of local


anesthetic in dental department. Determine the hormonal agent for emergency
medical aid.
А. Prednisolone *
B. Cermion
C. Vancomycin
D. Ribavirin
E. Probucol

636. A patient complained to a doctor about an appearance on the oral


mucosa a gray spots. He suffers from syphilis and takes bismuth preparation
which causes intoxication symptoms. What is the antidote should be used in this
case?
А. Unithiol *
B. Artykain
270
C. Fenoterol
D. Reserpine
E. Sulpiride

637. During the tooth extraction preparation, doctor indicated for patient
an agent that prevents bleeding. Determine this drug.
А. Vikasol *
B. Diakarb
C. Ethonium
D. Analgin
E. Benzohexonium

638. Patient B. 45 years, previously was instal with amalgam seal. A


month later, she complained of unwell feeling and was diagnosed with acute
mercury poisoning. What is the antidote advisable to appoint in this case?
A. Unithiol *
B. Atenolol
C. Enalapril
D.Ceftriaxone
E. Zidovudine
639. For mandibular biopsy procedure doctor used a neuroleptanalgesia.
What is the opioid analgesic that should be used in combination with droperidol
for this goal?
А. Fentanyl *
B. Emetine hydrochloride
C. Depo-medroxyprogesterone acetate
D. Metylmetioninsulfonium chloride
E. Fish Oil

640. Lincosamide antibiotic have been prescribed to a patient with


mandibular osteomyelitis. Determine this agent.
А. Lincomycin *
B. Kanamycin
C. Byseptol
D. Fuzydin sodium
E. Vancomycin

641. Wide spectrum antibiotic have been prescribed by the dentist for
odontogenic sepsis prevention that well-penetrates into bone tissue. Determine
this drug.
A. Doxycycline *
B. Nystatin
C. Tubocurarinum
D. Ethambutol
E. Pyrazinamide
271
642. Before the tooth extraction procedure dentist have done an anesthesia
by articaine and epinephrine combined agent. What did the dentist used?
А. Ultracaine *
B. Novocaine
C. Cocaine
D. Benzocaine
E. Codeine

643. Facial nerve paresis appeared in patient during a stomatological


procedure. Which of the anticholinestherase agents should be prescribed to the
patient?
A. Proserinum *
B. Fluoxamine
C. Sulmazole
D. Atenolole
E. Dipyridamole

644. For the xerostomia treatment have been prescribed a cholinestherase


drug that is widly use in ophtolmological practice in glaucoma. What is the drug
above?
A. Pilocarpine hydrochloride *
B. Klopamid
C. Estrone
D. Bronhofit
E. Suprastin
645. A patient diagnosed with bacterial stomatitis, caused by anaerobic
microflora. What synthetic antimicrobial agent should be given?
А. Metronidazol *
B. Carbocholine
C. Terazosin
D. Cyklodole
E Amphetamine
646. During tooth extraction the patient developed collapse. What drug
should be used to increase blood pressure?
A. Phenylephrine *
B. Arduan
C. Halothane
D. Apisarthron
E. Enflurane

647. A patient developed acute bronchospasm during dental examination.


Which drug should be administered urgently?
272
A. Epinephrine *
B. Chingaminum
C. Lovastatin
D. Procaine
E. Phenythoin

648. A patient was prescribed fluconazole after dental examination. What


disease in the oral cavity is an indication for use of fluconazole?
А. Candidomycose
В. Dental caries
С. Periodontitis
D. Bacterial inflammation
Е. Allergic gingivitis

649. The complex jaw joint tuberculosis therapy insludes isoniazid and
vitamin preparation which prevents its side effects (neuritis). What vitamin is
used for this purpose?
А. Pyridoxine*
В. Papaverine hydrochloride
С. Plathyphylline
D. Piracetam
Е. Trimeperidine

650. The examination of dental patient revealed stomatitis with bleeding


gums and pain during chewing. What vitamin preparations should be assigned to
him in the first place?
А. Ascorbic acid*
В. Aceclidine
С. Ambroxolum
D. Acetylsalicylic acid
Е. Аmitriptyline

651. After poliomyelitis a patient was appointed agent for recovery of skeletal
muscle tone. Determine the drug:
A. Neostigmine *
B. Nyzatidin
C. Sukralfat
D. Estrone
E. Hutalaks

652. A man who is working at the factory got the minor injuries of his
hands. What antiseptic drug from the halogens group should be used for
disinfecting hands scratches?
А. Solution of iodine*
B. Dibazol
273
C. Kanamycin
D. Dipyridamole
E. Pirenzepine

653. For the treatment of hypertensive patient was appointed angiotensin


II receptor antagonist. Which of the following drugs was prescribed?
А. Losartan*
B. Nitrazepam
C. Cyclodolum
D. Erythromycin
E. Galantamine.

654. For the treatment stroke patient doctor prescribed nootropic agent.
Point out this drug.
А. Piracetam*
B. Askorutin
C. Sermion
D. Vecuronium
E. Itipromin

655. A patient has chronic fatigue due to hard physical and mental work.
The doctor appointed psychomotor stimulant. Select this agent.
А.Caffeine*
B. Nicergoline
C. Talinololum
D. Ipratropium bromide
E. Analginum

656. A patient with impaired heart rate for prevention of attack of angina
pectoris was prescribed Ca channel blockers. Determine this drug.
А. Amiodarone*
B. Nakom
C. Sodium bromide
D. Tavegil
E. Logest

657. A patient was diagnosed gastric peptic ulcer. Indicate which drug
will reduce the function of gastric glands by blockade of H2-histamine
receptors?
A. Famotidine*
B. Syrepar
C. Trymetazyn
D. Analginum
E. Dyzopiramid

274
658. Patient with peptic ulcer was treated with omeprazole. The acidity of
gastric juice decreased significantly. What is the mechanism of this drug action?
A. Inhibition of Н+К+-АTPase*
B. Blockade of beta- adrenoceptors
C. Blockade of cyclooxygenase
D. Blockade of monoamineoxidase
E. Activation of Н-cholinoceptors

659. Aged patient is suffering from chronic constipation. Prescribe the


drug for treatment this condition.
А. Bisacodil*
В. Isoflurane
С. Leykohen
Д. Butorphanol
Е. Probucol

660. Patient is suffering from diarrhea. Prescribe the drug which acted by
binding with opioid receptors of intestine.
А. Loperamide*
В. Reaferon
С. Dihydroergotamine
Д. Amylnitrite
Е. Bismoverol

661. After gastric resection the megaloblastic anemia developed in


patient. What drug should be applied in this case?
А. Cyanocobalamin*
В. Pentoxifylline
С. Euphylline
D. Primaquine
Е. Phenobarbital

662. A pregnant woman with low generic activities was admitted to the
hospital. What preparation will be the most beneficial for stimulating of labor
activity in this case?
А. Oxytocin*
В. Aetacridini lactatis
С. Bupivacaine
D. Ephedrine
Е. Miconazole

663. A woman was admitted to the hospital with biliary colic. Which drug
from group of opioid analgesics with spasmolytic activity should be used?
А. Promedol*
B. Abciximab
275
C. Escitalopram
D. Oxandrolone
E. Zanamiviр

664. Patient after strok for prevention of thrombosis was treated with
antiplatelet agent from the group of nonselective cyclooxygenase inhibitors.
Point out this drug:
А. Acetylsalicylic acid *
B. Groprynozyn
C. Bupivacaine
D. Venlafaxine
E. Amfoglukamin

665. For the treatment of pregnant woman with tonsillitis doctor


prescribed antibiotic of narrow spectrum. Point out this antibiotic.

А. Penicillin sodium*
B. Laevomycetin
С. Тetracycline
D. Furacilinum
Е. Trimethoprim

666. A 5-year-old child was admitted to the hospital with pneumonia


caused by mycoplasma. Which antibiotic from macrolides group should be
assigned for its treatment?
А. Аzithromycin*
В. Tetracycline
С. Phthalazolum
D. Bactrim
E. Furacilinum

667. After applying eye drops in the patient with iris inflammation was mydriasis
and paralysis of accommodation. Which drug was prescribed by a doctor?
A. Аtropine *
B. Ferkoven
C. Аtenolol
D. Niridazol
E. Primaquine
668. A patient appealed to the doctor complaining of muscular rigidity, stiffness
of movement, permanent hand tremor. Diagnosis - Parkinson's disease. What
preparation is advisable to appoint a patient?
A. Levodopa *
B. Ferkoven
C. Sinestrol
D. Ketoprofen
276
E. Neomycin

669. A patient with rheumatism was prescribed antimalarial drug.


Determine the drug:
А. Chingaminum *
В. Tubocurarine hydrochloride
С. L-cet
D. Ferrum-lek
Е. Infusum Valerianae

670. A patient with chronic heart failure takes cardiac glycoside.


Determine the drug:
А. Digoxin *
В. Phenobarbitone
С. Heptral
D. Clindamycin
Е. Levamisole

671. A doctor prescribed a patient with acute heart failure digoxin. To


which pharmacological group this drug belongs to?
А. Cardiac glycosides *
В. Neuroleptics
С. Adrenoblokers
D. Antiprotozoal agents
Е. Cholinomimetics

672. According to medical examination a patient with ischemic heart


disease was found hyperlipidemia. A doctor prescribed a patient the drug that
violates the synthesis of cholesterol. Determine the drug:
A. Atorvastatin *
B. Nystatin
C. Pirenzepine
D. Reserpine
E. Xylometazoline

673. A patient suffers from hypertension and angina pectoris. Which of β-


adrenoblockers should be prescribed?
A. Propranolol *
B. Haloperidol
C. Bromocriptin
D. Tanninum
E. Levamisole

277
674. A man developed bronchospasm. What drug should be administered
to relieve bronchospasm?
A. Salbutamol *
B. Atenolol
C. Pindolol
D. Phetanolum
E. Timolol

675. A patient with acute pancreatitis was admitted to the surgical clinic.
Which drug should be administered urgently?
А. Contrycal *
В. Almagel
С. Pirenzepine
D. Norepinephrine
Е. Fraxiparine

676. A patient has impaired visual acuity, especially in the evening. What
vitamin preparation contribute to the restoration of vision?
А. Retinol *
В. Reserpine
С. Reparil
D. Retabolil
Е. Ranitidine

677. A child has bronchitis. The doctor appointed expectorant which


stimulates the synthesis of endogenous surfactant. Determine the drug:
А. Ambroxol *
B. Minizistone
C. Sitagliptin
D. Solutio Ringer-Locke
E. Nifuroxazide

678. A patient with rheumatoid arthritis was assigned non-steroidal anti-


inflammatory agent from selective cyclooxygenase-2 inhibitors. Determine the
drug:
А. Celecoxib *
B. Fluticasone propionate
C. Castor oil
D. Enoxaparin
E. Rosiglitazone

679. A 3-year-old child was diagnosed mixed helminthosis. Which drug


should be prescribed?
А. Mebendazole *
В. Riboflavine
278
С. Fraxiparine
D. Papaverine hydrochloride
Е. Ranitidine

GENERAL MICROBIOLOGY
Morphology
1. The organisms to be identified have a nucleus surrounded by a nuclear membrane.
Genetic material is concentrated predominantly in the chromosomes which consist of DNA strands
and protein molecules. These cells divide mitotically. Identify these organisms:
A. Eukaryotes B. Bacteriophages
C. Prokaryotes D. Viruses E. Bacteria
2. Infectious agents of various ultrastructures can be etiological agents of infectious
diseases. Which of the groups named below HAS NO cellular structure, protein synthesizing, enzyme
and energy systems?
A. Viruses B. Fungi C. Bacteria
D. Protozoa E. Rickettsia
3. Etiological factors for the infectious diseases are often microorganisms with various
ultrastructure. Which of the following microorganism groups relates to the eukaryotes?
A. Protozoa B. Viruses C. Viroids
D. Prions E. Scotobacteria
4. A child is presumably ill with diphtheria. A specimen of affected mucous membrane
of his pharynx was taken for analysis. The smear was stained and microscopic examination revealed
yellow rods with dark blue thickenings on their ends. What structural element of a germ cell was
revealed in the detected microorganisms?
A. Volutin granules B. Plasmids
C. Capsule D. Spores E. Flagella
5. On examination of a 6-year-old child the doctor noticed greyish film on the child’s
tonsils. Microscopy of the smear stained by Neisser method detected there Corynebacterium
diphtheria. What morphologic feature was the most indicative for determining the type of the agent?
A. Fence-like position of the agent’s cells
B. Spores that exceed cells in diameter
C. Localization of the causative agent within macrophages
D. Polar placement of volutin granules
E. Presence of the capsule
6. During the staining of sputum smear of a patient with suspected croupous
pneumonia the following reactives and stainers were used: gential violet solution, Lugol's solution,
960 spiritus and water fuchsin. What method of staining is used in this case?
A. Leffler's B. Gram's C. Ziehl-Neelsen's
D. Neisser's E. Romanovsky's
7. A smear of sputum from the patient with suspected lobar pneumonia was stained
with the use of the following stains and reagents: solution of gentian violet, Lugol’s solution, 96 o
alcohol, watermagenta. What staining method was applied in this case?
A. Gram B. Ziehl-Nielsen C. Romanovsky D. Neisser E. Leffler
8. In a bacteriological laboratory some bacterial smears had to be stained by Gram’s
method. For this purpose the following reagents were prepared: gentian violet, Lugol’s solution,
aqueous fuchsin solution. What other reagent is required?
A. 96% ethanol B. 5% sulfuric acid
C. Methylene blue solution
D. Carbolic fuchsin E. 3%hydrogen peroxide

279
9. When preparing a dental plague smear and staining it according the gram method, a
student during microscopy detected there various violet and pink microorganisms. What structural
component of microorganisms causes different response to stains?
A. Cell wall B. Cytoplasm
C. Cytoplasmic membrane D. Internal periplasmic space
E. Outer membrane
10. A smear of streptobacillus preparation stained by Ozheshko method has been
studied microscopically with oil immersion. What structural feature of the bacteria has been studied?
A. Spores B. Capsule C. Flagella
D. Inclusions E. Structure of cell wall
11. A specimen stained by Ozheshko method contains rod-like microorganisms stained
blue with round terminal components stained red. What are these components called?
A. Spores B. Cilia C. Flagella D. Capsules E. Mesosomas
12. The laboratory for especially dangerous infections conducts microscopic examination
of pathological material from a patient with suspected plague. The sample was stained by Burri-Gins
technique. What property of the causative agent can be identified by this technique?
A. Capsule formation B. Spore formation
C. Acid resistance D. Alkali resistance
E. Presence of volutin granules
13. When a smear is stained by Burry- Gins method a mucous structure that is tightly
bound with the cellular wall of bacteria and has well-defined outer boundaries can be detected. This
element of a bacteria cell is called:
A. Capsule B. Spore C. Filaments
D. Ribosomes E. Episomes
14. Capsuliferous bacteria has been detected during microbiological inspection of crude
drugs. What method of staining has been used to detect capsules?
A. Burri-Gins B. Ziehl–Neelsen
C. Neisser D. Gram E. Aujeszky
15. During microbiological inspection of crude drugs encapsulated bacteria were
revealed. What method was applied for capsule detection?
A. Burry-Gins B. Ziehl-Neelsen
C. Neisser D. Gram E. Ozheshko
16. Microbiological analysis of medicinal raw materials revealed capsular bacteria. What
stain method was used to detect the capsules?
A. Gin’s B. Ziehl-Neelsen’s
C. Neisser’s D. Gram’s E. Ozheshko’s
17. In course of long-term treatment of an infectious patient with penicillin, the
pathogen transformed into the L-form. What changes occur in the pathogen cell in case of L-
transformation?
A. Absence of a cell wall B. Absence of flagella
C. Absence of a capsule D. Absence of a spore
E. Absence of inclusions
19. The cell surface of pathogenic gram-negative bacteria can be covered in villi and cilia.
What function do these structures have?
A. RNA exchange between cells B. Inhibition of complement activity
C. Nutrient transport into the cell D. Resistance to antibody opsonization
E. Bacterial adhesion to the surface of the host cells, conjugation
20. During the skill-building session in microbiology the students need to stain the
prepared and fixed sputum smears obtained from a tuberculosis patient. What staining
technique should be used in this case?
A. Gram B. Giemsa C. Ziehl-Neelsen D. Burry
E. Gins
Special

280
1. Microscopy of a smear obtained from a patient with acute purulent periostitis
revealed gram-positive bacteria arranged in clusters resembling bunch of grapes. What
microorganisms is this morphology typical for?
A. Staphylococci
B. Sarcina
C. Tetracocci
D. Candida fungi
E. Streptococci
2. Analysis of sputum taken from a patient with suspected pneumonia revealed rather
elongated gram-positive diplococci with somewhat pointed opposite ends. What microorganisms
were revealed in the sputum?
A. Streptococcus pneumoniae
B. Staphylococcus aureus
C. Klebsiella pneumoniae
D. Neisseria meningitidis
E. Neisseria gonorrhoeae
3. Analysis of sputum taken from a patient with suspected pneumonia revealed slightly
elongated gram-positive diplococci with tapered opposite ends. What microorganisms were revealed
in the sputum?
A. Stretpococcus pneumoniae
B. Neasseria gonorrhoeae
C. Neisseria meningitidis
D. Staphylococcus aureus
E. Klebsiella pneumoniae
4. A patient has a suspected pneumonia. In his sputum there were revealed
grampositive diplococci, prolonged with the slightly pointed opposite ands. What microorganisms are
revealed in the sputum?
A. Staphylococcus aureus
B. Neisseria gonorrhoeae
C. Neisseria meningitidis
D. Klebsiella pneumoniae
E. Streptococcus pneumonia
5. Analysis of sputum taken from a patient with suspected pneumonia revealed slightly
elongated gram-positive diplococci with tapered opposite ends. What microorganisms were revealed
in the sputum?
A. Streptococcus pneumoniae B. Staphylococcus aureus
C. Klebsiella pneumoniae D. Neisseria meningitidis
E. Neisseria gonorrhoeae
6. Blood of a patient with presumable sepsis was inoculated into sugar broth. There
appeared bottom sediment. Repeated inoculation into blood agar caused growth of small
transparent round colonies surrounded by hemolysis zone. Examination of a smear from the
sediment revealed gram-positive cocci in form of long chains. What microorganisms are present in
blood of this patient?
A. Streptococci
B. Micrococci
C. Staphylococci
D. Tetracocci
E. Sarcina
7. Gramnegative bin-shaped diplococcus inside and outside of leucocytes were
detected on bacteriological examination of the purulent exudates from the cervix of the uterus.
Name the causative agent of purulent inflammation of the cervix of the uterus.
A. Haemophilus vaginalis
B. Chlamidia trachomatis
C. Calymmatobacterium granulomatis

281
D. Neisseria gonorrhoeae

E. Trichomonas vaginalis

8. Microscopic study of discharges from urethra of a patient suffering from acute


urethritis revealed bean-shaped microorganisms up to 1 micrometer in diameter arranged in pairs
and placed inside the leukocytes. What microorganisms are these?
A. Gonococci
B. Meningococci
C. Tetracocci
D. Streptococci
E. Staphylococci
9. While studying blood and mucus samples from the nasopharynx, a bacteriologist
took certain measures to conserve the pathogens in the material. Bacterioscopic study revealed the
presence of gram-negative cocci looking like coffee beans and arranged in pairs or tetrads. Name the
pathogen that was isolated by the bacteriologist:
A. Neisseria meningitidis
B. Staphilococcus aureus
C. Neisseria gonorrhoeae
D. Moraxella lacunata
E. Acinetobacter calcoaceticus
10. A young woman suddenly developed fever up to 39oC accompanied by a strong
headache. Examination revealed marked nuchal rigidity. Spinal puncture was performed. Gram-
stained smear of cerebrospinal fluid contained many neutrophils and Gram-negative diplococci.
What bacteria could be the cause of this disease?
A. Neisseria meningitidis
B. Streptococcus pneumoniae
C. Haemophilus influenza
D. Staphylococcus aureus
E. Pseudomonas aeruginosa
11. Microscopy of a smear taken from the film that appeared on the peptone water 6
hours after seeding and culturing of a fecal sample in a thermostat revealed mobile gram-negative
bacteria curved in form of a comma that didn’t make spores or capsules. What microorganisms were
revealed?
A. Vibrios
B. Spirochetes
C. Clostridia
D. Corynebacteria
E. Spirilla
12. Vomiting matters of a patient suspected of having cholera were delivered to the
bacteriological laboratory. The material was used for preparing a "hanging drop" specimen. What
type of microscopy will be applied for identification of the causative agent by its mobility?
A. Phase-contrast microscopy
B. Electron microscopy
C. Immune and electron microscopy
D. Fluorescence microscopy
E. Immersion microscopy
13. Patient with diarrhoea was admitted to the infection unit. Gramnegative curved rod-
like bacteria were founded on bacterioscopic examination of faecal masses. What is the most likely
disease in this patient?
A. Typhoid fever
B. Cholera
C. Diphtheria
D. Intestinal form of plague
E. Salmonellosis gastroenteritis
282
14. A man is suffering from diarrhea. In summer he spent his vacation in the south at the
sea coast. Bacteria with the following properties were detected in his feces: gram-negative curved
mobile monotrichous bacilli that do not produce spores or capsules. They are undemanding to
nutrient medium but require alkaline reaction (рН 8,5-9,5). Described are the agents of the following
enteric infection:
A. Cholera
B. Shigellosis
C. Typhoid fever
D. Colienteritis
E. Pseudotuberculosis
15. From the feces of a patient with acute gastroenteritis a pure culture of
microorganisms was obtained. The microorganisms are small mobile slightly curved gram-
negative bacilli that within 6 hours grow into a light blue film on the 1% alkaline peptone
water. Such properties are characteristic of the following microorganism:
A. Bacillus
B. Clostridium
C. Spirochete
D. Spirillum
E. Vibrio
16. A child is presumably ill with diphtheria. A specimen of affected mucous membrane
of his pharynx was taken for analysis. The smear was stained and microscopic examination revealed
yellow rods with dark blue thickenings on their ends. What structural element of a germ cell was
revealed in the detected microorganisms?
A. Volutin granules
B. Plasmids
C. Capsule
D. Spores
E. Flagella
17. On examination of a 6-year-old child the doctor noticed greyish film on the child’s
tonsils. Microscopy of the smear stained by Neisser method detected there Corynebacterium
diphtheria. What morphologic feature was the most indicative for determining the type of the agent?
A. Fence-like position of the agent’s cells
B. Spores that exceed cells in diameter
C. Localization of the causative agent within macrophages
D. Polar placement of volutin granules
E. Presence of the capsule
18. A smear from the tonsillar coating of a patient with suspected diphtheria was found
to contain blue bacilli with a thickening at the poles. What method of smear staining was used?
A. Leffler
B. Burri
C. Hins
D. Gram
E. Neisser
19. Microscopy of smear preparation stained with methylene blue revealed bacilli with
clublike expansions on their ends similar to C. diphtheriae. What additional method of staining
should be used to verify this assumption?
A. Neisser
B. Kozlovsky
C. Ziehl-Neelsen
D. D. Zdrodovsky
E. E. Aujeszky
20. A smear prepared from material obtained from patient with suspected diphtheria
contains yellow bacilli with blue grains at their ends. What staining was used in this case?
A. Ziehl-Nielsen

283
B. Kozlovsky

C. Romanovsky
D. Neisser

E. Loefler

21. There are several cases of children from boarding school suffering from sore throat.
Microscopy of tonsil smears stained according to Neisser method has revealed thin yellow bacilli with
dark brown grains on their ends placed in the shape of Roman numeral five. What infection can be
suspected in this case?
A. Diphtheria
B. Infectious mononucleosis
C. Listeriosis
D. Tonsillitis
E. Scarlet fever
22. Specimen of a patient’s sputum was stained with the following dyes and reagents:
Ziehl’s solution, methylene blue solution, 5% solutoin of sulfuric acid. What staining method was
applied?
A. Ziehl-Neelsen
B. Burri’s
C. Gram’s
D. Peshkov’s
E. Neisser’s
23. A consumptive patient has an open pulmonary form of disease. Choose what sputum
staining should be selected for finding out the tubercle (Koch’s) bacillus?
A. Method of Ziel-Neelsen B. Method of Romanowsky-Giemsa
C. Method of Gram D. Method of Neisser
E. Method of Burry-Gins
24. A centrifugate of urine sample obtained from patient with suspected renal
tuberculosis was used to make a slide mount for microscopy. What method should be used to stain
the slide and detect the causative agent?
A. Zielh-Neelsen stain B. Loeffler stain
C. Gram stain D. Aujeszky stain E. Burri stain
25. A patient complained about a carbuncle on his face. Examination results: neither
dense nor painful edema of subcutaneous cellular tissue, there is black crust in the middle of the
carbuncle and peripheral vesicular rash around it. Bacteriological examination revealed presence of
immobile streptobacilli able of capsulation. What microorganisms are causative agents of this
disease?
A. Bacillus antracis
B. Staptylococcus aureus
C. Bacillus anthracoides
D. Bacillus megaterium
E. Bacillus subtilis
26. A 34 year old male patient consulted a doctor about face carbuncle. Objectively: a
loose, painless edema of hypodermic tissue; black crust in the center of carbuncle, vesicular rash
around it. Microbiological examination revealed static streptobacilli capable of capsule building.
What microorganisms are the causative agents of this disease?
A. Bacillus antracis B. Staptylococcus aureus
C. Bacillus subtilis D. Bacillus anthracoides
E. Bacillus megaterium
27. Microscopic examination of a microbial culture revealed fusiform spore-forming
microorganisms
that get violet-blue Gram’s stain. What microorganisms were revealed?
A. Clostridia
284
B. Streptococci
C. Spirochaete
D. Actinomycete
E. Diplococci
28. On microscopic examination of leftovers of the canned meat eaten by patient with
severe food toxicoinfection the following was detected: gram-positive bacilli with subterminal
staining defect and configuration alteration of bacilli generally resembling a tennis racket. What
agent was detected?
A. C.botulinum
B. P.vulgaris
C. E.coli
D. S.aureus
E. S.enteritidis
29. Botulism agent causes severe food toxicoinfection. Point out the most characteristic
morphologic feature of botulism agent.
A. Gram-positive spore-forming bacilli with subterminal spore
B. Thick gram-positive non-sporeforming bacilli
C. Gram-positive spore-forming bacilli with terminal spore
D. Thin mobile spore-forming bacilli with central spore
E. Thick gram-positive non-sporeforming bacilli
30. The causative agent of botulism causes severe food poisoning. Specify the most
characteristic morphological feature of botulism causative agent:
A. Gram-positive bacillus with subterminal spore
B. Thick gram-positive non-spore-forming bacillus
C. Gram-positive bacillus with terminal spore
D. Thin mobile bacillus with central spore
E. Thick gram-positive bacillus without spores and flagella
31. A patient has food poisoning. Laboratory analysis revealed a culture of anaerobic
gram-positive spore-forming bacteria. What is the most likely kind of the isolated causative agent?
A. C. perfringens
B. Proteus vulgaris C. P. mirabilis
D. Vibrio parahemolyticus E. Esherichia coli
32. While studying a microslide obtained from the punctuate of a regional lymph node
and stained by Romanovsky-Giemsa method a physician revealed some light pink thin
microorganisms with 12-14 regular spiral coils and pointed ends, up to 10-13 micrometer long. This
might be the causative agent of the following disease:
A. Syphilis
B. Trypanosomiasis
C. Leptospirosis
D. Relapsing fever
E. Leishmaniasis
33. In the micropreparation made from patient’s regional lymph node punctate and
stained according to Romanovsky-Giemsa method, the doctor found out thin microorganisms with
12-14 equal ringlets and pale-pink sharp pointes 10-13 mkm in length. The pathogen of what disease
is it about?
A. Leishmaniasis
B. Leptospirosis
C. Surra
D. Syphilis
E. Relapsing fever
34. In a microslide of the patient’s regional lymph node stained with Giemsa method a
doctor detected thin microorganisms with 12-14 uniform tendrils with pointed tips, 10-13
micrometers in length, pale pink in color. In this case they can be identified as infectious agents of
the following disease:

285
A. Syphilis B. Trypanosomiasis
C. Leptospirosis D. Relapsing fever E. Leishmaniasis
35. Bacterioscopic examination of chancre material revealed some mobile, long,
convoluted microorganisms with 8-12 regular coils. These features are typical for:
A. Treponema
B. Borrellia
C. Leptospira
D. Vibrios
E. Campylobacter
36. A man died from an acute infectious disease accompanied by fever, jaundice,
haemorrhagic rash on the skin and mucous membranes as well as by acute renal insufficiency.
Histological examination of renal tissue (stained by Romanovsky-Giemsa method) revealed some
convoluted bacteria looking like C and S letters. What bacteria were revealed?
A. Leptospira
B. Treponema
C. Spirilla
D. Borrelia
E. Campilobacteria
37. Patients with similar complaints applied to the doctor: weakness, pain in the
intestines, disorder of GIT. Examination of the feces revealed that one patient with four nucleus cysts
should be hospitalized immediately. For what protozoa are such cysts typical?
A. Lamblia B. Dysenteric amoeba
C. Balantidium D. Trichomonas
E. Intestinal amoeba
38. Several patients with similar complaints came to the doctor. They all present with
weakness, pain in the intestines, indigestion. Feces analysis revealed the need for urgent
hospitalization of the patient, who had microbial cysts with four nuclei detected in his samples. Such
cysts are characteristic of the following protozoon:
A. Entamoeba histolytica.
B. Entamoeba coli.
C. Balantidium.
D. Trichomonad.
E. Lamblia.
39. Microscopy of dental plaque revealed unicellular organisms. Their cytoplasm had two
distinct layers, barely visible core, wide pseudopodia. The patient is most likely to have:
A. Entamoeba gingivalis B. Entamoeba histolytica
C. Trichomonas tenax D. Lamblia E. Entamoeba coli
40. Carious cavities of a 29-year-old patient contain the parasitic protozoa. It is
established that they relate to the Sarcodina class. Specify these single-celled organisms:
A. Entamoeba gingivalis
B. Entamoeba histolutica C. Entamoeba coli
D. Amoeba proteus E. Lamblia intestinalis
41. A patient complains of acute spastic abdominal pain, frequent urge to defecate,
liquid bloody feces with mucus. Laboratory analysis of fecal smear revealed inconstant in shape
organisms with erythrocyte. What is the most likely diagnosis?
A. Amebiasis.
B. Lambliasis. C. Schistosomiasis.
D. Intestinal trichomoniasis. E. Balantidiasis.
42. A 40-year-old patient presents with abdominal pain, frequent loose stools with
mucus and blood. Stool analysis revealed vegetative forms of some protozoa sized 30-40 microns,
with short pseudopodia, containing large amounts of phagocytosed erythrocytes. What protozoan
disease does the patient have?
A. Amebiasis
B. Leishmaniasis

286
C. Trichomoniasis
D. Giardiasis
E. Toxoplasmosis
43. Examination of duodenal contents revealed some pyriform protozoa with twin nuclei
and four pairs of flagella. There were two supporting filaments between the nuclei and a suctorial
disc on the ventral side. What representative of protozoa was revealed in this patient?
A. Lamblia
B. Toxoplasma
C. Leishmania
D. Intestinal trichomonad
E. Trypanosome
44. Examination of the duodenal contents revealed some pear-shaped protozoa with two
nuclei and four pairs of flagella. The organisms had also two axostyles between the nuclei and a
ventral adhesive disc. What protozoan representative was found in the patient?
A. Lamblia B. Toxoplasma C. Leishmania
D. Intestinal trichomonad E. Trypanosome
45. A duodenal content smear of a patient with indigestion contains protozoa 10-18 mcm
large. They have piriform bodies, 4 pairs of filaments, two symmetrically located nuclei in the
broadened part of body. What kind of the lowest organisms is it?
A. Lamblia
B. Dysentery ameba
C. Trichomonas
D. Intestinal ameba
E. Balantidium
46. A 13 year old child complains about poor appetite, pain in the right subcostal area.
Microscopical examination of duodenal contents revealed big pyriform cells with two nuclei. What
microorganism was revealed?
A. Lamblia
B. Trichomonas
C. Amoeba
D. Trypanosoma
E. Toxoplasma
47. Sanitary assessment of a pond, where the children from a recreation summer
camp take their swims, detected there oval cysts 50-60 micron in diameter, with 2 nuclei
visible in their cytoplasm (macronucleus and micronucleus). What protozoa do these cysts
belong to?
A. Toxoplasma B. Amoeba
C. Balantidium D. Lamblia E. Euglena
48. A patient working at a pig farm complains about paroxysmal abdominal pain, liquid
feces with admixtures of mucus and blood, headache, weakness, fever. Examination of large
intestine revealed ulcers from 1 mm up to several cm large, feces contained oval unicellular
organisms with cilia. What disease should be suspected?
A. Balantidiasis
B. Amebiasis
C. Toxoplasmosis
D. Lambliasis
E. Trichomoniasis
49. Slime, blood and protozoa 30-200 microns of length have been revealed in a man’s
feces. The body is covered with cilias and has correct oval form with a little bit narrowed forward and
wide round shaped back end. On the forward end a mouth is visible. In cytoplasm there are two
nucleuses and two short vacuoles. For whom are the described attributes typical?
A. Balantidium
B. Lamblia
C. Dysenteric amoeba

287
D. Trichomonas
E. Intestinal amoeba
50. A patient has symptoms of inflammation of urogenital tracts. Examination of a
vaginal smear revealed big monocellular, pear-shaped organisms with the pointed spike at the
posterior end of body, big nucleus and undulating membrane. What protozoa were found in the
smear?
A. Trichomonas vaginalis
B. Trichomonas hominis
C. Trichomonas buccalis
D. Trypanosoma gambiense
E. Lamblia intestinalis
51. A gynaecologist was examining a patient and revealed symptoms of genital tract
inflammation. A smear from vagina contains pyriform protozoa with a spine, flagella at their front;
there is also an undulating membrane. What disease can be suspected?
A. Urogenital trichomoniasis
B. Lambliasis C. Intestinal trichomoniasis

D. Toxoplasmosis E. Balantidiasis

A female patient has symptoms of inflammation of urogenital tracts. A smear from the
vaginal mucous membrane contained big unicellular pyriform organisms with a sharp spike on the
back end of their bodies; big nucleus and undulating membrane. What protozoa were revealed in the
smear?

B. Trichomonas vaginalis
C. Trichomonas hominis
D. Trichomonas buccalis
E. Trypanosoma gambiense
F. Lamblia intestinalis
52. Microscopical examination of discharges from the gums of a patient ill with
paradontosis revealed some protozoan pear-shaped organisms 6-13 micrometer long. The parasite
has one nucleus and undulating membrane, there are four flagella at the front of its body. What
protozoan were found?
A. Trichomonads
B. Leishmania C. Amoebae

D. Balantidia E. Lamblia

53. A smear from frothy and purulent vaginal discharges of a 42 y.o. woman was stained
by Romanovsky-Giemsa method. Its analysis revealed some microorganisms of flagellates class. What
microorganism were the most probably revealed?
A. Trichomonas vaginalis
B. Leishmania donovani
C. Trypanosoma gambiense
D. Trihomonas hominis
E. Lamblia intestinalis
54. A 42-year-old female has foamy purulent vaginal discharges. The smear stained by
Romanovsky-Giemsa’s method has been found to include flagellated bacteria. What is the most likely
microorganism that has been found by the doctor?
A. Trichomonas vaginalis
B. Trypanosoma gambiense
C. Leishmania donovani
D. Trihomonas hominis
E. Lamblia intestinalis

288
55. A lymph node punctate of a patient with suspected protozoa disease was examined.
Examination of the stained specimen (Romanovsky’s stain) revealed some crescent bodies with
pointed end, blue cytoplasm and red nucleus. What protozoan were revealed in the smears?
A. Toxoplasmas
B. Malarial plasmodiums
C. Dermotropic leishmania
D. Viscerotropic leishmania
E. Trypanosomes
56. A puncture sample taken from the lymph node of a patient with preliminary
diagnosis of protozoan disease has been investigated. The preparation was processed with Giemsa
stain and the following was detected: crescent-shaped bodies with pointed tips, blue cytoplasm and
red nuclei. What protozoa have been detected in the preparation?
A. Toxoplasma
B. Plasmodium malariae
C. Trypanosoma
D. Viscerotropic Leishmania
E. Dermatotropic Leishmania
57. Examination of a man revealed a protozoan disease that affected brain and caused
vision loss. Blood analysis revealed unicellular half-moon-shaped organisms with pointed end. The
causative agent of this disease is:
A. Toxoplasma
B. Leishmania
C. Lamblia
D. Amoeba
E. Trichmonad
58. A man is ill with a protozoan disease characterized by cerebral affection and loss of
sight. Blood analysis revealed halfmoon-shaped unicellular organisms with pointed ends. This disease
is caused by:
A. Toxoplasma
B. Leishmania
C. Lamblia
D. Amoeba
E. Trichomonad
59. The contents of vesicles that appeared on the mucous membrane of a patient with
variola were sent to a virological laboratory. Which of the listed changes were revealed during the
smear microscopy?
A. Paschen bodies B. Babes-Negri bodies

C. Guarnieri bodies D. Babes-Ernst bodies

E. Syncytium

60. A three-year-old child has had marked diarrhea for three days. Immune electron
microscopy of his excrements revealed bilayer pseudocovered capsid viruses that looked like small
spoke wheels. What viruses have been revealed?
A. Rotaviruses B. Coxsackie viruses C. ECHO viruses

D. Coronaviruse E. Reoviruses

61. An outbreak of an intestinal infection occurred in a kindergarten on the eve of New


Year holidays. Bacteriological examination of patients’ feces didn’t reveal any pathogenic bacteria.
Electron microscopy revealed roundish structures with clear outer edges and a thick core resembling
a wheel. Specify the most likely causative agent of this infection:
A. Rotavirus B. Adenovirus C. Coxsacki-virus

289
D. E. coli E. P.vulgaris

62. A 40 year old man noticed a reddening and an edema of skin in the area of his neck
that later developed into a small abscess. The incised focus is dense, yellowish-green. The pus
contains white granules. Histological examination revealed drusen of a fungus, plasmatic and
xanthome cells, macrophages. What type of mycosis is the most probable?
A. Actinomycosis B. Aspergillosis C. Candidosis

D. Sporotrichosis E. Coccidioidomycosis

63. A 40-year-old man developed skin redness and an swelling in the neck area, where
eventually a small abscess appeared. The section the focus is dense and yellow-green colored. In the
purulent masses there are white granules. Histologically there are fungal druses, plasma and
xanthome cells, and macrophages detected. Specify the most correct etiological name of this
pathological process?
A. Actinomycosis B. Furuncle

C. Carbuncle D. Syphilis E. Leprosy

64. Microscopical examination of an infiltrate removed from the submandibular skin area
of a 30 y.o. man revealed foci of purulent fluxing surrounded by maturing granulations and mature
connective tissue, the pus contains druses consisting of multiple short rod-like elements with one
end attached to the homogenous centre. What disease is it?
A. Actinomycosis
B. Tuberculosis C. Syphilis

D. Candidosis E. –

65. A 32-year-old patient who lives in the countryside consulted a doctor about a painful
swelling and a fistula in the submandibular region. Examination revealed an infiltration with a fistula
discharging thick pus and containing white granules. On dissection the infiltration tissues turned out
to be dense, yellow-green and had honeycomb structure because of multiple abscesses. What is the
most likely diagnosis?
A. Actinomycosis B. Tuberculosis C. Lepra

D. Syphilis E. Submandibular abscess

66. Microscopic examination of pus sample taken from mandibular fistula canal and
stained by Gram’s method has revealed druses with gram-positive coloring in the center and cone-
shaped structures with gram-negative coloring. Such morphology is characteristic of the agent of:
A. Fusobacteriosis B. Actinomycosis

C. Staphylococcal osteomyelitis

D. Anaerobic infection E. Candidiasis

290
Physiology of microbes
1. The sterile Petri dishes and pipettes are necessary to prepare for microbiological
tests in bacteriological laboratory. What way of sterilization should be applied in this case?
A. Dry-heat sterilization B. Tyndallization
C. Pasteurization D. Boiling
E. Steam sterilization in autoclave
2. What method should be applied for sterilization of heatproof and moistureproof
stomatological instruments in order to ensure total destruction of viruses, vegetative and spore
forms of microorganisms?
A. Autoclaving B. Boiling C. Pasteurization
D. Tyndallization E. Burning in the flame of gas burner
3. Which of the following sterilization methods ensures total death of microorganisms and
their spores during onetime thermal processing of an object?
A. Autoclaving B. Boiling C. Tyndallization
D. Pasteurization E. –
4. Meat peptone broth is prepared for sterilization in bacteriological laboratory. What
sterilization method is advisable?
A. Autoclaving B. Ignition C. Boiling D. Filtering E. Dry heat
5. Bacteriological laboratory has the task to sterilize nutrient mediums containing substances
that convert under the temperature over 1000C (urea, carbohydrates). What method of sterilization
should be used?
A. Fluid steam sterilization B. Autoclaving
C. Boiling D. Tindalization E. Pasteurization
6. What method of sterilization should be used during the manufacturing liquid dosage forms
containing proteins?
A. Filtering B. Boiling C. Gas sterilization
D. Autoclaving E. Pasteurization
7. In order to keep vitality and stability of eubiotics microorganisms in frozen state are dried
under conditions of high vacuum. What method is it?
A. Lyophilization B. Pasteurization
C. Tyndallization D. Inactivation E. Hybridization
8. What method ensures reliable sterilization of biological liquids (serums, solutions,
enzymes, vitamines etc.) that cannot be exposed to high temperatures?
A. Tyndallization B. Dry heat C. Flowing steam
D. Moist steam under pressure E. Flaming
9. Having completed work in a laboratory, a student must tidy up the workspace, perform
disinfection of the workbench and tools. What chemicals should be used for disinfection?
A. Chloramine
B. Hydrochloric acid
C. Formalin
D. Chloroform
E. Ether
10. In the surgical ward, the dressing material was undergoing sterilization in an autoclave.
Through an oversight of a nurse the mode of sterilization was changed and the temperature in the
autoclave reached only 100°C instead of the due 120°C.What microorganisms can stay viable under
these conditions?
A. Bacilli and clostridia B. Staphylococci and streptococci
C. Mold and yeast fungi D. Salmonella and klebsiella
E. Corynebacteria and mycobacteria
11. Passive and active transport of substances is realized through the cell membrane. Name
the type of active transport by which the membrane changes its structure:
A. Endocytosis B. Osmosis C. Filtration
D. Diffusion E. Facilitated diffusion

291
12. Those organisms which in the process of evolution failed to develop protection from
H2O2 can exist only in anaerobic conditions. Which of the following enzymes can break hydrogen
peroxide down?
A. Peroxidase and catalase B. Oxygenase and hydroxylase
C. Cytochrome oxidase, cytochrome B5
D. Oxygenase and catalase E. Flavin-dependent oxidase
13. A bacterial cell obtains nutrients by different ways. One of them is the facilitated diffusion
that is realized by special membrane carrier proteins. What are these proteins called?
A. Permeases B. Lyases C. Oxidoreductases D. Isomerases E. Ligases
14. Pathogenic microorganisms produce various enzymes in order to penetrate body tissues
and spread there. Point out these enzymes among those named below.
A. Hyaluronidase, lecithinase
B. Lyase, ligase C. Transferase, nuclease
D. Oxydase, catalase E. Esterase, protease
15. Pathogenic microorganisms are characterized by presence of aggression enzymes that
determine their virulence. Select an aggression enzyme:
A. Hyaluronidase B. Carbohydrase C. Transferase D. Oxidase E. Lyase
16. Pathological material taken from a patient suffering from pulpitis was inoculated onto
Kitt-Tarozzi cultural medium. It is planned to find the following microorganisms:
A. Anaerobic B. Acid-resistant
C. Acidophilic D. Haemolytic E. Aerobic
17. A patient was taken to a hospital with acute food poisoning caused by homemade canned
mushrooms. The product analysis revealed some microorganisms that develop only in the absence of
oxygen. What microorganisms caused the poisoning?
A. Obligate anaerobes B. Facultative anaerobes
C. Microaerophiles D. Obligate aerobes
E. Capnophiles
18. The causative agents of intestinal infections can grow at refrigerator temperatures, which
may cause infection in people. What type of temperature optimum do these microorganisms relate
to?
A. Psychrophilic B. Mesophilic C. Thermophilic
D. Anthropophilic E. Necrophilic
19. For cultivation of Brucella, pure cultures should be incubated in CO2 enriched
atmosphere. What type of breathing is typical for Brucella?
A. Capnophilic B. Facultative anaerobic
C. Obligate anaerobic D. Obligate aerobic E. Any
20. A patient underwent esophagogastroduodenoscopy. Analysis of the biopsy material
enabled doctors to diagnose him with helicobacteriosis. What property of the bacteria found in this
patient had to be obligatory taken into account during their cultivation?
A. Microaerophilic ability B. Presence of urease
C. Colonisation of gastral cells
D. Absence of spores and capsules
E. Presence of six polar flagella
21. In microbiology class students had been growing pure bacterial culture. Bacterial
inoculation of solid medium was performed to obtain separate visible colonies, resulting in two
colonies, R-type and S-type, grown in thermostat after one day of incubation. What microorganism
properties were described by students?
A. Cultural B. Tinctorial C. Biochemical
D. Morphologic E. Antigenic
22. A 55-year-old male patient was hospitalized to a surgical clinic for suspected septicemia.
What material should be taken for analysis?
A. Blood, sugar broth B. Liquor, serum agar
C. Urine, beef-extract broth
D. Pus, yolk saline agar E. Lymph node punctate, cysteine agar

292
23. A patient operated for acute paraproctitis undergoes antibacterial and detoxification
therapy. According patient state doctor suspected sepsis. What study will confirm the diagnosis?
A. Blood culture for a pathogen
B. X-ray of lungs C. Liver ultrasound
D. Determining the rate of microbial contamination of wound
E. Determining the rate of average-weight molecules
24. A 4-year-old child presents with general weakness, sore throat and deglutitive problem.
After his examination a doctor suspected diphtheria and sent the material to the bacteriological
laboratory. In order to determine the diphtheria causative agent the material should be inoculated
into the following differential diagnostic medium:
A. Blood tellurite agar B. Endo’s agar
C. Ploskyrev’s agar D. Sabouraud’s agar
E. Levenshtein-Yessen agar
25. It is suspected that the workers of a serum drugs plant at a regional hemotransfusion
station are carriers of pathogenic staphylococcus aureus. In order to detect staphylococcus carriage,
the material from the nasopharynx of the workers should be inoculated into the
following medium:
A. Egg-yolk-salt agar B. Endo agar
C. Meat infusion broth D. Kessler medium E. Blood agar
26. Microbiological studies of air in the pharmacy room revealed the presence of pathogenic
staphylococci. Select the medium in which you can detect the lecithinase activity of the isolated
microorganism:
A. Yolk-salt agar B. Blood agar
C. Bismuth sulfite agar D. Sugar agar E. Meat-extract agar
27. It was suspected that among workers of serum medications factory of regional
hemotransfusion station there are carriers of pathogenic cocci. What medium should the material
from nasopharynx of workers be inoculated of in order to reveal Staphylococcous carriage?
A. Yolk-salt medium B. Endo agar
C. Beef-extract broth D. Ressler’s medium E. Blood agar
28. Examination of air state in drugstore premises for preparation of injection drugs was
done by method of sedimentation. It revealed 5 small roundish colonies with zone of hemolysis
around them. Inoculations were made on the following cultural medium:
A. Blood agar B. Endo agar
C. Meat infusion agar D. Egg yolk and salt agar
E. Lewin’s agar
29. In order to establish the possible contamination of a medication with fungi, a nutrient
medium was inoculated, which resulted in growth of large cream-like colonies. What nutrient
medium was used in this case?
A. Sabouraud B. Lowenstein-Jensen
C. Roux D. Loeffler E. Finn-2
30. Crude herbal drugs must be examined for yeast-like fungi. What agar can ensure
development of these microorganisms so that associating microflora will grow very slowly or won’t
grow at all?
A. Sabouraud’s peptone agar
B. Endo agar C. Meat infusion agar
D. Milk-salt agar E. Blood agar
31. A 3 month old infant has got a white deposition on the mucous membrane of his mouth,
tongue and lips. The doctor suspected candidosis. What nutrient medium should be used for
inoculation of the material under examination in order to confirm this diagnosis?
A. Sabouraud B. Endo
C. Loewenstein-Jensen D. Roux E. Clauberg
32. A patient has a necrotizing phlegmon of his lower extremity. A doctor suspects a gas
gangrene. Microscopy reveals grampositive bacilli. In order to confirm the diagnosis further
bacteriological tests should include inoculation of the material into the following nutrient medium:

293
A. Kitt-Tarozzi medium B. Endo agar C. Levine agar
D. Meat-peptone agar E. Milk-salt agar
33. A lot of pyoinflammatory processes in oral cavity are caused by anaerobes. What nutrient
medium can be used for control of wound textile contamination by anaerobes?
A. Kitt-Tarozzi B. Endo C. Roux
D. Sabouraud’s E. Ploskirev’s
34. In 8 days after a surgery the patient develops tetatus. The surgeon suspects this condition
to be caused by suture material contaminated by tetanus agent. The material is delivered to a
bacteriological laboratory. What nutrient medium is required for primary inoculation of the suture
material?
A. Kitt-Tarozzi medium B. Endo agar C. Hiss medium
D. Sabouraud agar E. Egg-yolk salt agar
35. A 12-year-old boy has been hospitalized for suspected food poisoning. The fecal samples
were inoculated on the Endo agar, which resulted in growth of a large number of colorless colonies.
What microorganism is most likely to be EXCLUDED from the list of possible causative agents of the
disease?
A. Escherichia coli B. Salmonella enteritidis
C. Proteus vulgaris D. Pseudomonas aeruginosa
E. Yersinia enterocolitica
36. Red colonies spread in the large quantity in the Endo culture medium were revealed on
bacteriological stool examination of a 4-month-old baby with the symptoms of acute bowel infection.
What microorganism can it be?
A. Escherichia B. Salmonella C. Staphylococcus
D. Streptococcus E. Shigella
37. On bacteriological examination of the defecation of a 4-months-old baby with the
symptoms of acute bowel infection there were revealed red colonies spread in the large quantity in
the Endo environment. What microorganism can it be?
A. Staphylococcus B. Streptococcus
C. Shigella D. Salmonella E. Escherichia
38. During bacteriological examination of solutions prepared in a drug-store there appeared
red colonies with metallic shining that grew on Endo agar. What bacteria can they be?
A. Escherichia B. Shigella C. Staphylococci
D. Streptococci E. Salmonella
39. During bacteriological analysis of solutions prepared in a pharmacy some red colonies
with metallic glitter have grown on Endo agar. What microbes were revealed?
A.Escherichia B.Shigella C.Staphylococci D.Streptococci E. Salmonellа
40. Bacteriological examination of a patient with food poisoning required inoculation of a
pure culture of bacteria with the following properties: gramnegative movable bacillus that grows in
the Endo’s medium in form of colourless colonies. A representative of which species caused this
disease?
A. Salmonella B. Shigella C. Yersinia D. Esherichia E. Citrobacter
41. On bacteriological study of rinsing water of the patient with food poisoning, the pure
bacterial culture was inoculated with the following properties: gram-negative motile bacillus in the
Endo environment grows like achromic colony. Representative of what genus has caused the illness?
A. Yersinia B. Citrobacter C. Salmonella D. Shigella E. Escherichia
42. A patient was admitted to the infectious department of a hospital. His provisional
diagnosis was "acute gastroenteritis". Inoculation of feces on bismuth-sulfite agar induced growth of
black colonies with metallic glitter. What microorganisms should you think of?
A. Salmonella B. Escherichia C. Shigella D. Yersinia E. Brucella
43. After inoculation of the material obtained from the pharynx of an angina patient onto the
blood-tellurite agar, grey colonies could be observed. They were 4-5 mm in diameter, radially striated
(in form of rosettes). Microscopical examination revealed gram-positive bacilli with clavate swollen
ends arranged in form of wide-spread fingers. Identify these microorganisms:
A. Diphtheria corynebacteriae B. Clostridium botulinum

294
C. Diphtheroids D. Streptococci E. Streptobacilli
44. Inoculum from pharynx of a patient ill with angina was inoculated into bloodtellurite
agar. It resulted in growth of grey, radially striated (in form of rosettes) colonies 4-5 mm in diameter.
Grampositive bacilli with clublike thickenings on their ends placed in form of spread wide apart
fingers are visible by microscope. What microorganisms are these?
A. Diphtheria corynebacteriae B. Botulism clostridia
C. Diphtheroids D. Streptococci E. Streptobacilli
45. A sample taken from the pharynx of a patient with angina was inoculated on the blood-
tellurite agar. This resulted in growth of grey, radially striated (in form of rosettes) colonies up to 4-5
mm in diameter. Microscopically there can be seen gram-positive rods with club-shaped ends
arranged in form of spread fingers. What microorganisms are these?
A. Corynebacteria diphtheriae B. Clostridium botulinum
C. Diphtheroids D. Streptococci E. Streptobacilli
46. Blood of a patient with presumable sepsis was inoculated into sugar broth. There
appeared bottom sediment. Repeated inoculation into blood agar caused growth of small
transparent round colonies surrounded by hemolysis zone. Examination of a smear from the
sediment revealed gram-positive cocci in form of long chains. What microorganisms are present in
blood of this patient?
A.Streptococci B.Micrococci C.Staphylococci D.Tetracocci E.Sarcina
47. Examination of a patient with pustular skin lesions allowed to isolate a causative agent
that forms in the blood agar roundish yellow middle-sized colonies surrounded by haemolysis zone.
Smears from the colonies contain irregular-shaped clusters of gram-positive cocci. The culture is
oxidase- and catalasepositive, ferments mannitol and synthesizes plasmocoagulase. What causative
agent was isolated?
A. Staphylococcus aureus B. Streptococcus agalactiae
C. Streptococcus pyogenes D. Staphylococcus epidermidis
E. Staphylococcus saprophyticus
48. Purulent discharges of a patient with a mandibulofacial phlegmon contain spheroid
microorganisms making S-shaped colonies with golden pigment that produce lecithinase,
plasmocoagulase, hemolysin and decompose mannitol under anaerobic conditions. Specify the kind
of microorganisms that had caused the suppuration:
A. S. aureus B. Str. pyogenes C. Str. mutans
D. S. epidermidis E. Str. sanguis
49. From the purulent exudate of a patient with odontogenic phlegmon a pure culture of
Gram(+) microorganisms was segregated. This culture was lecithinously active, coagulated plasma of
a rabbit, decomposed mannitol under anaerobe conditions. What microorganism may have
contributed to the origin of suppurative complication?
A. S.aureus B. S.epidermidis
C. S.pyogenes D. S.viridans E. S.mutans
50. After inoculation of feces sample into the 1% alkaline peptonic water and 8-hour
incubation in the thermostat at a temperature of 37oC a culture in form of a tender bluish film has
grown. Such cultural properties are typical for the causative agent of the following disease:
A. Cholera B.Plague C.Typhoid fever D.Paratyphoid fever E.Dysentery
2018
From the feces of a patient with acute gastroenteritis a pure culture of microorganisms was
obtained. The microorganisms are small mobile slightly curved gram-negative bacilli that
within 6 hours grow into a light blue film on the 1% alkaline peptone water. Such properties
are characteristic of the following microorganism:
A. Bacillus B. Clostridium C. Spirochete D. Spirillum E.
Vibrio
Initial inoculation of water in 1% peptone water resulted in growth of a thin film on the
medium surface in 6 hours. Such cultural properties are characteristic of causative agent of the
following disease:
A. Dysentery B. Pseudotuberculosis C. Tuberculosis
295
D. Plague E. Cholera
In 8 days after a surgery the patient developed tetanus. The surgeon suspects this condition
to be caused by suture material contaminated by tetanus agent. The material is delivered to a
bacteriological laboratory. What nutrient medium is required for primary inoculation of the suture
material?
A. Kitt-Tarozzi medium. B. Endo agar.
C. Sabouraud agar. D. Egg-yolk salt agar. E. Hiss medium
A bacteriological laboratory received a sample of dried fish from an outbreak of food
poisoning. Inoculation of the sample on Kitt-Tarozzi medium revealed microorganisms resembling
tennis racket. These microorganisms are causative agents of the following disease:
A. Botulism. B. Diphtheria. C. Typhoid fever.
D. Salmonellosis. E. Dysentery.
Special
During inspectation of dental tools for sterility in one case gram-positive cocci were detected.
They were situated in clusters and yielded positive plasma coagulation reaction; the cocci were
fermenting mannitol in anaerobic conditions and exhibiting lecithinase activity. What microorganism
as detected?
A. St. saprophiticus
B. St. epidermidis
C. Corynebacterium xerosis
D. Staph. aureus
E. Str. pyogenes
Microbiological purity of tableted drugs had been tested at factory. Samples cultivation in
mannitol salt agar resulted in growth of golden-yellow colonies, microscopic examination of colonies
detected grampositive globular bacteria positioned in clusters; microorganisms had plasma
coagulation properties. What pure bacterial culture was obtained?
A. Staphylococcus aureus
B. Enterobacteriaceae
C. Staphylococcus epidermidis
D. Staph. saprophyticus
E. Pseudomonas aeruginosa
Bacilli were extracted from investigated sample. The bacilli are curved, extremely mobile,
gram-negative, form no spores or capsules, have anaerobic form of respiration. They form
transparent smooth colonies in alkaline agar, ferment saccharose and mannose into acid, produce
exotoxin, fibrinolysin, collagenase, and
hyaluronidase. What
agent was extracted?
A. Comma bacillus
B. Proteus
C. Dysentery bacillus
D. Blue pus bacillus
E. Colibacillus
Initial inoculation of water in 1% peptone water resulted in growth of a thin film on the
medium surface in 6 hours. Such cultural properties are characteristic of causative agent of the
following disesase:
A. Cholera
B. Plague
C. Tuberculosis
D. Dysentery
E. Pseudotuberculosis
A 4-year-old child presents with general weakness, sore throat and deglutitive problem. After
his examination a doctor suspected diphtheria and sent the material to the bacteriological
laboratory. In order to determine the diphtheria causative agent the material should be inoculated
into the following differential diagnostic medium:

296
A. Blood tellurite agar
B. Levenshtein-
Yessen agar
C. Ploskyrev’s
agar
D. Sabouraud’s
agar
E. Endo’s agar
A bacteriological laboratory received a sample of dried fish from an outbreak of food
poisoning. Inoculation of the sample on Kitt-Tarozzi medium revealed microorganisms resembling
tennis racket. These microorganisms are causative agents of the following disease:
A. Botulism. B. Diphtheria. C. Typhoid fever.
D. Salmonellosis. E. Dysentery.
A bacteriological laboratory studied the home-made dried fish which had caused a severe
food poisoning. Microscopy of the culture grown on the Kitt-Tarozzi medium revealed
microorganisms resembling
a tennis racket. What is the most
likely diagnosis?
A. Botulism
B. Salmonellosis
C. Cholera
D. Dysentery
E. Typhoid fever
A bacteriological laboratory has been investigating a sample of homemade dried fish that
was the cause of severe food poisoning. Microscopy of the culture inoculated in Kitt-Tarozzi medium
revealed microorganisms resembling a tennis racket. What diagnosis can be made?
A. Botulism
B. Salmonellosis C. Cholera
D. Dysentery E. Typhoid fever
A patient has severe catarrhal symptoms. Material growth on Bordet-Gengou agar showed
mercury-drop like colonies. Examination of the blood smears revealed some small
ovoid gram-negative
bacilli sized 1-3
microns. What
microorganisms
were isolated?
A. Bordetella
B. Corynebacteria
C. Mycobacteria
D. Meningococcus
E. Brucella
During bacteriological examination of sputum of a child with choking cough and fever there
were revealed glossy smooth colonies growing on casein-charcoal agar and reminding of mercury
drops. Microscopic examination revealed short Gram-negative bacteria. What microorganism was
secured from the sputum?
A. Bordetella pertussis
B. Haemophylus influenzae
C. Corynebacterium dyphtheriae
D.Klebsiella pneumoniae
E. Streptococcus pyogenes
On the base of the clinical data a child was diagnosed with atypical pneumonia resistant to
the effects of beta-lactam antibiotics. The patient’s sputum was cultured and incubated in a special
medium, which resulted in growth of microorganisms forming microscopic colonies with a dense
center (looking like fried eggs). What microorganism caused the disease?

297
A. Mycoplasma pneumoniae
B. Klebsiella pneumoniae
C. Str. pneumoniae
D. L. pneumophila
E. Chlamidia pneumoniae
From a medicinal herb a certain phytopathogenic microorganism was secured. In the nutrient
medium it forms "fried egg"colonies. What is the most likely agent?
A. Mycoplasma
B. Yeasts
C. Actinomycetes
D. Nocardia
E. Pseudoonas
Bacteriological examination of the urine of the patient with pyelonephritis revealed
microorganisms that produced yellow-green pigment and a characteristic odor in meat-peptone
agar. What are they called?
A. Pseudomonas
B. Escherichia C. Proteus
D. Klebsiella E. Azotobacter
Many diseases of medicinal plants are caused by bacteria of the Pseudomonas genus. Select
the bacteria
relating to this genus:
A. Blue pus bacillus
B. Colon bacillus
C. Proteus
D. Mycoplasma
E. Micrococci
Urine examination of a patient with acute cystitis revealed leukocytes and a lot of gram-
negative bacilli. Inoculation resulted in growth of colonies of mucous nature that formed green
soluble pigment. What microorganism is the most probable cause of the
disease?
A. Pseudomonas aeruginosa
B. Klebsiella pneumoniae
C. Escherihia coli
D. Proteus mirabilis
E. Salmonella enteritidis
A patient of surgical department complains about pain in the small of her back and in the
lower part of her belly; painful and frequent urination. Bacteriological examination of urine revealed
gram-negative oxidase-positive rod-like bacteria forming greenish mucoid colonies with specific
smell. What causative agent can it be?
A. Pseudomonas aeruginosa
B. Mycoplasma pneumonie C. Str.pyogenes
D. E. coli E. Proteus mirabilis
A patient has wound abscess. Bacteriological examination of the wound content revealed a
gram-negative bacillus which forms semi-transparent mucous colonies of blue-green color with a
pearlescent appearance on the beef-extract agar. Culture has a specific odor of violets or jasmine.
What type of pathogen was isolated from the patient’s wound?
A. P. aeruginosa B. P.vulgaris C. S.aureus D. S.pyogenes E. S.faecalis
A patient of oral surgery department has developed a purulent complication. Bacteriological
analysis of the wound discharge allowed to isolate a culture producing a blue-and-green pigment.
Which of the listed microorganisms may be a causative agent of the infection?
A. Pseudomonas aeruginosa B. Staph. epidermidis
C. B. subtilis D. Pr. vulgaris E. Klebsiella pneumoniae

298
A patient in the oral surgery department has got purulent complication. Bacteriological
analysis of the wound material found a culture that produces cyan pigment. What microorganism is
the most probable causative agent?
A. Pseudomonas aeruginosa B. Staph. epidermidis
C. B. subtilis D. Kleb. pneumoniae E. Pr.vulgaris
A patient being treated in the burns department has suppurative complication. The pus is of
bluish-green color that is indicative of infection caused by Pseudomonas aeruginosa. What factor is
typical for this causative agent?
A. Gram-negative stain B. Presense of spores
C. Coccal form D. Cell pairing E. Mycelium formation
Bacteriological inspection of disinfection quality at a pharmacy revealed a microorganism in
an utility room (in the sink). The microorganism has the following properties: mobile nonspore-
forming gram-negative bacteria that form capsular substance, grow well on ordinary nutrient media,
secrete the blue-green pigment. This microorganism is most likely to be of the following genus:
A. Pseudomonas B. Proteus C. Clostridium
D. Shigella E. Vibrio
A sample of a finished dosage form was found to be contaminated with some
microorganisms exhibiting the following properties: greenish fluorescent colonies of gram-negative
nonsporeforming bacilli that grew on the medium for the detection of pyocyanin. The bacilli release
the bluegreen pigment into the medium. What microorganisms contaminated the finished dosage
form?
A. Pseudomonas aeruginosa
B. Enterobacteriaceae C. Staphylococcus aureus
D. Staphylococcus epidermidis E. Staph. saprophyticus
During bacteriological examination of the purulent discharge obtained from a postoperative
wound an inoculation on meat infusion agar has been performed. The inoculation has resulted in
large colorless mucous colonies that in 24 hours with exposure to sunlight developed green-blue
pigmentation and smell of honey or jasmine. Bacterioscopy revealed gram-negative lophotrichea.
What bacterial culture is contained in purulent discharge?
A. Pseudomonas aeruginosa B. Klebsiella osaenae
C. Streptomyces griseus D. Proteus vulgaris E. Brucella abortus
A 3 m.o. baby has white film on the mucous membrane of his mouth, tongue and lips. A
doctor suspected candidosis. What nutrient medium should be applied for inoculation of the
material under examination in order to confirm this diagnosis?
A. Sabouraud’s B. Endo C. Jensen’s
D. Roux E. Clauberg’s
Virological laboratory has received patient’s nasopharyngeal lavage. What can be used to
single out influenza virus from the patient’s lavage?
A. Chick embryo
B. Endo’s medium
C. Meat infusion agar
D. Meat infusion broth
E. Lowenstein–Jensen medium

299
IMMUNILOGY, CHEMOTHERAPY, GENETICS, INFECTION
Immunity
1. Examination of a child who frequently suffers from infectious diseases revealed that IgG
concentration in blood serum was 10 times less than normal, IgA and IgM concentration was also
significantly reduced. Analysis showed also lack of B-lymphocytes and plasmocytes. What disease are
these symptoms typical for?
A. Bruton’s disease B. Swiss-type agammaglobulinemia
C. Dysimmunoglobulinemia
D. Louis-Bar syndrome E. Di George syndrome
2. Parents of a 5-year-old child report him to have frequent colds that develop into
pneumonias, presence of purulent rashes on the skin. Laboratory tests have revealed the following:
absence of immunoglobulins of any type; naked cells are absent from the lymph nodes punctate.
What kind of immune disorder is it?
A. X-linked hypogammaglobulinemia (Bruton type agammaglobulinemia)
B. Autosomal recessive agammaglobulinaemia (Swiss type)
C. Hypoplastic anemia D. Agranulocytosis E. Louis-Barr syndrome
3. A doctor examined a patient, studied the blood analyses, and reached a conclusion, that
peripheral immunogenesis organs are affected. What organs are the most likely to be affected?
A. Tonsils B. Thymus C. Kidneys
D. Red bone marrow E. Yellow bone marrow
4. A 32-year-old patient has purulent wound in the lower third of forearm. Smear of purulent
wound content has been made. What cells will be generally detected, if it is stained using
Romanovsky-Giemsa stain?
A. Neutrophil B. Eosinophil C. Lymphocyte
D. Erythrocyte E. Basocyte
5. Cellular composition of exudate largely depends on the etiological factor of inflammation.
What leukocytes are the first to be involved in the focus of inflammation caused by pyogenic
bacteria?
A. Neutrophil granulocytes
B. Monocytes
C. Myelocytes
D. Eosinophilic granulocytes
E. Basophils
6. A patient with clinical presentations of immunodeficiency has undergone immunological
tests. They revealed significant decrease in number of cells that form rosettes with sheep
erythrocytes. What conclusion can be drown on the ground of the analysis data?
A. Decrease in T-lymphocyte level B. Decrease in B-lymphocyte level
C. Decrease in natural killer level (NKcells)
D. Decrease in complement system level
E. Lack of effector cells of the humoral immunity
7. A patient with clinical presentations of immunodeficiency went through immunological
examinations. They revealed significant loss of cells that form rosettes with erythrocytes of a ram.
What conclusion can be made according to the analysis data?
A. Decrease of T-lymphocytes rate B. Decrease of B-lymphocytes rate
C. Decrease of natural killer cell rate
D. Decrease of complement system rate
E. Insufficiency of effector cells of humoral immunity
8. Donor skin transplantation was performed to a patient with extensive burns. On the 8-th
day the graft became swollen and changed colour; on the 11-th day graft rejection started. What
cells take part in this process?
A. T-lymphocytes B. Erythrocytes
C. Basophils D. Eosinophils E. B-lymphocytes

300
9. A female patient underwent liver transplantation. 1,5 month after it her condition became
worse because of reaction of transplant rejection. What factor of immune system plays the leading
part in this reaction?
A. T-killers B. Interleukin-1 C. Natural killers
D. B-lymphocytes E. T-helpers
10. A patient with skin mycosis has disorder of cellular immunity. The most typical
characteristic of it is reduction of the following index:
A. T-lymphocytes B. Immunoglobulin G C. Immunoglobulin E
D. B-lymphocytes E. Plasmocytes
11. Recovery from an infectious disease is accompanied by neutralization of antigens by
specific antibodies. What cells produce them?
A. Plasmocytes B. Fibroblasts C. Tissue basophils
D. Eosinophils E. T-lymphocytes
12. Throughout a year a 37-year-old woman periodically got infectious diseases of bacterial
origin, their course was extremely lingering, remissions were short. Examination revealed low level of
major classes of immunoglobulins. The direct cause of this phenomenon may be the following cell
dysfunction:
A. Plasmocytes B. Phagocytes C. Neutrophils
D. Macrophages E. Lymphocytes
13. Loose fibrous connective tissue of salivary glands contains oval average-sized cells which
synthesize antibodies. The cells have round eccentric nucleus and "spoke-wheel" chromatin pattern
made by small clumps of chromatin. What are these cells called?
A. Plasma cells B. Adipocytes C. Neutrophils
D. Fibroblasts E. Macrophages
14. A 37-year-old woman periodically got infectious diseases of bacterial origin, their course
was extremely lingering, remissions were short. Examination revealed low level of major classes of
immunoglobulins. The direct cause of this phenomenon may be the following cell dysfunction:
A. Plasmocytes B. Phagocytes C. Neutrophils
D. Macrophages E. Lymphocytes
15. Blood analysis of a 16-year-old girl suffering from the autoimmune inflammation of
thyroid gland revealed multiple plasmatic cells. Such increase in plasmocyte number is caused by
proliferation and differentiation of the following blood cells:
A. B-lymphocytes B. T-helpers C. Tissue basophils
D. T-killers E. T-supressors
16. Humoral immune response to an antigen results in generation of antibodies produced by
plasmacytes. Plasmacytes arise as a result of immunostimulated division from the following cells of
immune system:
A. B-lymphocytes B. Monocytes
C. Granulocytes D. T-helpers E. T-killers
17. In a patient with clinical signs of immunodeficiency the number and functional activity of
T and B lymphocytes are not changed. Defect with dysfunction of antigen-presentation to the
immunocompetent cells was found during investigation on the molecule level. Defect of what cells is
the most probable?
A. Т-lymphocytes, В-lymphocytes B. 0-lymphocytes
C. Fibroblasts, Т-lymphocytes, В-lymphocytes
D. NK-cells E. Macrophages, monocytes
18. A patient with clinical presentations of primary immunodeficiency displays disturbance of
antigen-presenting function by immunocompetent cells. What cells may have structure defect?
A. Macrophages, monocytes B. T-lymphocytes
C. B-lymphocytes D. Fibroblasts E. 0-lymphocytes
19. Live vaccine is injected into the human body. Increasing activity of what cells of
connective tissue can be expected?
A. Fibroblasts and labrocytes B. Adipocytes and adventitious cells
C. Macrophages and fibroblasts D. Plasmocytes and lymphocytes

301
E. Pigmentocytes and pericytes
20. In order to speed up healing of a wound of oral mucosa a patient was prescribed a drug
that is a thermostable protein occurring in tears, saliva, mother’s milk as well as in a new-laid hen’s
egg. It is known that this protein is a factor of natural resistance of an organism. What is it called?
A. Lysozyme B. Complement C. Interferon D. Interleukin E. Imanine
21. In order to administer general health improving therapy a parodontist intends to study
factors of nonspecific resistance of saliva and mucous secretion. Which of the following factors of
nonspecific resistance should be studied in the first line?
A. Lysozyme B. Secretory IgA
C. Properdin D. Interferon E. Complement
22. Lymphocytes and other cells of our body synthesize universal antiviral agents as a
response to viral invasion. Name these protein factors:
A. Interferon B. Interleukin-2 C. Cytokines
D. Interleukin-4 E. Tumor necrosis factor
23. Examination of patients with periodontitis revealed the interdependence between the
rate of affection of periodontal tissues and the amount of lysozymes in saliva and gingival liquid.
These results can be obtained during studying the following protection system of an organism:
A. Non-specific resistance B. Humoral immunity
C. Cellular immunity D. Autoresponsiveness E. Tolerance
24. Blood serum of a newborn contains antibodies to measles virus. What kind of immunity is
this indicative of?
A. Natural passive B. Natural active C. Artificial passive
D. Artificial active E. Heredoimmunity
25. A patient diagnosed with botulism has been prescribed antibotulinic serum for
treatment. What immunity will be formed in the given patient?
A. Antitoxic passive immunity B. Infection immunity
C. Antitoxic active immunity D. Antimicrobic active immunity
E. Antimicrobic passive immunity
26. For the specific prevention of influenza, the employees of an enterprise were vaccinated
with "Influvac". What type of immunity will develop in the body of the vaccinated?
A. Artificial active B. Innate congenital
C. Artificial passive D. Natural active E. Natural passive
27. In our country, routine preventive vaccinations against poliomyelitis involve using live
vaccine that is administered orally. What immunoglobulins are responsible for the development of
local post-vaccination immunity in this case?
A. Secretory IgA B. IgM C. IgG D. Serum IgA E. IgE
28. Various cells of the oral mucous membrane and antimicrobial substances synthesized by
these cells play an important part in the local immunity of the oral cavity. Specify the key factors for
the local immunity:
A. Secretory IgA B. B-lymphocytes C. IgG D. Macrophages E. Eosinophils
29. A child cut his leg with a piece of glass while playing and was brought to the clinic for the
injection of tetanus serum. In order to prevent the development of anaphylactic shock the serum
was administered by Bezredka method. What mechanism underlies this method of desensitization of
the body?
A. Binding of IgE fixed to the mast cells
B. Blocking the mediator synthesis in the mast cells
C. Stimulation of immune tolerance to the antigen
D. Stimulation of the synthesis of antigenspecific IgG
E. Binding of IgE receptors to the mast cells
30. A 10-year-old child cut his leg with a piece of glass and was sent to a clinic for an anti-
tetanus serum injection. In order to prevent the development of anaphylactic shock, the Besredka
desensitization method was applied. What mechanism underlies this method?
A. Binding to IgE fixed to mast cells
B. Inhibited synthesis of mast cells mediators

302
C. Stimulation of the immunological antigen tolerance
D. Stimulation of antigen-specific IgG2 synthesis
E. Binding of IgE receptors on mast cells.
31. A 10-year-old child cut his leg with a glass shard, when playing, and was delivered to
outpatient department to receive anti-tetanus serum. To prevent development of anaphylactic shock
the serum was introduced by Bezredka method. This method of organism hyposensitization is based
on the following mechanism:
A. Stimulation of antigen-specific IgG2
B. Stimulation of the immunological antigen tolerance
C. Stabilization of mast cell membranes
D. Blocking of mast cell mediators synthesis
E. Binding of mast cell-fixed IgE
32. A 27- year-old woman has dropped penicillin containing eye drops. In few minutes there
appeared feeling of itching, burning of the skin, lips and eyelids edema, whistling cough, decreasing
of BP. What antibodies take part in the development of this allergic reaction?
A. IgA and IgM B. IgM and IgG C. IgM and IgD
D. IgG and IgD E. IgE and IgG
33. Skin samples of a patient with bronchial asthma revealed allergen sensitization of poplar
fuzz. What factor of immune system plays the main part in development of this immunopathological
state?
A. IgE B. IgD C. IgM
D. Sensitized Т-lymphocytes E. –
34. A youth, aged 15, from childhood suffers from atopic dermatitis and allergy to the
shellfish. In the last 3 months after acquiring aquarium fish, rhinitis, conjunctivitis, itching in the nose
developed. What level of immunologic index should be defined in this case?
A. IgE B. IgG C. IgM
D. IgA E. Circulating immunocomplexes
35. A 7-year-old child complains of itching, papular erythematous rash, dry skin. Objectively:
there is lichenification in the popliteal fossae and antecubital spaces. What immunologic indicator if
found in the blood serum will verify the diagnosis (atopic dermatitis)?
A. Total IgE B. Secretory IgA
C. IgM D. IgG E. IgD
36. What condition may develop 15-30 minutes after re-administration of the antigen as a
result of the increased level of antibodies, mainly IgE, that are adsorbed on the surface of target cells,
namely tissue basophils (mast cells) and blood basophils?
A. Anaphylaxis B. Antibody-dependent cytotoxicity
C. Delayed-type hypersensitivity
D. Immune complex hyperresponsiveness E. Serum sickness
37. A 22-year-old woman ate some seafood. 5 hours later the trunk and the distal parts of
limbs got covered with small itchy papules which were partially fused together. After one day, the
rash disappeared spontaneously. Specify the hypersensitivity mechanism underlying these changes:
A. Atopy (local anaphylaxis) B. Systemic anaphylaxis
C. Cellular cytotoxicity D. Immune complex hypersensitivity
E. Antibody-dependent cell-mediated cytolysis
38. A 30-year-old patient has dyspnea fits, mostly at night. He has been diagnosed with
bronchial asthma. What type of allergic reaction according to the Gell-Coombs classification is most
likely in this case?
A. Anaphylactic B. Cytotoxic C. Stimulating
D. Immune complex E. Delayed-type hypersensitivity
39. During surgical manipulations a patient has been given novocaine injection for
anesthesia. 10 minutes later the patient developed paleness, dyspnea, hypotension. What type of
allergic reaction is it?
A. Anaphylactic immune reaction B. Cellulotoxic immune reaction
C. Aggregate immune reaction D. Stimulating immune reaction

303
E. Cell-mediated immune reaction
40. A 50 year old man who was referred to the hospital for treatment of cervical
lymphadenitis underwent test for individual sensitivity to penicillin. 30 seconds after he went hot all
over, AP dropped down to 0 mm Hg that led to cardiac arrest. What type of hypersensitivity reaction
is it?
A. Anaphylactic B. Delayed-type hypersensitivity
C. Complement-mediated cytotoxic
D. Immunocomplex-mediated E. –
41. A 50-year-old patient has been referred for treatment of neck lymphadenitis. His
individual penicillin sensitivity was tested. In 30 seconds fullbody fever raised in the patient and his
arterial blood pressure dropped to 0mm Hg followed by cardiac arrest. Resuscitation was
unsuccessful. Autopsy revealed acute venous hyperemia of viscera. Histological study revealed
mastcells (tissue basocytes) degranulation in the skin (at the area of injections), myocardium and
lungs. What kind of hypersensitivity reaction occurred in the patient?
A. Anaphylactic
B. Delayed-type hypersensitivity
C. Complement-mediated cytotoxic
D. Immune complex-mediated
E. –
42. Several minutes after a dentist administered novocaine for local anaesthesia of a
patient’s tooth, the following symptoms sharply developed in the patient: fatigue, skin itching.
Objectively the following can be observed: skin hyperemia, tachycardia, BP dropped down to 70/40
mm Hg. What kind of allergic reaction is this pathology?
A. Anaphylactic B. Cytotoxic C. Stimulating
D. Cell-mediated immune reaction E. Immune complex
43. A woman complaining of coryza, phonastenia, eyelids redness and lacrymation during
spring period came to the doctor. What type of allergic reaction by Gell ano Coombs classification
develops in this case?
A. Delayed type of hypersensitivity B. Immunocomplex
C. Stimulating D. Cytotoxic E. Anaphylactic
44. A patient who had been suffering for many years from bronchial asthma died from
asphyxia. The histological examination of his lungs revealed the following: much mucus with
eosinophiles contents in the lumen of bronchioles and small bronci, sclerosis of interalveolar septa,
dilation of alveolar lumen. Which of the mechanisms of allergy development is it?
A. Immunocomplex B. Cytolysis, caused by lymphocytes
C. Cytotoxic D. Reagin E. Granulomatosis
45. A patient with paroxysmal attacks of asphyxia, which appear after inhalation of different
aromatic substances has been made a diagnosis of bronchial asthma. Ig E rate is elevated. What type
of reaction is it the most typical for?
A. Anaphylactic B. Delayed type of hypersensitivity
C. Autoimmune D. Cytotoxic E. Immunocomplex
46. A patient suffering from periodical attacks caused by inhalation of different flavoring
substances was diagnosed with atopic bronchial asthma. IgE level was increased. This is typical for
the following type of reactions:
A. Anaphylactic reactions B. Cytotoxic reactions
C. Immunocomplex reactions D. Delayed-type hypersensitivity
E. Autoimmune reactions
47. During anaesthetization of the oral cavity mucous tunic a patient developed anaphylactic
shock (generalized vasodilatation, increase in vascular permeability along with escape of liquid to the
tissues). What type of hypersensitivity has the patient developed?
A. I type (anaphylactic) B. II type (antibody-dependent)
C. III type (immune complex) D. IV type (cellular cytotoxicity)
E. V type (granulomatosis)

304
48. A teenger had his tooth extracted under novocain anaesthesia. 10 minutes later he
presented with skin pallor, dyspnea, hypotension. When this reaction is developed and the allergen
achieves tissue basophils, it reacts with:
A. IgE B. IgА C. IgD D. IgМ E. T-lymphocytes
49. A surgeon used novocaine as an anaesthetic during surgical manipulations. 10 minutes
after it the patient became pale, he got dyspnea and hypotension. What type of allergic reaction is
it?
A. Anaphylactic B. Cytotoxic C. Immune complex
D. Stimulating E. Cell-mediated
50. During anesthesia of the oral mucosa a 37-year-old patient has had anaphylactic reaction
(widespread vasodilation, increased vascular permeability with liquid exiting the blood vessels and
penetrating in the tissues). What type of hypersensitivity reaction occurred in the patient?
A. Type I (anaphylactic) B. Type II (antibody-dependent)
C. Type III (immune complex)
D. Type IV (cell cytotoxicity) E. Type V (granulomatosis)
51. After anaesthetic application during tooth extraction the patient developed marked soft
tissue edema of the upper and lower jaw, skin rash on the face, reddening, and itching. What
pathological process results in such reaction to the anaesthetic?
A. Toxic action of drug
B. Disturbed lymph drainage
C. Allergy
D. Iflammation
E. Circulatory deficiency
52. A patient has been administered conduction anesthesia with novocaine in preparation for
tooth extraction. After the anesthesia administration the patient developed swelling and hyperemy
around the injection site, skin itch, general fatigue, motor agitation. Name the developed
complication:
A. Allergy
B. Idiosyncrasy
C. Tachyphylaxis
D. Drug dependence
E. Inflammation
53. A man with a long-term history of bronchial asthma died from asphyxia. What
mechanism accounts for the development of hypersensitivity reaction?
A. Reagine reaction B. Cytotoxic reaction
C. Immune complex reaction D. Lymphocyte-mediated cytolysis
E. Granulomatosis
In a woman with bronchial asthma, a viral infection provoked a fatal status asthmaticus.
Pulmonary histology shows a spasm and an edema of bronchioles. A marked infiltration with
lymphocytes, eosinophils, and other leukocytes, as well as degranulation of mast cells, can be
observed in their walls. What mechanism of hypersensitivity underlies the described changes?
A. Immune-mediated cytolysis
B. Immune complex mechanism
C. Reaginic hypersensitivity reaction
D. Inflammatory mechanism
E. Autoimmune mechanism
54. A 38 year old patient died during intractable attack of bronchial asthma. Histological
examination revealed mucus accumulations in bronchi’s lumen, a lot of mast cells (labrocytes) in
bronchi’s wall, some of these cells are degranulated, there are also many eosinophils. Name
pathogenesis of these changes in bronchi:
A. Atopy, anaphylaxis B. Cytotoxic, cytolytic effect of antibodies
C. Immune complex mechanism
D. Cell-mediated cytolysis E. Granulematosis

305
55. For the purpose of anaesthetization a patient got injection of local anesthetic. A few
minutes later the patient got dyspnea and tachycardia; he lost consciousness. What type of shock is
it?
A. Anaphylactic B. Cardiogenic C. Haemorrhagic
D. Traumatic E. Burn
56. In order to estimate antibiotic susceptibility of a patient doctors introduced him
intracutaneously 0,2 ml of penicillin solution. Ten minutes after introduction there appeared
hyperemy and edema. What type does this reaction relate to (according to Coomb’s and Gell’s
classification)?
A. Anaphylactic reaction B. Cytotoxic reaction
C. Reaction of Arthus phenomenon type
D. Delayed-type hypersensitivity E. Tuberculine reaction
57. A patient with acute bronchitis has been prescribed sulfanilamide drugs for treatment. In
an hour after administration the patient developed itching and vesicles filled with light transparent
liquid on the face, palms and soles. Name the mechanism of immune response:
A. Reaginic reaction
B. -
C. Antibody-mediated cytolysis
D. Immune complex-mediated hypersensitivity
E. Cell cytotoxicity
58. A few minutes afer repeated introduction of penicillin a patient got dyspnea, tongue
numbness, hyperemia and then skin pallor. The patient also lost consciousness. What is the cause of
such a grave condition?
A. Anaphylactic shock B. Serum sickness C. Hemolytic anemia
D. Acute glomerulonephritis E. Bronchial asthma
59. After a 10-year-old child had been bitten by a bee, he was delivered to a hospital. There
were lip, face and neck edemata. The patient felt hot and short of breath. Objectively: breathing was
laboured and noisy. There were foamy discharges from the mouth, cough. The skin was pale and
cold. There was bradypnoea. Heart sounds were muffled and arrhythmic. Thready pulse was present.
What diagnosis was made by the expert in resuscitation?
A. Anaphylactic shock B. Quincke’s edema C. Bronchial asthma
D. Acute cardiovascular collapse E. Cerebral coma
60. Every year during the plant blossoming a female patient develops acute catarrhal
inflammation of conjunctiva and nasal mucosa that is the clinical presentation of an allergy. These
symptoms relate to the following type of allergic reactions:
A. Anaphylactic B. Cytotoxic C. Immune complex
D. Cell-mediated E. Cellular dysfunction
61. After eating strawberries a child presented with itchy red spots on the skin (hives).
According to the classification of Coombs and Jell this reaction relates to the following type of allergic
reactions:
A. Reagin (anaphylactic) B. Immunocomplex
C. Cytotoxic D. Cell-mediated E. Stimulating
62. 30 minutes after drinking mango juice a child suddenly developed a local swelling in the
area of the soft palate, which impeded swallowing and, eventually, respiration. Mucosa of the
swollen area was hyperemic and painless. Blood test revealed moderate eosinophilia. Body
temperature was normal. Anamnesis states that the elder sister of the child has been suffering from
bronchial asthma attacks. What kind of edema has developed in the child?
A. Allergic
B. Inflammatory
C. Cardiac
D. Alimentary
E. Hepatic

306
63. 1 minute after the patient had been administered penicillin the patient’s arterial pressure
sharply dropped, pulse became thread, cold sweating and clonic convulsions began. Name this
condition.
A. Anaphylactic shock B. Traumatic shock
C. Burn shock D. Septic shock E. Cardiogenic shock
64. A 35 year old woman consulted a doctor about affection of arm skin and lower third of
forearm in form of a large edema, hyperemia, vesiculation and maceration. The disease developed
after using a laundry detergent "Lotos". The patient has been using it for a month. She hasn’t
suffered from dermatological diseases before. What is the most probable diagnosis?
A. Allergic dermatitis B. Dermatitis simplex C. Toxicoallergic dermatitis
D. Microbial eczema E. Localized neurodermatitis
65. A 22-year-old girl has been complaining of having itching rash on her face for 2 days. She
associates this disease with application of cosmetic face cream. Objectively: apparent reddening and
edema of skin in the region of cheeks, chin and forehead; fine papulovesicular rash. What is the most
likely diagnosis?
A. Allergic dermatitis B. Dermatitis simplex
C. Eczema D. Erysipelas E. Neurodermatitis
66. Hemotransfusion stimulated development of intravascular erythrocyte hemolysis. The
patient has the following type of hypersensitivity:
A. II type hypersensitivity (antibodydependent)
B. I type hypersensitivity (anaphylactic)
C. III type hypersensitivity (immune complex)
D. IV type hypersensitivity (cellular cytotoxicity)
E. V type hypersensitivity (granulomatosis)
67. The patient’s condition after blood transfusion has been aggravated by posttransfusion
shock. Name the type of allergic reaction causing this pathology.
A. Cytotoxic B. Anaphylactic C. Immune complex
D. Delayed-type hypersensitivity E. Receptor-mediated
68. During blood transfusion a patient has developed intravascular erythrocyte hemolysis.
What kind of hypersensitivity does the patient have?
A. II type (antibody-dependent) B. I type (anaphylactic)
C. III type (immune complex) D. IV type (cellular cytotoxicity)
E. IV type (granulomatosis)
69. On the 8th day since the patient was inoculated with antitetanic serum because of dirty
wound of his foot he has developed rising temperature up to 380С, pains in the joints, rash and itch.
The blood tests revealed leukopenia and thrombocytopenia. Allergic reaction of what type has
developed in this case?
A. Anaphylactic B. Cytotoxic
C. Delayed type of hypersensitivity
D. Stimulating E. Immunocomplex
70. A 16-year-old adolescent was vaccinated with DTP. In eight days there was stiffness and
pain in the joints, subfebrile temperature, urticarial skin eruption, enlargement of inguinal, cervical
lymph nodes and spleen. What kind of allergic reaction is observed?
A. Immunocomplex B. Hypersensitivity of immediate type
C. Cytoxic D. Hypersensitivity of delayed type E. –
71. A patient has been diagnosed with acute glomerulonephritis that developed after he had
had streptococcal infection. It is most likely that the affection of basal glomerular membrane is
caused by an allergic reaction of the following type:
A. Immune complex B. Anaphylactic
C. Cytotoxic D. Delayed E. Stimulating
72. 10 days after having quinsy caused by beta-hemolytic streptococcus a 6-year-old child
exhibited symptoms of glomerulonephritis. What mechanism of glomerular lesion is most likely in
this case?
A. Immunocomplex B. Cellular cytotoxicity

307
C. Anaphylaxis D. Atopy
E. Antibody-dependent cell-mediated cytolysis
73. A male patient has been diagnosed with acute post-streptococcal glomerulonephritis. It is
most likely that the lesion of the basement membrane of renal corpuscles was caused by the
following allergic reaction:
A. Immune complex B. Anaphylactic
C. Cytotoxic D. Delayed E. Stimulating
74. A 12-year-old child has developed nephritic syndrome (proteinuria, hematuria,
cylindruria) 2 weeks after the case of tonsillitis, which is a sign of affected glomerular basement
membrane in the kidneys. What mechanism is the most likely to cause the basement membrane
damage?
A. Immune complex B. Granulomatous C. Antibody-mediated
D. Reaginic E. Cytotoxic
75. A 30 year old woman has applied a lipstick with a fluorescent substance for a long time.
Then she got a limited erythema and slight peeling on her lip border, later there appeared transversal
striae and cracks. Special methods of microscopic examination of the affected area helped to reveal
sensibilized lymphocytes and macrophages in the connective tissue; cytolysis. What type of
immunological hypersensitivity was developed?
A. IV type (cellular cytotoxicity) B. I type (reaginic)
C. II type (antibody cytotoxicity)
D. III type (immune complex cytotoxicity) E. Granulomatosis
76. A 30 year old woman has been continuously using lipstick with a fluorescent substance
that led to development of a limited erythema on the prolabium, slight peeling, and later small
transversal sulci and fissures. Microscopic examination of the affected zone revealed in the
connective tissue sensibilized lymphocytes and macrophages, effects of cytolysis. What type of
immunological hypersensitivity has developed on the lip?
A. IV type (cellular cytotoxicity) B. I type (reagin type)
C. II type (antibody cytotoxicity)
D. III type (immune complex cytotoxicity) E. Granulomatosis
77. A patient visited a dentist with complaints of redness and edema of his mouth mucous
membrane in a month after dental prosthesis. The patient was diagnosed with allergic stomatitis.
What type of allergic reaction by Gell and Cumbs underlies this disease?
A. Delayed type hypersensitivity B. Cytotoxic
C. Immunocomplex D. Anaphylactic E. Stimulating
78. A woman has been applying a new cosmetic preparation for a week that resulted in eye-
lid inflammation accompanied by hyperemia, infiltration and painfulness. What type of allergic
reaction was developed?
A. IV B. I C. II D. III E. V
79. Tuberculine was injected intracutaneously to the child for tuberculin test. Marked
hyperemia, tissue infiltration developed on the place of injection in 24 hours. What mechanism
caused these modifications?
A. Cells cytotoxity B. Reagin type cytotoxity C. Granuloma formation
D. Immunocomplex cytotoxity E. Antibody cytotoxity
80. 48 hours after performing tuberculin test (Mantoux test) to a child a 10 mm papule
appeared on the spot of tuberculin introduction. What hypersensitivity mechanism underlies these
changes?
A. Cellular cytotoxicity B. Anaphylaxis
C. Antibody-dependent cytotoxicity
D. Immune complex cytotoxicity E. Granulomatosis
81. A 10-year-old child had the mantoux tuberculin test administered. 48 hours later a papule
up to 8 mm in diameter appeared on the site of the injection. What type of hypersensitivity reaction
developed after the tuberculin injection?
A. Type IV hypersensitivity B. Arthus phenomenon C. Seroreaction
D. Atopic reaction E. Type II hypersensitivity reaction

308
82. A 10 year old child was subjected to Mantoux test (with tuberculin). 48 hours later a
papule up to 8 mm in diameter appeared on the site of tuberculin injection. What type of
hyperesponsiveness reaction has developed after tuberculin injection?
A. Hyperresponsiveness type IV
B. Reaction of Arthus phenomenon type
C. Reaction of serum sickness type
D. Atopic reaction E. Hyperresponsiveness type II
83. A 10 year old child underwent Mantoux test (with tuberculin). 48 hours later there
apperaed a papule up to 8 mm in diameter on the site of tuberculin injection. Tuberculin injection
caused the following hypersensitivity reaction:
A. IV type hypersensitivity reaction
B. Arthus reaction C. Seroreaction
D. Atopic reaction E. II type hypersensitivity reaction
84. A 4 year old child had Mantoux test. 60 hours after tuberculin introduction a focal skin
hardening and redness 15 mm in diameter appeared. It was regarded as positive test. What type of
hypersensitivity reaction is this test based upon?
A. Delayed-type hypersensitivity
B. Immune complex-mediated hypersensitivity
C. Complement-mediated cytotoxic hypersensitivity
D. Immediate hypersensitivity E. –
85. A 45-year-old male died from disseminated tuberculosis. On autopsy the symptoms of
tuberculosis were confirmed by both microscopic and histological analyses. What kind of
hypersensitivity reaction underlies the process of granuloma development?
A. Delayed B. Antibody-dependent cytotoxicity
C. Complement-dependent cytotoxicity
D. Anaphylactic E. Immune complex
86. A child with suspected tuberculosis was given Mantoux test. After 24 hours the site of the
allergen injection got swollen, hyperemic and painful. What are the main components that
determine such response of the body?
A. Mononuclear cells, T-lymphocytes and lymphokines
B. Granulocytes, T-lymphocytes and IgG
C. Plasma cells, T-lymphocytes and lymphokines
D. B-lymphocytes, IgM
E. Macrophages, B-lymphocytes and monocytes
87. A child entering the school for the first time was given Mantoux test in order to
determine if there was a need for revaccination. The reaction was negative. What is the meaning of
this test result?
A. No cell-mediated immunity to tuberculosis
B. Availability of cell-mediated immunity to tuberculosis
C. No antibodies to the tuberculosis bacteria
D. No anti-toxic immunity to tuberculosis
E. Presence of antibodies to the tuberculosis bacteria
88. While registering the child to the school Mantoux's test was made to define whether
revaccination was needed test result is negative. What does this result of the test mean?
A. Absence of antitoxic immunity to the tuberculosis
B. Presence of antibodies for tubercle bacillus
C. Absence of antibodies for tubercle bacillus
D. Presence of cell immunity to the tuberculosis
E. Absence of cell immunity to the tuberculosis
89. A 16 y.o. boy from a countryside entered an educational establishment. Scheduled
Manteux test revealed that the boy had negative reaction. What are the most reasonable actions in
this case?
A. To perform BCG vaccination B. To repeat the reaction in a month
C. To perform serodiagnostics of tuberculosis

309
D. To isolate the boy temporarily from his mates
E. To perform rapid Price diagnostics
90. Medical examination of the first-year pupils included Mantoux test. 15 pupils out of 35
had negative reaction. What actions should be taken against children with negative reaction?
A. BCG vaccination B. Antitoxin vaccination
C. Rabies vaccination D. Repeat Mantoux test
E. Examination of blood serum
91. Planned mass vaccination of all newborn 5-7 day old children against tuberulosis plays an
important role in tuberculosis prevention. In this case the following vaccine is applied:
A. BCG
B. Diphteria and tetanus toxoids and pertussis vaccine
C. Diphtheria and tetanus anatoxin vaccine
D. Adsorbed diphtheria vaccine E. –
92. For tuberculosis prevention the newborns got an injection of a vaccine. What vaccine was
used?
A. BCG B. Mantoux C. DTaP vaccine
D. Anatoxin E. Oral polio vaccine (Sabin vaccine)
93. In a maternity hospital a newborn should receive vaccination against tuberculosis. What
vaccine should be chosen?
A. BCG vaccine
B. STI vaccine
C. EV vaccine
D. DPT vaccine
E. Tuberculin
94. There is a suspicion of active tuberculosis development in patient. The doctor has
appointed Mantoux test to make a diagnosis. What immunobiological agent has to be administered?
A. Tuberculine B. BCG vaccine C. DPT vaccine
D. Tularin test E. DT vaccine
95. A 6-year-old child with suspected active tuberculosis process has undergone diagnostic
Mantoux test. What immunobiological preparation was injected?
A. Tuberculin B. BCG vaccine
C. DTP vaccine D. Tularinum E. Td vaccine
96. The first grade pupils went through a medical examination aimed at selection of children
needing tuberculosis revaccination. What test was applied?
A. Mantoux test B. Schick test C. Cutaneous tularin test
D. Burne test E. Anthracene test
97. A virological laboratory obtained pathological material (mucous discharges from nasal
meatuses) taken from a patient with provisional diagnosis "influenza". What quick test will allow to
reveal specific viral antigen in the material under examination?
A. Direct and indirect immunofluorescence test
B. Direct and indirect fluorescence immunoassay
C. Hemagglutination inhibition assay D. Radioimmunoassay E. –
98. During the breakout of acute respiratory infection in order to diagnose influenza the
express-diagnosis, based on revealing of specific viral antigen in the examined material
(nasopharyngial lavage), is carried out. Which reaction is used for this?
A. Immunofluorescence B. Agglutination
C. Precipitation D. Opsonization E. Complement binding
99. Laboratory of extremely dangerous infections received a sample taken from a patient
with assumed cholera. What express-diagnostics method can confirm this diagnosis?
A. Immunofluorescence test
B. Complement binding reaction C. Agglutination test
D. Precipitation reaction E. Hemagglutination reaction

310
100. Serological diagnostics of infectious diseases is based upon specific interaction with
antigens. Specify the serological reaction that underlies adhesion of microorganisms when they are
affected by specific antibodies in presence of an electrolyte:
A. Agglutination reaction B. Precipitation reaction
C. Complement-binding reaction
D. Hemadsorption reaction E. Neutralization reaction
101. A physician is planning to diagnose an infectious disease by means of agglutination test.
What is required for this reaction apart from the serum of a patient?
A. Diagnosticum B. Diagnostic serum
C. Complement D. Hemolytic serum E. Anatoxin
102. To conduct serum diagnostics of typhoid fever a test is carried out, when diagnosticums
of three types of microorganisms are being added into different solutions of patient’s serum; then
agglutinate formation is checked. Name the author of that test.
A. Widal B.Wassermann
C. Ouchterlony D. Wright E. Sachs-Witebsky
103. For serological diagnostics of the whooping cough it was made large-scale reaction with
parapertussis and pertussis diagnosticums. At the bottom of the test-tubes with diagnosticum of
Bordetella parapertussis grain-like sediment formed. What antibodies have this reaction revealed?
A. Bacteriolysins B. Agglutinins
C. Antitoxins D. Opsonins E. Precipitins
104. A large-scale reaction with parapertussis and pertussis diagnosticums was made in order
to make serological diagnostics of the whooping cough. At the bottom of the test-tubes with
diagnosticum of Bordetella parapertussis a granular sediment formed. What antibodies did this
reaction reveal?
A. Agglutinins B.Precipitins C.Opsonins D.Bacteriolysins E.Antitoxins
105. For the purpose of retrospective diagnostics of recent bacterial dysentery it was decided
to perform serological examination of blood serum in order to determine antibody titer towards
Shiga bacilli. What of the following reactions should be applied?
A. Passive hemagglutination B. Bordet-Gengou test
C. Precipitation D. Hemolysis E. Bacteriolysis
106. Retrospective diagnostics of bacterial dysentery involved serological analysis of blood
serum intended for determination of Shigella antibody titer. Which of the following reactions should
be applied for this purpose?
A. Passive haemagglutination B. Complement binding
C. Precipitation D. Haemolysis E. Bacteriolysis
107. In order to establish the level of antidiphtheritic immunity in a child it was decided to
use a passive hemagglutination test. This task can be completed by the sensibilization of erythrocytes
by:
A. Diphtheria anatoxin B. Diphtheria antitoxin
C. Diphtheria bacillus antigens D. Antidiphtheric serum E. -
108. At a bacteriological laboratory animal skins are analyzed by means of Ascoli precipitaion
test. What is detected if the reaction is positive?
A. Anthrax agent antigens B. Anaerobic infection toxin
C. Brucellosis agent D. Yersinia surface antigen
E. Plague agent
109. The person was selling "homemade pork" sausages on the market. State sanitary
inspector suspected falcification of the sausages. With help of what serological immune reaction can
food substance be identified?
A. Immunofluorescence test B. Indirect hemagglutination test
C. Agglutination test D. Precipitation test
E. Complement- fixation test
110. A forensic laboratory received clothes of a citizen, who a day before was reported
missing. The clothes were found in a shed, there are red stains identified as blood by an expert. What
reaction should be performed to determine whether these red stains are dried human blood?

311
A. Complement binding
B. Enzyme immunoassay
C. Agglutination
D. Flocculation
E. Circular precipitation
111. In order to determine toxigenicity of diphtheria bacilli a strip of filter paper impregnated
with antitoxic diphtheria serum was put on the dense nutrient medium. There were also inoculated a
microbial culture under examination and a strain that is known to be toxigenic. If the microbial
culture under examination produces exotoxin, this will result in formation of:
A. Precipitin lines B. Haemolysis zones
C. Zones of diffuse opacification
D. Zones of lecithovitellinous activity E. Precipitin ring
112. In order to estimate toxogenicity of diphtheria agents obtained from patients the
cultures were inoculated on Petri dish with nutrient agar on either side of a filter paper strip that was
put into the center and moistened with antidiphtheric antitoxic serum. After incubation of
inoculations in agar the strip-like areas of medium turbidity were found between separate cultures
and the strip of filter paper. What immunological reaction was conducted?
A. Precipitation gel reaction B. Coomb’s test
C. Agglutination reaction D. Rings precipitation reaction
E. Opsonization reaction
113. A 7 year old girl was taken to an infectious diseases hospital. She had complaints of high
temperature, sore throat, general weakness. A doctor assumed diphtheria. What will be crucial proof
of diagnosis after defining pure culture of pathogenic organism?
A. Toxigenity test B. Detection of volutine granules
C. Cystinase test D. Hemolytic ability of pathogenic orhanism
E. Phagolysability
114. When examining a child the dentist found the deposit on both tonsils and suspected
atypical form of diphtheria. A smear was taken, and after the nutrient media inoculation the toxicity
of the isolated pure culture was determined. What reaction was used to determine the toxigenicity
of the isolated strain of diphtheria bacillus?
A. Gel precipitation reaction B. Agglutination reaction on a glass slide
C. Complement binding reaction
D. Hemolysis reaction E. Ring precipitation reaction
115. A patient has pure culture of diphtheria corynebacteria. What immunological reaction
should be used in order to determine bacteria toxigenity?
A. Precipitation in agar B. Agglutination
C. Complement binding D. Inhibition of hemagglutination
E. Indirect hemagglutination
116. A pregnant woman applied to a doctor with complaints typical for toxoplasmosis. The
doctor took a sample of her blood. What serological tests should be performed in this case?
A. Complement binding assay
B. Precipitation test C. Neutralization test
D. Widal’s test E. Wassermann test
117. A patient who came to the doctor because of his infertility was administered to make
tests for toxoplasmosis and chronic gonorrhoea. Which reaction should be performed to reveal
latent toxoplasmosis and chronic gonorrhoea in this patient?
A. RIHA - Reverse indirect hemagglutination assay
B. IFA - Immunofluorescence assay
C. RDHA - Reverse direct hemagglutination assay
D. Immunoblot analysis E. (R)CFT- Reiter's complement fixation test
118. Researchers of a bacteriological laboratory examine tinned meat for botulinic toxin. For
this purpose a group of mice was injected with an extract of the material under examination and
antitoxic antibotulinic serum of A, B, E types. A control group of mice was injected with the same
extract but without antibotulinic serum. What serological reaction was applied?

312
A. Neutralization B. Precipitation C. Complement binding
D. Opsonocytophagic E. Double immune diffusion
119. Bacteriological laboratory examines canned meat whether it contains botulinum toxin.
For this purpose an extract of test specimen and antitoxic antibotulinic serum of A, B, E types were
introduced to a group of mice under examination; a control group of mice got the extract without
antibotulinic serum. What serological reaction was applied?
A. Neutralization B. Precipitation C. Complement binding
D. Opsono-phagocytic E. Double immune diffusion
120. A patient has been hospitalized with provisional diagnosis of botulism. What serological
reaction should be used to reveal botulinum toxin?
A. Neutralization reaction B. Agglutination reaction
C. Bordet-Gengou test D. Precipitation reaction
E. Immunofluorescence test
121. Antigens of Sonne shigella placed on the objects of outdoor environment and foodstuffs
can be revealed by means of a certain test with application of a diagnostic test system that includes a
polystyrene tray with adsorbed specific antibodies. What reaction is it?
A. Immune-enzyme assay B. Immunofluorescence test
C. Passive inverse hemagglutination test
D. Direct hemagglutination test E. Immunoelectrophoresis test
122. In case of many infectious diseases patient’s blood may contain antigens of causative
agents. What reaction should be applied provided that the level of antigenemia is low?
A. Enzyme-linked immunosorbent assay
B. Agglutination test C. Indirect hemagglutination test
D. Latex agglutination test E. Immunoelectrophoresis
123. During many infectious diseases patient’s blood may contain antigenes of pathogens.
What reaction should be applied provided that antigenemia is at a low level?
A. Enzyme-linked immunosorbent assay
B. Agglutination reaction C. Indirect hemagglutination
D. Latex-agglutination E. Immunoelectrophoresis
124. Mass serological diagnosis of HIV infection is made by means of enzymelinked
immunosorbent assay techniques. What standard component of the reaction must be adsorbed on
the solid phase of the test system?
A. HIV antigens B. Monoclonal HIV antibodies
C. Enzyme-marked HIV antibodies
D. Specific immunoglobulins E. Substrates to determine enzyme
125. A patient has been hospitalized with provisional diagnosis of virus B hepatitis.
Serological reaction based on complementation of antigen with antibody chemically bound to
peroxidase or alkaline phosphatase has been used for disease diagnostics. Name this serological
reaction:
A. Immune-enzyme analysis B. Radioimmunoassay technique
C. Immunofluorescence test D. Bordet-Gengou test
E. Antigen-binding assay
126. Professional dentists belong to the risk group concerning professional infection with
viral hepatitis type B. Name an effective method for active prevention of this disease among the
dentists:
A. Vaccination with recombinant vaccine
B. Secure sterilization of medical instruments
C. Working with gum gloves on
D. Introduction of specific immunoglobuline
E. Introduction of interferonogenes
127. In order to eliminate occupational risks dental workers underwent vaccination. The
vaccine should protect them from a viral infection, whose pathogen may be found in blood of dental
patients who had had this infection or who are its chronic carriers. What vaccine was used?
A. Genetically engineered HBs antigen

313
B. Live measles vaccine C. Inactivated hepatitis A vaccine
D. Anti-rabies vaccine E. Subunit influenza vaccine
128. Specific prophylaxis involved application of a vaccine containing microorganisms and
exotoxin detoxicated by formalin. It relates to the following type of vaccine:
A. Combined B. Genetically engineered
C. Anatoxin D. Chemical E. Live
129. To prevent the seasonal influenza epidemics in the city hospitals, sanitary epidemic
station gave orders to immunize health care workers. Which of the following preparations should be
used for immunization?
A. Subunit vaccine B. Interferon C. Gamma-globulin
D. Rimantadine E. Amantadine
130. Bacterioscopic examination of a smear from the pharynx of a diphtheria suspect
revealed bacilli with volutine granules. What etiotropic drug should be chosen in this case?
A. Antidiphtheritic antitoxic serum B. Bacteriophage
C. Diphtheritic anatoxin D. Eubiotic E. Interferon
131. A patient with suspected diphtheria went through bacterioscopic examination.
Examination of throat swab revealed rod-shaped bacteria with volutin granules. What etiotropic
preparation should be chosen in this case?
A. Antidiphtheric antitoxic serum B. Bacteriophage
C. Diphtheria antitoxin D. Eubiotic E. Interferon
132. It is necessary to carry out preventive vaccination of a student group because of an
occurrence of diphtheria. Which preparation should be used for the creation of the artificial active
immunity?
A. Diphtheria anatoxin B. Specific immunoglobulin
C. DTP vaccine D. Inactivated bacteria vaccine
E. Anti-diphtheria serum
133. What preventive medications should be injected to a patient with open maxillofacial
trauma provided that he has never got prophylactic vaccination before?
A. Antitetanus immunoglobulin and anatoxin
B. Anticonvulsive drugs and anatoxin
C. Antitetanus serum and antibiotics
D. Diphtheria, tetanus toxoids and pertussis vaccine and antibiotics
E. Tetanus anatoxin and antibiotics
134. A laboratory received a food product that had been taken from the focus of food
poisoning and presumably contained botulinum toxin. To identify the type of toxin, the neutralization
reaction must be performed on white mice. What biological product is used in this reaction?
A. Antitoxic serum B. Normal serum
C. Antibacterial serum D. Diagnosticum E. Allergen
135. Vaccination is done by means of a toxin that has been neutralized by a formaldehyde
(0,4%) at a temperature 37 – 400C for four weeks. Ramond was the first to apply this preparation for
diphtheria prophylaxis. What preparation is it?
A. Anatoxin B. Immunoglobulin
C. Antitoxic serum D. Adjuvant E. Inactivated vaccine
136. Diphtheria exotoxin had been treated with 0,3-0,4% formalin and kept in a thermostat
for 30 days at a temperature of 40oC.What preparation was obtained as a result of these
manipulations?
A. Anatoxin B. Antitoxin C. Diagnosticum
D. Therapeutic serum E. Diagnostic serum
137. Biological preparations are subdivided into groups according to their purpose and
production principles. What group do the preparations for initiation of active immunity relate to?
A. Vaccines B. Immune sera C. Immunoglobulins
D. Monoclonal antibodies E. Bacteriophages
138. A pharmaceutical company received from a laboratory a delivery order of diagnostic
medications used for studying antigenic properties of causative agent. Name these preparations:

314
A. Diagnostic sera B. Allergens C. Diagnosticums
D. Immunoglobulins E. Bacteriophages
139. In an inhabited locality there is an increase of diphtheria during the last 3 years with
separate outbursts in families. What measure can effectively influence the epidemic process of
diphtheria and reduce the morbidity rate to single cases?
A. Immunization of the population
B. Hospitalization of patients C. Detection of carriers
D. Early diagnostics E. Disinfection in disease focus
140. An 11-year-old girl has been immunized according to her age and in compliance with the
calendar dates. What vaccinations should the children receive at this age?
A. Diphtheria and tetanus B. TB C. Polio D. Hepatitis B E. Pertussis
141. One of mass production drugs is produced by inactivation of bacterial exotoxin by
formalin. What is this drug for?
A. For active immunization B. For serodiagnostic assay
C. For passive immunization D. For toxinemia treatment
E. For immunocorrection
142. Anti-tetanus gamma globulin is produced by hyperimmunization of donors with tetanus
anatoxin. What class of immunoglobulins prevails in this preparation?
A. IgG B. IgA C. IgM D. IgE E. IgD
143. Vaccines are the artificial or natural preparations produced from bacteria, viruses and
other microorganisms, their chemical components and waste products. They are used for the active
immunization of humans and animals for the prevention and treatment of infectious diseases. The
attenuated vaccines consist of:
A. Viable microbes B. Dead microbes C. Anatoxin
D. Dead microbes and toxoid E. Immunoglobulins
144. A person has been in contact with influenza patient. What drug should be administered
for specific passive influenza prophylaxis?
A. Antigrippal immunoglobulin
B. Vaccine influenza virus inactivated
C. Leukocytic interferon D. Amizon E. Anaferon
145. A patient has been diagnosed with ARVI. Blood serum contains immunoglobulin M.
What stage of infection is it?
A. Acute B. Prodromal C. Incubation
D. Reconvalescence E. Carriage
146. A pregnant woman was detected with IgM to rubella virus. An obstetrician-gynecologist
recommended therapeutic abortion due to the high risk of teratogenic affection of the fetus.
Detection of IgM was of great importance as it is these specific immunoglobulins that:
A. Indicate recent infection B. Penetrate placental barrier
C. Have the largest molecular weight
D. Are associated with anaphylactic reactions
E. Are the main factor of antiviral protection
147. Protective function of saliva is based on several mechanisms, including the
presence of enzyme that has bactericidal action and causes lysis of complex capsular
polysaccharides of staphylococci and streptococci. Name this enzyme:
A. Lysozyme B. β-glucuronidase C. Oligo-l,6-glucosidase
D. Collagenase E. α-amylase
148. A patient with clinical signs of a primary immunodeficiency has functionally
disturbed mechanism of antigen-presentation to the immunocompetent cells. What cells are
likely to have structural defects?
A. T-Iymphocyte B. B-lymphocyte C. O-lymphocytes
D. Fibroblasts E. Macrophages, monocyte
149. Significant shortcoming of microscopy in infection diagnostics is its insufficient
information value due to morphological similarity between many species of microorganisms. What
immunoassay can significantly increase informativity of this method?

315
A. Fluorescence immunoassay. B. Radioimmunoassay.
C. Immune-enzyme assay. D. Coomb’s test. E.
Opsonization.
150. First-year schoolchildren have received tuberculin skin test (Mantoux test) at the school
nurse’s office. The purpose of this test was:
A. To detect parotitis in schoolchildren
B. To measure allergization rate toward rickettsia
C. To measure immune stress toward diphtheria
D. To determine the children that need to receive BCG vaccination
E. To preventively vaccinate against tuberculosis
160. A toxin neutralized with 0.4% formaldehyde under 37-40°C for 4 weeks is used
for
vaccination. This preparation was first used by Gaston Ramon for diphtheria
prevention. Name this preparation:
A. Immunoglobulin B. Anatoxin C. Antitoxic serum
D. Inactivated vaccine E. Adjuvant
170. Preventive vaccination against poliomyelitis is made with inactivated vaccine
introduced parenterally. What immunoglobulins create the postvaccinal immunity in this
case?
A. Serum. IgA, IgM B. IgE, IgM C. IgM, secretory IgA
D. IgM, IgG E. IgG, secretory IgA
180. The patient’s saliva has been tested for antibacterial activity. What saliva component
has antibacterial properties?
A. Lysozyme B. Ceruloplasmin C. Cholesterol D. Parotin E.
Amylase
181. A 10-year-old boy is brought to the physician by his parents because of fever, cough,
and fatigue. He has been admitted to the hospital five times because of pneumonia. Attempts to
induce immunity using a pneumococcal vaccine have failed. The first hospitalization was at the age of
12 months. Laboratory findings show marked reduction in all classes and subclasses of serum
immunoglobulins. Which of the following immune cells is most likely to be reduced in the peripheral
blood of this patients?
A. T-cells B. Neutrophils
C. Macrophages D. B-cells E. NK-cells
182. A 5-year-old child is diagnosed with Bruton’s disease (X-linked agammaglobulinemia)
that manifests itself in severe clinical course of bacterial infections and absence of B lymphocytes and
plasma cells. What changes of immunoglobulin content can be observed in blood serum of the child
with immunodeficiency?
A. Decreased IgA, IgM B. Increased IgA, IgM
C. Increased IgD, IgE D. No changes E. Decreased IgD, IgE
183. What condition may develop 15 30 minutes after re-administration of the antigen as a
result of the increased level of antibodies, mainly IgE, that are adsorbed on the surface of target cells,
namely tissue basophils (mast cells) and blood basophils?
A. Anaphylaxis B. Immune complex hyperresponsiveness
C. Delayed-type hypersensitivity D. Serum sickness
E. Antibody-dependent cytotoxicity
184. A 30-year old patient has dyspnea fits, mostly at night. He has been diagnosed with
bronchial asthma. What type of aller reaction according to the Gell-Coombs classification is most
likely in this case?
A. Anaphylactic B. Immune complex
C. Delayed-type hypersensitivity D. Stimulating
E. Cytotoxic
185. You work with the following specimens: 1) brucellosis topical vaccine; 2) leptospirosis
vaccine; 3) BCG vaccine; 4) adsorbed diphtheria-tetanus pertusisis vaccine (DPT vaccine); 5) tetanus
toxoid adsorbed. What kind of immunity do they produce?
316
A. Artificial active immunity B. Antibacterial immunity
C. Artificial passive immunity D. Non-sterilizing (infectious) immunity
E. Antitoxic immunity
186. Examination revealed that the patient has an insufficient immunoglobulin count. The
likely cause of this finding is a dysfunction of the following immune system cells?
A. Plasmablasts B. T-killers C. T-supressors D. T-helpers E. Plasma cells

Special
1. At the laboratory experiment the leukocyte culture was mixed with staphylococci.
neutrophile leukocytes engulfed and digested bacterial cells. This process is termed:
A. Phagocytosis
B. Pinocytosis
C. Diffusion
D. Facilitated diffusion
E. Osmosis
2. During the repeated Widal’s agglutination test it was noticed that the ratio of antibody
titers and O-antigens S.typhi in the patient’s serum had increased from 1:100 to 1:400. How would
you interpret these results?
A. The patient has typhoid fever
B. The patient is an acute carrier of typhoid microbes
C. The patient is a chronic carrier of typhoid microbes
D. The patient previously had typhoid fever
E. The patient was previously vaccinated against typhoid fever
3. A patient was brought into the infectional diseases hospital on the 8th day since the
disease onset. The patient complains of headache, malaise, and weakness. A sample of blood
was taken for the serological test. Widal agglutination test results with blood sample diluted
1:200 and typhoid fever O-diagnosticum were positive. What diagnosis can be made based on
the results of this test?
A. Typhoid fever
B. Leptospirosis
C. Tuberculosis
D. Dysentery
E. Cholera
4. To conduct serum diagnostics of typhoid fever a test is carried out, when diagnosticums of
three types of microorganisms are being added into different solutions of patient’s serum; then
agglutinate formation is checked. Name the author of that test.
A. Widal
B. Wassermann
C. Ouchterlony
D. Wright
E. Sachs-Witebsky
5. A child with diphtheria 10 days after injection of antitoxic antidiphtherial serum has
developed skin rash, accompanied by severe itch, rising temperature up to 38 0C and joints pain.
What is the cause of these symptoms?
A. Delayed type of hypersensitivity
B. Anaphylacsis
C. Contact allergy
D. Atopia
E. Serum sickness

317
6. A toxin neutralized with 0.4% formaldehyde under 37-40°C for 4 weeks is used for
vaccination. This preparation was first used by Gaston Ramon for diphtheria prevention.
Name this preparation:
A. Immunoglobulin
B. Anatoxin
C. Antitoxic serum
D. Inactivated vaccine E. Adjuvant
7. What diagnostic method should be used in industry to test the raw leather for presence of
B. antracis?
A. Microscopy with Burry-Gins stain
B. Microscopy with Aujeszky stain
C. Ascoli's thermo precipitation test
D. Bacteriological analysis
E. Serological test
8. There was a record of some anthrax cases among animals in a countryside. The spread of
disease can be prevented by means of immunization. What kind of vaccine should be used?
A. STI live vaccine
B. BCG vaccine
C. Salk vaccine
D. Sabin’s vaccine
E. Diphteria and tetanus toxoids and pertussis vaccine
9. In a village, a case of anthrax had been registered. Medical services began
epidemiologically indicated specific prophylaxis of population against anthrax. What preparation was
used for this purpose?
A. Live vaccine B. Inactivated vaccine
C. Chemical vaccine D. Genetically engineered vaccine
E. Anatoxin
10. An infectious diseases hospital admitted a veterinarian with assumed brucellosis. What
serologic test can confirm this diagnosis?
A. Wright’s agglutination reaction
B. Widal’s agglutination reaction
C. Ascoli’s precipitation reaction
D. Weigl’s agglutination reaction
E. Wassermann reaction of complement binding
11. A patient diagnosed with botulism has been prescribed antibotulinic serum for
treatment. What immunity will be formed in the given patient?
A. Antitoxic passive immunity B. Infection immunity
C. Antitoxic active immunity D. Antimicrobic active immunity
E. Antimicrobic passive immunity
12. Pathological material (mucosal excretion from the nasal passages) taken obtained from a
patient provisionally diagnosed with influenza was delivered to the virological laboratory. What quick
test allows detecting specific viral antigen in the investigated material?
A. Direct and indirect immunofluorescence (IF)
B. Reverse indirect haemagglutination (RIHA)
C. Radioimmunoassay (RIA)
D. Direct and indirect enzyme-linked immunosorbent assay (ELISA)
E. Hemagglutination inhibition assay (HAI)
13. A drugstore received a supply of a drug that is widely used for treatment of many virus
diseases since it is not virus specific. What drug is it?
A. Interferon B. Remantadin C. Metisazone
D. Immunoglobulin E. Vaccine
14. Preventive vaccination against poliomyelitis is made with inactivated vaccine
introduced parenterally. What immunoglobulins create the postvaccinal immunity in this
case?
318
A. Serum. IgA, IgM B. IgE, IgM C. IgM, secretory IgA
D. IgM, IgG E. IgG, secretory IgA
15. Dentists have high risk of contracting viral hepatitis type B in the course of their duties
and therefore are subject to mandatory vaccination. What vaccine is used in such cases?
A. Recombinant vaccine. B. Live vaccine. C. Anatoxin.
D. Inactivated vaccine. E. Chemical vaccine.
16. To determine the functional activity of blood corpuscles, a suspension of microorganisms
was introduced into the test tube with packed white cells. In this case, the cytoplasm of some cells
will contain phagocytized microorganisms. Which of the following cell types will show phagocytized
microorganisms?
A. Neutrophils and monocytes B. Lymphocytes and eosinophils
C. Monocytes and lymphocytes D. Lymphocytes and basophils
E. Lymphocytes and neutrophils
17. Some unicellular organisms, i.e. amoebae, feed via phagocytosis. What cells of the human
body use this method not as a means of feeding, but as a defensive mechanism against foreign
bodies (microorganisms, dust, ect.)?
A. Leucocytes B. Erytrocytes C. Myocytes D. Platelets E. Epithelial cells
18. Dental iplants were installed in a patient. Three weeks later, implant rejection occurred.
What blood cells play the largest role in this pathological process?
A. T lymphocytes B. Immunoglobulin E
C. B lymphocytes D. Immunoglobulins M E. Plasmacytes
19. A 20-year-old woman with intestinal polyposis has history of frequent fungal and viral
diseases. What part of the immune system is most likely to be deficient in this case?
A. Complement B. B-lymphocytes
C. Natural killers D. T-lymphocytes E. Phagocytes
20. A 16-year-old boy from the rural area entered the technical school. During a regular
Mantoux test, it turned out that this boy had a negative reaction. What tactics should the doctor
choose as the most rational in this case?
A. Repeat the test in a month B. Serodiagnosis of tuberculosis
C. Urgent isolation of the boy from his groupmates D. BCG vaccination
E. Express diagnostics of tuberculosis using the Price method

319
Chemotherapy
1. A patient with streptococcal infection of gums was prescribed a drug that contained beta-
lactam ring in its structure. Which drug relates to this group?
A. Benzylpenicillin B. Rifampicin
C. Erythromycin D. Streptomycin sulfate E. Chloramphenicol
2. A patient with streptococcal gingival infection was prescribed a medication that contains
beta lactam ring in its structure. What preparation belongs to this group?
A. Benzylpenicillin B. Rifampicin
C. Erythromycin D. Streptomicin sulfate E. Chloramphenicol
3. A patient suffering from syphilis was prescribed a drug the action of which based upon
disturbed generation of murein leading to death of the causative agent. What drug is it?
A. Benzylpenicillin sodium salt B. Bijochinol
C. Ciprofloxacin D. Azithromycin E. Doxycycline
4. To treat bronchitis the patient was prescribed a beta-lactam antibiotic. Its mechanism of
action is based on inhibition of murein production, which results in death of the causative agent.
Name this drug:
A. Azithromycin
B. Penicillin G Sodium Salt
C. Ciprofloxacin
D. Streptomycin
E. Bijochinol (Quinine bismuth iodine)
5. A 60-year-old patient was hospitalized to the surgical department because of infection
caused by blue pus bacillus (Pseudomonas aeruginosa) which is sensative to penicillin antibiotics.
Indicate which of the given penicillins has marked activity to the Pseudomonas aeruginosa?
A. Carbenicillin disodium B. Benzylpenicillin
C. Phenoxymethylpenicillin D. Oxacillin E. Methicillin
6. A 43-year-old patient is to be administered an antibiotic from the penicillin group which
would be resistant to penicillinase. What drug can be recommended?
A. Oxacillin B.Amoxicillin C.Carbenicillin D.Azlocillin E.Ampicillin
7. A patient has been diagnosed with bacillary dysentery. What drug of those listed below
should be prescribed?
A. Amoxicillin B. Benzylpenicillin sodium salt
C. Isonicotinic acid hydrazide (Isoniazid)
D. Itraconazole E. Acyclovir
8. A patient suffers from a severe life-threatening generalised septic infection. What group of
chemotherapeutical drugs should be prescribed in this case?
A. Cephalosporines B. Tetracyclines C. Sulfanilamides
D. Chloramphenicol group E. Macrolides
9. From urine of a 14-year-old boy with the exacerbation of secondary obstructive
pyelonephritis Pseudomonas aeruginosa was isolated with a titer of 1000000 microbes per 1 ml.
Which antibiotic is most advisable to be administered in this case?
A. Ciprofloxacin B. Ampicillin C. Cefazolinum
D. Azithromycin E. Chloramphenicol
10. A patient has been diagnosed with sepsis. It was decided to treat him with a drug from
the fluoroquinolone group. Specify this drug:
A. Ciprofloxacin B. Cefpirome C. Metronidazole
D. Ampicillin E. Cephalexin
11. Gonorrhoea was revealed in the patient on bacterioscopy of the smear from urethra.
Taking into account that medecines for gonorrhoea are fluorquinolones, patient should be
prescribed:
A. Ciprofloxacin B. Fluorouracil C. Cefazoline
D. Urosulfan E. Furazolidone
12. A patient has been diagnosed with gonorrhea. As fluoroquinolones are the drugs of
choice for treatment of gonorrhea the patient should be prescribed:

320
A. Ciprofloxacin
B. Furazolidone
C. Fluorouracil
D. Sulfacarbamide (Urosulfanum)
E. Cefazolin
13. A 54-year-old patient complains of frequent painful urination, chills, fever up to 38oC.
Urine test results: protein - 0,33 g/L, WBCs - up to 50-60 in the field of vision, RBCs - 5-8 in the field of
vision, gram-negative bacilli. Which of the listed antibiotics should be preferred in this case?
A. Ciprofloxacin B. Oxacillin C. Erythromycin
D. Tetracycline E. Tseporin
14. Patient with pneumonia has intolerance to antibiotics. Which of the combined
sulfanilamide medicines should be prescribed to the patient?
A. Biseptol B. Streptocid C. Aethazol
D. Natrium sulfacyl E. Sulfadimethoxine
15. A 30-year-old patient with pneumonia has been administered a 3-day course of an
antibiotic from the group of azalides that has bactericidal effect, prolonged action, the ability to bind
to phagocytic cells and accumulate in the infection foci. What drug has been administered?
A. Azithromycin B. Erythromycin C. Isoniazid
D. Benzylpenicillin sodium salt E. Ciprofloxacin
16. A 5-year-old child has been diagnosed with acute right distal pneumonia. Sputum
inoculation revealed that the causative agent is resistant to penicillin, but it is sensitive to macrolides.
What drug should be prescribed?
A. Azithromycin B. Tetracycline C. Gentamycin
D. Streptomycin E. Ampicillin
17. An infectious patient manifests sensibilization to penicillin. Which of the following
antibiotics is the safest to be applied in this case?
A. Erythromycin B. Bicillin C. Ampicillin D. Amoxicillin E. Oxacillin
18. A 1,5 y.o. child fell seriously ill: chill, body temperature rise up to 40,1 0C, then rapid
dropping to 36, 20 C, skin is covered with voluminous hemorrhagic rash and purple cyanotic spots.
Extremities are cold, face features are sharpened. Diagnosis: meningococcosis, fulminant form,
infection-toxic shock. What antibiotic must be used at the pre-admission stage?
A. Soluble Levomycetine succinate B. Penicillin
C. Lincomycin D. Gentamycin E. Sulfamonometoxin
19. A 26-year-old female patient with bronchitis has been administered a broad spectrum
antibiotic as a causal treatment drug. Specify this drug:
A. Doxycycline B. Interferon C. BCG vaccine
D. Ambroxol E. Dexamethasone
20. A patient with mandibular osteomyelitis has been administered an antibiotic from the
tetracycline group. Specify this drug:
A. Doxycycline hydrochloride B. Rifampicin
C. Streptomycin D. Oxacillin E. Amikacin
21. A stomatologists examined first-grade pupils and revealed that one of children had
yellowish brown teeth, two of them were split. Heretofore the pupil was treated with "some pills" on
account of pneumonia. What medication could have such a negative effect upon teeth?
A. Doxycycline B. Oxacillin C. Erythromycin
D. Ampicillin E. Biseptol
22. A stomatologists examined first-grade pupils and revealed that one of children had
yellowish brown teeth, two of them were split. Heretofore the pupil was treated with "some pills" on
account of pneumonia. What medication could have such a negative effect upon teeth?
A. Doxycycline B. Oxacillin C. Erythromycin
D. Ampicillin E. Biseptol
23. A patient with acne has been prescribed doxycycline hydrochloride. What
recommendations should be given to the patient, while he is taking this drug?
A. Avoid long stay in the sun

321
B. Take with large quantity of liquid, preferably milk
C. Take before meal
D. The course of treatment should not exceed 1 day
E. Do not take with vitamins
24. Administration of doxycycline hydrochloride caused an imbalance of the symbiotic
intestinal microflora. Specify the kind of imbalance caused by the antibiotic therapy:
A. Dysbacteriosis B. Sensibilization C. Idiosyncrasy
D. Superimposed infection E. Bacteriosis
25. The patient with pneumonia was treated with antibiotics for a long period. After
treatment patient complains of frequent and watery stool, abdominal pain. What is the reason of
intestine function disorder?
A. Intestinal disbacteriosis development
B. Antibiotics toxic influence on the GIT
C. Autoimmune reaction development
D. Bacteria toxins influence E. Hereditary enzyme defect
26. As a result of durative antibiotic therapy a 37-year old patient developed intestinal
dysbacteriosis. What type of drugs should be used in order to normalize intestinal microflora?
A. Eubiotics B. Sulfanilamides C. Bacteriophages
D. Autovaccines E. Vitamins
27. A patient underwent appendectomy. In the postoperative period he has been taking an
antibiotic. The patient complains about hearing impairment and vestibular disorders. What group of
antibiotics has such by-effects?
A. Aminoglycosides B. Penicillins
C. Tetracyclines D. Macrolides E. Cephalosporins
28. Mother of a 2 year old child consulted a stomatologist. In the period of pregnancy she
was irregularly taking antibiotics for an infectious disease. Examination of the child revealed incisor
destruction, yellow enamel, brown rim around the dental cervix. What drug has apparent teratogenic
effect?
A. Doxacycline B. Furosemide C. Ampiox
D. Xantinol nicotinate E. Octadine
29. Mother of a two year old child consulted a dentist. In the period of pregnancy she was
non-systematically taking antibiotics to treat an infectious disease. The child’s examination revealed
incisor destruction, yellow enamel, brown limbus of dental cervix. What preparation was mother
taking during her pregnancy?
A. Doxycycline B. Furosemide C. Ampiox
D. Xanthinol nicotinate E. Octadine
30. A 5 y.o. child’s enamel and dentin are striated with yellowish-brown stripes, the child has
also dentin exposure, multiple caries. It is known that the child’s mother had been taking antibiotics
during her pregnancy. What medication might have caused such by-effect?
A. Tetracycline B.Lincomycin C.Streptocid D.Nystatin E.Ampicillin
31. A 7 year old child is ill with bronchitis. It is necessary to administer him an antibacterial
drug. What drug of fluoroquinolone group is CONTRAINDICATED at this age?
A. Ciprofloxacin B. Ampicillin C. Amoxicillin
D. Sulfadimethoxine E. Ampiox
32. Patient was admitted to the infection unit with diagnosis of bacterial dysentery. On
laboratory studies it was revealed that causative element is sensitive to the many antimicrobial
medicines, but patient has anemia. What medicine is contra-indicated to the patient?
A. Enteroseptol B. Phthalazol C. Levomycetin
D. Ampicillin E. Furazolidone
33. A 50-year-old patient with typhoid fever was treated with Levomycetin, the next day his
condition became worse, temperature rised to 39,60С. What caused worthening?
A. Reinfection B. Allergic reaction
C. Irresponsiveness of an agent to the levomycetin
D. The effect of endotoxin agent E. Secondary infection addition

322
34. Purulent endometritis developed in a woman after delivery. Treating with antibiotics
inhibitors of murein synthesis was ineffective. Wide spectrum bactericidal antibiotic was
administered to her. In 6 hours temperature rapidly increased up to 400C with shiver. Muscle pains
have appeared. BP dropped down to 70/40 mmHg. Oliguria has developed. What is the main reason
for the development of this condition?
A. Endotoxic shock B. Toxic effect of preparation
C. Internal bleeding D. Anaphylactic shock E. Bacteremia
35. In order to prevent wound infection associated with surgical procedures a patient was
given a synthetic antiprotozoan drug with a high activity against Helicobacter pylori. Specify this
drug:
A. Metronidazole B. Doxycycline hydrochloride
C. Chingamin D. Acyclovir E. Isoniazid
36. A 52-year-old patient has the following diagnosis: systemic amebiasis with involvment of
intestines, liver, lungs. What drug should be prescribed?
A. Metronidasol B. Quiniofone C. Tetracycline
D. Quingamine E. Enteroseptol
37. Systemic amebiasis with involvment of intestines, liver, lungs was diagnosed in a 52-year-
old patient. What drug should be prescribed?
A. Quiniofone B. Enteroseptol C. Metronidasol
D. Tetracycline E. Quingamine
38. A patient consulted a doctor about bowels disfunction. The doctor established symptoms
of duodenitis and enteritis. Laboratory examination helped to make the following diagnosis:
lambliosis. What medication should be administered?
A. Metronidazole B. Erythromycin C. Monomycin
D. Chingamin E. Tetracycline
39. A female patient consulted a doctor about a sense of epigastric discomfort, nausea and
anorexia. A duodenal content analysis revealed lamblia. What drug should be prescribed?
A. Metronidazole B.Chingamin C.Rifampicin D.Isoniazid E.Acyclovir
40. A patient ill with amebiasis was prescribed a certain drug. The use of alcohol together
with this drug is contraindicated because the drug inhibits metabolism of ethyl alcohol. What drug is
it?
A. Metronidazole B. Reserpine C. Clonidine
D. Diazepam E. Aminazine
41. A 30 y.o. patient is diagnosed with amebic dysentery. This diagnosis was bacteriologically
confirmed. Name the preparation for its treatment:
A. Metronidazole B. Mebendazole C. Itrakonazole
D. Furacillin E. Acyclovir
42. A 30-year-old patient complains about having abdominal pain and diarrhea for five days;
body temperature rise up to 37, 5oC along with chills. The day before a patient had been in a forest
and drunk from an open water reservoir. Laboratory analyses enabled to make the following
diagnosis: amebic dysentery. What is the drug of choice for its treatment?
A. Metronidazole B. Furazolidonum C. Levomycetin
D. Phthalazol E. Emetine hydrochloride
43. A 30 year old patient consulted a doctor about having diarrhea and stomach aches for 5
days, temperature rise up to 37,50C with chills. The day before the patient was in a forest and drank
some water from an open pond. He was diagnosed with amebic dysentery that was bacteriologically
confirmed. Name the medication for treatment of this disease:
A. Metronidazole B. Furasolidone C. Chloramphenicol
D. Phthalazole E. Emethine hydrochloride
44. A patient consulted a stomatologist about purulent inflammation of his gums. What drug
will be the most effective if it is suspected that a causative agent is an anaerobe?
A. Metronidazole B. Gentamicin C. Oxacillin sodium
D. Co-trimoxazole E. Nitroxoline

323
45. A patient consulted a dentist about itching and burning in the oral cavity; high
temperature. The patient was diagnosed with trichomonal gingivostomatitis. What drug should be
chosen for his treatment?
A. Metronidazole B. Doxycycline hydrochloride
C. Ampicillin D. Gentamicin sulfate E. Nystatin
46. A dentist has detected symptoms of parodontosis in a patient. What antiprotozoal drug
should be prescribed?
A. Metronidazole B. Levamisole C. Griseofulvin
D. Mykoseptin E. Furazolidone
47. A healthy man is in a region with high risk of catching malaria. What drug should be
administered for individual chemoprophylaxis of malaria?
A. Chingamin B. Sulfalen C. Tetracycline
D. Metronidazole E. Biseptol
48. This drug has a destructive effect on erythrocytic forms of malarial plasmodia and
dysenteric amoebae. It is used for treatment and prevention of such diseases as malaria, amebiasis
and interstitial disease. What drug is it?
A. Chingamin B. Emetine hydrochloride
C. Tetracycline D. Erythromycin E. Quinine
49. UN volunteers have arrived in Nigeria to assist the locals in aftermath of earthquakes.
What drug should they prescribe for individual chemoprophylaxis of malaria?
A. Chingamin
B. Pyrantel
C. Pyrimethamine (Chloridinum)
D. Primaquine
E. Interferon (Laferon)
What drug is used for treatment of malaria, amoebic dysentery, and autoimmune diseases?
A. Intraconazole B. Dexamethasone
C. Streptomycin sulfate D. Co-trimoxazole (Biseptol)
E. Chingamin (Chloroquine)
50. After the second abortion a 23 year old woman has been diagnosed with toxoplasmosis.
Which drug should be used for toxoplasmosis treatment?
A. Co-trimoxazole B. Itraconazole
C. Mebendazole D. Azidothimidine E. Acyclovir
51. A patient was diagnosed with active focal pulmonary tuberculosis. What drug should be
prescribed in the first place?
A. Isoniazid B. Sulfalen C. Cyclocerine
D. Ethionamide E. Ethoxide
52. A patient suffers from pulmonary tuberculosis. During treatment neuritis of visual nerve
arose. What drug has caused this by-effect?
A. Isoniazid B. Ethambutol C. Kanamycin
D. Rifampicin E. Streptomycin
53. After 4 months of treatment for tuberculosis the patient began complaining of toes and
fingers numbness, sensation of creeps. He was diagnosed with polyneuritis. What antituberculous
drug might have caused these complications?
A. Isoniazid B. Rifampicin C. Ciprofloxacin
D. Sodium salt of benzylpenicillin E. Iodine solution
54. A patient suffering form tuberculosis was treated with rifampicin, which caused drug
resistance of tuberculosis mycobacteria. In order to reduce mycobacteria resistance, rifampicin
should be combined with the following drug:
A. Isoniazid B. Acyclovir C. Intraconazole
D. Metronidazole E. Amoxicillin
55. Following treatment with a highly efficient anti-tuberculosis drug a 48-yearold female
developed optic nerve neuritis, memory impairment, cramps. Which of these anti-TB drugs had the
patient taken?

324
A. Isoniazid B.PASA C.Rifampicin D.Ethambutol E.Kanamycin sulfate
56. A patient with pulmonary tuberculosis is prescribed the most effective antituberculosis
antibiotic. Name this drug:
A. Tetracycline
B. Furasolidone
C. Rifampicin
D. Bactrim (Co-trimoxazole)
E. Streptocide
57. Tuberculosis can be treated by means of combined chemotherapy that includes
substances with different mechanisms of action. What antituberculous medication inhibits
transcription of RNA into DNA in mycobacteria?
A. Rifampicin B. Isoniazid C. Streptomycin
D. Ethionamide E. Para-aminosalicylic acid
58. After starting treatment for pulmonary tuberculosis a patient complained about red tears
and urine. What drug could cause such changes?
A. Rifampicin B. Benzylpenicillin sodium salt
C. Benzylpenicillin potassium salt D. Biseptol-480 E. Cefazolin
59. A patient being treated for tuberculosis is suffering from hearing deterioration. What
drug causes this complication?
A. Streptomycin B. Isonicotinic acid hydrazide (Isoniazid)
C. Rifampicin D. Ethionamide E. Kanamycin sulphate
60. A patient who has been taking tetracycline for a long time has developed candidosis of
mucous membranes. What drug should administered for its treatment?
A. Itraconazole B. Griseofulvin C. Nitrofungin
D. Amphotericin E. Nitrofurantoin
61. A female who had been continuously taking antibiotics for an intestinal infection
developed a complication manifested by inflammation of the oral mucosa and white deposit.
Bacteriological study of the deposit samples revealed yeast fungi Candida albicans. Which of the
following medications is indicated for the treatment of this complication?
A. Fluconazole B. Biseptol C. Tetracycline
D. Furazolidone E. Polymyxin
62. Infectious diseases are treated with antibiotics (streptomycin, erythromycin,
chloramphenicol). They inhibit the following stage of protein synthesis:
A. Translation B. Transcription C. Replication
D. Processing E. Splicing
63. Streptomycin and other aminoglycosides prevent the joining of formyl-methionyl-tRNA
by bonding with 30S ribosomal subunit. This effect leads to disruption of the following process:
A. Translation initiation in eucaryotes
B. Translation initiation in procaryotes
C. Replication initiation in procaryotes
D. Transcription initiation in eucaryotes
E. Transcription initiation in procaryotes
64. A patient with bacterial pneumonia was prescribed benzylpenicillin. What is the
mechanism of its antibacterial effect?
A. Inhibition of synthesis of microorganism wall
B. Inhibition of intracellular protein synthesis
C. Abnormal permeability of cytoplasmic membrane
D. Inhibition of SH-groups of microorganism enzymes
E. Antagonism with p-amino-benzoic acid
65. A 19 year old woman suffers from primary syphilis. Doctor administered her complex
therapy that includes benzylpenicillin sodium salt. What is the mechanism of action of this drug?
A. It blocks synthesis of peptidoglycan of microbial membrane
B. It blocks synthesis of cytoplasm proteins
C. It blocks thiol enzymes

325
D. It blocks RNA synthesis E. It blocks DNA synthesis
66. A doctor prescribed a cephalosporin antibiotic to the patient after appendectomy for
infection prevention. Antimicrobial activity of this group of antibiotics is based upon the disturbance
of the following process:
A. Microbial wall formation B. Nucleic acid synthesis
C. Ribosome protein synthesis
D. Energy metabolism E. Choline esterase block
67. For infection prevention a patient who underwent appendectomy was prescribed a
cephalosporin antibiotic. Antimicrobial activity of these antibiotics is called forth by the disturbance
of the following process:
A. Microbial wall formation B. Nucleic acid synthesis
C. Ribosomal protein synthesis
D. Energy metabolism E. Cholinesterase block
68. Certain infections caused by bacteria are treated with sulphanilamides that block the
synthesis of bacterial growth factor. What is the mechanism of these drugs action?
A. They are antivitamins of paminobenzoic acid
B. They inhibit the folic acid absorption
C. They are allosteric enzyme inhibitors
D. They are involved in redox processes E. They are allosteric enzymes
69. A patient with herpetic stomatitis was prescribed acyclovir for topical application. What is
its mechanism of action?
A. It inhibits synthesis of nucleic acids of viruses
B. It inhibits virus penetration into cells C. It inhibits virus maturation
D. It increases the resistance of macroorganism cells to the viruses
E. It inhibits virion assembly
70. A patient is ill with herpetic stomatitis provoked by immunosuppression. What
preparation introduced intravenously, internally and locally can provide antiviral and
immunopotentiating effect?
A. Acyclovir B. Remantadinum C. Levamisole
D. Methisazonum E. Amoxicillin
71. On the 5-th day of the respiratory disease a 24 y.o. man has developed progressive
headaches systemic dizziness, feeling of seeing double, paresis of mimic muscles on the right,
choking while swallowing. Acute viral encephalitis has been diagnosed. What is the main direction of
urgent therapy?
A. Zovirax B. Glucocorticoids
C. Ceftriaxon D. Lasix E. Hemodesis
72. What preparations are used for prevention of fungal infection?
A. Fluconozol, Orungal, Nisoral
B. Rubomycin, Bleomycin, Mytomycin C
C. Cytosar, Cormyctin, Lomycitin
D. Captopril, Enalapril E. Isoniazid, Ftibazid, Pyrazinamid
73. Name the halogen-containing antiseptic with fungicidal properties, which is used to treat
dermatomycosis:
A. Iodine solution
B. Formalin solution
C. Methylene blue
D. Brilliant green
E. Boric acid solution
74. A patient developed burning sensation in the oral cavity and white fuzzy coating on the
tongue. Oral thrush is diagnosed. What drug of those listed below should be used?
A. Tetracycline. B. Amphotericin. C. Griseofulvin.
D. Nystatin. E. Gentamicin.

326
75. A 4-year-old child presents with numerous carious cavities and yellow-colored teeth. The
mother has a history of antibiotic treatment during her pregnancy. What antibiotic was the most
likely taken by the child’s mother?
A. Cefazolin B. Streptomycin sulfate
C. Ampicillin D. Erythromycin E. Doxycycline
76. Pathogenic staphylococcus was obtained from the purulent wound of the patient.
Its antibiotic sensitivity was determined to be as follows: penicillin growth inhibition zone - 8
mm; oxacillin - 9 mm, ampicillin - 10 mm, gentamicin - 22 mm, lincomycin - 11 mm. What
antibiotic should be chosen for treatment in this case?
A. Gentamicin B. Penicillin C. Ampicillin
D. Oxacillin E. Lincomycin
77. An 18-year-old patient has developed candidiasis after the case of pneumonia
treated with β-lactam antibiotic. What antimycotic agent should be prescribed?
A. Streptomycin B. Fluconazole C.
Phthalylsulfathiazole
D. Ampicillin E. Trimethoprim/sulfamethoxazole (Biseptol)
78. A patient with pulmonary tuberculosis is prescribed the most effective
antituberculous antibiotic. Name this drug:
A. Furasolidone B. Bactrim (Co-trimoxazole)
C. Streptocide D. Rifampicin E. Tetracycline
79. A patient with streptococcal infection of the gingiva was prescribed a drug with β-
lactam ring in its structure. What drug of those listed below belongs to this pharmacological
group?
A. Erythromycin B. Streptomycin sulfate
C. Benzylpenicillin D. Rifampicin E. Levomycetin (Chloramphenicol)
80. After a prolonged isoniazid treatment, the patient developed polyneuritis, paresthesia,
memory disorders, and convulsions. What is the most likely mechanism of the described isoniazid
side-effects?
A. Disruption of cell membrane synthesis B. Inhibition of protein synthesis
C. Inhibition of RNA-synthesis D. Inhibition of pyridoxal phosphate synthesis
E. Para-aminobenzoic acid antagonism
81. Mother of a 2-year-old child made an appointment with the dentist. She complains of
teeth destruction in her child. Examination shows that the milk teeth of the child are deformed,
carious, and have a brown border at their cervices. Medical history of the mother revealed that
during pregnancy she had been taking antibiotics without the doctor’s prescription. What group of
antibiotics with the most marked teratogenic effect was likely taken by the mother?
A. Penicillins B. Macrolides
C. Tetracyclines D. Aminoglycosides E. Cephalosporins

Special
1. Pathogenic staphylococcus was obtained from the purulent wound of the patient. Its
antibiotic sensitivity was determined to be as follows: penicillin growth inhibition zone - 8
mm; oxacillin - 9 mm, ampicillin - 10 mm, gentamicin - 22 mm, lincomycin - 11 mm. What
antibiotic should be chosen for treatment in this case?
A. Gentamicin B. Penicillin C. Ampicillin D. Oxacillin E.
Lincomycin
2. A patient with streptococcal gingival infection was prescribed a medication that contains
beta lactam ring in its structure. What preparation belongs to this group?
A. Benzylpenicillin B. Rifampicin C. Erythromycin D. Streptomicin sulfate E.
Chloramphenicol
3. A patient with streptococcal infection of gums was prescribed a drug that contained beta-
lactam ring in its structure. Which drug relates to this group?
A. Benzylpenicillin B. Rifampicin C. Erythromycin
327
D. Streptomycin sulfate E. Chloramphenicol
4. Gonorrhoea was revealed in the patient on bacterioscopy of the smear from urethra.
Taking into account that medecines for gonorrhoea are fluorquinolones, patient should be
prescribed:
A. Ciprofloxacin B. Fluorouracil C. Cefazoline D. Urosulfan E. Furazolidone
5. A patient has been diagnosed with gonorrhea. As fluoroquinolones are the drugs of choice
for treatment of gonorrhea the patient should be prescribed:
A. Ciprofloxacin B. Furazolidone
C. Fluorouracil D. Sulfacarbamide (Urosulfanum) E. Cefazolin
6. An 18-year-old patient has developed candidiasis after the case of pneumonia treated with
β-lactam antibiotic. What antimycotic agent should be prescribed?
A. Fluconazole B. Streptomycin C. Phthalylsulfathiazole
D. Ampicillin E. Trimethoprim/sulfamethoxazole
7. A patient needs to be prescribed a wide-spectrum fluoroquinolone agent. Select one such
agent from the list:
A. Ciprofloxacin B. Azlocillin C. Carbenicillin D. Chinoxydin E. Amoxicillin
8. Broadspectrum antibiotics can cause various complications, including intestinal
candidiasis. What drug is used for treatment of this complication?
A. Nystatin B. Griseofulvin C. Undecyne D. Amphotericin B E.
Gramicidin

328
Genetics
1. Ability to divide is characteristic of prokaryotic and eukaryotic cells. Prokaryotic cell
division is different from that of eukaryotic, but there is one molecular process that is the basis of
both types of division. Name this process.
A. DNA replication B. Transcription
C. Reparation D. Translation E. Gene amplification
2. From the nasopharynx of a 5-year-old child it was excreted a microorganism which is
identical to Corynebacterium diphtheriae dose according to morphological and biochemical signs.
Microorganism does not produce exotoxin. As a result of what process can this microorganism
become toxigenic?
A. Cultivation in the telluric environment
B. Chromosome mutation
C. Passing through the organism of the sensative animals
D. Phage conversion E. Growing with antitoxic serum
3. Because of suspected intrahospital infection in the neonatal department of the maternity
home the inspection was carried out. In some children and on some general things Staphylococcus
aureus was revealed. What properties of these cultures allow to establish their origin from one
source?
A. Antibioticogramma B. Antigenic structure
C. Biochemical activity D. Phagotype E. Chromogenesis
4. From the fecal sample of a patient Shigella sonne were isolated. What additional studies
are required to identify the source of infection?
A. Phage-typing of the isolated pure culture
B. Antibiogram C. Precipitation reaction
D. Complement-fixation reaction E. Neutralization reaction
5. In the surgical department of a dental clinic cases of hospital-acquired staphylococcal
infection were registered which was caused by strains with multiple drug resistance. Such feature
can be identified by presence of:
A. R-plasmids B. F-plasmids C. Exotoxins
D. Temperate bacteriophages E. Virulent bacteriophages
6. RNA that contains AIDS virus penetrated into a leukocyte and by means of reverse
transcriptase forced a cell to synthesize a viral DNA. This process is based upon:
A. Reverse transcription B. Operon repression
C. Reverse translation D. Operon depression
E. Convariant replication
7. During reproduction of some RNA-containing viruses that cause tumors in animals, genetic
information can be transmitted in the opposite direction from the RNA to the DNA via a specific
enzyme. The enzyme of reverse transcription is called:
A. Reverse transcriptase B. DNA polymerase
C. Ligase D. Primase E. Topoisomerase
8. It was revealed that T-lymphocytes were affected by HIV. Virus enzyme - reverse
transcriptase (RNA-dependent DNA polymerase) - catalyzes the synthesis of:
A. DNA on the matrix of virus mRNA
B. Virus informational RNA on the matrix of DNA
C. DNA on virus ribosomal RNA D. Viral DNA on DNA matrix
E. mRNA on the matrix of virus protein
9. A doctor was addressed by a 30-year old man. There is a probability of the patient being
HIV-positive. To clarify the diagnosis the doctor proposed to perform polymerase chain reaction. The
basic process in this kind of investigation is:
A. Gene amplification B. Transcription
C. Genetic recombination D. Genomic mutation
E. Chromosome mutation
10. In the course of evolution there developed molecular mechanisms for correction of
damaged DNA molecules. This process is called:

329
A. Reparation
B. Transcription
C. Translation
D. Replication
E. Processing
11. Genetic information is stored in DNA but does not participate directly in protein synthesis
within DNA cells. What process ensures transfer of genetic information into polypeptide chain?
A. Translation
B. Formation of rRNA
C. Formation of tRNA
D. Formation of iRNA
E. Replication
Special
A man is a carrier of HIV that is an RNA virus. The cells of this patient synthesize
viral DNA. This process is based on:
A. Transcription
B. Repair
C. Replication
D. Reverse transcription
E. Translation

Infection
1. A doctor made the diagnosis of gonorrhoea. It was known from the anamnesis that a
patient had had gonorrhoea before and he had been treated completely. What type of infection can
this new disease be attributed to?
A. Superinfection B. Reinfection C. Relapse
D. Secondary infection E. Autoinfection
2. A patient recovered from Sonne dysentery and was once more infected with the same
causative agent. What is such infection form called?
A. Reinfection B. Recidivation C. Superinfection
D. Persisting infection E. Chronic infection
3. A patient who suffered form syphilis took a course of antibiotic therapy and fully
recovered. Some time later he was infected again with Treponema pallidum. What form of infection
is it?
A. Reinfection B. Recurrence C. Secondary infection
D. Superinfection E. Complication
4. Material obtained from a patient contains several types of microorganisms (staphylococci
and streptococci) causative of the patient’s disease. Name this type of infection:
A. Mixed infection
B. Superinfection
C. Reinfection
D. Consecutive infection
E. Coinfection
5. 2 weeks since the blood transfusion a recipient has developed fever. What protozoal
disease can it be?
A. Trypanosomiasis B. Malaria
C. Amebiasis D. Toxoplasmosis E. Leishmaniasis
6. Two weeks after hemotransfusion a patient developed fever. What protozoal disease can
be suspected?
A. Malaria B. Toxoplasmosis C. Leishmaniasis
D. Amebiasis E. Trypanosomiasis
7. A pregnant woman was registered in an antenatal clinic and underwent complex
examination for a number of infections. Blood serum contained IgM to the rubella virus. What is this
result indicative of?

330
A. Of primary infection B. Of a chronic process
C. The woman is healthy D. Of exacerbation of disease
E. Of recurring infection with rubella virus
8. A pregnant woman was detected with IgM to rubella virus. An obstetrician-gynecologist
recommended therapeutic abortion due to the high risk of teratogenic affection of the fetus.
Detection of IgM was of great importance as it is these specific immunoglobulins that:
A. Indicate recent infection B. Penetrate placental barrier
C. Have the largest molecular weight
D. Are associated with anaphylactic reactions
E. Are the main factor of antiviral protection
9. A patient has been diagnosed with ARVI. Blood serum contains immunoglobulin M. What
is the stage of infection in this case?
A. Acute B. Prodromal C. Incubation
D. Reconvalescence E. Carriage
10. As an example of specific human parasites one can name Plasmodium falciparum, human
pinworm and some others. The source of parasite invasion is always a human. Such specific human
parasites cause the diseases that are called:
A. Anthroponoses B. Anthropozoonoses
C. Zoonoses D. Multifactorial diseases E. Infections
11. Examples of human-specific parasites are malaria plasmodium, enterobius vermicularis
and some other. The source of invasion of such parasites is always a human. Such human-specific
parasites cause diseases that are called:
A. Anthroponotic B. Zoonotic C. Anthropozoonotic
D. Infectious E. Multifactorial
12. A pregnant woman complains of vaginal mucosa irritation, itching and genital tracts
secretion. Bacterioscopy of vaginal smears revealed large gram-positive oval oblong cells that form
pseudomicelium. What is the most probable channel of infection?
A. Endogenous infection B. Sexual transmission
C. Contact infection D. Vector-borne transmission
E. Wound infection
13. A patient developed pyoinflammatory process of periodontal tissue caused by activation
of the microorganisms inherent in the body, which are the part of oral mucosal microflora. What
type of infection is it?
A. Relapse B. Autoinfection C. Reinfection
D. Exogenous infection E. Superinfection
14. Typical signs of food poisoning caused by C. botulinum include diplopia, swallowing and
respiration disorders. These signs develop due to:
A. Enterotoxic shock development
B. Enterotoxin action
C. Adenylate cyclase activation
D. Adhesion of the agent to enterocyte receptors
E. Exotoxin action
15. A laboratory has been investigating virulence of a diphtheria agent. In the process of the
experiment the infection was introduced intraperitoneally into test animals. The dosage of bacteria
resulting in 95% mortality of test animals was found. What unit of virulence measurement was
determined?
A. DLM
B. DCL
C. LD50
D. ID
E. LD5
16. A patient with signs of intestinal infection (vomiting, diarrhea, abdominal pain) has been
presenting with increasing symptoms of intoxication for three days. Papular rash appeared on the

331
uncovered skin areas and spread to the torso. A doctor suspected pseudotuberculosis. What
laboratory test allows confirming this diagnosis within the first week from the onset of disease?
A. Bacteriological
B. Microscopic
C. Serological
D. Allergic
E. Biological
17. Biological material taken from a patient contains several species of microorganisms
(staphylococci and streptococci) that are causative of the patient’s disease. Name this type of
infection:
A. Mixed infection B. Superinfection C. Reinfection
D. Consecutive infection E. Coinfection
18. A patient developed pyoinflammatory process of periodontal tissues caused by
activation of the microorganisms inherent in the body, which are a part of oral mucosal
microflora. What type of infection is it?
A. Exogenous infection B. Reinfection
C. Relapse D. Superinfection E. Autoinfection
19. A 12-year-old boy with clinical presentation of influenza has developed respiratory
mycoplasmosis. What type of infection has developed under these conditions?
A. Mixed infection B. Superinfection
C. Iatrogenic infection D. Relapse E. Autoinfection
20. A bacteriologist found in a sick child the causative agents of Flexner dysentery type 2,
Sonne dysentery type 1, and enteropathogenic colibacillus – O55/B5. Name this type of infection:
A. Mixed infection B. Superinfection
C. Secondary infection D. Carriage of pathogenic bacteria E. Reinfection
21. A man has a case of epidemic typhus 5 years ago. After an acute respiratory viral disease,
against the background of weakened immune system, he developed signs of typhus again. The
exacerbation occurred because of the causative agents, remaining in his body. What type of
infections is it?
A. Reinfection B. Co-infection C. Superinfection D. Secondary infection E.
Relapse

332
SPECIAL VIROLOGY
Influenza viruses
1. Ambulace brought to the hospital a patient with acute respiratory viral infection. The
illness began suddenly with temperature rise up to 39,90 C. He complains of headache in
frontotemporal lobes, pain in eyeballs, aching of the whole body, nose stuffiness, sore throat, dry
cough. At home he had a nasal hemorrhage twice. What type of acute respiratory viral infection is it?
A. Influenza B. Adenoviral infection
C. Parainfluenza D. RS-infection E. Enterovirus infection
2. Material taken from a patient with provisional diagnosis "influenza"was referred to a
laboratory. For virological examination the hemadsorption reaction was applied. This reaction can be
applied for detection of the following viruses:
A. Viruses containing hemagglutinins
B. All the simple viruses C. All the complex viruses
D. DNA-genomic viruses E. Any viruses
3. During the breakout of acute respiratory infection in order to diagnose influenza the
express-diagnosis, based on revealing of specific viral antigen in the examined material
(nasopharyngial lavage), is carried out. Which reaction is used for this?
A. Immunofluorescence B. Agglutination
C. Precipitation D. Opsonization E. Complement binding
4. A virological laboratory obtained pathological material (mucous discharges from nasal
meatuses) taken from a patient with provisional diagnosis "influenza". What quick test will allow to
reveal specific viral antigen in the material under examination?
A. Direct and indirect immunofluorescence test
B. Direct and indirect fluorescence immunoassay
C. Hemagglutination inhibition assay
D. Radioimmunoassay E. –
5. Pathological material (mucosal excretion from the nasal passages) taken obtained from a
patient provisionally diagnosed with influenza was delivered to the virological laboratory. What quick
test allows detecting specific viral antigen in the investigated material?
A. Direct and indirect immunofluorescence (IF)
B. Reverse indirect haemagglutination (RIHA)
C. Radioimmunoassay (RIA)
D. Direct and indirect enzyme-linked immunosorbent assay (ELISA)
E. Hemagglutination inhibition assay (HAI)
6. Virological laboratory has received patient’s nasopharyngeal lavage. What can be used to
single out influenza virus from the patient’s lavage?
A. Chick embryo B. Endo’s medium C. Meat infusion agar
D. Meat infusion broth E. Lowenstein–Jensen medium
7. Inoculation of hen’s embryos is the main method of detection of influenza virus. In order
to neutralize associated bacterial flora in the material under examination (nasopharyngeal lavage) it
is necessary to add beforehand:
A. Antibiotics B. Eubiotics C. Fluorescent serum
D. Leukocytic interferon E. Ant-influenza gamma globulin
8. A patient has been diagnosed with ARVI. Blood serum contains immunoglobulin M. What
is the stage of infection in this case?
A. Acute B. Prodromal C. Incubation
D. Reconvalescence E. Carriage
9. A patient fell ill the day before, the disease is acute with a predominance of general toxic
symptoms. With an account for the epidemic situation in the city, the doctor diagnosed the patient
with influenza A. What emergency etiotropic treatment must be administered to this patient?
A. Rimantadine B. Oxolinic ointment C. Gentamicin
D. Inactivated influenza vaccine E. Human gamma globulin
10. Pharmacy has received viricidal drugs. Choose the one used for influenza treatment from
the list given below.

333
A. Rimantadine B. Metisazone C. Levamisole
D. Azidothimidine E. Acyclovir
11. Epidemic of influenza was announced in a town. Which drug can be recommended for
the nonspecific prophylaxis of influenza?
A. Leukocytic interferon B. Anti-influenza vaccine
C. Anti-influenza immunoglobulin
D. Antibiotics E. Anti-influenza serum
12. A drugstore received a supply of a drug that is widely used for treatment of many virus
diseases since it is not virus specific. What drug is it?
A. Interferon B. Remantadin C. Metisazone
D. Immunoglobulin E. Vaccine
13. To prevent the seasonal influenza epidemics in the city hospitals, sanitary epidemic
station gave orders to immunize health care workers. Which of the following preparations should be
used for immunization?
A. Subunit vaccine B. Interferon
C. Amantadine D. Rimantadine E. Gamma-globulin
14. For the specific prevention of influenza, the employees of an enterprise were vaccinated
with "Influvac". What type of immunity will develop in the body of the vaccinated?
A. Artificial active B. Innate congenital
C. Natural active D. Artificial passive E. Natural passive
15. A person has been in contact with influenza patient. What drug should be administered
for specific passive influenza prophylaxis?
A. Antigrippal immunoglobulin
B. Vaccine influenza virus inactivated C. Leukocytic interferon
D. Amizon E. Anaferon
Poxviruses
1. The contents of vesicles that appeared on the mucous membrane of a patient with variola
were sent to a virological laboratory. Which of the listed changes were revealed during the smear
microscopy?
A. Paschen bodies B. Babes-Negri bodies
C. Guarnieri bodies D. Babes-Ernst bodies
E. Syncytium

Picornaviruses
1. A 3-year-old child has been taken to a pediatrician. He has no recent history of any
diseases. Objective examination revealed no pathology of the internal organs. The child needs the
routine immunization against the following disease:
A. Poliomyelitis B. Diphtheria and tetanus
C. Measles, rubella, parotitis
D. Pertussis E. Type B hepatitis
2. A 1,5 y.o. child fell ill acutely with high temperature 380C, headache, fatigue. The
temperature declined on the fifth day, muscular pain in the right leg occured in the morning, there
were no movements and tendon reflexes, sensitivity was reserved. What is the initial diagnosis?
A. Polyomyelitis B. Viral encephalitis C. Polyartropathy
D. Osteomyelitis E. Hip joint arthritis
3. A child is 9 months old. The patient’s body temperature is 36,7 oC, the skin is pale, humid,
there is pain in leg muscles. There is no extremities mobility, sensitivity is present. The child has been
diagnosed with poliomyelitis. The causative agent of this disease relates to the following family:
A. Picornavirus B. Paramyxovirus C. Tohovirus
D. Adenovirus E. Rotavirus
4. In our country, routine preventive vaccinations against poliomyelitis involve using live
vaccine that is administered orally. What immunoglobulins are responsible for the development of
local post-vaccination immunity in this case?
A. Secretory IgA B. IgM C. IgG

334
D. Serum IgA E. IgE
5. An 8-year-old child was hospitalized for fever up to 39,8oC, inertness, moderate headache,
vomiting. Examination revealed meningeal symptoms. Lumbar puncture was performed. The
obtained fluid had raised opening pressure, it was transparent, with the cell count of 450 cells per 1
mcL (mainly lymphocytes - 90%), glucose level of 2,6 mmol/l. What causative agent might have
caused the disease in the child?
A. Enterovirus B. Meningococcus C. Koch’s bacillus
D. Staphylococcus E. Pneumococcus
6. A culture of monkey cells (Vero) and a group of mouse sucklings were infected with an
inoculum taken from a child with provisional diagnosis "enterovirus infection". There was no
cytopathic effect on the cell culture but mouse sucklings died. What enteric viruses might have
caused disease of this child?
A. Coxsackie A B. Coxsackie B C. ECHO virus
D. Polioviruses E. Unclassified enteric viruses 68-71
7. A laboratory received a sample of water used in drug production for sanitary and
virusological analysis. What group of viruses will indicate fecal contamination of water and thus the
need for its additional purification?
A. Picornaviridae B. Herpesviridae C. Orthomyxoviridae
D. Retroviridae E. Flaviviridae
8. A paitent, who works as a milkmaid, has made an appointment with a dentist with
complaints of aphtha-shaped rash on the mucosa of oral cavity. The doctor detected rash on her
hands in the area of nail plates. What agent causes this disease?
A. Foot-and-mouth disease virus B. Cytomegalovirus
C. Vesicular stomatitis D. Herpesvirus E. Coxsackie B virus
2018
Preventive vaccination against poliomyelitis is made with inactivated vaccine introduced
parenterally. What immunoglobulins create the postvaccinal immunity in this case?
A. Serum. IgA, IgM B. IgE, IgM C. IgM, secretory IgA
D. IgM, IgG E. IgG, secretory IgA

Rotaviruses
1. A three-year-old child has had marked diarrhea for three days. Immune electron
microscopy of his excrements revealed bilayer pseudocovered capsid viruses that looked like small
spoke wheels. What viruses have been revealed?
A. Rotaviruses B. Coxsackie viruses C. ECHO viruses
D. Coronaviruse E. Reoviruses
2. An outbreak of an intestinal infection occurred in a kindergarten on the eve of New Year
holidays. Bacteriological examination of patients’ feces didn’t reveal any pathogenic bacteria.
Electron microscopy revealed roundish structures with clear outer edges and a thick core resembling
a wheel. Specify the most likely causative agent of this infection:
A. Rotavirus B. Adenovirus C. Coxsacki-virus
D. E. coli E. P.vulgaris

Rabies viruses
1. A hospitalized patient bitten by a rabid animal has an abusive wound of shin. What kind of
vaccine must be given to prevent rabies?
A. Anti-rabies vaccine B. DTaP
C. Td D. BCG E. TABte
2. In the area being the epicenter of the registered rabies cases among wild animals a 43-
year-old man presented to a clinic and claimed to have been bitten by a stray dog. He was given a
course of anti-rabies vaccine. This preparation relates to the following type of vaccines:
A. Attenuated B. Inactivated C. Molecular
D. Toxoids E. Synthetic

335
3. In the area that is the epicenter of the registered rabies cases among wild animals a 43-
year-old man arrived at a clinic and claimed to have been bitten by a stray dog. He was given a
course of anti-rabies vaccine. This preparation belongs to the following type of vaccines:
A. Attenuated
B. Inactivated
C. Molecular
D. Toxoids
E. Synthetic
4. A female patient bitten by a stray dog came to a surgery. Wide lacerated wounds were
localized on the patient’s face. What treatment-and prevention aid should be rendered in order to
prevent rabies?
A. Immunization with the antirabic vaccine
B. Combined antibiotic therapy
C. Hospitalization, injection of DTP
D. Hospitalization, medical surveillance
E. Urgent injection of normal gammaglobulin
5. A man who was bitten by the unknown dog applied to the surgeon. Wide ragged wounds
were localized on the face. What curative-prophylactic aid should be given to prevent rabies?
A. Start immunization with rabies vaccine
B. Prescribe combined antibiotic therapy
C. Immediate injection of DPT vaccine
D. Hospitalize the patient and keep under the doctor’s supervision
E. Immediately inject normal gamma globulin
6. Brain autopsy revealed an edema, hyperemia, and small hemorrhages in the medulla
oblongata. Microscopically chromatolysis, hydropia and nerve cell necrosis are observed; within the
cytoplasm of hippocampal nerve cells there are eosinophilic structures (Negri bodies) detected. What
diagnosis corresponds with the described morphological signs?
A. Rabies
B. Encephalomyelitis
C. Meningococcal meningitis
D. Brucellosis
E. Encephalitis

Vesicular stomatitis virus


1. A patient has some vesicles on the mucous membrane of the oral cavity, lips and nose. A
dentist suspected vesicular stomatitis. What analysis will allow to confirm the diagnosis?
A. Recovery of virus from the vesicular fluid B. Allergy test
C. Recovery of bacteria from the vesicular fluid
D. Contamination of animals with the vesicular fluid
E. Microscopy of the vesicular fluid

Herpesviruses
1. A patient consulted a dentist about pains, reddening and swelling of gums. The dentist
assumed herpetic gingivostomatitis. What virus might have caused this disease?
A. Herpes simplex virus type 1 B. Herpes simplex virus type 2
C. Herpes zoster D. Cytomegalic virus E. Epstein-Barr virus
2. A 26-year-old male patient complains of a rash on the upper lip skin, which arose on a
background of influenza with high-grade fever and is accompanied by pain and burning. The rash has
been present for 3 days. Objectively: the skin of the upper lip is edematic and erythematous,
grouped vesicles are filled with serous fluid and have a rough surface. What is the most likely
diagnosis?
A. Herpetic vesicular dermatitis B. Eczema
C. Contact dermatitis D. Dermatitis herpetiformis
E. Erythema multiforme

336
3. A 27-year-old sexually active female complains of numerous vesicles on the right sex lip,
itch and burning. Eruptions regularly turn up before menstruation and disappear 8-10 days later.
What is the most likely diagnosis?
A. Herpes simplex virus B. Bartholinitis C. Primary syphilis
D. Cytomegalovirus infection E. Genital condylomata
4. A 7 y.o. girl has mild form of varicella. Headache, weakness, vertigo, tremor of her limbs,
ataxia, then mental confusion appeared on the 5th day of illness. Meningeal signs are negative.
Cerebrospinal fluid examination is normal. How can you explain these signs?
A. Encephalitis B. Meningitis C. Meningoencephalitis
D. Myelitis E. Neurotoxic syndrome
5. On the 21 day after appearance of vesiculous chickenpox rash a 7-year-old child developed
ataxia, nystagmus, intention tremor, muscle hypotonia. Liquor analysis shows insignificant
lymphocytic pleocytosis, slightly increased protein rate. What complication is it?
A. Encephalitis B. Acute nephritis C. Pneumonitis
D. Purulent meningitis E. Postherpetic neuralgia
6. A patient complained about general weakness, fever, painful rash on his trunk skin . He has
been suffering from this for 3 days. Objectively: lateral surface of trunk on the left is hyperemic and
edematic, there are some groups of vesicles with serous and haemorrhagic contents. What is the
most probable diagnosis?
A. Herpes zoster B. Contact dermatitis simplex
C. Contact allergic dermatitis
D. Microbial eczema E. Herpetiform Duhring’s dermatosis
7. A 50-year-old woman is being treated for shingles in a neurology unit. What reactivated
virus causes this disease?
A. Varicella zoster virus (chickenpox virus)
B. Herpes simplex virus type 1 C. Herpes simplex virus type 2
D. Measles virus E. Cytomegalovirus
8. A 67-year-old male patient complains of rash, severe pain in the subscapular region on the
right. Objectively: skin in the right subscapular region is covered with linearly arranged pink-red
edematous lesions that are somewhat infiltrated, and have clear boundaries. On the lesion surface
there are vesicles with transparent exudate. What is the most likely diagnosis?
A. Herpes zoster B. Duhring dermatitis
C. Erysipelas D. Atopic dermatitis E. Impetigo
9. 2 days ago a patient presented with acute pain in the left half of chest, general weakness,
fever and headache. Objectively: between the 4 and 5 rib on the left the skin is erythematous, there
are multiple groups of vesicles 2-4 mm in diameter filled with transparent liquid. What diease are
these symptoms typical for?
A. Herpes zoster B. Pemphigus C. Herpes simplex
D. Streptococcal impetigo E. Herpetiform Duhring’s dermatosis
10. A 3-year-old child has continuous fever, lymph nodes are enlarged, the amount of
lymphocytes in blood is significantly increased. Enzymelinked immunosorbent assay (ELISA) revealed
antigen of Epstein-Barr virus. What diagnosis can be made based on the information given above?
A. Infectious mononucleosis B. Burkitt’s lymphoma
C. Herpetic lymphadenopathy D. Cytomegalovirus infection
E. Generalized infection caused by herpes-zoster
11. A doctor examines a 17-year-old girl. The following is detected: pharyngitis, cervical
lymphadenopathy, fever. The preliminary diagnosis is infectious mononucleosis. What method of
investigation allows to confirm this diagnosis at the disease onset?
A. Determining antibodies IgМ to Epstein-Barr virus
B. Microscopy of blood smear according to Giemsa method
C. Determining antibodies IgG to Epstein-Barr virus
D. Sabin-Feldman dye test
E. Determining the amount of C-reactive Protein

337
12. An 18 year old patient was admitted to a hospital with complaints of headache,
weakness, high temperature, sore throat. Objectively: enlargement of all groups of lymph nodes was
revealed. The liver is enlarged by 3 cm, spleen - by 1 cm. In blood: leukocytosis, atypical lymphocytes
- 15%. What is the most probable diagnosis?
A. Infectious mononucleosis B. Acute lymphoid leukosis
C. Diphtheria D. Angina E. Adenoviral infection
13. Often the cause of secondary immunodeficiency is an infectious affection of an organism,
when agents reproduce directly in the cells of immune system and destroy them. Specify the
diseases, during which the described above occurs:
A. Poliomyelitis, viral hepatitis A
B. Q fever, typhus
C. Tuberculosis, mycobacteriosis
D. Infectious mononucleosis, AIDS
E. Dysentery, cholera
14. An HIV-positive patient’s cause of death is acute pulmonary insufficiency resulting from
pneumonia. Pathohistological investigation of lungs has revealed transformed cells resemble owl’s
eye. Name the pneumonia causative agent:
A. Cytomegalovirus B. Pneumococcus
C. Influenza virus D. Candida fungi E. Toxoplasma
15. What chemotherapeutic agent is a drug of choice for treatment of herpes?
A. Acyclovir B. Rifampicin C. Chingamin
D. Doxycycline hydrochloride E. Metronidazole
16. A patient has herpetic rash. What medication should be administered?
A. Acyclovir B. Gentamycin C. Clotrimazole
D. Benzylpenicillin sodium salt E. Biseptol
17. A patient is ill with herpetic stomatitis provoked by immunosuppression. What
preparation introduced intravenously, internally and locally can provide antiviral and
immunopotentiating effect?
A. Acyclovir B. Remantadinum C. Levamisole
D. Methisazonum E. Amoxicillin
18. A patient consulted a dentist about a lesion of his oral mucosa. He was diagnosed with
herpetic stomatitis. Which of the following drugs will have an effect on etiotropic factor?
A. Acyclovir B. Dimedrol C. Paracetamol D. Levamisole E. Furacilinum
19. A patient has herpetic conjunctivitis. What etiotropic drug should be administered?
A. Acyclovir B. Ampicillin C. Methisazonum D. Furagin E. Tetracycline
20. A patient with herpetic stomatitis was prescribed acyclovir for topical application. What is
its mechanism of action?
A. It inhibits synthesis of nucleic acids of viruses
B. It inhibits virus penetration into cells C. It inhibits virus maturation
D. It increases the resistance of macroorganism cells to the viruses
E. It inhibits virion assembly
21. A medical student was hospitalized into the infectious diseases unit on the 2nd day
after the disease onset; the patient is suspected to have infectious mononucleosis. What results
of laboratory analysis can confirm this diagnosis immediately on the day of the
hospitalization?
A. IgM antibodies to herpes simplex virus were detected
B. Herpesvirus was isolated
C. Fourfold increase in number of antibodies to Epstein-Barr virus was detected
D. IgM antibodies to Epstein-Barr virus were detected
E. Cytomegalovirus antibodies were detected
22. A patient came to a dentist complaining of fever and characteristic small vesicles on the
buccal, palatal, and lingual mucosa. The dentist suspects herpetic stomatitis. What additional test is
necessary to confirm the diagnosis?
A. Inoculation of chick chorioallantoic membrane or brain tissue of white mice

338
B. Precipitation reaction
C. Inoculation on Rappaport medium
D. Inoculation on medium 199 with addition of bovine serum
E. Inoculation on Eagle medium
23. A 17-year-old girl is undergoing an examination. She has signs of pharyngitis,
lymphadenopathy of the neck, and fever and was provisionally diagnosed with infectious
mononucleosis. What test can confirm this diagnosis at the onset of the disease?
A. Microscopy of blood smear using Romanovsky-Giemsa method
B. Measuring the levels of C-reactive protein
C. Measuring of IgG to Epstein-Barr virus
D. Sabin-Feldman dye test
E. Measuring the antibodies (IgM) to Epstein-Barr virus

Arboviruses
1. A patient with clinical signs of encephalitis was delivered to the infectious diseases
hospital. Anamnesis registers a tick bite. Hemagglutination-inhibition reaction helped to reveal
antibodies to the causative agent of tick-borne encephalitis in the dilution 1:20 which is not
diagnostic. What actions should the doctor take after he had got such result?
A. To repeat the examination with serum taken 10 days later
B. To examine the same serum C. To apply more sensitive reaction
D. To repeat examination with another diagnosticum
E. To deny diagnosis of tick-borne encephalitis
2. After a thorough examination the patient who had returned from Central Asia to Ukraine
was diagnosed with spring-summer encephalitis. Its pathogen might have entered the body through
the bite of the following arthropod:
A. Dog-louse B. Taiga tick C. Mosquito
D. Itch mite E. Argasid tick (ornithodorus papillipes)
3. After a thorough examination, a man, who has returned to Ukraine from the Central Asia,
was provisionally diagnosed with spring-summer encephalitis. In such cases, the causative agent is
transmitted via bites of a certain arthropod. Name this arthropod:
A. Ixodes persulcatus
B. Sarcoptes scabiei
C. Phlebotominae
D. Ornithodorus papillipes
E. Ixodes ricinus

Measles virus
1. A 4-year-old girl died suddenly with symptoms of asphyxia. Autopsy revealed white spots
on the buccal mucosa; large blotches of rash on the skin of face, trunk and extremities; conjunctivitis,
edema with foci of necrosis on the laryngeal mucosa; giant-cell pneumonia on microscopy. What is
the most likely diagnosis?
A. Measles B. Scarlet fever C. Influenza
D. Meningococcal infection E. Typhus
2. A child is 4 years old, has been ill for 5 days. There are complaints of cough, skin rash, to-
38,2oC, face puffiness, photophobia, conjunctivitis. Objectively: there is bright, maculo-papulous, in
some areas confluent rash on the face, neck, upper chest. The pharynx is hyperemic. There are
seropurulent discharges from the nose. Auscultation revealed dry rales in lungs. What is the most
likely diagnosis?
A. Measles B. Rubella C. Scarlet fever
D. Adenoviral infection E. Enterovirus exanthema
3. A child, aged 4, has being ill for 5 days, suffers from cough, skin rash, t0- 38,2 °C, facial
hydropy, photosensitivity, conjunctivitis. On the face, neck, upper part of the chest there is bright

339
maculopapular rash with areas of merging. Hyperemic throat. Seropurulent nasal discharge. In lungs
there are dry crackles. What is the most probable preliminary diagnosis?
A. Measles B. Rubella C. Scarlet fever
D. Adenovirus infection E. Enterovirus exanthema
4. Such presentations as catarrhal conjunctivitis, pharyngitis, laryngotracheo-bronchitis,
white spots on the buccal mucosa in the region of lower premolar teeth, maculopapular rash on face,
body and extremities are typical for the following disease:
A. Measles B. Spotted fever C. Scarlet fever
D. Meningococcal infection E. Influenza
5. A 7 year old child had an acute onset of disease. Pediatrician stated that mucous
membrane of face is hyperemic and covered with a lot of mucus. Mucous membrane of cheeks has
whitish stains. Next day the child’s skin of face, neck, body was covered with coarsely-papular
rash.What disease may be presumed?
A. Measles B. Scarlet fever C. Diphteria
D. Meningococcemia E. Allergic dermatitis
6. There is a 7-year-old child with complains of cough, lacrimation, rhinitis, skin rash,
photophobia and three-day-long fever as high as 38oC. Physical examination has revealed the
following: conjunctivitis; bright red maculopapular rash covering the skin of face, neck and torso;
hyperemic pharynx; serous purulent secretions from the nose; dry rales in the lungs. What is the
most probable diagnosis?
A. Measles B. Scarlet fever C. Rubella
D. Adenovirus infection E. Chicken pox
7. A 5-year-old child developed an acute disease starting from body temperature rise up to
38,5oC, running nose, cough and conjunctivitis. On the 4th day the child presented with maculo-
papular rash on face. Body temparature rose again up to 39,2oC. Over the next few days the rash
spread over the whole body and extremities. Mucous membrane of palate was hyperemic, there was
whitish deposition on cheek mucous membrane next to molars. What is your provisional diagnosis?
A. Measles B. Acute viral respiratory infection
C. Yersinia D. Rubella E. Enterovirus diseases
8. A 3 year old child has been suffering from fever, cough, coryza, conjunctivitis for 4 days. He
has been taking sulfadimethoxine. Today it has fever up to 39oC and maculopapular rash on its face.
Except of rash the child’s skin has no changes. What is your diagnosis?
A. Measles B. Allergic rash C. Rubella
D. Scarlet fever E. Pseudotuberculosis
9. Blood serum of a newborn contains antibodies to measles virus. What kind of immunity is
this indicative of?
A. Natural passive B. Natural active C. Artificial passive
D. Artificial active E. Heredoimmunity
10. Examination of a child who has recently recovered from measles revealed in the soft
tissues of cheeks and perineum some inaccurate, edematic, red-and-black, slightly fluctuating areas.
What complication is it?
A. Humid gangrene B. Dry gangrene
C. Gas gangrene D. Pressure sore E. Trophic ulcer
11. A 6 year old child was delivered to the hospital because of measles pneumonia. On the
mucous membrane of a cheek a dentist revealed an ill-defined greish area 2х2,5 cm large. Soft
tissues are edematic and foul-smelling. The most probable diagnosis of the dentist should be:
A. Noma B. Gangrenous stomatitis
C. Pustular stomatitis D. Phlegmonous stomatitis
E. Ulcerous stomatitis
12. A measles outbreak has occurred in a maternity clinic. What class of maternal antibodies
can provide a newborn with immunity to measles virus?
A. IgE B. IgG C. IgM D. IgA E. IgD

340
13. A 1.5-year-old boy, who previously received no regular immunization, was in contact with
measles patient. For urgent specific prevention, the child was administered donor gamma globulin.
What type of immunity develops in this case?
A. Natural B. Post-vaccination C. Local D. Antitoxic E.
Passive

Rubella virus
1. A 2 y.o. girl has been ill for 3 days. Today she has low–grade fever, severe catarrhal
symptoms, non-abundant maculopapular rash on her buttocks and enlarged occipital glands. What is
your diagnosis?
A. Rubella B. Scarlet fever C. Measles
D. Adenoviral infection E. Pseudotuberculosis
2. A pregnant woman was registered in an antenatal clinic and underwent complex
examination for a number of infections. Blood serum contained IgM to the rubella virus. What is this
result indicative of?
A. Of primary infection B. Of a chronic process
C. The woman is healthy D. Of exacerbation of a chronic disease
E. Of recurring infection with rubella virus
3. A pregnant woman was detected with IgM to rubella virus. An obstetrician-gynecologist
recommended therapeutic abortion due to the high risk of teratogenic affection of the fetus.
Detection of IgM was of great importance as it is these specific immunoglobulins that:
A. Indicate recent infection
B. Penetrate placental barrier
C. Have the largest molecular weight
D. Are associated with anaphylactic reactions
E. Are the main factor of antiviral protection
4. A 36 y.o. woman is in the 12-th week of her first pregnancy. She was treated for infertility
in the past. She contacted a child who fell ill with rubella 2 days after their meeting. Woman doesn’t
know if she has ever been infected with rubella. What is the adequate tactics?
A. Monitoring of the specific IgG, IgM with the ELISA
B. Fetus wastage C. Immunoglobulin injection
D. Cyclovin prescription E. Interferon prescription

Hepatitis viruses
1. A 20 y.o. patient was admitted to the hospital with complaints of having skin and sclera
icteritiousness, dark urine, single vomiting, appetite loss, body temperature rise up to 38 0 C for 2
days. Three weeks ago he went in for fishing and shared his dishes with friends. Objectively: the
patient is flabby, t - 36, 80 C, skin and scleras are icteritious, liver sticks from under the costal margin
by 3 cm, it is sensitive; spleen isn’t palpable. Urine is dark, stool is partly acholic. What is the most
probable diagnosis?
A. Virus A hepatitis B. Leptospirosis C. Infectious mononucleosis
D. Hemolytic anemia E. Intestinal yersiniosis
2. A hepatitis outbreak was registered in a settlement. This episode is connected with water
factor. What hepatitis virus could have caused the infective outbreak in this settlement?
A. E B. C C. D D. G E. B
3. In a township there were registered an outbreak of hepatitis, which was attributed to
water supply. What hepatitis virus could be the cause of the outbreak in this township?
A. Hepatitis E virus B. Hepatitis C virus C. Hepatitis D virus
D. Hepatitis G virus E. Hepatitis B virus
4. A patient has been admitted to the infectious diseases department for malaise, fever up to
38oC, jaundice. A few months ago, the patient underwent blood transfusion. The doctor suspected
viral hepatitis B. What are the principal methods of laboratory diagnosis of hepatitis B?

341
A. Serological and gene diagnostics
B. Virus isolation in cell culture and its identification by CPE
C. Detection of virions in blood by electron microscopy
D. Isolation of the virus in laboratory animals (Nt)
E. Isolation of the virus in chicken embryos
5. The donor who for a long time didn't donate the blood was investigated with IFA method.
Anti-HBs antibodies were revealed. What does positive result of IFA in this case mean?
A. Chronic hepatitis В B. Acute hepatitis B
C. Acute hepatitis C D. Chronic hepatitis С E. Previous hepatitis B
6. Examination of a 27-year-old donor who had not donated blood for a long time revealed
HBs antibodies detected by ELISA method. In this case, the positive ELISA results indicate that the
donor:
A. Had hepatitis B
B. Has acute hepatitis B C. Has acute hepatitis C
D. Has chronic hepatitis B E. Has chronic hepatitis C
7. Immune-enzyme reaction revealed in blood serum HBs-antigen. What disease is this
antigene associated with?
A. Viral hepatitis type B B. Viral hepatitis A
C. AIDS D. Tuberculosis E. Syphilis
8. Immune-enzyme assay has detected HBs antigen in blood serum. What disease is it
characteristic of?
A. Viral hepatitis type B
B. Viral hepatitis type A
C. AIDS
D. Tuberculosis
E. Syphilis
9. During surgical operation a blood transfusion was made. The blood must be checked to
find antigens of some disease. What disease is expected to be found?
A. Viral hepatitis B B. Viral hepatitis A
C. Adenovirus D. Enterovirus E. Viral hepatitis E
10. Hepatitis B is diagnosed through laboratory tests that determine the presence of HBA-
DNA in blood serum of the patient. What reference method is applied for this purpose?
A. Polymerase chain reaction B. Hybridization method
C. Hybridization signal amplification method
D. Ligase chain reaction method E. ELISA diagnostic method
11. A patient has been hospitalized with provisional diagnosis of virus B hepatitis. Serological
reaction based on complementation of antigen with antibody chemically bound to peroxidase or
alkaline phosphatase has been used for disease diagnostics. What is the name of the applied
serological reaction?
A. Immune-enzyme analysis B. Radioimmunoassay technique
C. Immunofluorescence test D. Bordet-Gengou test
E. Antigen-binding assay
12. Professional dentists belong to the risk group concerning professional infection with viral
hepatitis type B. Name an effective method for active prevention of this disease among the dentists:
A. Vaccination with recombinant vaccine
B. Secure sterilization of medical instruments
C. Working with gum gloves on
D. Introduction of specific immunoglobuline
E. Introduction of interferonogenes
13. Dentists are at increased risk of being infected with the type B hepatitis. What
preparation should be used for reliable active prevention of this disease?
A. Recombinant vaccine of HBsAg proteins
B. Live type B hepatitis vaccine C. Specific immunoglobulin
D. Monoclonal HBsAg antibodies E. Antihepatitis serum

342
14. In order to eliminate occupational risks dental workers underwent vaccination. The
vaccine should protect them from a viral infection, whose pathogen may be found in blood of dental
patients who had had this infection or who are its chronic carriers. What vaccine was used?
A. Genetically engineered HBs antigen
B. Live measles vaccine C. Inactivated hepatitis A vaccine
D. Subunit influenza vaccine E. Anti-rabies vaccine
15. A 32 year old patient suffering from chronic viral hepatitis complains about dull pain in
the right subcostal area, nausea, dry mouth. Objectively: liver dimensions are 13-21-11 cm (according
to Kurlov), spleen is by 2 cm enlarged, aspartate aminotransferase is 3,2 micromole/l·h, alanine
aminotransferase - 4,8 millimole/l·h. Serological study revealed HBeAg, high concentration of DNA
HBV. What drug should be chosen for treatment of this patient?
A. α-interferon B. Acyclovir C. Remantadinum
D. Arabinoside monophosphate E. Essentiale-forte
16. It is known that infectious type B hepatitis is a systemic disease caused by the type B
hepatitis virus and characterized by a predominant liver affection. Choose from the below given list
the drugs for the etiotropic therapy of this infection:
A. Acyclovir B. Penicillin C. Tetracycline
D. Sulfanilamides E. Fluoroquinolones
17. A patient is registered for regular check-ups. Laboratory analyses foe viral hepatitis
diagnostics are made. In the blood serum only antibodies to HBsAg are detected. Such result is
indicative of:
A. Viral hepatitis type A. B. Past case of viral hepatitis type B.
C. Acute viral hepatitis type C. D. Acute viral hepatitis type B.
E. Chronic viral hepatitis type C.
18. Dentists have high risk of contracting viral hepatitis type B in the course of their duties
and therefore are subject to mandatory vaccination. What vaccine is used in such cases?
A. Recombinant vaccine. B. Live vaccine. C. Anatoxin.
D. Inactivated vaccine. E. Chemical vaccine.
In a township there was registered an outbreak of hepatitis, which had supposedly spread
through the water supply. What hepatitis virus could be the cause of the outbreak in this
township?
A. Hepatitis E virus B. Hepatitis C virus C. Hepatitis B virus
D. Hepatitis G virus E. Hepatitis D virus
19. During laboratory diagnostics of hepatitis C, it is necessary to detect the presence of
antibodies to hepatitis C virus in the patient’s blood serum. What test should be conducted in this
case?
A. Enzyme-linked immuno sorbent assay (ELISA)
B. Nucleic acid hybridization with signal amplification C. DNA probe
method
D. Ligase chain reaction E. Nucleic acid hybridization
20. To test donor blood for hepatitis B antigens, it is necessary to use highly sensitive
detection methods. What test should be used for this purpose?
A. Solid-phase enzyme-linked immunosorbent assay B. Indirect
hemagglutination
C. Complement binding D.
Immunoelectrophoresis
E. Indirect immunofluorescence
21. Lately, the laboratory diagnostics of hepatitis B includes detecting the presence of viral
DNA in the patient’s blood. What reaction is used to determine it?
A. Polymerase chain reaction B. Hemagglutination inhibition reaction
C. Indirect hemagglutination reaction D. Enzyme-linked immunosorbent assay
E. Complement fixation reaction
22. Hepatitis В is diagnosed through laboratory tests that determine the presence of HBA-
DNA in blood serum of the patient. What reference method is applied for this purpose?

343
A. Polymerase chain reaction B. Hybridization signal amplification method
C. ELISA diagnostic method D. Ligase chain reaction method
E. Hybridization method
23. It is known that hepatitis D virus belongs to defective viruses and can reproduce in the
host cells only in the presence of virus of:
A. Hepatitis G B. Hepatitis E C. Hepatitis C D. Hepatitis A E. Hepatitis B

HIV
1. RNA that contains AIDS virus penetrated into a leukocyte and by means of reverse
transcriptase forced a cell to synthesize a viral DNA. This process is based upon:
A. Reverse transcription B. Operon repression
C. Reverse translation D. Operon depression
E. Convariant replication
2. During reproduction of some RNA-containing viruses that cause tumors in animals, genetic
information can be transmitted in the opposite direction from the RNA to the DNA via a specific
enzyme. The enzyme of reverse transcription is called:
A. Reverse transcriptase B. DNA polymerase
C. Ligase D. Primase E. Topoisomerase
3. The examination of blood serum of a patient with immunodeficiency signs revealed
antybodies to gP120 and gP41 proteins. The presence of what infection of this patient does it
confirm?
A. НLTV-1-infection B. TORCH-infection
C. ЕСНО-infection D. НВV-infection E. HIV-infection
4. Blood analysis of a patient showed signs of HIVinfection (human immunodeficiency virus).
Which cells does HIV-virus primarily affect?
A. Cells that contain receptor T4 (T-helpers)
B. Cells that contain receptor IgM (B-lymphocytes)
C. Specialized nervous cells (neurons)
D. Mast cells E. Proliferating cells
5. A patient consulted an immunologist about diarrhea, weight loss within several months,
low-grade fever, enlarged lymph nodes. The doctor suspected HIV infection. What
immunocompetent cells must be studied in the first place?
A. Helper T-lymphocytes B. Suppressor T-lymphocytes
C. B-lymphocytes D. Monocytes E. Plasma cells
6. HIV displays the highest tropism towards the following blood cells:
A. T-helpers B. T-suppressors C. T-killers
D. Thrombocytes E. Erythrocytes
7. Examination of a young man in the AIDS centre produced a positive result of immune-
enzyme assay with HIV antigens. Patient’s complaints about state of his health were absent. What
can the positive result of immune-enzyme assay be evidence of?
A. HIV infection B. Being ill with AIDS
C. Being infected with HBV D. Having had AIDS recently
E. HBV persistence
8. It was revealed that T-lymphocytes were affected by HIV. Virus enzyme - reverse
transcriptase (RNA-dependent DNA polymerase) - catalyzes the synthesis of:
A. DNA on the matrix of virus mRNA
B. Virus informational RNA on the matrix of DNA
C. DNA on virus ribosomal RNA D. Viral DNA on DNA matrix
E. mRNA on the matrix of virus protein
9. T-lymphocytes are determined to be affected with HIV. In this case viral enzyme reverse
transcriptase (RNA-dependent DNA polymerase) catalyzes the synthesis of:
A. DNA based on the viral RNA matrix

344
B. Viral DNA based on DNA matrix
C. Viral RNA based on DNA matrix
D. Viral protein based on RNA matrix
E. Informational RNA based on viral protein matrix
10. Quite often the cause of secondary immunodeficiency is an infection involvement, when
the causative agents propagate directly in the cells of immune system and destroy it. The following
diseases are characterized by:
A. Infectious mononucleosis, AIDS
B. Tuberculosis, mycobacteriosis C. Poliomyelitis, type A hepatitis
D. Dysentery, cholera E. Q-febris, epidemic typhus
11. Often the cause of secondary immunodeficiency is an infectious affection of an organism,
when agents reproduce directly in the cells of immune system and destroy them. Specify the
diseases, during which the described above occurs:
A. Poliomyelitis, viral hepatitis A
B. Q fever, typhus
C. Tuberculosis, mycobacteriosis
D. Infectious mononucleosis, AIDS
E. Dysentery, cholera
12. The immunoblot detected gp120 protein in the blood serum. This protein is typical for
the following disease:
A. HIV-infection B. Virus B hepatitis C. Tuberculosis
D. Syphilis E. Poliomyelitis
13. HIV has gp41 and gp120 on its surface interacts with target cells of an organism. Which of
the following human lymphocyte antigens is gp120 complementary bound with?
A. CD 4 B. CD 3 C. CD 8 D. CD 19 E. CD 28
14. A 26 year old manual worker complained of 3 weeks history of fevers and fatigue, weight
loss with no other symptoms. Physical findings: Temperature 37,6oC, Ps- 88 bpm, blood pressure
115/70mmHg, superficial lymph nodes (occipital, submental, cervical, axillary) are enlarged, neither
tender nor painful. Rubella-like rash on the trunk and extremities. Herpes simplex lesions on the lips.
Candidosis of oral cavity. What infectious disease would you suspect?
A. HIV infection B. Influenza C. Rubella
D. Infectious mononucleosis E. Tuberculosis
15. A 24 y.o. woman presents with prolonged fever, nocturnal sweating. She’s lost weight for
7 kg during the last 3 months. She had irregular intercourses. On examination: enlargement of all
lymphaden groups, hepatolienal syndrom. In blood: WBC – 2,2 x 109/L. What is the most likely
diagnosis?
A. HIV-infection B. Lymphogranulomatosis C. Tuberculosis
D. Chroniosepsis E. Infectious mononucleosis
16. A 35-year-old female patient has HIV at the AIDS stage. On the skin of the lower
extremities and palatine mucosa there appeared rusty red spots, bright red nodules of various sizes.
One of the nodules was taken for histological study. It revealed a lot of randomly distributed thin-
walled vessels lined with endothelium, the bundles of spindle cells containing hemosiderin. What
kind of tumor developed in the patient?
A. Kaposi’s sarcoma B. Hemangioma
C. Burkitt’s lymphoma D. Lymphangioma
E. Fibrosarcoma
17. An HIV-positive patient’s cause of death is acute pulmonary insufficiency resulting from
pneumonia. Pathohistological investigation of lungs has revealed transformed cells resemble owl’s
eye. Name the pneumonia causative agent:
A. Cytomegalovirus B. Pneumococcus
C. Influenza virus D. Candida fungi E. Toxoplasma
18. A doctor examined a patient with recurrent aphthous stomatitis with concominant
candidosis and decided to eliminate a possibility of HIV-infection. What examination can help to clear
the situation up and make a provisional diagnosis?

345
A. Immune-enzyme analysis B. Gel precipitation reaction
C. Reaction of hemagglutination inhibition
D. Phase-contrasr microscopy E. Reaction of hemagglutination
19. Medical examination of a 19-year-old worker revealed generalized lymphadenopathy
mainly affecting the posterior cervical, axillary and ulnar lymph nodes. There are multiple injection
marks on the elbow bend skin. The man denies taking drugs, the presence of injection marks ascribes
to influenza treatment. Blood count: RBCs - 3,2x1012/l, Hb- 100 g/l, WBCs - 3,1x109/l, moderate
lymphopenia. What study is required in the first place?
A. ELISA for HIV B. Immunogram C. Sternal puncture
D. X-ray of lungs E. Lymph node biopsy
20. The 28 y.o. woman applied to doctor because of limited loss of the hair. In the anamnesis
- she had frequent headache indisposition, arthromyalgia, fever, irregular casual sexual life, drug
user. RW is negative. What examination must be done first?
A. Examination for HIV B. Examination for neuropathology
C. Examination for gonorrhea
D. Examination for fungi E. Examination for trichomoniasis
21. Mass serological diagnosis of HIV infection is made by means of enzyme-linked
immunosorbent assay techniques. What standard component of the reaction must be adsorbed on
the solid phase of the test system?
A. HIV antigens B. Monoclonal HIV antibodies
C. Specific immunoglobulins D. Enzyme-marked HIV antibodies
E. Substrates to determine enzyme activity
22. A doctor was addressed by a 30-year old man. There is a probability of the patient being
HIV-positive. To clarify the diagnosis the doctor proposed to perform polymerase chain reaction. The
basic process in this kind of investigation is:
A. Gene amplification B. Transcription
C. Genetic recombination D. Genomic mutation
E. Chromosome mutation
23. A man is a carrier of HIV that is an RNA virus. The cells of this patient synthesize
viral DNA. This process is based on:
A. Transcription B. Repair C. Replication
D. Reverse transcription E. Translation

346
SPECIAL BACTERIOLOGY
Staphylococcus
1. At the laboratory experiment the leukocyte culture was mixed with staphylococci.
neutrophile leukocytes engulfed and digested bacterial cells. This process is termed:
A. Phagocytosis B. Pinocytosis C. Diffusion
D. Facilitated diffusion E. Osmosis
2. In the surgical department of a hospital there was an outbreak of hospital infection that
showed itself in often postoperative wound abscesses. Bacteriological examination of pus revealed
aurococcus. What examination shall be conducted to find out the source of this causative agent
among the department personnel?
A. Phagotyping B. Microscopical examination
C. Serological identification D. Biochemical identification
E. Estimation of antibiotic susceptibility
3. Because of suspected intrahospital infection in the neonatal department of the maternity
home the inspection was carried out. In some children and on some general things Staphylococcus
aureus was revealed. What properties of these cultures allow to establish their origin from one
source?
A. Antibioticogramma B. Antigenic structure
C. Biochemical activity D. Phagotype
E. Chromogenesis
4. In the surgical department of a dental clinic cases of hospital-acquired staphylococcal
infection were registered which was caused by strains with multiple drug resistance. Such feature
can be identified by presence of:
A. R-plasmids B. F-plasmids C. Exotoxins
D. Temperate bacteriophages E. Virulent bacteriophages
5. Staphylococci grow well in ordinary media but inoculation of blood and egg-yolk salt agar
should be done to separate pure bacterial cultures from diseased tissue. What is the purpose of
those media?
A. To define disease-producing factor
B. To define tinctorial properties C. To study antigenic properties
D. To define bacterial mobility E. To define antibiotic susceptibility
6. During inspectation of dental tools for sterility in one case gram-positive cocci were
detected. They were situated in clusters and yielded positive plasma coagulation reaction; the cocci
were fermenting mannitol in anaerobic conditions and exhibiting lecithinase activity. What
microorganism as detected?
A. St. saprophiticus
B. St. epidermidis
C. Corynebacterium xerosis
D. Staph. aureus
E. Str. pyogenes
7. Microbiological purity of tableted drugs had been tested at factory. Samples cultivation in
mannitol salt agar resulted in growth of golden-yellow colonies, microscopic examination of colonies
detected grampositive globular bacteria positioned in clusters; microorganisms had plasma
coagulation properties. What pure bacterial culture was obtained?
A. Staphylococcus aureus B. Enterobacteriaceae
C. Staphylococcus epidermidis D. Staph. saprophyticus
E. Pseudomonas aeruginosa
8. Examination of a patient with pustular skin lesions allowed to isolate a causative agent
that forms in the blood agar roundish yellow middle-sized colonies surrounded by haemolysis zone.
Smears from the colonies contain irregularshaped clusters of gram-positive cocci. The culture is
oxidase- and catalasepositive, ferments mannitol and synthesizes plasmocoagulase. What causative
agent was isolated?
A. Staphylococcus aureus B. Streptococcus agalactiae
C. Streptococcus pyogenes D. Staphylococcus epidermidis E. -

347
9. A 65-year-old man has purulent abscess on his neck. Analyses revealed a culture of gram-
positive cocci with plasmocoagulase activity. This culture relates most likely to:
A. Staphylococcus aureus B. Streptococcus pyogenes
C. Staphylococcus epidermidis D. Staph. saprophyticus E. –
10. From the purulent exudate of a patient with odontogenic phlegmon a pure culture of
Gram(+) microorganisms was segregated. This culture was lecithinously active, coagulated plasma of
a rabbit, decomposed mannitol under anaerobe conditions. What microorganism may have
contributed to the origin of suppurative complication?
A. S.aureus B. S.epidermidis C. S.pyogenes
D. S.viridans E. S.mutans
11. Purulent discharges of a patient with a mandibulofacial phlegmon contain spheroid
microorganisms making S-shaped colonies with golden pigment that produce lecithinase,
plasmocoagulase, hemolysin and decompose mannitol under anaerobic conditions. Specify the kind
of microorganisms that had caused the suppuration:
A. S. aureus B. Str. pyogenes C. Str. mutans
D. S. epidermidis E. Str. sanguis
12. A 15-year-old patient consulted a dermatologist about a painful lump in the armpit.
Objectively: there is a walnut-sized node, lymphadenitis, infiltration of the surrounding tissues. The
patient has been diagnosed with hidradenitis. What is the most likely causative agent of this disease?
A. Staphylococci B. Streptococci C. Proteus vulgaris
D. Pseudomonas aeruginosa E. Mixed infection
13. Microscopy of a smear obtained from a patient with acute purulent periostitis revealed
gram-positive bacteria arranged in clusters resembling bunch of grapes. What microorganisms is this
morphology typical for?
A. Staphylococci B.Sarcina C.Tetracocci D.Candida fungi E.Streptococci
14. Pathogenic staphylococcus was obtained from the purulent wound of the patient.
Its antibiotic sensitivity was determined to be as follows: penicillin growth inhibition zone - 8
mm; oxacillin - 9 mm, ampicillin - 10 mm, gentamicin - 22 mm, lincomycin - 11 mm. What
antibiotic should be chosen for treatment in this case?
A. Gentamicin B. Penicillin C. Ampicillin
D. Oxacillin E. Lincomycin
15. During an outbreak of a hospital-acquired infection, pure cultures of S. aureus were
grown after inoculation of the samples obtained from the nasopharynges of the medical personnel
and from wound drainage of the surgical patients. What tests are necessary to determine the likely
sourse of infection?
A. Phage-typing of the obtained cultures B. Sero-identification
C. Antibiotic sensitivity test D. Repeated inoculations E. Biochemical
profiles

Streptococcus
1. Analysis of sputum taken from a patient with suspected pneumonia revealed rather
elongated gram-positive diplococci with somewhat pointed opposite ends. What microorganisms
were revealed in the sputum?
A. Streptococcus pneumoniae
B. Staphylococcus aureus C. Klebsiella pneumoniae
D. Neisseria meningitidis E. Neisseria gonorrhoeae
2. Analysis of sputum taken from a patient with suspected pneumonia revealed slightly
elongated gram-positive diplococci with tapered opposite ends. What microorganisms were revealed
in the sputum?
A. Stretpococcus pneumoniae
B. Neasseria gonorrhoeae
C. Neisseria meningitidis

348
D. Staphylococcus aureus
E. Klebsiella pneumoniae
3. A patient has a suspected pneumonia. In his sputum there were revealed grampositive
diplococci, prolonged with the slightly pointed opposite ands. What microorganisms are revealed in
the sputum?
A. Staphylococcus aureus B. Neisseria gonorrhoeae
C. Neisseria meningitidis D. Klebsiella pneumoniae
E. Streptococcus pneumoniae
4. A 22 y.o. man complains of acute throat pain, increasing upon swallowing during 3 days.
Body temperature 38,30 C, neck lymph nodules are slightly enlarged and painful. Pharyngoscopically
– tonsilar hyperemia, enlargement and edema, tonsils are covered by round yellow fibrinous patches
around crypts openings. Beta-haemolytic streptococcus in swab analysis. What is the diagnosis?
A. Acute membranous tonsilitis
B. Acute follicular tonsilitis C. Pharyngeal diphtheria
D. Infectious mononucleosis E. Pharyngeal candidosis
5. A 10-year-old girl was admitted to a hospital with carditis presentations. It is known from
the anamnesis that two weeks ago she had exacerbation of chronic tonsillitis. What is the most likely
etiological factor in this case?
A. Streptococcus B.Staphylococci C.Pneumococci D.Klebsiella E.Proteus
6. A 10-year-old child has painful swallowing, neck edema, temperature rise up to 39 oC, the
whole body is covered with bright-red petechial rash. Back of the throat and tonsils are hyperemic,
the tongue is crimsoncolored. Tonsillar surface is covered with isolated grayish-colored necrosis nidi.
What disease is it?
A. Scarlet fever B. Meningococcal nasopharyngitis
C. Diphtheria D. Influenza E. Measles
7. A child is 10 years old. The following presentations have developed: sharp pain during
swallowing, swollen neck, body temperature rise up to 39,0oC, bright-red finely papular rash all over
the body. Pharynx and tonsils are sharply hyperemic ("flaming pharynx"), "crimson tongue". On the
tonsils surface there are isolated greyish necrosis focuses. What disease it might be?
A. Scarlet fever B. Meningococcal nasopharyngitis
C. Diphtheria D. Influenza E. Measles
8. A boy is 7 y.o. Objectively: against the background of hyperemic skin there is knobby
bright-pink rash on his forehead, neck, at the bottom of abdomen, in the popliteal spaces; nasolabial
triangle is pale. Examination of oropharyngeal surface revealed localized bright-red hyperemia;
tonsils are swollen, soft, lacunas contain pus, tongue is crimson. Cervical lymph nodes are enlarged,
dense and painful. What is the most probable diagnosis?
A. Scarlet fever B. Rubella C. Whooping cough
D. Diphtheria E. Infectious mononucleosis
9. A 9-year-old boy has acute onset of disease: sore throat, body temperature rise up to
o
39,5 C; on the second day diffuse skin rash was detected all over his skin except for nasolabial
triangle. On examination of oral cavity: crimson tongue, "flaming pharynx", necrotic tonsillitis. What
diagnosis is the most likely?
A. Scarlet fever B. Measles C. Diphtheria
D. Influenza E. Meningococcemia
10. A 7 y.o. girl fell ill abruptly: fever, headache, severe sore throat, vomiting. Minute bright
red rash appear in her reddened skin in 3 hours. It is more intensive in axillae and groin. Mucous
membrane of oropharynx is hyperemic. Greyish patches is on the tonsills. Submaxillary lymph nodes
are enlarged and painful. What is your diagnosis?
A. Scarlet fever B. Measles C. Rubella
D. Pseudotuberculosis E. Enteroviral infection
11. In a 2-year-old child with catarrhal presentations and skin rash a pediatrician suspected
scarlet fever. The child was given intracutaneously a small dose of serum antibody to the
streptococcal erythrogenic toxin; on the site of injection the rash disappeared. What do the reaction
results mean?

349
A. The clinical diagnosis was confirmed
B. The child has hypersensitivity to the erythrogenic toxin
C. The disease wasn’t caused by haemolytic streptococcus
D. The whole serum dose may be injected intravenously
E. The child has very weak immune system
12. A 2 y.o. child has catarrhal effects and skin eruption. A doctor suspected scarlet fever.
The child was injected intracutaneously with some serum to the erythrogenic streptococcus toxin, on
the spot of injection the eruption disappeared. What do the reaction results mean?
A. They confirm the clinical diagnosis
B. The child has hypersensitivity to the erythrogenic toxin
C. The disease was caused by non-hemolytic streptococcus
D. The complete dose of serum could be introduced intravenously
E. The child’s immune system is very weakened
13. A 7 year old child often suffers from streptococcus angina. Doctor suspected
development of rheumatism and administered serological examination. The provisional diagnosis will
be most probably confirmed by presence of antibodies to the following streptococcus antigen:
A. O-streptolysin B. C-carbohydrate C. M-protein
D. Erythrogenic toxin E. Capsular polysaccharide
14. A 40-year-old woman was diagnosed with glomerulonephritis based on her clinical
symptoms and the results of urine analysis. Anamnesis states chronic tonsillitis. What
microorganisms are the most likely cause for her kidney damage?
A. Streptococci B. Staphylococci C. Escherichia
D. Mycoplasma E. Meningococci
15. A male patient has been diagnosed with acute post-streptococcal glomerulonephritis. It is
most likely that the lesion of the basement membrane of renal corpuscles was caused by the
following allergic reaction:
A. Immune complex B. Anaphylactic C. Cytotoxic
D. Delayed E. Stimulating
16. A patient has been diagnosed with acute glomerulonephritis that developed after he had
had streptococcal infection. It is most likely that the affection of basal glomerular membrane is
caused by an allergic reaction of the following type:
A. Immune complex B. Anaphylactic
C. Cytotoxic D. Delayed E. Stimulating
17. 10 days after having quinsy caused by beta-hemolytic streptococcus a 6-year-old child
exhibited symptoms of glomerulonephritis. What mechanism of glomerular lesion is most likely in
this case?
A. Immunocomplex B. Anaphylaxis C. Cellular cytotoxicity
D. Atopy E. Antibody-dependent cell-mediated cytolysis
18. Blood of a patient with presumable sepsis was inoculated into sugar broth. There
appeared bottom sediment. Repeated inoculation into blood agar caused growth of small
transparent round colonies surrounded by hemolysis zone. Examination of a smear from the
sediment revealed gram-positive cocci in form of long chains. What microorganisms are present in
blood of this patient?
A. Streptococci B. Micrococci C. Staphylococci
D. Tetracocci E. Sarcina
19. Microscopy of a dental plaque revealed a large number of cocci arranged in pairs and
strings, as well as Gram-positive bacilli which were likely to be the cause of cariogenesis. What
microorganism associations are involved in the development of dental caries?
A. S. mutans, streptococci and lactobacilli
B. S. salyvarius, streptococci and lactobacilli
C. S. mutans, streptococci and corynebacteria
D. S. aureus and lactobacilli E. S.salyvarius, streptococci and enterococci
20. A patient with streptococcal gingival infection was prescribed a medication that contains
beta lactam ring in its structure. What preparation belongs to this group?

350
A. Benzylpenicillin B. Rifampicin C. Erythromycin
D. Streptomicin sulfate E. Chloramphenicol
21. A patient with streptococcal infection of gums was prescribed a drug that contained beta-
lactam ring in its structure. Which drug relates to this group?
A. Benzylpenicillin B. Rifampicin C. Erythromycin
D. Streptomycin sulfate E. Chloramphenicol
22. During examination of a patient a dentist revealed a lot of "white spots zones of enamel
demineralization. What microorganisms take part in the development of this process?
A. Streptococcus mutans B. Streptococcus salivarius
C. Streptococcus pyogenes D. Veilonella parvula
E. Staphylococcus epidermidis
23. A 40-year-old woman was diagnosed with glomerulonephritis based on her clinical
symptoms and the results of urine analysis. Anamnesis states chronic tonsillitis. What
microorganisms are the most likely cause for her kidney damage?
A. Mycoplasma B. Staphylococci
C. Meningococci D. Escherichia E. Streptococci
24. Analysis of sputum taken from a patient with suspected pneumonia revealed slightly
elongated gram-positive diplococci with tapered opposite ends. What microorganisms were revealed
in the sputum?
A. Streptococcus pneumoniae B. Staphylococcus aureus
C. Klebsiella pneumoniae D. Neisseria meningitides E. Neisseria
gonorrhoeae
25. A patient with streptococcal infection of the gingiva was prescribed a drug with β-
lactam ring in its structure. What drug of those listed below belongs to this pharmacological
group?
A. Erythromycin B. Streptomycin sulfate
C. Benzylpenicillin D. Rifampicin E. Levomycetin (Chloramphenicol)
26. Microscopy of the sputum of a patient with lobar pneumonia revealed a large number of
gram-positive lancet-shaped encapsulated diplococci. What causative agent is it?
A. Streptococcus pneumoniae B. Klebsiella pneumoniae
C. Chlamidia pneumonia D. Staphylococcus aureus E. Escherichia
coli
27. A 10-year-old child has painful swallowing, neck edema, temperature rise up to 39.0°C,
the whole body is covered with bright-red petechial rash. Back of the throat and tonsils are
hyperemic, the tongue is crimsoncolored. Tonsillar surface is covered with isolated grayish-colored
necrosis nidi. What disease is it?
A. Scarlet fever B. Measles
C. Meningococcal nasopharyngitis D. Influenza E. Diphtheria

Gonococci
1. Bacteriological examination of purulent discharges from the urethra revealed gram-
negative bacteria looking like coffee beans. They were localized in the leukocytes and could
decompose glucose and maltose to acid. These are the causative agents of the following disease:
A. Gonorrhoea B. Syphilis C. Melioidosis
D. Soft chancre E. Veneral lymphogranulomatosis
2. Gramnegative bin-shaped diplococcus inside and outside of leucocytes were detected on
bacteriological examination of the purulent exudates from the cervix of the uterus. Name the
causative agent of purulent inflammation of the cervix of the uterus.
A. Haemophilus vaginalis B. Chlamidia trachomatis
C. Calymmatobacterium granulomatis
D. Neisseria gonorrhoeae E. Trichomonas vaginalis

351
3. Bacteriological analysis of purulent discharges from urethra revealed presence of gram-
negative bacteria resembling of coffee beans, which were able to decompose glucose and maltose
into acid. They were found in the leukocytes. These bacteria are causative agents of the following
disease:
A. Gonorrhoea B. Syphilis C. Melioidosis
D. Ulcul molle E. Venereal lymphogranulomatosis
4. Bacteriological examination of purulent discharges from urethra revealed some bacteria
that had negative Gram’s stain, resembled of coffee corns, decomposed glucose and maltose up to
acid. They were located in leukocutes. What disease do they cause?
A. Gonorrhea B. Syphilis C. Pseudocholera
D. Soft chancre E. Venereal lymphogranulomatosis
5. Microscopy of a female patient’s swabs made from vaginal secretion revealed
gramnegative bean-shaped diplococci. What provisional diagnosis can be made?
A. Gonorrhoea B. Syphilis C. Clamidiosis
D. Mycoplasmosis E. Toxoplasmosis
6. Microscopic study of discharges from urethra of a patient suffering from acute urethritis
revealed bean-shaped microorganisms up to 1 micrometer in diameter arranged in pairs and placed
inside the leukocytes. What microorganisms are these?
A. Gonococci B. Meningococci C. Tetracocci
D. Streptococci E. Staphylococci
7. A newborn child has hyperemia, edema of mouth mucous membrane, small erosions with
viscous muco-purulent discharge. Examination of muco-pus smears reveals a great number of
leukocytes containing gram-negative diplococci. The same microorganisms can be found outside the
leukocytes. What is the most probable diagnosis?
A. Gonococcal stomatitis B. Toxoplasmosis
C. Prenatal syphilis
D. Staphylococcal stomatitis E. Blennorrhea
8. An 18 year old woman consulted a gynecologist about the pain in the lower part of
abdomen, fever up to 37,5oC, considerable mucopurulent discharges from the genital tracts, painful
urination. Vaginal and speculum examination results: the urethra is infiltrated, cervix of the uterus is
hyperemic, erosive. The uterus is painful, ovaries are painful, thickened; fornixes are free.
Bacterioscopy test revealed diplococcus. What diagnosis is the most probable?
A. Recent acute ascending gonorrhea B. Trichomoniasis
C. Candydomycosis D. Chronic gonorrhea E. Chlamydiosis
9. On the fifth day after a casual sexual contact a 25-year-old female patient consulted a
doctor about purulent discharges from the genital tracts and itch. Vaginal examination showed that
vaginal part of uterine cervix was hyperemic and edematic. There was an erosive area around the
external orifice of uterus. There were mucopurulent profuse discharges from the cervical canal,
uterine body and appendages exhibited no changes. Bacterioscopic examination revealed bean-
shaped diplococci that became red after Gram’s staining. What is the most likely diagnosis?
A. Acute gonorrheal endocervicitis
B. Trichomonal colpitis C. Candidal vulvovaginitis
D. Bacterial vaginism E. Clamydial endocervicitis
10. A 28-year-old patient has been admitted to the gynecological department three days
after a casual coitus. She complains about pain in her lower abdomen and during urination, profuse
purulent discharges from the vagina, body temperature rise up to 37,8oC. The patient was diagnosed
with acute bilateral adnexitis. Supplemental examination revealed: the 4th degree of purity of the
vaginal secretion, leukocytes within the whole visual field, diplococcal bacteria located both intra-
and extracellularly. What is the etiology of acute adnexitis in this patient?
A. Gonorrheal B. Colibacterial C. Chlamydial
D. Trichomonadal E. Staphylococcal
11. A patient consulted a venereologist about painful urination, reddening of the external
opening of urethra, profuse purulent discharges from the urethra. He considers himself to be ill for 3
days. He also associates the disease with a casual sexual contact that took place for about a week

352
ago. If provisional diagnosis "acute gonorrheal urethritis" will be confirmed, then bacteriological
study of urethral discharges will reveal:
A. Gram-negative diplococci B. Gram-positive diplococci
C. Spirochaete D. Proteus vulgaris E. Mycoplasma
12. A female woman has been clinically diagnosed with gonorrhea. Which of the following
studies can be used to confirm the diagnosis?
A. Microscopy of the pathological material
B. Disinfection of laboratory animals C. Bacteriophage test
D. Hemagglutination reaction E. Immobilization reaction
13. Clinical diagnosis of a female patient was gonorrhoea. What examination method can be
applied for confirmation of this diagnosis?
A. Microscopy of pathological material
B. Infection of laboratory animals C. Test with bacteriophage
D. Hemagglutination reaction E. Immobilization reaction
14. A 30-year-old female patient has been delivered to the gynaecological department with
complaints of acute pain in the lower abdomen and body temperature 38,8oC. In history: sexual life
out of wedlock and two artificial abortions. Gynaecological examination reveals no changes of
uterine. The appendages are enlarged and painful on both sides. Vaginal discharges are purulent and
profuse. What study is required to confirm a diagnosis?
A. Bacteriological and bacterioscopic analysis
B. Hysteroscopy C. Curettage of uterine cavity
D. Colposcopy E. Laparoscopy
15. On admission a 35-year-old female reports acute abdominal pain, fever up to 38,8oC,
mucopurulent discharges. The patient is nulliparous, has a history of 2 artificial abortions. The patient
is unmarried, has sexual contacts. Gynecological examination reveals no uterus changes. Appendages
are enlarged, bilaterally painful. There is profuse purulent vaginal discharge. What study is required
to confirm the diagnosis?
A. Bacteriologic and bacteriascopic studies
B. Hysteroscopy C. Curettage of uterine cavity
D. Vaginoscopy E. Laparoscopy
16. A patient who came to the doctor because of his infertility was administered to make
tests for toxoplasmosis and chronic gonorrhoea. Which reaction should be performed to reveal
latent toxoplasmosis and chronic gonorrhoea in this patient?
A. RIHA - Reverse indirect hemagglutination assay
B. RDHA - Reverse direct hemagglutination assay
C. IFA - Immunofluorescence assay D. Immunoblot analysis
E. (R)CFT- Reiter's complement fixation test
17. A doctor made the diagnosis of gonorrhoea. It was known from the anamnesis that a
patient had had gonorrhoea before and he had been treated completely. What type of infection can
this new disease be attributed to?
A. Superinfection B. Reinfection
C. Secondary infection D. Relapse E. Autoinfection
18. Gonorrhoea was revealed in the patient on bacterioscopy of the smear from urethra.
Taking into account that medecines for gonorrhoea are fluorquinolones, patient should be
prescribed:
A. Ciprofloxacin B. Fluorouracil C. Cefazoline
D. Urosulfan E. Furazolidone
19. A patient has been diagnosed with gonorrhea. As fluoroquinolones are the drugs of
choice for treatment of gonorrhea the patient should be prescribed:
A. Ciprofloxacin
B. Furazolidone
C. Fluorouracil
D. Sulfacarbamide (Urosulfanum)
E. Cefazolin

353
19. An ophthalmologist suspects blennorrhea (gonococcal conjunctivitis) in a child with signs
of suppurative keratocojunctivitis. What laboratory diagnostics should be conducted to confirm the
diagnosis?
A. Microscopy and bacteriological analysis.
B. Serum diagnostics and allergy test. C. Biological analysis and
phagodiagnostics.
D. Biological analysis and allergy test. E. Microscopy and serum diagnostics.
20. The ophthalmologist noticed a purulent discharge from conjunctiva of a newborn.
Microscopy of the smear obtained from conjunctiva found there a large number of leukocytes, as
well as gram-negative bean-shaped diplococci located inside leukocytes. What is the causative agent
of this disease?
A. Staphylococcus aureus B. Neisseria catarrhalis
C. Streptococcus pyogenes D. Staphylococcus epidermidis E. Neisseria
gonorrhoae

Meningococci
1. While studying blood and mucus samples from the nasopharynx, a bacteriologist took
certain measures to conserve the pathogens in the material. Bacterioscopic study revealed the
presence of gram-negative cocci looking like coffee beans and arranged in pairs or tetrads. Name the
pathogen that was isolated by the bacteriologist:
A. Neisseria meningitidis B. Staphilococcus aureus
C. Neisseria gonorrhoeae D. Moraxella lacunata
E. Acinetobacter calcoaceticus
2. Bacterioscopy of nasopharyngeal mucus taken from a 2,5 year old child with
nasopharyngitis revealed gram-negative diplococci looking like coffee grains. What organs of the
child are most likely to be affected if these microorganisms penetrate the blood?
A. Brain tunics B. Cardiac valves C. Renal glomeruli
D. Urogenital tracts E. Lymph nodes
3. A young woman suddenly developed fever up to 39 oC accompanied by a strong headache.
Examination revealed marked nuchal rigidity. Spinal puncture was performed. Gram-stained smear of
cerebrospinal fluid contained many neutrophils and Gram-negative diplococci. What bacteria could
be the cause of this disease?
A. Neisseria meningitidis B. Streptococcus pneumonia
C. Haemophilus influenza D. Staphylococcus aureus
E. Pseudomonas aeruginosa
4. A 5 y.o. child had a temperature rise up to 400C, acute headache, vomiting, anxiety, chill. 4
days later there appeared hemorrhagic skin eruption, oliguria and adrenal insufficiency that caused
death. Bacteriological examination of smears from the child’s pharynx revealed meningococcus.
What disease form was revealed?
A. Meningococcemia B. Meningococcal meningitis
C. Meningoencephalitis D. Meningicoccal nasopharyngitis E. –
5. A 5 year old child has the following symptoms: body temperature up to 40 oC, acute
headache, vomiting, anxiety, shiver. 4 days later there appeared hemorrhagic skin rash, oliguria and
adrenal insufficiency that caused death. Bacteriological examination of pharyngeal smears revealed
meningococcus. What form of meningococcal infection was it?
A. Meningococcemia B. Meningococcal meningitis
C. Meningoencephalitis D. Meningococcal nasopharyngitis E. –
6. In winter a 3-year-old child has sharp rise of body temperature up to 40 oC. Hemorrhagic
rash is observed on the skin and mucosa. Bean-shaped gram-negative microorganisms situated in
pairs are detected in the blood. What provisional diagnosis can be made?
A. Meningococcosis B. Gonorrhea
C. Scarlet fever D. Influenza E. Diphtheria

354
7. A 4 month old child fell seriously ill: body temperature rose up to 38,5oC, the child became
inert and had a single vomiting. 10 hours later there appeared rash over the buttocks and lower
limbs in form of petechiae, spots and papules. Some haemorrhagic elements have necrosis in the
center. What is the most probable disease?
A. Meningococcemia B. Rubella C. Influenza
D. Haemorrhagic vasculitis E. Scarlet fever
8. The disease of a 21 y.o. patient began with raise of temperature up to 39,0 0C, headache,
chill, repeated vomiting. Rigidity of occipital muscles is determined. The analysis of liquor revealed:
cytosis - 1237 in 1 ml, including: 84% of neutrophils, 16 % of lymphocytes. On bacterioscopy: gram-
negative cocci are found in liquor. What is the most probable disease?
A. Meningococcal infection: purulent meningitis
B. Meningococcal infection: serous meningitis
C. Secondary purulent meningitis
D. Serous meningitis E. Infectious mononucleosis
9. A 1,5 y.o. child fell seriously ill: chill, body temperature rise up to 40,10 C, then rapid
dropping to 36, 20 C, skin is covered with voluminous hemorrhagic rash and purple cyanotic spots.
Extremities are cold, face features are sharpened. Diagnosis: meningococcosis, fulminant form,
infection-toxic shock. What antibiotic must be used at the pre-admission stage?
A. Soluble Levomycetine succinate B. Penicillin
C. Lincomycin D. Gentamycin E. Sulfamonometoxin

Escherichia
1. A child with suspected colienteritis was delivered to the infectious disease hospital.
Colibacillus was obtained from the child’s feces. How to determine whether this bacillus is of
pathogenic variety?
A. By means of bacteriophage typing
B. Agglutination reaction with serum O
C. Microscopy of stained smears
D. Based on the nature of its growth in Endo medium
E. Based on its biochemical properties
2. Red colonies spread in the large quantity in the Endo culture medium were revealed on
bacteriological stool examination of a 4-month-old baby with the symptoms of acute bowel infection.
What microorganism can it be?
A. Escherichia B. Salmonella C. Staphylococcus
D. Streptococcus E. Shigella
3. On bacteriological examination of the defecation of a 4-months-old baby with the
symptoms of acute bowel infection there were revealed red colonies spread in the large quantity in
the Endo environment. What microorganism can it be?
A. Staphylococcus B. Streptococcus
C. Shigella D. Salmonella E. Escherichia
4. From the defecation of a 6-year-old ill child, who has artificial feeding, the intestinal
bacillus with antigen structure 0-111 is excreted. What is the diagnosis?
A. Food poisoning B. Dysentery-like disease
C. Gastroenteritis D. Coli-enteritis E. Cholera-like disease
5. Stool culture test of a 6-month-old bottlefed baby revealed a strain of intestinal rod-
shaped bacteria of antigen structure 0-111.What diagnosis can be made?
A. Colienteritis
B. Gastroenteritis
C. Choleriform disease
D. Food poisoning
E. Dysentery-like disease
6. Among junior children of an orphanage an outbreak of intestinal infection with signs of
coli-enteritis was registered. In order to identify isolated causative agent it is necessary to:
A. Study antigenic properties of the causative agent

355
B. To determine sensitivity to antibiotics
C. To study sensitivity to bacteriophages
D. To study biochemical properties of the causative agent
E. To study virulence of the causative agent
7. 12 year old child has the ulcer disease of stomach. What is the etiology of this disease?
A. Intestinal bacillus B. Helicobacter pylori
C. Salmonella D. Lambliosis E. Influenza
8. A 12-year-old boy has been hospitalized for suspected food poisoning. The fecal samples
were inoculated on the Endo agar, which resulted in growth of a large number of colorless colonies.
What microorganism is most likely to be EXCLUDED from the list of possible causative agents of the
disease?
A. Escherichia coli B. Salmonella enteritidis
C. Proteus vulgaris D. Pseudomonas aeruginosa
E. Yersinia enterocolitica
9. Feces of a child with an acute intestinal infection were inoculated onto Endo medium,
which resulted in the growth of numerous raspberry-red colonies with a metallic sheen. It indicates
that the colibacillus is the likely causative agent of the disease. What property of the causative agent
was used to distinguish it from other intestinal bacteria?
A. Antigenic structure
B. Capsule formation
C. Toxigenicity
D. Oxidase activity
E. Fermentation of lactose

Salmonella
1. Bacteriological examination of a patient with food poisoning required inoculation of a pure
culture of bacteria with the following properties: gramnegative movable bacillus that grows in the
Endo’s medium in form of colourless colonies. A representative of which species caused this disease?
A. Salmonella B. Shigella C. Yersinia D. Esherichia E. Citrobacter
2. On bacteriological study of rinsing water of the patient with food poisoning, the pure
bacterial culture was inoculated with the following properties: gram-negative motile bacillus in the
Endo environment grows like achromic colony. Representative of what genus has caused the illness?
A. Yersinia B. Citrobacter C. Salmonella D. Shigella E. Escherichia
3. It was reported an outbreak of food poisoning connected with consumption of pastry that
had been stored at a room temperature and had duck eggs as one of the ingredients. What
microorganisms might have caused this disease?
A. Salmonella B. Colon bacilli
C. Staphylococci D. Legionella E. Comma bacilli
4. A 33-year-old male patient developed a condition that had a stormy clinical course: chills,
fever up to 39oC, vomiting, epigastric pain, diarrhea with watery smelly feces. 6 hours before, he ate
a raw egg, fried potatoes with stewed meat, drank some juice. What pathogen is likely to have
caused this condition?
A. Salmonella B. Colibacillus C. Campylobacter
D. Shigella E. Vibrio cholerae
5. A 10 month old boy has been ill for 5 days after consumption of unboiled milk. Body
temperature is 38 − 39oC, there is vomiting, liquid stool. The child is pale and inert. His tongue is
covered with white deposition. Heart sounds are muffled. Abdomen is swollen, there is borborygmus
in the region of ubbilicus, liver is enlarged by 3 cm. Stool is liquid, dark-green, with admixtures of
mucus, 5 times a day. What is the most probable diagnosis?
A. Salmonellosis B. Staphylococcal enteric infection
C. Escherichiosis D. Acute shigellosis E. Rotaviral infection

356
6. A nurse of the kindergarten was taken to the hospital with complaints of acute pain in
parumbilical region, convulsions of lower limbs, multiple bile vomiting, frequent watery foul feces of
green color in huge amounts. At the same time all the staff in the kindergarden got ill. Two days ago
all of them ate cottage cheese with sour cream. General condition of patients is of moderate
severity. Temperature 38,2oC. Heart tones: rhythmic and muted. Heart rate 95/min, arterial pressure:
160 mm/Hg. Abdomen is slightly swollen, painful. Liver +2 cm. What is the most likely diagnosis?
A. Salmonellosis B. Dysentery C. Cholera
D. Food toxic infection E. Enterovirus infection
7. A 28 year old patient was admitted to the clinic with complaints of the temperature rise up
to 39,0oC, headache, weakness, constipation on the 9th day of the disease. On examination: single
roseolas on the skin of the abdomen are present. The pulse rate is 78 bpm. The liver is enlarged by 2
cm. What is the most probable diagnosis?
A. Typhoid fever B. Leptospirosis C. Brucellosis
D. Sepsis E. Malaria
8. A 28 y.o. male patient was admitted to the hospital because of high temperature 39 0C,
headache, generalized fatigue, constipation, sleep disorder for 9 days. There are sporadic roseolas on
the abdomen, pulse- 78 bpm, liver is enlarged for 2 cm. What is the most probable diagnosis?
A. Abdominal typhoid B. Typhus
C. Sepsis D. Brucellosis E. Leptospirosis
9. A patient with complaints of 3-daylong fever, general weakness, loss of appetite came to
visit the infectionist. The doctor suspected enteric fever. Which method of laboratory diagnosis is the
best to confirm the diagnosis?
A. Detachment of blood culture
B. Detachment of myeloculture C. Detachment of feces culture
D. Detachment of urine culture E. Detachment of pure culture
10. A 50 year old locksmith was diagnosed with typhoid fever. The patient lives in a separate
apartment with all facilities. Apart of him there are also 2 adults in his family. What actions should be
taken about persons communicating with the patient?
A. Bacteriological study B. Antibiotic prophylaxis
C. Isolation D. Dispensary observation E. Vaccination
11. During the repeated Widal’s agglutination test it was noticed that the ratio of antibody
titers and O-antigens S.typhi in the patient’s serum had increased from 1:100 to 1:400. How would
you interpret these results?
A. The patient has typhoid fever
B. The patient is an acute carrier of typhoid microbes
C. The patient is a chronic carrier of typhoid microbes
D. The patient previously had typhoid fever
E. The patient was previously vaccinated against typhoid fever
12. To conduct serum diagnostics of typhoid fever a test is carried out, when diagnosticums
of three types of microorganisms are being added into different solutions of patient’s serum; then
agglutinate formation is checked. Name the author of that test.
A. Widal B.Wassermann C. Ouchterlony
D. Wright E. Sachs-Witebsky
13. A 50-year-old patient with typhoid fever was treated with Levomycetin, the next day his
condition became worse, temperature rised to 39,60С. What caused worthening?
A. Reinfection B. Irresponsiveness of an agent to the levomycetin
C. Allergic reaction D. The effect of endotoxin agent
E. Secondary infection addition
14. A patient was hospitalized into the infectious diseases unit on the 11th day since the
disease onset and provisionally diagnosed with typhoid fever. What biological material should be
collected from the patient for the analyzes at this stage?
A. Roseola secretion B. Blood serum C. Bile D. Feces E. Urine
15. A patient was brought into the infectional diseases hospital on the 8th day since the
disease onset. The patient complains of headache, malaise, and weakness. A sample of blood

357
was taken for the serological test. Widal agglutination test results with blood sample diluted
1:200 and typhoid fever O-diagnosticum were positive. What diagnosis can be made based on
the results of this test?
A. Typhoid fever B. Leptospirosis C. Tuberculosis
D. Dysentery E. Cholera
16. A 43-year-old man seeks evaluation at an emergency department with complaints of
fever with chills, malaise, diffuse abdominal pain for over a week, diarrhea and loss of appetite. He
says that his symptoms ard progressively getting worse. He recalls that the fever began slowly znd
climbed its way up stepwise to the current 39.8°C. His blood pressure is 110/70 mm Hg. A physical
exam reveals as coated tongue, enlarged spleen and rose spots on the abdomen. Serologic study
shows the agglutinin O titer of 1:200 by the Widal test. Which of the following is the most likely
causative organism for this patient’s condition?
A. Salmonella typhi B. Enterohemorrhagic E. coli
C. Leptospira interrogans D. Mycobacterium tuberculosis
E. Vibrio cholera
17. After a celebratory dinner, several people, who were eating a cake with duck eggs cream,
developed food poisoning. What genus of bacteria is the most likely cause of food poisoning in this
case?
A. Salmonella B. Clostridium
C. Corynebacterium D. Yersinia E. Shigella
18. A man with suspected typhoid fever was admitted to the infectious diseases hospital on
the 3rd day of illness. What microbiological method should be used for diagnostics in this case?
A. Method of isolation of bile culture B. Method of isolation of the causative agent from
CSF
C. Method of isolation of a blood culture D. Method of isolation of a stool culture
E. Method of isolation of a urine culture
19. A man has a case of epidemic typhus 5 years ago. After an acute respiratory viral disease,
against the background of weakened immune system, he developed signs of typhus again. The
exacerbation occurred because of the causative agents, remaining in his body. What type of
infections is it?
A. Reinfection B. Co-infection C. Superinfection D. Secondary infection E.
Relapse
20. Autopsy of the body revealed waxy degeneration of the rectus abdominis muscles. In the
terminal segment of the small intestine the are ulcers 3-5 cm in diameter. The ulcer walls are covered
in a crumbling gray-1yellow substance. The ulcer edges are moderately raised above the mucosa.
Widal test is positive. Make the diagnosis:
A. Typhoid fever B. Nonspecific ulcerative colitis
C. Crohn’s disease D. Dysentery
E. Relapsing fever
Shigella
1. Autopsy of a 46-year-old man revealed multiple brown-and-green layers and
hemmorhages on the mucous membrane of rectum and sigmoid colon; slime and some blood in
colon lumen; histologically - fibrinous colitis. In course of bacteriological analysis of colon contents
S.sonne were found. What is the most probable diagnosis?
A. Dysentery B. Cholera C. Salmonellosis
D. Yersiniosis E. Crohn’s disease
2. A patient has been suffering from diarrhea for 5 day. On the fifth day colonoscopy
revealed that membrane of rectum was inflamed, there were greyish-green films closely adhering to
the subjacent tissue. What is the most probable diagnosis?
A. Dysentery B. Nonspecific ulcerous colitis
C. Typhoid fever D. Salmonellosis E. Crohn’s disease
3. A 71-year-old man had been presenting with diarrhea for 10 days. The feces had
admixtures of blood and mucus. He was delivered to a hospital in grave condition and died 2 days
later. Bacteriological analysis revealed Shigella. What was the main disease?

358
A. Dysentery B. Typhoid fever C. Salmonellosis
D. Nonspecific ulcerous colitis E. Yersiniosis
4. A 28 y.o. man fell seriously ill, he feels chill, has got a fever, body temperature raised up to
38,50C, paroxysmal pain in the left iliac region, frequent defecation in form of fluid bloody and
mucous mass. Abdomen palpation reveals painfulness in its left half, sigmoid colon is spasmed. What
is the most probable diagnosis?
A. Acute dysentery B. Amebiasis C. Colibacillosis
D. Nonspecific ulcerative colitis
E. Malignant tumors of large intestine
5. A 30-year-old patient complains of paroxysmal abdominal pain, frequent liquid stools up to
10 times a day. Throughout the first 3 days he had a fever, since the 2nd day of disease there were
scant liquid stools mixed with mucus. On palpation: tenderness of all colon segments. Sigmoid colon
was found spastic. What is your provisional diagnosis?
A. Acute dysentery B. Intestinal amebiasis
C. Salmonellosis D. Cholera E. Balantidiasis
6. A 6-year-old child complains of frequent liquid stool and vomiting. On the 2nd day of
disease the child presented with inertness, temperature rise up to 38,2 oC, Ps- 150 bpm, scaphoid
abdomen, palpatory painful sigmoid colon, defecation 10 times a day with liquid, scarce stool with
mucus and streaks of green. What is a provisional diagnosis?
A. Shigellosis B. Salmonellosis C. Escherichiosis
D. Intestinal amebiasis E. Yersiniosis
7. An 8-year-old boy fell ill acutely: he presents with fever, weakness, headache, abdominal
pain, recurrent vomiting, then diarrhea and tenesmus. Stools occur 12 times daily, are scanty,
contain a lot of mucus, pus, streaks of blood. His sigmoid gut is tender and hardened. What is your
diagnosis?
A. Dysentery B. Salmonellosis C. Cholera
D. Staphylococcal gastroenteritis E. Escherichiosis
8. A patient with suspected dysentery has been admitted to the infectious diseases hospital.
Which basic method of laboratory diagnosis must be applied in the first place?
A. Bacteriological B. Serological C. Allergic
D. Biological E. Microscopic
9. A patient was taken to the hospital with complaints of headache, high temperature,
frequent stool, stomach pain with tenesmus. Doctor made a clinical diagnosis dysentery and sent the
material (excrements) to the bacteriological laboratory for analysis. What diagnostic method should
the laboratory doctor use to confirm or to disprove the clinical diagnosis?
A. Bacteriological B. Biological C. Bacterioscopic
D. Serological E. Allergic
10. The infectious diseases department of a hospital admitted a patient with nausea, liquid
stool with mucus and blood streaks, fever, weakness. Dysentery was suspected. What method of
laboratory diagnostics should be applied to confirm the diagnosis?
A. Bacteriological B. Serological C. Mycological
D. Microscopic E. Protozoological
11. A patient diagnosed with acute dysentery has been treated for 3 days in an infectious
diseases hospital. On admission there were complaints of high temperature, stomachache and fluid
excrements with mucus as often as 8-10 times a day. What sample should be taken for analysis?
A. Feces B. Urine C. Bile D. Liquor E. Blood
12. A patient recovered from Sonne dysentery and was once more infected with the same
causative agent. What is such infection form called?
A. Reinfection B. Recidivation C. Superinfection
D. Persisting infection E. Chronic infection
13. From the fecal sample of a patient Shigella sonne were isolated. What additional studies
are required to identify the source of infection?
A. Phage-typing of the isolated pure culture
B. Antibiogram C. Precipitation reaction

359
D. Complement-fixation reaction E. Neutralization reaction
14. For the purpose of retrospective diagnostics of recent bacterial dysentery it was decided
to perform serological examination of blood serum in order to determine antibody titer towards
Shiga bacilli. What of the following reactions should be applied?
A. Passive hemagglutination B. Bordet-Gengou test
C. Precipitation D. Hemolysis E. Bacteriolysis
15. Retrospective diagnostics of bacterial dysentery involved serological analysis of blood
serum intended for determination of Shigella antibody titer. Which of the following reactions should
be applied for this purpose?
A. Passive haemagglutination B. Complement binding
C. Precipitation D. Haemolysis E. Bacteriolysis
16. Retrospective diagnostics of old bacillary dysentery required serologic examination of
blood serum in order to determine blood titer to the shigella. What reaction should be applied for
this purpose?
A. Reaction of passive hemagglutination
B. Bordet-Gengou test C. Precipitation reaction
D. Hemolysis reaction E. Bacteriolysis reaction
17. Antigens of Sonne shigella placed on the objects of outdoor environment and foodstuffs
can be revealed by means of a certain test with application of a diagnostic test system that includes a
polystyrene tray with adsorbed specific antibodies. What reaction is it?
A. Immune-enzyme assay B. Immunofluorescence test
C. Passive inverse hemagglutination test
D. Direct hemagglutination test E. Immunoelectrophoresis test
18. A patient has been diagnosed with bacillary dysentery. What drug of those listed below
should be prescribed?
A. Amoxicillin B. Benzylpenicillin sodium salt
C. Isonicotinic acid hydrazide (Isoniazid)
D. Itraconazole E. Acyclovir
19. Patient was admitted to the infection unit with diagnosis of bacterial dysentery. On
laboratory studies it was revealed that causative element is sensitive to the many antimicrobial
medicines, but patient has anemia. What medicine is contra-indicated to the patient?
A. Enteroseptol B. Phthalazol C. Levomycetin
D. Ampicillin E. Furazolidone
20. A mother of 4-year-old child complains that the child developed elevated body
temperature, tenesmus, diarrhea, and abdominal pain attacks. The child attends a preschool facility.
Laboratory analysis detected mucus and blood admixtures in the child’s feces. Name the changes
that occur in the gastrointestinal tract during dysentery:
A. Enterocolitis B. Enteritis
C. Gastroenteritis D. Gastritis E. Colitis
21. A 23-year-old woman presents to the emergency department complaining of bloody
diarrhea, fatigue and confusion. A few days earlier, she went to a fast food restaurant for a birthday
party. Her friends are experiencing similar symptoms. Laboratory studies show anemia. Which of the
following would you most likely obtain for microbiologic testing?
A. Stool B. Bile C. Cerebrospinal fluid D. Urine
E. Blood
22. Shigella capable of producing exotoxin was obtained from a patient diagnosed with
dysentery. What Shigella species is it?
A. Shigella dysenteriae B. Shigella flexneri
C. Shigella sonnei D. Shigella boydii E. Shigella Newcastle

Cholera

360
1. Microscopy of a smear taken from the film that appeared on the peptone water 6 hours
after seeding and culturing of a fecal sample in a thermostat revealed mobile gram-negative bacteria
curved in form of a comma that didn’t make spores or capsules. What microorganisms were
revealed?
A. Vibrios B. Spirochetes C. Clostridia D. Corynebacteria E. Spirilla
2. Vomiting matters of a patient suspected of having cholera were delivered to the
bacteriological laboratory. The material was used for preparing a "hanging drop" specimen. What
type of microscopy will be applied for identification of the causative agent by its mobility?
A. Phase-contrast microscopy B. Electron microscopy
C. Immune and electron microscopy
D. Fluorescence microscopy E. Immersion microscopy
3. Bacilli were extracted from investigated sample. The bacilli are curved, extremely mobile,
gram-negative, form no spores or capsules, have anaerobic form of respiration. They form
transparent smooth colonies in alkaline agar, ferment saccharose and mannose into acid, produce
exotoxin, fibrinolysin, collagenase, and hyaluronidase. What agent was extracted?
A. Comma bacillus
B. Proteus
C. Dysentery bacillus
D. Blue pus bacillus
E. Colibacillus
4. Patient with diarrhoea was admitted to the infection unit. Gramnegative curved rod-like
bacteria were founded on bacterioscopic examination of faecal masses. What is the most likely
disease in this patient?
A. Typhoid fever B. Intestinal form of plague C. Diphtheria
D. Cholera E. Salmonellosis gastroenteritis
5. 6 hours after the initial inoculation of water sample into 1% peptone water, the growth of
a culture in form of a thin pellicle on the medium surface was registered. Such cultural properties are
typical for the causative agent of the following disease:
A. Cholera B. Plague C. Tuberculosis
D. Dysentery E. Pseudotuberculosis
6. Initial inoculation of water in 1% peptone water resulted in growth of a thin film on the
medium surface in 6 hours. Such cultural properties are characteristic of causative agent of the
following disesase:
A. Cholera B. Plague C. Tuberculosis
D. Dysentery E. Pseudotuberculosis
7. After inoculation of feces sample into the 1% alkaline peptonic water and 8-hour
incubation in the thermostat at a temperature of 37 oC a culture in form of a tender bluish film has
grown. Such cultural properties are typical for the causative agent of the following disease:
A. Cholera B. Plague C. Typhoid fever
D. Paratyphoid fever A E. Dysentery
8. A man is suffering from diarrhea. In summer he spent his vacation in the south at the sea
coast. Bacteria with the following properties were detected in his feces: gram-negative curved
mobile monotrichous bacilli that do not produce spores or capsules. They are undemanding to
nutrient medium but require alkaline reaction (рН 8,5-9,5). Described are the agents of the following
enteric infection:
A. Cholera B. Shigellosis C. Typhoid fever
D. Colienteritis E. Pseudotuberculosis
9. A patient had been suffering from profuse diarrhea and vomiting for 2 days. He died from
acute dehydration. Autopsy revealed that the intestinal wall was edematic and hyperemic, with
multiple haemorrhages in the mucous membrane. Intestine lumen contains whitish fluid resembling
of rice water. What disease caused death?
A. Cholera B. Dysentery C. Salmonellosis
D. Typhoid fever E. Enterocolitis

361
10. A patient with marked manifestations of exsicosis died in the infectious disease hospital.
Postmortem examination results: the corpse with contracted muscles, dry skin and mucous
membranes, thick and dark blood in veins, edematous plethoric mucosa, distended bowel loops, the
lumen contains about 4 liters of rice-water fluid. What is the most likely diagnosis?
A. Cholera B. Enteric fever C. Dysentery
D. Anthrax, intestinal form E. Yersiniosis
11. Autopsy of a 42-year-old man revealed a distinctly dilated lumen of small intestine filled
with rice-water-like liquid. The intestine wall was edematic with lots of petechial haemorrhages on
the mucosa. What infectious disease is the described enteritis typical for?
A. Cholera B. Dysentery C. Salmonellosis
D. Amebiasis E. Typhoid fever
12. The disease began acutely. The frequent watery stool developed 6 hours ago. The body’s
temperature is normal. Then the vomiting was joined. On examination: his voice is hoarse, eyes are
deeply sunken in the orbits. The pulse is frequent. Blood pressure is low. There is no urine. What is
the preliminary diagnosis?
A. Cholera B. Toxic food-borne infection
C. Salmonellosis D. Dysentery E. Typhoid fever
13. A man in grave condition was delivered to the admission ward of a hospital on the 2nd
day of illness. Examination revealed body temperature of 36,1oC, sharpened features of face, dry skin
that makes a fold, aphonia, convulsive twitching of some muscle groups. Acrocyanosis is present.
Heart sounds are muffled, Ps is 102 bpm, AP is 50/20mm Hg. Abdomen is soft, drawn-in, painless.
Anuria is present. Stool is liquid in form of rice water. What is the most probable diagnosis?
A. Cholera B. Acute dysentery
C. Salmonellosis D. Escherichiosis E. Intestinal amebiasis
13. From the feces of a patient with acute gastroenteritis a pure culture of
microorganisms was obtained. The microorganisms are small mobile slightly curved gram-
negative bacilli that within 6 hours grow into a light blue film on the 1% alkaline peptone
water. Such properties are characteristic of the following microorganism:
A. Bacillus B. Clostridium C. Spirochete D. Spirillum E. Vibrio
14. Initial inoculation of water in 1% peptone water resulted in growth of a thin film
on the medium surface in 6 hours. Such cultural properties are characteristic of causative
agent of the following disease:
A. Dysentery B. Pseudotuberculosis C. Tuberculosis D. Plague E.
Cholera

Diphtheria
1. A child is presumably ill with diphtheria. A specimen of affected mucous membrane of his
pharynx was taken for analysis. The smear was stained and microscopic examination revealed yellow
rods with dark blue thickenings on their ends. What structural element of a germ cell was revealed in
the detected microorganisms?
A. Volutin granules B. Plasmids C. Capsule
D. Spores E. Flagella
2. On examination of a 6-year-old child the doctor noticed greyish film on the child’s tonsils.
Microscopy of the smear stained by Neisser method detected there Corynebacterium diphtheria.
What morphologic feature was the most indicative for determining the type of the agent?
A. Fence-like position of the agent’s cells
B. Spores that exceed cells in diameter
C. Localization of the causative agent within macrophages
D. Polar placement of volutin granules
E. Presence of the capsule
3. A smear from the tonsillar coating of a patient with suspected diphtheria was found to
contain blue bacilli with a thickening at the poles. What method of smear staining was used?
A. Leffler B. Burri C. Hins D. Gram E. Neisser

362
4. Microscopy of smear preparation stained with methylene blue revealed bacilli with
clublike expansions on their ends similar to C.diphtheriae. What additional method of staining should
be used to verify this assumption?
A. Neisser B. Kozlovsky C. Ziehl-Neelsen
D. Zdrodovsky E. Aujeszky
5. There are several cases of children from boarding school suffering from sore throat.
Microscopy of tonsil smears stained according to Neisser method has revealed thin yellow bacilli with
dark brown grains on their ends placed in the shape of Roman numeral five. What infection can be
suspected in this case?
A. Diphtheria B. Infectious mononucleosis
C. Listeriosis D. Tonsillitis E. Scarlet fever
6. A sample taken from the pharynx of a patient with angina was inoculated on the blood-
tellurite agar. This resulted in growth of grey, radially striated (in form of rosettes) colonies up to 4-5
mm in diameter. Microscopically there can be seen gram-positive rods with club-shaped ends
arranged in form of spread fingers. What microorganisms are these?
A. Corynebacteria diphtheriae B. Clostr. botulinum
C. Diphtheroids D. Streptococci E. Streptobacilli
7. After inoculation of the material obtained from the pharynx of an angina patient onto the
blood-tellurite agar, grey colonies could be observed. They were 4-5 mm in diameter, radially striated
(in form of rosettes). Microscopical examination revealed gram-positive bacilli with clavate swollen
ends arranged in form of wide-spread fingers. Identify these microorganisms:
A. Diphtheria corynebacteria B. Clostr. botulinum
C. Diphtheroids D. Streptococci E. Streptobacilli
8. Inoculum from pharynx of a patient ill with angina was inoculated into bloodtellurite agar.
It resulted in growth of grey, radially striated (in form of rosettes) colonies 4-5 mm in diameter.
Grampositive bacilli with clublike thickenings on their ends placed in form of spread wide apart
fingers are visible by microscope. What microorganisms are these?
A. Diphtheria corynebacteria B. Botulism clostridia
C. Diphtheroids D. Streptococci E. Streptobacilli
9. A 4-year-old child presents with general weakness, sore throat and deglutitive problem.
After his examination a doctor suspected diphtheria and sent the material to the bacteriological
laboratory. In order to determine the diphtheria causative agent the material should be inoculated
into the following differential diagnostic medium:
A. Blood tellurite agar B. Endo’s agar
C. Ploskyrev’s agar D. Sabouraud’s agar E. Levenshtein-Yessen agar
10. From the nasopharynx of a 5-year-old child it was excreted a microorganism which is
identical to Corynebacterium diphtheriae dose according to morphological and biochemical signs.
Microorganism does not produce exotoxin. As a result of what process can this microorganism
become toxigenic?
A. Cultivation in the telluric media B. Chromosome mutation
C. Passing through the organism of the sensative animals
D. Phage conversion E. Growing with antiserum
11. Autopsy of a dead 6-year-old child revealed a marked edema of the soft tissues of neck
and enlarged tonsils. Pharyngeal mucosa was covered with numerous dense whitish-yellow pellicles
exposing deep ulcers after their removal. What infectious disease caused the death of the child?
A. Diphtheria B. Parainfluenza C. Scarlet fever
D. Whooping cough E. -
12. While examining a patient an otolaryngologist noticed hyperaemia and significantly
edematous tonsils with a grayish film upon them. Microscopical examination of this film revealed
some gram-positive bacilli placed at an angle with each other. What disease might be suspected?
A. Diphtheria B. Angina C. Scarlet fever
D. Meningococcal nasopharyngitis E. Epidemic parotitis
13. During examination of a 6-yearold child a doctor revealed greyish films on the pharyngeal
tonsils. Their removal provoked moderate haemorrhage. Bacterioscopy revealed gram-positive

363
clublike bacteria. What symptoms will develop in this child within the next few days if no specific
treatment is provided?
A. Toxic lesions of myocard, liver and kidney
B. Pulmonary edema C. Strong paroxysmal cough
D. Papulous skinrash E. Intermittent fever
14. A 5 y.o. girl has high temperature and sore throat. Objectively: soft palate edema, tonsills
are covered with grey films that can be hardly removed and leave deep bleeding tissue injuries. What
disease is the most probable?
A. Pharyngeal diphtheria B. Vincent’s angina
C. Lacunar angina D. Infectious mononucleosis
E. Necrotic angina
15. A diseased child has a high fever, sore throat, swelling of submandibular lymph nodes.
Objectively: pharyngeal mucosa is edematous, moderately hyperemic, the tonsils are enlarged,
covered with grayish membrane tightly adhering to the tissues above. Attempts to remove the
membrane produce the bleeding defects. What disease are these presentations typical for?
A. Diphtheria B. Catarrhal tonsillitis
C. Scarlet fever D. Meningitis E. Measles
16. A woman complains of high temperature to 380C, mild pain in the throat during 3 days.
On examination: angle lymphatic nodes of the jaw are 3 cm enlarged, palatinel tonsils are enlarged
and coated with grey plaque which spreads to the uvula and frontal palatinel arches. What is the
most probable diagnosis?
A. Larynx diphtheria B. Infectious mononucleosis
C. Vincent’s angina D. Agranulocytosis E. Oropharyngeal candidosis
17. A 4-year-old boy had untimely vaccination. He complains of painful swallowing,
headache, inertness, fever. Objectively: the child is pale, has enlarged anterior cervical lymph nodes,
swollen tonsils with cyanotic hyperemia, tonsils are covered with gray-white pellicles which cannot
be easily removed. When the pellicles are forcibly removed, the tonsils bleed. What is the most likely
diagnosis?
A. Oropharyngeal diphtheria B. Lacunar tonsillitis
C. Pseudomembranous tonsillitis
D. Infectious mononucleosis E. Follicular tonsillitis
18. A 24 year old patient complains about general weakness, dizziness, body temperature
rise up to 37,5oC, sore throat, neck edema, enlargement of submaxillary lymph nodes. Objectively:
mucous membrane of oropharynx is edematic and cyanotic, tonsils are enlarged and covered with
films that spread beyond the tonsils and cannot be easily removed. What is the leading mechanism
of this illness’ development?
A. Action of bacterial exotoxin
B. Action of bacterial endotoxin C. Allergic
D. Accumulation of suboxidated products E. Bacteriemia
19. From pharynx of a child with suspected diphtheria a pure culture of microorganisms was
isolated. Their morphological, tinctorial, cultural and biochemical properties appeared to be typical
for diphtheria causative agents. What study should be conducted in order to make a conclusion that
this is a pathogenic diphtheria bacillus?
A. Estimation of toxigenic properties
B. Estimation of proteolytic properties
C. Estimation of urease activity
D. Estimation of cystinase activity
E. Estimation of ability to decompose starch
20. Pure culture of microorganisms was obtained from pharynx of a child with suspected
diphtheria. Morphologic, tinctorial, cultural, and biochemical properties of the microorganisms were
studied and revealed to be characteristic of diphtheria agents. What investigation should be
additionally performed to make a conclusion, that these microorganisms are pathogenic diphtheria
bacilli?
A. Determine toxigenic properties B. Determine proteolytic properties

364
C. Determine urease activity D. Determine cystinase activity
E. Determine amylolytic activity
21. In order to determine toxigenicity of diphtheria bacilli a strip of filter paper impregnated
with antitoxic diphtheria serum was put on the dense nutrient medium. There were also inoculated a
microbial culture under examination and a strain that is known to be toxigenic. If the microbial
culture under examination produces exotoxin, this will result in formation of:
A. Precipitin lines B. Haemolysis zones
C. Zones of diffuse opacification
D. Zones of lecithovitellinous activity E. Precipitin ring
22. In order to estimate toxogenicity of diphtheria agents obtained from patients the cultures
were inoculated on Petri dish with nutrient agar on either side of a filter paper strip that was put into
the centre and moistened with antidiphtheric antitoxic serum. After incubation of inoculations in
agar the strip-like areas of medium turbidity were found between separate cultures and the strip of
filter paper. What immunological reaction was conducted?
A. Precipitation gel reaction B. Coomb’s test
C. Agglutination reaction
D. Rings precipitation reaction E. Opsonization
23. When examining a child the dentist found the deposit on both tonsils and suspected
atypical form of diphtheria. A smear was taken, and after the nutrient media inoculation the toxicity
of the isolated pure culture was determined. What reaction was used to determine the toxigenicity
of the isolated strain of diphtheria bacillus?
A. Gel precipitation reaction
B. Agglutination reaction on a glass slide
C. Complement binding reaction
D. Hemolysis reaction E. Ring precipitation reaction
24. A 7 year old girl was taken to an infectious diseases hospital. She had complaints of high
temperature, sore throat, general weakness. A doctor assumed diphtheria. What will be crucial proof
of diagnosis after defining pure culture of pathogenic organism?
A. Toxigenity test B. Detection of volutine granules
C. Hemolytic ability of pathogenic orhanism
D. Cystinase test E. Phagolysability
25. A 7 y.o. girl was admitted to the infectious diseases hospital with fever, sore throat,
common weakness. A doctor suspected diphtheria. What would be crucial for diagnosis confirmation
after pure culture of causative agent had been singled out?
A. Toxigenity test B. Detection of volutine granules
C. Hemolytic ability of a causative agent
D. Cystinase test E. Phagolysability
26. A patient has pure culture of diphtheria corynebacteria. What immunological reaction
should be used in order to determine bacteria toxigenity?
A. Precipitation in agar B. Agglutination
C. Complement binding D. Inhibition of hemagglutination
E. Indirect hemagglutination
27. In order to establish the level of antidiphtheritic immunity in a child it was decided to use
a passive hemagglutination test. This task can be completed by the sensibilization of erythrocytes by:
A. Diphtheria anatoxin B. Diphtheria antitoxin
C. Diphtheria bacillus antigens
D. Antidiphtheric serum E. -
28. A patient with suspected diphtheria went through bacterioscopic examination.
Examination of throat swab revealed rod-shaped bacteria with volutin granules. What etiotropic
preparation should be chosen in this case?
A. Antidiphtheric antitoxic serum B. Bacteriophage
C. Diphtheria antitoxin D. Eubiotic E. Interferon
29. Bacterioscopic examination of a smear from the pharynx of a diphtheria suspect revealed
bacilli with volutine granules. What etiotropic drug should be chosen in this case?

365
A. Antidiphtheritic antitoxic serum B. Bacteriophage
C. Interferon D. Eubiotic E. Diphtheritic anatoxin
30. A child with diphtheria 10 days after injection of antitoxic antidiphtherial serum has
developed skin rash, accompanied by severe itch, rising temperature up to 38 0C and joints pain.
What is the cause of these symptoms?
A. Delayed type of hypersensitivity B. Anaphylacsis
C. Contact allergy D. Atopia E. Serum sickness
31. A 16-year-old adolescent was vaccinated with DTP. In eight days there was stiffness and
pain in the joints, subfebrile temperature, urticarial skin eruption, enlargement of inguinal, cervical
lymph nodes and spleen. What kind of allergic reaction is observed?
A. Immunocomplex B. Hypersensitivity of immediate type
C. Cytoxic D. Hypersensitivity of delayed type E. –
32. It is necessary to carry out preventive vaccination of a student group because of an
occurrence of diphtheria. Which preparation should be used for the creation of the artificial active
immunity?
A. Diphtheria anatoxin B. Specific immunoglobulin
C. DTP vaccine D. Inactivated bacteria vaccine
E. Anti-diphtheria serum
33. Vaccination is done by means of a toxin that has been neutralized by a formaldehyde
(0,4%) at a temperature 37 – 400C for four weeks. Ramond was the first to apply this preparation for
diphtheria prophylaxis. What preparation is it?
A. Anatoxin B. Immunoglobulin C. Antitoxic serum
D. Adjuvant E. Inactivated vaccine
34. Diphtheria exotoxin had been treated with 0,3-0,4% formalin and kept in a thermostat for
30 days at a temperature of 40oC.What preparation was obtained as a result of these manipulations?
A. Anatoxin B. Antitoxin C. Diagnosticum
D. Therapeutic serum E. Diagnostic serum
35. In an inhabited locality there is an increase of diphtheria during the last 3 years with
separate outbursts in families. What measure can effectively influence the epidemic process of
diphtheria and reduce the morbidity rate to single cases?
A. Immunization of the population
B. Hospitalization of patients C. Detection of carriers
D. Early diagnostics E. Disinfection in disease focus
36. An 11-year-old girl has been immunized according to her age and in compliance with the
calendar dates. What vaccinations should the children receive at this age?
A. Diphtheria and tetanus B. TB
C. Polio D. Hepatitis B E. Pertussis
36. In a closed community it is necessary to determine community members immunity to
diphtheria and verify the need for their vaccination. What investigation is necessary in this case?
A. Check medical records for vaccination.
B. Test community members for diphtheria bacillus carriage.
C. Determine antitoxin titer by means of indirect hemagglutination assay.
D. Determine diphtheria antibody titer.
E. Determine community members immunity to diphtheria bacillus.
37. What drugs are used for specific treatment of diphtheria?
A. Placental gamma globulin. B. Anatoxin.
C. Native plasma. D. Antitoxic serum. E. Antibiotics.
38. A toxin neutralized with 0.4% formaldehyde under 37-40°C for 4 weeks is used
for vaccination. This preparation was first used by Gaston Ramon for diphtheria prevention.
Name this preparation:
A. Immunoglobulin B. Anatoxin C. Antitoxic serum
D. Inactivated vaccine E. Adjuvant

366
39. A 6-year-old girl with diphtheria is administrated an intravenous injection of diphtheria
antitoxin. Ten days after the initial administration of drug, she develops a pruritic rash, fever, and
arthralgias. Which of the following is the most likely diagnosis?
А. Serum sickness B. Delayed type hypersensitivity
C. Atopy D. Allergic contact dermatitis
E. Anaphylaxis
40. A child was hospitalized with diagnosis of diphtheria. What should be given to this child
for specific therapy?
A. Diphtheria antitoxin serum, antibiotics B. Diphtheria
bacteriophage
C. Diphtheria vaccines: DPT, DT, diphtheria vaccine D. Codivac vaccine,
sulfanilamides
E. Diphtheria anatoxin, antibiotics
41. To determine toxogenicity of diphtheria causative agents obtained from patients, the
cultures were inoculated in a Petri dish with nutrient agar, bilaterally to a strip of filter paper spotted
with antidiphtheric antitoxic serum and situated in the center of the Petri dish. After incubation of
the inoculated cultures in the agar, strip-like areas of medium turbidity formed between some of the
cultures and the filter paper. What immunological test was conducted?
A. Agar gel precipitation test B. Opsonization test
C. Agglutination test D. Coombs test E. Ring precipitin
test
42. A 5-year-old kindergartener has diphtheria. To find the carriers of the disease among the
kindergarten staff, samples of pharyngeal mucus were obtained from the employers. One of the
employers had gram-positive bacilli in her sample. They were situated at an angle to each other and
colored unevenly, when stained according to Loeffler. What method can confirm that this carrier is
dangerous to other people?
A. Toxin production test B. Mouse neutralization test
C. Immunofluorescence test D. Complement fixation test E. Serum
agglutination test
43. A smear prepared from material obtained from patient with suspected diphtheria
contains yellow bacilli with blue grains at their ends. What staining was used in this case?
A. Ziehl-Nielsen B. Kozlovsky
C. Romanovsky D. Neisser E. Loefler
44. There are several cases of children from boarding school suffering from sore throat.
Microscopy of tonsil smears stained according Neisser method has revealed thin yellow bacilli with
dark brown grains on their ends placed in the shape of Roman numeral five. What infection can be
suspected in this case?
A. Diphtheria B. Scarlet fever
C. Listeriosis D. Tonsilitis E. Infectious mononucleosis
45. Bacteriology testing is one of the methods for laboratory diagnostics of diphtheria. To
grow the colonies of C. diphtheriae, it is necessary to know the proper conditions for causative agent
cultivation. What nutrient media are optimal for Corynebacterium diphtheriae cultivation?
A. Sugar meat pepton broth, sugar meat pepton agar B. Salt egg yolk agar
C. Endomedium, Ploskirev medium D. Serum agar, ascitic agar
E. Blood agar, tellurite blood agar

Tuberculosis
1. A bacteriological laboratory received sputum sample of a patient suffering from
tuberculosis. Bacterioscopic examination of smears and detection of tuberculosis bacillus can be
realized by one of enrichment methods that involves processing of sputum only with solution of
caustic soda. What is this method called?
A. Homogenization B. Inactivation
C. Flotation D. Filtration E. Neutralization

367
2. Specimen of a patient’s sputum was stained with the following dyes and reagents: Ziehl’s
solution, methylene blue solution, 5% solutoin of sulfuric acid. What staining method was applied?
A. Ziehl-Neelsen B. Burri’s C. Gram’s D. Peshkov’s E. Neisser’s
3. A consumptive patient has an open pulmonary form of disease. Choose what sputum
staining should be selected for finding out the tubercle (Koch’s) bacillus?
A. Method of Ziel-Neelsen B. Method of Romanowsky-Giemsa
C. Method of Gram D. Method of Neisser
E. Method of Burry-Gins
4. Study of bacteriological sputum specimens stained by the Ziel-Neelsen method revealed
some bright-red acid-resistant bacilli that were found in groups or singularly. When inoculated onto
the nutrient media, the signs of their growth show up on the 10-15 day. These bacteria relate to the
following family:
A. Micobacterium tuberculosis B. Yersinia pseudotuberculosis
C. Histoplasma dubrosii D. Klebsiella rhinoscleromatis
E. Coxiella burnettii
5. Microscopy of stained (Ziehl-Neelsen staining) smears taken from the sputum of a patient
with chronic pulmonary disease revealed red bacilli. What property of tuberculous bacillus was
shown up?
A. Acid resistance B. Alkali resistance C. Alcohol resistance
D. Capsule formation E. Sporification
6. A bacteriological laboratory has received smears from the sputum of a patient with a
chronic pulmonary disease. Microscopical examination of the smears stained by the Ziehl-Neelsen
technique revealed red bacilli. What property of the tuberculosis bacillus has shown itself?
A. Acid resistance B. Alkali resistance C. Alcohol resistance
D. Capsule formation E. Spore formation
7. Sputum smears of a patient with chronic pulmonary disease were stained by Ziehl-Neelsen
method and analyzed in the bacteriological laboratory. Microscopy revealed red bacillus. What
property of tuberculosis myobacteria was found?
A. Acid resistance B. Alkali resistance C. Alcohol resistance
D. Encapsulation E. Spore-formation
8. While registering the child to the school Mantoux's test was made to define whether
revaccination was needed test result is negative. What does this result of the test mean?
A. Absence of antitoxic immunity to the tuberculosis
B. Presence of antibodies for tubercle bacillus
C. Absence of antibodies for tubercle bacillus
D. Presence of cell immunity to the tuberculosis
E. Absence of cell immunity to the tuberculosis
9. A child entering the school for the first time was given Mantoux test in order to determine
if there was a need for revaccination. The reaction was negative. What is the meaning of this test
result?
A. No cell-mediated immunity to tuberculosis
B. Availability of cell-mediated immunity to tuberculosis
C. No antibodies to the tuberculosis bacteria
D. No anti-toxic immunity to tuberculosis
E. Presence of antibodies to the tuberculosis bacteria
10. A 45 year old male died from disseminated tuberculosis. On autopsy the symptoms of
tuberculosis were confirmed by both microscopical and histological analyses. What kind of
hypersensitivity reaction underlies the process of granuloma development?
A. Delayed B. Antibody-dependent cytotoxicity
C. Complement-dependent cytotoxicity
D. Anaphylactic E. Immune complex
11. 48 hours after performing tuberculin test (Mantoux test) to a child a 10 mm papule
appeared on the spot of tuberculin introduction. What hypersensitivity mechanism underlies these
changes?

368
A. Cellular cytotoxicity B. Anaphylaxis
C. Antibody-dependent cytotoxicity
D. Immune complex cytotoxicity E. Granulomatosis
12. Tuberculine was injected intracutaneously to the child for tuberculin test. Marked
hyperemia, tissue infiltration developed on the place of injection in 24 hours. What mechanism
caused these modifications?
A. Cells cytotoxity B. Reagin type cytotoxity
C. Granuloma formation
D. Immunocomplex cytotoxity E. Antibody cytotoxity
13. A 4 year old child had Mantoux test. 60 hours after tuberculin introduction a focal skin
hardening and redness 15 mm in diameter appeared. It was regarded as positive test. What type of
hypersensitivity reaction is this test based upon?
A. Delayed-type hypersensitivity
B. Immune complex-mediated hypersensitivity
C. Complement-mediated cytotoxic hypersensitivity
D. Immediate hypersensitivity E. –
14. A 10-year-old child had the mantoux tuberculin test administered. 48 hours later a papule
up to 8 mm in diameter appeared on the site of the injection. What type of hypersensitivity reaction
developed after the tuberculin injection?
A. Type IV hypersensitivity reaction
B. Arthus phenomenon C. Seroreaction
D. Type II hypersensitivity reaction E. Atopic reaction
15. A 10 year old child was subjected to Mantoux test (with tuberculin). 48 hours later a
papule up to 8 mm in diameter appeared on the site of tuberculin injection. What type of
hyperesponsiveness reaction has developed after tuberculin injection?
A. Hyperresponsiveness reaction type IV
B. Reaction of Arthus phenomenon type
C. Reaction of serum sickness type
D. Atopic reaction E. Hyperresponsiveness reaction type II
16. A child suspected for tuberculosis underwent Mantoux test. 24 hours after allergen
injection there appeared a swelling, hyperaemia and tenderness. What are the main components in
the development of this reaction?
A. Mononuclears, T-lymphocytes and lymphokines
B. Granulocytes, T-lymphocytes and IgG
C. Plasmatic cells, T-lymphocytes and lymphokines
D. B-lymphocytes, IgM E. Macrophages, B-lymphocytes and monocytes
17. A patient was diagnosed with active focal pulmonary tuberculosis. What drug should be
prescribed in the first place?
A. Isoniazid B. Sulfalen C. Cyclocerine
D. Ethionamiden E. Ethoxide
18. A patient suffers from pulmonary tuberculosis. During treatment neuritis of visual nerve
arose. What drug has caused this by-effect?
A. Isoniazid B. Ethambutol C. Kanamycin
D. Rifampicin E. Streptomycin
19. After 4 months of treatment for tuberculosis the patient began complaining of toes and
fingers numbness, sensation of creeps. He was diagnosed with polyneuritis. What antituberculous
drug might have caused these complications?
A. Isoniazid B. Rifampicin C. Ciprofloxacin
D. Sodium salt of benzylpenicillin E. Iodine solution
20. A patient suffering form tuberculosis was treated with rifampicin, which caused drug
resistance of tuberculosis mycobacteria. In order to reduce mycobacteria resistance, rifampicin
should be combined with the following drug:
A. Isoniazid B. Acyclovir C. Intraconazole D. Metronidazole E. Amoxicillin

369
21. Tuberculosis can be treated by means of combined chemotherapy that includes
substances with different mechanisms of action. What antituberculous medication inhibits
transcription of RNA into DNA in mycobacteria?
A. Rifampicin B. Isoniazid C. Streptomycin
D. Ethionamide E. Para-aminosalicylic acid
22. A patient with pulmonary tuberculosis is prescribed the most effective antituberculosis
antibiotic. Name this drug:
A. Tetracycline
B. Furasolidone
C. Rifampicin
D. Bactrim (Co-trimoxazole)
E. Streptocide
23. The 32-year-old patient has been taking antituberculosis drugs. Later he noticed that his
urine had become redorange in color. What drug is conductive to this phenomenon?
A. Rifampicin B. Isoniazid C. Pyrazinamide
D. Ethambutol E. Streptomycin sulphate
24. After starting treatment for pulmonary tuberculosis a patient complained about red tears
and urine. What drug could cause such changes?
A. Rifampicin B. Benzylpenicillin sodium salt
C. Benzylpenicillin potassium salt
D. Biseptol-480 E. Cefazolin
25. Following treatment with a highly efficient anti-tuberculosis drug a 48-yearold female
developed optic nerve neuritis, memory impairment, cramps. Which of these anti-TB drugs had the
patient taken?
A. Isoniazid B. PASA C. Rifampicin
D. Ethambutol E. Kanamycin sulfate
26. A patient being treated for tuberculosis is suffering from hearing deterioration. What
drug causes this complication?
A. Streptomycin B. Isonicotinic acid hydrazide (Isoniazid)
C. Rifampicin D. Ethionamide E. Kanamycin sulphate
27. A 16 y.o. boy from a countryside entered an educational establishment. Scheduled
Manteux test revealed that the boy had negative reaction. What are the most reasonable actions in
this case?
A. To perform BCG vaccination
B. To repeat the reaction in a month
C. To perform serodiagnostics of tuberculosis
D. To isolate the boy temporarily from his mates
E. To perform rapid Price diagnostics
28. Medical examination of the first-year pupils included Mantoux test. 15 pupils out of 35
had negative reaction. What actions should be taken against children with negative reaction?
A. BCG vaccination B. Antitoxin vaccination
C. Rabies vaccination D. Repeat Mantoux test
E. Examination of blood serum
29. Planned mass vaccination of all newborn 5-7 day old children against tuberulosis plays an
important role in tuberculosis prevention. In this case the following vaccine is applied:
A. BCG B. Diphteria and tetanus toxoids and pertussis vaccine
C. Diphtheria and tetanus anatoxin vaccine
D. Adsorbed diphtheria vaccine E. –
30. For tuberculosis prevention the newborns got an injection of a vaccine. What vaccine was
used?
A. BCG B. Mantoux C. DTaP vaccine
D. Anatoxin E. Oral polio vaccine (Sabin vaccine)
31. In a maternity hospital a newborn should receive vaccination against tuberculosis. What
vaccine should be chosen?

370
A. BCG vaccine
B. STI vaccine
C. EV vaccine
D. DPT vaccine
E. Tuberculin
32. There is a suspicion of active tuberculosis development in patient. The doctor has
appointed Mantoux test to make a diagnosis. What immunobiological agent has to be administered?
A. Tuberculine B. BCG vaccine C. DPT vaccine
D. Tularin test E. DT vaccine
33. A 6-year-old child with suspected active tuberculosis process has undergone diagnostic
Mantoux test. What immunobiological preparation was injected?
A. Tuberculin B. BCG vaccine C. DTP vaccine D. Tularinum E. Td vaccine
34. The first grade pupils were examined in order to sort out children for tuberculosis
revaccination. What test was applied for this purpose?
A. Mantoux test B. Schick test C. Anthraxine test
D. Burnet test E. Supracutaneous tularin test
35. During the skill-building session in microbiology the students need to stain the
prepared and fixed sputum smears obtained from a tuberculosis patient. What staining
technique should be used in this case?
A. Gram B. Giemsa C. Ziehl-Neelsen D. Burry
E. Gins
36. A patient with pulmonary tuberculosis is prescribed the most effective
antituberculous antibiotic. Name this drug:
A. Furasolidone B. Bactrim (Co-trimoxazole)
C. Streptocide D. Rifampicin E. Tetracycline
To treat tuberculosis, an antibiotic that colors urine red is prescribed. Name the antibiotic:
A. Amoxicillin. B. Rifampicin. C. Nitroxoline.
D. Erythromycin. E. Cefotaxime.
37. First-year schoolchildren have received tuberculin skin test (Mantoux test) at the school
nurse’s office. The purpose of this test was:
A. To detect parotitis in schoolchildren
B. To measure allergization rate toward rickettsia
C. To measure immune stress toward diphtheria
D. To determine the children that need to receive BCG vaccination
E. To preventively vaccinate against tuberculosis
38. A 36-year-old man provisionally diagnosed with renal tuberculosis has undergone urinary
sediment analysis. Microscopy revealed acid-fast bacteria, but Pryce method detected no cord factor.
Name the most reliable method of investigation that can confirm or refute this provisional diagnosis:
A. Inoculation of laboratory animals B. Phage typing of the obtained culture
C. Allergy skin test D. Toxigenicity testing
E. Serological identification of the causative agent
39. A 6-year-old boy is brought to the pediatrician by his mother, who complains of low-
grade fever, chronic cough and night sweats in her child. She describes the cough as productive,
producing white sputum that is sometimes streaked with blood. She also says that her son has lost
some weight in the last month. His vital signs include blood pressure of 115/75 mm Hg, heart rate of
110/min., respiratory rate of 18/min. and temperature of 36,6°C. On physical examination, the
patient is ill looking. Pulmonary auscultation reveals some fine crackles in the right upper lobe. The
pediatrician suspects an active infection and performs Mantoux test. Intradermal injection of which
of the following substances has been most likely used by pediatrician for screening test in this clinical
case?
A. Tuberculin B. –
C. Tetanus and diphtheria toxoids vaccine (Td) D. Bacillus Calmette-Guerin
(BCG) vaccine
E. Diphtheria-tetanus toxoids-acellular pertussis vaccine (DTaP)

371
40. After a prolonged isoniazid treatment, the patient developed polyneuritis, paresthesia,
memory disorders, and convulsions. What is the most likely mechanism of the described isoniazid
side-effects?
A. Disruption of cell membrane synthesis B. Inhibition of protein synthesis
C. Inhibition of RNA-synthesis D. Inhibition of pyridoxal phosphate synthesis
E. Para-aminobenzoic acid antagonism
41. A centrifugate of urine sample obtained from patient with suspected renal tuberculosis
was used to make a slide mount for microscopy. What method should be used to stain the slide and
detect the causative agent?
A. Zielh-Neelsen stain B. Loeffler stain
C. Gram stain D. Aujeszky stain E. Burri stain
42. A 16-year-old boy from the rural area entered the technical school. During a regular
Mantoux test, it turned out that this boy had a negative reaction. What tactics should the doctor
choose as the most rational in this case?
A. Repeat the test in a month B. Serodiagnosis of tuberculosis
C. Urgent isolation of the boy from his groupmates D. BCG vaccination
E. Express diagnostics of tuberculosis using the Price method
43. A man is being treated for chronic pneumonia for a long time. Microscopy of sputum
smears stained using Ziehl-Nielsen method reveals red bacilli 0.25x4 microns in size, located
separately or sometimes in small clusters. What disease can be suspected?
A. Pneumococcal pneumonia B. Pulmonary actinomycosis
C. Pulmonary candidiasis D. Pulmonary tuberculosis
E. Influenza pneumonia

Lepra
1. Microscopic analysis of tissue sampling from affected area of mucous membrane of oral
cavity revealed bacillus in form of accumulations that looked like a pack of cigarettes. Ziehl-Neelsen
staining gives them red colour. What kind of pathogenic organism was most likely revealed in tissue
sampling?
A. M.leprae B. M.tuberculosis C. A.bovis
D. A.israilii E. M.avium
2. Granulomas containing lymphocytes and macrophages were detected during analysis of
skin biopsy material. Among macrophages there are large cells with fat inclusions, which contain
microorganisms in spheric packages (Virchow’s cells). The following disease is based on the described
type of hypersensitivity:
A. Leprosy B. Syphilis C. Tuberculosis
D. Rhinoscleroma E. Epidemic typhus

Plague
1. During the examination of a patient, who had been to the mountain pasture and had been
hospitalized in a bad condition with fever, the doctor found out the enlargement of inguinal lymph
nodes to 8 cm, which were attached to the surrounding tissues, immovable, the skin above them was
red and tender. The microscopic examination of the node revealed acute serohemorrhagic
inflammation. What disease is it typical for?
A. Brucellosis B. Syphilis C. Anthrax
D. Tularemia E. Plague
2. A patient presents with fever, chill and cough. From his sputum the ovoid Gram-negative
bipolar-stained bacilli with a delicate capsule were secured. What is the most likely diagnosis?
A. Plague B. Tuberculosis C. Leptospirosis
D. Brucellosis E. Toxoplasmosis

372
3. The patient has developed pain in the axillary area, rise of temperature developed 10
hours ago. On examination: shaky gait is marked, the tongue is coated by white coating. The pulse is
frequent. The painful lymphatic nodules are determined in the axillary area. The skin is erythematous
and glistering over the lymphatic nodules. What is the most probable diagnosis?
A. Bubonic plague B. Acute purulent lymphadenitis
C. Lymphogranulomatosis D. Anthrax E. Tularemia
4. A patient has got pain in the axillary area, rise of temperature developed 10 hours ago. On
examination: shaky gait is evident, the tongue is coated with white deposit. The pulse is frequent.
The painful lymphatic nodes are revealed in the axillary area. The skin over the lymph nodes is
erythematous and glistering. What is the most probable diagnosis?
A. Bubonic plague B. Acute purulent lymphadenitis
C. Lymphogranulomatosis D. Anthrax E. Tularemia
5. A 45-year-old patient, a sailor, was hospitalized on the 2nd day of the disease. A week ago
he returned from India. Complains of body temperature of 41 oC, severe headache, dyspnea, cough
with frothy rusty sputum. Objectively: the patient is pale, mucous membranes are cyanotic,
breathing rate - 24/min, tachycardia is present. In lungs: diminished breath sounds, moist rales over
both lungs, crepitation. What is the most likely diagnosis?
A. Pneumonic plaque B. Miliary tuberculosis
C. Influenza D. Ornithosis E. Sepsis
6. The laboratory for especially dangerous infections conducts microscopic examination of
pathological material from a patient with suspected plague. The sample was stained by Burri-Gins
technique. What property of the causative agent can be identified by this technique?
A. Capsule formation B. Spore formation
C. Acid resistance D. Alkali resistance
E. Presence of volutin granules
7. A puncture sample has been taken from the inguinal lymph nodes of a patient
provisionally diagnosed with plague. The sample was inoculated into a hard nutrient medium. What
shape will the colonies have, if the diagnosis is confirmed?
A. “Dewdrops”
B. “Shagreen leather”
C. “Lace handkerchief”
D. “Mercury drops”
E. “Lion’s mane”
8. Dwellers of a village noticed mass mortality of rats in some farms. It was suspected that
the animals might have died from plague. What postmortal analyses should be conduced in order to
establish the causative agent of the infection as soon as possible?
A. Ring precipitation reaction
B. Agglutination reaction C. Passive agglutination reaction
D. Neutralization reaction E. Complement-binding reaction
9. On the territory of a certain region the mass death of rodents was observed. It was
assumed that it may be caused by plague agent. What serological reaction should be applied for
quick determination of antigen of this epizootic agent?
A. Precipitation reaction B. Agglutination reaction
C. Reaction of passive hemagglutination
D. Bordet-Gengou test E. Neutralization reaction
10. On a certain territory mass death of rodents was registered. It was suspected that their
death might have been caused by plague. What serological reaction should be applied for quick
identification of antigen of the causative agent of this epizooty?
A. Precipitation B. Agglutination
C. Passive hemagglutination
D. Complement binding E. Neutralization
2018

373
2 days after a hunter cut a ground squirrel’s body, he developed fever up to 390C, his lymph
nodes enlarged. Later he developed pneumonia with serohemorrhagic exudate that contained egg-
shaped microorganisms with bipolar staining. What provisional diagnosis can be made in this case?
A. Brucellosis. B. Anthrax. C. Tetanus.
D. Plague. E. Pseudotuberculosis.

Tularemia
1. On examination of a patient with disease onset 5 days ago the doctor suspected tularemia
and prescribed the patient tularin intracutaneously. What is the purpose of this drug administration
in the patient?
A. Allergy diagnostics
B. Prognosis for the disease
C. Treatment
D. Treatment evaluation
E. Prevention

Antrax
1. A smear of streptobacillus preparation stained by Ozheshko method has been studied
microscopically with oil immersion. What structural feature of the bacteria has been studied?
A. Spores B. Capsule C. Flagella
D. Inclusions E. Structure of cell wall
2. The territory of an old burial ground for animal refuse that hasn’t been used for over 50
years is meant for house building. But soil investigation showed the presence of viable spores of a
causative agent causing a very dangerous disease. What microorganism might have been preserved
in soil for such a long period of time?
A. Bacillus anthracis B. Francisella tularensis
C. Brucella abortus D. Yersinia pestis
E. Mycobacterium bovis
3. It is planned to use the territory of an old cattle burial ground (which is not used for more
than 50 years) for building houses. But ground analysis revealed presence of the pathogen of the
very dangerous illness. Which of the indicated microorgonisms is likely to remain in the ground for
such a long time?
A. Mycobacterium bovis B. Brucella abortus
C. Yersinia pestis D. Francisella tularensis E. Bacillus anthracis
4. A patient complained about a carbuncle on his face. Examination results: neither dense
nor painful edema of subcutaneous cellular tissue, there is black crust in the middle of the carbuncle
and peripheral vesicular rash around it. Bacteriological examination revealed presence of immobile
streptobacilli able of capsulation. What microorganisms are causative agents of this disease?
A. Bacillus antracis B. Staptylococcus aureus
C. Bacillus anthracoides D. Bacillus megaterium
E. Bacillus subtilis
5. A 34 year old male patient consulted a doctor about face carbuncle. Objectively: a loose,
painless edema of hypodermic tissue; black crust in the center of carbuncle, vesicular rash around it.
Microbiological examination revealed static streptobacilli capable of capsule building. What
microorganisms are the causative agents of this disease?
A. Bacillus antracis B. Staptylococcus aureus
C. Bacillus subtilis D. Bacillus anthracoides
E. Bacillus megaterium
6. A worker of a cattle farm consulted a surgeon about fever up to 40 oC, headache,
weakness. Objective examination of his back revealed hyperaemia and a dark red infiltration up to 5

374
cm in diameter with black bottom in the center and some pustules. What disease are these
presentations typical for?
A. Anthrax B. Plaque C. Tularemia
D. Furuncle E. Abscess
7. A 43 y.o. patient was admitted to the hospital with complaints of high temperature of the
body and severe headache. On examination: carbuncle is revealed on the forearm. There are intense
edema around it, insignificant pain, regional lymphadenitis. The patient is a worker of cattle-ranch.
What disease is it necessary to think about first?
A. Anthrax B. Carcinoma of skin C. Erysipelas
D. Erysipeloid E. Eczema
8. A 49-year-old countryman got an itching papule on the dorsum of his right hand. In the
centre there is a vesicle with serosanginous exudate. Within the next 2 days the patient developed a
painless edema of hand and forearm. On the 4th day the temperature rose to 38,5oC, in the right
axillary region a large painful lymph node was found. One daybefore the onset of the disease the
patient had examined a dead calf. What is the most likely diagnosis?
A. Cutaneous anthrax B. Bubonic plague
C. Carbuncle D. Lymphocutaneous tularemia
E. Erysipelas
9. Quite often, the soil may contain a number of pathogenic microorganisms. The causative
agents of the following disease may exist in the soil for a long time:
A. Anthrax B. Diphtheria C. Viral hepatitis
D. Pertussis E. Dysentery
10. At a bacteriological laboratory animal skins are analyzed by means of Ascoli precipitaion
test. What is detected if the reaction is positive?
A. Anthrax agent antigens
B. Brucellosis agent C. Anaerobic infection toxin
D. Plague agent E. Yersinia surface antigen
11. There was a record of some anthrax cases among animals in a countryside. The spread of
disease can be prevented by means of immunization. What kind of vaccine should be used?
A. STI live vaccine B. BCG vaccine
C. Salk vaccine D. Sabin’s vaccine
E. Diphteria and tetanus toxoids and pertussis vaccine
12. In a village, a case of anthrax had been registered. Medical services began
epidemiologically indicated specific prophylaxis of population against anthrax. What preparation was
used for this purpose?
A. Live vaccine B. Inactivated vaccine
C. Chemical vaccine D. Genetically engineered vaccine
E. Anatoxin
13.What diagnostic method should be used in industry to test the raw leather for presence of
B. antracis?
A. Microscopy with Burry-Gins stain B. Microscopy with Aujeszky stain
C. Ascoli's thermo precipitation test
D. Bacteriological analysis E. Serological test
14. A 43-year-old cattle farm worker is brought to the surgeon with fever, malaise, and
inflamed lesions on his hands and arms. He reports that about 2 weeks before his presentation at the
hospital he noticed small, painless, pruritic papules that quickly enlarged and developed a central
vesicle. The vesicles developed into erosion and left painless necrotic ulcers with black, depressed
eschar. Gram’s staining of the ulcer reveals gram-positive spore-forming bacilli. Which of the
following diseases is the most likely cause of these findings?
A. Anthrax B. Chickenpox C. Syphilis D. Tularemia E.
Plague
15. The bacteriological laboratory needs to prepare for analysis of materials that are
suspected to be contaminated with spores of anthrax causative agent. What diagnostic preparation
allows for quick detection of these spores?

375
A. Anti-anthrax fluorescent serum B. Standard anthrax antigen
C. Anti-anthrax immunoglobulin D. Enzyme-tagged immunoglobulin
E. Monoclonal antibodies to anthrax causative agent
16. A bioterrorist has mailed an envelope with a powder that is suspected to contain anthrax
causative agent. This envelope can remain dangerous for a long time, because anthrax causative
agent:
A. Forms a protein capsule B. Forms a polysaccharide capsule
C. Is a spore-former D. Forms flagella
E. Belongs to actinomycetes

Brucella
1. An infectious diseases hospital admitted a veterinarian with assumed brucellosis. What
serologic test can confirm this diagnosis?
A. Wright’s agglutination reaction B. Widal’s agglutination reaction
C. Ascoli’s precipitation reaction D. Weigl’s agglutination reaction
E. Wassermann reaction of complement binding
2. A veterenary attendant working at a cattle farm complains of joint pain, fever,
indisposition and sweating at nighttime that he has been experiencing for a month. Giving the regard
to such presentations and occupational history the doctor suspected brucellosis. What material
taken from this patient is to be analyzed in a common microbiological laboratory?
A. Blood serum B. Spinal fluid C. Vomit mass
D. Urine E. Feces
3. A 40-year-old female farmworker has been diagnosed with brucellosis and administered
causal chemotherapy. What group of drugs will be used for this purpose?
A. Antibiotic B. Donor immunoglobulin
C. Inactivated therapeutic vaccine
D. Antitoxic serum E. Polyvalent bacteriophage
4. For cultivation of Brucella, pure cultures should be incubated in CO2 enriched atmosphere.
What type of breathing is typical for Brucella?
A. Capnophilic B. Facultative anaerobic
C. Obligate anaerobic D. Obligate aerobic E. Any

Anaerobes
1. Microscopic examination of a microbial culture revealed fusiform spore-forming
microorganisms that get violet-blue Gram’s stain. What microorganisms were revealed?
A. Clostridia B. Streptococci C. Spirochaete
D. Actinomycete E. Diplococci
2. Those organisms which in the process of evolution failed to develop protection from H2O2
can exist only in anaerobic conditions. Which of the following enzymes can break hydrogen peroxide
down?
A. Peroxidase and catalase B. Oxygenase and hydroxylase
C. Cytochrome oxidase, cytochrome B5
D. Oxygenase and catalase E. Flavin-dependent oxidase
3. Pathological material taken from a patient suffering from pulpitis was inoculated onto Kitt-
Tarozzi cultural medium. It is planned to find the following microorganisms:
A. Anaerobic B. Acid-resistant C. Acidophilic
D. Haemolytic E. Aerobic
4. A patient was taken to a hospital with acute food poisoning caused by homemade canned
mushrooms. The product analysis revealed some microorganisms that develop only in the absence of
oxygen. What microorganisms caused the poisoning?
A. Obligate anaerobes B. Facultative anaerobes
C. Microaerophiles D. Obligate aerobes E. Capnophiles

376
5. A lot of pyoinflammatory processes in oral cavity are caused by anaerobes. What nutrient
medium can be used for control of wound textile contamination by anaerobes?
A. Kitt-Tarozzi B. Endo C. Roux
D. Sabouraud’s E. Ploskirev’s E. -
6. A patient has a necrotizing phlegmon of his lower extremity. A doctor suspects a gas
gangrene. Microscopy reveals grampositive bacilli. In order to confirm the diagnosis further
bacteriological tests should include inoculation of the material into the following nutrient medium:
A. Kitt-Tarozzi medium B. Endo agar
C. Meat-peptone agar D. Levine agar E. Milk-salt agar
7. In 8 days after a surgery the patient develops tetatus. The surgeon suspects this condition
to be caused by suture material contaminated by tetanus agent. The material is delivered to a
bacteriological laboratory. What nutrient medium is required for primary inoculation of the suture
material?
A. Sabouraud agar
B. Endo agar
C. Hiss medium
D. Kitt-Tarozzi medium
E. Egg-yolk salt agar
8. A patient consulted a dentist about limited (restricted) mouth opening (trismus). He has a
history of a stab wound of the lower extremity. What infection may cause these symptoms?
A. Tetanus B. Brucellosis C. Whooping cough
D. Wound anaerobic infection E. Tularemia
9. A patient with convulsive contractions of facial muscles was admitted to the infectious
disease ward. From a scratch on his lower right extremity analysts isolated bacteria with terminal
endospores that gave them drumstick appearance. What bacteria are compliant with given
description?
A. Clostridium tetani B. Clostridium botulinum
C. Clostridium perfringens D. Bacillus anthracis
E. Bacillus cereus
10. A 47-year-old male patient consulted a dentist about difficult mouth opening (lockjaw or
trismus). The patient has a history of a stab wound of the lower extremity. What infection can be
manifested by these symptoms?
A. Tetanus B. Brucellosis C. Whooping cough
D. Anaerobic wound infection E. Tularemia
11. On the 15-th day after a minor trauma of the right foot a patient felt malaise, fatigability,
irritability, headache, high body temperature, feeling of compression, tension and muscular twitching
of his right crus. What disease can it be?
A. Tetanus B. Anaerobic gas gangrene
C. Erysipelas D. Acute thrombophlebitis
E. Thromboembolism of popliteal artery
12. A 65 y.o. woman complains of complicated mouth opening following foot trauma 10 days
ago. Next day she ate with difficulties, there were muscles tension of back, the back of the head and
abdomen. On the third day there was tension of all muscle groups, generalized convulsions every 10-
15 min. What is the most probable diagnosis?
A. Tetanus B. Tetania C. Meningoencephalitis
D. Hemorrhagic stroke E. Epilepsy
13. A 45-year-old patient complains of body temperature rise up to 40oC, general weakness,
headache, painfulness and spastic muscle contractions around the wound in the shin. He received
this wound 5 days ago when working in his garden. He requested no medical care back then. What
wound infection can be suspected?
A. Tetanus B. Anthrax C. Erysipelas
D. Gram-positive E. Gram-negative

377
14. A 45-year-old patient complains of fever up to 40oC, general weakness, headache and
spasmodic contraction of muscles in the region of a shin wound. The patient got injured five days ago
when tilling soil and didn’t seek medical attention. What kind of wound infection can be suspected?
A. Tetanus B. Anthrax C. Erysipelas
D. Gram-positive E. Gram-negative
15. A 38-year-old male complains of tonic tension of the masticatory muscles, so that he
cannot open his mouth. 12 days before, he was bitten by an unknown dog. Objectively: there is
pronounced tension and twitching of the masticatory muscles. What is the most likely diagnosis?
A. Tetanus B. Rabies C. Hysteria
D. Trigeminal neuralgia E. Apyretic tetanus
16. Soil microflora often includes the representatives of pathogenic microorganisms. Specify
the diseases, whose causative agents may say viable in the soil for a long time:
A. Tetanus and gas anaerobic infection
B. Tuberculosis and mycobacterioses
C. Colibacillosis and cholera
D. Leptospirosis and plague E. Typhoid fever and dysentery
17. What preventive medications should be injected to a patient with open maxillofacial
trauma provided that he has never got prophylactic vaccination before?
A. Antitetanus immunoglobulin and anatoxin
B. Anticonvulsive drugs and anatoxin
C. Antitetanus serum and antibiotics
D. Diphtheria, tetanus toxoids and pertussis vaccine and antibiotics
E. Tetanus anatoxin and antibiotics
18. Anti-tetanus gamma globulin is produced by hyperimmunization of donors with tetanus
anatoxin. What class of immunoglobulins prevails in this preparation?
A. IgG B. IgA C. IgM D. IgE E. IgD
19. On the 8th day since the patient was inoculated with antitetanic serum because of dirty
wound of his foot he has developed rising temperature up to 380С , pains in the joints, rash and itch.
The blood tests revealed leukopenia and thrombocytopenia. Allergic reaction of what type has
developed in this case?
A. Anaphylactic B. Cytotoxic C. Delayed type of hypersensitivity
D. Stimulating E. Immunocomplex
20. A 16-year-old adolescent was vaccinated with DTP. In eight days there was stiffness and
pain in the joints, subfebrile temperature, urticarial skin eruption, enlargement of inguinal, cervical
lymph nodes and spleen. What kind of allergic reaction is observed?
A. Immunocomplex B. Hypersensitivity of immediate type
C. Cytoxic D. Hypersensitivity of delayed type E. –
21. A child cut his leg with a piece of glass while playing and was brought to the clinic for the
injection of tetanus toxoid. In order to prevent the development of anaphylactic shock the serum
was administered by Bezredka method. What mechanism underlies this method of desensitization of
the body?
A. Binding of IgE fixed to the mast cells
B. Blocking the mediator synthesis in the mast cells
C. Stimulation of immune tolerance to the antigen
D. Stimulation of the synthesis of antigenspecific IgG
E. Binding of IgE receptors to the mast cells
22. A 10-year-old child cut his leg with a piece of glass and was sent to a clinic for an anti-
tetanus serum injection. In order to prevent the development of anaphylactic shock, the Besredka
desensitization method was applied. What mechanism underlies this method?
A. Binding to IgE fixed to mast cells
B. Inhibited synthesis of mast cells mediators
C. Stimulation of the immunological antigen tolerance
D. Stimulation of antigen-specific IgG2 synthesis
E. Binding of IgE receptors on mast cells.

378
23. A 10-year-old child cut his leg with a glass shard, when playing, and was delivered to
outpatient department to receive anti-tetanus serum. To prevent development of anaphylactic shock
the serum was introduced by Bezredka method. This method of organism hyposensitization is based
on the following mechanism:
A. Stimulation of antigen-specific IgG2
B. Stimulation of the immunological antigen tolerance
C. Stabilization of mast cell membranes
D. Blocking of mast cell mediators synthesis
E. Binding of mast cell-fixed IgE
24. Typical manifestations of food poisoning caused by C. botulinum are double vision,
abnormal functioning of the swallowing and breathing. These symptoms develop as a result of:
A. Exotoxin effects B. Enterotoxin effects
C. Enterotoxic shock development D. Activation of adenylate cyclase
E. Pathogen adhesion to the enterocyte receptors
25. Typical signs of food poisoning caused by C. botulinum include diplopia, swallowing and
respiration disorders. These signs develop due to:
A. Enterotoxic shock development
B. Enterotoxin action
C. Adenylate cyclase activation
D. Adhesion of the agent to enterocyte receptors
E. Exotoxin action
26. After consumption some tinned meat a patient had diplopia, acute headache, deglutition
disorder, hard breathing, muscle weakness. The diagnosis was botulism. What factor of pathogenicity
are the clinic presentations of this disease connected with?
A. Exotoxin B.Hemolysin C.Endotoxin
D.Plasmocoagulase E.Fibrinolysin
27. When examining a patient with a suspicion of food toxicoinfection, a doctor on duty has
detected symptoms characteristic of botulism. The patient named the meals he had eaten the day
before. What is the most probable cause of infection?
A. Homemade canned meat B. Custard pastry from private bakery
C. Sour cream from local dairy factory
D. Strawberries from suburban vegetable garden E. Fried eggs
28. When examining a patient presumptively diagnosed with food toxicoinfection, a doctor
on duty has detected symptoms characteristic of botulism. The patient named the meals he ate the
day before. What is the most probable cause of infection?
A. Homemade canned meat
B. Custard pastry from a private bakery
C. Sour cream from a local dairy factory
D. Strawberries from a suburban vegetable garden
E. Fried eggs
29. An outbreak of food poisoning was recorded in an urban settlement. The illness was
diagnosed as botulism on the grounds of clinical presentations. What foodstuffs should be chosen for
analysis in the first place in order to confirm the diagnosis?
A. Tinned food B. Potatoes C. Pasteurized milk
D. Boiled meat E. Cabbage
30. A bacteriological laboratory studied the home-made dried fish which had caused a severe
food poisoning. Microscopy of the culture grown on the Kitt-Tarozzi medium revealed
microorganisms resembling a tennis racket. What is the most likely diagnosis?
A. Botulism B. Salmonellosis C. Cholera
D. Dysentery E. Typhoid fever
31. A bacteriological laboratory has been investigating a sample of homemade dried fish that
was the cause of severe food poisoning. Microscopy of the culture inoculated in Kitt-Tarozzi medium
revealed microorganisms resembling a tennis racket. What diagnosis can be made?
A. Botulism

379
B. Salmonellosis
C. Cholera
D. Dysentery
E. Typhoid fever
32. On microscopic examination of leftovers of the canned meat eaten by patient with severe
food toxicoinfection the following was detected: gram-positive bacilli with subterminal staining
defect and configuration alteration of bacilli generally resembling a tennis racket. What agent was
detected?
A. C. botulinum
B. P. vulgaris
C. E. coli
D. S. aureus
E. S. enteritidis
33. Botulism agent causes severe food toxicoinfection. Point out the most characteristic
morphologic feature of botulism agent.
A. Gram-positive spore-forming bacilli with subterminal spore
B. Thick gram-positive non-sporeforming bacilli
C. Gram-positive spore-forming bacilli with terminal spore
D. Thin mobile spore-forming bacilli with central spore
E. Thick gram-positive non-sporeforming bacilli
34. The causative agent of botulism causes severe food poisoning. Specify the most
characteristic morphological feature of botulism causative agent:
A. Gram-positive bacillus with subterminal spore
B. Thick gram-positive non-spore-forming bacillus
C. Gram-positive bacillus with terminal spore
D. Thin mobile bacillus with central spore
E. Thick gram-positive bacillus without spores and flagella
35. The patient 25 y.o. was admitted on the 1st day of the disease with complaints of double
vision in the eyes, heavy breathing. The day before the patient ate homemade mushrooms. On
objective examination: paleness, widened pupils, disorder of swallowing, bradycardia, constipation
are marked. What is the diagnosis?
A. Botulism B. Yersiniosis C. Leptospirosis
D. Salmonellosis, gastrointestinal form E. Lambliasis
36. In the morning a patient had nausea, abdominal discomfort, single vomiting, dry mouth.
In the evening, the patient presented with the increasing general weakness, double vision, difficult
swallowing of solid food. Objectively: ptosis, mydriasis, anisocoria, absence of gag and pharyngeal
reflex, dry mucous membranes. The previous evening the patient had dinner with canned food and
alcohol. What is the presumptive diagnosis?
A. Botulism B. Poliomyelitis C. Food toxicoinfection
D. Acute ischemic stroke E. Intoxication with unknown poison
37. A 12-year-old boy presents with nausea, frequent repeated vomiting that first occurred
after eating canned vegetables. Objectively: the patient has dry mucous membranes, muscular
hypotonia, anisocoria, mydriasis, dysphagia and dysarthria. What is the most likely diagnosis?
A. Botulism B. Shigellosis C. Salmonellosis
D. Cholera E. Yersiniosis
38. Patient with vomiting, dizziness, sensation of dubble vision, difficult swallowing was
admitted to the hospital. Doctor suspects botulism. What diagnostic methods should be used for
diagnosis approving?
A. - B. Bacteriological, mycological C. Biological test, bacteriological
D. Allergic test, serological E. Protozoological, microscopical
39. A patient has been hospitalized with provisional diagnosis of botulism. What serological
reaction should be used to reveal botulinum toxin?
A. Neutralization reaction B. Agglutination reaction
C. Bordet-Gengou test D. Precipitation reaction

380
E. Immunofluorescence test
40. Researchers of a bacteriological laboratory examine tinned meat for botulinic toxin. For
this purpose a group of mice was injected with an extract of the material under examination and
antitoxic antibotulinic serum of A, B, E types. A control group of mice was injected with the same
extract but without antibotulinic serum. What serological reaction was applied?
A. Neutralization B. Precipitation C. Complement binding
D. Double immune diffusion E. Opsonocytophagic
41. A laboratory received a food product that had been taken from the focus of food
poisoning and presumably contained botulinum toxin. To identify the type of toxin, the neutralization
reaction must be performed on white mice. What biological product is used in this reaction?
A. Antitoxic serum B. Normal serum C. Antibacterial serum
D. Diagnosticum E. Allergen
42. Bacteriological laboratory examines canned meat whether it contains botulinum toxin.
For this purpose an extract of test specimen and antitoxic antibotulinic serum of A, B, E types were
introduced to a group of mice under examination; a control group of mice got the extract without
antibotulinic serum. What serological reaction was applied?
A. Neutralization B. Precipitation C. Complement binding
D. Double immune diffusion E. Opsonophagocytic
43. A bacteriological laboratory tests canned meat for botulinum toxin. Extract of the tested
material and ABE botulinum antitoxin serum was introduced into the test group of mice; a control
group of mice received the extract without antibotulinic serum. What serological reaction was used?
A. Neutralization B. Precipitation C. Complement binding
D. Double immunodiffusion E. Opsonophagocytic
44. A 12 year old girl complains about abrupt weakness, nausea, dizziness, vision impairment.
The day before she ate home-made stockfish, beef. Examination revealed skin pallor, a scratch on
the left knee, dryness of mucous membranes of oral pharynx, bilateral ptosis, mydriatic pupils. The
girl is unable to read a simple text (mist over the eyes). What therapy would be the most adequate in
this case?
A. Parenteral introduction of polyvalent antibotulinic serum
B. Parenteral disintoxication C. Parenteral introduction of antibiotics
D. Gastric lavage E. Parenteral introduction of antitetanus serum
45. A patient diagnosed with botulism has been prescribed antibotulinic serum for
treatment. What immunity will be formed in the given patient?
A. Antitoxic passive immunity
B. Infection immunity
C. Antitoxic active immunity
D. Antimicrobic active immunity
E. Antimicrobic passive immunity
46. A patient has food poisoning. Laboratory analysis revealed a culture of anaerobic gram-
positive spore-forming bacteria. What is the most likely kind of the isolated causative agent?
A. C. perfringens B. Proteus vulgaris C. P. mirabilis
D. Vibrio parahemolyticus E. Esherichia coli
47. Gram-positive spore-forming bacilli were extracted in anoxic environment from the
patient’s wound contaminated with soil. Cultivation on a blood-glucose agar resulted in growth of
the colonies surrounded with hemolysis zone. What agent was extracted from the wound?
A. Clostridium perfringens
B. Clostridium botulinum
C. Staphylococcus aureus
D. Pseudomonas aeruginosa
E. Esherichia coli
48. On the 5th day after a surgery for colon injury a patient complains of bursting pain in the
postoperative wound, weakness, drowsiness, headache, fever up to 40oC. Objectively: the skin
around the wound is swollen, there is gas crepitation. The wound discharges are scarce foul-smelling,
of dark-gray color. What is the most likely diagnosis?

381
A. Anaerobic clostridial wound infection B. Abscess
C. Postoperative wound infection D. Erysipelas E. Phlegmon
49. 4 days after a patient received a gunshot wound of the middle third of the thigh soft
tissues his condition suddenly began deteriorating. There are complaints of bursting pain in the
wound; pain increases during the last 12 hours. Edema of skin and hypodermic tissue quickly grows.
Body temperature is 38,2oC, heart rate is 102/min. The wound edges gape, are dull in color; the
muscles, viable as of day before, now protrude into the wound, look boiled, are dull in color, have
dirty-grey coating and fall apart when being held with forceps. What infection has developed in the
wound?
A. Anaerobic B. Aerobic gram-negative C. Putrid
D. Aerobic gram-positive E. Diphtheria of wound
50. A laboratory received a material from a patient’s wound. Preliminary diagnosis is gaseous
gangrene. What microbiological method should be applied to determine species of causative agent?
A. Bacteriological B. Allergic C. Bacterioscopic D. Serological E. RIA
51. The following spore-forming bacteria can be preserved in soil over a long period of time:
clostridia of tetanus, botulism, anaerobic gas infection. Name the way with which these
microorganisms get into soil.
A. With feces B. With urine C. With water
D. With industrial waters E. With expectoration
52. A patient consulted a stomatologist about purulent inflammation of his gums. What drug
will be the most effective if it is suspected that a causative agent is an anaerobe?
A. Metronidazole B. Gentamicin C.Oxacillin D.Co-trimoxazole E.-
52. In 8 days after a surgery the patient developed tetanus. The surgeon suspects this
condition to be caused by suture material contaminated by tetanus agent. The material is delivered
to a bacteriological laboratory. What nutrient medium is required for primary inoculation of the
suture material?
A. Kitt-Tarozzi medium. B. Endo agar.
C. Sabouraud agar. D. Egg-yolk salt agar. E. Hiss medium
53. A bacteriological laboratory received a sample of dried fish from an outbreak of food
poisoning. Inoculation of the sample on Kitt-Tarozzi medium revealed microorganisms resembling
tennis racket. These microorganisms are causative agents of the following disease:
A. Botulism. B. Diphtheria. C. Typhoid fever.
D. Salmonellosis. E. Dysentery.
54. A 37-year-old male was admitted to a hospital complaining of abdominal pain, difficulty
in swallowing and breathing, constipation, and nausea. He developed respiratory failure and required
endotracheal intubation and ventilation. Two days before, the patient consumed dried salted fish
bought from an artisanal producer. Laboratory investigation for infectious pathogen was performed
using Kitt-Tarozzi’s method. Observation under a bright field microscopy revealed the presence of
typical microorganisms with “tennis racket” appearance. Which of the following is the most likely
diagnosis?
A. Botulism B. Cholera
C. Nontyphoidal Salmonella infection D. Typhoid fever E. Shigella
infection

Bordetella
1. For serological diagnostics of the whooping cough it was made large-scale reaction with
parapertussis and pertussis diagnosticums. At the bottom of the test-tubes with diagnosticum of
Bordetella parapertussis grain-like sediment formed. What antibodies have this reaction revealed?
A. Bacteriolysins B. Agglutinins C. Antitoxins
D. Opsonins E. Precipitins
2. A large-scale reaction with parapertussis and pertussis diagnosticums was made in order
to make serological diagnostics of the whooping cough. At the bottom of the test-tubes with

382
diagnosticum of Bordetella parapertussis a granular sediment formed. What antibodies did this
reaction reveal?
A. Agglutinins B. Precipitins C. Opsonins
D. Bacteriolysins E. Antitoxins
3. A patient has been suffering from elevated temperature and attacks of typical cough for 10
days. Doctor administered inoculation of mucus from the patient’s nasopharynx on the agar. What
microorganism is presumed?
A. Pertussis bacillus B. Pfeiffer’s bacillus
C. Listeria D. Klebsiella E. Staphylococcus
4. A patient has severe catarrhal symptoms. Material growth on Bordet-Gengou agar showed
mercury-drop like colonies. Examination of the blood smears revealed some small ovoid gram-
negative bacilli sized 1-3 microns. What microorganisms were isolated?
A. Bordetella B. Corynebacteria C. Mycobacteria
D. Meningococcus E. Brucella
5. During bacteriological examination of sputum of a child with choking cough and fever
there were revealed glossy smooth colonies growing on casein-charcoal agar and reminding of
mercury drops. Microscopic examination revealed short Gram-negative bacteria. What
microorganism was secured from the sputum?
A. Bordetella pertussis B. Haemophylus influenzae
C. Corynebacterium dyphtheriae
D. Klebsiella pneumoniae E. Streptococcus pyogenes
6. A family has two children. The younger child is under the year. The child has developed
spastic cough attacks. Similar clinical presentation was observed in the elder preschool child one
month ago. The doctor suspects pertussis infection. What method enables retrospective diagnostics
of this disease?
A. Serological B. Biological C. Bacteriological D. Molecular biological E.
Microscopy

Treponema
1. While studying a microslide obtained from the punctuate of a regional lymph node and
stained by Romanovsky-Giemsa method a physician revealed some light pink thin microorganisms
with 12-14 regular spiral coils and pointed ends, up to 10-13 micrometer long. This might be the
causative agent of the following disease:
A. Syphilis B. Trypanosomiasis C. Leptospirosis
D. Relapsing fever E. Leishmaniasis
2. In the micropreparation made from patient's regional lymph node punctate and stained
according to Romanovsky-Giemsa method, the doctor found out thin microorganisms with 12-14
equal ringlets and pale-pink sharp pointes 10-13 mkm in length. The pathogen of what disease is it
about?
A. Leishmaniasis B. Leptospirosis C. Surra
D. Syphilis E. Relapsing fever
3. In a microslide of the patient’s regional lymph node stained with Giemsa method a doctor
detected thin microorganisms with 12-14 uniform tendrils with pointed tips, 10-13 micrometers in
length, pale pink in color. In this case they can be identified as infectious agents of the following
disease:
A. Syphilis
B. Trypanosomiasis
C. Leptospirosis
D. Relapsing fever
E. Leishmaniasis
4. Bacterioscopic examination of chancre material revealed some mobile, long, convoluted
microorganisms with 8-12 regular coils. These features are typical for:
A. Treponema B. Borrellia C. Leptospira D. Vibrios E. Campylobacter

383
5. An 18-year-old patient has enlarged inguinal lymph nodes, they are painless, thickened on
palpation. In the area of genital mucous membrane there is a small-sized ulcer with thickened edges
and "laquer"bottom of greyish colour. What is the most probable diagnosis?
A. Syphilis B. Tuberculosis C. Lepra D. Trophic ulcer E. Gonorrhea
6. A 14-year-old patient was diagnosed with Hutchinson’s triad: barrel-shaped incisors,
parenchymatous keratitis and deafness. The revealed presentations are consistent with the following
disease:
A. Syphilis B. Toxoplasmosis C. Lepra D. Tuberculosis E. -
7. A dentist examined a 5-year-old boy and found him to have a saddle nose, high-arched
palate, natiform skull. Both front maxillary incisors are peg-shaped and have a crescent-shaped notch
in the cutting edge. Lymph nodes are not changed. What is the provisional diagnosis?
A. Late congenital syphilis B. Early congenital syphilis
C. Tertiary syphilis D. Fluorosis E. Rickettsiosis
8. During examination of the patient’s oral cavity a dentist noticed deformation of the teeth
and a crescent indentation of the upper right incisor. The teeth are undersized, barrel-shaped – tooth
cervix is wider than its edge. The patient uses a hearing aid, suffers from visual impairment. What
type of syphilis affects teeth in such way?
A. Primary
B. Early congenital
C. Neurosyphilis
D. Secondary
E. Late congenital
9. A 32 y.o. man is divorced, has an irregular sexual life. He complains of falling out of hair in
the region of eyelashes, eyebrows, scalp. Objectively: diffuse alopecia is observed, eyebrow margin is
absent, eyelashes are stair-like (Pinkus’ sign). What examination should be carried out first of all?
A. Wasserman test, IFT B. T.pallidum Immobilization Test (TPI)
C. Detection of the nasal mucous membrane for M.leprae
D. Consultation of neuropathist E. CBC
10. A patient who suffered form syphilis took a course of antibiotic therapy and fully
recovered. Some time later he was infected again with Treponema pallidum. What form of infection
is it?
A. Reinfection B. Recurrence
C. Superinfection D. Secondary infection E. Complication
11. A patient had been provisionally diagnosed with syphilis. A laboratory assistant took the
blood serum for an immunologic test based on the detection of antibodies preventing the movement
of treponemas and causing their death. What reaction was used for the diagnosis?
A. Immobilization B. Complement binding
C. Agglutination D. Precipitation E. Neutralization
12. A 32-year-old patient undergoing dental examination was found to have some rash-like
lesions resembling secondary syphilis in the oral cavity. The patient was referred for the serological
study with the purpose of diagnosis confirmation. In order to detect antibodies in the serum, living
Treponema were used as diagnosticum. What serological test was performed?
A. Immobilization B. Passive hemagglutination
C. Precipitation D. Complement binding E. Neutralization
13. A patient suffering from syphilis was prescribed a drug the action of which based upon
disturbed generation of murein leading to death of the causative agent. What drug is it?
A. Benzylpenicillin sodium salt
B. Bijochinol C. Ciprofloxacin D. Azithromycin E. Doxycycline
14. A 19 year old woman suffers from primary syphilis. Doctor administered her complex
therapy that includes benzylpenicillin sodium salt. What is the mechanism of action of this drug?
A. It blocks synthesis of peptidoglycan of microbial membrane
B. It blocks synthesis of cytoplasm proteins C. It blocks thiol enzymes
D. It blocks RNA synthesis E. It blocks DNA synthesis

384
15. A 23-year-old man developed a perforation in his hard palate, a dense formation with
clear margins was detected in this area. After a surgery, microscopy of excised formation shows
there a large focus of caseous necrosis surrounded with a granulation tissue with endovasculitis and
a cellular infiltration consisting of lymphocytes and epithelioid cells with predominance of plasma
cells. What is the most likely disease in this case?
A. Syphilis B. Tuberculosis C. Leprosy D. Scleroma E.
Sarcoma
16. The Wasserman reaction is markedly positive (++++) in a 30-year-old man. What
infectious disease is diagnosed using the Wasserman reaction?
A. Brucellosis B. Syphilis C. Poliomyelitis D. Influenza E. Tuberculosis
17. A 14-year-old boy was diagnosed with Hutchinson’s triad: barrel-shaped teeth,
parenchymatous keratitis, and deafness. These signs are characteristic of:
A. Syphilis B. Opisthorchiasis C. Toxoplasmosis D. Tuberculosis E. Leprosy

Leptospira
1. A man was admitted to the hospital on the 5th day of disease that manifested itself by
jaundice, muscle aching, chill, nose bleedings. In course of laboratory diagnostics a bacteriologist
performed darkfield microscopy of the patient’s blood drop. Name a causative agent of this disease:
A. Leptospira interrogans B. Borrelia dutlonii
C. Calymmatobacterium granulomatis
D. Bartonella bacilloformis E. Rickettsia mooseri
2. A patient was admitted to the hospital on the 7th day of the disease with complaints of
high temperature, headache, pain in the muscles, especially in calf muscles. Dermal integuments and
scleras are icteric. There is hemorrhagic rash on the skin. Urine is bloody. The patient was fishing two
weeks ago. What is the most likely diagnosis?
A. Leptospirosis B. Yersiniosis C. Salmonellosis
D. Brucellosis E. Trichinellosis
3. A 33 year old patient was delivered to the infectious diseases department on the 7-th day
of disease. He complained about great weakness, high temperature, pain in the lumbar area and leg
muscles, icteritiousness, dark color of urine, headache. The acute disease started with chill, body
temperature rise up to 40oC, headache, pain in the lumbar area and sural muscles. Icterus turned up
on the 4th day, nasal and scleral haemorrhages came on the 5th day. Fever has lasted for 6 days.
Diuresis - 200 ml. What is the most probable diagnosis?
A. Leptospirosis B. Typhoid fever
C. Virus A hepatitis
D. Sepsis E. Yersiniosis
4. A 25-year-old patient was delivered to an infectious diseases unit on the 3rd day of illness
with complaints of headache, pain in lumbar spine and gastrocnemius muscles, high fever, chill.
Objectively: condition of moderate severity. Scleras are icteric. Pharynx is hyperemic. Tongue is dry
with dry brown coating. Abdomen is distended. Liver is enlarged by 2 cm. Spleen is not enlarged.
Palpation of muscles, especially gastrocnemius muscles, is painful. Urine is dark in color. Stool is
normal in color. The most likely diagnosis is:
A. Leptospirosis B. Infectious mononucleosis
C. Malaria D. Viral hepatitis A E. Yersiniosis
5. A man died from an acute infectious disease accompanied by fever, jaundice,
haemorrhagic rash on the skin and mucous membranes as well as by acute renal insufficiency.
Histological examination of renal tissue (stained by Romanovsky-Giemsa method) revealed some
convoluted bacteria looking like C and S letters. What bacteria were revealed?
A. Leptospira B. Treponema C. Spirilla
D. Borrelia E. Campilobacteria

Protozoa

385
1. Etiological factors for the infectious diseases are often microorganisms with various
ultrastructure. Which of the following microorganism groups relates to the eukaryotes?
A. Protozoa B. Viruses C. Viroids D. Prions E. Scotobacteria
2. In order to prevent wound infection associated with surgical procedures a patient was
given a synthetic antiprotozoan drug with a high activity against Helicobacter pylori. Specify this
drug:
A. Metronidazole
B. Doxycycline hydrochloride
C. Chingamin
D. Acyclovir
E. Isoniazid
3. A dentist has detected symptoms of parodontosis in a patient. What antiprotozoal drug
should be prescribed?
A. Metronidazole
B. Levamisole
C. Griseofulvin
D. Mykoseptin
E. Furazolidone
Entamoeba
4. Patients with similar complaints applied to the doctor: weakness, pain in the intestines,
disorder of GIT. Examination of the feces revealed that one patient with four nucleus cysts should be
hospitalized immediately. For what protozoa are such cysts typical?
A. Lamblia B. Dysenteric amoeba C. Balantidium
D. Trichomonas E. Intestinal amoeba
5. A patient complains of frequent bowel movements and stool with blood admixtures
("raspberry jelly" stool). Microscopic examination revealed large mononuclear cells with absorbed
red blood cells. What protozoon is this morphological structure typical for?
A. Entamoeba histolytica
B. Giardia lamblia C. Campylobacter jejuni
D. Toxoplasma gondii E. Balantidium coli
6. A 40-year-old patient presents with abdominal pain, frequent loose stools with mucus and
blood. Stool analysis revealed vegetative forms of some protozoa sized 30-40 microns, with short
pseudopodia, containing large amounts of phagocytosed erythrocytes. What protozoan disease does
the patient have?
A. Amebiasis B. Leishmaniasis C. Trichomoniasis
D. Giardiasis E. Toxoplasmosis
7. A patient with suspected liver abscess was admitted to the surgical department. The
patient had been staying for a long time on business in one of African countries and fell repeatedly ill
with acute gastrointestinal disorders. What protozoal disease may the patient be now ill with?
A. Amebiasis B. Trypanosomosis C. Leishmaniasis
D. Malaria E. Toxoplasmosis
8. Among public catering workers examined by doctors of sanitary and epidemiologic station
often occur asymptomatic parasite carriers. This means that a healthy person carries cysts that infect
other people. Such parasitizing is possible for the following causative agent:
A. Dysenteric amoeba B. Malarial plasmodium
C. Intestinal trichomonad D. Dermatotropic leishmania
E. Viscerotropic leishmania
9. When doctors of a sanitary and epidemiologic institution examine employees of public
catering establishments they often reveal asymptomatic parasitosis, that is when a healthy person is
a carrier of cysts that infect other people. What causative agent can parasitize in such a way?
A. Dysenteric amoeba B. Malarial plasmodium
C. Enteral trichomonad D. Dermatotropic leishmania
E. Viscerotropic leishmania

386
10. Microscopy of dental plaque revealed unicellular organisms. Their cytoplasm had two
distinct layers, barely visible core, wide pseudopodia. The patient is most likely to have:
A. Entamoeba gingivalis B. Entamoeba histolytica
C. Lamblia D. Trichomonas tenax E. Entamoeba coli
11. Carious cavities of a 29-year-old patient contain the parasitic protozoa. It is established
that they relate to the Sarcodina class. Specify these single-celled organisms:
A. Entamoeba gingivalis B. Entamoeba histolutica
C. Entamoeba coli D. Amoeba proteus E. Lamblia intestinalis
12. A 52-year-old patient has the following diagnosis: systemic amebiasis with involvment of
intestines, liver, lungs. What drug should be prescribed?
A. Metronidazole B. Quiniofone C. Tetracycline
D. Quingamine E. Enteroseptol
13. Systemic amebiasis with involvment of intestines, liver, lungs was diagnosed in a 52-year-
old patient. What drug should be prescribed?
A. Quiniofone B. Enteroseptol C. Metronidazole
D. Tetracycline E. Quingamine
14. A patient ill with amebiasis was prescribed a certain drug. The use of alcohol together
with this drug is contraindicated because the drug inhibits metabolism of ethyl alcohol. What drug is
it?
A. Metronidazole B. Reserpine C. Clonidine
D. Diazepam E. Aminazine
15. A 30-year-old patient complains about having abdominal pain and diarrhea for five days;
body temperature rise up to 37, 5oC along with chills. The day before a patient had been in a forest
and drunk from an open water reservoir. Laboratory analyses enabled to make the following
diagnosis: amebic dysentery. What is the drug of choice for its treatment?
A. Metronidazole B. Furazolidonum C. Levomycetin
D. Phthalazol E. Emetine hydrochloride
16. A 30 year old patient consulted a doctor about having diarrhea and stomach aches for 5
days, temperature rise up to 37,50C with chills. The day before the patient was in a forest and drank
some water from an open pond. He was diagnosed with amebic dysentery that was bacteriologically
confirmed. Name the medication for treatment of this disease:
A. Metronidazole B. Furasolidone C. Chloramphenicol
D. Phthalazole E. Emethine hydrochloride
17. A 30 y.o. patient is diagnosed with amebic dysentery. This diagnosis was bacteriologically
confirmed. Name the preparation for its treatment:
A. Metronidazole B. Mebendazole C. Itrakonazole
D. Furacillin E. Acyclovir
18. What drug is more advisable for the patient with amebic dysentery?
A. Metronidazole B. Pyrantel C. Levamisole D. Bicillin-5
E. Benzylpenicillin sodium salt (Penicillin G sodium salt)
Gardia
19. Examination of duodenal contents revealed some pyriform protozoa with twin nuclei and
four pairs of flagella. There were two supporting filaments between the nuclei and a suctorial disc on
the ventral side. What representative of protozoa was revealed in this patient?
A. Lamblia B. Toxoplasma C. Leishmania
D. Intestinal trichomonad E. Trypanosome
20. Examination of the duodenal contents revealed some pear-shaped protozoa with two
nuclei and four pairs of flagella. The organisms had also two axostyles between the nuclei and a
ventral adhesive disc. What protozoan representative was found in the patient?
A. Lamblia B. Toxoplasma C. Leishmania
D. Intestinal trichomonad E. Trypanosome
21. A duodenal content smear of a patient with indigestion contains protozoa 10-18 mcm
large. They have piriform bodies, 4 pairs of filaments, two symmetrically located nuclei in the
broadened part of body. What kind of the lowest organisms is it?

387
A. Lamblia B. Dysentery ameba
C. Trichomonas D. Intestinal ameba E. Balantidium
22. A 13 year old child complains about poor appetite, pain in the right subcostal area.
Microscopical examination of duodenal contents revealed big pyriform cells with two nuclei. What
microorganism was revealed?
A. Lamblia B.Trichomonas C.Amoeba D. Trypanosoma E.Toxoplasma
23. A female patient consulted a doctor about a sense of epigastric discomfort, nausea and
anorexia. A duodenal content analysis revealed lamblia. What drug should be prescribed?
A. Metronidazole B.Chingamin C.Rifampicin D.Isoniazid E.Acyclovir
24. A patient consulted a doctor about bowels disfunction. The doctor established symptoms
of duodenitis and enteritis. Laboratory examination helped to make the following diagnosis:
lambliosis. What medication should be administered?
A. Metronidazole B. Erythromycin
C. Monomycin D. Chingamin E. Tetracycline
Balantidium
25. A patient working at a pig farm complains about paroxysmal abdominal pain, liquid feces
with admixtures of mucus and blood, headache, weakness, fever. Examination of large intestine
revealed ulcers from 1 mm up to several cm large, feces contained oval unicellular organisms with
cilia. What disease should be suspected?
A. Balantidiasis B. Amebiasis C. Toxoplasmosis
D. Lambliasis E. Trichomoniasis
26. Slime, blood and protozoa 30-200 microns of length have been revealed in a man’s feces.
The body is covered with cilias and has correct oval form with a little bit narrowed forward and wide
round shaped back end. On the forward end a mouth is visible. In cytoplasm there are two nucleuses
and two short vacuoles. For whom are the described attributes typical?
A. Balantidium B. Lamblia C. Dysenteric amoeba
D. Trichomonas E. Intestinal amoeba
Leishmania
27. Parents with ill child came to the infectionist. They worked in one of the Asian countries
for a long time. Child has eathy colored skin, loss of appetite, laxity, enlarged liver, spleen, peripheral
glands. What protozoan illness can this child have?
A. Amebiasis B. Balantidiasis C. Visceral leishmaniasis
D. Lambliasis E. Toxoplasmosis
28. A patient has roundish ulcers on his face, inflammation and enlargement of lymph nodes.
These symptoms turned up as a result of mosquito bites. Laboratory examination of discharge from
the ulcers revealed unicellular aflagellar organisms. What is the most probable diagnosis?
A. Dermatotropic leishmaniasis B. Toxoplasmosis
C. Scabies D. Trypanosomiasis E. Myasis
29. A group of Ukrainian tourists returning from Samarqand was bringing with them gerbils.
During examination in customs office ulcers were detected on the skin of the animals. What protozoa
is the most likely to cause the disease in the animals, if mosquitoes are the carriers?
A. Leishmania tropica major B. Balantidium coli
C. P. falciparum D. T. cruzi E. Toxoplasma gondii
Trypanosoma
30. While examining a blood smear taken form a patient and stained by Romanovsky’s
method a doctor revealed some protozoa and diagnozed the patient with Chagas disease. What
protozoan is the causative agent of this disease?
A. Trypanosoma cruzi B. Toxoplasma gondii
C. Leishmania donovani D. Leishmania tropica
E. Trypanosoma brucei
31. In the South and Central America there can be found a species of trypanosomes that is
the causative agent of Chagas disease. What animal is the infection carrier specific to this disease?
A. Gnat
B. Mosquito

388
C. Tsetse fly
D. Cockroach
E. Triatomine bug
32. A businessman came to India from South America. On examination the physician found
that the patient was suffering from sleeping-sickness. What was the way of invasion?
A. As a result of bug’s bites
B. As a result of mosquito’s bites
C. With contaminated fruits and vegetables
D. Through dirty hands E. After contact with a sick dogs
Trichomonas
33. A patient has symptoms of inflammation of urogenital tracts. Examination of a vaginal
smear revealed big monocellular, pear-shaped organisms with the pointed spike at the posterior end
of body, big nucleus and undulating membrane. What protozoa were found in the smear?
A. Trichomonas vaginalis B. Trichomonas hominis
C. Trichomonas buccalis D. Trypanosoma gambiense
E. Lamblia intestinalis
34. A gynaecologist was examining a patient and revealed symptoms of genital tract
inflammation. A smear from vagina contains pyriform protozoa with a spine, flagella at their front;
there is also an undulating membrane. What disease can be suspected?
A. Urogenital trichomoniasis
B. Lambliasis C. Intestinal trichomoniasis
D. Toxoplasmosis E. Balantidiasis
35. A female patient has symptoms of inflammation of urogenital tracts. A smear from the
vaginal mucous membrane contained big unicellular pyriform organisms with a sharp spike on the
back end of their bodies; big nucleus and undulating membrane. What protozoa were revealed in the
smear?
A. Trichomonas vaginalis B. Trichomonas hominis
C. Trichomonas buccalis D. Trypanosoma gambiense
E. Lamblia intestinalis
36. Microscopical examination of discharges from the gums of a patient ill with paradontosis
revealed some protozoan pear-shaped organisms 6-13 micrometer long. The parasite has one
nucleus and undulating membrane, there are four flagella at the front of its body. What protozoan
were found?
A. Trichomonads B. Leishmania C. Amoebae
D. Balantidia E. Lamblia
37. A smear from frothy and purulent vaginal discharges of a 42 y.o. woman was stained by
Romanovsky-Giemsa method. Its analysis revealed some microorganisms of flagellates class. What
microorganism were the most probably revealed?
A. Trichomonas vaginalis B. Leishmania donovani
C. Trypanosoma gambiense
D. Trihomonas hominis E. Lamblia intestinalis
38. A 42-year-old female has foamy purulent vaginal discharges. The smear stained by
Romanovsky-Giemsa’s method has been found to include flagellated bacteria. What is the most likely
microorganism that has been found by the doctor?
A. Trichomonas vaginalis B. Leishmania donovani
C. Trypanosoma gambiense
D. Trihomonas hominis E. Lamblia intestinalis
39. A patient consulted a dentist about itching and burning in the oral cavity; high
temperature. The patient was diagnosed with trichomonal gingivostomatitis. What drug should be
chosen for his treatment?
A. Metronidazole B. Ampicillin C. Gentamicin sulfate
D. Nystatin E. Doxycycline hydrochloride
Plasmodium

389
40. As an example of specific human parasites one can name Plasmodium falciparum, human
pinworm and some others. The source of parasite invasion is always a human. Such specific human
parasites cause the diseases that are called:
A. Anthroponoses B. Zoonoses C. Anthropozoonoses
D. Infections E. Multifactorial diseases
41. A patient who has recently come from an endemic area presents with elevated body
temperature, headache, chills, malaise, that is with the symptoms which are typical for a common
cold. What laboratory tests are necessary to confirm or to refute the diagnosis of malaria?
A. Microscopy of blood smears
B. Study of lymph node punctate
C. Urinalysis D. Study of cerebrospinal fluid
E. Microscopy of bone marrow punctuate
42. According to the data of WHO, for about 250 mln of Earth population fall ill with malaria.
This disease is mostly spread in tropical and subtropical regions. Range of its spread falls into the
area of the following mosquitoes:
A. Anopheles B. Culex C. Aedes D. Mansonia E. Culiseta
43. A patient has been brought to the hospital with the complaints of headache, pain in left
hypochondrium. He has been ill for 1,5 weeks. The sudden illness began with the increase of body
temperature up to 39, 90C. In 3 hours the temperature decreased and hydropoiesis began. The
attacks repeat rhythmically in 48 hours. The patient had visited one an African country. The doctors
have suspected malaria. What method of laboratory diagnostics is necessary to use?
A. Blood examination B. Immunological tests
C. Stool examination D. Urine examination E. -
44. A journalist’s body temperature has sharply increased in the morning three weeks after
his mission in India, it was accompanied with shivering and bad headache. A few hours later the
temperature decreased. The attacks began to repeat in a day. He was diagnosed with tropical
malaria. What stage of development of Plasmodium is infective for anopheles-female?
A. Gametocytes B.Shizontes C.Merozoites
D.Microgamete E.Sporozoites
45. A 47-year-old patient came to see a doctor on the 7th day of disease. The disease
developed very fast: after the chill body temperature rose to 40 oC and lasted up to 7 hours, then
dropped abruptly, which caused profuse sweat. There were three such attacks occur in once in two
days. Two days ago the patient arrived from Africa. Objectively: pale skin, subicteric sclera,
significantly enlarged liver and spleen. What is the cause of fever attacks in this disease?
A. Erythrocytic schizogony
B. Tissue schizogony C. Exotoxin of a causative agent
D. Gametocytes E. Endotoxin of a causative agent
46. A 34-year-old male visited Tajikistan. After return, he complains of fever up to 40oC which
occurs every second day and is accompanied by chills, sweating. Hepatosplenomegaly is present.
Blood test results: RBC- 3x1012/l, Нb- 80 g/l, WBC- 4x109/l, eosinophils - 1%, stab neutrophils - 5%,
segmented neutrophils - 60%, lymphocytes - 24%, monocytes - 10%, ESR - 25 mm/h. What is the
provisional diagnosis?
A. Malaria B. Infectious mononucleosis
C. Sepsis D. Typhoid fever E. Leptospirosis
47. A 23 year old female patient complains about periodical chill and body temperature rise
up to 40oC, sense of heat taking turns with profuse sweating. The patient has had already 3 attacks
that came once in two days and lasted 12 hours. She has lived in Africa for the last 2 months. Liver
and spleen are enlarged. In blood: erythrocytes - 2,5x1012/l. What is the most probable diagnosis?
A. Malaria B. Spotted fever C. Sepsis
D. Haemolytic anaemia E. Leptospirosis
48. 2 weeks since the blood transfusion a recepient has developed fever. What protozoal
disease can it be?
A. Trypanosomiasis B. Malaria
C. Amebiasis D. Toxoplasmosis E. Leishmaniasis

390
49. Two weeks after hemotransfusion a patient developed fever. What protozoal disease can
be suspected?
A. Malaria B. Toxoplasmosis C. Leishmaniasis
D. Amebiasis E. Trypanosomiasis
50. A healthy man is in a region with high risk of catching malaria. What drug should be
administered for individual chemoprophylaxis of malaria?
A. Chingamin B. Sulfalen C. Tetracycline
D. Metronidazole E. Biseptol
51. This drug has a destructive effect on erythrocytic forms of malarial plasmodia and
dysenteric amoebae. It is used for treatment and prevention of such diseases as malaria, amebiasis
and interstitial disease. What drug is it?
A. Chingamin B. Emetine hydrochloride
C. Tetracycline D. Erythromycin E. Quinine
52. What drug should be administered for individual prevention of malaria?
A. Chingamin B. Rifampicin C. Ampicillin
D. Gentamicin E. Biseptol (Co-Trimoxazolel)
53. UN volunteers have arrived in Nigeria to assist the locals in aftermath of earthquakes.
What drug should they prescribe for individual chemoprophylaxis of malaria?
A. Chingamin
B. Pyrantel
C. Pyrimethamine (Chloridinum)
D. Primaquine
E. Interferon (Laferon)
54. In preparation for business trip abroad the doctor was prescribed a histoschizontocidal
antimalarial drug as a personal means of disease prevention. What drug was given to the doctor?
A. Chingamin B. Mefloquine C. Biseptol (Co-Trimoxazole)
D. Quinine E. Doxycycline
Toxoplasma
55. A lymph node punctate of a patient with suspected protozoa disease was examined.
Examination of the stained specimen (Romanovsky’s stain) revealed some crescent bodies with
pointed end, blue cytoplasm and red nucleus. What protozoan were revealed in the smears?
A. Toxoplasmas B. Malarial plasmodiums
C. Dermotropic leishmania D. Viscerotropic leishmania
E. Trypanosomes
56. A puncture sample taken from the lymph node of a patient with preliminary diagnosis of
protozoan disease has been investigated. The preparation was processed with Giemsa stain and the
following was detected: crescent-shaped bodies with pointed tips, blue cytoplasm and red nuclei.
What protozoa have been detected in the preparation?
A. Toxoplasma
B. Plasmodium malariae
C. Dermatotropic Leishmania
D. Viscerotropic Leishmania
E. Trypanosoma
57. Examination of a man revealed a protozoan disease that affected brain and caused vision
loss. Blood analysis revealed unicellular half-moon-shaped organisms with pointed end. The
causative agent of this disease is:
A. Toxoplasma B.Leishmania C.Lamblia D.Amoeba E.Trichmonad
58. A man is ill with a protozoan disease characterized by cerebral affection and loss of sight.
Blood analysis revealed halfmoon-shaped unicellular organisms with pointed ends. This disease is
caused by:
A. Toxoplasma B.Leishmania C.Lamblia D.Amoeba E.Trichomonad
59. A woman who was infected with toxoplasmosis during the pregnancy has a child with
multiple congenital defects. This is a result of:
A. Teratogenesis B. Cancerogenesis C. Recombination

391
D. Chemical mutogenesis E. Biological mutogenesis
60. A woman delivered a dead child with multiple developmental defects. What protozoan
disease might have caused the intrauterine death?
A. Toxoplasmosis B. Leishmaniasis C. Malaria
D. Amebiasis E. Lambliasis
61. A woman gave birth to a stillborn baby with numerous malformations. What protozoan
disease could cause intrauterine death?
A. Toxoplasmosis
B. Leishmaniasis
C. Malaria
D. Amebiasis
E. Lambliasis
62. A married couple applied to the genetic consultation in order to consult about their child
with multiple abnormalities (microcephaly, idiocy etc). The woman has had an illnesses during her
pregnancy but she didn’t take any teratogens or mutagens. The parents’ and the child’s karyotype is
normal. Anamnesis study revealed that the family kept a cat. What gravidic disease caused the child’s
abnormalities?
A. Toxoplasmosis B. Leishmaniasis C. Dysentery
D. Balantidiasis E. Trichomoniasis
63. A patient’s preliminary diagnosis is toxoplasmosis. What material was used for
diagnostics of this disease?
A. Blood B. Feces C. Urine
D. Duodenal contents E. Sputum
64. A patient who came to the doctor because of his infertility was administered to make
tests for toxoplasmosis and chronic gonorrhoea. Which reaction should be performed to reveal
latent toxoplasmosis and chronic gonorrhoea in this patient?
A. RIHA - Reverse indirect hemagglutination assay
B. RDHA - Reverse direct hemagglutination assay
C. IFA - Immunofluorescence assay D. Immunoblot analysis
E. (R)CFT- Reiter's complement fixation test
65. A pregnant woman applied to a doctor with complaints typical for toxoplasmosis. The
doctor took a sample of her blood. What serological tests should be performed in this case?
A. Complement binding assay B. Precipitation test
C. Neutralization test D. Widal’s test E. Wassermann test
66. After the second abortion a 23 year old woman has been diagnosed with toxoplasmosis.
Which drug should be used for toxoplasmosis treatment?
A. Co-trimoxazole B. Itraconazole C. Mebendazole
D. Azidothimidine E. Acyclovir
67. Sanitary assessment of a pond, where the children from a recreation summer camp
take their swims, detected there oval cysts 50-60 micron in diameter, with 2 nuclei visible in
their cytoplasm (macronucleus and micronucleus). What protozoa do these cysts belong to?
A. Toxoplasma B. Amoeba C. Balantidium D. Lamblia
E. Euglena
68. Several patients with similar complaints came to the doctor. They all present with
weakness, pain in the intestines, indigestion. Feces analysis revealed the need for urgent
hospitalization of the patient, who had microbial cysts with four nuclei detected in his samples. Such
cysts are characteristic of the following protozoon:
A. Entamoeba histolytica. B. Entamoeba coli.
C. Balantidium. D. Trichomonad. E. Lamblia.
69. A patient complains of acute spastic abdominal pain, frequent urge to defecate, liquid
bloody feces with mucus. Laboratory analysis of fecal smear revealed inconstant in shape organisms
with erythrocyte. What is the most likely diagnosis?
A. Amebiasis. B. Lambliasis. C. Schistosomiasis.
D. Intestinal trichomoniasis. E. Balantidiasis.

392
70. A patient with probable liver abscess was delivered to a surgical department. The patient
for a long time had been on an assignment in an African country and had recurrent cases of
gastrointestinal disturbance. What protozoan disease can it be?
A. Toxoplasmosis B. Leishmaniasis
C. Malaria D. Amebiasis E. Trypanosomiasis

Rickettsia
1. A patient with suspicion on epidemic typhus was admitted to the hospital. Some arachnids
and insects have been found in his flat. Which of them may be a carrier of the pathogen of epidemic
typhus?
A. Lice B. Spiders C. Bed-bugs D. Cockroaches E. Houseflies
2. A sick man with high temperature and a lot of tiny wounds on the body has been admitted
to the hospital. Lice have been found in the folds of his clothing. What disease can be suspected in
the patient?
A. Epidemic typhus B. Tularemia
C. Scabies D. Malaria E. Plague
3. A 28 y.o. homeless male was admitted to the hospital because of initial diagnosis
"influenza". Roseolo-petechiae rash has appeared on the trunk and internal surfaces of the limbs on
the fifth day. Temperature is 410C, euphoria, face and sclera’s hyperemia, tongue tremor,
tachycardia, splenomegaly, excitement. What is the most probable diagnosis?
A. Typhus B. Alcogolic delirium
C. Leptospirosis D. Abdominal typhoid E. Measles
4. A 28 y.o. patient without permanent residence was admitted to the hospital with the
preliminary diagnosis influenza. On the fith day of illness he got a maculopapular petechial rash on
his body and internal surfaces of extremities. Body temperature is 410 C, euphoria, face hyperemia,
sclera reddening, tongue tremor, tachycardia, splenomegaly, excitement. What is the most probable
diagnosis?
A. Epidemic typhus B. Delirium alcoholicum
C. Leptospirosis D. Measles E. Typhoid fever
5. A 28-year-old patient was hospitalized with preliminary diagnosis "influenza". Roseolous-
petechial rash appeared on the 5th day of disease on the trunk. The temperature is 41 oC. Hyperemia
of face, reddening of scleras, tremor of tongue, tachycardia, splenomegaly are present. What is the
most likely diagnosis?
A. Epidemic typhus B. Measles C. Alcohol delirium
D. Leptospirosis E. Typhoid fever
6. A 55-year-old patient with a characteristic rash, fever, dizziness has been admitted to a
hospital. He has been provisionally diagnosed with typhus. No similar cases have been reported. In
his youth (15 years old) the patient suffered typhus in a boarding school. What disease is it?
A. Brill’s disease B. Typhoid fever C. Measles D. Rubella E. Cholera
7. 5 days before, a 26-year-old female patient developed an acute condition. Objectively:
marked headache, vomiting, weakness, poor appetite, temperature up to 39 oC. Objectively: the
patient is in a moderately grave condition, excited. The face is hyperemic, sclerae are injected. The
tongue is coated with brown fur. The trunk and limbs are covered with plentiful roseolous and
petechial rash. Hepatosplenomegaly is present. Complement binding reaction with Rickettsia
prowazekii is positive with the titer of 1:640. What drug should be administered?
A. Doxycycline B. Chloramphenicol C. Penicillin
D. Streptomycin E. Metronidazole

Chlamydia
1. A 35-year-old patient has been admitted to a hospital for pain in the left sternoclavicular
and knee joints, lumbar area. The disease has an acute character and is accompanied by fever up to
38oC. Objectively: the left sternoclavicular and knee joints are swollen and painful. In blood: WBCs -

393
9, 5x109/l, ESR - 40 mm/h, CRP - 1,5 millimole/l, fibrinogen - 4,8 g/l, uric acid - 0,28 millimole/l.
Examination of the urethra scrapings reveals chlamydia. What is the most likely diagnosis?
A. Reiter’s syndrome B. Rheumatic arthritis C. Gout
D. Bechterew’s disease E. Rheumatoid arthritis

Mycoplasma
1. On the base of the clinical data a child was diagnosed with atypical pneumonia resistant to
the effects of beta-lactam antibiotics. The patient’s sputum was cultured and incubated in a special
medium, which resulted in growth of microorganisms forming microscopic colonies with a dense
center (looking like fried eggs). What microorganism caused the disease?
A. Mycoplasma pneumoniae B. Klebsiella pneumoniae
C. Str. pneumoniae D. L. pneumophila E. Chlamidia pneumoniae
2. From a medicinal herb a certain phytopathogenic microorganism was secured. In the
nutrient medium it forms "fried egg"colonies. What is the most likely agent?
A. Mycoplasma B.Yeasts C.Actinomycetes D.Nocardia E.Pseudomonas

Legionella
1. A 22-year-old patient is a clerk. His working day runs in a conditioned room. In summer he
was taken by an acute disease with the following symptoms: fever, dyspnea, dry cough, pleural pain,
myalgia, arthralgia. Objectively: moist rales on the right, pleural friction rub. X-ray picture showed
infiltration of the inferior lobe. In blood: WBC - 11 · 109/l, stab neutrophils - 6%, segmented
neutrophils - 70%, lymphocytes - 8%, ESR - 42 mm/h. What is the etiological factor pneumonia?
A. Legionella B. Mycoplasma C. Streptococcus
D. Staphylococcus E. Pneumococcus

Gardnerella
1. A 21-year-old female patient consulted a gynecologist about itching, burning, watery
vaginal discharges with a fish-like smell. Speculum examination revealed that the cervical and vaginal
mucosa was of a normal pink color. Vaginal examination revealed no alterations of the uterus and
appendages. Gram-stained smears included clue cells. What is the most likely pathology?
A. Bacterial vaginosis (gardnerellosis) B. Chlamydiosis
C. Gonorrhea D. Trichomoniasis E. Candidiasis

Pseudomonas
1. Bacteriological examination of the urine of the patient with pyelonephritis revealed
microorganisms that produced yellow-green pigment and a characteristic odor in meat-peptone
agar. What are they called?
A. Pseudomonas B.Escherichia C.Proteus D.Klebsiella E.Azotobacter
2. A 60-year-old patient was hospitalized to the surgical department because of infection
caused by blue pus bacillus (Pseudomonas aeruginosa) which is sensative to penicillin antibiotics.
Indicate which of the given penicillins has marked activity to the Pseudomonas aeruginosa?
A. Carbenicillin disodium B. Benzylpenicillin
C. Methicillin D. Oxacillin E. Methylpenicillin
3. Urine examination of a patient with acute cystitis revealed leukocytes and a lot of gram-
negative bacilli. Inoculation resulted in growth of colonies of mucous nature that formed green
soluble pigment. What microorganism is the most probable cause of the disease?
A. Pseudomonas aeruginosa B. Klebsiella pneumoniae
C. Escherihia coli D. Salmonella enteritidis
E. Proteus mirabilis
4. A patient of surgical department complains about pain in the small of her back and in the
lower part of her belly; painful and frequent urination. Bacteriological examination of urine revealed
gram-negative oxidase-positive rod-like bacteria forming greenish mucoid colonies with specific
smell. What causative agent can it be?
A. Pseudomonas aeruginosa B. Proteus mirabilis C. E.coli

394
D. Str.pyogenes E. Mycoplasma pneumonie
5. A patient has wound abscess. Bacteriological examination of the wound content revealed a
gram-negative bacillus which forms semi-transparent mucous colonies of blue-green color with a
pearlescent appearance on the beef-extract agar. Culture has a specific odor of violets or jasmine.
What type of pathogen was isolated from the patient’s wound?
A. P. aeruginosa B. P. vulgaris C. S. aureus
D. S. pyogenes E. S. faecalis
6. A patient of oral surgery department has developed a purulent complication.
Bacteriological analysis of the wound discharge allowed to isolate a culture producing a blue-and-
green pigment. Which of the listed microorganisms may be a causative agent of the infection?
A. Pseudomonas aeruginosa
B. Proteus vulgaris C. Bacillus subtilis
D. Klebsiella pneumoniae E. Staphylococcus epidermidis
7. A patient in the oral surgery department has got purulent complication. Bacteriological
analysis of the wound material found a culture that produces cyan pigment. What microorganism is
the most probable causative agent?
A. Pseudomonas aeruginosa
B. Proteus vulgaris C. Bacillus subtilis
D. Klebsiella pneumoniae E. Staphylococcus epidermidis
8. A patient being treated in the burns department has suppurative complication. The pus is
of bluish-green color that is indicative of infection caused by Pseudomonas aeruginosa. What factor
is typical for this causative agent?
A. Gram-negative stain B. Presense of spores
C. Coccal form D. Cell pairing E. Mycelium formation
9. Bacteriological inspection of disinfection quality at a pharmacy revealed a microorganism
in an utility room (in the sink). The microorganism has the following properties: mobile nonspore-
forming gram-negative bacteria that form capsular substance, grow well on ordinary nutrient media,
secrete the blue-green pigment. This microorganism is most likely to be of the following genus:
A. Pseudomonas B. Proteus C. Clostridium
D. Shigella E. Vibrio
10. A sample of a finished dosage form was found to be contaminated with some
microorganisms exhibiting the following properties: greenish fluorescent colonies of gram-negative
nonsporeforming bacilli that grew on the medium for the detection of pyocyanin. The bacilli release
the bluegreen pigment into the medium. What microorganisms contaminated the finished dosage
form?
A. Pseudomonas aeruginosa
B. Enterobacteriaceae C. Staphylococcus aureus
D. Staphylococcus epidermidis E. Staphylococcus saprophyticus
11. During bacteriological examination of the purulent discharge obtained from a
postoperative wound an inoculation on meat infusion agar has been performed. The inoculation has
resulted in large colorless mucous colonies that in 24 hours with exposure to sunlight developed
green-blue pigmentation and smell of honey or jasmine. Bacterioscopy revealed gram-negative
lophotrichea. What bacterial culture is contained in purulent discharge?
A. Pseudomonas aeruginosa
B. Proteus vulgaris
C. Klebsiella osaenae
D. Streptomyces griseus
E. Brucella abortus
12. Many diseases of medicinal plants are caused by bacteria of the Pseudomonas genus.
Select the bacteria relating to this genus:
A. Blue pus bacillus B. Colon bacillus
C. Proteus D. Mycoplasma E. Micrococci
13. A patient suffers from severe postoperative pseudomonadous infection. What of the
following antibiotics should be administered in this case?

395
A. Amicacin sulfate B. Benzylpenicillin
C. Cephazolin D. Erythromycin E. Doxycycline
14. From urine of a 14-year-old boy with the exacerbation of secondary obstructive
pyelonephritis Pseudomonas aeruginosa was isolated with a titer of 1000000 microbes per 1 ml.
Which antibiotic is most advisable to be administered in this case?
A. Ciprofloxacin B. Ampicillin C. Cefazolinum
D. Azithromycin E. Chloramphenicol
15. Urinalysis of a patient with acute cystitis shows leukocytes and a large number of gram-
negative bacilli. Inoculation has resulted in the growth of mucous colonies that produce a green
soluble pigment. What microorganism is the most likely cause of the patient’s disorder?
A. Pseudomonas aeruginosa B. Proteus mirabilis
C. Escherichia coli D. Klebsiella pneumoniae
E. Salmonella enteritidis
16. A woman hospitalized in the surgical department complains of pain in her lower
abdomen and in the small of her back, frequent and painful urination. Urine culture test revealed
gram-negative oxidase-positive bacilli that formed green mucoid colonies with specific smell. What
causative agent can be suspected?
A. Pseudomonas aeruginosa B. Mycoplasma pneumonia
C. Str.pyogenes D. E.coli E. Proteus mirabilis

Helicobacter
1. Impression smear of mucosa biopsy material has been obtained from a patient with peptic
ulcer disease of the stomach. Gram-negative arcuate bent microorganisms were detected, urease
activity test was positive. What microorganisms were detected in the patient?
A. Helicobacter
B. Spirochete
C. Spirilla
D. Leptospira
E. Treponema
2. A male patient has been diagnosed with gastric ulcer. Bacteriological examination of
biopsy material from the affected part of stomach revealed small colonies of gram-negative, oxide
reductase-positive flexibacteria that grew on the chocolate agar on the fifth day. Which of the
following microorganisms is the most likely causative agent?
A. Helicobacter pylori B. Campilobacter jejuni
C. Campilobacter fetus D. Mycoplasma hominis
E. Chlamydia trachomatis
3. A patient underwent esophagogastroduodenoscopy. Analysis of the biopsy material
enabled doctors to diagnose him with helicobacteriosis. What property of the bacteria found in this
patient had to be obligatory taken into account during their cultivation?
A. Microaerophilic ability B. Presence of urease
C. Absence of spores and capsules
D. Colonisation of gastral cells E. Presence of six polar flagella
4. A 42-year-old patient with gastric ulcer has a disbalance between the aggressive and
defensive factors. Which of the following factors contributes to the development of gastric ulcer?
A. Helicobacter pylori B. Mucin
C. Hydrocarbonate D. Prostaglandin E. Prostacyclin
5. During fibergastroscopy a patient with ulcer disease of the stomach, the mucosal biopsy is
taken from the area of an ulcer. Impression smear is prepared from biopsy material and stained by
Gram method; the rest of biopsy material is tested for urease activity. Microscopy of the impression
smear revealed gram-negative spiral-shaped microorganisms, urease activity test is positive. What
bacteria were detected?
A. Helicobacter pylori B. Campilobacter jejuni
396
C. Treponema pallidum D. Spirilla minor
E. Shigella flexneri
6. A 32 year old patient complains about heartburn and dull pain in the epigastrium that
appear 2-3 hours after meal. Exacerbations happen in spring and in autumn. The patient has food
intolerance of eggs and fish. Objectively: stomach palpation reveals painfulness in the
gastroduodenal area. Electrophasoduodenoscopy revealed a 5 mm ulcer on the anterior wall of
duodenum. Urease test is positive. What is the most probable leading mechanism of disease
development?
A. Helicobacterial infection B. Dietary allergy
C. Autoantibody production
D. Reduced prostaglandin synthesis
E. Disorder of gastric motor activity
7. In order to prevent wound infection associated with surgical procedures a patient was
given a synthetic antiprotozoan drug with a high activity against Helicobacter pylori. Specify this
drug:
A. Metronidazole B. Doxycycline hydrochloride
C. Chingamin D. Acyclovir E. Isoniazid
8. A 6-year-old child has duodenal ulcer. What antibacterial drug should be co-administered
together with metronidazole and De-Nol in order to eradicate Helicobacter pylori infection?
A. Amoxicillin B. Tetracycline C. Oleandomycin
D. Biseptol E. Sulfadimethoxinum
9. In order to accurately identify etiology of peptic ulcer disease, polymerase chain reaction
(PCR) analysis was performed on gastric biopsy specimen taken from 47-year-old patient. Eventually,
the test result was positive for H. pylori. Which of the following was most likely detected in PCR
analysis?
A. Bacterial DNA B. Bacterial toxin
C. H. pylori bacterium itself D. Bacterial enzymes – urease and catalase
E. Bacterial antigen
10. Before a surgery the patient was prescribed a synthetic antiprotozoal drug for prevention
of wound infection. The prescribed drug is highly effective against Helicobacter pylori. Name this
drug:
A. Metronidazole B. Chingamin (Chloroquine)
C. Doxycycline hydrochloride D. Aciclovir E. Isoniazid

Actinomycetes
1. A 40 year old man noticed a reddening and an edema of skin in the area of his neck that
later developed into a small abscess. The incised focus is dense, yellowish-green. The pus contains
white granules. Histological examination revealed drusen of a fungus, plasmatic and xanthome cells,
macrophages. What type of mycosis is the most probable?
A. Actinomycosis B. Aspergillosis C. Candidosis
D. Sporotrichosis E. Coccidioidomycosis
2. A 40-year-old man developed skin redness and an swelling in the neck area, where
eventually a small abscess appeared. The section the focus is dense and yellow-green colored. In the
purulent masses there are white granules. Histologically there are fungal druses, plasma and
xanthome cells, and macrophages detected. Specify the most correct etiological name of this
pathological process?
A. Actinomycosis B. Furuncle C. Carbuncle
D. Syphilis E. Leprosy
3. Microscopical examination of an infiltrate removed from the submandibular skin area of a
30 y.o. man revealed foci of purulent fluxing surrounded by maturing granulations and mature
connective tissue, the pus contains druses consisting of multiple short rod-like elements with one
end attached to the homogenous centre. What disease is it?
A. Actinomycosis B. Tuberculosis
C. Syphilis D. Candidosis E. –

397
4. A 32-year-old patient who lives in the countryside consulted a doctor about a painful
swelling and a fistula in the submandibular region. Examination revealed an infiltration with a fistula
discharging thick pus and containing white granules. On dissection the infiltration tissues turned out
to be dense, yellow-green and had honeycomb structure because of multiple abscesses. What is the
most likely diagnosis?
A. Actinomycosis B. Tuberculosis C. Lepra
D. Syphilis E. Submandibular abscess
5. Microscopic examination of pus sample taken from mandibular fistula canal and stained by
Gram’s method has revealed druses with gram-positive coloring in the center and cone-shaped
structures with gram-negative coloring. Such morphology is characteristic of the agent of:
A. Fusobacteriosis
B. Actinomycosis
C. Staphylococcal osteomyelitis
D. Anaerobic infection
E. Candidiasis
6. A 40-year-old man developed skin redness and swelling in the neck area, where eventually
a small abscess appeared. On section the focus is dense and yellow-green colored, in the purulent
masses there are white granules. Histologically there are fungal druses, plasma and xanthoma cells,
and macrophages detected. Specify the most correct etiological name of this pathological process:
A. Syphilis B. Carbuncle C. Actinomycosis D. Leprosy E.
Furuncle

Candida
1. A 3 month old infant has got a white deposition on the mucous membrane of his mouth,
tongue and lips. The doctor suspected candidosis. What nutrient medium should be used for
inoculation of the material under examination in order to confirm this diagnosis?
A. Sabouraud B. Endo
C. Loewenstein-Jensen D. Roux E. Clauberg
2. A 3 m.o. baby has white film on the mucous membrane of his mouth, tongue and lips. A
doctor suspected candidosis. What nutrient medium should be applied for inoculation of the
material under examination in order to confirm this diagnosis?
A. Sabouraud’s B. Endo C. Jensen’s
D. Roux E. Clauberg’s
3. Examination of a child revealed some whitish spots looking like coagulated milk on the
mucous membrane of his cheeks and tongue. Analysis of smears revealed gram-positive oval yeast-
like cells. What causative agents are they?
A. Candida B. Staphylococci C. Diphtheria bacillus
D. Actinomycetes E. Fusobacteria
4. During examination of a 3-month old infant a pediatrician revealed that the baby’s oral
mucosa and tongue were covered with a thick white deposit. In the material taken from the affected
site a bacteriologist revealed the presence of yeast fungi giving the reasons for suspecting a fungal
infection which occurs most often in children of this age, namely:
A. Candidiasis B. Favus C. Epidermophytosis
D. Actinomycosis E. Trichophytia
5. Microscopic examination of a Gramstained scrape from patient’s tongue revealed oval,
round, elongated chains of dark-violet gemmating cells. What disease can be caused by this causative
agent?
A. Candidosis B. Actinomycosis C. Diphtheria
D. Staphylococcic infection E. Streptococcic infection
6. A 9 y.o. child has been taking antibiotics on account of bronchopneumonia for a long time.
There appeared pain and burning in the area of mucous membrane of his lips and tongue.
Objectively: mucous membrane of lips and tongue has caseous and grey plaques that can be easily
removed by a spatula leaving hyperemia foci on their spot. Microscopic examination of the plaques
revealed mycelium. What is the most probable diagnosis?

398
A. Candidous cheilitis B. Exfoliative cheilitis
C. Leukoplakia D. Contactant allergic cheilitis
E. Manganotti’s cheilitis
7. After continuous treatment with antibiotics a patient got symptoms of stomatitis.
Examination of specimens of oral mucous membrane revealed some oval polymorphous Gram-
positive microorganisms arranged in clusters. What microorganism may be the cause of such
manifestations?
A. С.albicans B. C.perfringens C. S.aureus
D. S.pyogenes E. H.pylori
8. After long-term antibiotic treatment a patient has developed whitish spots on the oral
mucosa. Gram-positive oval budding cells were detected in the smear preparations. What causative
agents were detected?
A. Candida fungi
B. Staphylococci
C. Sarcinae
D. Actinomycete
E. Tetracocci
9. A 70-year-old man has developed prosphetic stomatitis. Apart of this he was found to have
an evident lesion of mouth corners. Microscopical examination revealed large ovoid gram-positive
cells. What microorganisms are most likely to be the leading etiological agent of such a lesion?
A. Candida fungi B. Streptococci C. Staphylococci
D. Neisseria E. Corynebacteria
10. An 18-year-old patient has developed candidiasis after the case of pneumonia treated
with β-lactam antibiotic. What antimycotic agent should be prescribed?
A. Fluconazole B. Streptomycin C. Phthalylsulfathiazole
D. Ampicillin E. Trimethoprim/sulfamethoxazole
11. A patient who has been taking tetracycline for a long time has developed candidosis of
mucous membranes. What drug shoul administered for its treatment?
A. Itraconazole B. Griseofulvin C. Nitrofungin
D. Amphotericin E. Nitrofurantoin
12. A female who had been continuously taking antibiotics for an intestinal infection
developed a complication manifested by inflammation of the oral mucosa and white deposit.
Bacteriological study of the deposit samples revealed yeast fungi Candida albicans. Which of the
following medications is indicated for the treatment of this complication?
A. Fluconazole B. Biseptol C. Tetracycline
D. Furazolidone E. Polymyxin
13. A pregnant woman complains of vaginal mucosa irritation, itching and genital tracts
secretion. Bacterioscopy of vaginal smears revealed large gram-positive oval oblong cells that form
pseudomicelium. What is the most probable channel of infection?
A. Endogenous infection B. Sexual transmission C. Contact infection
D. Vector-borne transmission E. Wound infection
A 58-year-old male patient visited his dentist with the chief complaint of itching and burning
sensation in his mouth. On intraoral examination, diffuse white patches were seen on his tongue,
right and left buccal mucosa, as well as on his hard palate and soft palatal region. The potassium
hydroxide (KOH) preparation of the specimen revealed non-pigmented septate hyphae.
Administration of which of the following is the most appropriate initial step in treatment of this
patient?
A. Nystatin B. Tetracycline C. Gentamicin D. – E. Penicillin
13. An 18-year-old patient has developed candidiasis after the case of pneumonia treated
with β-lactam antibiotic. What antimycotic agent should be prescribed?
A. Streptomycin B. Fluconazole C.
Phthalylsulfathiazole
D. Ampicillin E. Trimethoprim/sulfamethoxazole (Biseptol)

399
14. A man uses dentures. The dentist has noticed mucosal lecions with a white coating in his
oral cavity. Microscopy of the coating detected large oval Gram-positive cells. What microorganisms
have caused stomatitis in the patient?
A. Yeast-like fungi of Candida genus B. Actinomycetes
C. Streptococci D. Oral spirochetes E. Oral trichomonas
15. Broadspectrum antibiotics can cause various complications, including intestinal
candidiasis. What drug is used for treatment of this complication?
A. Nystatin B. Griseofulvin C. Undecyne D. Amphotericin B E.
Gramicidin

Fungi
1. A patient with skin mycosis has disorder of cellular immunity. The most typical
characteristic of it is reduction of the following index:
A. T-lymphocytes B. Immunoglobulin G C. Immunoglobulin E
D. B-lymphocytes E. Plasmocytes
2. In order to establish the possible contamination of a medication with fungi, a nutrient
medium was inoculated, which resulted in growth of large cream-like colonies. What nutrient
medium was used in this case?
A. Sabouraud B. Lowenstein-Jensen
C. Roux D. Loeffler E. Finn-2
3. Crude herbal drugs must be examined for yeast-like fungi. What agar can ensure
development of these microorganisms so that associating microflora will grow very slowly or won’t
grow at all?
A. Sabouraud’s peptone agar B. Endo agar
C. Milk-salt agar D. Meat infusion agar E. Blood agar
4. What preparations are used for prevention of fungal infection?
A. Fluconozol, Orungal, Nisoral
B. Rubomycin, Bleomycin, Mytomycin C
C. Cytosar, Cormyctin, Lomycitin
D. Captopril, Enalapril E. Isoniazid, Ftibazid, Pyrazinamid
5. Name the halogen-containing antiseptic with fungicidal properties, which is used to treat
dermatomycosis:
A. Iodine solution
B. Formalin solution
C. Methylene blue
D. Brilliant green
E. Boric acid solution
Sanitary microbiology
1. During the regular sanitary epidemiological inspection of a pharmacy, the bacteriological
analysis of air was performed. The air was found to have bacilli, yeast fungi, hemolytic streptococci,
micrococci. Which of the detected microorganisms indicate the direct epidemic danger?
A. Haemolytic streptococci B. Micrococci
C. Bacilli D. Yeast fungi E. –
2. Presence of pathogenic microorganisms in the air can be detected by presence of sanitary
representative bacteria. Choose bacteria that are indicators of direct epidemiological danger:
A. Hemolytic streptococci B.Sarcina C.Molds D.Yeast E.Micrococci
3. Sanitary microbiological analysis of the indoor air of a pharmacy carried out in summer
revealed presence of Streptococcus haemolyticus and Streptococcus viridians at the rate of 40
microorganisms per 1 m3. Specify the microbiological characteristic of the air:
A. Contaminated B. Within the permissible limits
C. Almost pure D. Pure
E. These microorganisms are not the determinants of the air quality

400
4. Sanitary and bacteriological examination of air in drug-store premises revealed increased
content of sanitary representative microorganisms. What microorganisms are these?
A. Golden staphylococcus and hemolytic streptococcus
B. Diphtheria and tuberculosis bacilli C. Colon and blue pus bacilli
D. Epidermal staphylococcus and sarcina E. Enterococci and citrobacter
5. During sanitary and bacteriological examination of air in a drugstore it was revealed that
the air had high concentration of sanitary meaningful microorganisms. What microorganisms are
these?
A. Staphylococcus aureus and hemolytic streptococcus
B. Diphtheritic and tuberculous bacilli C. Colibacilli and blue pus bacilli
D. Epidermal staphylococcus and Sarcina E. Enterococci and Citrobacter
6. Sanitary-biologic examination of air in a drugstore revealed a sanitary-indicative
microorganism. Name it:
A. Staphylococcus aureus B. Colon bacillus C. Fecal enterococcus
D. α-haemolytic streptococcus E. Citrobacter
7. Routine investigation of microbiological sanitary condition of air in a hospital is performed
once in 3 months. What microorganism is the sanitary indicator of air condition in an enclosed
space?
A. S.aureus
B. E.coli
C. E.faecalis
D. P.aeruginosa
E. C.perfringens
8. Sanitary bacteriological research on water by the membrane filter method revealed two
red colonies on a membrane filter (Endo agar) through which 500 ml of analyzed water were passed.
Calculate the coli index and coli titer of the analyzed water:
A. 4 and 250 B. 2 and 500 C. 250 and 4
D. 500 and 2 E. 250 and 2
9. During sanitary and bacteriological testing of water with the membrane filter technique
there were revealed two red colonies on a membrane filter (Endo agar) through which 500 ml of
water was filtred. Calculate the coli index and coli titer of the analyzed water:
A. 4 and 250 B. 2 and 500 C. 250 and 4
D. 500 and 2 E. 250 and 2
10. After the sanitary and bacteriological study of tap water the following results were
obtained: the total number of bacteria in 1,0 ml was 80, coli index was 3. How would you interpret
the study results?
A. Water is safe to be consumed
B. Water is of doubtful quality
C. Water is of highly doubtful quality
D. Water is contaminated E. Water is highly contaminated
11. Bacteriological analysis of tap water has resulted in the following: total bacterial count in
1,0 ml of water is 80, coli index is 3. What would be the conclusion?
A. The water is safe for consumption
B. The water quality is doubtful
C. The water quality is extremely doubtful
D. The water is polluted
E. The water is extremely polluted
12. A laboratory received a sample of water used in drug production for sanitary and viral
analysis. What group of viruses will indicate fecal contamination of water and thus the need for its
additional purification?
A. Picornaviridae B. Herpesviridae C. Flaviviridae
D. Retroviridae E. Orthomyxoviridae

401
13. In an urban settlement situated on the riverbank, an outbreak of hepatitis A was
registered. The disease might have water origin. This assumption can be confirmed by growth of the
following values of water quality:
A. Number of coli-phages B. Escherichia coli index
C. Oxidability D. Index of fecal coli-forms
E. Presence of benign leptospirosis pathogen
14. Basing upon the data of laboratory assessment of sanitary state of soil in a certain
territory, the soil was found to be low-contaminated according to the sanitary indicative value;
contaminated according to the coli titer; low-contaminated according to the anaerobe titer (Cl.
perfringens). This is indicative of:
A. Fresh fecal contamination B. Old fecal contamination
C. Insufficient intensity of soil humification
D. Constant entry of organic protein contaminations
E. Insufficient insolation and aeration of soil
15. After the water supply system had been put into operation in a new residential area, the
medical officers of sanitary and epidemiological station measured total microbial number in the
water. Name the maximum permissible value of this indicator for potable water:
A. 1000. B. 500. C. 400. D. 100. E. 10.
16. A bacteriological laboratory conducts the analysis of potable water quality. Microbial
number of the water sample is approximately 100. What microorganisms were accounted for in this
case?
E. All bacteria that have grown on a nutrient medium B. Opportunistic
pathogenic bacteria
C. Enteropathogenic bacteria and viruses D. Colibacilli
E. Human and animal pathogenic bacteria

402
№ krok 2020
Тема Drugs acting on cardiovascular system
Текст задания A patient was delivered into the admission room. He has the signs of
acute heart failure: pallor, acrocyanosis, frequent and shallow
respiration. What medicine of those listed below is indicated in this
case?
Правильный ответ Corglycon (Convallatoxin)
B Cordiamin (Niketamide)
C Digitoxin
D Adrenaline hydrohloride
E Nitroglycerine
№ krok 2020
Тема Drugs acting on the CNS
Текст задания A patient with signs of anxiety, fear, uncertaity and mental strain
was prescribed diazepam. What mechanism of tranquilizing action
can be observed in this case?
Правильный ответ Interaction with benzodiazepine receptors
B Interaction with serotonin receptors
C Interaction with adrenergic receptors
D Interaction with cholinergic receptors
E Interaction with dopamine receptors
№ krok 2020
Тема Drugs acting on the CNS
Текст задания
A patient wih severe toothache that lasted for several days made no
appointment with a doctor and engaged in self-tretment instead. As a
resalt, his tooth needs to be extracted. What analgesic increases the
probability of a hemorrhage developing after the tooth is extracted?
Правильный ответ Acetylsalicilyc asid
B Dimedrol (Diphenhydramine)
C Codeine phosphate
D Paracetampl
E Analgin (Metamizole)
№ krok 2020
Тема Antimicrobial agents
Текст задания A dentist is used a solution of potassium permanganate as an
antiseptic. This preparation has a bactericidal affect because of:
Правильный ответ Atomic oxygen
B Manganese oxide
C Potassium hudroxide
D Potassium
E Potassium oxide
№ krok 2020
Тема Antimicrobial agents
Текст задания A patint with streptococcal pneumonia was prescribed an
antimicrobial agent that disrupts the formation of microbial
membranes. Name this drug
Правильный ответ Benzylpenicillin sodium salt
B Erythromycin
C Azithromycin
D Gentamicin
E Doxycycline
№ krok 2020
Тема Antimicrobial agents
Текст задания A 4-year-old chold has numerous carious cavities and yellow-
colored teeth. It isknown that during her pregnancy the childs
mother was undergoing an antibiotic treatment. What medicine was
likely being taken by the childs mother?
Правильный ответ Doxycycline
B Erythromycin
C Cefazolin
D Ampicillin
E Streptomicin sulfate
№ krok 2020
Тема Drugs acting on metabolism, blood and immunity
Текст задания After a glucose-lowering therapy, a patient with diabetes mellitus
developed hypoglycemia. What hormone, taken in excess, could
have caused this hupoglycemic condition?
Правильный ответ Insulin
B Thyroxine
C Glucagon
D Adrenaline
E Cortisol
№ krok 2020
Тема Drugs acting on cardiovascular system
Текст задания During dental manipulation, the patient developed an angina
pectoris attack. What droup of drugs needs to be prescribed ror the
emergency aid in this case?
Правильный ответ Antianginal drugs
B Antiarrhythmic drugs
C Antihypertensive drugs
D Cardiotonics
E Respiratiry stimulants
№ krok 2020
Тема Antimicrobial agents
Текст задания A doctor has made a diagnosis of gingivitis and recommended the
patient to rinse the oral cavity with an oxidizing agent. Specify this
agent
Правильный ответ Hydrogen peroxide
B Brilliant green
C Salicylic asid
D Boric acid
E Phenol
№ krok 2020
Тема Drugs acting on cardiovascular system
Текст задания A patient has arrythmia. What medicine needs to be prescribed in
this case?
Правильный ответ Amiodarone
B Nitroglycerine
C Imizine
D Cavinton
E Euphillin
№ krok 2020
Тема Drugs acting on metabolism, blood and immunity
Текст задания A 60-year-old man with diabetes mellitus was prescribed insulin.
What tipe of pharmacologycal therapy is it?
Правильный ответ Replacement
B Symptomatic
C Preventive
D Etiotropic
E Pathogenetic
№ krok 2020
Тема Drugs acting on the CNS
Текст задания A cessation of morfine administration after its long-term use leads to
the development of severe mental, neurological and somatic
disoders.name this condition
Правильный ответ Withdrowal
B Sensitisation
C Idiosyncrasy
D Toleranse
E Cumulation
№ krok 2020
Тема Agents influencing on afferent and efferent innervation
Текст задания For a surgerry in the maxillofacial area, cholinergic agents are used
to decrease salivation. What drug of yhose listed bellow would you
recommendfor this purpose?
Правильный ответ Atropine sulfate
B Adrenaline hydrohloride
C Proserin
D Lobeline hydrochloride
E Dithylinum
№ krok 2020
Тема Drugs affecting the gastrointestinal tract
Текст задания
A patient with peptic ulser disease of the stomach is prescribed a
drug that blocks histamine H2 reseptors.Select this drug from the list
Правильный ответ Famotidine
B Bisacodil
C Atropine sulfate
D Omeprazole
E Dithylinum
№ krok 2020
Тема Drugs acting on cardiovascular system
Текст задания What nitrate drug would you recommend to a patient with ischemic
heart disease for prevention of angina pectoris attacks?
Правильный ответ Isosorbide mononitrate
B Lovastatin
C Nitroglycerine
D Mentol
E Lisinipril
№ krok 2020
Тема Drugs acting on cardiovascular system
Текст задания A certain drug with potent natriuretic action is usually prescribed
fordehydration therapy of cerebral and pulmonary edemas. Name
this drug
Правильный ответ Furosemide
B Spironolactone
C Mannitol
D Theophylline
E Etacrynic aced
№ krok 2020
Тема Agents influencing on afferent and efferent innervation
Текст задания During an appointment with the dentist, a patient developed a
bronchial asthma attack.what does this patient need to be prescribed
to terminate the bronchospasm?
Правильный ответ Salbutamol
B Anaprilin
C Benzohexonium
D Methacin
E Droperidol
№ krok 2020
Тема Antimicrobial agents
Текст задания A patient is diagnosed with stomatites caused by herpes simplex
virus, type 1 and 2. What medicine that is an analogue of
nucleosides and is converted by thymidine kinase can provide highly
effective selective antiviral therapy?
Правильный ответ Acyclovir
B Rimantadine
C Laferon
D Acetylcystein
E Oxolin
№ krok 2017
Тема Drugs acting on the CNS
A 24-year-old patient has been administered glutamic acid to treat
epilepsy. Medicinal effect in this case occurs not due to glutamate
Текст задания itself, but due to the product of its decarboxylation:
Правильный ответ γ-aminobutyric acid
B Histamine 4-monooxygenase
C Serotonin
D Dopamine
E Taurine
№ krok 2017
Тема Agents influencing on afferent and efferent innervations
To perform conduction anesthesia a patient had been administered
a drug used in dental surgery. The patient developed the symptoms
of poisoning: central nervous system excitation followed by
paralysis and acute cardiovascular insufficiency (collapse).
Additionally there were allergic reactions (itching, swelling,
Текст задания erythema). Name this drug:
Правильный ответ Lidocaine
B Suxamethonium chloride
C Thiopental sodium
D Tubocurarin chloride
E Pipecuronium bromide
№ krok 2017
Тема Drugs acting on metabolism, blood and immunity
Enzymes and other active substances regulating connective tissue
density and permeability are being produced in connective tissue
cells. What enzyme drug is used to make the connective tissue
Текст задания growths looser and more permeable?
Правильный ответ Lydase
B Amylase
C Lipase
D Cocarboxylase
E Cholinesterase
№ krok 2017, 2016
Тема Drugs acting on the CNS
A schizophrenic patient has been prescribed aminazine. What
pharmacodynamic action of this drug justifies its prescription in this
Текст задания case?
Правильный ответ Antipsychotic
B Antiemetic
C Hypothermic
D Muscle relaxant
E Hypotensive
№ krok 2017
Тема Drugs acting on the CNS
A patient with maxillofacial joint arthritis has come to a dentist. The
dentist prescribed an ointment with an anti-inflammatory agent
Текст задания that is a pyrazolone derivative. Name this agent:
Правильный ответ Butadion (Phenylbutazone)
B Mefenamic acid
C Ibuprofen
D Indometacin
E Diclofenac sodium
№ krok 2017, 2015
Тема Drugs acting on cardiovascular system
A 55-year-old man with acute heartfailure has been administered a
quick-relief cardiac glycoside. Which of the following drugs has
Текст задания been given to the patient?
Правильный ответ Strophanthin
B Adonisidum
C Digitoxin
D Celanid
E Milrinone
№ krok 2017
Тема Drugs acting on cardiovascular system
A patient suffering from ciliary arrhythmia with anamnesis of
bronchial asthma should be prescribed an anti-arrhythmic drug.
Текст задания What antiarrhythmic drug is CONTRAINDICATED in this case?
Правильный ответ Anaprilin (Propranolol)
B Ajmaline
C Verapamil
D Nifedipine
E Novocainamide (Procainamide)
№ krok 2017
Тема Drugs acting on metabolism, blood and immunity
A woman, who had undergone mastectomy due to breast cancer,
was prescribed a course of radiation therapy. What vitamin
preparation has marked antiradiation effect due to its antioxidant
Текст задания activity?
Правильный ответ Tocopherol acetate
B Ergocalciferol
C Riboflavin
D Cyanocobalamin
E Folic acid
№ krok 2017
Тема Agents influencing on afferent and efferent innervations
A 45-year-old patient with essential hypertension, who has been
taking an antihypertensive drug for 4 days, notes that his blood
pressure is normalized; however the patient complains of
Текст задания sleepiness and sluggishness. What drug is the patient taking?
Правильный ответ Clophelin (Clonidine)
B Prazosin
C Captopril
D Enalapril
E Apressin (Hydralazine)
№ krok 2017
Тема Drugs acting on cardiovascular system + General pharmacology
To terminate hypertensive crisis the patient was administered
solution of magnesium sulfate. What route of drug administration
Текст задания should be chosen?
Правильный ответ Intravenous
B Duodenal
C Rectal
D Oral
E Intra-arterial
№ krok 2017
Тема Agents influencing on afferent and efferent innervations
To treat peptic ulcer disease of the stomach a patient has been
prescribed famotidine. Specify the mechanism of action of this
Текст задания drug:
Правильный ответ H2 histamine receptors block
B Effect on ion channels of cell membranes
C Antienzyme action
D Physicochemical interaction
E Effect on cell membrane transport system
№ krok 2017
Тема Agents influencing on afferent and efferent innervations
Due to overdosage of a vasodilator a 58-year-old patient has
developed acute vascular insufficiency. What drug would be
Текст задания advisable for termination of this pathological condition?
Правильный ответ Mesaton (Phenylephrine)
B Euphyllin (Aminophylline)
C Dopamine
D Cordiamin
E Aethimizolum (Methylamide)
№ krok 2017
Тема Drugs acting on cardiovascular system

To treat chronic heart failure a patient takes digoxin. What diuretic


Текст задания can increase digoxin toxicity due to increased excretion of K+ ions?
Правильный ответ Hydrochlorothiazide
B Spironolactone
C Panangin
D Silibor
E Lisinopril
№ krok 2017, 2016
Тема Drugs acting on cardiovascular system
A patient with symptoms of acute heart failure, namely pallor,
acrocyanosis, and rapid shallow breathing, has been delivered to an
emergency department. Which of these drugs is indicated in this
Текст задания case?
Правильный ответ Corglycon
B Digitoxin
C Cordiamine
D Nitroglycerine
E Adrenalin hydrochloride
№ krok 2017
Тема Drugs acting on metabolism, blood and immunity
A patient suffering from acute vascular purpura is prescribed a first-
generation antihistamine with local anaesthetic, antispasmodic, and
Текст задания sedative action. Specify this drug:
Правильный ответ Dimedrol (Diphenhydramine)
B Diazolin (Mebhydrolin)
C Dithylin
D Droperidol
E Dibazol (Bendazol)
№ krok 2017
Тема Drugs acting on metabolism, blood and immunity
A patient with autoimmune thyroiditis has been prescribed a
Текст задания peptide hormonal agent. Specify this agent:
Правильный ответ L-thyroxin
B Trimethoprim
C Triamcinolone
D Triquilar
E Tamoxifen
№ krok 2017
Тема Antimicrobial agents

To prevent wound infection associated with surgical procedures a


patient was given a synthetic antiprotozoan drug demonstrating
Текст задания high activity against Helicobacter pylori. Specify this drug:
Правильный ответ Metronidazole
B Doxycycline hydrochloride
C Chingamin
D Acyclovir
E Isoniazid
№ krok 2017, 2016
Тема Agents influencing on afferent and efferent innervations
A patient in a grave condition has been delivered to an admission
ward. Examination revealed pupil mydriasis, no reaction to the
light, considerable reddening and dryness of skin and mucous
membranes. What drug could have caused the intoxication
Текст задания symptoms?
Правильный ответ Atropine sulphate
B Proserin
C Adrenalin hydrochloride
D Pilocarpine hydrochloride
E Dithylinum
№ krok 2017
Тема Agents influencing on afferent and efferent innervations
A patient with bronchial asthma has developed a bronchial spasm
during the visit to a dentist. Name the drug necessary to arrest the
Текст задания spasm:
Правильный ответ Salbutamol
B Naphthizin
C Mesaton (Phenylephrine)
D Anaprilin (Propranolol)
E Bisoprolol
№ krok 2017
Тема Drugs acting on the CNS
Due to severe pain syndrome a patient has been prescribed a
Текст задания narcotic analgesic. Specify the prescribed drug:
Правильный ответ Morphine
B Analgin (Metamizole)
C Nimesulid
D Dimexid
E Indometacin
№ krok 2017
Тема Drugs acting on the CNS
A patient suffering from arthritis is prescribed a COX2 selective
Текст задания inhibitor with anti-inflammatory action. Specify this drug:
Правильный ответ Celecoxib
B Butadion (Phenylbutazone)
C Dimexid
D Indometacin
E Analgin (Metamizole)
№ krok 2017
Тема Antimicrobial agents
A doctor has made a diagnosis of gingivitis and recommended the
patient to rinse the oral cavity with an oxidizing agent. Specify this
Текст задания agent:
Правильный ответ Hydrogen peroxide
B Boric acid
C Salicylic acid
D Phenol
E Brilliant green
№ krok 2017
Тема Agents influencing on afferent and efferent innervations
In dental practice atropine sulfate is used to decrease salivation.
Текст задания This drug belongs to the following group:
Правильный ответ Muscarinic antagonists
B α-adrenergic agonists
C β-adrenoceptor antagonists
D Ganglionic blockers
E Muscle relaxants
№ krok 2016
Тема Drugs acting on cardiovascular system
A 50-year-old patient with hypertensive crisis had been
administered magnesium sulfate, which led to abrupt decrease in
blood pressure. The side effects of magnesium sulfate can be
Текст задания prevented if the following drug is administered:
Правильный ответ Calcium chloride
B Potassium chloride
C Trilon B
D Sodium bromide
E Sodium sulfate
№ krok 2016
Тема Drugs acting on metabolism, blood and immunity
A patient has a slowly healing fracture. What medicine can be used
Текст задания to accelerate formation of connective tissue matrix?
Правильный ответ Methyluracil
B Prednisolone
C Cyclophosphan
D Methotrexate
E Cyclosporine
№ krok 2016
Тема General pharmacology

A 17-year-old girl has attempted to commit suicide by overdosing


on phenobarbital. Upon arrival at the site an emergency doctor
urgently performed gastric lavage on the patient and introduced
bemegride and solution of sodium hydrocarbonate intravenously.
Текст задания The doctor introduced sodium hydrocarbonate to:
Правильный ответ Increase renal excretion of phenobarbital
B Stimulate respiration
C Normalise blood pressure
D Inactivate phenobarbital
E Wake up the patient
№ krok 2016
Тема Drugs acting on metabolism, blood and immunity
A child with signs of rickets has been prescribed a certain
liposoluble vitamin drug by pediatrician and dentist. This drug
affects the metabolism of phosphorus and calcium in the body and
facilitates calcium accumulation in bone tissue and dentine. If its
content in the body is insufficient, there develop disruptions of
ossification process, dental structure and occlusion. Name this
Текст задания drug:
Правильный ответ Ergocalciferol
B Retinol acetate
C Tocopherol acetate
D Menadione (Vicasolum)
E Thyroidin
№ krok 2016
Тема Antimicrobial agents
Having completed work in a laboratory, a student must tidy up the
workspace, perform disinfection of the workbench and tools. What
Текст задания chemicals should be used for disinfection?
Правильный ответ Chloramine
B Hydrochloric acid
C Formalin
D Chloroform
E Ether
№ krok 2016, 2015
Тема Drugs acting on the CNS
A 56-year-old woman is registrated with a psychoneurologic
dispensary due to her suffering from epilepsy, specifically, minor
Текст задания attacks (pti mal). What drug is the most efficient in this case?
Правильный ответ Sodium valproate
B Phenobarbital
C Trihexyphenidyl
D Levodopa
E Phenytoin
№ krok 2016, 2015
Тема Agents influencing on afferent and efferent innervations
A 60-year-old patient with a history of bronchial asthma has had
several attacks during the day. What is the optimal drug to be used
Текст задания for attacks prevention?
Правильный ответ Salbutamol
B Isadrinum
C Adrenaline hydrochloride
D Dobutamine
E Methacinum
№ krok 2016
Тема Drugs acting on the CNS

A patient suffering from parkinsonism has been prescribed


levodopa, which resulted in rapid improvement of the patient’s
Текст задания condition. What mechanism of action is characteristic of this drug?
Правильный ответ Stimulation of dopamine synthesis
B Block of muscarinic receptors
C Stimulation of dopamine receptors
D Anticholinesterase action
E Stimulation of muscarinic receptors
№ krok 2016
Тема Drugs acting on the CNS
A 45-year-old woman has addressed a doctor with complaints of
rapid mood swings, tearfulness, apathy. Antidepressants prescribed
for her treatment are monoamine oxidase inhibitors. These drugs
have a certain effect on catecholamines, which results in their
Текст задания medicinal action. Name this effect:
Правильный ответ Increased concentration
B Increased deaminization
C Decreased concentration
D Neutralization activation
E Inhibition of back transfer
№ krok 2016
Тема Agents influencing on afferent and efferent innervations
A patient with limb fracture should be prescribed a depolarizing
Текст задания muscle relaxant for brief surgical invasion. Name this drug:
Правильный ответ Dithylinum
B Tubocurarin chloride
C Cytisinum (Cytitonum)
D Atropine sulfate
E Azamethonium bromide (Pentaminum)
№ krok 2016
Тема General pharmacology
A 50-year-old patient has been referred for treatment of neck
lymphadenitis. His individual penicillin sensitivity was tested. In 30
seconds fullbody fever raised in the patient and his arterial blood
pressure dropped to 0 mm Hg followed by cardiac arrest.
Resuscitation was unsuccessful. Autopsy revealed acute venous
hyperemia of viscera. Histological study revealed mast cells (tissue
basocytes) degranulation in the skin (at the area of injections),
myocardium and lungs. What kind of hypersensitivity reaction
Текст задания occurred in the patient?
Правильный ответ Anaphylactic
B Delayed-type hypersensitivity
C Complement-mediated cytotoxic
D Immune complex-mediated
E -
№ krok 2016
Тема General pharmacology
A patient with heart failure and tachycardia has been prescribed
digoxin. After 5 days of taking digoxin the patient’s heart rate was
normalized. After 2 weeks the patient addressed a doctor due to
continuous decrease of heart rate down to 52/min. What
Текст задания phenomenon has caused such changes of heart rate?
Правильный ответ Cumulation
B Idiosyncrasy
C Tachyphylaxis
D Tolerance
E Allergy
№ krok 2016
Тема Antimicrobial agents
Sulfanilamides are applied as antimicrobal agents in clinical
practice. Sulfanilamide treatment, however, can result in hemolytic
anemia development in patients that suffer from genetic defect of
the following enzyme of pentose phosphate metabolism in
Текст задания erythrocytes:
Правильный ответ Glucose-6-phosphate dehydrogenase
B Hexokinase
C Transketolase
D Transaldolase
E Pyruvate kinase
№ krok 2016
Тема General pharmacology
For pain relief a patient has taken simultaneously a tablet of
paracetamol and a tablet of diclofenac sodium. What type of drug
Текст задания interaction did the patient use for self-treatment?
Правильный ответ Additive synergism
B Potentiated synergism
C Synergic antagonism
D Competitive antagonism
E Non-competitive antagonism
№ krok 2016
Тема Drugs acting on cardiovascular system

A 60-year-old patient has taken a drug to relieve angina pectoris


attack; in several minutes the pain felt in the breastbone area
abated, but it was followed by feeling of vertigo, headache,
Текст задания tinnitus, and hyperemia of face. What drug has the patient taken?
Правильный ответ Nnitroglycerine
B Validol
C Nifedipine
D Verapamil
E Amiodarone
№ krok 2016
Тема Drugs acting on cardiovascular system
For relief of hypertensive crisis a doctor has administered a patient
a drug that apart from antihypertensive effect has also sedative,
spasmolytic and anti-convulsive action. The drug was taken
parenterally. When it is taken enterally it acts as a laxative and
Текст задания cholagogue. What drug was administered?
Правильный ответ Magnesium sulfate
B Dibasolum
C Reserpine
D No-spa
E Apressin
№ krok 2016
Тема Drugs acting on metabolism, blood and immunity

A 13-year-old girl has been prescribed a certain drug for treatment


of megaloblastic anemia. This drug stimulates a transfer from
megaloblastic haemopoiesis to normoblastic, participates in
synthesis of purine and pyrimidine bases, activates proteine and
Текст задания methionine synthesis. What drug does the patient take?
Правильный ответ Cyanocobalamin
B Ferric sulfate
C Haemostimulinum
D Erythropoietin
E Rosehip tea
№ krok 2016
Тема General pharmacology
A patient has been administered conduction anesthesia with
novocaine in preparation for tooth extraction. After the anesthesia
administration the patient developed swelling and hyperemy
around the injection site, skin itch, general fatigue, motor agitation.
Текст задания Name the developed complication:
Правильный ответ Allergy
B Idiosyncrasy
C Tachyphylaxis
D Drug dependence
E Inflammation
№ krok 2016
Тема Drugs acting on metabolism, blood and immunity

A 30-year-old patient after a case of viral hepatitis type B has


developed complaints of continuous nasal hemorrhages. What drug
Текст задания would be the most advisable for treatment of this condition?
Правильный ответ Menadione (Vicasolum)
B Nadroparin calcium (Fraxiparine)
C Folic acid
D Dipiridamol
E Asparcam
№ krok 2016
Тема Agents influencing on afferent and efferent innervations
A patient is being prepared for a surgery - partial mandibular
resection. What drug should be administered to decrease
Текст задания salivation?
Правильный ответ Atropine sulfate
B Carbacholine
C Armine
D Tubocurarin chloride
E Lobeline
№ krok 2016
Тема Antimicrobial agents
A dentist has detected symptoms of parodontosis in a patient.
Текст задания What anti-protozoal drug should be prescribed?
Правильный ответ Metronidazole
B Levamisole
C Griseofulvin
D Mykoseptin
E Furazolidone
№ krok 2015
Тема Agents influencing on afferent and efferent innervations
To perform conduction anesthesia a patient had been administered
a drug used in dental surgery. It was followed by the symptoms of
poisoning: central nervous system excitation with following
paralysis, and acute cardiovascular insufficiency (collapse).
Additionally there were allergic reactions (itching, swelling,
Текст задания erythema). Name this drug.
Правильный ответ Lidocaine
B Suxamethonium chloride
C Thiopental sodium
D Tubocurarin chloride
E Pipecuronium bromide
№ krok 2015
Тема Agents influencing on afferent and efferent innervations
A patient in a collaptoid state has been given an injection of
mesatonum for the correction of blood pressure. What is the
Текст задания mechanism of this drug action?
Правильный ответ It stimulates α-adrenergic receptors
B It stimulates β-adrenergic receptors
C It blocks α-adrenergic receptors
D It blocks β-adrenergic receptors
E It stimulates α- and β-adrenergic receptors
№ krok 2015
Тема Drugs acting on the CNS
A 42-year-old female patient consulted a doctor about pain in the
knee joints. Objectively there is swelling, redness, hyperthermia in
the region of these joints. Laboratory testing revealed positive
acute phase reactants. What drugs should be used to treat this
Текст задания patient?
Правильный ответ Anti-inflammatory drugs
B Narcotic analgesics
C Antidepressants
D Antibiotics
E Sulfonamides
№ krok 2015
Тема Antimicrobial agents
A patient being treated for tuberculosis is suffering from hearing
Текст задания deterioration. What drug causes this complication?
Правильный ответ Streptomycin
B Isonicotinic acid hydrazide (Isoniazid)
C Kanamycin sulphate
D Ethionamide
E Rifampicin
№ krok 2015
Тема Drugs acting on cardiovascular system
A patient complains about retrosternal pain, dyspnea and
palpitation. After examination he was diagnosed with coronary
heart disease and prescribed verapamil. What is the mechanism of
Текст задания its action?
Правильный ответ It blocks calcium channels
B It blocks α-adrenoreceptors
C It blocks β-adrenoreceptors
D It blocks potassium channels
E It blocks sodium channels
№ krok 2015
Тема Drugs acting on cardiovascular system
A doctor has prescribed clonidine for rapid relief of hypertensic
crisis. What group does this drug belong to according to its
Текст задания mechanism of action?
Правильный ответ Central α2-adrenoceptor agonist
B α-adrenergic blocking agent
C Nonselective α-adrenoceptor agonist
D Selective α1-adrenoceptor agonist
E Selective β1-adrenergic blocking agent
№ krok 2015
Тема Antimicrobial agents
A patient has been diagnosed with bacillary dysentery. What drug
Текст задания of those listed below should be prescribed?
Правильный ответ Amoxicillin
B Benzylpenicillin sodium salt
C Isonicotinic acid hydrazide (Isoniazid)
D Itraconazole
E Acyclovir
№ krok 2015
Тема Agents influencing on afferent and efferent innervations
A patient was taken to a hospital with dizziness, dry mouth,
mydriatic pupils, accommodation disorder, tachycardia, difficult
urination, intestinal atony. These symptoms might have been
Текст задания caused by overdose of the following drug:
Правильный ответ Atropine sulfate
B Furosemide
C Clonidine
D Captopril
E Prazosin
№ krok 2015
Тема Drugs acting on metabolism, blood and immunity
A 30-year-old driver complains of allergic rhinitis that usually
excerbates in spring. He has been administered an anti-histamine
drug with a slight sedative effect and 24-hour period of action.
Текст задания Which of the listed drugs has been administered?
Правильный ответ Loratadine
B Dimedrol
C Heparin
D Vicasol
E Oxytocin
№ krok 2015
Тема Antimicrobial agents
A patient suffers from a severe life-threatening generalised septic
infection. What group of chemotherapeutical drugs should be
Текст задания prescribed in this case?
Правильный ответ Cephalosporines
B Tetracyclines
C Sulfanilamides
D Chloramphenicol group
E Macrolides
№ krok 2015
Тема Drugs acting on the CNS
To prevent possible negative effect upon the gastric mucsa a
patient with rheumatoid arthritis was administered a nonsteroid
anti-inflammatory drug - a COX-2 selective inhibitor. Specify this
Текст задания drug:
Правильный ответ Celecoxib
B Analgine
C Acetylsalicinic acid
D Butadion
E Ibuprofen
№ krok 2015
Тема Antimicrobial agents
A 46-year-old patient consulted a doctor about pustular rash on the
skin of the limbs. What antiseptic should be administered to the
Текст задания patient?
Правильный ответ Alcohol solution of iodine
B Insulin
C Prednisolone
D Sibazon
E Heparin
№ krok 2015
Тема Drugs acting on the CNS

A patient suffering from stenocardia takes 100 mg of acetylsalicilic


Текст задания acid daily. What is the effect of acetylsalicilic acid in this patient?
Правильный ответ Inhibition of thrombocyte aggregation
B Inhibition of blood coagulation
C Dilatation of coronary vessels
D Prothrombin rate reduction
E Cholesterol rate reduction
№ krok 2015
Тема Drugs acting on the CNS
A 45-year-old female patient has neurosis with irritability, insomnia,
amotivational anxiety. What tranquilizer will be able to eliminate all
Текст задания symptoms of the disease?
Правильный ответ Diazepam
B Paracetamol
C Piracetam
D Caffeine-sodium benzoate
E Levodopa
№ krok 2015
Тема Drugs acting on the CNS + Antimicrobial agents
A patient is diagnosed with acute morphine hydrochloride
poisoning. Choose the oxidant drug to be prescribed for gastric
Текст задания lavage.
Правильный ответ Potassium permanganate
B Chloramine
C Sulfocamphocainum (Procaine+Sulfocamphoric acid)
D Cerigel
E Chlorhexidine digluconate
№ krok 2015
Тема Drugs acting on metabolism, blood and immunity
A 26-year-old woman at 40 weeks’ gestation was admitted to the
maternity ward. Examination revealed that the cervix was open,
but uterine contractions were absent. The doctor gave her a
Текст задания hormonal drug to induce labor. Specify this drug:
Правильный ответ Oxytocin
B Hydrocortisone
C Estrone
D Testosterone
E ACTH
№ krok 2015
Тема Drugs acting on cardiovascular system
A female patient suffering from coronary artery disease has been
prescribed amiodarone that has antianginal action. What other
Текст задания action does this drug have?
Правильный ответ Antiarrhythmic
B Analgesic
C Local anaesthetic
D Anti-inflammatory
E Anti-shock
№ krok 2015
Тема General pharmacology
A patient with acute cardiac failure has been taking cardiac
glycoside drug for a long time. He has developed the following
symptoms: nausea, fatigue, extrasystole. What is the cause of this
Текст задания symptoms?
Правильный ответ Material cumulation
B Idiosyncrasy
C Acquired tolerance
D Functional cumulation
E Drug addiction
№ krok 2015
Тема Agents influencing on afferent and efferent innervations
Novocaine acts as an anesthetic by making nerve fibers unable to
conduct stimulation. What mechanism of action regarding
Текст задания membrane’s permeability to ions does this drug have?
Правильный ответ Sodium ion-selective channels blockade
B Potassium ion-selective channels blockade
C Calcium ion-selective channels blockade
D Sodium-potassium pump blockade
E Sodium-proton pump blockade
№ krok 2015, 2012
Тема Agents influencing on afferent and efferent innervations
A patient presented to a hospital with complaints about quick
fatigability and significant muscle weakness. Examination revealed
an autoimmune disease that causes functional disorder of
receptors in the neuromuscular synapses. This will result in the
Текст задания disturbed activity of the following mediator:
Правильный ответ Acetylcholine
B Noradrenaline
C Dopamine
D Serotonin
E Glycine
№ krok 2015
Тема Drugs acting on cardiovascular system
A 60-year-old patient has taken a drug to relieve angina pectoris
attack; in several minutes pain felt in the breastbone area abated,
but it was followed by feeling of vertigo, headache, tinnitus, and
Текст задания hyperemia of face. What drug has he taken?
Правильный ответ Nnitroglycerine
B Validol
C Nifedipine
D Verapamil
E Amiodarone
№ krok 2015
Тема Antimicrobial agents
A dentist has detected symptoms of parodontosis in a patient.
Текст задания What anti-protozoal drug should be prescribed?
Правильный ответ Metronidazole
B Levamisole
C Griseofulvin
D Mykoseptin
E Furazolidone
№ krok 2015
Drugs acting on cardiovascular system+ Drugs acting on
Тема metabolism, blood and immunity
A 50-year-old patient with a hypertensive crisis had been
administered magnesium sulfate, which led to an abrupt decrease
in blood pressure. The side effects of magnesium sulfate can be
Текст задания avoided if the following drug is administered:
Правильный ответ Calcium chloride
B Potassium chloride
C Trilon B
D Sodium bromide
E Sodium sulfate
№ krok 2014
Тема Agents influencing on afferent and efferent innervations
A patient with arterial hypertension has developed a bronchial
asthma attack. Which of the following bronchodilators may
Текст задания provoke a hypertensive crisis?
Правильный ответ Ephedrine hydrochloride
B Salbutamol
C Aminophylline
D Berotec
E Isadrine
№ krok 2014
Тема Drugs acting on cardiovascular system
A paroxysm of tachycardia occurred in a patient undergoing a
dental procedure. Which of the following drugs should be used to
Текст задания relieve it?
Правильный ответ Verapamil
B Isadrinum
C Atropine
D Dipheninum
E Nitroglycerine
№ krok 2014
Тема Drugs acting on the CNS
To perform a scheduled surgery on the upper jaw, a surgeon
decided to apply ataralgesia. What medications are used for this
Текст задания manipulation?
Правильный ответ Tranquilizers
B General anesthetics
C Narcotic analgesics
D Non-narcotic analgesics
E Sedatives
№ krok 2014
Тема Drugs acting on cardiovascular system
A 55-year-old mae patient with acute heart failure has been
administered a quickrelief cardiac glycoside. Which of the following
Текст задания drugs has been given to the patient?
Правильный ответ Strophanthin
B Adonisidum
C Digitoxin
D Celanid
E Milrinone
№ krok 2014
Тема Antimicrobial agents
Penicillin group which would be resistant to penicillinase. What
Текст задания drug can be recommended?
Правильный ответ Oxacillin
B Amoxicillin
C Carbenicillin
D Azlocillin
E Ampicillin
№ krok 2014
Тема Drugs acting on the CNS
A 49-year-old patient has psychotic manifestations in form of
psychomotor agitation, auditory and visual hallucinations. What
Текст задания drug is indicated in the described case?
Правильный ответ Haloperidol
B Sodium bromide
C Diazepam
D Valerian tincture
E Chlordiazepoxide
№ krok 2014
Тема Drugs acting on cardiovascular system
A 58-year-old patient with essential hypertension complains about
general health deterioration. She has been administered
Текст задания methyldopa. Specify the mechanism of this drug action:
Правильный ответ Inhibtion of noradrenaline synthesis
B Increase in acetylcholine synthesis
C Blockade of β-adrenergic receptors
D Blockade of α-adrenergic receptors
E Inhibition of angiotensin-converting factor
№ krok 2014, 2013
Тема Antimicrobial agents
A 30-year-old patient with pneumonia has been administered a 3-
day course of an antibiotic from the group of azalides that has
bactericidal effect, prolonged action, the ability to bind to
phagocytic cells and accumulate in the infection foci. What drug has
Текст задания been administered?
Правильный ответ Azithromycin
B Erythromycin
C Benzylpenicillin sodium salt
D Isoniazid
E Ciprofloxacin
№ krok 2014
Тема Drugs acting on cardiovascular system
A 57-year-old patient with coronary artery disease has been
administered an antianginal agent that activates guanylate cyclase
Текст задания and accumulates myocardial cGMP. What drug is it?
Правильный ответ Isosorbide mononitrate
B Dipyridamole
C Panangin
D Validol
E Verapamil
№ krok 2014
Тема Antimicrobial agents
A 24-year-old patient with catarrhal tonsillitis has ben administered
a drug from the group of sulfonamides. Specify the mechanism of
Текст задания sulfonamide antibacterial action:
Правильный ответ Competitive antagonism of PABA
B Disruption of the cell wall protein synthesis
C Reduction of membrane permeability
D Inhibition of sulfhydryl groups of thiol enzymes
E Protein coagulation
№ krok 2014
Тема Drugs acting on the CNS
A 39-year-old patient with arthritis of the temporomandibular joint
has been administered diclofenac sodium. It must be kept in mind
Текст задания that the side effect of prolonged use of this drug is:
Правильный ответ Ulcerogenicity
B Teratogenicity
C Ototoxicity
D Carcinogenicity
E Drug dependence
№ krok 2014
Тема Antimicrobial agents
A female who had been continuously taking antibiotics for an
intestinal infection developed a complication manifested by
inflammation of the oral mucosa and white deposit. Bacteriological
study of the deposit samples revealed yeast fungi Candida albicans.
Which of the following medications is indicated for the treatment
Текст задания of this complication?
Правильный ответ Fluconazole
B Biseptol
C Tetracycline
D Furazolidone
E Polymyxin
№ krok 2014
Тема Antimicrobial agents
For the treatment of periostitis a 35-year-old patient should be
administered an antibiotic with a high ability to penetrate into the
Текст задания bone tissue. Specify this drug:
Правильный ответ Doxycycline hydrochloride
B Streptomycin
C Erythromycin
D Chloramphenicol
E Kanamycin
№ krok 2014, 2013
Тема Drugs acting on the CNS
A patient with a severe maxillofacial trauma has been delivered to
the emergency department. What drug should be given this patient
Текст задания to relieve pain shock?
Правильный ответ Promedol
B Sydnocarb
C Ibuprofen
D Pantogam
E Mydocalm
№ krok 2014
Тема Drugs acting on metabolism, blood and immunity
After using a toothpaste a 27-year-old patient has developed
Quincke’s edema. Administer a drug from the group of histamine
Текст задания H1-receptor antagonists for the treatment of this condition:
Правильный ответ Dimedrol
B Paracetamol
C Digoxin
D Chlorpromazine
E Analgin
№ krok 2014
Тема Agents influencing on afferent and efferent innervations
After a patient had taken a blocking agent, his heart rate (HR)
increased. Pressing on the eyeballs didn’t result in the expected
reflectory decrease in heart rate. What exactly was blocked by the
Текст задания drug in the pacemaker cells?
Правильный ответ M-cholinergic receptors
B α1-adrenergic receptors
C β-adrenergic receptors
D C a2+-L-type channels
E Fast N a+ channels
№ krok 2014
Тема Agents influencing on afferent and efferent innervations
A 36- year-old patient has gastric ulcer (with increased acidity).
Which of the listed below drugs will reduce the secretion of
Текст задания hydrochloric acid with a minimum of side effects?
Правильный ответ Famotidine
B Atropine
C Pirenzepine
D Almagel
E -
№ krok 2014
Тема Drugs acting on metabolism, blood and immunity
Activation of free radical processes is a universal mechanism that
triggers cell death. What inhibitors of this process should be
administered as a part of therapeutic interventions intended for the
Текст задания treatment of generalized periodontitis?
Правильный ответ Tocopherol, ascorbate
B Riboflavin, pyridoxine
C Calciferol, naphthoquinone
D Thiamin, folate
E Cobalamin, pantothenic acid
№ krok 2014
Тема Drugs acting on the CNS
With the purpose of analgesia, a narcotic analgesic has been used
with a benzodiazepine drug. What drug has been used to potentiate
Текст задания analgesia?
Правильный ответ Diazepam
B Chlorprothixene
C Triftazin
D Carbamazepine
E Imizinum
№ krok 2013
Тема Antimicrobial agents
Certain infections caused by bacteria are treated with
sulphanilamides that block the synthesis of bacterial growth factor.
Текст задания What is the mechanism of these drugs action?
Правильный ответ They are antivitamins of para-aminobenzoic acid
B They inhibit the folic acid absorption
C They are allosteric enzyme inhibitors
D They are involved in redox processes
E They are allosteric enzymes
№ krok 2013
Тема Drugs acting on metabolism, blood and immunity

A patient with periodontitis has been administered a glucocorticoid


Текст задания drug in form of an ointment. Specify this ointment:
Правильный ответ Prednisolone
B Tetracycline
C Decamine
D Ampicillin
E Erythromycin
№ krok 2013
Тема Drugs acting on metabolism, blood and immunity
A patient has a systemic inflammatory lesion of connective tissue.
Which antiinflammatory drug will reduce all the inflammatory
Текст задания phases?
Правильный ответ Prednisolone
B Contrycal
C Phenylbutazone
D Indomethacin
E Diclofenac sodium
№ krok 2013
Тема Agents influencing on afferent and efferent innervations
During a surgery, the anesthesiologist used a ganglionic blocker for
controlled hypotension. What drug was given the patient in this
Текст задания case?
Правильный ответ Hygronium
B Benzohexonium
C Pirilenum
D Pentaminum
E Pachycarpinum
№ krok 2013
Agents influencing on afferent and efferent innervations+ Drugs
Тема acting on cardiovascular system
A patient with acute heart failure refractory to cardiac glycosides
was given an injection of dobutamine. What is the mechanism of
Текст задания action of this drug?
Правильный ответ Stimulation of β-1-adrenergic receptors
B Complexation with membrane phospholipids
C Inhibition of K+, Na+- ATPase
D Inhibition of phosphodiesterase activity
E Increase of n.vagus tonus
№ krok 2013
Тема Drugs acting on cardiovascular system + General pharmacology
A patient with chronic heart failure has been taking digoxin for
several months on an outpatient basis. At a certain stage of
treatment, he got symptoms of drug overdose. What effect
Текст задания underlies the development of this complication?
Правильный ответ Material accumulation
B Adaptation
C Sensibilization
D Functional cumulation
E Tachyphylaxis
№ krok 2013
Тема Drugs acting on metabolism, blood and immunity
In the framework of complex treatment of gingivitis a patient has
been administered a drug that stimulates leucopoiesis, accelerates
wound healing, enhances the growth and proliferation of cells, has
the antiinflammatory effect. It is applied for treatment of
leukopenias of different genesis, in the dental practice it is used for
treatment of inflammatory diseases of the oral mucosa. Identify the
Текст задания drug:
Правильный ответ Pentoxylum
B Mercaptopurine
C Methotrexate
D Cyanocobalamin
E Coamide
№ krok 2013
Тема Drugs acting on metabolism, blood and immunity
A patient with rheumatoid arthritis has been given hydrocortisone
for a long time. He has developed hyperglycemia, polyuria,
glycosuria, thirst. These complications of treatment result from the
Текст задания activation of the following process:
Правильный ответ Gluconeogenesis
B Glycogenolysis
C Glycogenesis
D Glycolysis
E Lipolysis
№ krok 2013, 2011
Тема Drugs acting on the CNS
Depressions and emotional disorders result from noradrenaline,
serotonin and other biogenic amines deficiency in brain.
Concentration of these compounds in synapses can be increased by
means of antidepressants that inhibit the activity of the following
Текст задания enzyme:
Правильный ответ Monoamine oxidase
B Diamine oxidase
C L-amino acid oxidase
D D-amino acid oxidase
E Phenylalanine-4-monooxigenase
№ krok 2013
Тема Antimicrobial agents

A patient has been diagnosed with sepsis. It was decided to treat


Текст задания him with a drug from the fluoroquinolone group. Specify this drug:
Правильный ответ Ciprofloxacin
B Cefpirome
C Metronidazole
D Ampicillin
E Cephalexin
№ krok 2013
Тема Agents influencing on afferent and efferent innervations
Curarelike substances (dithylinum) make it impossible for skeletal
Текст задания muscles to contract because they block:
Правильный ответ Neuromuscular synapses
B Central synapses
C Ganglionic synapses
D Membrane conduction of excitement
E Proprioceptors
№ krok 2013

Тема Preperations for pharmacotherapy of different systems pathology


A patient has acute laryngotracheitis with nonproductive cough
Текст задания that is very exhaustive. Prescribe an antitussive drug:
Правильный ответ Glaucine
B Ambroxol
C Mucaltin
D Неrbа Тhermopsidis
E Acetylcystein
№ krok 2013
Тема Drugs acting on the CNS

A patient with a malignant neoplasm on the upper jaw had been


administered morphine hydrochloride for analgesia. The injection
induced respiratory depression, pupil constriction, cyanosis,
Текст задания hypothermia, loss of consciousness. What antidote must be used?
Правильный ответ Naloxone
B Atropine sulfate
C Droperidol
D Promedol
E Adrenalin hydrochloride
№ krok 2013
Тема Antimicrobial agents
A patient with mandibular osteomyelitis has been administered an
Текст задания antibiotic from the tetracycline group. Specify this drug:
Правильный ответ Doxycycline hydrochloride
B Rifampicin
C Streptomycin
D Oxacillin
E Amikacin
№ krok 2013
Тема Antimicrobial agents
A patient has herpetic conjunctivitis. What etiotropic drug should
Текст задания be administered?
Правильный ответ Acyclovir
B Ampicillin
C Methisazonum
D Furagin
E Tetracycline
№ krok 2013
Тема Drugs acting on the CNS
A patient has the pronounced pain syndrome induced by neuralgia.
What drug from the group of nonsteroidal anti-inflammatory drugs
Текст задания will reduce pain sensitivity?
Правильный ответ Diclofenac sodium
B Codeine phosphate
C Ketamine hydrochloride
D Lidocaine hydrochloride
E Droperidol
№ krok 2013
Тема Drugs acting on the CNS
A patient with arthritis and varicose veins has been taking a
nonsteroidal antiinflammatory drug for a long time, which caused
the thrombosis of cutaneous veins. Which of the listed drugs might
Текст задания have caused this complication?
Правильный ответ Celecoxib
B Indomethacin
C Aspirin
D Phenylbutazone
E Ibuprofen
№ krok 2013
Тема Antimicrobial agents
After starting treatment for pulmonary tuberculosis a patient
complained about red tears and urine. What drug could cause such
Текст задания changes?
Правильный ответ Rifampicin
B Benzylpenicillin sodium salt
C Benzylpenicillin potassium salt
D Biseptol-480
E Cefazolin
№ krok 2013
Тема Drugs acting on cardiovascular system
A patient with chronic heart failure had been taking digitoxin for
several months, during digitalization the following symptoms
developed: headache, nausea, diarrhea, loss of appetite, impaired
color vision, bradycardia. What antidote should be administered to
Текст задания reduce the intoxication symptoms?
Правильный ответ Unithiol
B Atropine sulfate
C Prednisolone
D Adrenalin hydrochloride
E Naloxone
№ krok 2013
Тема Antimicrobial agents
Following treatment with a highly-efficient anti-tuberculosis drug a
48-year-old female developed optic nerve neuritis, memory
impairment, cramps. Which of these anti-TB drugs had the patient
Текст задания taken?
Правильный ответ Isoniazid
B PASA
C Rifampicin
D Ethambutol
E Kanamycin sulfate
№ krok 2012, 2007
Тема Drugs acting on metabolism, blood and immunity
Examination of a child who hasn’t got fresh fruit and vegetables
during winter revealed numerous subcutaneous hemorrhages,
gingivitis, carious cavities in teeth. What vitamin combination
Текст задания should be prescribed in this case?
Правильный ответ Ascorbic acid and rutin
B Thiamine and pyridoxine
C Folic acid and cobalamin
D Riboflavin and nicotinamide
E Calciferol and ascorbic acid
№ krok 2012, 2011, 2010
Тема Drugs acting on cardiovascular system
A 65-year-old patient with chronic heart failure has been taking
digitoxin in self-administered dosages for a long time. She was
admitted to the hospital for general health aggravation, arrhythmia,
nausea, reduced diuresis, insomnia. What is the primary action to
Текст задания be taken?
Правильный ответ To withhold digitoxin
B To reduce digitoxin dosage
C To administer strophanthine intravenously
D To administer digoxin
E To give an intravenous injection of calcium gluconate solution
№ krok 2012
Тема Drugs acting on metabolism, blood and immunity
A patient has myocardial infarction with thrombosis of the left
coronary artery. What pharmocological preparation group should
Текст задания be used to reestablish blood flow?
Правильный ответ Fibrinolysis activators
B Narcotic analgesics
C β–adrenergic blockers
D Angiotensin-converting enzyme inhibitors
E Glucocorticoids
№ krok 2012, 2008
Тема Antimicrobial agents
For the purpose of disinfection of nonmetallic surgical instruments
the formaldehyde solution was used. What group does this
anticeptic preparation belong to according to its chemical
Текст задания structure?
Правильный ответ Aliphatics
B Aromatics
C Alcohols
D Halogenated compounds
E Detergents
№ krok 2012, 2011
Тема Agents influencing on afferent and efferent innervations
Before the infiltration anaesthesia a patient had been tested for
sensitivity to novocaine. The reaction turned out to be positive.
Which of the below listed drugs can be used for anaesthetization in
Текст задания this case?
Правильный ответ Lidocaine
B Procainamide hydrochloride
C Trimecaine
D Anesthezin
E Tetracaine
№ krok 2012, 2007
Тема Antimicrobial agents
A stomatologists examined first-grade pupils and revealed that one
of children had yellowish brown teeth, two of them were split.
Heretofore the pupil was treated with "some pills"on account of
pneumonia. What medication could have had such a negative effect
Текст задания upon teeth?
Правильный ответ Doxycycline
B Oxacillin
C Erythromycin
D Ampicillin
E Biseptol
№ krok 2012
Тема Drugs acting on metabolism, blood and immunity
A patient has a slowly healing fracture. What medicine can be used
Текст задания to accelerate formation of connective tissue matrix?
Правильный ответ Methyluracil
B Prednisolone
C Cyclophosphan
D Methotrexate
E Cyclosporine
№ krok 2012
Тема Drugs acting on metabolism, blood and immunity
Leukoses are treated with antimetabolite methotrexate. What
Текст задания vitamin is its antagonist?
Правильный ответ Folic acid
B Cyanocobalamin
C Phyllochinone
D Piridoxine
E Rutin
№ krok 2012, 2011, 2010
Тема Drugs acting on the CNS
A patient consulted a dentist about the temporomandibular joint
arthritis. The dentist administered an ointment containing
Текст задания diclofenac sodium. What is its mechanism of action?
Правильный ответ Cyclooxigenase inhibition
B Phospholipase inhibition
C Opiate receptor activation
D Opiate receptor block
E Cyclooxigenase activation
№ krok 2012, 2011, 2010
Тема Drugs acting on the CNS+ General pharmacology
While under barbituric anaesthesia a 65-year-old male patient
developed respiratory inhibition. Anesthesiologist made him a 10
ml intravenous injection of 0,5% bemegride solution. The patient’s
condition got better, the pulmonary ventilation volume increased.
What phenomenon underlies the interaction of these
Текст задания medivcations?
Правильный ответ Direct antagonism
B Indirect antagonism
C Unilateral antagonism
D Direct synergism
E Indirect synergism
№ krok 2012
Тема Antimicrobial agents
After the second abortion a 23-year-old woman has been
diagnosed with toxoplasmosis. Which drug should be used for
Текст задания toxoplasmosis treatment?
Правильный ответ Co-trimoxazole
B Itraconazole
C Mebendazole
D Azidothimidine
E Acyclovir
№ krok 2012
Тема Antimicrobial agents
After 4 months of treatment for tuberculosis the patient began
complaining of toes and fingers numbness, sensation of creeps. He
was diagnosed with polyneuritis. What antituberculous drug might
Текст задания have caused these complications?
Правильный ответ Isoniazid
B Rifampicin
C Ciprofloxacin
D Sodium salt of benzylpenicillin
E Alcohol iodine solution
№ krok 2012
Тема Agents influencing on afferent and efferent innervations
An elderly patient has chronic constipations induced by large
Текст задания intestine hypotonia. What drug should be administered?
Правильный ответ Bisacodyl
B Sodium sulphate
C Castor oil
D Atropine
E Novocaine amide
№ krok 2012
Тема Drugs acting on the CNS
A patient with epilepsy and depressive reaction has been
administered a drug that reduced epilepsy manifestations and
Текст задания improved the patient’s psychic condition.
Правильный ответ Sodium valproate
B Ethosuxemide
C Amitriptyline
D Phenytoin
E Phenobarbital
№ krok 2012, 2010, 2009
Тема Drugs acting on metabolism, blood and immunity
A patient suffering from non-insulin-dependent diabetes mellitus
was prescribed glibenclamid internally. What is the mechanism of
Текст задания its hypoglycemic action?
Правильный ответ It stimulates generation of endogenous insulin by beta cells
B It inhibits gluconeogenesis in liver
C It intensifies utilization of glucose by peripheral tissues
D It inhibits glucose absorption in the bowels
E It inhibits alpha glucosidase and polysaccharide breakdown
№ krok 2012, 2009
Тема Agents influencing on afferent and efferent innervations
A man got poisoned with mushrooms. They contain muscarine that
stimulates muscarinic cholinoreceptors. What symptom is typical
Текст задания for poisoning with inedible mushrooms?
Правильный ответ Miosis
B Mydriasis
C Bronchi dilation
D Heart rate rise
E Arterial pressure rise
№ krok 2012, 2011
Тема Drugs acting on metabolism, blood and immunity
A 12-year-old child presents with intolerance to some foodstuffs.
Their consumption causes an allergic reaction in form of itching skin
eruption. What antihistaminic drug should be administered that
won’t have any negative impact on the child’s school studies (with
Текст задания no sleepiness effect)?
Правильный ответ Loratadine
B Dimedrol
C Sodium diclofenac
D Aminophylline
E Mesatonum
№ krok 2012
Тема Drugs acting on metabolism, blood and immunity
A 46-year-old female patient needs a surgery in the maxillofacial
region. It is known that the patient is disposed to increased
hemocoagulation. What natural anticoagulant can be used in order
Текст задания to prevent thrombosis?
Правильный ответ Heparin
B Hirudin
C Sodium citrate
D Fibrinolysin
E None of the listed drugs
№ krok 2012, 2009
Тема Antimicrobial agents
A patient consulted a dentist about itching and burning in the oral
cavity; high temperature. The patient was diagnosed with
trichomonal gingivostomatitis. What drug should be chosen for his
Текст задания treatment?
Правильный ответ Metronidazole
B Ampicillin
C Doxycycline hydrochloride
D Gentamicin sulfate
E Nystatin
№ krok 2012, 2010
Тема Antimicrobial agents
A patient suffering form stomatitis was prescribed oral rinsing.
Which antiseptic from the oxidant group is the most suitable for
Текст задания this purpose?
Правильный ответ Potassium permanganate
B Boric acid
C Alcoholic iodine solution
D Ethyl alcohol
E Chloramine
№ krok 2012
Тема Drugs acting on the CNS+ Drugs acting on cardiovascular system
A patient has been diagnosed with transmural myocardial
infarction. What drug should be given in order to prevent
Текст задания cardiogenic shock?
Правильный ответ Promedol
B Reserpin
C Octadine
D Phentolamine
E Analgin
№ krok 2012
Тема Drugs acting on cardiovascular system

A patient with ischemic heart disease has been administered an


antianginal drug that reduces the myocardial oxygen consumption
Текст задания and improves blood supply of myocardium. What drug is it?
Правильный ответ Nitroglycerine
B Validol
C Propranolol
D Promedol
E Retabolil
№ krok 2012
Тема Drugs acting on the CNS
A patient presents with dysfunction of cerebral cortex accompanied
by epileptic seizures. He has been administered a biogenic amine
synthetized from glutamate and responsible for central inhibition.
Текст задания What substance is it?
Правильный ответ Gamma-amino butyric acid
B Serotonin
C Dopamine
D Acetylcholine
E Histamine
№ krok 2012
Agents influencing on afferent and efferent innervations+ Drugs
Тема acting on cardiovascular system
For relief of hypertensive crisis a doctor administered a patient a
drug that apart from antihypertensive effect has also sedative,
spasmolytic and anti-convulsive effect. The drug was taken
parenterally. When it is taken enterally it acts as a laxative and
Текст задания cholagogue. What drug was administered?
Правильный ответ Magnesium sulfate
B Dibasolum
C Reserpine
D No-spa
E Apressin
№ krok 2012
Тема Agents influencing on afferent and efferent innervations
A patient with chronic bronchitis has been administered an
expectorant that disintegrates disulphide bonds of sputum
glycosaminoglycan thus reducing its viscosity. The patient has been
also warned about possible bronchospasm. What drug has been
Текст задания administered?
Правильный ответ Acetylcysteine
B Libxine
C Bromhexine
D Thermopsis herb
E Sodium hydrocarbonate
№ krok 2012
Тема Drugs acting on metabolism, blood and immunity
A patient with diabetes mellitus complicated by angiopathy has
been recommended a drug which is a sulphonyl urease derivate of
the second generation. It improves microcirculation and is known
Текст задания for its relatively good tolerance. What drug is it?
Правильный ответ Glibenclamide
B Glibutidum
C Insulin
D Acarbose
E Adrenalin
№ krok 2012
Тема Agents influencing on afferent and efferent innervations
A patient in grave condition has been deliveredf to the admission
ward. Examination revealed pupil mydriasis, no reaction to the
light, considerable reddening and dryness of skin and mucous
membranes. What drug might have caused the intoxication
Текст задания symptoms?
Правильный ответ Atropine sulphate
B Proserin
C Adrenalin hydrochloride
D Pilocarpine hydrochloride
E Dithylinum
№ krok 2011
Тема Agents influencing on afferent and efferent innervations
Dentists commonly practice local anaesthetization by applying
novocain solution with 0,1% adrenalin solution. The added
Текст задания adrenalin induces:
Правильный ответ Local vasoconstriction
B Local vasodilatation
C Arterial pressure drop
D Decrease in vascular resistance
E Arterial pressure rise
№ krok 2011
Тема Drugs acting on the CNS
Before a tooth extraction a 48-year-old female patient received an
injection of diazepam. Anxiolytic effect of this drug can be
Текст задания explained by:
Правильный ответ Interaction with benzodiazepine receptors
B β-adrenoreceptor block
C M-cholinoreceptor activation
D Dopamine receptor block
E α-adrenoreceptor block
№ krok 2011
Тема Agents influencing on afferent and efferent innervations

In order to reduce salivation before a stomatological procedure a


dentist gave his patient 10 drops of 0,1% solution of atropine
sulfate perorally. 30 minutes later the patient started complaining
of acute pain in the eyeballs, misty vision, headache, palpitation.
Текст задания These symptoms were eliminated by means of the following drug:
Правильный ответ Physostigmine
B Aceclidine
C Cytiton
D Carbacholine
E Phosphacol
№ krok 2011
Тема Drugs acting on metabolism, blood and immunity
A 20-year-old patient complains of morbid thirst and huperdiuresis
(up to 10 l daily). Glucose concentration in blood is normal but it is
absent in urine. The patient has been diagnosed with diabetes
insipidus. What hormonal drug is the most appropriate for
Текст задания management of this disorder?
Правильный ответ Vasopressin
B Cortisol
C Thyroxin
D Oxytocin
E Insulin
№ krok 2011, 2010
Тема Drugs acting on metabolism, blood and immunity
A patient diagnosed with acute pancreatitis was admitted to the
surgical department. Which drug administration would be
Текст задания pathogenetically grounded?
Правильный ответ Contrical
B Tripsin
C Chymotripsin
D Pancreatin
E Fibrinolysin
№ krok 2011
Тема Drugs acting on cardiovascular system
A patient with essential hypertension has been prescribed
Текст задания captopril. What is its mechanism of action?
Правильный ответ Inhibition of angiotensin-converting enzyme activity
B β-adrenoreceptor block
C α-adrenoreceptor block
D Angiotensin II receptor block
E Peripheral vasodilatating effect
№ krok 2011
Тема Agents influencing on afferent and efferent innervations
A patient with bronchial asthma has been administered inhalations
of 0,5% isadrin solution. This helped to relieve bronchiospasms but
the patient started complaining of heart pain and palpitation. What
Текст задания is the cause of these presentations?
Правильный ответ β1-adrenoreceptor stimulation
B β2-adrenoreceptor stimulation
C α-adrenoreceptor stimulation
D M-cholinoreceptor activation
E Inhibition of acetylcholine synthesis
№ krok 2011
Тема Antimicrobial agents
During examination of first-grade pupils a dentist revealed that one
child had brown-yellow teeth and two split teeth. According to the
child, previously he had been treated for pneumonia with
"some"pills. What drug might have had such a negative impact on
Текст задания teeth?
Правильный ответ Doxycycline
B Oxacillin
C Erythromycin
D Ampicillin
E Biseptol
№ krok 2011
Тема Drugs acting on metabolism, blood and immunity
A nurse accidentally injected a nearly double dose of insulin to a
patient with diabetes mellitus. The patient lapsed into a
hypoglycemic coma. What drug should be injected in order to help
Текст задания him out of coma?
Правильный ответ Glucose
B Lidase
C Insulin
D Somatotropin
E Noradrenaline
№ krok 2011
Тема Antimicrobial agents
A patient who has been taking tetracycline for a long time has
developed candidosis of mucous membranes. What drug shoul
Текст задания administered for its treatment?
Правильный ответ Itraconazole
B Griseofulvin
C Nitrofungin
D Amphotericin
E Nitrofurantoin
№ krok 2011
Тема Drugs acting on the CNS
A male patient waiting for tooth extraction has developed a strong
sense of anxiety. Which drug should be given to him in order to
Текст задания relieve him of this discomfort?
Правильный ответ Diazepam
B Aminazine
C Analgin
D Aethimizolum
E Carbamazepine
№ krok 2011
Тема Agents influencing on afferent and efferent innervations
During a visit to a dentist a patient has developed collapse. What
Текст задания drug can be applied to manage this situation?
Правильный ответ Mesaton
B Strophanthine
C Propanolol
D Nitroglycerine
E Seduxen
№ krok 2011
Тема Antimicrobial agents
A patient with herpetic stomatitis was prescribed acyclovir for
Текст задания TOPICal application. What is its mechanism of action?
Правильный ответ It inhibits synthesis of nucleic acids of viruses
B It inhibits virus penetration into cells
C It inhibits virus maturation
D It increases the resistance of macroorganism cells to the viruses
E It inhibits virion assembly
№ krok 2011
Тема Antimicrobial agents

An infectious patient manifests sensibilization to penicillin. Which


Текст задания of the following antibiotics is the safest to be applied in this case?
Правильный ответ Erythromycin
B Bicillin
C Ampicillin
D Amoxicillin
E Oxacillin
№ krok 2011
Тема Drugs acting on cardiovascular system
Among the specific hypocholesterolemic drugs the most effective
are those blocking the synthesis of endogenic cholesterol in liver.
Which of the below listed drugs has such mechanism of
Текст задания hypocholesterolemic action?
Правильный ответ Lovastatin
B Probucol
C Clofibrate
D Linaethol
E Allilcepum
№ krok 2011, 2010
Тема Drugs acting on cardiovascular system
A 60-year-old patient consulted a doctor about retrosternal pain
arising immediately after physical exercise. He was prescribed
nitroglycerin. The medication relieved retrosternal pain but the
patient got acute headache. What is the likely mechanism of this
Текст задания side effect?
Правильный ответ Intracranial pressure rise
B α-adrenoreceptor block
C Phosphodiesterase block
D Reduced accumulation of calcium ions
E Inhibited formation of mediators in brain
№ krok 2011
Тема Drugs acting on the CNS+ General pharmacology

A patient diagnosed with morphinism has been admitted to the


narcological department. A doctor noted a decrease in
pharmacological activity of morphine. Repetitive use of a drug may
Текст задания result in tolerance to its effect, and this phenomenon is called:
Правильный ответ Addiction
B Cumulation
C Tachyphylaxis
D Antagonism
E Allergy
№ krok 2011
Тема Agents influencing on afferent and efferent innervations
A patient has a spasm of smoot muscles of bronchi. As the first aid
it would be physiologically appropriate to inject the patient the
Текст задания antagonists of the following receptors:
Правильный ответ M -cholinoreceptors
B alpha-adrenoreceptors
C N -cholinoreceptors
D beta-adrenoreceptors
E Adenosine receptors
№ krok 2011
Тема Agents influencing on afferent and efferent innervations
On the 2-3 day after stomach resection a patient is still
experiencing a failure of intestinal peristalsis. In order to stimulate
the motility of gastrointestinal tract the following drug should be
Текст задания administered:
Правильный ответ Proserin
B Prazosin
C Cyclodolum
D Atropine sulphate
E Noradrenaline hydrotartrate
№ krok 2010
Тема Agents influencing on afferent and efferent innervations
A patient who takes a blocker of membrane cytoreceptors of
efferent conductor synapses of autonomic nervous system
Текст задания complains about dry mouth. What receptors are blocked?
Правильный ответ Muscarinic cholinoreceptors
B Nicotinic cholinoreceptors
C H2-receptors
D α-adrenoreceptors
E β-adrenoreceptors
№ krok 2010
Тема Drugs acting on metabolism, blood and immunity
A patient has increased pyruvate concentration in blood. Large
amount of it is excreted with urine. What vitamin deficiency is
Текст задания observed?
Правильный ответ B1
B E
C B3
D B6
E B2
№ krok 2010, 2008
Тема Agents influencing on afferent and efferent innervations
Introduction of a local anesthetic to a patient resulted in the
development of anaphylactic shock. What is the leading mechanism
Текст задания of blood circulation disturbance?
Правильный ответ Decrease of vascular tone
B Hypervolemia
C Pain
D Activation of sympathoadrenal system
E Reduction of contractile myocardium function
№ krok 2010
Тема Agents influencing on afferent and efferent innervations

In a surgical department of a stomatological polyclinic a patient is


being prepared for tooth extraction. What drug should be added to
Текст задания the solution of a local anaesthetic in order to prolong its action?
Правильный ответ Adrenalin hydrochloride
B Noradrenaline hydrotartrate
C Isadrine
D Salbutamol
E Octadine
№ krok 2010
Тема Drugs acting on metabolism, blood and immunity
A patient complains about shin pain which is getting worse during
walking. Objectively: there is an edema and reddening along the
vein. A doctor administered a direct coagulant to be applied
Текст задания TOPICally. What drug can be applied for this purpose?
Правильный ответ Heparin ointment
B Salicylic ointment
C Troxevasin ointment
D Butadion ointment
E Thrombin
№ krok 2010
Тема Drugs acting on metabolism, blood and immunity
A patient is found to have increased permeability of blood vessel
walls accompanied by increased gingival haemorrhage, petechial
skin haemorrhages, dedentition. What pathology is observed in this
Текст задания patient?
Правильный ответ Hypovitaminosis C
B Hypervitaminosis D
C Hypervitaminosis C
D Hypovitaminosis D
E Hypovitaminosis A
№ krok 2010
Тема Agents influencing on afferent and efferent innervations
While being at the dentist’s a patient had an attack of bronchial
asthma. The dentist applied a β-adrenomimetic drug in form of
Текст задания inhalations. What drug was applied?
Правильный ответ Salbutamol
B Aminophylline
C Adrenaline hydrochloride
D Atropine sulfate
E Ephedrine hydrochloride
№ krok 2010
Тема Drugs acting on the CNS
A female patient suffering from chronic hepatitis complains about
an increased susceptibility to barbiturates, which previously
induced no syptoms of intoxication. This may be explained through
Текст задания the folllowing liver dysfunction:
Правильный ответ Metabolic
B Cholepoietic
C Hemodynamic
D Hemopoietic
E Phagocytal
№ krok 2010
Тема Drugs acting on the CNS
A patient with toxic paralysis of respiratory centre was given
several cordiamin injections intended to stimulate the respiratory
Текст задания centre. What side effect may arise?
Правильный ответ Clonic convulsions
B Tonic convulsions
C Arrhythmia
D Collapse
E Bronchiospasm
№ krok 2010
Тема Drugs acting on the CNS
After parenteral introduction of a medication a patient fell into a
coma. He had Cheyne-Stokes respiration, apparently miotic pupils.
The patellar reflex was preserved. What medication might have
Текст задания caused the intoxication?
Правильный ответ Morphine hydrochloride
B Aminazine
C Diazepam
D Analgine
E Phenobarbital
№ krok 2010
Тема Agents influencing on afferent and efferent innervations
A patient has applied eye drops containing atropine which resulted
Текст задания in persistent mydriasis. Which muscle was blocked?
Правильный ответ Pupil-contracting
B Pupil-dilating
C Ciliate
D Rectus
E Oblique
№ krok 2010
Тема Drugs acting on cardiovascular system
A patient has coronary heart disease. For its treatment he was
prescribed an antianginal drug that activates guanylate cyclase and
accumulates cyclic guanosine monophosphate in the miocardium
Текст задания cells. What drug is it?
Правильный ответ Isosorbide mononitrate
B Dipiridamol
C Panangine
D Validol
E Verapamil
№ krok 2010
Тема Drugs acting on the CNS
A patient was admitted to a surgical department for an operation.
He has to undergo neuroleptanalgesia. To achieve
neuroleptanalgesia it would be rational to combine fentanyl with
Текст задания the following medicine:
Правильный ответ Droperidol
B Cholosasum
C Salbutamol
D Pilocarpine
E Fraxiparine
№ krok 2010
Тема Drugs acting on the CNS
To prevent possible negative effect upon the gastric mucsa a
patient with rheumatoid arthritis was administered a nonsteroid
antiinflammatory drug - a COX-2 selective inhibitor. Specify this
Текст задания drug:
Правильный ответ Celecoxib
B Analgine
C Acetylsalicinic acid
D Butadion
E Ibuprofen
№ krok 2010, 2009
Тема Antimicrobial agents
A patient suffering from syphilis was prescribed a drug the action of
which based upon disturbed generation of murein leading to death
Текст задания of the causative agent. What drug is it?
Правильный ответ Benzylpenicillin sodium salt
B Bijochinol
C Ciprofloxacin
D Azithromycin
E Doxycycline hydrochloride
№ krok 2010, 2009
Тема General pharmacology
A patient who has been taking a certain drug for a long time cannot
discontinue the use of it because this causes psychic and somatic
disfunctions. The syndrome occuring at refraining from the use of a
Текст задания drug is called:
Правильный ответ Abstinence
B Sensitization
C Idiosyncrasy
D Tachyphylaxis
E Cumulation
№ krok 2010, 2008
Тема Drugs acting on cardiovascular system
A patient in a cardiological department has arrhythmia. A doctor
administered him amyodaron. What is the main mechanism of
Текст задания amyodaron’s antiarrhythmic action?
Правильный ответ It blocks mostly potassium channels
B It inhibits cholinoreceptors
C It stimulates histamine receptors
D It activates serotonin receptors
E It alters myocardium susceptibility to the acetylcholine
№ krok 2010
Тема Drugs acting on cardiovascular system
A patient with edemata was prescribed a K+-retaining diuretic -
Текст задания aldosterone antagonist. What drug is it?
Правильный ответ Spironolactone
B Digoxin
C Procainamide hydrochloride
D Clonidine
E Alopurinole
№ krok 2010
Тема Antimicrobial agents
A patient consulted a dentis about a lesion of his oral mucosa. He
was diagnosed with herpetic stomatitis. Which of the following
Текст задания drugs will have an effect on etiotropic factor?
Правильный ответ Acyclovir
B Dimedrol
C Paracetamol
D Levamisole
E Furacilinum
№ krok 2010
Тема Antimicrobial agents
A patient suffering form tuberculosis was treated with rifampicin,
which caused drug resistance of tuberculosis mycobacteria. In order
to reduce mycobacteria resistance, rifampicin should be combined
Текст задания with the following drug:
Правильный ответ Isoniazid
B Acyclovir
C Intraconazole
D Metronidazole
E Amoxicillin
№ krok 2009
Тема Agents influencing on afferent and efferent innervations
A patient who takes blocker of membrane cytoreceptors of efferent
conductor synapses of autonomic nervous system complains about
Текст задания dry mouth. What receptors are blocked?
Правильный ответ Muscarinic cholinoreceptors
B Nicotinic cholinoreceptors
C H2-receptors
D α-adrenoreceptors
E β-adrenoreceptors
№ krok 2009
Тема Drugs acting on metabolism, blood and immunity
Parodontosis is treated by means of antioxidants. Which of the
Текст задания following natural compounds is used as an antioxidant:
Правильный ответ Tocopherol
B Thiamine
C Gluconate
D Pyridoxine
E Choline
№ krok 2009
Тема Agents influencing on afferent and efferent innervations
A patient had an attack of bronchial asthma in the dentist’s office.
The attack was arrested by salbutamol. This drug relates to the
Текст задания following group of therapeutic agents:
Правильный ответ β2-adrenomimetics
B α-adrenomimetics
C β1-β2-adrenomimetics
D Sympatholytics
E α-β-adrenomimetics
№ krok 2009
Тема Agents influencing on afferent and efferent innervations
A patient with acute duodenal ulcer was admitted to the hospital.
Analysis of gastric juice revealed hyperfunction of secretion and
acid-forming in stomach. Choose a drug that can reduce secretory
Текст задания function of stomach due to inhibition of H2-receptors:
Правильный ответ Ranitidine
B Extract of dry belladonna
C Atropine
D Methacin
E Platyphyllin
№ krok 2009
Agents influencing on afferent and efferent innervations+ Drugs
Тема acting on cardiovascular system
A patient was administered clonidine to be taken parenterally in
case of abrupt rise of arterial pressure. What is its mechanism of
Текст задания action?
Правильный ответ Stimulation of central α2-adrenoreceptors
B Block of nicotinic cholinoreceptors of ganglia
C Block of α1- and α2-adrenoreceptors
D Block of α1-adrenoreceptors
E Stimulation of central imidazole1-receptors
№ krok 2009, 2008
Тема General pharmacology
A patient was prescribed a drug with apparent lipophilic properties.
Текст задания What is the main mechanism of its absorption?
Правильный ответ Passive diffusion
B Filtration
C Active transport
D Pinocytosis
E Binding with transport proteins
№ krok 2009
Тема General pharmacology
A patient noticed symptoms of approaching attack of bronchial
asthma and took several tablets one by one at short intervals out of
the doctor’s control. Short-term improvement of his condition
came only after taking the first two tablets. Next intakes of a drug
didn’t improve his condition. Reduction of the drug effectiveness
Текст задания was caused by:
Правильный ответ Tachyphylaxis
B Cumulation
C Addiction
D Dependence
E Idiosyncrasy
№ krok 2009
Тема Antimicrobial agents
A patient with streptococcal infection of gums was prescribed a
drug that contained beta-lactam ring in its structure. Which drug
Текст задания relates to this group?
Правильный ответ Benzylpenicillin
B Rifampicin
C Erythromycin
D Streptomycin sulfate
E Chloramphenicol
№ krok 2009
Тема General pharmacology
It was necessary to determine absolute gustation thresholds of a
healthy man for different substances. The lowest threshold will be
Текст задания observed for the following substance:
Правильный ответ Quinine
B Sodium chloride
C Glucose
D Saccharose
E Citric acid
№ krok 2009
Тема Agents influencing on afferent and efferent innervations
A female patient suffering from acute bronchitis complains about
respiratory obstruction and cough with thick viscous sputum. She
was prescribed a mucolytic agent that stimulates surfactant
Текст задания synthesis. What mucolytic agent was prescribed?
Правильный ответ Ambroxolum
B Sodium hydrocarbonate
C Morphine hydrochloride
D Glaucin
E Theophylline
№ krok 2009
Тема Antimicrobial agents
A patient was admitted to a hospital with poisoning with unsound
food. His stomach was lavaged with solution of potassium
Текст задания permanganate. What is its mechanism of action?
Правильный ответ Release of atomic oxygen
B Release of chlorine
C Release of iodine
D Disturbance of synthesis of respiratory chain enzymes
E Destruction of bacteria membranes
№ krok 2009
Тема Drugs acting on cardiovascular system
A 70 year old patient suffering from cardiac insufficiency has been
uncontrolledly taking digoxin that resulted in extrasystole,
vomiting, vision impairment, anxiety, sleep disturbance, reduction
of diuresis. Application of drugs relating to the following group will
Текст задания be pathogenetically reasonable in this case:
Правильный ответ Donators of sulfhydric groups
B Stimulants of β1-adrenoreceptors
C Angiotensin II receptor blockers
D Potassium channel blockers
E Donators of nitrogen oxide
№ krok 2009
Тема Drugs acting on metabolism, blood and immunity
In order to speed up healing of the thermal injury it is required to
prescribe a drug that facilitates epithelization of skin and mucous
Текст задания membranes. What drug is it?
Правильный ответ Retinol acetate
B Tocopherol acetate
C Nicotinic acid
D Ergocalciferol
E Ascorbic acid
№ krok 2009
Тема Drugs acting on cardiovascular system
A patient with ventricular arrhythmia was admitted to the
Текст задания cardiological department. What drug should be administered?
Правильный ответ Amiodarone
B Amlodipine
C Drotaverine
D Aminazine
E Proserin
№ krok 2009
Drugs acting on cardiovascular system+ Agents influencing on
Тема afferent and efferent innervations
A patient with essential hypertension was admitted to the
cardiological department. In order to lower arterial pressure a
doctor prescribed a drug that blocks β1 and β2-adrenoreceptors.
Текст задания What drug is it?
Правильный ответ Propranolol
B Proserin
C Celecoxib
D Prednisolone
E Indometacin
№ krok 2009
Тема Drugs acting on the CNS
A patient with myocardium infarction was delivered to the
resuscitation department. What drug should be injected to the
Текст задания patient for prophylaxis of pain shock?
Правильный ответ Promedol
B Analgin
C Paracetamol
D Celecoxib
E Naloxone
№ krok 2009
Тема Drugs acting on metabolism, blood and immunity
A patient with myocardium infarction was admitted to the
resuscitation department. What drug should beinjected to the
Текст задания patient in order to prevent thrombosis?
Правильный ответ Heparin
B Chingamin
C Thyroxine
D Biseptol-480
E Dimedrol
№ krok 2009
Тема Drugs acting on cardiovascular system
A patient suffers from chronic left-ventricular insufficiency. What
Текст задания drug should be prescribed?
Правильный ответ Digoxin
B Bemegride
C Etimizol
D Vinpocetine
E Pyracetam
№ krok 2009, 2008
Тема Agents influencing on afferent and efferent innervations

A patient suffering from chronic bronchitis takes a synthetic


Текст задания mucolytic drug that stimulates sputum liquefaction. What drug is it?
Правильный ответ Acetylcysteine
B Diazepam
C Heparin
D Furosemide
E Enalapril
№ krok 2009
Тема Drugs acting on metabolism, blood and immunity
A patient has allergic rhinitis with profuse mucous discharges,
itching, frequent sneezing. What drug should be chosen if you know
Текст задания that it selectively blocks histamine receptors?
Правильный ответ Loratadine
B Mesatonum
C Adrenaline hydrochloride
D Naphthizin
E Prednisolone
№ krok 2009
Тема Drugs acting on the CNS
A patient with myocardium infarction was admitted to the
cardiological department. In order to relieve his pain it was decided
to potentiate action of fentanyl by a certain neuroleptic. What is
Текст задания the most suitable neuroleptic for neuroleptanalgesia?
Правильный ответ Droperidol
B Aminazine
C Triftazine
D Haloperidol
E Sulpiride
№ krok 2009
Тема Agents influencing on afferent and efferent innervations
A patient was delivered to the admission ward with poisoning with
an insecticide of anticholinesterase action. What drug able to block
Текст задания muscarinic cholinoreceptors should be prescribed?
Правильный ответ Atropine sulfate
B Pilocarpine hydrochloride
C Dithylinum
D Benzohexonium
E Mesatonum
№ krok 2008
Тема Agents influencing on afferent and efferent innervations
Dentists widely apply local anaesthesia adding adrenalin to an
Текст задания anaesthetic solution. What is the purpose of this method?
Правильный ответ Local vasoconstriction
B Local vasodilatation
C Lowering of arterial pressure
D Local reduction of vascular resistance
E Microcirculation improvement
№ krok 2008
Тема Agents influencing on afferent and efferent innervations
A patient with accute attack of duodenal ulcer was admitted to a
hospital. Analysis of his gastric juice revealed intensification of
secretory and acid-forming stomach functions. Choose a drug that
will reduce secretory stomach function due to blockade of H2-
Текст задания receptors:
Правильный ответ Ranitidine
B Belladonna extract
C Atropine
D Methacin
E Platyphyllin
№ krok 2008
Тема Agents influencing on afferent and efferent innervations
After a surgical procedure a patient felt ill with enteroparesis. What
medication from the group of anticholinesterase drugs should be
Текст задания prescribed?
Правильный ответ Proserin
B Carbacholine
C Aceclydine
D Pilocarpine
E Acetylcholine
№ krok 2008
Тема Drugs acting on metabolism, blood and immunity
For treatment of skin diseases with apparent inflammation in the
maxillofacial area the TOPICal glucocorticoids are applied. What
Текст задания preparation has the minimal resorptive effect?
Правильный ответ Flumethasoni pivalas
B Prednisolone
C Hydrocortisone
D Triamcinolone
E Dexamethasone
№ krok 2008
Тема Drugs acting on the CNS
A patient who attempted suicide in a state of serious depression
was delivered to a hospital by an ambulance. What drugs should be
Текст задания administered?
Правильный ответ Antidepressants
B Sedative
C Neuroleptics
D Tranquillizers
E Lithium salts
№ krok 2008
Тема Drugs acting on cardiovascular system
A patient has acute cardiac insufficiency resulting from essential
Текст задания hypertension. What drug is the most appropriate in this case?
Правильный ответ Corglycon
B Digoxin
C Cardiovalene
D Caffeine
E Cordiamin
№ krok 2008
Drugs acting on cardiovascular system+ Agents influencing on
Тема afferent and efferent innervations
A patient suffers from bradyarrhythmia caused by hypertension.
Текст задания What drug should be administered?
Правильный ответ Platyphyllin hydrotartate
B Clonidine
C Papaverine hydrochloride
D Reserpine
E Methyldopa
№ krok 2008
Тема Drugs acting on metabolism, blood and immunity
A student applied to a doctor and asked him to prescribe a drug for
treatment of allergic rhinitis he fell ill with during linden flowering.
Текст задания What drug can be applied?
Правильный ответ Loratadine
B Noradrenaline hydrotartate
C Propranolol
D Ambroxol
E Losartan
№ krok 2008
Тема Antimicrobial agents
For the preparation of the burned skin surface of a patient a certain
medication was applied. Its antiseptic properties are provided by
free oxygen released in presence of organic substances. What
Текст задания medication is it?
Правильный ответ Potassium permanganate
B Furacillin
C Chlorhexidine
D Alcoholic iodine solution
E Sodium hydrocarbonate
№ krok 2008
Тема Antimicrobial agents
A 30 y.o. patient is diagnosed with amebic dysentery. This diagnosis
was bacteriologically confirmed. Name the preparation for its
Текст задания treatment:
Правильный ответ Metronidazole
B Mebendazole
C Itrakonazole
D Furacillin
E Acyclovir
№ krok 2008
Тема Drugs acting on cardiovascular system
Being at a dentist a patient had an attack of stenocardia. What drug
Текст задания from the nitrate group should be applied in this case?
Правильный ответ Nitroglycerine
B Menthol
C Talinolole
D Erinit
E Validol
№ krok 2008
Тема Drugs acting on the CNS
A patient who has been traeted in a neurological clinic with
sedatives for a long time has the following complications: cough,
rhinitis, lacrimation. What preparation might have caused such
Текст задания disorders?
Правильный ответ Sodium bromide
B Diazepam
C Valerian
D Phenazepam
E Reserpine
№ krok 2008
Тема Drugs acting on the CNS
A patient with acute poisoning with morphine was delivered to the
hospital ward. What specific antagonist of narcotic analgesics is to
Текст задания be applied in this case?
Правильный ответ Naloxone
B Paracetamol
C Methacin
D Digoxin
E Unithiol
№ krok 2008
Тема Drugs acting on cardiovascular system
A patient with edemata was prescribed a +-retaining diuretic -
Текст задания aldosterone antagonist. What drug is it?
Правильный ответ Spironolactone
B Digoxin
C Procainamide hydrochloride
D Clonidine
E Alopurinole
№ krok 2008
Тема Drugs acting on the CNS
To subdue the fever and relieve tooth ache a patient was
prescribed paracetamol. What is the action mechanism of this
Текст задания medication?
Правильный ответ Cycloxygenase blocking
B Monoamine oxidase blocking
C Lipoxygenase blocking
D Cholinesterase blocking
E Phosphodiesterase blocking
№ krok 2008
Тема Agents influencing on afferent and efferent innervations
A dentist was examining a patient and noticed excessive salivation.
The dentist applied a medication inducing dryness of oral cavity.
Текст задания What medication is it?
Правильный ответ Atropine sulfate
B Phentolamine
C Pilocarpine hydrochloride
D Proserin
E Galantamine
№ krok 2008
Тема Antimicrobial agents
A patient is ill with herpetic stomatitis provoked by
immunosuppression. What preparation introduced intravenously,
internally and locally can provide antiviral and immunopotentiating
Текст задания effect?
Правильный ответ Acyclovir
B Remantadinum
C Levamisole
D Methisazonum
E Amoxicillin
№ krok 2007
Тема Drugs acting on cardiovascular system
A woman with ischemic heard disease has been taking an
antianginal medication that has the following properties: dilates
coronary arteries, peripheral vessels (arterial and venous), reduces
the need of myocardium in oxygen, improves endocardial blood
Текст задания circulation. Name this preparation:
Правильный ответ Nitroglycerin
B Validol
C Papaverine
D Dibasol
E Aminophylline
№ krok 2007
Тема Agents influencing on afferent and efferent innervations
A 25 year old patient had in the dentist’s room a sudden attack of
bronchial asthma. The doctor gave him salbutamol in the form of
Текст задания inhalation. What is the mechanism of action of this preparation?
Правильный ответ Stimulates β2-adrenoreceptors
B Stimulates α-adrenoreceptors
C Blocks 1-histamine receptors
D Blocks phosphodiesterase
E Blocks М-cholinergic receptors
№ krok 2007
Тема Agents influencing on afferent and efferent innervations
During the ether narcosis a patient had evident bradycardia with
threat of cardiac arrest. What medication should be used to
accelerate heartbeat under condition of narcosis that shouldn’t be
Текст задания interrupted?
Правильный ответ Atropine
B Caffeine
C Adrenaline
D Camphor
E Isadrine
№ krok 2007
Тема Antimicrobial agents
A patient with stomatitis was prescribed a medication of
sulfanamide group. What is the mechanism of its antibacterial
Текст задания effect?
Правильный ответ Competitive antagonism with para-aminobenzoic acid
B Synthesis disturbance of cell membrane proteins
C Decrease of membrane permeability
D Inhibition of sulfhydric groups of thiol enzymes
E Protein coagulation
№ krok 2007
Тема Drugs acting on the CNS
An unconscious patient was admitted to the hospital. Objectively:
cold skin, miotic pupils, heavy breathing, Chaine-Stokes’ periodicity,
low arterial pressure, overfull urinary bladder. What caused the
Текст задания poisoning?
Правильный ответ Narcotic analgetics
B Tranquilizers
C Nonnarcotic analgetics
D Muscarinic receptor blockers
E -
№ krok 2007
Тема Antimicrobial agents
A 30 year old patient consulted a doctor about having diarrhea and
stomach aches for 5 days, temperature rise up to, 50 with chills.
The day before the patient was in a forest and drank some water
from an open pond. He was diagnosed with amebic dysentery that
was bacteriologically confirmed. Name the medication for
Текст задания treatment of this disease:
Правильный ответ Metronidazole
B Furasolidone
C Chloramphenicol
D Phthalazole
E Emethine hydrochloride
№ krok 2007
Тема Antimicrobial agents
A surgeon cleansed his hands with 70% solution of ethyl alcochol
before an operation. What is the main mechanism of preparation’s
Текст задания antiseptic effect upon microorganisms?
Правильный ответ Dehydratation of protoplasm proteins
B Blockade of sulfhydric groups of enzyme systems
C Oxidation of organic protoplasm components
D Interaction with aminogroups of protoplasm proteins
E Interaction with hydroxyl enzyme groups
№ krok 2007
Тема Antimicrobial agents
Formaldehyde solution was applied for desinfection of nonmetallic
instruments of surgical department. What chemical series does this
Текст задания antiseptic preparation belong to?
Правильный ответ Aliphatic series
B Aromatic series
C Alcochols
D Halogenated compaunds
E Detergents
№ krok 2007
Тема Agents influencing on afferent and efferent innervations

A patient with acute condition of duodenal ulcer was admitted to


the hospital. Gastric juice analysis has shown increase of secretory
and acid-producing function of stomach. Choose a medication that
Текст задания will reduce secretory function due to blockade of H2-receptors:
Правильный ответ Ranitidine
B Beladonna bell extraction
C Atropine
D Methacin
E Platyphyllin
№ krok 2007
Agents influencing on afferent and efferent innervations+ Drugs
Тема acting on cardiovascular system
Name the drug group that can reduce need of myocardium for
Текст задания oxygene, decrease force of heartbeat and inhibit lipolysis:
Правильный ответ β-adrenoceptor blockers
B α-adrenoceptor blockers
C Sympatholytics
D Selective β-adrenoceptor agonists
E α-adrenoceptor agonists
№ krok 2007
Тема Drugs acting on metabolism, blood and immunity
A patient with thyreotoxicosis was prescribed a medication that
inhibits enzyme systems taking part in synthesis of thyroid gland
Текст задания hormones. What medication is it?
Правильный ответ Mercazolile
B Diiodotyrosine
C Potassium iodide
D Thyreoidine
E Radioactive iodine
№ krok 2007
Тема Antimicrobial agents
A patient with tuberculosis was prescribed a certain medication as a
part of complex therapy - derivative of isonicotinic acid hydrazide.
Текст задания Name this preparation:
Правильный ответ Isoniazide
B Streptomycin sulfate
C Cephaloridine
D Rifampicin
E Kanamycin
№ krok 2007
Тема Agents influencing on afferent and efferent innervations

After a surgical procedure a patient has got an enteroparesis. What


Текст задания anticholinesterase medication should be prescribed in this case?
Правильный ответ Proserin
B Carbacholine
C Aceclidine
D Pilocarpine
E Acetylcholine
№ krok 2007
Тема Drugs acting on metabolism, blood and immunity
A patient has a slowly healing fracture. What medcine can be used
Текст задания to accelerate formation of connective tissue matrix?
Правильный ответ Methyluracil
B Prednisolone
C Cyclophosphan
D Methotrexate
E Cyclosporine
№ krok 2007
Тема Agents influencing on afferent and efferent innervations
Removal of a foreign body from patient’s eye involves local
anesthesia with lidocaine. What is the action mechanism of this
Текст задания medication?
Правильный ответ It disturbs passing of Na+ through the membrane
B It blocks passing of nitric oxide
C It inhibits cytochrome oxidase activity
D It reduces dehydrogenase activity
E It reduces passage of neuromediators
№ krok 2007
Тема Drugs acting on the CNS
A patient with myocardium infarction was prescribed an analgetic
in order to stop pain syndrome. The patient felt better but
overdose caused weakness, myosis, respiratory depression. What
Текст задания medication was prescribed?
Правильный ответ Morphine
B Baralgine
C Sedalgine
D Ibuprofen
E Paracetamol
№ krok 2007
Тема Antimicrobial agents
A patient complains of painful cracks in mouth angles. What
sulfanilamide can be recommended for local treatment of angular
Текст задания stomatitis?
Правильный ответ Streptocide liniment
B Synthomycine liniment
C Prednisolone ointment
D Butadione ointment
E Tetracycline ointment
№ krok 2007
Тема Antimicrobial agents
What antihelmintic medication is used for stimulation of immune
Текст задания system in case of chronic generalized periodontitis?
Правильный ответ Levamisole
B Piperidine adipinate
C Pumpkin seeds
D Pyranthel
E Chloxil
№ krok 2007
Тема Drugs acting on cardiovascular system
A patient has acute cardiac insufficiency resulting from essential
hypertension. What medication will be the most appropriate in this
Текст задания case?
Правильный ответ Corglycone
B Digoxin
C Cardiovalen
D Caffeine
E Cordiamin
№ krok 2007
General pharmacology+ Agents influencing on afferent and efferent
Тема innervations

A patient was operated on account of abdominal injury with


application of tubocurarin. At the end of operation, after the
respiration had been restored, the patient got injection of
gentamicin. It caused a sudden respiratory standstill and relaxation
Текст задания of skeletal muscles. What effect underlies this phenomenon?
Правильный ответ Potentiation
B Cumulation
C Antagonism
D Habituation
E Sensitization
№ krok 2007
Тема Antimicrobial agents
A patient with streptococcal gingival infection was prescribed a
medication that contains beta lactam ring in its structure. What
Текст задания preparation belongs to this group?
Правильный ответ Benzylpenicillin
B Rifampicin
C Erythromycin
D Streptomicin sulfate
E Chloramphenicol
№ krok 2019
Тема Drugs acting on cardiovascular system
Текст задания The patient is in the state of cardiogenic shock, he needs to be
given a non-glycoside cardiotonic drug. What will be the drug
of choise in this case?
Правильный ответ Dobutamine
B Amrinone
C Caffeine
D Cordiamin (Nikethamide)
E Ethimizol
№ krok 2019
Тема Drugs acting on metabolism, blood and immunity
Текст задания A 30-year-old patient after a case of viral hepatitis type B has
developed complaints of continuous nasal hemorrhages. What
drug would by the most advisable for treatment of this
condition?
Правильный ответ Menadione (Vicasolum)
B Asparcam
C Folic acid
D Nadroparin calcium (Fraxiparine)
E Dipiridamol
№ krok 2019
Тема Antimicrobial agents
Текст задания A patient with streptococcal pneumonia was prescribed an
antimicrobial agent that disrupts microbial membranes. Name
this drug
Правильный ответ Benzylpenicillin sodium salt
B Gentamicin sulfate
C Erythromycin
D Azithromicin
E Doxycycline hydrochloride
№ krok 2019
Тема Drugs acting on cardiovascular system
Текст задания After a tooth extraction, the patient developed acute heart
failure. What drug should be prescribed in this case?
Правильный ответ Strophantin
B Convallaria majalis tincture
C Cordigitum
D Digitoxin
E Adonisid
№ krok 2019
Тема Drugs acting on the CNS
Текст задания A patient with signs of anxiety, fear, uncertainty, and mental
strain was prescribed diazepam. What mechanism of
tranquilizing action can be observed in this case?
Правильный ответ Interaction with benzodiazepine receptors
B Interaction with cholinergic receptors
C Interaction with adrenergic receptors
D Interaction with dopamine receptors
E Interaction with serotonin receptors
№ krok 2019
Тема Drugs acting on the CNS
Текст задания To treatment a bronchial asthma attack that developed in the
patient during the tooth extraction, the patient was given
salbutamol. This drug belongs to th following pharmacological
group
Правильный ответ Beta-2-adrenergic agonist
B Adaptogens
C Muscarinic agonists
D Narcotic analgetics
E Analeptics
№ krok 2019
Тема Drugs acting on metabolism, blood and immunity
Текст задания A 25-year-old young man complains of general weakness,
rapid fatigability, irritability, redused working ability, and
bleeding gums. What vitamin deficiency is the most likely
cause of this condition?
Правильный ответ Ascorbic acid
B Ribaflavin
C Retinol
D Folic asid
E Thiamine
№ krok 2019
Тема Drugs acting on metabolism, blood and immunity
Текст задания An ophthalmologist has detected increased time od darkness
adaptation in the patient`s eye. What vitamin deficiency can
cause this sign?
Правильный ответ A
B C
C K
D E
E D
№ krok 2019
Тема Agents influencing on afferent and efferent innervations
Текст задания Curariform drugs are used to immobilize the patient during a
surgery. Their mechanism of action is based on the blockade of

Правильный ответ Nicotininic acetylcholine reseptors of sceletel muscles


B Acetilcholine release into the synaptic cleft
C Muscarinic acetylcholine reseptors of smooth muscles
D Noradrenaline release inti the synaptic cleft
E Condition of excitation in the nerve fibers
№ krok 2019
Тема Agents influencing on respiratory system
Текст задания A child presents with dry cough. What non-narcotic antitussive
drug can relieve the patient`s condition?
Правильный ответ Glaucine hydrochloride
B Codeine phosphate
C Potassium iodide
D Althaea officialis roots
E Morphine hydrochloride
№ krok 2019
Тема Drugs acting on metabolism, blood and immunity
Текст задания A patient with hypochromic anemia was prescribed an iron-
containing drug for intravenous administration only. Name this
drug
Правильный ответ Fercoven
B Mannitol
C Etacrinic ased
D Furosemide
E Dichlothiazide (Hydrochlorothiazide)
№ krok 2019
Тема Drugs acting on cardiovascular system
Текст задания A certain drug with potent natriuretic action is assually
prescribed for dehudratation therapy of cerebral and pulmonary
edenas. Name this drug
Правильный ответ Furosemide
B Etacrinic aside
C Theophylline
D Mannitol
E Spironolacton
№ krok 2019
Тема Drugs acting on metabolism, blood and immunity
Текст задания A woman was diagnosed with peptic ulser of the stomach. She
has a long history of rheumatoid arthritis. What drugs are the
likely cause of thise desiase in the patient?
Правильный ответ Glucocorticoids
B Antyhypertensive drugs
C Antibiotics
D H2 blockers
E Antihistamines
№ krok 2019
Тема Agents influencing on afferent and efferent innervations
Текст задания Complics therapy of a patient with bronchopneumonia
accompanied by cough includes a certain mucolytic agent that
depolymerizes mucoproteins. Name this drug
Правильный ответ Acetilcysteine
B Codeine
C Strophanthin
D Atenolol
E Neodicumarin
№ krok 2019
Тема Agents influencing on the gastrointestinal system
Текст задания A patirnt with peptic ulcer disease of the stomach is prescribed
a drug that blocks histamine H2 reseptors. Select thise drug
from the list
Правильный ответ Famotidine
B Atropine sulfate
C Dithylin (Suxamethonium)
D Bisacodil
E Omeprasol
№ krok 2019
Тема Drugs acting on cardiovascular system
Текст задания To treat ischemic heart disease, a patient was prescribed a beta-
adrenergic blocking agent. After a time he developed a cough
and bronchospasm. What drug can cause these side effects?

Правильный ответ Anaprilin (Propranolol)


B Talinolol
C Metoprolol
D Atenolol
E Phenihidine (Nifedipine)
№ krok 2019
Тема Agents influencing on afferent and efferent innervations
Текст задания Prior to tooth extraction under a local anesthesia, the patient
was tested for novocaine allergy. The test result was positive.
What substance can be used to administer anesthesia in this
case?
Правильный ответ Lidocaine
B Sodium valproate
C Procainamide
D Acetylsalicylic asid
E Analgin (Metamisole)
№ krok 2019
Тема Agents influencing on afferent and efferent innervations
Текст задания A 70-year-old patient is brought to the emergency department
by his son because of blurry vision and dysarthria. His son
says, `Father is always thirsty and has difficulty with urination`.
Examination reveals dry skin, cutaneus vasodilation,
nonreactive mydriasis, and hyperthermia. Drug overdose is
suspected. Which of the following drugs is the most likely
cause of thise patient`s toxicity?

Правильный ответ Atropine


B Reserpine
C Metamisole
D Clonidine
E Carbochol
№ krok 2019
Тема Antimicrobial agents
Текст задания A 58-year-old male patient visited his dentist with the chief
complaint of itching and burning sensation in his mouth. On
intraoral examination, diffuse white patches were seen oh his
tonge, right and left buccal mucosa, as well as on his hard
palate and soft palatal region. The potassium hydroxide (KON)
preparation of the specimen revealed non-pigmented septate
hyphae. Administration of which of the following is the most
appropriate initial step in treatment of this patient?

Правильный ответ Nystatin


B Penicillin
C Gentamicin
D Tetracyclin
E –
№ krok 2019
Тема Drugs acting on metabolism, blood and immunity
Текст задания A 35-year-old woman is brought to the physician because of a
4-month history of progressive weakness of both lower limbs.
She notes difficulty climbing stairs and complains of lethargy
and loss of muscle bulk. Her diet consists primary of `polished`
rice. A diagnosis of dry berybery is suspected. Deficiency of
the following vitamins is most likely to be detected in her
blood?
Правильный ответ Vitamin B1 (thiamine)
B Vitamine B6 (pyridoxine)
C Vitamine B3 (niacin)
D Vitamin B2 (riboflavin)
E Vitamine C (ascorbic asid)
№ krok 2019
Тема Drugs acting on the CNS
Текст задания A 42-year-old female comes to the physician 2 days after the
sudden onset of pain and swelling of right knee. She has had no
injury. Examination of the right knee shows warmth, erythema,
and effusions. Laboratory studies show an increase in the
consentration of acute phase reactants. Which of the following
is the most appropriate pharmacotherapy for this patient?

Правильный ответ Nonsteroid anti-inflametory drugs (NSAIDs)


B Sulfonomides
C Antibiotics
D Opioids
E Antidepressants
№ krok 2018
Тема Drugs acting on the CNS
Текст задания For two weeks a woman has been takingthemixturefor
neurasthenia, which was prescribed by a neurologist. Her
general state slightly improved but shortly she started
complaining of rhinitis, conjunctivitis, skinrashes, fatigue, and
memory impairment. What group of drugs can have such a side
effect?
Правильный ответ Bromine salts
B Valerian preparations
C Motherwort preparations
D Adaptogens
E Hop preparations
№ krok 2018
Тема Antimicrobial agents
Текст задания A patient developed burning sensation in the oral cavity and
white fuzzy coating on the tongue. Oral thrush is diagnosed.
What drug of those listed below should be used?
Правильный ответ Nystatin
B Amphotericin
C Griseofulvin
D Tetracycline
E Gentamicin
№ krok 2018
Тема Drugs acting on the CNS
Текст задания A dental patient was prescribed a psychosedative for his fear of
pain. What drug would be the most effective in this case?

Правильный ответ Diazepam


B Aminazine
C Lithium carbonate
D Valerian tincture
E Sodium bromide
№ krok 2018
Тема Drugs acting on metabolism, blood and immunity
Текст задания A patient with skin burns was delivered to a hospital. To clean
the wound from necrotic tissues and mucus the doctor
prescribed an enzymatic drug for topical treatment. Name this
drug:
Правильный ответ Tripsin
B Pancreatin
C Asparaginase
D Pepsin
E Streptokinase
№ krok 2018
Тема Drugs acting on metabolism, blood and immunity
Текст задания A patient with parodontosis was prescribed a fat-soluble
vitamin that actively participates in redox processes in the
organism. This antioxidant is a growth factor, has
antixerophthalmic action, and contributes to maintenance of
normal vision. In dental practice it is used to accelerate
mucosal re-epithelization during parodontosis. Name this
substance:
Правильный ответ Retinol acetate
B Ergocalciferol
C Tocopherol acetate
D Menadione (Vicasolum)
E Cyanocobalamin
№ krok 2018
Тема Agents influencing on afferent and efferent innervations
Текст задания It is necessary to decrease pumping a bility of the patient’s
heart. What membrane cytoreceptors must be blocked to
achieve this effect?
Правильный ответ β-adrenergic receptors
B α-adrenergic receptors
C α- andβ-adrenergicreceptors
D Muscarinic acetylcholine receptors
E Nicotinic acetylcholine receptors
№ krok 2018
Тема Drugs acting on cardiovascular system
Текст задания Resuscitation unit received a patient with acute poisoning
caused by unidentified medicine. To quickly excrete the
poisonfrom the patient’s body, forced diuresis was induced.
What substance was used to perform this procedure?
Правильный ответ Furosemide
B Hydrochlorothiazide
C Omeprazole
D Spironolactone
E Dithylinum (Suxamethonium chloride)
№ krok 2018
Тема Drugs acting on gastrointestinal system
Текст задания A patient with acute pancreatitis presents with significantly
increased urine diastase content. What proteolysis inhibitor
must be included into complex therapy of this patient?
Правильный ответ Contrykal (Aprotinin)
B Festal
C Pancreatine
D Digestal
E Mezym forte
№ krok 2018
Тема Agents influencing on afferent and efferent innervations
Текст задания A patient suffering fromacute bronchitis with difficult
expectoration was prescribed acetylcysteine. What drug action
will provide curative effect?
Правильный ответ Mucoproteins depolymerization
B Activation of bronchial ciliated epithelium
C Reflex stimulation of bronchiolar peristalsis
D Alkalinization of sputum
E Stimulation of the bronchial glands
№ krok 2018
Тема Drugs acting on cardiovascular system
Текст задания A patient with ischemic heart
diseasepresentswithincreasedbloodplasma content of
triglycerides and very lowdensity lipoproteins. What drug
should be prescribed?
Правильный ответ Fenofibrate
B Famotidine
C Amiodarone
D Lisinopril
E Dobutamine
№ krok 2018
Тема General pharmacology
Текст задания In what organ biotransformation (metabolic transformation) of
most medicinal agents occurs upon their introduction into an
organism?
Правильный ответ Liver
B Kidneys
C Intestine
D Skin
E Lungs
№ krok 2018
Тема Drugs acting on the CNS
Текст задания A dentist prescribed the patient with maxillofacial arthritis
diclofenac sodium. What is the mechanism of action of this
drug?
Правильный ответ Cyclooxygenase-2 inhibition
B Catalase inhibition
C Opiate receptors activation
D Opiate receptors block
E Phosphodiesterase activation
№ krok 2018
Тема Antimicrobial agents
Текст задания To treat tuberculosis, an antibiotic that colors urine red is
prescribed. Name this antibiotic:
Правильный ответ Rifampicin
B Erythromycin
C Amoxicillin
D Nitroxoline
E Cefotaxime
№ krok 2018
Тема Agents influencing on afferent and efferent innervations
Текст задания A 38-year-old woman developed an attack of bronchial asthma.
What bronchial spasmolytic for emergency medical aid is a
beta-2-adrenergic agonist?
Правильный ответ Salbutamol
B Adrenaline
C Ipratropium bromide
D Platyphyllin
E Atropine
№ krok 2018
Тема Agents influencing on afferent and efferent innervation
Текст задания A patient has been administered conduction anesthesia with
novocaine in preparation for tooth extraction. After the
anesthesia administration the patient developed swelling and
hyperemy around the injection site, skin itch, general fatigue,
motor agitation. Name the developed complication:
Правильный ответ Allergy
B Idiosyncrasy
C Tachyphylaxis
D Drug dependence
E Inflammation
№ krok 2018
Тема Drugs acting on metabolism, blood and immunity
Текст задания A 26-year-old woman presents with skin rashes and itching
after eating citrus fruits. Prescribe her a drug that is an
H1histamine receptor antagonist:
Правильный ответ Dimedrol (Diphenhydramine)
B Acetylsalicylic acid
C Menadione (Vicasolum)
D Analgin (Metamizole)
E Paracetamol
№ krok 2018
Тема Drugs acting on metabolism, blood and immunity
Текст задания A patient with megaloblastic anemia was taking a water-
soluble vitamin. Name this substance:
Правильный ответ Cyanocobalamin
B Thiamine chloride
C Tocopherol acetate
D Ascorbic acid
E Pyridoxine
№ krok 2018
Тема Agents influencing on afferent and efferent innervation
Текст задания A patient with dislocated jaw was given a short-acting muscle
relaxant by a doctor. Name this drug:
Правильный ответ Dithylinum (Suxamethonium chloride)
B Procaine
C Cytitonum (Cytisine)
D Papaverine hydrochloride
E Pyridostigmine hydrobromide
№ krok 2018
Тема Antimicrobial agents
Текст задания To treat osteomyelitis, a patient was prescribed an antibiotic
that easily penetrates into bone tissue. Name this drug:

Правильный ответ Lincomycin hydrochloride


B Streptomycin sulfate
C Cefazolin
D Polymyxin B
E Amphotericin B

You might also like